You are on page 1of 967

1.

A 65yo man presents with painless hematuria, IVU is normal, prostate is mildly enlarged with mild
frequency. What is the most appropriate next step?
a. US Abdomen
b. Flexible cystoscopy
c. MRI
d. Nuclear imaging
e. PSA
Q. 1. What is the key?
Q. 2. Points in favour of the key.
Ans. 1. The key is B. Flexible cystoscopy.
Ans. 2. Painless hematuria in an elderly (here 65 years old man) indicates carcinoma bladder for which flexible
cystoscopy is done.
An elderly gentleman complaining of painless hematuria : always exclude bladder cancer

The most important and definite Investigation for bladder cancer is a cystoscopy+ Biopsy.
Initially : Urine microscopy but it does not rule out CA.

Other causes of painless hematuria are rhabdomyolysis , coagulation disorder , prostate cancer , hemolytic
anemia , renal tumor , and polycystic kidney disease

you can exclude those by absence of :


1- History of crush injury for rhabdomyolysis
2- No bleeding from other orifices for coagulation disorder
3- No symptoms of prostatism for Prostate Cancer
4- No signs of anemia
5- No tenderness in loin or masses (renal tumor)
6- No hypertension (in polycystic kidney)

although other investigations like Mid urine sample , IVU , may show UTI , other findings like filling defects ,
etc.. they dont help with diagnosis and prognosis

Diagnosis : Bladder CA. (1 in 10,000)


Most common : Transistional cell CA. 3x in MEN of 50+ age.
Inc factors :
Smoking, schistosomiasis, rubber dye industries, White ppl, recurrent infections.

Symptoms :
Painless hematuria (on and off)
Pain in lower abdomen
Treatment :
TUR with 1 chemotherapy within 24 hours. If needed, BCG is used for next chemo cycles.

2. A 74 yo smoker presented to his GP with cough and SOB. Exam revealed pigmentation of the oral
mucosa and also over the palms and soles. Tests show that he is diabetic and hypokalemic. What is the most
probable dx?
a. Pseudocushing syndrome
b. Conns disease
c. Ectopic ACTH
d. Cushings disease
e. Hypothyroidism

Q. 1. What is the likely key?


Q. 2. Please explain the key.
Ans. 1. The key is C. Ectopic ACTH.
Ans. 2. The patient is smoker and probably developed small cell lung cancer which is working as a tumour
producing ectopic ACTH resulting in pigmentation. Resultant raised cortisol is leading to diabetes and
hypokalemia.
The features can be explained by increased levels of ACTH and adrenocortical hormones.
So the question is : ectopic or pituitary ACTH excess? It seems to be from an ectopic source since the patient is
smoker and has SOB and cough ( Lung tumor whether small cell CA bronchus or carcinoid tumor- both may
secrete ectopic ACTH ) . Furthermore , Cushing's disease is often the result of pituitary ACTH-secreting
adenoma that also causes pressure symptoms like headache and visual disturbances which are absent in this
case.
Ectopic ACTH increase the secretion of aldosterone from adrenal gland and aldosteronism causes
hypernatraemia and hypokalaemia.
Cortisol is a form of stress hormone. So it induces glycogenolysis causing increase in blood glucose.
No pigmentation in conn
Conns must have hypertension & not necessarily hypokalemia but it presents with signs of hypokalemia like
weakness quadriparesis cramps.

Why not Cushings?


SCLC is a direct cause of ectopic ACTH (statement is clear cut - Smoker). Further ectopic acth can lead to
cushings at later stages but its major cause is use of steroids and pituitary adenoma while ectopic acth is
down the list. And if cushings happen, the major indicative symptoms are stria, moon face, easily fractured
bones, plethora.

Diagnosis : Small Cell Lung CA causing ectopic ACTH.


Main reason : SMOKING for years. At Least 20 so age goes up to 50 to 60. Male.
Symptoms :
Persistent cough, hemoptysis, chest n shoulder pains, SOB, clubbing. +/- pleural effusion, pneumonia, pins
and needles in arm n shoulder sensation.

Invs :
Initial : CXR. Shows shadowing
Confirmatory : CT chest THEN Biopsy thru bronchoscope or transthoracic needle biopsy. Depends on the
location.
Pleural Tap can be done if pleural effusion.
Treatment :
Surgery, radio and chemo.
Prognosis : Good if early diagnosed. Bad if late.

3. A 44yo woman has lost weight over 12 months. She has also noticed episodes where her heart beats rapidly
and strongly. She has a regular pulse rate of 90bpm. Her ECG shows sinus rhythm. What is the most
appropriate inv to be done?
a. Thyroid antibodies
b. TFT
c. ECG
d. Echocardiogram
e. Plasma glucose
Q. 1. What is the key?
Q. 2. What is the diagnosis?
Q. 3. What is the significance of episodes of rapid strong heart beat?
Ans. 1. The key is B. TFT.
Ans. 2. Thyrotoxicosis [weight loss over 12 months, episodes of rapid strong heart beet (thyrotoxicosis induced
paroxysmal atrial fibrillation) points towards the diagnosis of thyrotoxicosis].
Ans. 3. Episodes of rapid strong heart beat indicates thyrotoxicosis induced paroxysmal atrial fibrillation.

Diagnosis :
Hyperthyroidism/t
hyrotoxicosis.
Gender : more in
females. 20 to 50
age.
Most common : Graves'
Symptoms :
Irritable, always on the go, losing weight despite increase appetite, palpitations, heat intolerance, sweating,
Diarrhea, SOB, itch, very light periods, increase risk of AF and osteoporosis.
Meds that cause it : Amiodarone and lithium.
Invs : TFTS. Low TSh and high T4.
Treatment :
1. Carbimazole for 12 to 18 months.
Pregnancy - Propylthiouracil
2. Radioiodine. Should not be pregnant and conceive for at least 6 months. Father at least 4 months.
3. Surgery
4. Beta blockers (propranolol, atenolol)
Follow up every year is very imp.

4. 79yo anorexic male complains of thirst and fatigue. He has symptoms of frequency, urgency
and terminal dribbling. His urea and creatinine levels are high. His serum calcium is 1.9 and he is anemic. His
BP is 165/95 mmHg. What is the most probable dx?
a. BPH
b. Prostate carcinoma
c. Chronic pyelonephritis
d. Benign nephrosclerosis

Explanation of Question no. 4:


First to say in this case (almost all features goes in favour of prostatic carcinoma like- frequency, urgency and
terminal dribbling are features of prostatism; Age, anorexia and anaemia favours carcinoma prostate
diagnosis and it would be accurate presentation if it was hypercalcaemia. But given calcium level is of
hypocalcaemic level and it is the main cause of discrepancy of this question). Renal failure can be an
association of malignant disease and can cause high BP. Thirst is a feature of hypercalcaemia (here may be
erroneously calcium level is given in hypocalcaemic level ; probably a bad recall). Prostate biopsy is the
confirmatory diagnosis and others like PSA is suggestive. This is what I could point out. If there is any better
explanation please place it to correct the answer- any one please.

Calcium 2.1-2.6 mmol/l


I think this patient has CKD secondary to prostate CA which leads to hypocalcemia
due to vit D def.
Osteoblastic metastases Occasional patients with widespread osteoblastic metastases, particularly those
with breast or prostate cancer, have hypocalcemia.

Diagnosis : Prostate Cancer


Most common CA in men of uk. 1 in 8 men. After 65.
Risk factors: Fatty diet, exposure to cadmium, ageing & family history.

Symptoms :
Poor stream, hesitancy, dribbling, frequency, urgency, poor emptying.

Invs :
Examine. PSA levels.
Confirmatory test : Biopsy.

Grading : Gleason Score.


4 or less - well differentiated. 10 yr risk of local progression 25%
5 - 7 - moderately differentiated. 50% risk
Over 7 - poorly differentiated. 75% risk

Risk assessment PSA levels.


Low - <10 and gleason score 6 or below
Intermediate - psa 10 to 20 or gleason score 7
High - psa >20 or gleason 8 to 10.

Staging : MRI preferred over CT.


Treatment : Surgery. Radical prostatectomy. S/E impotence, incontinence of urine.
Radiotherapy. External and internal (brachytherapy)
HRT to stop TESTOSTERONE.
Medicines - LHRH. Goserelin, leuprorelin, triptorelin (act on pitutary) and Flutamide, cyproterone (anti
androgenic)
Prognosis : variable. Depends on the stage.
Complications :
UTi, AKI, CKD, sexual dysfunction, metastasis.

Note : Prostate CA has increased risk with HYPERCALCEMIA. Not hypo. So the statement seems to be wrong.
Even BPH has nothing to do with Ca levels.
Benign nephrosclerosis is due to long standing HTN. No link to prostate found.

5. A 64yo man has recently suffered from a MI and is on aspirin, atorvastatin and ramipril. He has
been having trouble sleeping and has been losing weight for the past 4 months. He doesnt feel
like doing anything he used to enjoy and has stopped socializing. He says he gets tired easily and
cant concentrate on anything. What is the most appropriate tx?
a. Lofepramine
b. Dosulepin
c. Citalopram
d. Fluoxetine
e. Phenelzine
Ans. The key is C. Citalopram. [Citalopram is the antidepressant of choice in IHD]
Citalopram is associated with dose-dependent QT interval prolongation and is contra-indicated in patients
with known QT interval prolongation or congenital long QT syndrome.

Sertraline can also be used in patients with IHD for depression. sertraline is considered the drug of choice
post-MI
1. Depression with obesity=fluoxetine (It helps without weight loss)
2. Depression with sexual dysfunction=mirtazapine
3. Post stroke depression use nortriptyline (TCA)
4. Depression with obsessive compulsive disorder=clomipramine (TCA)
5. Depression with ischemic heart disease=SSRI e. g citalopram
6. A 67yo man after a stroke, presents with left sided ptosis and constricted pupil. He also has loss
of pain and temp on the right side of his body and left side of his face. Which part of the brain is
most likely affected?
a. Frontal cortex
b. Cerebellum
c. Pons
d. Medulla
e. Parietal cortex
Q. 1. What is the key?
Q. 2. What is the name of this condition?
Ans. 1. The key is D. Medulla.
Ans. 2. The name of the condition is Lateral medullary syndrome [ipsilateral Horner syndrome and
contralateral loss of pain and temperature sense]

Lateral medullary syndrome, also known as Wallenberg's syndrome, occurs following occlusion of the
posterior inferior cerebellar artery

Cerebellar features
ataxia
nystagmus

Brainstem features
ipsilateral: dysphagia, facial numbness, cranial nerve palsy e.g. Horner's
contralateral: limb sensory loss
Lateral medullary or Wallenberg's syndrome:
Usually from occlusion of the vertebral artery.
Occasionally from occlusion of the posterior inferior cerebellar artery.
Involvement of the vestibular system causes nausea, vomiting and vertigo.
Ipsilateral features:
o Ataxia from cerebellar involvement.
o Horner's syndrome from damage to descending sympathetic fibres.
o Reduced corneal reflex from descending spinal tract damage.
o Nystagmus.
o Hypacusis.
o Dysarthria.
o Dysphagia.
o Paralysis of palate, pharynx, and vocal cord.
o Loss of taste in the posterior third of the tongue.
Contralateral findings:
o Loss of pain and temperature sensation in the trunk and limbs (anterior
spinothalamic tract).
o Tachycardia and dyspnoea (cranial nerve X).
o Palatal myoclonus (involuntary jerking of the soft palate, pharyngeal muscles and
diaphragm).

7. A 60yo man presents with dysphagia and pain on swallowing both solids and liquids. A barium
meal shows gross dilatation of the esophagus with a smooth narrowing at the lower end of the
esophagus. What is the SINGLE most likely cause of dysphagia?
a. Achalasia
b. Myasthenia gravis
c. Esophageal carcinoma
d. Esophageal web
e. Systemic sclerosis
Ans. The key is A. Achalasia.

Achalasia typically presents in middle-age and is equally common in men and women
Investigations
manometry: excessive lower oesophageal sphincter tone which doesn't relax on swallowing -
considered most important diagnostic test
barium swallow shows grossly expanded oesophagus, fluid level, 'bird's beak' appearance. This is in
contrast to the rat's tail appearance of carcinoma of the oesophagus
CXR: wide mediastinum, fluid level
Gold standard - Manometry

Treatment
intra-sphincteric injection of botulinum toxin
Heller cardiomyotomy for fit young patients.
balloon dilation for old unwell patients.

Complications : Aspiration pneumonia, perforation, GERD, Oesophagus CA.


Dysphagia
The table below gives characteristic exam question features for conditions causing dysphagia:

Dysphagia may be associated with weight loss, anorexia or vomiting during eating
Oesophageal
cancer Past history may include Barrett's oesophagus, GORD, excessive smoking or alcohol use

Oesophagitis May be history of heartburn

Odynophagia but no weight loss and systemically well

Oesophageal There may be a history of HIV or other risk factors such as steroid inhaler use
candidiasis

Achalasia Dysphagia of both liquids and solids from the start

Heartburn

Regurgitation of food - may lead to cough, aspiration pneumonia etc

Pharyngeal pouch More common in older men

Represents a posteromedial herniation between thyropharyngeus and cricopharyngeus


muscles

Usually not seen but if large then a midline lump in the neck that gurgles on palpation

Typical symptoms are dysphagia, regurgitation, aspiration and chronic cough. Halitosis
may occasionally be seen

Systemic sclerosis Other features of CREST syndrome may be present, namely Calcinosis, Raynaud's
phenomenon, oEsophageal dysmotility, Sclerodactyly, Telangiectasia

As well as oesophageal dysmotility the lower oesophageal sphincter (LES) pressure is


decreased. This contrasts to achalasia where the LES pressure is increased

Myasthenia gravis Other symptoms may include extraocular muscle weakness or ptosis

Dysphagia with liquids as well as solids


Globus hystericus May be history of anxiety

Symptoms are often intermittent and relieved by swallowing

Usually painless - the presence of pain should warrant further investigation for organic
causes

8. A man undergoes a pneumonectomy. After surgery, invs show hyponatremia. What could be the
cause of the biochemical change?
a. Removal of hormonally active tumor
b. Excess dextrose
c. Excess colloid
d. Excessive K+
e. Hemodilution
Ans. The key is A. Removal of hormonically active tumour.

Small cell lung carcinoma produces acth like peptide which stimulates aldosterone secretion causing
hypernatremia. Removal of that will lead to hyponatremia.
OHCM 170...Lung tumors may secrete both ACTH and ADH. If it was an ACTH secreting tumor then it's removal
may cause hyponatremia. As ACTH helps in absorption of Na and water by releasing Aldosterone from adrenal
gland. On the other hand if it was an SIADH secreting tumor then opposite would happen.

9. A pregnant lady came with pain in her calf muscle with local rise in temp to the antenatal clinic.
What tx should be started?
a. Aspirin
b. LMWH
c. Paracetamol
d. Cocodamol
e. Aspirin and heparin
Ans. The key is B. LMWH.

During pregnancy :
Start LMWH and continue throughout pregnancy. Stop the injections 24 hours before labour and then
restart them 4 hours post op. Warfarin is Contraindicated in pregnancy.
If NO pregnancy :
the protocol is different.
- LMWH stat
- Start Warfarin within 24 hours
- Monitor INR and withdraw LMWH when value is 2.0
- Depending on provoked or non provoked, give Warfarin for 3 and 6 months respectively and then stop.
- IVC filter is used when anticoagulants fail
- Compression stockings to all patients to prevent 'Post-phlebitic limb changes'
Wells' diagnostic algorithm[1]

Score one point for each of the following:


Active cancer (treatment ongoing or within the previous six months, or palliative).
Paralysis, paresis or recent plaster immobilisation of the legs.
Recently bedridden for three days or more, or major surgery within the previous 12 weeks,
requiring general or regional anaesthesia.
Localised tenderness along the distribution of the deep venous system (such as the back of the
calf).
Entire leg is swollen.
Calf swelling by more than 3 cm compared with the asymptomatic leg (measured 10 cm below
the tibial tuberosity).
Pitting oedema confined to the symptomatic leg.
Collateral superficial veins (non-varicose).
Previously documented DVT.

Subtract two points if an alternative cause is considered at least as likely as DVT.

The risk of DVT is likely if the score is two or more, and unlikely if the score is one or less.

invs :
initial - Duplex USG
Gold standard - invasive venography

10. A 53yo female presents with an acute painful hot knee joint. She is a known case of RA. On
examination, the knee is red, tender and swollen. The hamstring muscles are in spasm. Her
temp is 38.5C and BP is 120/80mmHg. What is the SINGLE best next inv?
a. Joint aspiration for cytology and culture and sensitivity
b. Joint aspiration for positive birefrengent crystals
c. Joint aspiration for negative birefrengent crystals
d. Blood culture
e. Serum uric acid
Q. 1. What is the likely key here?
Q. 2. Is there any link in septic arthritis and rheumatoid arthritis?
Q. 3. What is the likely organism in this age group?
Q. 4. What is the likely organism in younger age group?
Ans. 1. A. Joint aspiration for cytology and culture and sensitivity.
Ans. 2. Any chronically arthritic joint is predisposed to infection. Moreover chronic use of steroid in Rh.
arthritis is one of the important predisposing factor.
Ans. 3. Staphylococcus
Ans. 4. Neisseria gonorrhoeae

RA always involves bilateral symmetrical joints with morning stiffness. The patient presented with new
complaint which is monoarticular, swollen n hot. It's clearly Septic arthritis n u do joint aspiration. Chronic
use of steroids is one of the important predisposing factors.

Diagnosis : Septic Arthritis due to persistent Rheumatoid Arthritis.


The classic picture is a single swollen joint with pain on active or passive movement.
It is more common in patients with prior joint damage, as in gout, rheumatoid arthritis and systemic
connective tissue disorders.
Fever and rigors. Chest wall pains.
Treatment :
Flucloxacillin and for MRSA - Vancomycin.
Penicillin with Gentamicin is being used as well.

11. An 80yo man presented with pain in his lower back and hip. He also complains of waking up in
the night to go to the washroom and has urgency as well as dribbling. What is the most likely dx?
a. BPH
b. Prostatitis
c. UTI
d. Prostate carcinoma
e. Bladder carcinoma
Q. 1. What is the likely key?
Q. 2. What are the points in favour of your diagnosis?
Q. 3. What are the investigations?
Q. 4. What are the treatment options for carcinoma prostate?
DISCUSSED IN MCQ 4.
Ans. 1. D. Prostate carcinoma.
Ans. 2. Age, nocturia, urgency and dribbling points towards prostate pathology. Pain of lower back and hip
points towards bony metastases from prostate cancer.
Ans. 3. Blood test for PSA; Prostate biopsy; MRI [if initial biopsy is negative, to decide repeat biopsy]. Source
NICE.
Ans. 4. Treatment options: 1. Active treatment [i) radical prostatectomy ii) radical radiotherapy iii) hormone
therapy iv) brachytherapy v) pelvic radiotherapy vi) orchidectomy]
2. Active surveillance
3. Watchful waiting
4. Palliative care [Source: NICE].

12. An 18yo female has periorbital blisters. Some of them are crusted, others secreting pinkish fluid. What is
the most likely dx?
a. Shingles
b. Chicken pox
c. Varicella
d. Rubella
e. Measles
Q.1. What is the likely key?
Q. 2. Which nerve is involved here?
Q. 3. Is this disease unilateral or bilateral?
Ans. 1. A. Shingles
Ans. 2. Ophthalmic division of trigeminal nerve.
Ans. 3. Typically shingles is unilateral.

Short note everywhere. Mcq covers it up.


Treatment :
Refer to ophthalmologist. Ocular lubricants, cool compressors, topical steroids,
Botulinum toxin injection if neurotrophic ulcers form.
13. A 29yo lady who is a bank manager is referred by the GP to the medical OPC due to a long hx of tiredness
and pain in the joints. An autoimmune screen result showed smooth muscle
antibodies positive. What is the most appropriate next inv?
a. ECG
b. TFT
c. LFT
d. Serum glucose
e. Jejunal biopsy
Q. 1. What is the likely key?
Q. 2. What is the diagnosis?
Q. 3. What is the definitive investigation?
Q. 4. What is the treatment?
Ans. 1. C. LFT
Ans. 2. Autoimmune hepatitis.
Ans. 3. Definitive investigation is liver biopsy
Ans. 4. Steroid [start with high dose prednisolone]. Azathioprine is commonly added with steroid to reduce its
dose as steroid has more side effects than azathioprine.

Diagnosis : Autoimmune hepatitis.

Symptoms :
Tiredness, fatigue, mild pruritus, amenorrhea, pleuritis, abdominal discomfort, Oedema, Skin rashes, acne,
weight loss. Nausea is prominent.

Signs :
Hepatomegaly, splenomegaly, spider angiomata, ascites, encephalopathy, jaundice in 50%

The autoantibodies present include antinuclear antibody (ANA), anti-smooth muscle antibody (ASMA), anti-
liver-kidney microsomal-1 (anti-LKM-1) antibody, antibodies against soluble liver antigen (anti-SLA),
antimitochondrial antibody (AMA) and antiphospholipid antibodies.

Invs :
1. Autoantibodies. Typical is ASMA.
2. Ig G is raised. (Can lead to hyperviscosity syndrome)
3. LFTs. ALT and AST are raised. ALP maybe normal.
4. USG abdomen.
5. Liver biopsy - Confirmatory.

Treatment :
Steroids.
Budesonide + Azathioprine
Prednisolone + Azathioprine
+/- methotrexate, Anti TNF alpha drugs
Liver transplant.
14. A 5yo with recurrent chest pain, finger clubbing with offensive stool. Choose the single most
likely inv?
a. Endomysial/Alpha gliadin antibody
b. Sweat test
c. Barium meal
d. ECG
e. Glucose tolerance test
Q. 1. What is the likely key?
Q. 2. What is the diagnosis?
Q. 3. What is the mode of inheritance?
Ans. 1. B.
Ans. 2. Cystic fibrosis
Ans.3. Autosomal recessive.

DIAGNOSIS : Cystic FIbrosis. Mutation of CFTR on chromosome 7.


It cause dehydration. Hence, bronchiectasis, bowel obstruction and bacterial
growth,

Signs
Finger clubbing.
Cough with purulent sputum.
Crackles.
Wheezes (mainly in the upper lobes).

INVS :

Sweat Test. Chloride concentration > 60. Sinus X ray or CT scan - Opacities of sinuses.

Lung function tests, LFTs, Sputum microbiology.

17. A man with suspected active TB wants to be treated at home. What should be done to prevent
the spread of disease?
a. Immediate start of the tx with Anti-TB drugs
b. All family members should be immediately vaccinated with BCG vaccine
c. Patient should be isolated in a negative pressure chamber in his house
d. Universal prevention application protocol
Ans. d. Universal prevention application protocol.
This protocol isnt available anywhere on the internet. Everybody is suggesting D on the
basis of exclusion.
18. A 7yo child is brought to the ED with a 1 day hx of being listless. On examination, the child is
drowsy with an extensive non-blanching rash. What advice would you give the parents?
a. All family members need antibiotic therapy
b. Only the mother should be given rifampicin prophylaxis
c. All family members need isolation
d. All family members should be given rifampicin prophylaxis
Q. 1. What is the likely key?
Q. 2 What is the probable diagnosis?
Q. 3. What is the diagnostic investigation?
Q. 4. What is the initial management?
Ans. 1. D. All family member should be given rifampicin prophylaxis
Ans. 2. Meningococcal disease.
Ans. 3. Blood or CCF PCR
Ans. 4. Prehospital management: Benzyl penicillin or cefotaxime.

DIAGNOSIS : Meningococcal infection. It can be meningococcal Septicemia which is without Meningitis, or


can be Meningococcal Meningitis. This Mcq doesnt have meningitis signs so we will stick to M.Septicemia.

Caused by N.Meningitidis
Signs and Symptoms :
Most common and important - Non - blanching rash
Fever, headache
May have : Stiff neck, back rigidity, bulging fontanelle (in infants), photophobia.
Altered mental state, unconsciousness, toxic/moribund state, Kernig's sign (pain and resistance on passive
knee extension with hips fully flexed) and Brudziski's sign (hips flex on bending the head forward)

Pre-Hospital Management :
Call 999 and give Benzyl penicillin or Cefotaxime

INVESTIGATIONS :
Blood cultures.
FBC (WCC), CRP, U&Es, renal function tests, LFTs.
Blood test for polymerase chain reaction (PCR): perform whole blood real-time PCR testing -
(EDTA sample) - for N. meningitidis to confirm a diagnosis of meningococcal disease
Investigations for disseminated intravascular coagulation: prothrombin time is elevated,
activated partial thromboplastin time (aPTT) is elevated, platelet count is reduced and the
fibrinogen level is low.
Lumbar puncture - once the patient is stable
Aspirate from other sterile sites suspected of being infected (eg, joints) for microscopy, culture
and PCR.

TREATMENT :

Choice of antibiotics in hospital :


o Ceftriaxone is usually given to those over 3 months
o Cefotaxime and amoxicillin are usually given to those under 3 months.
o Vancomycin is given in addition, to those who have recently travelled outside the UK
or have had prolonged or multiple exposure to antibiotics.
CHEMOPROPHYLAXIS :
To close contacts of cases, irrespective of vaccination status - for example, those who have had
prolonged close contact with the case in a household-type setting during the seven days before
onset of illness
Ciprofloxacin and rifampicin are both recommended by Public Health England (PHE) but ciprofloxacin
is the preferred choice for most individuals.
Ciprofloxacin can be used in all ages and in pregnancy; it is easily available in a single dose and does
not interfere with oral contraceptives (but is contra-indicated if there has been previous sensitivity):
o Adults and children aged >12 years - 500 mg orally stat.
o Children aged 5-12 years - 250 mg orally stat.
o Children aged <5 years - 30 mg/kg up to a maximum of 125 mg orally stat.
Rifampicin is no longer the drug of choice as, although it is licensed for chemoprophylaxis, it has
several disadvantages including important drug interactions

19. A 47yo man has a temp of 39C and is delirious. He has developed blisters mainly on his trunk,
which appeared a few hours ago. He is well and not on any medications. He last travelled 5
months ago to Italy. Which of the following is the most likely dx?
a. Shingles
b. Chicken pox
c. Pemphigoid
d. Bullous pemphigus
Q 1. What is the likely key?
Q. 2. Why delirium in this patient?
Q. Is his travel history has any link to the development of this disease?
Ans. 1. B. Chicken pox
Ans. 2. Adults more commonly develop a more generalized brain inflammation ("encephalitis") whose
symptoms may include delirium and seizures.
Ans. 3. Incubation period of chicken-pox is 10-21 days. So this travel history is not significant.

DIAGNOSIS : Chicken POX.


Why not Shingles?
Shingles is painful, while mcq does not mention it.
Shingles have dermatomal distribution while chickenpox is mainly peripheral and
truncal in distribution.
Chicken pox has 10 - 21 days incubation period, and so does shingles, so this 5 months thing is wrongly
stated.
The patient is delirious - suggesting ENCEPHALITIS which is the main complication of C.pox.

INVS :
Scraping the blisters and the skin and checking it with immunohistochemical and PCR.
CXR and LP for its complications. (Pneumonia and Encephalitis)

Management

Chickenpox in an otherwise healthy individual


Simple advice regarding adequate fluid intake, minimising scratching if possible and that the first
1-2 days they are most infectious.
Symptomatic treatment - eg, analgesia and antipyretics such as paracetamol. There is a possible
association with non-steroidal anti-inflammatory drugs (NSAIDs) and risk of necrotising soft
tissue infections.
Pruritus can be helped by sedating antihistamines and emollients. Calamine lotion is no longer
recommended, as when it dries it ceases to be effective. Secondary infection may require
antibiotics.
Aciclovir should be considered if the patient presents within 24 hours
Anyone with encephalitis - should be admitted to hospital.
20. A 64yo pt has been having freq episodes of secretory diarrhea, which is extremely watery, with large amts
of mucus. A dx of villous adenoma was made after endoscopy. What electrolyte
abnormality is most likely in this pt?
a. Hyperkalemia
b. Hypernatremia
c. Hyponatremia
d. Hypokalemia
e. Hypercalcemia
Q. 1. What is the likely key?
Q. 2. Why this occurs?
Ans. 1. D
Ans. 2. There is active potassium secretion causing high fecal potassium concentration leading to this
hypokalaemia in secretory diarrhea. There is also reduced potassium absorption in diarrhea.

Electrolyte loss in diarrhoea (Na, K, Mg, Cl). But potassium loss is more marked especially in severe, chronic
and when associated with mucous loss. Villous adenomas secrete protein n potassium rich mucus so
hypoalbuminemia n hypokalemia

Hypokalaemia is usually defined as a serum concentration of potassium <3.5 mmol/L. It can be classified as
follows:

Mild - 3.1 - 3.5 mmol/L


Moderate - 2.5 - 3.0 mmol/L
Severe - <2.5 mmol/L

Typical ECG findings when potassium is <3.0 mmol/L:

Flat T waves
ST depression
Prominent U waves

NB: the QT interval may appear prolonged, but this is usually a pseudo-prolongation as the flattened T
waves merge into the U waves.

MAIN CAUSES OF HYPOKALEMIA.

KIDNEY causes :

Thiazide or loop diuretics (the most common cause)


Renal tubular acidosis
Hypomagnesaemia
Hyperaldosteronism - eg, Conn's syndrome, renal artery stenosis, Cushing's disease
Tubulo-interstitial renal disease due to Sjgren's syndrome or systemic lupus erythematosus
Excess liquorice ingestion
Activation of the renin-angiotensin system - eg, Bartter's syndrome or Gitelman's syndrome

Via the GI tract:


Diarrhoea
Vomiting (bicarbonate diuresis)
Intestinal fistulae
Villous adenoma
Pyloric stenosis
Laxative abuse
Bowel preparation with oral sodium phosphate solution

Via the skin:

Burns
Increased sweating - eg, exercising in a hot climate
Increased loss in sweat - eg, cystic fibrosis

21. A pt with an acute gout attack came to the ED. What drug should be given to relieve symptoms?
a. NSAIDs
b. Allopurinol
c. Ibuprofen
Q. 1. What is the key?
Q. 2. What is the acute management of gout?
Ans. 1. A
Ans. 2. Oral NSAIDs commenced immediately and continue for 1 2 weeks; Colchicine can be effective
alternative but is slower to work than NSAIDs. Intra articular corticosteroids are highly effective in acute gouty
monoarthritis.

DIAGNOSIS : GOUT

Cause : Interleukin - 1

Risk factors
Male sex, Meat, Seafood, Alcohol (10 or more grams per day), Diuretics, Obesity, Hypertension,
Coronary heart disease, Diabetes mellitus, Chronic renal failure, High triglycerides

Pharmacological therapeutic options include:

Non-steroidal anti-inflammatory drugs (NSAIDs)


Colchicine
Corticosteroids

NOTE : This MCQ has NSAIDS and Ibuprofen in options which is from the same group
but the preferred NSAIDS are : Diclofenac, naproxen and indomethacin.
Colchicine is particularly appropriate when NSAIDs are poorly tolerated, in patients with heart failure and in
those who are on anticoagulants.
30-35 mg of prednisolone reported a low incidence of side-effects
Allopurinol should never be started during an acute attack. Wait for 1-2 weeks after the attack
resolves.
Co-prescribe colchicine or a low dose non-steroidal anti-inflammatory drug (NSAID) to prevent an
attack of gout whilst initiating therapy, and continue until after hyperuricaemia has settled (usually a
total of three months).
If an acute attack develops during treatment, maintain the dose of allopurinol but add colchicine or
NSAIDs.

22. A pt was lying down on the operating table in a position with his arms hanging down for 3 hours. Soon
after he woke up, he complains of numbness and weakness in that hand and has limited wrist
movement/wrist drop and sensory loss over dorsum of that hand, weakness of extension of the fingers and
loss of sensation at the web of the thumb. What structure is likely to be damaged?
a. Radial nerve
b. Median nerve
c. Ulnar nerve
d. Axillary nerve
e. Suprascapular nerve

ANS is A. Radial Nerve.


Radial nerve, in spiral grove on humerus is pressed and damaged. It is also called saturday night palsy.

23. A pt who was previously on 120mg slow release oral morphine has had his dose increased to
200mg. He is still in significant pain. He complains of drowsiness and constipation. What is the
next step in the management?
a. Increase slow release morphine dose
b. Fentanyl patch
c. Replace morphine with oral hydromorphone
d. Replace morphine with oxycodone
e. Subcutaneous morphine
Q. 1. What is the likely key? D - Replace morphine with oxycodone.
Q. 2. Why not other options? X

Morphine Start with oral solution 510mg/4h PO with an equal breakthrough dose as often as required. A
double dose at bedtime can enable a good nights sleep. Patient needs will vary greatly and there is no
maximum dose; aim to control symptoms with minimum side-effects. If not effective, increase doses in 30
50% increments (5mg10mg20mg30mg45mg). Change to modified release preparations (eg MST Continus
12h) once daily needs are known by totalling 24h use and dividing by 2. Prescribe 1/6th of the total daily dose
as oral solution for breakthrough pain. Side effects (common) are drowsiness, nausea/vomiting, constipation
and dry mouth. Hallucinations and myoclonic jerks are signs of toxicity and should prompt dose review.
If the oral route is unavailable try morphine/diamorphine IV/SC. If difficulty tolerating morphine/diamorphine,
try oxycodone PO/IV/SC/PR, starting at an equivalent dose. It is as effective as morphine and is a useful 2nd-
line opioid with a different range of receptor activity. There are also fentanyl transdermal patches which
should usually be started under
specialist supervision (after opioid dose requirements have been established). Remove
after 72h, and place a new patch at a different site. 45mg oral morphine/24h is approximately equivalent to a
12mcg/h fentanyl patch.
24. A 40yo woman notices increasing lower abdominal distention with little/no pain. On
examination, a lobulated cystic mass is felt and it seems to be arising from the pelvis. What is
the most appropriate inv?
a. CA 125
b. CA 153
c. CA 199
d. CEA
e. AFP
Q. 1. What is the likely key?
Q. 2. What is the likely diagnosis?
Ans. 1. A
Ans. 2. Ovarian ca. X

DIAGNOSIS : Ovarian CA
Age - 60 to 80 women

SYMPTOMS :
Early symptoms are often vague, such as abdominal discomfort, abdominal distension or
bloating, urinary frequency or dyspepsia. Constitutional symptoms include fatigue, weight loss,
anorexia and depression.
It most commonly presents with a pelvic or abdominal mass that may be associated with pain.
Abdominal, pelvic or back pain is usually a late sign
It may cause abnormal uterine bleeding.
Often associated with ascites. One third of patients with ascites also have a pleural effusion.

INVESTIGATIONS :

CA-125 test.
If this is reported as raised (35 IU/mL or greater) arrange pelvic and abdominal ultrasound scans.
CT is the investigation of choice in the UK

Treatment :

The standard comprehensive surgical staging approach consists of a total abdominal hysterectomy and
bilateral salpingo-oophorectomy (TAH and BSO)

The standard regime is paclitaxel and carboplatin given intravenously every three weeks for six cycles.

Radiotherapy.

PRGNOSIS :

Stage I: 92%
Stage II: 55%
Stage III: 21.9%
Stage IV: 5.6%
25. A resident of a nursing home presented with rashes in his finger webs and also on his abdomen, with
complaints of itching which is severe at night. He was dx with scabies. What the best tx for
his condition?
a. 0.5% permethrin
b. Doxycycline
c. 5% permethrin
d. Reassure
e. Acyclovir
Q. 1. What is the likely key?
Q. 2. Will you consider any other treatment beside this?
Ans. 1. C
Ans. 2. Scabies outbreaks in nursing homes and cases of crusted scabies may require combination therapy
consisting of topical application of permethrin and 2 oral doses of ivermectin at 200 mcg/kg (administered 1
wk apart). X

DIAGNOSIS : SCABIES

signs and symptoms develop after 3-4 weeks. Symptoms reappear within 1-3 days if the person is
re-infested due to prior sensitisation.
The most common presenting symptom is widespread itching. This is usually worse at night and
when the person is warm. A history of several family members all suffering with itch is strongly
suggestive of scabies. Scratching predisposes to secondary bacterial infection.
Lesions may be papules, vesicles, pustules, and nodules. Erythematous papular or vesicular lesions
are usually seen in the sites of the burrows. The more widespread, symmetrical, itchy, papular
eruption is not in the areas of burrows or obvious mite activity. This is most commonly seen
around the axillae, the peri-areolar region of the breasts in women, and the abdomen, buttocks,
and thighs.
The 'wake' sign is specific for scabies, can be seen with the naked eye and points towards the
location of the mite.
Hyperkeratotic crusted lesions called as Crusted Norwegian Scabies

INVS :
Ink Burrow Test.
Diagnosis can be confirmed by taking a skin scraping from an affected area.

TREATMENT :
First line : Permethrin 5%
2nd line : malathion 0.5% aqueous liquid

26. A 34yo alcoholic is found passed out in front of a local pub. The ambulance crew informs you
that he was sweating when they found him and there were cans of cider lying empty around
him. What is the initial stage of inv?
a. Capillary blood sugar
b. CT head
c. MRI head
d. ABG
e. MCV
key: A
Hypoglycemia: blood glucose <3.0 mmol/L
alcohol exerts hypoglycemic effects through inhibiting both gluconeogenesis and glycogenolysis , especially
when the drinker already has starvation or adrenocortical insufficiency.
Here we need to find the causes of passed out..first thing first.glucose for hypo...ABG can help to find acidosis
which is likely to find in alcohol poisoning..MCV it will b increased in alcoholic but we are more likely to look
for anaemia in this as there may b H&M which could lead pt to pass out..CT head for SDH.

Risk factors
Tight glycaemic control.
Malabsorption.
Injection into lipohypertrophy sites.
Alcohol.
Insulin prescription error (notable in hospitalised patients).
Long duration of diabetes.
Renal dialysis.
Drug interactions between hypoglycaemic agents - eg, quinine, selective serotonin reuptake
inhibitors (SSRIs).
Impaired renal function.
Lack of anti-insulin hormone function - eg, Addison's disease, hypothyroidism.

Initially
Glucose 10-20 g is given by mouth, either in liquid form or as granulated sugar (two teaspoons) or
sugar lumps
Repeat capillary blood glucose after 10-15 minutes; if the patient is still hypoglycaemic then the
above can be repeated (probably up to 1-3 times).

If hypoglycaemia causes unconsciousness, or the patient is unco-operative

75-80 ml 20% glucose or 150-160 ml of 10% glucose (the volume will be determined by the clinical
scenario).
25 ml of 50% glucose concentration is viscous, making it more irritant and more difficult to
administer, and is rarely used now.

Once the patient regains consciousness, oral glucose should be administered, as above.

If the patient is at home, or intravenous (IV) access cannot be rapidly established


Glucagon 1 mg should be given by intramuscular (IM), or subcutaneous (SC) injection.
This dose is used in insulin-induced hypoglycaemia (by SC, IM, or IV injection), in adults and in
children over 8 years (or body weight over 25 kg). NB: 1 unit of glucagon = 1 mg of glucagon.

The patient must be admitted to hospital if hypoglycaemia is caused by an oral antidiabetic drug, because the
hypoglycaemic effects of these drugs may persist for 12-24 hours and ongoing glucose infusion or other therapies
such as octreotide (see under 'Hypoglycaemia which causes unconsciousness or fitting is an emergency', below)
may be required.
Glucagon
Glucagon can have variable absorption, as it is given SC or IM. It has a relatively slow onset of action and relies
on glycogen stores. Therefore, it may not be effective in cachectic patients, those with liver disease, and in
young children. It is contra-indicated in insulinoma and phaeochromocytoma. It also causes more insulin to
be released and creates the potential for secondary rebound hypoglycaemia.

27. A young boy fell on his outstretched hand and has presented with pain around the elbow. He
has absent radial pulse on the affected hand. What is the most likely dx?
a. Dislocated elbow
b. Angulated supracondylar fx
c. Undisplaced fx of radial head
d. Posterior dislocation of shoulder
Q. 1. What is the key?
Q. 2. What is the cause of absent radial pulse?
Q. 3. What is the immediate management?
Ans. 1. b.
Ans. 2. Damage or occlusion of the brachial artery is the cause of absent radial pulse.
Ans. 3. Open reduction to fix the occluded artery.

This scenario is classic for supracondylar fx,the distal humerus is displaced posteriorly causes vascular and
nerve injuries which if not addressed properly leads to volkmann contracture
posterior dislocation can compromise vascular supply but falling on outstretched hand causes it unlikely.
Other causes of pulseless radial artery except the two above ?
congenital absence,embolization, cervical rib, thoracic outlet syndrome
falling on an outstretched hands can lead to different type of fractures , like frac of clavicle , radius ,
supracondylar etc. but here it says that radial pulse is not felt ,which is a dreaded complication of
supracondylar frac

Mechanism of injury in elbow fractures and dislocation

Radial head and neck fractures Fall on to an outstretched hand

Elderly - indirect trauma by pull of triceps and brachioradialis


Olecranon fractures Children - direct blow to elbow

Fractures of the coronoid Fall on to an extended elbow as for elbow dislocation


process
Fractures of the distal humerus Fall on to an extended outstretched hand

Intercondylar fractures Direct or indirect blow to elbow

Condylar fractures Direct blow to a flexed elbow

Capitellum fracture Fall on to an outstretched hand, or direct trauma

Fall on to an extended elbow


Elbow dislocation Common in sport in the young

28. A 65yo woman presented with transient arm and leg weakness as well as a sudden loss of vision in the left
eye. Her symptoms resolved within the next couple of hours. What is the most
appropriate next inv?
a. CT brain
b. Echo
c. Doppler USG
d. Arteriography
e. 24h ECG
Q. 1. What is the key?
Q. 2. What is the likely diagnosis?
Q. 3. What will be seen on dopplar USG?
Q. 4. What is the management?
Ans. 1. The key is c.
Ans. 2. TIA
Ans. 3. Carotid artery narrowing
Ans. 4. Aspirin 300 mg daily for 2 weeks then aspirin 75mg daily and modified released dipiridamol 200mg 12
hourly.

unilateral blindness is typical sign for carotid stenosis


The duration is no more than 24 hours in tia
The most common source of emboli is the carotids, usually at the bifurcation.
People who have had a suspected TIA who need brain imaging (ie the vascular territory or pathology is
uncertain) should undergo diffusion-weighted MRI except where contra-indicated, in which case CT
should be used.
People who have a suspected TIA at high risk of stroke (eg, an ABCD2 score of 4 or above - see below)
in whom vascular territory or pathology is uncertain should undergo urgent brain imaging (preferably
diffusion-weighted MRI).
People with a suspected TIA at low risk of stroke (eg, an ABCD2 score of less than 4) in whom vascular
territory or pathology is uncertain should undergo brain imaging within one week of onset of
symptoms (preferably diffusion-weighted MRI).

Driving
Group 1 (car or motorcycle)

Must not drive for one month.


No need to notify DVLA after a single TIA.
Multiple TIAs over a short period: require three months free from further attacks before resuming
driving and DVLA should be notified.

Group 2 (lorry or bus)

Licence refused or revoked for one year following a stroke or TIA.

Assessment of the risk of stroke


An ABCD2 score of more than 4 suggests high risk of an early stroke.

Scoring System for Risk of Stroke after TIA (ABCD2 Score)

Age Age >60 1

Blood pressure BP>140 systolic and/or >90 diastolic 1

Clinical features Unilateral weakness 2

Speech disturbance without weakness 1

Other 0
Duration of symptoms >60 minutes 2

10-59 minutes 1

<10 minutes 0

Diabetes Presence of diabetes 1

RCP recommendations
Patients with suspected TIA who are at high risk of stroke (eg, an ABCD2 score of 4 or above)
should receive:
o Aspirin or clopidogrel (each as a 300 mg loading dose and then 75 mg daily) and a
statin started immediately.
o NB: clopidogrel is not licensed for the management of TIA and therefore the National
Institute for Health and Care Excellence (NICE) and the British National Formulary
(BNF) recommend aspirin plus modified-release dipyridamole.
o Specialist assessment and investigation within 24 hours of onset of symptoms.
People with crescendo TIA (two or more TIAs in a week), atrial fibrillation or those on
anticoagulants should be treated as being at high risk of stroke even if they may have an ABCD2
score of 3 or below.
Patients with suspected TIA who are at low risk of stroke (eg, an ABCD2 score of 3 or below)
should receive:
o Aspirin or clopidogrel (each as a 300 mg loading dose and then 75 mg daily) and a
statin.
o NB: clopidogrel is not licensed for the management of TIA and therefore NICE and the
BNF recommend aspirin plus modified-release dipyridamole.
Patients with TIA in atrial fibrillation should be anticoagulated in the TIA clinic once intracranial
bleeding has been excluded and if there are no other contra-indications.

29. A man complains of loss of sensation in his little and ring finger. Which nerve is most likely to be involved?
a. Median nerve
b. Ulnar nerve
c. Radial nerve
d. Long thoracic nerve
e. Axillary nerve
Ans. b.
30. A young man complains of double vision on seeing to the right. Which nerve is most likely to be involved?
a. Left abducens
b. Right abducens
c. Left trochlear
d. Right trochlear
e. Right oculomotor
Ans. b.

31. A 45yo man keeps having intrusive thoughts about having dirt under the bed. He cant keep
himself from thinking about these thoughts. If he tries to resist, he starts having palpitations.
What is the most likely dx?
a. OC personality
b. OCD
c. Schizophrenia
d. Panic disorder
e. Phobia
Q. 1. What is the key?
Q. 2. What is the managemment?
Ans. 1. b.
Ans. 2. CBT 1st line. SSRIs.
intrusive thoughts and anxiety when trying to fight them mean OCD.

32. A 33yo man presents with an itchy scaly annular rash on his thigh after a walk in the park. Which of the
following drugs will treat his condition?
a. Erythromycin
b. Doxycycline
c. Penicillin
d. Amoxicillin
Q. 1. What is the key?
Q. 2. What is the diagnosis?
Ans. 1. b.
Ans. 2. Lyme disease.

Lyme disease is caused by the spirochaete Borrelia burgdorferi and is spread by ticks

Features
early: erythema chronicum migrans + systemic features (fever, arthralgia)
CVS: heart block, myocarditis
neuro: cranial nerve palsies, meningitis

Investigation
serology: antibodies to Borrelia burgdorferi

Management
doxycycline if early disease. Amoxicillin is an alternative if doxycycline is contraindicated (e.g.
pregnancy)
ceftriaxone if disseminated disease
Jarisch-Herxheimer reaction is sometimes seen after initiating therapy: fever, rash, tachycardia after
first dose of antibiotic (more commonly seen in syphilis, another spirochaetal disease)

33. A pt with cerebral mets has polyuria and polydipsia. What part of the brain would be affected?
a. Cerebral cortex
b. Cerebellum
c. Diencephalon
d. Pons
e. Medulla
Q. 1. What is the key?
Q. 2. What is the diagnosis?
Ans. 1. c.
Ans. 2. Cranial diabetes insipidus.

diencephalon contains hypothalamus centre for thirst.


Other causes of polyuria and thirst.
Psychogenic or primary polydipsia (PP).
Diabetes mellitus.
Other osmotic diureses - eg, hypercalcaemia.
Diuretic abuse.

Investigation
high plasma osmolality, low urine osmolality
water deprivation test

34. A 32yo man presented with painless hematuria. He is hypertensive but the rest of the exam is
unremarkable. What is the most likely dx?
a. Polycystic kidneys
b. Ca bladder
c. Ca prostate
d. TTP
e. HUS
Q. 1. What is the key?
Q. 2. What are the points to justify your answer?
Q. What is the investigation of choice?
Ans. 1. A.
Ans. 2. Painless haematuria at an younger age with hypertension.
Ans. 3. Renal ultrasound.

autosomal dominant
Gross haematuria following trauma is a classic presenting feature of ADPKD,Advise against
participating in contact sports which risk abdominal trauma
polycystic kidneys can produce excess erythropoietin and hence raise Hb
Angiotensin-converting enzyme (ACE) inhibitors or angiotensin-II receptor antagonists are the
preferred choice

35. A 45yo female complains of pain in the inner side of her right thigh. She was dx with benign
ovarian mass on the right. Which nerve is responsible for this pain?
a. Femoral nerve
b. Obturator nerve
c. Iliohypogastric nerve
d. Ovarian branch of splanchic nerve
e. Pudendal nerve
Ans. B. [The Obturator nerve is responsible for the sensory innervation of the skin of the medial aspect of the
thigh].
36. A 37yo lady strongly believes that a famous politician has been sending her flowers every day
and is in love with her. However, this is not the case. What is the most likely dx?
a. Erotomania
b. Pyromania
c. Kleptomania
d. Trichotillomania
e. Grandiosity
Ans. 1. A. [Erotomania is a type of delusion in which the affected person believes that another person, usually
a stranger, high-status or famous person, is in love with them].
Pyromania fail to resist impulses to deliberately start fires, in order to relieve tension or for instant
gratification.
Kleptomania is the inability to refrain from the urge to steal items.
Trichotillomania compulsive urge to pull out one's hair, leading to noticeable hair loss and balding.
Grandiosity refers to an unrealistic sense of superiority.

37. A 3yo child has been brought with facial lacerations. On examination he has some cuts over his right cheek
and under the eye. The GCS on initial evaluation is 15. What is the appropriate next inv?
a. Skull XR
b. Facial XR
c. CT scan
d. MRI
e. Observation
Ans. b.
If u have cuts over ur cheek..there is high chance that there can be a fracture of a facial bone..hence by
observing u dont want to wait for a hematoma to form then go for a facial xr Skull xray isnt of any value as the
gcs is 15!

38. A 73yo woman has lymphadenopathy and splenomegaly. She feels well but has had recurrent
chest infections recently. Choose the single most likely blood film findings?
a. Atypical lymphocytes
b. Excess of mature lymphocytes
c. Plasma cells
d. Multiple immature granulocytes with blast cells
e. Numerous blast cells
Q. 1. What is the key?
Q. 2. What is the diagnosis?
Q. 3. Points in favour of your answer?
Ans. 1. B.
Ans. 2. CLL
Ans. 3. Age of patient (usually above 50 yrs), lymphadenopathy and splenomegaly, appearance of lymphocytes
(mature lymphocytes but functionally not normal). Repeated chest infection points towards abnormal
function of lymphocytes against infection.

It's CLL because in CML the risk age is 40-60 years, and in CBC there should be increased myeloid cells(which is
absent in options). If I exclude these factors, then the possible DX would be CLL and film finding is B.
1. Acute lymphoblastic leukemia- abnormal immature lymphocytes, (can be immature B or T lymphocytes)
called lymphoblasts. 2. Chronic lymphocytic leuaemia- Excess of mature lymphocytes. 3. Acute myeloid
leukaemia- blast cells (abnormal immature white cells) derived from myeloid stem cells. 4. Chronic myeloid
leukaemia- near normal granulocytes developed from abnormal stem cells (these are mature cells).
Also age is a factor ALL in any age but common in child, AML- age over 50, CLL common over age 60, CML- in
adults and commoner with increasing age.

39. A lady presents with itching around the breast and greenish foul smelling discharge from the
nipple. She had a similar episode before. What is the most likely dx?
a. Duct papilloma
b. Duct ectasia
c. Breast abscess
d. Periductal mastitis
e. Mammary duct fistula
Q. 1. What is the key?
Q. 2. What other options (breast conditions) frequently come in plab mcq?
Ans. 1. Key is b.
Ans. 2. 1. Breast ca 2. Duct papilloma and intraductal papilloma (both are same thing) 3. Mammary duct fistula
4. Breast abscess. X

Mammary duct Dilatation of the large breast ducts


ectasia
Most common around the menopause

May present with a tender lump around the areola +/- a green nipple discharge

If ruptures may cause local inflammation, sometimes referred to as 'plasma cell


mastitis'

Duct papilloma Local areas of epithelial proliferation in large mammary duct

May present with blood stained discharge

Breast abscessMore common in lactating women Red, hot tender swelling purulent discharge.

Periductal mastitis occurs when the ducts under the nipple become inflamed and infected. It's a benign
condition (not cancer), which can affect women of all ages but is more common in younger women. Symptoms
include: the breast becoming tender and hot to the touch. the skin may appear reddened.

Mammary duct fistulaThis is a communication between the skin and a major subareolar breast duct.
It may occur following incision and drainage of a non-lactating abscess, spontaneous discharge of a periareolar
mass or following biopsy of a periductal inflammatory mass.
Treatment is by excision under antibiotic cover.

40. A young male whose sclera was noted to be yellow by his colleagues has a hx of taking OTC
drugs for some pain. Tests showed raised bilirubin, ALT and AST normal. The provocation test
with IV nicotinic acid is positive and produces further rise in the serum bilirubin levels. What is
the most likely dx?
a. Acute hepatitis
b. Drug hypersensitivity
c. Gilberts syndrome
d. Acute pancreatitis
Q. 1. Does nicotinic acid provocation test can differentiate between CLD and Gilberts?
Q. 1. What is the key?
Q. 2. What are the points in favour of your diagnosis?
Ans. 1. C.
Ans. 2. Only bilirubin is increased but not the liver enzymes. Also positive nicotinic acid provocation test is in
its favour. X
Normal AST and ALT rules out any possible insult to the liver.
drug hypersensitivity will not give positive nicotinic acid provocation test.
autosomal recessive condition of defective bilirubin conjugation due to a deficiency of UDP glucuronyl
transferase
unconjugated hyperbilirubinemia (i.e. not in urine)
diagnosed around puberty, and aggravated by intercurrent illness, stress, fasting or after
administration of certain drugs
Investigation and management
investigation: rise in bilirubin following prolonged fasting or IV nicotinic acid
no treatment required

*********************
41. A 24yo biker has been rescued after being trapped under rocks for almost 12h. He complains of reddish
brown urine. His creatinine is 350umol/L and his urea is 15mmol/L. What is the most
imp step in the management of this patient?
a. Dialysis
b. IV NS
c. IV dextrose
d. IV KCl
e. Pain relief
Q. 1. What is the key?
Q. 2. What is the likely diagnosis?
Ans. 1. Key is B.
Ans. 2. Rhabdomyolysis. X

Crush syndrome is characterised by:

Hypovolaemic shock
Hyperkalaemia
Metabolic acidosis .
Acute kidney injury.
Disseminated intravascular coagulation (DIC).
ECG may show changes secondary to hyperkalaemia.

In the adult, a saline infusion of 1,500 ml/hour should be initiated during extrication. Early, vigorous hydration
(10 litres/day) helps preserve renal function.
42. A 74yo man who has been a smoker since he was 20 has recently been dx with SCLC. What
serum electrolyte picture will confirm the presence of SIADH?
a. High serum Na, low serum osmolarity, high urine osmolarity
b. Low serum Na, low serum osmolarity, high urine osmolarity
c. Low serum Na, high serum osmolarity, high urine osmolarity
d. High serum Na, low serum osmolarity, low urine osmolarity
e. High serum Na, high serum osmolarity, low urine osmolarity
Ans. b.
SIADH means excess ADH so water retention which will lead to dilutional hyponatremia and decrease in serum
osmolarity and sebsequent increase in urine osmolarity
Management: treat the cause and restrict fluid.
if severe: salt+- loop diuretics
vaptans can be used.
43. A man brought into the ED after being stabbed in the chest. Chest is bilaterally clear with
muffled heart sounds. BP is 60/nil. Pulse is 120bpm. JVP raised. What is the most likely dx?
a. Pulmonary embolism
b. Cardiac tamponade
c. Pericardial effusion
d. Hemothorax
e. Pneumothorax
Ans. b.
Beck's triad : low bp , muffled HS, raised jvp .. Cardiac tamponade
Features
dyspnoea
raised JVP, with an absent Y descent - this is due to the limited right ventricular filling
tachycardia
hypotension
muffled heart sounds
pulsus paradoxus
Kussmaul's sign (much debate about this)
ECG: electrical alternans

44. A 50yo pt is admitted for elective herniorraphy. Which of the following options will lead to a
postponement of the operation?
a. SBP 110mmHg
b. MI 2 months ago
c. Hgb 12g/dl
d. Pain around hernia
e. Abdominal distention
Key is B. After MI elective surgery should not be done before 6 months post MI.
Criteria for postponing elective surgery. ...Hb <10 ,
Plt count <50000
Systolic BP... <90
Uncontrolled HTN, DM , asthma
MI within 3 months
45. A 32yo woman of 39wks gestation attends the antenatal day unit feeling very unwell with
sudden onset of epigastric pain associated with nausea and vomiting. Her temp is 36.7C. Exam:
she is found to have RUQ tenderness. Her blood results show mild anemia, low platelets,
elevated liver enzymes and hemolysis. What is the most likely dx?
a. Acute fatty liver of pregnancy
b. Acute pyelonephritis
c. Cholecystitis
d. HELLP syndrome
e. Acute hepatitis
Q. 1. What is the key?
Q. 2. What is the main treatment
Ans. 1. D.
Ans. 2. The main treatment is to deliver the baby as soon as possible [as early as after 34 weeks if multisystem
disease is present].
HELLP syndrome is a group of symptoms that occurs in pregnant women who have pre-eclampsia or eclampsia
and who also show signs of liver damage and abnormalities in blood clotting.
H aemolysis
EL (elevated liver) enzymes
LP (low platelet) count

46. A woman comes with an ulcerated lesion 3 cm in the labia majorum. What is the lymphatic drainage of this
area?
a. External iliac
b. Superficial inguinal LN
c. Para-aortic
d. Iliac
e. Aortic
Ans. Key is B. Superficial inguinal LN.

47. A man post-cholecystectomy presented with jaundice, fever and dark urine. What is the most
diagnostic inv?
a. ERCP
b. USG Abdomen
c. CT Scan
d. MRCP
e. MRI
Q. 1. What is the key?
Q. 2. What is the likely diagnosis?
Ans. 1. A. ERCP
Ans. 2. Cholangitis. Post cholescytectomy syndrome?
investigation of choice in post cholecystectomy syndrome is ercp

Charcot's triad of right upper quadrant (RUQ) pain, fever and jaundice
Management
intravenous antibiotics
endoscopic retrograde cholangiopancreatography (ERCP) after 24-48 hours to relieve any obstruction

ERCP contraindications:
Acute pancreatitis (unless persistently raised or worsening bilirubin suggests ongoing obstruction)
Previous pancreatoduodenectomy
Coagulation disorder if sphincterotomy planned
Recent myocardial infarction
Inadequate surgical back-up
History of contrast dye anaphylaxis
Poor health condition for surgery
Severe cardiopulmonary disease

48. A 79yo stumbled and sustained a minor head injury 2 weeks ago. He has become increasingly
confused, drowsy and unsteady. He has a GCS of 13. He takes warfarin for Afib. What is the most likely dx?
a. Extradural hemorrhage
b. Cerebellar hemorrhage
c. Epidural hemorrhage
d. Subdural hemorrhage
e. Subarachnoid hemorrhage
Q. 1. What is the key?
Q. 2. What is the management?
Ans. 1. D.
Ans. 2. 1st line: Evacuation by burr hole craniostomy. 2nd line: Craniotomy if the clot is organized.
The gradual onset of symptoms supports the dx
Subdural hematoma as old shrunken /alcoholic brains are prone to develop tear in the veins which bleed
slowly and eventually the hematoma gets big enough to show the symptoms
Also in this case the patient is on warfarin

Type of injury Notes

Extradural Often results from acceleration-deceleration trauma or a blow to the side of the head.
(epidural) The majority of epidural haematomas occur in the temporal region where skull
haematoma fractures cause a rupture of the middle meningeal artery.

Features

features of raised intracranial pressure


some patients may exhibit a lucid interval

Subdural Bleeding into the outermost meningeal layer. Most commonly occur around the frontal
haematoma and parietal lobes.

Risk factors include old age, alcoholism and anticoagulation.

Slower onset of symptoms than a epidural haematoma.

Subarachnoid Usually occurs spontaneously in the context of a ruptured cerebral aneurysm but may
be seen in association with other injuries when a patient has sustained a traumatic
haemorrhage brain injury

worst headache.

49. A 25yo female complains of intermittent pain in her fingers. She describes episodes of
numbness and burning of the fingers. She wears gloves whenever she leaves the house. What is
the most probable dx?
a. Kawasaki disease
b. Takayasu arteritis
c. Buergers disease
d. Embolism
e. Raynauds phenomenon
Ans is e.
wearing glove is the catch phrase
Takayasu;pulseless disease, will present with other symptoms like unequal pulses,diziness,weakness
fr buergers hx of smoking...age usually more then 40.buerger's disease should present with smoking hx of an
aged male.
Management
first-line: calcium channel blockers e.g. nifedipine
IV prostacyclin infusions: effects may last several weeks/months

50. A 22yo lady has been unwell for some time. She came to the hospital with complaints of fever
and painful vesicles in her left hear. What is the most probable dx?
a. Acne
b. Herpes zoster
c. Chicken pox
d. Insect bite
e. Cellulitis
Q. 1. What is the key?
Q. 2. What is the specific name of the condition?
Ans. 1. Herpes Zoster
Ans. 2. Herpes zoster oticus/Ramsay hunt syndrome.

Ramsey hunt syndrome


Painful vesicles in her left ear
Features
auricular pain is often the first feature
facial nerve palsy
vesicular rash around the ear
other features include vertigo and tinnitus

Management
oral aciclovir and corticosteroids are usually given
51. A 5yo girl had earache and some yellowish foul smelling discharge, perforation at the attic and
conductive hearing loss. She has no past hx of any ear infections. What is the most appropriate
dx?
a. Acute OM
b. OM with effusion
c. Acquired cholesteatoma
d. Congenital cholesteatoma
e. Otitis externa
Q. 1. What is the key?
Q.2. What are the points in favour of your diagnosis?
Ans. 1. The key is c. Acquired cholesteatoma.

Ans. 2. Ans. 1. The key is c. Acquired cholesteatoma.


Ans. 2. acquired cholesteatomas develop as a result of chronic middle ear infection and are usually associated
with perforation of the tympanic membrane at the attic (mass is seen in attic with perforation at pars flaccida-
in contrast to medial to tympanic membrane which is in congenital). Clinical presentation usually consists of
conductive hearing loss, often with purulent discharge from the ear
In congenital
mass medial to the tympanic membrane
normal tympanic membrane
no previous history of ear discharge, perforation or ear surgery.

52. A female with T1DM would like to know about an deficiency of vitamins in pregnancy that can
be harmful. A deficiency of which vitamin can lead to teratogenic effects in the child?
a. Folic acid
b. Vit B12
c. Thiamine
d. Riboflavin
e. Pyridoxine

Ans. A. Folic acid.

Diet: To prevent neural tube defects (NTD) and cleft lip, all should have folate rich foods + folic acid 0.4mg

daily >1 month pre-conception till 13wks (5mg/day if past NTD, on anti epileptics, obese (BMI 30), HIV+ve on

co-trimoxazole prophylaxis, diabetic or sickle cell disease.

Smoking: decreases ovulations, causes abnormal sperm production ( less penetrating capacity),

rates of miscarriage (2), and is associated with preterm labour and lighter-for-dates babies placenta praevia

and abruption. Reduced reading ability in smokers children up to 11yrs old shows that long term effects are

important.
Alcohol consumption: High levels of consumption are known to cause the fetal alcohol syndrome. Mild

drinking eg 12U/wk has not been shown to adversely affect the fetus. Especially harmful in weeks 3-

8.Miscarriage rates are higher among drinkers of alcohol

53. A 23yo woman has been having pain at the base of her thumb, the pain is reproduced when
lifting her 3 month old baby or changing diapers and also with forceful abduction of the thumb
against resistance. What is the likely cause?
a. Avascular necrosis of scaphoid
b. Trigger finger.
c. De Quervains tenosynovitis
Q. 1. What is the key?
Q. 2. How will you diagnose the case?
Ans. 1. The key is c. De Quervains tenosinovitis.
Ans. 2. Can be diagnosed by Finkelsteins test:
The physician grasps the thumb and the hand is ulnar deviated sharply. If sharp pain occurs along the distal
radius (top of forearm, about an inch below the wrist), de Quervain's syndrome is likely.

De Quervain's tenosynovitis is a common condition in which the sheath containing the extensor pollicis brevis
and abductor pollicis longus tendons is inflamed. It typically affects females aged 30 - 50 years old

Features
pain on the radial side of the wrist
tenderness over the radial styloid process
abduction of the thumb against resistance is painful
Finkelstein's test: with the thumb is flexed across the palm of the hand, pain is reproduced by
movement of the wrist into flexion and ulnar deviation

Management
analgesia
steroid injection
immobilisation with a thumb splint (spica) may be effective
surgical treatment is sometimes required

54. A 6m child presents with fever and cough. His mother has rushed him to the ED asking for help. Exam:
temp=39C and the child is feeding poorly. Dx?
a. Bronchiolitis
b. Asthma
c. Bronchitis
Q. 1. What is the key?
Q.2. What is the management?
Ans. 1. A. Bronchiolitis.
Ans. 2. Management: 1. Oxygen inhalation 2. Nasogastric feeding. DONT USE: i) bronchodilator ii) steroid iii)
antibiotics routinely. [OHCS, page-160]
Acute bronchiolitis is the big lung infection in infants
Symptoms: coryza precedes cough, low fever, tachypnoea, wheeze, inspiratory crackles, apnoea, intercostal
recession cyanosis.
Cause: Winter respiratory syncytial virus. Others: Mycoplasma, parainfluenza, adenoviruses. Those <6 months
old are most at risk.
Signs prompting admission: Poor feeding, >50 breaths/min, apnoea, dehydration, rib recession, patient or
parental exhaustion

Tests: If severe: CXR (hyperinflation); blood gases/SpO2; FBC.

Treatment: O2 (stop when SpO2 92%); nasogastric feeds. 5% need ventilating (mortality 1%; 33% if
symptomatic congenital heart disease). Dont use bronchiodilators and steroids routinely

55. A 75yo man collapsed while walking in his garden. He recovered fully within 30 mins with BP
110/80 mmHg and regular pulse of 70bpm. He has a systolic murmur on examination. His
activities have been reduced lately which he attributes to old age. What is the definitive
diagnostic inv that will assist you with his condition?
a. ECG
b. Echo
c. 24h ECG monitoring
d. 24h BP monitoring
e. Prv CIN
Q. 1. What is the key?
Q. 2. What are the possible causes of this syncope?
Ans. 1. B. Echo.
Ans. 2. i) Aortic stenosis more likely in elderly. ii) hypertrophic cardiomyopathy less likely in this age as
presentation may present in an earlier age.

Aortic stenosis (AS)


Causes: Senile calcification is the commonest. Others: congenital (bicuspid valve, Williams syndrome,
rheumatic heart disease.
Presentation: Think of AS in any elderly person with chest pain, exertional dyspnoea or syncope. The classic
triad includes angina, syncope, and heart failure (usually after age 60). Also: dyspnoea; dizziness; faints;
systemic emboli if infective endocarditis; sudden death.
Signs: Slow rising pulse with narrow pulse pressure (feel for diminished and delayed carotid upstrokeparvus
et tardus); heaving, non-displaced apex beat; LV heave; aortic thrill; ejection systolic murmur (heard at the
base, left sternal edge and the aortic area, radiates to the carotids).
There may be an ejection click (pliable valve) or an S4 (said to occur more often with bicuspid valves, but not
in all populations).
Tests: ECG: P-mitrale, LVH with strain pattern; LBBB or complete AV block (calcified ring). CXR: LVH; calcified
aortic valve post-stenotic dilatation of ascending aorta.
Echo: diagnostic.
Doppler echo can estimate the gradient across valves
Cardiac catheter can assess: valve gradient; LV function; coronary artery disease
Management: If symptomatic, prognosis is poor without surgery.
If moderate-to-severe and treated medically, mortality can be as high as 50% at 2yrs, therefore prompt valve
replacement is usually recommended.
In asymptomatic patients with severe AS and a deteriorating ECG, valve replacement is also recommended. If
the patient is not medically fit for surgery, percutaneous valvuloplasty/replacement (TAVI = transcatheter
aortic valve implantation) may be attempted.

56. A 35yo man with a hx of schizophrenia is brought to the ER by his friends due to drowsiness. On
examination he is generally rigid. A dx of neuroleptic malignant syndrome except:
a. Renal failure
b. Pyrexia
c. Elevated creatinine kinase
d. Usually occurs after prolonged tx
e. Tachycardia
Ans. Key is D. Usually after prolonged tx. It usually occurs within 10 days of starting treatment. renal failure
is the wrong answer as neuroleptic syndrome can lead to renal failure so we have to give IV fluids to prevent
it.
Cause: antipsychotics or dopamenergic drugs (levodopa)
Management: STOP the drug causing it. IV fluids, Dantrolene, Bromocriptine
57. A 33yo drug addict wants to quit. She says she is ready to stop the drug abuse. She is supported by her
friends and family. What drug tx would you give her?
a. Benzodiazepines
b. Diazipoxide
c. Lithium
d. Methadone
e. Disulfiram
Q.1. What is the key.
Q.2. What drugs should you use in i) tobacco abuse and in ii) alcohol abuse?
Ans. 1. Key is d. Methadone. (used in opiate abuse). Nalexone is the antidote.
Ans. 2. i) tobacco abuse: a) bupropion ii) alcohol: a) acamprosate decreases craving b) disulfirum is a
deterrent.

58. A 16m child presents with drooling, sore throat and loss of voice. He has fever with a temp of
38.2C. What is your next step towards management?
a. Direct pharynoscopy
b. Call ENT surgeon
c. Call anesthesiologist
d. IV fuilds
e. Start antibiotics
Q. 1. What is the key?
Q. 2. What is the diagnosis?
Q. What is the urgent management?
Ans. 1.Key is C. Call anesthesiologist.
Ans. 2. Diagnosis is Acute epiglottitis.
Ans. 3. In given case urgent intubation is needed to secure airway to prevent blockage of respiration.

Differential: croup. Croup has barking cough which is worse at night and there is no drooling of saliva NO
COUGH IN EPIGLOTITTIS. Croup caused by parainfluenza while epiglottitis is caused by H.influenze
Acute epiglottitis is rarer than croup but mortality is high: 1% if respiratory distress. Its an emergency as
respiratory arrest can occur.
Presentation: Often, history is short, septicaemia is rapid, and cough is absent. Also: sore throat (100%), fever
(88%), dyspnoea (78%), voice change (75%), dysphagia (76%), tender anterior neck cellulitis (27%),
hoarseness (21%), pharyngitis (20%), anterior neck nodes (9%), drooling (head for ward tongue out), prefers to
sit, refusal to swallow,
Cause: Haemophilus (vaccination has reduced prevalence); Strep pyogenes.
Investigation: Fibre-optic laryngoscopy remains the 'gold standard' for diagnosing epiglottitis
Management: Take to ITU; dont examine throat (causes resp. arrest). Give O2 by mast, Give nebulized
adrenaline, IV dexamethasone, antibiotics, antipyretics until the anesthetist arrives. Definitive management is
intubation

59. A 62yo woman complains of unsteadiness when walking. On examination she has pyramidal
weakness of her left lower limb and reduced pain and temp sensation on right leg and right side
of trunk up to the umbilicus. Joint position sense is impaired at her left great toe but is normal
elsewhere. She has a definite left extensor plantar response and the right plantar response is
equivocal. Where is the lesion?
a. Left cervical cord
b. Midline mid-thoracic cord
c. Right mid-thoracic cord
d. Left mid-thoracic cord
e. Left lumbo-sacral plexus
Q. 1. What is the key?
Q. 2. What is the name of this condition?
Ans. 1. The key is d. Left mid-thoracic cord.
Ans. 2. Brown-sequard syndrome.

Pain & temperature: carried by lateral spinothalamic tract dicussate to the opposite side within the spinal
cord. Vibration, proprioception: carried by the medial lemniscal system decussate at the junction of pons
and medulla. Upper motor neurons in pyramidal tract also decussate close to medulla. So from this we know
that the lesion is on the left side. And since the symptoms are below the umbillicus it rules out a cervical
lesion. Lesion of lumbo sacral plexus will impair the whole of the lower limb. So the correct answer is D.

Brown Sequard syndrome:


A lesion in one half of the spinal cord (due to hemisection or unilateral cord lesion)
Presentation: Ipsilateral UMN weakness below the lesion (severed corticospinal tract, causing spastic
paraparesis, brisk reflexes, extensor plantars) Ipsilateral loss of proprioception and vibration (dorsal column
severed) Contralateral loss of pain and temperature sensation (severed spinothalamic tract which has
crossed over
Causes: Bullet, stab, dart, kick, tumour, disc hernia, cervical spondylosis, MS, neuroschistosomiasis, myelitis,
septic emboli (eg meningococcal).
Imaging: MRI
60. A 26yo man present to ED with increasing SOB on left side and chest pain. He has been a heavy smoker for
the past 4 years. He doesnt have any past med hx. What is the likely dx?
a. Pulmonary embolism
b. MI
c. Asthma
d. Pleural effusion
e. Pneumothorax
Q. 1. What is the key?
Q. 2. What are the points in favour of your diagnosis?
Q. 3. What is the cause of the disease in this case?
Ans. 1. The key is e. Pneumothorax.
Ans. 2. Increased shortness of breath and chest pain with no past medical history.
Ans. 3. Heavy smoking. Tobacco is a risk factor for spontaneous pneumothorax.
Pneumothorax
Causes Often spontaneous (especially in young thin men) due to rupture of a subpleural bulla.
Other causes: asthma; COPD; TB; pneumonia; lung abscess; carcinoma; cystic fibrosis; lung fibrosis;
sarcoidosis; connective tissue disorders (Marfans sy., EhlersDanlos sy.), trauma; iatrogenic (subclavian CVP
line insertion, pleural aspiration/ biopsy, transbronchial biopsy, liver biopsy, +ve pressure ventilation).
Symptoms: There may be no symptoms (especially if fit, young and small pneumothorax) or there may be
sudden onset of dyspnoea and/or pleuritic chest pain. Patients with asthma or COPD may present with a
sudden deterioration. Mechanically ventilated patients may present with hypoxia or an increase in ventilation
pressures.
Signs: Reduced expansion, hyper-resonance to percussion and diminished breath sounds on the affected side.
With a tension pneumothorax, the trachea will be deviated away from the affected side

Management:
61. A pt with hepatocellular ca has raised levels of ferritin. What is the most probable cause?
a. Hemochromatosis
b. A1 antitrypsin def
c. Cystic fibrosis
Ans. Haemochromatosis.

Haemochromatosis... Autosomal recessive.


SYMPTOMS bronzing of skin, DM, hepatomegly, arthropathy. Can also cause infertility, arrhythmias,
neurological symptoms. Liver fibrosis, cirrhosis & HCC.
INVESTIGATIONS: Serum ferritin then genetic testing for HFE mutations.
TREATMENT: phlebotomy OR liver transplant

62. A woman has electric pains in her face that start with the jaw and move upwards. Her corneal
reflexes are normal. What is the most likely dx?
a. Atypical face pain
b. Trigeminal neuralgia
c. Tempero-mandibular joint dysfunction
d. GCA
e. Herpes zoster
Q. 1. What is the key?
Q. 2. What are the options mentioned are possible causes of absent corneal reflex?
Ans. 1. Key is b. Trigeminal neuralgia.
Ans. 2. Possible options are 1. Trigeminal neuralgia 2. Herpes zoster ophthalmicus

Trigerminal Neuralgia...Facial pains. PRESENTATION: The episodes are sporadic and sudden and often like
'electric shocks', lasting from a few seconds to several minutes. Pain is unilateral, brief, stabbing, recurrent in
the distribution of CN5. Can be provoked by light touch to the face, eating, cold winds, or vibrations typically
occurs after shaving, brushing teeth.
Cause is a compression of CN5.
No Investigations
TREATMENT: Carbamezapine is the first line. Rhizotomy (surgery) may also be done

63. A 32yo man presented with slow progressive dysphagia. There is past hx of retro-sternal
discomfort and he has been treated with prokinetics and H2 blockers. What is the probably dx?
a. Foreign body
b. Plummer vinson syndrome
c. Pharyngeal pouch
d. Peptic stricture
e. Esophageal Ca
Q. 1. What is the key?
Q. 2. What is the underlying cause of this stricture?
Ans. 1. The key is D. Peptic stricture.
Ans. 2. The underlying cause is Gastro-oesophageal reflux.

Points not in favor of CA: Age (32yrs), no anemia, anorexia, lethargy etc mentioned.

Peptic Stricture
PRESENTATION: heartburn, dysphagia, impaction of food, weight loss, and chest pain. There can be
progressive dysphagia, weight loss & anemia.
CAUSES: History of GERD, corrosive intake, drugs like NSAIDs
INVESTIGATIONS: Endoscopy (risk of perforation) Barium swallow
TREATMENT: Benign: endoscopic baloon dilation. Malignant: oesophagectomy

64. A 56yo man comes with hx of right sided weakness & left sided visual loss. Where is the
occlusion?
a. Ant meningeal artery
b. Mid meningeal artery
c. Mid cerebral artery
d. Carotid artery
e. Ant cerebral artery
f. Ant communicating artery
Q. 1. What is the key?
Q. 2. How will you differentiate between middle cerebral artery occlusion from anterior cerebral artery
occlusion?
Ans. 1. The key is d. Carotid artery.
Ans. 2.
i) Middle cerebral artery occlusion: paralysis or weakness of contralateral face and arm
(faciobracheal). Sensory loss of the contralateral face and arm.
ii) Anterior cerebral artery occlusion: paralysis or weakness of the contralateral foot and leg.
Sensory loss at the contralateral foot and leg.

Carotid Artery occlusion:


PRESENTATION: Patients may present with TIAs or CVEs.
Typical symptoms are contralateral weakness or sensory disturbance, ipsilateral blindness, and (if the
dominant hemisphere is involved) dysphasia, aphasia or speech apraxia.
Carotid bruit may or may not be present
INVESTIGATIONS: For diagnosis: CAROTID ANGIOGRAPHY GOLD STANDARD. MR angio and angio CT can also
be used.
Echo colour Doppler ultrasonography is the screening method of choice
TREATMENT: Medical: Antiplatelets, Anti HTN, Statins
Surgery: Carotid endartarectomy. Symptomatic patients with greater than 50% stenosis and healthy,
asymptomatic patients with greater than 60% stenosis warrant consideration for carotid endarterectomy.

65. A young college student is found in his dorm unconscious. He has tachyarrhythmia and high
fever. He also seems to be bleeding from his nose, which on examination shows a perforated
nasal septum. What is the most likely dx?
a. Marijuana OD
b. Cocaine OD
c. Heroin OD
d. Alcohol OD
e. CO poisoning
Q. 1. What is the key?
Q. 2. What are the points that favours the diagnosis in given question?
Q. 3. What are other important findings?
Ans. 1. Key is B. Cocaine overdose.
Ans. 2. Points in favour: i) Tachyrhythmia ii) High fever iii) perforated nasal septum iv) unconsciousness
Ans. 3. Other findings: i) Psychiatric: anxiety, paranoia ii) Tachypnoea iii) Increased energy and talking rapidly
iv) Dilated pupils. Also: [rhabdomyolysis, metabolic acidosis, convulsion].

COCAINE may be snored, taken via IV or smoked.


PRESENTATION: occasional use produces euphoria, increased alertness and feelings of self-confidence and
competence
frequent repeated use causes tachycardia, twitching, insomnia and anxiety
ADDICTION: can result in perforated nasal septum, psych problems.
The patient may present in anxiety, paranoia, they may ask for help.
MANAGEMENT: CBT, self help groups. Benzodiazepines are first line drugs, anti depressents like SSRIs but
donot use with cocaine (causes SSRI syndrome). Beta blockers for anxiety

66. A 56yo pt whose pain was relieved by oral Morphine, now presents with progressively
worsening pain relieved by increasing the dose of oral morphine. However, the pt complains
that the increased morphine makes him drowsy and his is unable to carry out his daily activities.
What is the next step in his management?
a. Oral oxycodone
b. Oral tramadol
c. PCA
d. IV Fentanyl
e. Diamorphine
Ans. Key is oral oxycodon.

If there are intolerable side effects to morphine go for oral oxycodone


Pain ladder: NSAIDs, Mild opioids, strong opioids.
Once on one step of the ladder do not go back.
NSAIDs are good for bone pain.
Morphine Start with oral solution 510mg/4h PO with an equal breakthrough dose
as often as required. A double dose at bedtime can enable a good nights sleep. Patient
needs will vary greatly and there is no maximum dose; aim to control symptoms
with minimum side-effects. If not effective, increase doses in 3050% increments
(5mg10mg20mg30mg45mg). Change to modified release preparations (eg
MST Continus 12h) once daily needs are known by totalling 24h use and dividing by
2. Prescribe 1/6th of the total daily dose as oral solution for breakthrough pain. Sideeff
ects (common) are drowsiness, nausea/vomiting, constipation and dry mouth.
Hallucinations and myoclonic jerks are signs of toxicity and should prompt dose review.
If the oral route is unavailable try morphine/diamorphine IV/SC (see BOX for
conversions). If difficulty tolerating morphine/diamorphine, try oxycodone PO/IV/SC/
PR, starting at an equivalent dose. It is as effective as morphine and is a useful 2nd-line
opioid with a different range of receptor activity. 61 OxyNorm is the oral liquid form.
There are also fentanyl transdermal patches which should usually be started under
specialist supervision (after opioid dose requirements have been established). Remove
after 72h, and place a new patch at a different site. 45mg oral morphine/24h
is approximately equivalent to a 12mcg/h fentanyl patch.

67. A 30yo man presents with a 5cm neck mass anterior to the sternocleido-mastoid muscle on the left side in
its upper third. He states that the swelling has been treated with antibiotics for
infection in the past. Whats the most likely cause?
a. Branchial cyst
b. Parotitis
c. Pharyngeal pouch
d. Thyroglossal cyst
e. Thyroid swelling
Q. 1. What is the key?
Q. 2. Justify your answer.
Ans. 1. The key is A. Branchial cyst.
Ans. 2. i) Branchial cyst is anterior triangular lump. [parotid is also anterior triangular lump but it regresses
with appropriate treatment]. ii) pharyngeal pouch is posterior triangular lump. iii) Thyroglossal is midline
lump. iv) thyroid swelling moves with swallowing.

Branchial cysts emerge under the anterior border of sternocleidomastoid where the upper third meets the
middle third (age <20yrs).
CAUSE: Due to non-disappearance of the cervical sinus (where 2nd branchial arch grows down over 3rd and
4th)
TREAT by excision
If lump in the supero-posterior area of the anterior triangle, is it a parotid tumour

68. An 18yo man is rushed into the ER by his friends who left him immediately before they could be
interviewed by staff. He is semiconscious, RR=8/min, BP=120/70mmHg, pulse=60bpm. He is
noted to have needle track marks on his arms and his pupils are small. What is the single best
initial tx?
a. Insulin
b. Naloxone
c. Methadone
d. Gastric lavage
Q. 1. What is the key?
Q. 2. What is the diagnosis?
Q. 3. What are the points in favour of the diagnosis?
Ans.1. The key is B. Naloxone.
Ans. 2. The diagnosis is opiate overdose.
Ans. 3. Points in favour are: i) reduced consciousness ii) RR 8/min (12<) iii) hypotension (here lower normal) iv)
miosis v) needle track marks on his arms.

Opioid overdose presents with the usual Adverse effects of opioids. This is a typical presentation. Treatment
IS WITH NALOXONE IV/IM/SC
OPIOID WITHDRAWAL SYMPTOMS:
Sweating. Watering eyes. Rhinorrhoea Yawning Feeling hot and cold. Anorexia and abdominal cramps.
Nausea, vomiting and diarrhoea. Tremor. Insomnia, restlessness, anxiety and irritability. Generalised
aches and pains. Tachycardia, hypertension. Goose flesh (goosebumps). Dilated pupils. Increased
bowel sounds. Coughing.
COMPLICATIONS: Skin infection at injection sites (can be severe; necrotising fasciitis can occur).
Septicaemia. Infective endocarditis. HIV infection. Hepatitis A, B and C infection. Tuberculosis infection.

TREATMENT: Methadone or buprenorphine. Stabilize the patient on either of the two.


Naltrexone can be used once the patient is detoxified.

69. A 30yo man and wife present to the reproductive endocrine clinic because of infertility. The man is tall, has
bilateral gynecomastia. Examination of the testes reveals bilateral small, firm testes.
Which of the following inv is most helpful in dx?
a. CT of pituitary
b. Chromosomal analysis
c. Measure of serum gonadotropins
d. Measure of serum testosterone
Q. 1. What is the key?
Q. 2. What is the diagnosis?
Q. 3. What are the points in favour of your diagnosis?
Ans. 1. The key is B. Chromosomal analysis.
Ans. 2. The diagnosis is Klinefelters syndrome. (xxy)
Ans. 3. The points in favour are: i) Infertility ii) Tall stature iii) Bilateral gynaecomastia iv) Bilateral small firm
testes.

Klinefelters syndrome: (47,XXY, 48,XXYY polysomy or a mosaic 47,XXY/46,XY) Turners is XO with NO Barr
body.
PRESENTATION: Infertility & small testis (most common & most imp) gynecomastia, lack of secondary sexual
characteristics, tall and slender and learning disablities (delayed speech, behavioral problems)
Investigations: Before birth via amniocentesis or CVS.
Later serum testosterone is low. FSH & LH are high (FSH>LH)
Chromosome karyotyping gives the deifinitive diagnosis
TREATMENT: 1. Testosterone replacement. 2.Intracytoplasmic injection of sperm. 3. Surgery for gynecomastia

70. An 18yo female just received her A-Level results and she didnt get into the university of her
choice. She was brought into the ED after ingestion of 24 paracetamol tablets. Exam: confused
and tired. Initial management has been done. Inv after 24h: normal CBC, ABG = pH7.1, PT=17s,
Bilirubin=4umol/L, creatinine=83umol/L. What is the next step in management?
a. Observation for another 24h
b. Refer to psychologist
c. Give N-Acetylcysteine
d. Discharge with psychiatry referral
e. Liver transplantation
Q. 1. What is the key?
Q. 2. What are the indications of this management?
Ans. 1. The key is E. Liver transplantation.
Ans. 2. King's College Hospital criteria for liver transplantation in paracetamol-induced acute liver failure.
arterial pH <7.3 or arterial lactate >3.0 mmol/L after adequate fluid resuscitation, OR
if all three of the following occur in a 24-hour period:
Creatinine >300 mol/L.
PT >100 seconds (INR >6.5).
Grade III/IV encephalopathy.

PARACETAMOL POISONING: >150mg/kg or 12g total


PRESENTATION: Hepatic damage shown by deranged LFTs occurs after 24hrs. Patients may develop encaph,
hypoglycemia, ARF
INVESTIGATIONS: Paracetamol levels: 4hrs post ingestion, if time is >4hr or staggered overdose

Any alcohol taken (acute alcohol ingestion will inhibit liver enzymes and may reduce the production of the
toxin NAPQI, whereas chronic alcoholism may increase it)

MANAGEMENT:
If presentation is within the first 4 hours give activated charcoal
All patients who have a timed plasma paracetamol level plotted on or above the line drawn between
100 mg/L at 4 hours and 15 mg/L at 15 hours after ingestion, should receive acetylcysteine.
If time unknown (even in staggered dose) give N-Acetyl cysteine without delay
NAC most effective in the first 8 hrs.
NAC can be given during pregnancy
Beware if the patient is on any P450 enzyme inducer medicines as they increase the toxicity
Refer to ICU if there is fulminant liver failure - those treated with N-acetylcysteine (NAC) to the medical
team and all para-suicides to the psychiatric team.

71. A 75yo alcoholic presents with a mass up to umbilicus, urinary dribbling, incontinence, and
clothes smelling of ammonia. What is the next step in management?
a. Urethral catheter
b. Suprapubic catheter
c. Antibiotics
d. Condom catheter
e. Nephrostomy
Q. 1. What is the key?
Q. 2. What is the cause of this retention?
Ans. 1. The key is A. Urethral catheter.
Ans. 2. Alcohol consumption (it is rather a less common cause of urinary retention).

ACUTE URINARY RETENTION

Causes of urinary retention


In men - BPH, meatal stenosis, paraphimosis, penile constricting bands, phimosis, prostate cancer.
In women - prolapse (cystocele, rectocele, uterine), pelvic mass (gynaecological malignancy, uterine
fibroid, ovarian cyst), retroverted gravid uterus.
In both - bladder calculi, bladder cancer, faecal impaction, gastrointestinal or retroperitoneal
malignancy, urethral strictures, foreign bodies, stones.
Infectious and inflammatory:
In men - balanitis, prostatitis and prostatic abscess.
In women - acute vulvovaginitis, vaginal lichen planus and lichen sclerosis, vaginal pemphigus.
In both - bilharzia, cystitis, herpes simplex virus (particularly primary infection), peri-urethral abscess,
varicella-zoster virus.
Drug-related:
Up to 10% AUR episodes are thought to be attributable to drugs. Those known to increase risk include:
Anticholinergics (eg, antipsychotic drugs, antidepressant agents, anticholinergic respiratory agents).
Opioids and anaesthetics. Alpha-adrenoceptor agonists. Benzodiazepines. NSAIDs Detrusor relaxants.
Calcium-channel blockers Antihistamines. Alcohol.
Neurological:
More often causing chronic retention but may cause AUR:
Autonomic or peripheral nerve (eg, autonomic neuropathy, diabetes mellitus, Guillain-Barr syndrome,
pernicious anaemia, poliomyelitis, radical pelvic surgery, spinal cord trauma, tabes dorsalis).
Brain (eg, cardiovascular disease (CVD), MS, neoplasm, normal pressure hydrocephalus, Parkinson's
disease).
Spinal cord (eg, invertebral disc disease, meningomyelocele, MS, spina bifida occulta, spinal cord
haematoma or abscess, spinal cord trauma, spinal stenosis, spinovascular disease, transverse myelitis,
tumours, cauda equina).
Other:
In men - penile trauma, fracture, or laceration.
In women - postpartum complications (increased risk with instrumental delivery, prolonged labour and
Caesarean section),[2]urethral sphincter dysfunction (Fowler's syndrome).
In both - pelvic trauma, iatrogenic, psychogenic.

MANAGEMENT: Foleys catheter first line. If contraindicated or failed refer to urology or try suprapubic
catheterization

COMPLICATIONS: UTIs, Renal failure, Post retention diureses and hematuria

72. In CRF, main cause of Vit D deficiency is the failure of:


a. Vit D absorption in intestines
b. 25 alpha hydroxylation of Vit D
c. Excess Vit D loss in urine
d. 1 alpha hydroxylation of Vit D
e. Availability of Vit D precursors
Ans. The key is D. 1 alpha hydroxylation of Vit D [kidney] [25 alpha hydroxilation of Vit D- liver].

VITAMIN D: FUNCTION: Absorption of calcium and phosphorus from GIT


SOURCES: Oily fish, fortified food, skin synthesized VitD using sunlight
Deficiency causes Rickets in children and ostomalacia in adults
Causes of Deficiency: Increased demand in pregnancy, breast feeding, Malabsorptive diseases like Crohns,
coeliac, pancreatic insufficiency, CKD, Liver diseases
Treatment: Take vit D supplements in the form of calciferol. Tablets or injections. Injections can be effective
for upto 6 months.
All pregnant and breastfeeding women should take a daily supplement containing 10 micrograms of vitamin D
All children aged 6months to 6years should take daily vitD supplements in the form of drops
73. Pt with puffiness of face and rash showing cotton wool spots on fundoscopy. Whats the dx?
a. Macular degeneration
b. Hypertensive retinopathy
c. Diabetic background
d. Proliferative diabetic retinopathy
e. SLE
Q. 1. What is the key?
Q. 2. Why there is puffyness of face?
Q. 3. Why there is cotton wool spots on fundoscopy? What is the most common ocular manifestation of SLE?
Ans. 1. The key is SLE.
Ans. 2. Puffiness is due to lupus nephritis.
Ans. 3. SLE, can involve the retina. The classic lesion of SLE is a white fluffy appearing lesion within the retina
known as a cotton wool spot. The most common ocular manifestation in SLE is Keratoconjunctivits sicca.

SLE. Female male 5:1. More common in asians, afro caribbeans


Cause: HLA DR-2 DR-3 association, Environmental factors like UV rays, EBV and drugs (chlorpromazine,
methyldopa, hydralazine, isoniazid, d-penicillamine and minocycline)
Presentation: SLE is a remitting and relapsing illness
Raynauds phenomenon
Arthritis: Early morning stiffness, non erosive, no swelling peripheral, symmetrical
Photosensivity: malar rash. Precipitated by sunlight, sparing the nasolabial fold. Erythematous, raised &
pruritic
Discoid lupus eryhtamatosus: well damarcated with scaling, on sun exposed areas
Mouth ulcers
Pulmonary: pleurisy, fibrosing alveolitis
Renal: Nephritis is often asymptomatic. Glomerulonephritis is common in lupus pts
Neuro: depression and anxiety are common. There may be seizures, meningitis, psychosis
Vasculitis

Criteria for diagnosis: if any 4 of these 11 are present not necessarily at the same time.
Malar rash.
Discoid lupus.
Photosensitivity.
Oral or nasopharyngeal ulcers.
Non-erosive arthritis involving two or more peripheral joints.
Pleuritis or pericarditis.
Renal involvement with persistent proteinuria or cellular casts.
Seizures or psychosis.
Haematological disorder: haemolytic anaemia or leukopenia or lymphopenia or thrombocytopenia.
Immunological disorder: anti-DNA antibody or anti-Sm or antiphospholipid antibodies.
A positive antinuclear antibody.
Investigations: FBC: Anemia, thrombocytopenia may be seen.
ESR IS RAISED BUT CRP IS USUALLY NORMAL
Antibodies: ANA: screening test but not diagnostic, 95% sensitive. Anti DNA: diagnostic and show disease
activity. High specificity. Anti-Sm is the most specific antibody but 30-40% sensitive. Anti-SSA (Ro) or Anti-SSB
(La) are present in 15% of patients
Anti-RNP may indicate mixed connective tissue disease with overlap SLE, scleroderma, and myositis
Anti-histone: drug-induced lupus ANA antibodies are often this type
anticardiolipin antibodies and lupus anticoagulant should be checked in lupus patients, as they are associated
with APLS.
Complement C3 and C4 levels are reduced
TREATMENT: Avoid sun exposure
NSAIDs for musculoskeletal pains
Steroids: Effective but can be harmful for CVS, osteoporosis. High-dose prednisolone is reserved for
life-threatening SLE
Hydroxychloroquine remains first-line treatment for patients with mild SLE, especially for those with
arthralgia, skin rashes, alopecia, and oral or genital ulceration
Cyclophosphamide is reserved for treatment of life-threatening disease, particularly lupus nephritis,
vasculitis and cerebral disease
Mycophenolate mofetil is as effective as cyclophosphamide in inducing remission in lupus nephritis
Mycophenolate mofetil is more effective than azathioprine in maintenance therapy for preventing
relapse
Azathioprine is used as a steroid-sparing agent. As an alternative to cyclophosphamide, azathioprine is
much safer but probably less effective
Intravenous immunoglobulins are increasingly being used in the treatment of resistant lupus and also
have a role in patients who have concomitant infection and active lupus, for whom
immunosuppression treatment is often inappropriate.
Belimumab is licensed as adjunctive therapy in patients with active, autoantibody-positive SLE with a
high degree of disease activity despite standard therapy
Fertility is normal and pregnancy is safe in mild or stable lupus
COCP should be used with caution

74. A 35yo man presents with progressive breathlessness. He gave a hx of polyarthralgia with
painful lesions on the shin. CXR: bilateral hilar lymphadenopathy. Whats the most likely dx?
a. Bronchial asthma
b. Cystic fibrosis
c. Sarcoidosis
d. Bronchiectasis
e. Pneumonia
Q. 1. What is the key?
Q.2 . What is the specific name of this condition? What is the triad?
Ans. 1. The key is C. Sarcoidosis.
Ans. 2. Lofgren syndrome. The triad is i) Erythema nodosum ii) Bilateral hilar lymphadenopathy iii)
Arthralgia.

Sarcoidosis:
Presentation: Lungs are in involved in more than 90% cases of sarcoidosis. There is interstitial lung disease.
The painful skin lesion is erythema nodosum. Also look for Lupus pernio (chronic raised hardened, often
purple lesion) may be seen on the face.
Lofgren syndrome is often a part of sarcoidosis. The triad is i) Erythema nodosum ii) Bilateral hilar
lymphadenopathy iii) Arthralgia
Sarcoidosis is a multisystem disease and can involve any system/organ
Tests: ESR is often raised. Serum ACE enzyme levels are raised in 60% of times
Plain CXR may show bilateral hilar or paratracheal lymphadenopathy. High resolution CT should be done.
There will be restricitve pattern of disease on pulmonary function tests.
Transbronchial biopsy can demonstrate the presence of non-caseating granulomata, giving a more accurate
diagnosis
Bronchioalveolar lavage may also be done

75. A child presents with clean wound, but he has never been immunized as his parents were
worried about it. There is no contraindication to immunization, what is the best management?
a. Full course of DTP
b. 1 single injection DT
c. 1 single injection DTP
d. Only Ig
e. Antibiotic
Ans. The key is A. Full course of DTP.
Vaccination is at 2,3,4 months of age for children under 10 yrs of age. If a dose is missed just give the next
dose and no need to repeat the previous doses and just complete the 3 doses.
For >10 yrs it is same with an interval of at least 1 month between doses.
BOOSTERS: Age <10 yr should receive boosters 3 years after completing the 3 doses it is DPT. 2nd booster is
10yrs after the 1st booster.
Age >10 yrs receive boosters 5yrs after completing the initial 3 doses. 2nd booster is 10yrs after the 1st
booster.
Where there is no reliable history of previous immunisation, it should be assumed that they are unimmunised,
and the full UK recommendations should be followed

76. A 65yo HTN man presents with lower abdominal pain and back pain. An expansive abdominal mass is

palpated lateral and superior to the umbilicus. What is the single most discriminating inv?

a. Laparascopy
b. KUB XR

c. Pelvic US

d. Rectal exam

e. Abdominal US

Q. 1. What is the key?

Q. 2. What is the diagnosis?

Q. 3. What are the points given here in favour of your diagnosis?

Ans. 1. The key is E. Abdominal US.

Ans. 2. The diagnosis is Abdominal aortic aneurism.

Ans. 3. Points in favour of AAA are i) hypertension ii) abdominal pain iii) back pain iv) expansile abdominal

mass lateral and superior to the umbilicus.

RISK Factors include : Family Hx , tobacco smoking is an important factor.

Male sex.

Increasing age.

Hypertension.

Chronic obstructive pulmonary disease.

Hyperlipidaemia

UNRUPTURED AAA is commonly asymptomatic and is an accidental finding.

Ruptured AAA may present with:

Pain in the abdomen, back or loin - the pain may be sudden and severe.

Syncope, shock or collapse:

The degree of shock varies according to the site of rupture and whether it is contained - eg, rupture into the

peritoneal cavity is usually dramatic, with death before reaching hospital; whereas rupture into the

retroperitoneal space may be contained initially by a temporary seal forming.


Ultrasound is simple and cheap; it can assess the aorta to an accuracy of 3 mm. It is used for initial assessment

and follow-up.

SCANS :

CT Scan provides more anatomical details - eg, it can show the visceral arteries, mural thrombus, the 'crescent

sign' (blood within the thrombus, which may predict imminent rupture) and para-aortic inflammation. CT with

contrast can show rupture of the aneurysm.

MRI angiography may be used.

If size exceeds 5.5cm, we go for surgery.

77. A 55yo man has had severe pain in the right hypochondrium for 24h. The pain comes in waves and is

accompanied by nausea. Nothing seems to relieve the pain. He feels hot and sweaty but has normal temp.

What is the most appropriate next inv:

A.US Abdomen

b. ERCP

c. MRCP

d. Serum amylase

e. UGI endoscopy

Q. 1. What is the key?

Q. 2. What is the diagnosis?

Q. 3. What are the points in favour of your diagnosis?

Ans. 1. The key is A. US abdomen.

Ans. 2. The diagnosis is biliary colic.

Ans. 3. Points in favour- i) severe right hypochondrial pain. ii) colicky nature of the pain (comes in waves) iii)

nausea iv) absence of fever iv) absence of jaundice.

Biliary colic :
The pain starts suddenly in the epigastrium or right upper quadrant (RUQ) and may radiate round to the back

in the interscapular region.

Contrary to its name, it often does not fluctuate but persists from 15 minutes up to 24 hours, subsiding

spontaneously or with analgesics.

Nausea or vomiting often accompanies the pain, which is visceral in origin and occurs as a result of distension

of the gallbladder due to an obstruction or to the passage of a stone through the cystic duct.

Differential diagnosis include reflux, peptic ulcers, irritable bowel syndrome, relapsing pancreatitis and

tumours - eg, stomach, pancreas, colon or gallbladder. Two or more of these conditions may overlap, so the

diagnosis may not be easy.

ULTRASOUND is the best way to demonstrate stones, being 90-95% sensitive.

78. A 67yo man has deteriorating vision in his left eye. He has longstanding COPD and is on multiple drug

therapy. What single medication is likely to cause this visual deterioration?

a. B2 agonist

b. Corticosteroid

c. Diuretic

d. Theophylline

Q. 1. What is the key?

Q. 2. What is the cause of deteriorating vision?

Ans. 1. The key is B. Corticosteroid.

Ans. 2. Prolonged corticostiroids [also topical i.e. eye drop] can cause cataract.

79. A woman who returned from abroad after 3 weeks of holiday complains of severe diarrhea of 3 weeks. She

also developed IDA and folic acid def. What condition best describes her situation?

a. Jejunal villous atrophy

b. Chronic diarrhea secretions


c. Malabsorption

d. Increased catabolism

e. Increased secretions of acid

Q. 1. What is the key?

Q. 2. What are the points in favour?

Q. 3. What are the signs of deficiency may be present?

Ans. 1. The key is C. Malabsorption.

Ans. 2. Diarrhoea, IDA and folic acid deficiency.

Ans. 3. Iron-deficiency anaemia.

Folate deficiency or vitamin B12 deficiency.

Bleeding, resulting from low vitamin K.

Oedema, which occurs in protein/calorie malnutrition.

Tropical sprue is seen in residents of, and visitors to, tropical areas and it tends to begin with an acute episode

of diarrhoea, fever and malaise before settling into a more chronic presentation of steatorrhea,

malabsorption, nutritional deficiency, anorexia, malaise and weight loss. Folate deficiency is a significant part

of the clinical picture.

80. A 35yo male is bitterly annoyed with people around him. He thinks that people are putting ideas into his

head. What is the single most likely dx?

a. Thought block

b. Thought insertion

c. Thought broadcasting

d. Thought withdrawal

e. Reference

Q. 1. What is the key?

Q. 2. In which disease you will find this feature?


Ans. 1. The key is B. Thought insertion.

Ans. 2. It is seen in schizophrenia.

Symptoms called disorders of thought possession may also occur in schizophrenia. These include:

Thought insertion. This is when someone believes that the thoughts in their mind are not their own and that

they are being put there by someone else

.Thought withdrawal. This is when someone believes that thoughts are being removed from their mind by an

outside agency.

Thought broadcasting. This is when someone believes that their thoughts are being read or heard by others.

Thought blocking. This is when there is a sudden interruption of the train of thought before it is completed,

leaving a blank. The person suddenly stops talking and cannot recall what he or she has been saying.

81. A 10yo girl presents with hoarseness of the voice. She is a known case of bronchial asthma and has been

on oral steroids for a while. What is the most likely cause of hoarseness?

a. Laryngeal candidiasis

b. Infective tonsillitis

c. Laryngeal edema

d. Allergic drug reaction

e. Ludwigs angina

Hoarseness may be a feature of laryngeal obstruction - so can be a warning of impending airway obstruction.

This may occur in:

Infections - acute epiglottitis, diphtheria, croup, laryngeal abscess, laryngitis

Inflammation/oedema - airway burns, anaphylaxis, physical trauma, angio-oedema, hereditary angio-oedema.

Vocal cord immobility - laryngeal nerve palsy (depending on the position of the cords) or cricoarytenoid joint

disease.
Immuno compromised states lead to fungal infections. These include HIV, Diabetes etc. Patients taking long

term steriods (inhalar or oral), antibiotics n those having vitb12 and folic acid deficiecy are at a risk of having

oral thrush.

Q. 1. What is the key?

Q. 2. What is the reason for this condition?

Ans. 1. The key is A. Laryngeal candidiasis.

Ans. 2. Steroids predisposes to fungal infection.

82. A lady with breast cancer has undergone axillary LN clearance. She develops arm swelling after being stung

by a bee. What is the most likely mechanism responsible for the swelling?

a. Lymphedema

b. Cellulitis

c. Hypersensitivity reaction

d. DVT

e. Fluid retention

Q. 1. What is the key?

Q. 2. What is the reason for this condition?

Ans. 1. The key is A. Lymphoedema.

Ans. 2. Reason is compromised lymphatic drainage of arm due to axillary LN clearance.

83. A 34yo pt presents with 50% partial thickness burns. What should be the most appropriate management?

a. IV fluids calculated from the time of hospital arrival

b. IV fluids calculated from the time of burn

c. No IVF

d. IV dextrose stat

e. Burns ointment

Q. 1. What is the key?


Q. 2. How the calculation of fluid is made?

Ans. 1. The key is B. IV fluids calculated from the time of burn.

Ans. 2. Resuscitation fluids required in the first 24 hours from the time of injury.

For adults: 3 ml (in partial thickness burn) of Hartmanns solution/kg body weight/% total

Body surface area.

Half of this calculated volume is given in the first 8 hours and the other half is given over the following 16

hours.

84. A 54yo man has recently been dx with moderate depression. He has hx of MI and is suffering from

insomnia. What is the drug of choice for him?

a. Citalopram

b. Lofepramine

c. ECT

d. Haloperidol

e. Diazepam

Ans. Key is A. Citalopram. [Citalopram is the antidepressant of choice post MI].

85. A man presented with cellulitis and swelling. He was started on flucloxacillin. What other medication do

you want to add?

a. Vancomycin

b. Penicillin

c. Metronidazole

d. Ceftriaxone

e. Amoxicillin

Q. 1. What is the key?

Q. 2. Is it justified to add this drug? If justified please mention why?


Ans. 1. The key is B. Penicillin.

Ans. 2. Custom and practice has traditionally combined the use of benzylpenicillin and flucloxacillin in the

management of hospitalised patients with cellulitis. In most cases this is not seen as practical or necessary.

Flucloxacillin covers both beta-haemolytic streptococci and penicillinase-resistant staphylococci.

But for exam purpose, treatment of cellulitis = Benzylpenicillin + Flucloxacillin.

Drug of choice for cellulitis is flucloxacillin. IT Is sufficient alone. If needed to add something, add penicilin ,, or

add doxycycline if exposed to salt water,,, add erythromycin if there is penicillin allergy, or add ciprofloxacin if

exposed to fresh water.

86. A 24yo college student presents with nausea, vomiting, headache, neck stiffness and a fever of 38.4C.

What is the most appropriate empirical antibiotic to be started?

a. Ceftriaxone

b. Penicillin

c. Gentamicin

d. Tazobactam

e. Meropenem

Ans. The key is A. Ceftriaxone. [In OHCM-Cefotaxime <55yrs and Cefotaxime + Ampicillin if age >55yrs].

One should start benzyl penicillin before admission. After admission, ideally cefotaxime should be given as per

new guidelines. But, as there was no option of cefotaxime, we would go for ceftriaxone (also 3rd gen

cephalosporin)

87. A man with prosthetic heart valve underwent hemicolectomy and after some days complains of left

hypochondriac pain, fever and has a systolic murmur. What is the next inv to ascertain the cause of HF?

a. CT

b. Blood culture

c. ECG
d. MRI

e. Radioactive thyroid scan

Infective Endocarditis

Risk factors:

Cardiac conditions considered to increase a patient's risk of developing infective endocarditis:

Valvular heart disease with stenosis or regurgitation. Valve replacement.

Structural congenital heart disease, including surgically corrected or palliated structural conditions, but

excluding: Isolated atrial septal defect. Fully repaired ventricular septal defect. Fully repaired patent ductus

arteriosus. Closure devices that are judged to be endothelialized.

Investigations

Nonspecific signs of infection - eg, elevated CRP or ESR, leukocytosis, anaemia and microscopic haematuria.

CXR: as part of the initial assessment.

Electrocardiogram is useful to detect the 10% of patients who will develop conduction defects.

Blood cultures:

Should be taken prior to starting treatment in all cases. Meticulous aseptic technique is required.

Echocardiography

Q. 1. What is the key?

Q. 2. What is the diagnosis?

Q. 3. Why have you made this diagnosis?

Q. 4. What are the important risk factors for this condition?

Ans. 1. The key is B. Blood culture.

Ans. 2. The diagnosis is infective endocarditis.

Ans. 3. Fever + new murmur = endocarditis until proven otherwise.


Ans. 4. Important risk factors: dermatitis, IV injections, renal failure, organ transplantation, DM, post operative

wound. Risk factors for abnormal valves: aortic or mitral valve disease, tricuspid valve in IV drug users,

prosthetic valves.

88. A 45yo man with posterior gastric ulcer presented with severe excruciating pain which subsided after

conservative treatment. 10 days later he developed swinging pyrexia. US shows a collection in the

peritoneum. What will be the most likely location of the collection?

a. Hepatorenal pouch

b. Left paracolic gutter

c. Subphrenic

d. Pelvic cavity

e. Lesser sac

Ans. The key is E. Lesser sac.

89. A 23yo lady was prescribed with azithromycin 1gm for her chlamydial pelvic infection. She has got a new

boyfriend for the last 2 months. She has recently started contraception to avoid conception. Which of the

following contraception method will be affected by azithromycin?

a. Barrier

b. IUCD

c. POP

d. COCP

Ans. None of them! Before it was thought that hepatic enzyme inhibitor drugs may affect COCP but later it

was established that actually there is no such significant effect. Only drugs like rifampicin and rifambuin can

cause this. No other antibiotic alters COCP levels. Moreover, POP is not affected by any antibiotic other than

rifampicin. Barrier method has nothing to do with any antibiotic as its a mechanical method. IUCD has no

proved interaction with antibiotics.


90. An 11yo boy is being checked by the diabetic specialist nurse. His HbA1c was high and he has been

skipping meals recently. He has been unhappy at school. Which single member of the clinical team would you

refer him to next?

a. GP

b. Pediatrician

c. Dietician

d. Clinical psychologist

Ans. The key is D. Clinical psychologist. [Unhappy at school, skipping meals these are psychological issue. He

needs psychological counseling].

There was a discussion on plab forum that the answer should be pediatrician , but here the problem is

psychological. Had he missed any medication, he would have had to see pediatrician.

91. A 35yo man who has served in the army presents with lack of interest in enjoyable activities and feeling

low. He doesnt feel like reading the news or watching movies as he believes there is

violence everywhere. What is the most appropriate first line therapy?

a. Citalopram

b. Lofepramine

c. CBT

d. Chlordiazepoxide

e. Desensitization

Q. 1. What is the key?

Q. 2. What is the diagnosis?

Q. 3. What is the first line treatment?

Q. 4. Here why 1st line treatment is not considered?

Ans. 1. The key is C. CBT


Ans. 2. The diagnosis is depressive illness.

Ans. 3. In depressive illness 1st line therapy is SSRI

Ans. 4. In this patient abnormal thinking of presence of violence everywhere is the trigger for his depression

and in this situation CBT gives the best result.

[It is not post traumatic stress disorder as constant vivid flashbacks of the experience which is the main

feature of PTSD is absent here. In the given scenario depression has a trigger of abnormal thought process that

there is violence everywhere! So cognitive behavioural therapy is the best treatment here (though in typical

depression drug of first choice is SSRI- according to samson note)].

(personally not sure about this one as pt has all the features of ptsd except the flashbacks. Though, the answer

would still be cbt )

92. A man has reducible bulge below the pubic tubercle, and on occlusion of the deep inguinal ring, cough

impulse is present. What is the most likely dx?

a. Direct inguinal

b. Indirect inguinal

c. Femoral

d. Spigelian

e. Lumbar

Q. 1. What is the key?

Q. 2. What are the points in favour of your answer?

Ans. 1. The key is C. Femoral hernia.

Ans. 2. It is just below the pubic tubercle that is just below the inguinal ligament.

Note: this question is a very bad recall as hernia below pubic tubercle is femoral and cough impulse felt in

occluded deep ring is seen in inguinal hernia. In femoral hernia positive cough impulse is found in femoral ring.

Features of femoral hernia:


Below and lateral to the pubic tubercle

More common in women, particularly multiparous ones

High risk of obstruction and strangulation

Surgical repair is required

Other hernias :

Types of abdominal wall hernias:

Type of hernia Details

Inguinal hernia Inguinal hernias account for 75% of abdominal wall hernias. Around 95% of patients are

male; men have around a 25% lifetime risk of developing an inguinal hernia.

Above and medial to pubic tubercle

Strangulation is rare

Femoral hernia Below and lateral to the pubic tubercle

More common in women, particularly multiparous ones

High risk of obstruction and strangulation

Surgical repair is required

Umbilical hernia Symmetrical bulge under the umbilicus


Paraumbilical Asymmetrical bulge - half the sac is covered by skin of the abdomen directly above or
hernia below the umbilicus

Epigastric hernia Lump in the midline between umbilicus and the xiphisternum

Most common in men aged 20-30 years

Incisional hernia May occur in up to 10% of abdominal operations

Spigelian hernia Also known as lateral ventral hernia

Rare and seen in older patients

A hernia through the spigelian fascia (the aponeurotic layer between the rectus abdominis
muscle medially and the semilunar line laterally)

Obturator hernia A hernia which passes through the obturator foramen. More common in females and
typical presents with bowel obstruction

Richter hernia A rare type of hernia where only the antimesenteric border of the bowel herniates
through the fascial defect

93. A 48yo woman is admitted to ED with a productive cough and moderate fever. She has often central chest

pain and regurgitation of undigested food most times but doesnt suffer from acid reflux. These symptoms

have been present for the last 3.5 months which affects both food and drink. A CXR shows an air-fluid level

behind a normal sized heart. What is the most likely dx?

a. Pharyngeal pouch

b. Hiatus hernia

c. Bulbar palsy

d. Achalasia
e. TB

Q. 1. What is the key?

Q. 2. What are the points in favour?

Ans. 1. The key is D. Achalasia.

Ans. 2. Points in favour: Aspiration pneumonia due to retained food and fluid in oesophagus. Regurgitation of

undigested food without acid reflux. Dysphagia for both food and drink. Air-fluid level behind heart.

Why it is not hiatus hernia? Ans. Differentiating point:-i) In hiatus hernia usually you will get associated GORD

[particularly in sliding hernia which is the most common (99%). However in rolling hernia there may be no

reflux]. ii) In hiatus hernia x-ray chest may demonstrate a retrocardiac gas-filled structure rather than a air-

fluid level iii) Also in hiatus hernia there may be nausea or vomiting.

Why it is not pharyngeal pouch? Ans. In pharyngeal pouch there will be halitosis.

Achlasia has been discussed before in detail.

94. A 64yo man has been waking up in the middle of the night to go to the bathroom. He also had difficulty in

initiating micturition and complains of dribbling. A dx of BPH was made after a transrectal US guided biopsy

and the pt was prepared for a TURP. What electrolyte abnormality is highly likely due to this surgery?

a. Hypokalemia

b. Hypocalcemia

c. Hyperkalemia

d. Hyponatremia

e. Hypernatremia

Q. 1. What is the key?

Q. 2. Why this happens?

Ans. 1. The key is D. Hyponatremia.


Ans. 2. Absorption of fluid used for bladder irrigation to flush out blood clots and IV fluids all may lead to

hypervolaemia and dilutional hyponatremia.

95. A 56yo lady has developed severe right sided headache which worsens whenever she comes to bright light

since the last 4 days. She feels nauseated, but doesnt vomit. What is the most likely

dx?

a. SAH

b. Brain tumor

c. Migraine

d. Cluster headache

e. Subdural headache

Q. 1. What is the key?

Q. 2. What is the type of the given case?

Q. 3. What are the points in favour of mentioned type?

Ans. 1. The key is C. Migraine.

Ans. 2. It is migraine without aura. There is presence of trigger (bright light)

Ans. 3. Criteria of migraine without aura: 5 headaches lasting 4-72 hours + nausea/vomiting (or

photo/phono-phobia) + any 2 of: i) unilateral ii) pulsating iii) worsen by routine activity [OHCM, 9th edition,

page-462].

It should be noted that as a general rule 5-HT receptor agonists are used in the acute treatment of migraine

whilst 5-HT receptor antagonists are used in prophylaxis. NICE produced guidelines in 2012 on the

management of headache, including migraines.

Acute treatment

first-line: offer combination therapy with an oral triptan and an NSAID, or an oral triptan and
paracetamol
for young people aged 12-17 years consider a nasal triptan in preference to an oral triptan
if the above measures are not effective or not tolerated offer a non-oral preparation of
metoclopramide* or prochlorperazine and consider adding a non-oral NSAID or triptan

Prophylaxis

prophylaxis should be given if patients are experiencing 2 or more attacks per month. Modern
treatment is effective in about 60% of patients.
NICE advise either topiramate or propranolol 'according to the person's preference, comorbidities and
risk of adverse events'. Propranolol should be used in preference to topiramate in women of child
bearing age as it may be teratogenic and it can reduce the effectiveness of hormonal contraceptives
if these measures fail NICE recommend 'a course of up to 10 sessions of acupuncture over 5-8 weeks'
or gabapentin
NICE recommend: 'Advise people with migraine that riboflavin (400 mg once a day) may be effective in
reducing migraine frequency and intensity for some people'
for women with predictable menstrual migraine treatment NICE recommend either frovatriptan (2.5
mg twice a day) or zolmitriptan (2.5 mg twice or three times a day) as a type of 'mini-prophylaxis'
pizotifen is no longer recommend. Adverse effects such as weight gain & drowsiness are common

*caution should be exercised with young patients as acute dystonic reactions may develop

96. A 35yo man presented with hematuria, abdominal swelling and has a BP of 190/140. What is the most

diagnostic inv?

a. Cystoscopy

b. USG

c. CT

d. Renal biopsy

e. Urine analysis

Q. 1. What is the key?

Q. 2. What is the diagnosis?

Q. 3. What will be the USG findings to establish diagnosis in given case?

Ans. 1. The key is B. USG.

Ans. 2. The diagnosis is ADPKD.


Ans. 3. In given case patients age is 35. So the USG diagnostic criteria is: Age 18 39 yrs>3 unilateral or,

bilateral cysts (here bilateral means if 1 + 1 it is enough).

Autosomal dominant polycystic kidney disease (ADPKD) is the most common inherited cause of kidney

disease, affecting 1 in 1,000 Caucasians. Two disease loci have been identified, PKD1 and PKD2, which code for

polycystin-1 and polycystin-2 respectively

ADPKD type 1 ADPKD type 2

85% of cases 15% of cases

Chromosome 16 Chromosome 4

Presents with renal failure earlier

The screening investigation for relatives is abdominal ultrasound:

Ultrasound diagnostic criteria (in patients with positive family history)

two cysts, unilateral or bilateral, if aged < 30 years


two cysts in both kidneys if aged 30-59 years
four cysts in both kidneys if aged > 60 years
97. A young man is brought to the ED after a RTA. His GCS on initial evaluation is 6. What is the most

appropriate next step?

a. CT

b. MRI

c. IV fluids

d. Skull XR

e. Secure airway

Ans. The key is E. Secure airway.


In a case of Road Traffic Accident. or any trauma, management starts with A- airway (includes cervical

immobility), B- (breathing), C (circulation) , D (disability) . ETT SHOULD BE CONSIDERED IN ALL PATIENTS WITH

GCS BELOW 8.

98. A 65yo man presented with frank hematuria. He has no other urinary symptoms. What is the most

appropriate next step that will lead to the dx?

a. IVU

b. US Abdomen

c. Cystoscopy

d. Mid-stream urine for culture

e. Transrectal US

Q. 1. What is the key?

Q. 2. What is the diagnosis?

Q. 3. What are the reasons for this diagnosis?

Q. 4. If there is painless haematuria in young (say 25-30yrs) what diagnosis will come first?

Ans.1. Key is C. Cystoscopy.

Ans. 2. Bladder cancer.

Ans. 3. Age 65, asymptomatic haematuria.

Ans. 4. ADPKD [at the beginning there is very few or no symptoms]

Bladder CA has already been discussed in mcq # 1.

99. A 30yo woman had a gradual decrease of visual acuity since the last 3 years. Now she has a disability due

to very low vision. Whats the dx?

a. Glaucoma

b. Cataract

c. Macular degeneration

d. Retinitis pigmentosa
e. Keratitis

Q. 1. What is the key?

Q. 2. Why it is not the other given D/D s?

Ans. 1. The key is D. Retinitis pigmentosa.

Ans. 2. i) It is not angle closure glaucoma as angle closure glaucoma occurs usually after the age of 50; In open

angle glaucoma visual loss is not evenly gradual rather occurs a bit suddenly at its later part. It is not cataract

as cataract occurs usually in elderly. In macular degeneration near blindness does not occur rather causes

inability to identify face or cannot read small prints; otherwise peripheral vision is not that depressed. In

keratitis will be pain, redness, photophobia and vision is ok.

Retinitis pigmentosa primarily affects the peripheral retina resulting in funnel vision

Features

night blindness is often the initial sign


funnel vision (the preferred term for tunnel vision)
fundoscopy: black bone spicule-shaped pigmentation in the peripheral retina, mottling of the retinal
pigment epithelium
Associated diseases

Refsum disease: cerebellar ataxia, peripheral neuropathy, deafness, ichthyosis


Usher syndrome
abetalipoproteinemia
Lawrence-Moon-Biedl syndrome
Kearns-Sayre syndrome
Alport's syndrome

100. A 27yo lady has had an uncomplicated pregnancy so far. She came to the hospital 2h ago after her water
broke. The midwife is looking at her now. She has regular contractions. P.V exam revealed 2cm dilated cervix.
Vital signs are normal. What stage of labour is she in?
a. Second stage

b. First stage

c. Latent stage

d. Third stage

e. Active phase
Ans. The key is B. First stage starts with softening of cervix with start of opening of cervix and ends when

cervix is fully dilated (i.e. 10 cm dilated). [There is nothing named latent stage but latent phase which is up

to 4cm dilatation. So, the preferred option is first stage here].

Stages of Labour

First stage

The first stage begins with regular contractions (when the fetal presenting part has descended into the true

pelvis), or on admission to hospital with obvious signs of labour.

The first stage ends when the cervix is fully dilated (10 cm).

First stage can be divided into:

Latent or quiet phase: Contractions are not particularly painful and at 5- to 10-minute intervals. Contractions

become stronger with shorter intervals, although the cervix is still dilating relatively slowly, with membranes

possibly breaking later in this phase.

Active phase:Starts with the cervix 3-4 cm dilated and is associated with more rapid dilatation normally at 0.5-

1.0 cm/hour. Once the cervix is dilated to 9 cm, towards the end of the active phase, contractions may be

more painful and women may want to push. Pushing is undesirable at this stage; there is the need to establish

by vaginal examination whether the cervix is fully dilated. During this time the fetal head descends into the

maternal pelvis and the fetal neck flexes.

While the length of established first stage of labour varies between women, first labours last on average 8

hours (unlikely 18 hours). Second and subsequent labours last on average 5 hours (unlikely 12 hours).

However if the first stage does not appear to be progressing, the cause needs to be determined.

Second stage:
This starts when the cervix is fully dilated and ends with the birth of the baby:

Contractions are stronger, occur at 2- to 5-minute intervals and last 60-90 seconds.

The fetal head descends deeply into the pelvis and rotates anteriorly so that the back of the fetal head is

behind the mother's symphysis pubis (98% of cases).

The second stage is said to be active once the baby is visible and the woman usually also wants to assist what

have become expulsive contractions by pushing.

The fetal head becomes more visible with each contraction until a large part of the head can be seen.

The head is now born with first the forehead, then the nose, mouth and chin.

The head rotates to allow the shoulders to be born next, followed by the trunk and legs.

After this, the baby should start to breathe and to cry loudly.

Third stage:

This stage starts with the birth of the baby and ends with the delivery of the placenta and membranes:

Separation of the placenta occurs immediately after birth due to forceful uterine contractions along with

retraction of the uterus, thus greatly reducing the size of the placental bed.

It normally takes up to 5 minutes, but can take longer.

Haemorrhaging is prevented by the contraction of uterine muscle fibres closing off the blood vessels that were

supplying the placenta.

Without active management, after 10-20 minutes, separation is shown by a gush of blood, prominence of the

fundus in the abdomen and apparent lengthening of the umbilical cord.

101. A 2yo boy fell off his tricycle and hurt his arm. He got up to start crying, but before there was
any sound, he went pale, unconscious and rigid. He recovered after 1-2 mins but remained pale.
After an hour he was back to normal. His mother says she was afraid he was going to die, and
that he had a similar episode 3 months prior after falling down some steps. What single inv is
indicated?
a. CT head
b. EEG
c. CBC
d. None
e. Skeletal survey
Q. 1. What is the key?
Q. 2. What is the diagnosis?
Ans. 1. The key is D. None.
Ans. 2. Diagnosis is breath holding spell.

102. A 29yo woman had just delivered a still born vaginally, following a major placental abruption.
Choose the single most likely predisposing factor for developing PPH in this lady?
a. Retained product
b. DIC
c. Fibroid uterus
d. Uterine infection
e. Large placental site
Q. 1. What is the key?
Q. 2. What are the causes of this condition here?
Ans. 1. The key is B. DIC.
Ans. 2. Pregnancy itself is a risk factor for DIC. Placental abruption is a more common cause of DIC.
Other causes of pregnancy related DIC are: eclampsia, retention of a dead fetus, amniotic fluid embolism,
retained placenta or bacterial sepsis.
103. A 28yo woman has delivered with rotational forceps after an 8h labor and 3h second stage.
Choose the single most likely predisposing factor for PPH for this pt?
a. Atonic uterus
b. Cervical/vaginal trauma
c. Retained product
d. Preterm labor
e. Uterine infection
Ans. The key is B. Cervical/vaginal trauma. [complication of forceps delivery].

Primary PPH is the loss of greater than 500mL (defi nitions vary) in the first
24h after delivery
Causes: uterine atony (90%), genital tract trauma (7%), clotting disorders (3%)
Risks: Antenatal Previous PPH or retained placenta BMI>35kg/m2 Maternal Hb<8.5g/dl at onset of labour
Antepartum haemorrhage Multiparity 4+ Maternal age 35y+ Uterine malformation or fibroids A large
placental site (twins, severe rhesus disease, large baby) Low placenta, Overdistended uterus
(polyhydramnios, twins) Extravasated blood in the myometrium (abruption).
In labour Prolonged labour (1st, 2nd or 3rd stage) Induction or oxytocin use Precipitant labour
Operative birth or caesarean section. Book mothers with risk factors for obstetric unit delivery.
Treatment: Give oxytocin 5U slowly IV for atonic uterus.
Attach oxygen, Give IV fluids, maintain systolic >100mmHg, Transfuse blood.
Is the placenta delivered? If it is, is it complete? If not, explore the uterus. If the placenta is complete, put
the patient in the lithotomy position with adequate analgesia and good lighting. Check for and repair trauma.
If the placenta has not been delivered but has separated, attempt to deliver it by controlled cord traction
after rubbing up a uterine contraction. If this fails, ask an experienced obstetrician to remove it under general
anaesthesia.Beware renal shut down.

104. A 50yo man has had anterior resection of the rectum for carcinoma. He expressed concerns
about control of post-op pain in discussions with the anaesthetist before surgery. What is the
best management strategy?
a. Oral diclofenac
b. Oral codeine
c. IM morphine
d. IM dihydrocodeine
e. Ondansetron oral
Ans. The key is C. IM morphine. [Post operative pain is severe pain which needs strong opioid analgesics].
Oral route will not be suitable for this patient as he is having a major abdominal surgery so most probably he
will be NPO post operatively. Dihydrocodeine is useful for mild to moderate pain but since its a major surgery
and the patient is himself worried about the pain good analgesia should be maintained. Ondensetron is an anti
emetic not an analgesic. So the most suitable option here is IM Morphine as it is a strong analgesic most
suitable for severe pain.
105. A 73yo male presents with enlarged cervical nodes. He has had recurrent infections over the last year. His
conjunctiva is pale. Choose the single cell type you will find on the blood film.
a. Granulocyte without blast cells
b. Myelofibroblasts
c. Plasma cells
d. Mature lymphocytes
Q. 1. What is the key?
Q. 2. What is the diagnosis?
Q. 3. What are the points in favour of your diagnosis?
Ans. 1. The key is D. Mature lymphocytes.
Ans. 2. The diagnosis is CLL.
Ans. 3. It is CLL because of his age (73 yrs). Other supportive features are cervical lymphadenpathy, recurrent
infections (mature but functionally defective lymphocytes), and pale conjunctiva (anaemia).

It is a typical presentation of CLL with reurrent infections, symmetrical lymphadenopathy, anemia. There can
also be hepatosplenomegaly and thrombocytopenia leading to patechae
On blood film there will be B cell lymphocytosis often with smudge cells. There are mature but functionally
impaired lymphocytes as they escape apoptosis. In bone marrow there is lymphocytic replacement of bone
marrow cells. Mainstay of treatment is chemotherapy.
106. A 45yo lady has 10m hx of SOB. She is found to have irregularly irregular pulse and loud P2 with fixed
splitting and ejection systolic murmur in left 2nd ICS. What is the probable dx?
a. TOF
b. ASD
c. VSD
d. PDA
e. CoA
Q. 1. What is the key?
Q. 2. What is the diagnosis?
Ans. 1. The key is B. Atrial septal defect.
Ans. 2. Diagnosis is ASD with atrial fibrillation. [i) atrial fibrillation = irregularly irregular pulse. ii) ASD = SOB,
fixed splitting with loud P2, ESM in pulmonary area]. This pictures are of atrial septal defect itself though
similar findings we get in pulmonary hypertension. One should not misdiagnose SOB, ESM in pulmonary area
and loud P2 as pulmonary hypertension in the given case.
Fixed splitting is the clincher in this question.
VSD: Pansystolic murmur
PDA: machinery murmur
ASD is acyanotic condition. Ostium seccundum is the most common cause. There is left to right shunting of
blood leading to dyspnoea/heart failure eg at age 4060. There may be pulmonary hypertension, cyanosis,
arrhythmia, haemoptysis, and chest pain.
SIGNS: AF; raised JVP; wide, fixed split S2; pulmonary ejection systolic murmur
If an embolus from DVT of lower limb passes to the brain and causes ischemia it can only pass from vein to
artery through ASD.
INVESTIGATIONS: Echo is diagnositic
Tx: In children closure by surgery before 10yrs of age, In adults transcatheter closure is now more common
than surgery

107. A 5m baby present with recurrent vomiting. Mother noticed some of the vomitus is blood
stained. Choose the single most likely inv?
a. Upper GI endoscopy
b. Barium meal
c. US
d. Colonoscopy
e. CT abdomen
Ans. The key is A. upper GI endoscopy.

Haematemesis (unless swallowed blood - eg, following a nosebleed or ingested blood from a cracked nipple in
some breast-fed infants) may suggest an important and potentially serious bleed from the oesophagus,
stomach or upper gut.

Projectile vomiting, non bilious: Pyloric stenosis


Bilious vomiting: Call for senior help, consider duodenal obstruction.
108. A 76yo is treated with HTN. He suffers from pain and redness at the MTP joint of his right big toe. Which
of the following anti-HTN cause this symptoms?
a. Losartan
b. Bendroflumethiazide
c. Ramipril
d. Bisoprolol
e. Verapamil
Q. 1. What is the key?
Q. 2. What is the diagnosis?
Q. 3. What is the cause of the disease?
Ans. 1. The key is B. Bendroflumethiazide
Ans. 2. Diagnosis is acute gout.
Ans. 3. Thiazide diuretics may cause hyperuricemia and thus precipitate acute gout.

Thiazide diuretics are contraindicated in gout!


In gout mostly large joints are involved like ankle, knee, foot. But small joints of hands can also be involved.
It is caused by deposition of monosodium urate crystals in and near joints, precipitated, for example, by
trauma, surgery, starvation, infection or diuretics.
CAUSES: Hereditary, dietary purines, alcohol excess, diuretics, leukaemia, cytotoxics
(tumour lysis).
INVESTIGATIONS: Polarized light microscopy of synovial fluid: negatively birefringent crystals (while those of
pseudogout are positively birefringenent).... Serum urate may or may not be raised. Punched out erosions on
X Ray in advanced disease.
TREATMENT: ACUTE: NSAIDs (indomethacin), colchicine if NSAIDs are contra indicated like peptic ulcer, heart
disease. In renal failure both are problematic so use steroids.
CHRONIC: Start if >1 attack in 12 months, tophi or renal stones. Use allopurinol. Aim is plasma urate
<0.3mmol/L
In acute attack allopurinol is CI as it exacerbates the attack, wait until 3 weeks after acute attack to start
allopurinol. But once on allopurinol no need to stop it during acute attacks.
Febuxostat and probenicid are alternatives.
109. A 33yo male involved in a street fight presents with bruises and deformity in the upper part of
his leg. XR shows fx of the neck of fibula. What is the single most associated nerve injury?
a. Sciatic nerve
b. Gluteal nerve
c. Musculocutaneous nerve
d. Lateral peroneal nerve
e. Tibial nerve
f. Femoral nerve
Ans. is D. Lateral peroneal nerve. [Lateral peroneal nerve is other name of superficial peroneal nerve].

110. A 35yo man presents with hx of dyspepsia. H.Pylori antibodies are negative. No improvement is seen
after 1m of tx. What is the next step?
a. Urea breath test
b. Gastroscopy
c. CT
d. MRI
Q. 1. What is the key?
Q. 2. What may be the D/D here?
Q. 3. At this age what are the indications of this procedure?
Ans. 1. Gastroscopy.
Ans. 2. Not responding to treatment D/D is: i) Zollinger Elison syndrome ii) Ca stomach
Ans. 3. Indications of gastroscopy in a 35 yo man (man of age <50yrs): i) Acute symptoms with H/O previous
episode (PUD) ii) Alarm features [weight loss, anaemia, vomiting, hematemesis and melaena, dysphagia,
palpable abdominal mass], fear of cancer, evidence of organic disease.

Urgent specialist referral - two-week rule


If the patient has dyspepsia at any age with any of the following alarm symptoms: [13]
Chronic GI bleeding.
Progressive unintentional weight loss.
Progressive dysphagia.
Persistent vomiting.
Iron-deficiency anaemia.
Epigastric mass.
Suspicious barium meal.
NB: patients aged 55 years or older with unexplained and persistent recent-onset dyspepsia should be
referred urgently for endoscopy

If age less than 55 and no alarm signs, try life style modifications and simple antacids. If no improvement then
do H.pylori testing (antibodies). If it is positive do eradication and review in 4 weeks.
If resolved, no further action required.
If symptoms are not resolved, do urea breath test.
If it is positive, again eradication for H.pylori
If it is negative, do upper GI endoscopy

If the initial H.pylori testing was negative give PPIs or H2 blockers for 4 weeks and review if symptoms resolve
no action needed if they dont resolve do upper GI endoscopy.
111. A 15yo male has bilateral ankle edema. His BP=110/70mmHg and urinalysis shows protein++++.
What is the most likely dx?
a. HUS
b. IgA nephropathy
c. Membranous GN
d. Minimal change GN
e. Nephrotic syndrome
Q. 1. What is the key?
Q. 2. What are the points in favour of your diagnosis?
Q. 3. What is the treatment?
Ans. 1. The key is D. Minimal change disease.
Ans. 2. Points in favour: i) Age 15 ii) Ankle oedema iii) Normotension iv) Heavy proteinuria.
Ans. 3. Treatment of choice is steroid (prednisolone). Failure of steroid or frequent relapse (>3)
cyclophosphamide.

Most common cause of nephrotic in children is minimal change disease. There will be hypoalbuminemia and
peripheral edema too. Electron microscopy shows effacement of podocyte foot processes.. MCD has albumin
selective proteinuria. Treatment is with steroids.
IgA nephropathy is nephritic and will also show HTN and microscopic hematuria and follows upper resp tract
infection.
Membranous GN also presents as nephrotic but age and since MCD is most common we choose MCD.
Nephrotic syndrome itself is not a diagnosis.
112. A 28yo man has developed a red, raised rash on trunk after playing football. His PMH shows he had
childhood asthma. The rash is becoming increasingly itchy. What is the most appropriate tx?
a. Oral chlorpheneraime
b. Oral amoxicillin
c. IM adrenaline
d. Nebulized salbutamol
e. Histamine
Q. What is the key?
Q. 2. What is the diagnosis?
Ans. 1. The key is A. Oral chlorpheneramine.
Ans. 2. Diagnosis is Atopy (allergy).

Since it is an allergic reaction only 2 options are suitable. A & C. IM adrenaline is used in anaphylactic shock
which can occur due to allergy. But this is just a mild allergic reaction here so anti histamine
(chlorpheneramine) is adequate.

Anaphylaxis presents with:


Sudden onset and rapid progression of symptoms.
Life-threatening airway and/or breathing and/or circulation problems
Patient will be mostly in shock.
Mostly commonly caused by certain foods like peanuts, pulses, fish, eggs. Also by venom (bee,wasps) and
drugs like antibiotics.
Treatment:ABCDE, Oxygen, IM Adrenaline. <6yrs0.15ml, 6-12yrs 0.3ml, >12 yrs 0.5ml 1:1000

113. A 72yo man has been advised to have antibiotic prophylaxis for some years now before dental tx. He has
never experienced chest pain. Three weeks ago, he noticed breathlessness on exertion and for one week he
had orthopnea. His pulse is normal. What is the most probable dx?
a. Aortic regurgitation
b. Ischemic mitral regurgitation
c. Mitral valve prolapse
d. Pulmonary stenosis
e. Mitral valve stenosis
Ans. The kay is E. Mitral valve stenosis.

The patient has mitral stenosis or Aortic regurgitation. he is given prophylaxis for infective endocarditis.
According to OHCM, such prophylaxis has no benefit and should not be given.
RISK FACTORS for IE: aortic or mitral valve disease; tricuspid valves in IV drug users; coarctation; patent ductus
arteriosus; VSD; prosthetic valves

Mitral Stenosis: Presentation: dyspnoea; fatigue; palpitations; chest pain; systemic emboli; haemoptysis;
chronic bronchitis-like picture
CAUSES: Rheumatic, congenital, mucopolysaccharidoses, endocardial fibroelastosis, malignant carcinoid,
prosthetic valve.
SIGNS: Malar flush on cheeks (due to cardiac output); low-volume pulse; AF common;
tapping, non-displaced, apex beat (palpable S1). On auscultation: loud S1; opening snap (pliable valve);
rumbling mid-diastolic murmur (heard best in expiration, with patient on left side
ECG shows P-mitrale ECHO is diagnostic. CXR: left atrial enlargement (double shadow in right cardiac
silhouette)
TREATMENT: balloon valvuloplasty (if pliable, non-calcified valve), open mitral valvotomy
or valve replacement.
Complications: Pulmonary hypertension, emboli, pressure from large LA on local
structures, eg hoarseness (recurrent laryngeal nerve), dysphagia (oesophagus),
bronchial obstruction; infective endocarditis

AORTIC REGUGITATION: CAUSES Acute: Infective endocarditis, ascending aortic dissection,


chest trauma.
SYMPTOMS: Exertional dyspnoea, orthopnoea, and paroxysmal nocturnal dyspnoea. palpitations, angina,
syncope, CCF
Signs: Collapsing (water-hammer) pulse (p40); wide pulse pressure; displaced, hyperdynamic apex beat; high-
pitched early diastolic murmur (heard best in expiration, with patient sitting forward).

The diagnosis here is mitral stenosis because of the normal pulse. I think the information in the question is
too deficient for such a disease and diagnosis!
FEVER + NEW MURMUR IS ENDOCARDITIS UNTIL PROVEN OTHERWISE

114. A 37yo woman presents with fatigue. Exam: angular stomatitis, no koilonychea. Choose the single cell
type you will find on the blood film.
a. Macrocytes
b. Microcytes
c. Granulocytes wthout blast cells
d. Blast cells
Q. 1. What is the key?
Q. 2. What is the cause here?
Q. 3. What are the points in favour of mentioned cause?
Ans. 1. The given key is A. Macrocytes.
Ans. 2. The cause here is VIT. B12 or folate deficiency.
Ans. 3. Points in favour of Vit. B12 or folate deficiency: i) fatigue (anaemia) ii) angular stomatitis (can be seen
in Vit. B12 or folate deficiency) iii) absence of koilonychea is against IDA.

SIGNS in ANEMIA:
Koilocychia (spoon shaped nails) iron deificiency anemia
atrophic glossitis in iron def.
post cricoid webs (plummer vinson syndrome)
Angular stomatitis (cheilosis) in both vit B12 and iron def.
glossitis (beefy-red sore tongue) Vit. B12 def.

115. A 4yo boy with a febrile convulsion lasting eight minutes has been given IV lorazepam to control them.
What is the single most likely serious side effect?
a. Amnesia
b. Anaphylactic shock
c. Apnea
d. Bronchospasm
e. Cardiac arrhythmia
Ans. The key is C. Apnoea.
Due to respiratory depression caused by benzodiazepines. They can also cause amnesia but it wont be in acute
setting.

116. A 4wk girl has been dx of having breast milk jaundice. She is otherwise well. What is the single most
appropriate management?
a. Continue breastfeeding
b. Exchange transfusion
c. Increase fluid intake
d. Phototherapy
e. Stop breastfeeding
Q. 1. What is the key?
Q. 2. What is breast milk jaundice?
Q. 3. What type of hyperbilirubinemia occurs in breast milk jaundice?
Q. 4. What is the cause of this jaundice?
Ans. 1. The key is A. Continue breast feeding.
Ans. 2. If jaundice lasts past the first week of life in a breastfed baby who is otherwise healthy, the condition
may be called "breast milk jaundice."

Ans. 3. Unconjugated hyperbilirubinaemia.

Ans. 4. Cause of breast milk jaundice: factors in a mother's milk that help a baby absorb bilirubin from the
intestine.
Hyperbilirubinaemia (<200mol/L) after 24h is usually physiological
Visible jaundice within 24h of birth is always abnormal. Causes: Sepsis or Rhesus haemolytic disease: +ve
direct Coombs test.
Prolonged jaundice (not fading after 14 days) Causes: breastfeeding; sepsis, (UTI & TORCH, hypothyroidism;
cystic fibrosis; biliary atresia if conjugated and pale stools.

If the jaundice is between 1-14 days no intervention is needed unless it is severe in which case phototherapy
or exchange transfusion is done.

117. A 12yo girl when playing in the garden accidentally stepped on a hive and was bitten several
times. She has numerous wheals on her body and complains of severe itching. What is the single
most appropriate management?
a. Oral antihistamine
b. IV antihistamine
c. IM adrenaline
d. Oral ciprofloxacin
e. Reassurance
Ans. The given key is C. IM adrenaline which is a wrong key. The correct answer is A. Oral antihistamine.
Followings are the indications of adrenaline in anaphylaxis:
1. Horseness of voice
2. Wheeze
3. Shortness of breath
4. Shock
5. Stridor
6. Swelling of the tongue and cheek
7. Facial swelling

Consider anaphylaxis when there is compatible history of rapid-onset severe allergic-type reaction with
respiratory difficulty and/or hypotension, especially if there are skin changes present and the treatment of
anaphylaxis is IM adrenaline not anti histamine
Adrenaline can be repeated after 5mins.
And since she is bitten by bee several times it a risk factor for anaphylaxis.

118. A term baby born to a 30yo woman of blood group A-ve develops severe jaundice within the
first 24h of birth. What is the most likely dx?
a. Hereditary spherocytosis
b. G6PD
c. ABO incompatibility
d. Rh incompatibility
e. Physiological jaundice
Ans. The key is D. Rh incompatibility.

As mentioned in the Q116 neonatal jaundice within 24hrs of birth could be either because of sepsis or Rh
incompatibility.
Mother is always Rh- and the baby is Rh +. Fetal antigen crosses the placenta and the mother produces the
antibodies against the antigen. Which cross the placenta in subsequent pregnancies as a result of secondary
response (greater in magnitude) and cause hemolysis of the fetal blood.
ABO incompatibility: (mother O; baby A or B, or mother A and baby B, or vice
versa) DCT +ve in 4%; indirect Coombs +ve in 8%. Maternal IgG anti-A or anti-
B haemolysin is always present
119. A 4yo girl is found to have bounding pulse and continuous machinery murmur. What is the most probable
dx?
a. TOF
b. ASD
c. VSD
d. PDA
e. CoA
Ans. The key is D. PDA.
Machinery murmur is the clincher for PDA.
VSD has a pansystolic murmur
ASD ejection systolic and fixed splitting

PDA PRESENTATION: Usually asymptomatic. Acyanotic disease. A large-shunt PDA may cause lower respiratory
tract infection as well as feeding difficulties and poor growth during infancy, with failure to thrive because of
heart failure.
ECHO IS DIAGNOSTIC
MANAGEMENT: Indomethacin can be used but not useful in term infants. Closure is indicated if the patient is
symptomatic at any stage of life or if asymptomatic but with great left heart load.
Surgery is used where non surgical method can not be used. In asymptomatic infant we wait till 1 yr for
spontaneous closure of PDA if that does not occur it can be closed by surgery at any time.
In preterms indomethacin or ibuprofen may be used.
Most common complication is infective endocarditis.

120. A 12yo child with episodes of sudden bluish discoloration and brief loss of consciousness. Exam: clubbing,
central cyanosis, systolic thrill with systolic ejection murmur in 2nd left ICS. What is the most probable dx?
a. TOF
b. ASD
c. VSD
d. PDA
e. CoA
Ans. The key A. TOF.

ASD, VSD, PDA are all acyanotic congenital heart diseases. TOFF is the most common cyanotic congenital heart
disease that survives to adulthood.
Typical features:
1 Ventricular septal defect (VSD)
2 Pulmonary stenosis (most imp feature)
3 Right ventricular hypertrophy
4 The aorta overriding the VSD
During a hypoxic spell, the child becomes restless and agitated and may cry inconsolably. Toddlers may squat,
which is typical of TOF. Clubbing, difficulty of feeding, failure to thrive all are features.
Chest Xray Shows BOOT SHAPED HEART. Echo is also done.
In acute stage give O2, place the child in knee chest position, give morphine. Surgery is required within 1st yr
of life otherwise mortality is 95%.
121. An 8yo child who is tall for his age and has a refractory error for which he wears glasses has
presented with severe crushing chest pain. What is the most likely dx?
a. Fragile X syndrome
b. Prader-willi syndrome
c. DiGeorge syndrome
d. Marfans syndrome
Q. 1. What is the key?
Q. 2. What is the cause of this severe crushing chest pain?
Q. 3. What are the most common cardiac abnormalitis found in this disease?
Ans. 1. The key is D. Marfans syndrome.
Ans. 2. Cause of severe crushing chest pain may be aortic dissection.
Ans. 3. Most common cardiac abnormalities in Marfans syndrome are: dilatation of the aorta and mitral
regurgitation.

Marfans syndrome diagnosis:


Major criteria (diagnostic if >2): Lens dislocation (ectopia lentis) aortic dissection or
dilatation; dural ectasia; skeletal features: arachnodactyly (long spidery fingers), armspan > height, pectus
deformity, scoliosis, pes planus. Minor signs: Mitral valve prolapse, high-arched palate, joint hypermobility.
Diagnosis is clinical.
DANGER IS AORTIC DISSECTION. Surgery is done when aorta >5cm
Can also cause pneumothorax.
122. A 4yo child presents with pain of spontaneous onset in his knee of 2 days duration. He has
developed mild fever in the 2nd day. He can walk but has a limp. Exam: painful restriction in the
right hip. What is the most probable dx?
a. Osteosarcoma
b. Septic arthritis
c. TB arthritis
d. Exostosis
e. Osteomyelitis
Q. 1. What is the key?
Q. 2. What are the points in favour of your diagnosis?
Ans. Given key is E. Osteomyelitis which is a wrong key. The correct answer is B. Septic arthritis.
Ans. Points in favour of diagnosis: i) Pain in joints (knee and hip) ii) Fever iii) Painful restricted movement of
joint.

Not sure about the correct answer here. But i think osteomyelitis.
Osteomyelitis mostly has a primary source of infection via which the infection spreads to bone.
PRESENTATION: Pain of gradual onset over the course of a few dayswith tenderness,
warmth, and erythema at the affected part; unwillingness to move. Vertebrae and distal femur mostly
affected.
Diagnosis: FBC, ESR, CRP, blood culture. Bone biopsy and culture is gold standard. Staph aureus (MR the most
common organism found.)
Treatment Drain abscesses and remove sequestra by open surgery. Antibiotics: vancomycin 1g/12h and
cefotaxime 1g/12h IVI until the organism and its sensitivities are known. Fusidic acid or clindamycin can also
be used.

Septic Arthritis: Exclude septic arthritis in any acutely inflamed joint, as it can destroy a joint
in under 24h. Knee & hip joint are most commonly involved.
Risk factors for septic arthritis include:
Increasing age
Diabetes mellitus
Rheumatoid arthritis
Joint surgery
Hip or knee prosthesis
Skin infection in combination with joint prosthesis
Infection with HIV
Diagnosis: Urgent joint aspiration for synovial fluid microscopy and culture is
the key investigation. The joint is usually swollen, warm, tender and exquisitely painful on movement.
Flucloxacillin or clindamycin as empirical treatment.
123. A man with anterior resection and end to end anastomosis done complains of severe pain in the chest
and abdominal distension. What is the most appropriate inv likely to review the cause this deterioration?
a. XR abdomen
b. Exploratory laparoscopy
c. CT
d. US
e. Laparotomy
Ans. The key is E. Laparotomy. [diagnostic and therapeutic].

124. Pt with hx of alcoholism, ataxic gait, hallucinations and loss of memory. He is given acamprosate. What
other drug can you give with this?
a. Chlordiazepoxide
b. Thiamine
c. Diazepam
d. Disulfiram
e. Haloperidol
Q. 1. What is the key?
Q. 2. What is the diagnosis?
Q. 3. What are the points in favour of diagnosis?
Ans. 1. The key is B. Thiamine.
Ans. 2. The diagnosis is Wernickes encephalopathy.
Ans. 3. Points in favour of diagnosis: i) history of alcoholism ii) ataxic gait iii) hallucination iv) memory loss.

Thiamine (vitamin B1) deficiency with a classical triad of 1 confusion 2 ataxia (wide-based gait) and 3
ophthalmoplegia (nystagmus, lateral rectus or conjugate gaze palsies). Always
consider this diagnosis in alcoholics: it may also present with memory disturbance.
TREATMENT: early treatment is essential to prevent progression to the irreversible Korsakoff syndrome.
Alcoholics can present with hypoglycemia so make sure you give thiamine BEFORE glucose as glucose can
precipitate wernickes encaph.

125. A 35yo male builder presented with sudden onset of severe abdominal pain. He was previously fit and
well other than taking ibuprofen for a long term knee injury. On examination he is in severe pain,
pulse=110bpm, BP=110/70mmHg and has a rigid abdomen. What is the most likely dx?
a. Biliary peritonitis
b. Ischemic colon
c. Pancreatic necrosis
d. Perforated diverticulum
e. Perforated peptic ulcer
Ans. The key is E. Perforated peptic ulcer. [NSAIDs induced perforation].
Peritonitis (Perforation of peptic ulcer/duodenal ulcer, diverticulum, appendix,
bowel, or gallbladder) Signs: prostration, shock, lying still, +ve cough test tenderness ( rebound/percussion
pain), board-like abdominal rigidity, guarding and no bowel sounds. Erect CXR may show gas under the
diaphragm.
NB: acute pancreatitis causes these signs, but does not require a laparotomy
so dont be caught out and always check serum amylase

126. A woman 5 days post-op for bilateral salphingo-oopherectomy and abdominal hysterectomy has
developed abdominal pain and vomiting a/w abdominal distension and cant pass gas. No bowel sounds
heard, although well hydrated. What is the most appropriate next step?
a. XR abdomen
b. Exploratory laparoscopy
c. CT
d. USG
e. Barium enema
Q. 1. What is the key?
Q. 2. What is the diagnosis?
Q. 3. What are the causes of it?
Q. 4. What is the management?
Ans. 1. The key is A. X-ray abdomen.
Ans. 2. The diagnosis is paralytic ileus.
Ans. 3. Causes of paralytic ileus: i) electrolyte imbalance ii) gastroenteritis iii) appendicitis iv) pancreatitis v)
surgical complications and vi) certain drugs.
Ans. 4. Management of paralytic ileus: i) nil by mouth ii) nasogastric suction to alleviate the distension and
remove the obstruction.

Bowel sounds are absent in paralytic ileus But bowel sounds are exaggerated in mechanical obstruction.
Ileus and incomplete small bowel obstruction can be conservatively managed while strangulation large bowel
obstruction requires surgery.
CT can confirm the level of obstruction.

127. A 30yo man complains of hoarseness of voice. Exam: unilateral immobile vocal cord. What is the most
probable dx?
a. Graves disease
b. Hematoma
c. Unilateral recurrent laryngeal nerve injury
d. External laryngeal nerve injury
e. Tracheomalacia
Ans. The key is C. unilateral recurrent laryngeal nerve injury.

Causes: 30% are cancers (larynx in ~40%; thyroid, oesophagus, hypopharynx,


bronchus, or malignant node). 25% are iatrogenic, ie after parathyroidectomy. Other causes: CNS disease
(polio; syringomyelia); TB; aortic aneurysm;
Symptoms: Symptoms of vocal cord paralysis are:
Hoarseness with breathy voice with a weak cough.
Repeated coughing/aspiration (weak sphincter + supraglottic sensation).
Exertional dyspnoea (glottis is too narrow to allow much air flow).

Nerve damaged with injury of superior thyroid artery: External laryngeal nerve
Nerve damaged with injury to inferior thyroid artery: Recurrent laryngeal nerve
128. A 38yo woman has delivered after an induced labor which lasted 26h. choose the single most likely
predisposing factor for postpartum hemorrhage?
a. Atonic uterus
b. Cervical/vaginal trauma
c. Rupture uterus
d. Fibroid uterus
e. Age of mother
Ans. The key is A. Atonic uterus.

Primary PPH is the loss of greater than 500mL (definitions vary) in the first 24h after delivery
Causes: uterine atony (90%), genital tract trauma (7%), clotting disorders (3%)
Risks: Antenatal Previous PPH or retained placenta BMI>35kg/m2 Maternal Hb<8.5g/dl at onset of labour
Antepartum haemorrhage Multiparity 4+ Maternal age 35y+ Uterine malformation or fibroids A large
placental site (twins, severe rhesus disease, large baby) Low placenta, Overdistended uterus
(polyhydramnios, twins) Extravasated blood in the myometrium (abruption).
In labour Prolonged labour (1st, 2nd or 3rd stage) Induction or oxytocin use Precipitant labour
Operative birth or caesarean section. Book mothers with risk factors for obstetric unit delivery.
Treatment: Give oxytocin 5U slowly IV for atonic uterus.
Attach oxygen, Give IV fluids, maintain systolic >100mmHg, Transfuse blood.
Is the placenta delivered? If it is, is it complete? If not, explore the uterus. If the placenta is complete, put
the patient in the lithotomy position with adequate analgesia and good lighting. Check for and repair trauma.
If the placenta has not been delivered but has separated, attempt to deliver it by controlled cord traction
after rubbing up a uterine contraction. If this fails, ask an experienced obstetrician to remove it under general
anaesthesia.Beware renal shut down.

129. A 32yo woman in tears describing constant irritability with her 2 small children and inability to relax. She
describes herself as easily startled with poor sleep and disturbed nightmares following a house fire a year ago,
while the family slept. What is the single best tx?
a. Rassurance
b. Relaxation therapy
c. Quetiapine
d. Lofepramine
e. Fluoxetine
Q. 1. What is the key
Q. 2. What is the diagnosis?
Q. 3. What are the points in favour of your diagnosis?
Ans. 1 The key is E. Fluoxetine. The key is probably a wrong key. Likely correct key is B. Relaxation therapy
Ans. 2. The diagnosis is post traumatic stress disorder.
Ans. 3. Points in favour of PTSD: i) H/O stressor (house fire a year ago) ii) Nightmares of the stressor iii) Hyper
arousal (very anxious and inability to relax (leading to irritability) iv) associated depression (poor sleep,
tearful).
Note: Fluoxetin and peroxetin are the drugs of choice in PTSD. CBT is the non-pharmacological treatment.
PTSD:
Symptoms: Fearful; horrified; dazed Helpless; numb, detached Emotional responsiveness
Intrusive thoughts Derealization Depersonalization Dissociative amnesia Reliving of events
Avoidance of stimuli Hypervigilance Lack of Concentration Restlessness Autonomic arousal: pulse; BP;
sweating Headaches; abdo pains
Signs: Suspect this if symptoms become chronic, with these
signs (may be delayed years): difficulty modulating arousal; isolated-avoidant modes of living; alcohol abuse;
numb to emotions and relationships; survivor guilt; depression; altered world
view in which fate is seen as untamable, capricious or absurd, and life can yield no meaning
or pleasure.
Treatment: Watchful waiting for mild cases.
For severe cases: CBT or eye movement desensitization and reprocessing is done. Drug treatment is not
recommended but in case it is needed prescribe mirtazapine or paroxetine.

So i agree in this question it is PTSD and B should be the answer.


130. A 22yo woman with longstanding constipation has severe ano-rectal pain on defecation. Rectal exam:
impossible due to pain and spasm. What is the most probable dx?
a. Anal hematoma
b. Anal fissure
c. Anal abscess
d. Protalgia fugax
e. Hemorrhoids
Ans. The key is B. Anal fissure.
Anal fissures: Acute If less than 6weeks, >6wks chronic.

Causes: Most are due to hard faeces. Spasm may constrict the inferior rectal artery, causing ischaemia, making
healing difficult and perpetuating the problem.

History of constipation almost always present. Examination is almost impossible due to severe pain.

Treatment: Acute: Increase fluid intake, fiber diet. Bulk forming laxatives are first line. Topical anesthetics are
used. Lactulose can be tried.

Chronic: Topical GTN is the first line and mainstay of treatment. If ineffective for >8wks surgical referral for
use of botulinum toxin.

131. A 20yo student attends the OPD with complaint of breathlessness on and off, cough and sputum. His
sleep is disturbed and skin is very dry in flexural areas of the body. Exam: tachypnea, hyperresonant
percussion and wheezing on auscultation. What is the most likely dx?
a. Extrinsic allergic alveolitis
b. Asthma
c. Wegeners granulomatosis
d. COPD
e. Cystic fibrosis
Q. What is the key?
Q. What are the diagnostic criteria?
Ans. The key is B. Asthma.
Ans. 2. Diagnostic criteria of asthma: i) Airway hyper-responsiveness to certain stimuli ii) Recurrent variable
airflow limitation usually reversible iii) presents as wheezing, breathlessness, chest tightness and cough.

ASTHMA.
Symptoms: Dyspnea, wheeze, cough (with or without sputum), chest tightness (4 most important) particularly
if symptoms are worse at night or early morning and in response to certain triggers like cold, exercise,
allergens. Symptoms exacerbated by use of NSAIDs and Beta blockers. Mostly there is history of allergy (atopy)
as in this question there is history of dry skin. Try to find the precipitating factor.
Signs Tachypnoea; audible wheeze; hyperinflated chest; hyperresonant percussion
note; reduced air entry ; widespread, polyphonic wheeze.

Management: CHRONIC (LONG TERM)


132. A pt with thought disorder washes hands 6x each time he uses the toilet. What is the best
management?
a. Psychodynamic therapy
b. CBT
c. Antipsychotics
d. Refer to dermatology
e. Reassure
Q. 1. What is the key?
Q. 2. What is the diagnosis?
Ans. 1. The key is B. CBT.
Ans. 2. The diagnosis is obsessive compulsive disorder.

OCD:
Compulsions are senseless, repeated rituals. Obsessions are stereotyped, purposeless words, ideas, or phrases
that come into the mind.
Repetitive behavior and an urge to do it.
Treatment: CBT is first line. Clomipramine (start with 25mg/day PO) or SSRIs (eg fluoxetine)

133. A 25yo woman presented to her GP on a routine check up. Upon vaginal exam, she was fine except for
finding of cervical ectropion which was painless but mild contact bleeding on touch. What is the next
management?
a. Endometrial ablation
b. Cervical smear
c. Colposcopy
d. Antibiotics
e. Vaginal US
f. Pack with gauze and leave to dry
Q. 1. What is the key?
Q. 2. Points in favour of key.
Ans. 1. The key is D. Antibiotics. WRONG KEY!
Ans. 2. Points in favour of antibiotic: Ectropion and contact bleeding can occur in infection. In the given case
swab is taken to establish or rule out infection. As this is not in options then the best response is antibiotics. If
improves with antibiotics then repeat smear in 6 months.

There is a red ring around the os because the endocervical epithelium has extended its territory over the paler
epithelium of the ectocervix. Ectropions extend temporarily under hormonal influence during puberty, with
the combined Pill, and during pregnancy. As columnar epithelium is soft and glandular, ectropion is prone to
bleeding, to excess mucus production, and to infection. Treatment: Once a normal cervical smear has been
confirmed, it is actively managed only if there are symptoms. After stopping any oestrogen-containing
contraceptive, treatment options are controversial but include diathermy, cryotherapy, surgery with laser
treatment and microwave therapy.

SO THE CORRECT ANSWER IS B.


134. A 32yo had a normal vaginal delivery 10 days ago. Her uterus has involuted normally. Choose the single
most likely predisposing factor for PPH?
a. Retained product
b. DIC
c. Uterine infection
d. Von Willebrand disease
e. Primary PPH
Q. 1. What is the key?
Q. 2. What type of PPH it would be?
Ans. 1. The key is C. uterine infection.
Ans. 2. Secondary PPH

Loss of >500ml blood in the first 24hrs after delivery is PRIMARY PPH.
Secondary PPH: This is excessive blood loss from the genital tract after 24h from delivery. It usually occurs
between 5 and 12 days and is due to infections (most common cause) (endometritis) or retained placenta.
Look for history of extended labour, difficult third stage, ragged placenta, PPH.
Symptoms: Abdominal pain. Offensive smelling lochia. Abnormal vaginal bleeding - PPH. Abnormal vaginal
discharge. Dyspareunia. Dysuria.
Signs: are those of sepsis. Tachycardia, fever, rigors, suprapubic tenderness.
Treatment: For endometritis: IV antibiotics if there are signs of severe sepsis. If less systemically unwell, oral
treatment may be sufficient. Piperacilin and tazobectum may be used.
If RPOC are suspected, elective curettage with antibiotic cover may be required. Surgical measures should be
undertaken if there is excessive or continuing bleeding, irrespective of ultrasound findings

135. A 37yo man slipped while he was walking home and fell on his out stretched hand. He complains of pain
in the right arm. XR showed fx of the head of radius. What is the single most associated
nerve injury?
a. Radial nerve
b. Musculocutaneous nerve
c. Median nerve
d. Ulnar nerve
Q. 1. What is the key?
Q. 2. What is the root value?
Ans. 1. The key is A. Radial nerve.
Ans. 2. Root value of radial nerve: C5,6,7,8 and T1.

136. A butcher stabbed accidently his groin. He bled so much that the towel was soaked in blood and
BP=80/50mmHg, pulse=130bpm. What % of circulatory blood did he lose?
a. <15%
b. 15-30%
c. 30-40%
d. 40-50%
e. >50%
Q. 1. What is the key?
Q. 2. What is the classification of blood loss according to vital sign?
Ans. 1. The key is C. 30-40%
Ans. 2. Hypovolemic shock Classification:
1. Class 1 up to 15% of blood volume lost: pulse <100; systolic BP normal; pulse pressure normal;
Respiratory rate 14-20; urine output greater than 30 ml/hour.
2. Class 2 15%-30% blood volume lost: pulse 100-120; systolic blood pressure normal; pulse pressure
decreased; respiratory rate 20-30; urine output 20-30 ml/hour.
3. Class 3 30%-40% blood volume lost: pulse 120-140; systolic BP decreased; pulse pressure decreased,
respiratory rate 30-40; urine output 5-15 ml/hr
4. Class 4, blood loss of greater than 40%: pulse rate >140; systolic BP decreased; pulse pressure
decreased respiratory rate >35; urine output negligible.

137. A 67yo man presents with palpitations. ECG shows an irregular rhythm and HR=140bpm. He is otherwise
stable, BP=124/80 mmHg. What is the most appropriate management?
a. Bisoprolol
b. ACEi
c. Ramipril
d. Digoxin
Ans. The key is A. Bisoprolol.

The patient has Atrial fibrillation. Irregularly irregular pulse and tachycardia.

Agents used to control rate in patients with atrial fibrillation


beta-blockers
calcium channel blockers
digoxin (not considered first-line anymore as they are less effective at controlling the heart rate during
exercise. However, they are the preferred choice if the patient has coexistent heart failure)

Agents used to maintain sinus rhythm in patients with a history of atrial fibrillation
sotalol
amiodarone
flecainide
others (less commonly used in UK): disopyramide, dofetilide, procainamide, propafenone, quinidine

TREATMENT CHOICE:
In the given question since the patient is above the age of 65 so rate control is done! For which either a beta
blocker or calcium channel blocker is used!
138. A 78yo man is depressed after his wifes death. He has been neglecting himself. His son found him in a
miserable state when he went to visit. The son cant deal with his father. What is the appropriate
management?
a. Voluntary admission to psychiatry ward
b. Hand over to social worker
c. Request son to move in with father
d. Send pt to care home
Ans. The key is A. Voluntary admission to psychiatry ward.
139. An old alcoholic presents with cough, fever, bilateral cavitating consolidation. What is the most probable
cause?
a. Gram +ve diplococcic
b. Coagulase +ve cocci
c. Gram ve cocci
d. AFB
e. Coagulase ve cocci
Q. 1. What is the key?
Q. 2. What is the organism?
Ans. 1. The key is B. Coagulase +ve cocci.
Ans. 2. Name of organism is Staphylococcus aureus.

Legionella: hotel stay, foreign travel, flu like symptoms, hyponatremia, pleural effusion. TEST: urinary
antigen. CXR shows bi-basal consolidation
Mycoplasma Pneumonae: Rash (erythema multiforme), unusual symptoms (abd pain, dry cough), long
duration of symptoms, hyponatremia, Diagnosis by serology. CXR: reticular-nodular shadowing or patchy
consolidation
Staphylococcal pneumonia may complicate influenza infection and is seen most frequently in the
elderly and in intravenous drug users or patients with underlying disease. Shows bilateral cavitations.
Pneumonia associated with COPD: H.influenze (more likely) or P.aeruginosa
P.aeruginosa: Common in bronchiectasis or CF. Also causes hospital acquired infection.
Klebsiella pneumoniae is classically in alcoholics
Strept pneumonia: Associated with herpes labialis. commoner in the elderly, alcoholics, post-
splenectomy, immunosuppressed and patients with chronic heart failure or pre-existing lung disease
Pneumocystis pneumonia (PCP) causes pneumonia in the immunosuppressed (eg HIV). CXR may be
normal or show bilateral perihilar interstitial shadowing. Diagnosis: visualization of the organism in induced
sputum, bronchoalveolar lavage, or in a lung biopsy specimen

SO i think it is either klebsiella (gram - rod) or streptococcus as these are the ones common in alcoholics but
bilateral cavitations do point in favor of staphylococcus.

140. A 67yo man had successful thrombolysis for an inf MI 1 month ago and was discharged after 5 days. He is
now re admitted with pulmonary edema. What is the most probable dx?
a. Aortic regurgitation
b. Ischemic mitral regurgitation
c. Mitral valve prolapse
d. Pulmonary stenosis
e. Rheumatic mitral valve stenosis
Ans. The key is B. Ischaemic mitral regurgitation. [ Causes of Ischaemic mitral regurgitation: left ventricular
remodeling and dysfunction, annular dilation/dysfunction, and mechanical dyssynchrony].

Complications OF MI:
Cardiac arrest
Unstable angina
Bradycardias or heart block
cardiogenic shock
Tachyarrhythmias:
Consider implantable cardiac defibrillator
Right ventricular failure (RVF)/infarction
Pericarditis
DVT & PE:
Systemic embolism:
Cardiac tamponade
Mitral regurgitation
Ventricular septal defect
Late malignant ventricular arrhythmias
Dresslers syndrome
Left ventricular aneurysm

Mitral regurgitation: May be mild (minor papillary muscle dysfunction) or severe


(chordal or papillary muscle rupture or ischaemia). Presentation: Pulmonary oedema.
Treat LVF and consider valve replacement.

141. A 60yo lady who had stroke 3 years ago now reports having increased dyspnea on exertion and atrial
fibrillation. CXR: straight left border on the cardiac silhouette. What is the most probable
dx?
a. Aortic regurgitation
b. Ischemic mitral regurgitation
c. Mitral valve prolapse
d. Pulmonary stenosis
e. Rheumatic mitral valve stenosis
Q. 1. What is the key?
Q. 2. What are the points in favour of your answer?
Ans. 1. The key is E. Rheumatic mitral valve stenosis.
Mitral Stenosis: Presentation: dyspnoea; fatigue; palpitations; chest pain; systemic emboli; haemoptysis;
chronic bronchitis-like picture
CAUSES: Rheumatic, congenital, mucopolysaccharidoses, endocardial fibroelastosis, malignant carcinoid
(rare), prosthetic valve.
SIGNS: Malar flush on cheeks (due to inc cardiac output); low-volume pulse; AF common;
tapping, non-displaced, apex beat (palpable S1). On auscultation: loud S1; opening
snap (pliable valve); rumbling mid-diastolic murmur (heard best in expiration,
with patient on left side
ECG show P-mitrale ECHO is diagnostic. CXR: left atrial enlargement
(double shadow in right cardiac silhouette)
TREATMENT: balloon valvuloplasty (if pliable, non-calcified valve), open mitral valvotomy
or valve replacement.
Complications: Pulmonary hypertension, emboli, pressure from large LA on local
structures, eg hoarseness (recurrent laryngeal nerve), dysphagia (oesophagus),
bronchial obstruction; infective endocarditis

Ans. 2. Points in favour: i) Dyspnoea on exertion ii) Straight left border of the cardiac silhouette. Iii) Atrial
fibrillation is a common association.
142. A 60yo diabetic complains of pain in thigh and gluteal region on walking up the stairs for the last 6
months. She is a heavy smoker and has ischemic heart disease. What is the most appropriate dx?
a. Thromboangitis Obliterans
b. Sciatica
c. DVT
d. Atherosclerosis
e. Embolus
Q. 1. What is the key?
Q. 2. What are the points in favour?
Ans. 1. The key is D. Atherosclerosis.
Ans. 2. i) It is not sciatica as sciatica pain is worse when sitting. There may be weekness, numbness, difficulty
moving the leg or foot. A constant pain on one side of the rear. A shooting pain that makes it difficult to stand
up. ii) It is not DVT as no swelling, warmth or redness of skin are there iii) It is not thromboangitis obliterans as
pulses are ok, no colour change or reduced hair growth, no ulceration or gangrene iv) no embolism as no pain
(rest pain), no numbness, no redness or itching or rash, no ulceration of skin.
This patient has intermittent claudication due to atherosclerosis,
Symptoms Cramping pain is felt in the calf, thigh, or buttock after walking for a given distance (the
claudication distance) and relieved by rest. Ulceration, gangrene, and foot pain at resteg burning pain at
night relieved by hanging legs over side of bedare the cardinal features of critical ischaemia.
Fontaine classification for peripheral arterial disease: 1. Asymptomatic 2. Intermittent
claudication 3.Ischaemic rest pain 4. Ulceration/gangrene (critical ischaemia)
Signs: Absent femoral, popliteal or foot pulses; cold, white leg(s); atrophic skin;
punched out ulcers (often painful); postural/dependent colour change; a vascular
(Buergers) angle of <20 and capillary filling time >15s are found in severe ischaemia
Imaging: Colour duplex USS is 1st line (non-invasive and readily available). If considering
intervention then MR/CT angiography
MANAGEMENT: Stop smoking, exercise, treat HTN, antiplatelet (clopidogrel). Advise exercise until maximum
tolerable pain.
Vasoactive drugs may be used. If PAD is advancing consider:
Percutaneous transluminal angioplasty (PTA) is used for disease limited to a single arterial segment.
Surgical reconstruction: arterial bypass
Amputation. If all fail. Knee should be saved if possible.

143. A 3yo child who looks wasted on examination has a hx of diarrhea on and off. The mother
describes the stool as bulky, frothy and difficult to flush. What is the single inv most likely to
lead to dx?
a. Sweat chloride test
b. Anti-endomysial antibodies
c. LFT
d. US abdomen
e. TFT
Q. 1. What is the key?
Q. 2. What is the diagnosis?
Ans. 1. The key is B. Anti-endomysial antibody
Ans. 2. The diagnosis is celiac disease.

Coeliac Disease: Suspect this in all those with diarrhoea + weight loss or anaemia (esp. if iron or B12). It is a T-
cell-mediated autoimmune disease of the small bowel in which prolamin
(alcohol-soluble proteins in wheat, barley, rye oats) intolerance causes villous atrophy
and malabsorption (including of bile acids)
Investigations: FBC, Dec feritin, dec vit.B12
Antibodies: alpha -gliadin, transglutaminase and anti-endomysial 95% specific. Duodenal biopsy shows
subtotal villous atrophy.
Treatment Lifelong gluten-free diet

144. A 45yo woman has had severe epigastric and right hypochondrial pain for a few hours. She has a normal
CBC, serum ALP is raised, normal transaminase. 3 months ago she had a
cholecystectomy done. What is the most appropriate inv?
a. US abdomen
b. ERCP
c. MRCP
d. CT abdomen
e. Upper GI endoscopy
Q. 1. What is the key?
Q. 2. What is the diagnosis?
Ans. 1. The key is B. ERCP.
Ans. 2. Diagnosis is choledocolithiasis.

Right upper quadrant pain think of gall stones. And since the LFTs here show obstructive picture ALP
increased with normal transaminases the obstruction is most probably in the biliary tract CBD.
ERCP: Endoscopic retrograde cholangiopancreatography (ERCP)
Indications: No longer routinely used for diagnosis, it still has a significant therapeutic role: sphincterotomy
for common bile duct stones; stenting of benign or malignant strictures and obtaining brushings to diagnose
the nature of a stricture.

MRCP: MRCP (magnetic resonance cholangiopancreatography) gives detail of the biliary system and the
pancreatic duct. MRCP has excellent sensitivity and specificity for diagnosing common bile duct stoneswhen
these are >6mm both are 99% (although accuracy is lower for stones <6mmand is the imaging modality of
choice.

But here since we need to remove the stones as well so we use ERCP.
145. A 53yo woman presented with pain in the eye, blurry vision and clumsiness for 3 months. She has a hx of
difficulty in swallowing and weakness in her right upper limb 2y ago. What is the inv of choice?
a. CSF analysis
b. EEG
c. EMG
d. MRI brain
e. Visual evoked response test
Q. What is the key?
Q. What is the diagnosis?
Ans. 1. The key is D. MRI brain.
Ans. 2. Diagnosis is multiple sclerosis.

Multiple sclerosis: Discrete plaques of demyelination occur at multiple CNS sites. Early exposure to
sunlight/vit. D is important, and vit. D status relates to prevention of MS,
PRESENTATION: Usually monosymptomatic: unilateral optic neuritis (pain on eye
movement and rapid central vision); numbness or tingling in the limbs; leg weakness; brainstem or cerebellar
symptoms (eg diplopia, ataxia). The disease has a relapsing - remitting course.
INVESTIGATIONS: This is clinical, as no test is pathognomonic. MRI is sensitive but not specific for plaque
detection.
CSF: Oligoclonal bands of IgG on electrophoresis suggest CNS inflammation but does not confirm MS.. Delayed
visual, auditory, and somatosensory evoked potentials.
MANAGEMENT: Stress free life. Give vit. D to achieve serum 25(OH)D levels of 50nmol/L
Methylprednisolone shortens relapse doesnt alter overall prognosis.
Beta interferon reduces replaces by 30% but does not reduce overall disability
Monoclonal antibodies: Alemtuzumab and natalizumab reduce relapses by 68%.
Glatiramer in secondary progressive
Azathioprine in relapsing-remiting
Palliation: Spasticity: Baclofen, diazepam, dantrolene or tizanidine.
Tremor: Botulinum toxin
Urgency/frequency: If post-micturition residual urine >100mL, teach intermittent self-catheterization; if
<100mL, try tolterodine

146. A 55yo male presents with malaise and tiredness. Exam: spleen approaching RIF, no
lymphadenopathy. Choose the single cell type?
a. Helmet shaped cell
b. Sickle cell
c. Granulocyte without blast cells
d. Blast cells
Q. 1. What is the key?
Q. 2. What is the diagnosis?
Q. 3. What are the diagnostic features?
diagnosis is CML.
Ans. 1. The key is C. Granulocyte without blast cells.
Ans. 2. The diagnosis is CML.
Ans. 3. Diagnostic features are i) increased number of mature granulocytes ii) huge splenomegaly.

Causes of Splenomegaly: If massive, think of: chronic myeloid leukaemia, myelofibrosis, malaria (or
leishmaniasis)
Splenomegaly with fever Infection (malaria, SBE/IE hepatitis, EBV,TB, CMV, HIV)
With lymphadenopathy Glandular fever, leukemia/lymphoma, Sjogrens syndrome
With purpura Septicemia, typhus, DIC, amyloid, meningococcemia
CML: It is a myeloproliferative disorder, common ge of occurence is 40-60 years.
Philadelphia chromosome (Ph) Present in >80% of those with CML t(9:22)
Symptoms Mostly chronic and insidious: weightloss, tiredness, fever, sweats. There
may be features of gout (due to purine breakdown), bleeding (platelet dysfunction),
and abdominal discomfort (splenic enlargement). ~30% are detected by chance.
Signs Splenomegaly (>75%)often massive. Hepatomegaly, anaemia, bruising (fi g 2).
Tests WBC increased (often >100109/L) with whole spectrum of myeloid cells, ie increased neutrophils,
myelocytes, basophils, eosinophils. Hb dec or normal, platelets variable. Urate increased B12increased. Bone
marrow hypercellular.
Treatment: is by chemotherapy or stem cell transplantation.

147. A 6yo pt comes with easy bruising in different places when she falls. CBC: WBC=25, Hgb=10.9, Plt=45. Her
paul brunnel test +ve. What is the most likely dx?
a. Glandular fever
b. ITP
c. Trauma
d. NAI
e. Septicemia
Q. 1. What is the key?
Q. 2. What are the lab. Values that suggests the diagnosis here?
Ans. 1. The key is A. Glandular fever.
Ans. 2. Suggestive lab. Values: WBC=25 (leucocytosis), Hgb=10.9 (usually patient is not anaemic), Plt=45
(thrombocytopenia-leading to easy bruising), Positive paul bunnel test.
INFECTIOUS MONONUCLEOSIS
Caused by EBV, spread by saliva or droplets. EBV also causes certain cancers (Hogdkins, burkitts and
nasopharyngeal CA)
Symptoms: Sore throat, inc T, anorexia, malaise, lymphadenopathy (esp. posterior
triangle of neck), palatal petechiae, splenomegaly, fatigue/mood
Blood film Lymphocytosis and atypical lymphocytes (large, irregular nuclei)
Heterophil antibody test (Monospot, Paul Bunnell) 90% show heterophil antibodies by 3wks, disappearing
after ~3 months PCR may also be done.
Treatment: None usually needed.Avoid contact sports for 8 weeks. Avoid alcohol. Steroid or acyvlovir may be
given but there is not much benefit.
Never give ampicillin or amoxicillin for sore throats as they often cause a severe
rash in those with acute EBV infection

148. A 41yo woman who has completed her family, has suffered from extremely heavy periods for many
years. No medical tx has worked. She admits that she would rather avoid open surgery.
After discussion, you collectively decide on a procedure that wouldnt require open surgery or
GA. Select the most appropriate management for this case.
a. Endometrial ablation
b. Hysterectomy
c. Fibroid resection
d. Myomectomy
e. Uterine artery embolization
Ans. The key is uterine artery embolization.
Treating menorrhagia:
Drugs
Progesterone-containing IUCDs, eg Mirena should be considered 1st line treatment for those wanting
contraception. effective for bleeding and also reduce the size of fibroid uterus.
2nd line recommended drugs are antifibrinolytics, antiprostaglandins or the
Pill. Antifibrinolytics Taken during bleeding these reduce loss (by 49%)eg tranexamic acid CI:
thromboembolic disease
Antiprostaglandins eg mefenamic acid 500mg/8h PO pc (CI: peptic ulceration) taken duringdays of bleeding
particularly help if there is also dysmenorrhoea.
COCP can also be used if they are not contraindicated..
3rd line recommendation is progestogens IM or norethisterone
Rarely gonadotrophin (LHRH) releasing hormones are used

Surgery Endometrial resection is suitable for women who have completed


their families and who have <10wk size uterus and fibroids <3cm. Contraception will be required.
For women wishing to retain fertility who have fibroids >3cm consider uterine
artery embolization or myomectomy
Women not wishing to retain fertility, with a uterus >10wk size and fibroids >3cm may benefit from
hysterectomy, vaginal hysterectomy being the preferred route.

149. A girl with hx of allergies visited a friends farm. She got stridor, wheeze and erythematous rash. What is
the most appropriate tx?
a. 0.25ml IM adrenaline
b. 0.25ml PO adrenaline
c. 0.25ml IV adrenaline
d. IV chlorphearamine
Ans. The key is A. 0.25 ml IM adrenaline [Presence of stridor and wheeze are suggestive of anaphilaxis and
treatment option is adrenaline].
Consider anaphylaxis when there is compatible history of rapid-onset severe allergic-type reaction with
respiratory difficulty and/or hypotension, especially if there are skin changes present and the treatment of
anaphylaxis is IM adrenaline not anti histamine

Treatment: ABCDE, Oxygen, IM Adrenaline 1:1000. <6yrs 0.15ml, 6-12yrs 0.3ml; >12 yrs 0.5ml 1:1000

Since the age of the girl is not mentioned here and options A is the answers supposing the girl was 6-12 yrs of
age.

150. A 5yo boy is referred to the hospital and seen with his father who is worried that he has been listless. He
is not sure why his GP suggested he should come to the ED and is keen to get some tablets and go home.
Exam: tired and irritable, swelling around eyes. Renal biopsy: remarkable
for podocyte fusion on EM. What is the most probable dx?
a. NAI
b. Myelodysplastic disease
c. HSP
d. Membranous GN
e. Minimal change GN
Ans. The key is E. Minimal change glomerulonephritis. [Podocyte fusion on electron microscopy]
Most common cause of nephrotic in children is minimal change disease. There will be hypoalbuminemia and
peripheral edema too. Electron microscopy shows effacement of podocyte foot processes.. MCD has albumin
selective proteinuria. Treatment is with steroids.

151. A 6yo boy is brought to the hospital for a 3rd episode of sore throat in 1 month. He is found
bleeding from gums and nose and has pale conjunctiva. Whats the single cell type?
a. Clumped platelets
b. Microcytes
c. Granulocyte without blast cells
d. Blast cells
e. Mature lymphocytes
Q. 1. What is the key?
Q. 2. What is the diagnosis?
Q. 3. What are the points that favour diagnosis?
Ans. 1. The key is D. Blast cells.
Ans. 2. The diagnosis is ALL
Ans. 3. Points in favour: i) Age-6yrs ii) recurrent infection (sorethroat) due to neutropenia and abnormal
lymphoblasts which cannot protect from infection iii) thrombocytopenia causing gum and nose bleeding. Iii)
anaemia (pale conjunctiva) due to reduced red cell production from marrow occupation by blast cells. [Here
debate came why it is not aplastic anaemia? There is no risk factor mentioned for this patient for aplastic
anaemia. There may be congenital aplastic anaemia but again it would present earlier in life. So it goes more
with leukaemia but it cannot be confirmed unless we do bone marrow aspiration.]

ALL: This is a malignancy of lymphoid cells, affecting B or T lymphocyte cell lines, arresting
maturation and promoting uncontrolled proliferation of immature blast
cells, with marrow failure and tissue infiltration.
Causes: Genetic susceptibility, environmental factors (ionizing radiations) Downs syndrome.
Commonest cancer of childhood.
Signs and symptoms: (varies depending on the type of infiltration)
Marrow failure (Aplastic anemia): Anaemia (Hb), infection (WCC), and bleeding (platelets).
Infiltration: Hepatosplenomegaly, lymphadenopathysuperficial or mediastinal, orchidomegaly, CNS
involvementeg cranial nerve palsies, meningism.
INVESTIGATIONS: Characteristic blast cells on blood film and bone marrow
CXR and CT scan to look for mediastinal and abdominal lymphadenopathy.
Lumbar puncture should be performed to look for CNS involvement.
TREATMENT: Blood transfusions, prophylactic antibiotics, IV antibiotics in case of infection. Main stay of
treatment is chemotherapy.
Prognosis Cure rates for children are 7090%; for adults only 40%

152. A 23yo man has been stabbed in the back and has SOB. The trachea is not deviated, he has engorged
neck veins and absent breath sounds on the right. What is the most appropriate dx?
a. Tension pneumothorax
b. Cardiac tamponade
c. Simple pneumothorax
d. Hemothorax
e. Pleural effusion
Q. 1. What is the key?
Q. 2. What are the point in favour of your answer?
Ans. 1. The key is A. Tension pneumothorax.
Ans. 2. Points in favour: i) Stab wound in the back ii) SOB iii) Engorged neck vein iv) Absent breath sound.

**WHY I THINK IT IS HEMOTHORAX!!

153. A 44yo pt comes with right hemiparesis. Exam: left sided ptosis and left dilated pupil. Where is the
lesion?
a. Cerebral infarct
b. Cerebellar infarct
c. Medulla oblongata
d. Pons
e. Midbrain
Q. 1. What is the key?
Q. 2. What is the name of this condition?
Ans. 1. The key is E. Midbrain.
Ans. 2. Weber syndrome [presence of ipsilateral oculomotor nerve palsy and contralateral hemiparesis or
hemiplagia].

Ptosis + miosis = horners syndrome Ptosis + mydriasis= oculomotor nerve palsy.


CN3 nucleus lies in the midbrain.
Fore brain: CN 1,2
Mid brain: CN 3,4
Pons: CN 5,6,7,8
Medulla: CN 9,10,12

Webers syndrome (superior alternating hemiplegia) Ipsilateral oculomotor


nerve palsy with contralateral hemiplegia, due to infarction of one-half of the midbrain,
after occlusion of the paramedian branches of the basilar or posterior cerebral
arteries

154. A 50yo man has a stab wound to his left anterior chest at the level of the 4th ICS. He has a BP 80mmHg,
pulse=130bpm. His neck veins are dilated and his heart sounds are faint. His trachea is central. What is the
most appropriate dx?
a. Cardiac tamponade
b. Diaphragmatic rupture
c. Fractured ribs
d. Tension pneumothorax
e. Traumatic rupture of aorta
Q. 1. What is the Key?
Q. What are the points in favour of your answer?
Ans. 1. The key is Cardiac tamponade.
Ans. 2. Points in favour: i) Systolic BP 80 mmHg ii) Pulse 130 bpm iii) Engorged neck vein iv) Faint heart sounds
v) Trachea is central.

Essence:Pericardial fluid collects intrapericardial pressure rises heart cannot fill pumping stops.
Causes: Trauma, lung/breast cancer, pericarditis, myocardial infarct, bacteria, eg TB.
Signs: Falling BP, a rising JVP, and muffled heart sounds (Becks triad); JVP on inspiration
(Kussmauls sign); pulsus paradoxus (pulse fades on inspiration) (also in severe asthma). Echocardiography
may be diagnostic. CXR: globular heart; left heart border convex or straight; right cardiophrenic angle <90.
ECG: electrical alternans
Management: Prompt pericardiocentesis. While waiting give O2, IV fluids, monitor ECG.

155. A 15yo boy has a soft painless swelling in the left scrotum, blue in color and can be compressed. What is
the most appropriate next step?
a. Analgesia
b. Antibiotic
c. Biopsy
d. Immediate surgery
e. Reassurance
Q. 1. What is the key?
Q.2. What is the name of this condition?
Ans. 1. The key is E. Reassurance.
Ans. 2. Name of the condition is Varicocele.

Testicular lump = cancer until proven otherwise.


Acute, tender enlargement of testis = torsion until proven otherwise
Diagnosing scrotal masses
Cannot get above inguinoscrotal hernia or hydrocele extending proximally
Separate and cystic epididymal cyst
Separate and solid epididymitis/varicocele
Testicular and cystic hydrocele
Testicular and solidtumour, haematocele, granuloma, orchitis, gumma. USS may help.

Varicocele Dilated veins of pampiniform plexus. Left side more commonly affected.
Often visible as distended scrotal blood vessels that feel like a bag of worms.
Patient may complain of dull ache. Associated with subfertility, but repair (via surgery
or embolization) seems to have little effect on subsequent pregnancy rates.

156. A 12yo pt presents with copious diarrhea. Exam: urine output=low, mucous membrane=dry, skin
turgor=low. What is the most appropriate initial management?
a. Antibiotic
b. Antimotility
c. Anti-emetic
d. Fluid replacement
e. Reassurance
Q. 1. What is the key?
Q. 2. What is the diagnosis and why?
Ans. 1. The key is D. Fluid replacement.
Ans. 2. Diagnosis is severe dehydration. Points in favour: i) low urine output ii) dry mucous membrane and iii)
low skin turgor.

Gastroenteritis Rotavirus is the most common cause of gastroenteritis in infants and children.
Norovirus (most common cause in adults)

ASSESSMENT OF DEHYDRATION:
Mild dehydration: Decreased urine output.
5% dehydration: Dry mucous membranes; decreased urine output.
10% dehydration: The above + sunken fontanele, inc pulse; hoarse cry; dec skin turgor.
>10%: The above, but worse, with: shock, drowsiness, and hypotension.

MANAGEMENT
Mild: Treated at home by oral rehydrating therapy.
Moderate: Oral fluids, via NG or IV fluids can be used. Rapid rehydration involves 4 hours of
10mL/kg/h 0.9% NaCl then maintenance after if needed. Monitor U & Es
Severe: If not in shock oral or NG route can be used. If in shock. 0.9% saline 20mL/kg IVI bolus, while
calculations are performed. Continuously monitor pulse, BP, ECG. Continue with boluses until the signs of
shock ease.
Then give the daily requirement + fluid deficit

157. A 60yo smoker presents with cramp-like pain in the calves relieved by rest and non-healing ulcers. Exam:
cold extremities with lack of hair around the ankles, absent distal pulses. What is the
most probable dx?
a. Intermittent claudication
b. Chronic ischemia of the limbs
c. Buergers disease
d. DVT
e. DM
Q. 1. What is the key?
Q. 2. Points that support your diagnosis.
Ans. 1. The key is B. Chronic ischaemia of the limb.
Ans. 2. Intermittent claudication is a symptom not diagnosis. It is not buergers disease as buerger occur in
more younger heavy smoker (before the age of 50yrs) mostly limited to the extremities, It is not DVT as dvt
pain or tenderness is not of an intermittent claudication pattern. Again in DM there is no intermittent
claudication.

158. An otherwise healthy 13yo boy presents with recurrent episodes of facial and tongue swelling and
abdominal pain. His father has had similar episodes. What is the most likely dx?
a. C1 esterase deficiency
b. HIV
c. Mumps
d. Sarcoidosis
e. Sjogrens syndrome
Q. 1. What is the key?
Q. 2. What is the name of this condition?
Q. 3. Why it is not acquired?
Ans. 1. The key is A. C1 esterase inhibitor deficiency.
Ans. 2. Hereditary angioedema.
Ans. 3. Acquired angioedema usually manifest after the age of 40 yrs.

The oedema is triggered by increased permeability of the blood vessels. The net result is episodes of massive
local oedema, ie angio-oedema. (In angio-oedema, the swelling is subcutaneous or submucosal rather than
epidermal, so urticaria is absent.) It can mimic anaphylaxis.
Type I has low levels of C1-INH (C1 esterase inhibitor) (the majority of cases).
Type II has impaired function of C1-INH.

Clinical features
Recurrent episodes of angio-oedema and/or abdominal pain - may involve: Laryngeal oedema - can be fatal:
Pointers to a diagnosis of HAE are:
Family history.
Recurrent episodes of non-urticarial swelling lasting >24 hours, and unresponsive to antihistamines.
Laryngeal oedema.
Recurrent, unexplained abdominal pain and vomiting.
Symptoms starting in childhood and worsening in adolescence.
The recommended initial tests are:
Serum complement factor 4 (C4) level.
C1 inhibitor (C1-INH) antigenic protein level.
C1-INH function (if available).

Management involves:
Emergency treatment of attacks
Patient education and awareness; may need own supply of emergency treatment.
Good links with A&E departments.
Prophylaxis:
o Short-term cover for procedures - eg, dental treatment.
o Long-term prophylactic drugs if required.
o Avoidance of triggers.
Testing of family members is recommended owing to the potential seriousness of an attack.
Drugs:
Plasma-derived C1-INH, A bradykinin receptor inhibitor, Antifibrinolytic drugs - eg, tranexamic acid,
Attenuated androgens - eg, danazol.
159. A 25yo had an LSCS 24h ago for fetal distress. She now complains of intermittent vaginal
bleeding. Observations: O2 sat=98% in air, BP=124/82mmHg, pulse=84bpm, temp=37.8C. The
midwife tells you that she had a retained placenta, which required manual removal in the OT.
Choose the most appropriate C-Section complication in this case?
a. Retained POC
b. Aspiration pneumonitis
c. Endometritis
d. Uterine rupture
e. DIC
Q. 1. What is the key?
Q. 2. What are the points in favour?
Ans. 1. The key is C. Endometritis.
Ans. 2. More handling of tissue like manual removal of placenta, intermittent vaginal bleeding and raised
temperature points toward infective process like endometritis.

This is secondary PPH.


Secondary PPH: This is excessive blood loss from the genital tract after 24h from delivery. It usually occurs
between 5 and 12 days and is due to infections (most common cause) (endometritis) or retained placenta.
Look for history of extended labour, difficult third stage, ragged placenta, PPH.
Symptoms: Abdominal pain. Offensive smelling lochia. Abnormal vaginal bleeding - PPH. Abnormal vaginal
discharge. Dyspareunia. Dysuria.
Signs: are those of sepsis. Tachycardia, fever, rigors, suprapubic tenderness.
Treatment: For endometritis: IV antibiotics if there are signs of severe sepsis. If less systemically unwell, oral
treatment may be sufficient. Piperacilin and tazobectum may be used.
If RPOC are suspected, elective curettage with antibiotic cover may be required. Surgical measures should be
undertaken if there is excessive or continuing bleeding, irrespective of ultrasound findings

160. A 30yo woman has brief episodes of severe shooting pain in the rectum. Rectal examination and flexible
sigmoidoscopy are normal. What is the most probable dx?
a. Anal hematoma
b. Anal fissure
c. Rectal carcinoma
d. Proctalgia fugax
e. Piles
Ans. is D. Proctagia fugax [normal rectal examination and flexible sigmoidoscopy excludes other options].

Shooting pain mostly neuropathic pain.

Proctalgia fugax Idiopathic (could be because of spasm of muscles) , intense, brief, stabbing/crampy rectal
pain,often worse at night. Very short lived pain. The mainstay of treatment is reassurance. Inhaled salbutamol
or topical GTN (0.20.4%) or topical diltiazem (2%) may help.

161. A 78yo male, DM and HTN, had a fall and since then is unable to walk. He presents with
deformity and tenderness over the right hip area. XR=fx of femur neck. What is the single most
associated nerve injury?
a. Sciatic nerve
b. Gluteal nerve
c. Lateral peroneal nerve
d. Tibial nerve
e. Femoral nerve
Ans. The key is A. Sciatic nerve.

Fractures of the femoral neck are far more common in the elderly but fractures of the femoral shaft and
supracondylar fractures most often occur in adolescents and young adults. Hip fracture is the most common
reason for admission to an orthopaedic trauma ward. Intertrochanteric fractures affect the base of the
femoral neck, May disrupt the blood supply to the femoral head, leading to avascular necrosis.

Posterior dislocation of the hip


This is caused by major force to a flexed knee and hip - eg, when knees strike the dashboard in a road
traffic accident.
Posterior dislocations account for the majority of hip dislocations.
The affected leg is shortened and internally rotated with flexion and adduction at the hip. This
appearance may not occur if there is also a femoral shaft fracture.
Diagnosis is usually obvious on AP X-ray
Treatment: ABC, Pain management, reduction under GA. Allis technique

Complications
These include:
Sciatic nerve injury: pain in the distribution of the sciatic nerve, loss of sensation in the posterior leg
and foot and loss of dorsiflexion (peroneal branch) or plantar flexion (tibial branch) of the foot.
Vascular injury: not as frequent as with anterior dislocations.
Avascular necrosis of the femoral head: risk increases the longer the hip is dislocated. [1]
Secondary osteoarthritis.
Anterior dislocation of the hip
This is much less common.
It causes pain in the hip and inability to walk or adduct the leg.
The leg is externally rotated, abducted, and extended at the hip.

Complications
These include damage to the femoral nerve, artery and vein:
Injury to the femoral nerve may occur, resulting in paralysis and numbness in the femoral nerve
distribution.
Injury to the femoral artery may produce arterial insufficiency in the leg

162. A 20yo man has a head on collision in a car. On presentation his is breathless, has chest pain and fx of 5-
7th rib. CXR confirms this. What is the most appropriate initial action in this pt?
a. Antibiotics
b. Analgesia
c. O2 by mask
d. Physiotherapy
e. Refer to surgeon
Ans. The key is C. O2 by mask. [There was debate in this forum that pain relief should be given first which will
automatically relieve breathing problem. But others told O2 first]. O2 first is the correct answer!
[http://www.urmc.rochester.edu/encyclopedia/content.aspx?contenttypeid=22&contentid=flailchest]

ABCDE always comes first.

163. A 28yo man with complains of headache and nose bleeds also has pain in the lower limbs on
exertion. Exam: radio-femoral delay, cold legs with weak pulse and mild systolic murmur with
normal S1S2. What is the most probable dx?
a. TOF
b. ASD
c. VSD
d. PDA
e. CoA
Ans. The key is coarctation of aorta. [headache and nosebleeds - >hypertension, pain in lower limb on exertion
-> as reduced blood supply to leg due to coarctation, radio-femoral delay, cold legs with week pulse, mid-
systolic murmur are all features of coarctation of aorta].
Radio femoral delay is a clincher for coarctation of aorta.

Coarctation of the aorta Congenital narrowing of the descending aorta. More common in boys.
Associations: Bicuspid aortic valve; Turners syndrome.
Signs: Radiofemoral delay
(femoral pulse later than radial); weak femoral pulse; BP; scapular bruit; systolic murmur (best heard over
the left scapula).
Complications: Heart failure; infective endocarditis. Coarctation of the aorta is associated with berry
aneurysms which if ruptured cause Sub arachnoid hemorrhage.
Tests: CT or MRI-aortogram, CXR shows rib notching.
Cardiac catheterisation
To confirm the diagnosis when this is not clear at ultrasound.
To determine the gradient across the coarctation (with a gradient in excess of 20 mm Hg considered to
be significant).
To assess other abnormalities and the overall haemodynamic picture when considering therapeutic
options in more detail.
Therapeutically using balloon angioplasty with or without stent implantation.

Treatment: Surgery or balloon dilatation stenting.

164. A 23yo male has a tonic clonic seizure whilst at college. His GCS is 12, BP=120/77mmHg,
HR=99bpm. What is the most appropriate inv for his condition?
a. CT
b. MRI
c. Serum blood glucose
d. Serum drug levels
Ans. The key is C. Serum blood glucose [it is also possible that he may have taken drug, even though first we
have to do serum glucose as its presence can be very easily managed and it needs urgent management to save
life. If it is excluded then we can look for other causes which may be not fatal in short time as hypoglycaemia].
This case will be treated as status epilepticus. Normally status occurs in a patient with known epilepsy but
here no such history is given. Even in a person who presents with his first seizure we need to look for
hypoglycemia first before going onto later tests.

Status epilepticus:
This means seizures lasting for >30min, or repeated seizures without intervening
consciousness.
Also consider eclampsia in mind if the patient is female and the abdomen is distended.

Investigations
Bedside glucose, the following tests can be done once treatment has started: lab glucose,
ABG, U&E, Ca2+, FBC, ECG.
Consider anticonvulsant levels, toxicology screen, LP, culture blood and urine, EEG,
CT, carbon monoxide level. Pulse oximetry, cardiac monitor.

Treatment: Try to control seizure in less than 20mins as there could be permanent brain damage.
1. IV lorazepam:0.1mg/kg. Repeat if no response in 10mins. Be careful about respiratory depression. If there is
no IV access give PR Diazepam.
2.Buccal midazolam: Alternative oral route. squirt half the volume between the lower gum and the cheek on
each side.
3.Phenytoin infusion: 1520mg/kg IVI (roughly 1g if 60kg, and 1 . 5g if 80kg; max 2g. 2nd line! Dont use if
bradycardic or hypotensiv. ECG monitoring is recommended.
Diazepam infusion: eg 100mg in 500mL of 5% dextrose. It is most unusual for seizures to remain unresponsive
following this. If they do, allow the idea to pass through your mind that they could be pseudoseizures,
particularly if there are odd features (pelvic thrusts; resisting attempts to open lids and your attempts to do
passive movements; arms and legs flailing around).
4. Dexamethasone: 10mg IV if vasculitis/cerebral oedema (tumour) possible.
5. General anaesthesia: For refractory status: get anaesthetist/ICU involved early

165. A 20yo man complains of recent onset of itching which followed a viral infection. There are
numerous wheals of all sizes on his skin particularly after he has scratched it. These can last up
to an hour. What is the most probable dx?
a. Uremia
b. Urticaria
c. Psychogenic itching
d. Atopic eczema
e. Primary biliary cirrhosis
Ans. The key is B. Urticaria.
Urticaria Signs: wheals, rapid onset after taking drug association with angio-oedema /anaphylaxis. It can
result from both immunological and non-immunological mechanisms.
Causes: Drugs:morphine & codeine cause direct mast cell degranulation; penicillins & cefalosporins trigger IgE

responses; NSAIDs; ACEi.


Clinical diagnosis. No investigations required.
Management:
Find the cause and avoid/treat it.

Antihistamines:
Non-sedating H1 antihistamines are the mainstay of treatment
In pregnancy chlorphenamine is often the first choice of antihistamine.

166. A 75yo lady who had mitral valve replacement 13 yrs ago has developed recurrent
breathlessness. Her husband has noticed prominent pulsation in her neck. She complains of
abdominal pain and ankle swelling. What is the most probable dx?
a. Aortic regurgitation
b. Mitral regurgitation
c. Mitral stenosis
d. Tricuspid regurgitation
e. Pulmonary stenosis
Ans. The key is D. Tricuspid regurgitation. [Points in favour: i) recurrent breathlessness if the cause is LV
dysfunction, ii) prominent pulsation in the neck giant v waves, iii) abdominal pain pain in liver on exertion,
ankle swelling; These are features of tricuspid regurgitation. Reference:- OHCM, 9th edition, page- 142]

Pulmonary stenosis is mostly congenital. But it can be caused by rheumatic fever and in this case the patient
could have developed rheumatic fever given his history of valve replacement. But there is no mention of
abdominal pain in pulmonary stenosis so i guess thats the differentiating point here.
Aortic regurgitation, mitral stenosis and regurgitation do not involve abdominal pain and ankle swelling. And
the JVP will not be raised.

Tricuspid regurgitation
Causes: Functional (RV dilatation; eg due to pulmonary hypertension induced by LV failure); rheumatic fever;
infective endocarditis (IV drug abuser); carcinoid syndrome; congenital (eg ASD, AV canal, Ebsteins anomaly,
ie downward displacement of the tricuspid valve drugs (eg ergot derived dopamine agonists,fenfluramine).
Symptoms: Fatigue; hepatic pain on exertion; ascites; oedema and also dyspnoea and orthopnoea if the cause
is LV dysfunction.
Signs: Giant v waves and prominent y descent in JVP, RV heave; pansystolic murmur, heard best at lower
sternal edge in inspiration; pulsatile hepatomegaly; jaundice; ascites.
Management: Treat underlying cause. Drugs: diuretics, digoxin, ACE-i. Valve replacement (~10% 30-day
mortality).
.
167. A 45yo T1DM had an annual check up. Ophthalmoscopy showed dot and blot hemorrhage + hard exudate
and multiple cotton wool spots. What is the next step in management?
a. Reassurance and annual screening only
b. Urgent referral to ophthalmologist
c. Laser therapy
d. Non-urgent referral to ophthalmologist
e. Nothing can be done
Ans. The key is D. Non-urgent referral to ophthalmologist. [It is pre-proliferative retinopathy so non-urgent
referral; If proliferative (with neovascularization) urgent referral].

Diabetic retinopathy Blindness is preventable. Annual retinal screening mandatory


for all patients not already under ophthalmology care. Pre-symptomatic screening enables laser
photocoagulation to be used, aimed to stop production of angiogenic factors from the ischaemic retina.
Indications: maculopathy or proliferative retinopathy.
Background retinopathy: Microaneurysms (dots), haemorrhages (blots) and
hard exudates (lipid deposits). Refer if near the macula, eg for intravitreal triamcinolone.
Pre-proliferative retinopathy: Cotton-wool spots (eg infarcts), haemorrhages,
venous beading. These are signs of retinal ischaemia. Non urgent Refer to a specialist.
Proliferative retinopathy: New vessels form. Needs urgent referral.
Maculopathy: (hard to see in early stages). Suspect if acuity. Prompt laser, intra vitreal steroids or anti-
angiogenic agents may be needed in macular oedema.

168. A 2m baby who has ambiguous genitalia presents to the ED with vomiting. Labs:
Na+=125mmol/L, K+=6mmol/L. What is the most likely dx?
a. Fragile X syndrome
b. Turners syndrome
c. Noonan syndrome
d. Congenital adrenal hyperplasia
Q. 1. What is the key?
Q. 2. What are the points in favour?
Ans. 1. The key is D. Congenital adrenal hyperplasia
Ans. 2. Points in favour: i) ambiguous genitalia ii) salt wasting manifested as hyponatremia and hyperkalemia
(In mild forms of salt-wasting adrenal hyperplasia, salt wasting may not become apparent until an illness
stresses the child). [here hyperkalaemia inspite of vomiting is indicating the disease].

Congenital adrenal hyperplasia (From secretion of androgenic hormones deficiency of 21-hydroxylase, 11-
hydroxylase, or 3--hydroxysteroid dehydrogenase).
Cortisol is inadequately produced, and the consequent rise in ACTH leads to adrenal hyperplasia and
overproduction of androgenic cortisol precursors. CAH is a leading cause of male pseudohermaphroditism.
Signs: Vomiting, dehydration, and ambiguous genitalia. Girls may be masculinized. Boys may seem normal at
birth, but have precocious puberty, or ambiguous genitalia (androgens in 17-hydroxylase deficiency), or
incomplete masculinization (hypospadias with cryptorchidism
from 3-hydroxysteroid dehydrogenase). Hyponatraemia and hyperkalaemia are common. Plasma 17-
hydroxyprogesterone Increased in 90%; Increased urinary 17-ketosteroids (not in 17-hydroxylase deficit).
Management of adrenocortical crisi: Urgent treatment is needed
0.9% saline IVI (35g Na+/day), glucose, fludrocortisone and hydrocortisone in neonate IV stat then
maintanance dose.
CAH can lead to addisons disease later in life due to delayed onset.

169. A 40yo man collapsed at home and died. The GPs report says he suffered from T2DM and
BMI=35. What is the most likely cause of death?
a. Myocardial Infarction
b. Diabetes mellitus
c. Heart failure
d. Pulmonary embolism
e. Renal failure
Q. 1. What is the key?
Q. 2. Why the patients death was unnoticed?

Ans. 1. The key is A. MI.


Ans. 2. In diabetics MI become painless when the patient develop autonomic neuropathy (till there is no
autonomic neuropathy diabetic patients will feel MI pain). In this case the disease was unnoticed as it was a
painless attack.
It is one of the complications of Diabetes.
Vascular disease Chief cause of death. MI is 4-fold commoner in DM and is more
likely to be silent. Stroke is twice as common. Women are at high risk.
Address other risk factorsdiet, smoking, hypertension. Suggest a statin (eg simvastatin 40mg
nocte) for all, even if no overt IHD, vascular disease or microalbuminuria. Fibrates
are useful for triglycerides and reduced HDL. Aspirin 75mg reduces vascular events
(if past stroke or MI) and is good as statin co-therapy (safe to use in diabetic retinopathy;
use in primary prevention is disappointing, at least at 100 mg/day.

170. A 38yo pt presented with tingling, numbness, paraesthesia, resp stridor and involuntary spasm of the
upper extremities. She has undergone surgery for thyroid carcinoma a week ago. What is the most likely dx?
a. Thyroid storm
b. Hyperparathyroidism
c. Unilateral recurrent laryngeal nerve injury
d. External laryngeal nerve injury
e. Hypocalcemia
Q. 1. What is the key?
Q. 2. What is the cause of this condition?
Q. 3. Why there is respiratory stridor?

Ans. 1. The key is E. Hypocalcaemia.


Ans. 2. Hypocalcaemia may be due to accidental parathyroid gland removal during thyroidectomy.
Ans. 3. Laryngospasm is a feature in hypocalcaemia which may cause stridor.
Thyroid storm causes thyrotoxicosis (inc heart rate, palpitations, weight loss, tremors, heat intolerance etc)
hyperparathyroidism causes hypercalcemia (bone pains, kidney stones, confusion, psychosis), nerve injuries
only explain the stridor.

Causes of hypocalcemia:
Treatment
Mild symptoms: give calcium 5mmol/6h PO, with daily plasma Ca2+ levels.
In chronic kidney disease:May require alfacalcidol
Severe symptoms: give 10mL of 10% calcium gluconate (2.25mmol) IV over 30min,
and repeat as necessary. If due to respiratory alkalosis, correct the alkalosis

171. A 50yo chronic smoker came to OPD with complaint of chronic productive cough, SOB and
wheeze. Labs: CBC=increase in PCV. CXR >6ribs seen above the diaphragm in midclavicular line. ABG=pO2
decreased. What is the most likely dx?
a. Interstitial lung disease
b. Wegeners granulomatosis
c. Ca bronchi
d. COPD
e. Amyloidosis
Q. 1. What is the key?
Q. 2. What are the points in favour?
Ans. 1. The key is D. COPD.
Ans. 2. Points in favour: i) Age 50 yrs ii) Chronic smoker iii) Chronic productive cough, SOB and Wheeze iv)
Raised PCV secondary to chronic hypoxaemia v) Low set diaphragm and widened horizontal ribs vi)
Hypoxaemia on ABG.

COPD:
COPD is a common progressive disorder characterized by airway obstruction (FEV1 <80% predicted; FEV1/FVC
<0.7. It includes emphysema and chronic bronchitis.
COPD is favoured by: Age of onset >35yrs Smoking (passive or active) or pollution related Chronic
dyspnoea Sputum production Minimal diurnal or day-to-day FEV1 variation.
Chronic bronchitis is defIned clinically as cough, sputum production on most days for 3 months of 2 successive
yrs. Symptoms improve if they stop smoking.
Emphysema is defined histologically as enlarged air spaces distal to terminal bronchioles, with destruction of
alveolar walls.

Pink puffers have inc alveolar ventilation, a near normal PaO2 and a normal or low PaCO2. They are breathless
but are not cyanosed. They may progress to type 1 respiratory failure.
Blue bloaters have decreased alveolar ventilation, with a low PaO2 and a high PaCO2. They are cyanosed but
not breathless and may go on to develop cor pulmonale. Their respiratory centres
are relatively insensitive to CO2 and they rely on hypoxic drive to maintain respiratory
effort

Symptoms Cough; sputum; dyspnoea; wheeze.


Signs Tachypnoea; use of accessory muscles of respiration; hyperinflation; cricosternal distance (<3cm);
decreased chest expansion; resonant or hyperresonant percussion note; quiet breath sounds (eg over bullae);
wheeze; cyanosis; cor pulmonale.
Complications Acute exacerbations infection; polycythaemia; respiratory failure; cor pulmonale (oedema;
raised JVP); pneumothorax (ruptured bullae); lung carcinoma.
Tests FBC: Raised PCV. CXR: Hyperinflation (>6 anterior ribs seen above diaphragm in midclavicular line); flat
hemidiaphragms; large central pulmonary arteries; peripheral
vascular markings; bullae. ECG: Right atrial and ventricular hypertrophy (cor pulmonale).
ABG: Reduced PaO2 hypercapnia. Lung function: obstructive + air trapping (FEV1 <80% of predicted. FEV1 :
FVC ratio <70%
TREATMENT OF CHRONIC STABLE PATIENTS.

172. A 44yo pt has sudden onset of breathlessness and stridor few minutes after extubation for
thyroidectomy. The pat had longstanding goiter for which he had the surgery. What is the most
likely dx?
a. Thyroid storm
b. Hematoma
c. Unilateral recurrent laryngeal nerve injury
d. External laryngeal nerve injury
e. Tracheomalacia
Ans. The key is tracheomalacia.

173. A 15yo boy presents with generalized edema. His urinalysis reveals protein +++, eGFR =110. What is the
most likely dx?
a. IgA nephropathy
b. Membranous nephropathy
c. Minimal change disease
d. PSGN
e. Lupus nephritis
Ans. The key is C. Minimal change disease. [Points in favour: i) Age 15 yrs ii) Generalized oedema iii) Protein in
urine +++ vi) Normal eGFR of 110 (Normal range- 90 to 120 mL/min)].

Most common cause of nephrotic syndrome in children is minimal change disease. There will be
hypoalbuminemia and peripheral edema too. Electron microscopy shows effacement of podocyte foot
processes.. MCD has albumin selective proteinuria. Treatment is with steroids.

174. A 72yo man is receiving chemotherapy for SCLC. He has his 4th tx 8 days ago. He has a cough with some
green sputum but feels well. Temp=37.6C. Chest exam = few coarse crepitations in the right base. HR=92bpm.
CBC: Hgb=12.5g/dL, WBC=1.1, Neutrophils=0.6, Plt=89. Sputum, urine and blood culture sent to microbiology.
What is the most appropriate management?
a. Broad spectrum antibiotics IV
b. Broad spectrum antibiotics PO
c. GCSF
d. Postpone tx until bacteriology results available
e. Reassure and send home
Q. 1. What is the key?
Q. 2. What is the Diagnosis?
Q. 3. What is the treatment of low WBC count?
Ans. 1. The key is A. Broad spectrum antibiotics IV
Ans. 2. The diagnosis is lower respiratory tract infection.
Ans. 3. GCSF subcutaneously. [it is the treatment of chemotherapy induced neutropenia]

Use of antibiotics in neutropenia:


Treat any known infection promptly.
If T >38C or T >37.5C on 2 occasions, >1h apart, or the patient is toxic, assume
septicaemia and start blind combination therapyeg piperacillintazobactam(+ vancomycin,if Gram +ve
organisms suspected or isolated, eg Hickman line sepsis). Check local preferences. Continue until afebrile for
72h or 5d course, and until neutrophils >0.5109/L. If fever persists despite antibiotics, think of CMV, fungi (eg
Candida; Aspergillus) and central line infection.
Consider treatment for Pneumocystis eg co-trimoxazole, ie trimethoprim
20mg/kg + sulfamethoxazole 100mg/kg/day PO/IV in 2 daily doses). Remember TB.

Avoid IM injections as they can lead to hematomas.

In this question the neutrophil count is 0.6 and temp is 37.6. So we should still start Iv antibiotics as they are
dangerously close the ranges given in the text above.
Granulocytes colony stimulating factors are used to produce neutrophils and is used in preventing sepsis but
the patient here is almost in sepsis! GCSF are mostly used in myeloproliferative disorders.

175. A 25yo woman with T1DM has delivered a baby weighing 4.5kg. Her uterus is well contracted. Choose the
single most likely predisposing factor for PPH from the options?
a. Atonic uterus
b. Cervical/vaginal trauma
c. Retained POC
d. Large placental site
e. Rupture uterus
Q. 1. What is the key?
Q. Reason for your answer.
Ans. 1. The key is B. Cervical/vaginal trauma
Ans. 2. The baby is a big baby. If patients uterus was not well contracted we would fear of atonic uterus! But
as uterus is well contracted it is not atonic uterus. Rather most likely cause is trauma dring delivery of this big
baby.

Primary PPH is the loss of greater than 500mL (definitions vary) in the first 24h after delivery
Causes: uterine atony (90%), genital tract trauma (7%), clotting disorders(3%)
Risks: Antenatal Previous PPH or retained placenta BMI>35kg/m2 Maternal Hb<8.5g/dl at onset of labour
Antepartum haemorrhage Multiparity 4+ Maternal age 35y+ Uterine malformation or fibroids A large
placental site (twins, severe rhesus disease, large baby) Low placenta, Overdistended uterus
(polyhydramnios, twins) Extravasated blood in the myometrium (abruption).
In labour Prolonged labour (1st, 2nd or 3rd stage) Induction or oxytocin use Precipitant labour
Operative birth or caesarean section. Book mothers with risk factors for obstetric unit delivery.
Treatment: Give oxytocin 5U slowly IV for atonic uterus.
Attach oxygen, Give IV fluids, maintain systolic >100mmHg, Transfuse blood.
Is the placenta delivered? If it is, is it complete? If not, explore the uterus. If the placenta is complete, put
the patient in the lithotomy position with adequate analgesia and good lighting. Check for and repair trauma.
If the placenta has not been delivered but has separated, attempt to deliver it by controlled cord traction
after rubbing up a uterine contraction. If this fails, ask an experienced obstetrician to remove it under general
anaesthesia.Beware renal shut down.

176. A 23yo lady presents with headache. Exam: photophobia and generalized rash that doesnt
blanch on pressure. What must be done immediately?
a. IV benzylpenicillin
b. Isolate pt
c. Gown and mask
d. Blood culture
Ans. The key is A. IV benzylpenicillin.

When to act: Headache, pyrexia, neck stiffness, altered mental state: if any 2 co-exist and not yet in hospital,
give benzylpenicillin 1.2g IM/IV before admitting.

Organisms: Meningococcus or pneumococcus. Less commonly Haemophilus influenzae;


Listeria monocytogenes. CMV, cryptococcus or TB if immunocompromised eg HIV +ve organ transplant,
malignancy.

Features
Early: Headache, leg pains, cold hands and feet, abnormal skin colour.
Later:
Meningism: neck stiffness, photophobia, Kernigs sign (pain + resistance on passive knee extension with hip
fully flexed).
Decreased Conscious level , coma.
Seizures (~20%) focal CNS signs (~20%) opisthotonus
Petechial rash (non-blanching ; may only be 1 or 2 spots, or none).
Signs of galloping sepsis: slow capillary refill; DIC; dec BP. inc T and pulse: inc or normal.

Management:
Start antibiotics immediately.
<55yrs: cefotaxime 2g/6h slow IV.
>55yrs: cefotaxime as above + ampicillin 2g IV/4h (for Listeria).
Prophylaxis: (discuss with public health/ID) Household contacts in droplet range. Give rifampin or
ciprofloxacin.

If in this question there was an option for IV cefotaxime that would have been the correct answer since here
the patient has presented in the hospital. Benzylpenicilline is given before admission or before taking the
patient to the hospital

177. A 4yo baby has generalized tonic-clonic seizure and fever of 39C. his mother informs you that this has
happened 3-4x before. What is the most likely dx?
a. Febrile convulsion
b. Absence seizures
c. Epilepsy
d. Partial complex seizure
Ans: The key is C. Epilepsy! Probably wrong key! Epilepsy doesnt occur with fever! Likely correct key is A.
Febrile convulsion.

FEBRILE CONVULSION is a single tonicclonic, symmetrical generalized seizure lasting <20min, occurring as T
rises rapidly in a febrile illnesstypically in a normally developing child (5yrs old).
Think of meningo-encephalitis, CNS lesion, epilepsy, trauma, hypoglycemia, dec Ca2+, or dec Mg2+ if:
Focal CNS signs or CNS abnormality Previous history of epilepsy The seizure lasts >15min There is >1
attack in 24h.
Examination: Find any infection; if any neck stiffness consider meningitis. :
Management: Put in recovery position; if fit is lasting >5min: lorazepam IV, buccal midazolam or diazepam PR.
Tepid sponging if hot; paracetamol syrup
Labs: Consider FBC, U&E, Ca2+, glucose, MSU, CXR, ENT swabs.
Avoid LP in the postictal period. If you suspect meningitis, then treat now.
Parental education: Allay fear (a child is not dying during a fit). For the 30% having
recurrences, teach carers to use buccal midazolam or rectal diazepam 0.5mg/ kg
Further prevention: Diazepam PR during fevers has a role; other anticonvulsants are never needed. Explain
that all fevers (eg vaccination-associated) should prompt oral antipyretics.
Prognosis: In typical febrile convulsions there is no progress to epilepsy in 97%.
178. A middle aged Asian presents with episodes of fever with rigors and chills for last 1y. Blood film: ring form
plasmodium with schaffners dots in RBCs. What is the drug to eradicate this infection?
a. Doxycycline
b. Mefloquine
c. Proguanil
d. Quinine
e. Artesonate
Q. 1. What is the key?
Q. 2. What does Shuffners dot in RBC indicate?
Ans. 1. The key is B. Mefloquine.
Ans. 2. Shuffners dot indicates, it is plasmodium ovale or plasmodium vivax infestation.

MALARIA:
Plasmodium vivax and ovale: cyclical fever every 48 hours.
P.malariae: Cyclical fever every 72 hours. Can cause glomerulonephritis. Rarely fatal.
P. falciparum: fevr 36-48hrs cyclical. Fulminant disease.
Presentation: 3 phases:
1 Shivering (1h): I feel so cold.
2 Hot stage (26h): T 41C, flushed, dry skin; nausea/vomiting; headache.
3 Sweats (~3h) as T falls
Also malaise, fatigue, anorexia, myalgias...
Signs: Anaemia, jaundice, and hepatosplenomegaly. No rash or lymphadenopathy
Protective factors: G6PD lack; sickle- cell trait; melanesian ovalocytosis;
Complications: Hemolytic anemia can occur.
5 grim signs: 1 dec Consciousness/coma (cerebral malaria) 2 Convulsions 3 Coexisting chronic illness 4 Acidosis
(eg esp bad if HCO3 <15mmol/L) 5 Renal failure
Diagnosis: Serial thin & thick blood films.
P. vivax ring forms partly hidden by Schuffners dots. P. malariae: ring and band forms
P. falciparum: sausage-like gametocytes in RBC ghosts.

Treatment: If the patient has taken prophylaxis, dont use the same drug for treatment.
If species unknown or mixed infection, treat as P. falciparum. Nearly all P. falciparum is resistant to
chloroquine and in many areas also to Fansidar (pyrimethamine + sulfadoxine).
Chloroquine is 1st choice for benign malarias in most parts of the world. Never rely on chloroquine if used
singly for prophylaxis.
Treating uncomplicated P. ovale, P. vivax, & P. malariae: Chloroquine base: 10mg/
kg, then 5mg/kg at 6h, 24h and 48h. In resistant cases, try Malarone (atovaquone and proguanil) ,quinine, or
Riamet. Primaquine dose in P. vivax: 500g/kg (max.30mg) daily for 14d; P. ovale: 250g/kg (max. 15mg)
daily for 14dgiven after chloroquine to treat liver stage and prevent relapse. Screen for G6PD deficiency
first. CI: pregnancy. P. malariae does not need primaquine.
Treating uncomplicated P.falciparum malaria:
Artemether-lumefantrineif >35kg: 4 tabs stat, then 4 tablets at 8, 24, 36, 48 and 60h. Artesunate-
amodiaquine; if a fixed combination pill is available.
Dihydroartemisinin-naphthoquine Dihydroartemisinin piperaquine. Atovaquone-proguanil. can be used.
In pregnancy: Artemisinins are OK in children and pregnancy from 13 weeks;
(use quinine + clindamycin in 1st trimester).
In addition give symptomatic treatment for fever, blood transfusion if required.
Prophylaxis:
If little/no chloroquine resistance: Proguanil 200mg/24h+chloroquine base 300mg/wk.
If chloroquine-resistant P. falciparum: Mefloquine 250mg/wk (18d before to 4wks after trip) or doxycycline
100mg/d (1d before to 4wks after) or atovaquone 250mg + proguanil 100mg (Malarone) 1 tab/d (1d before
travel to 7d after).

179. A 35yo woman had an uneventful lap chole 18h ago. She has a pulse=108bpm, temp 37.8C. There are
signs of reduced air entry at the right base but the CXR doesnt show an obvious abnormality. What is the
most appropriate management strategy?
a. Cefuroxime PO
b. Ceftriaxone IV
c. Chlorpheniramine PO
d. Chest physiotherapy
e. Reassure
Q. 1. What is the key?
Q. 2. What is the diagnosis?
Ans. 1. The key is D. Chest physiotherapy.
Ans. 2. Atelactasis.
Best visible on CT scan and not on chest xray. Mostly occurs as a complication of anaesthesia.
Arrange physiotherapy and antibiotics.

180. A 20yo pop star singer complains of inability to raise the pitch of her voice. She attributes this to the
thyroid surgery she underwent a few months back. What is the most likely dx?
a. Thyroid storm
b. Bilateral recurrent laryngeal nerve injury
c. Unilateral recurrent laryngeal nerve injury
d. External laryngeal nerve injury
e. Thyroid cyst
Ans. The key is D. External laryngeal nerve injury.

Complications of thyroid surgery:


1. Bleeding, which may cause tracheal compression.
2. Recurrent laryngeal nerve injury:
Innervates all of the intrinsic muscles of the larynx, except the cricothyroid muscle.
Patients with unilateral vocal fold paralysis present with postoperative hoarseness. Presentation is
often subacute and voice changes may not present for days or weeks. Unilateral paralysis may resolve
spontaneously.
Bilateral vocal fold paralysis may occur following a total thyroidectomy and usually presents
immediately after extubation. Both vocal folds remain in the paramedian position, causing partial
airway obstruction.
Superior (external) laryngeal nerve injury:
o The external branch provides motor function to the cricothyroid muscle.
o Trauma to the nerve results in an inability to lengthen a vocal fold and thus to create a
higher-pitched sound.
o The external branch is probably the most commonly injured nerve in thyroid surgery.
o Most patients do not notice any change but the problem may be career-ending for a
professional singer.
Hypoparathyroidism: the resulting hypocalcaemia may be permanent but is usually transient. The
cause of transient hypocalcaemia postoperatively is not clearly understood.
Thyrotoxic storm: is an unusual complication of surgery but is potentially lethal.
Infection: occurs in 1-2% of all cases. Peri-operative antibiotics are not recommended for thyroid
surgery.
Hypothyroidism.
Damage to the sympathetic trunk may occur but is rare.

181. A 28yo woman at 39wk gestation is in labor. She develops abdominal pain and HR=125bpm,
BP=100/42mmHg, temp=37.2C and saturation=99%. Exam: lower abdomen is exquisitely tender. CTG=prv
normal, now showing reduced variability and late deceleration develops with slow recovery. She has had 1 prv
LSCS for a breech baby. Choose the most appropriate CS
complication for this lady?
a. Endometritis
b. UTI
c. Urinary tract injury
d. Pleurisy
e. Uterine rupture
Ans. The key is E. Uterine rupture.

Uterine rupture: Its an obstetrical emergency


Causes: ~70% of UK ruptures are due to dehiscence of caesarean section scars.
Other risk factors: Obstructed labour in the multiparous, especially if oxytocin is used Previous
cervical surgery High forceps delivery Internal version Breech extraction. Rupture is usually during the
third trimester or in labour.

Vaginal birth after caesarean (trial of scar): Vaginal birth will be successful in 7276%. Endometritis, need for
blood transfusion, uterine rupture and perinatal death are commoner than repeated elective C section.

Signs and symptoms Rupture is usually in labour. In a few (usually a caesarean scar dehiscence) rupture
precedes labour. Pain is variable, some only having slight pain and tenderness over the uterus. In others pain
is severe.
Vaginal bleeding is variable and may be slight (bleeding is intraperitoneal). Unexplained
maternal tachycardia, sudden maternal shock, cessation of contractions, disappearance of the presenting part
from the pelvis, and fetal distress are other presentations. Postpartum indicators of rupture: continuous PPH
with a well-contracted uterus; if bleeding continues postpartum after cervical repair; and whenever shock is
present.

Management If suspected in labour, perform laparotomy, deliver the baby by caesarean section, and explore
the uterus. If rupture is small Repair or if vagina or cervix are involved in the tear hysterectomy may be
needed.

182. An 8m infant presented with FTT and constipation. Exam: large tongue and fam hx of prolonged neonatal
jaundice. What is the most likely dx?
a. Downs syndrome
b. Fragile X syndrome
c. Praderwilli syndrome
d. DiGeorge syndrome
e. Congenital hypothyroidism
Q. 1. What is the key?
Q. 2. What are the points in favour?
Congenital Hypothyroidism: Thyroid hormone is necessary for growth and neurological development.
Signs: May be none at birthor prolonged neonatal jaundice, widely opened posterior fontanelle, poor
feeding, hypotonia, and dry skin are common. Inactivity, sleepiness, slow feeding, little crying, and
constipation may occur. Look for coarse dry hair, a flat nasal
bridge, a protruding tongue, hypotonia, umbilical hernia, slowly relaxing reflxes, pulse, and poor growth and
mental development if it has not been picked up. Other later signs: dec IQ, delayed puberty (occasionally
precocious), short stature, delayed dentition.
Universal neonatal screening: Cord blood or filter paper spots (at ~7 days, from heel prick) allow early
diagnosis (theGuthrie card).
Tests: Decreased T4, Increased TSH (but undetectable in secondary hypothyroidism), Decreased I131 uptake,
dec Hb. Bone age is less than chronological age. As it is unwise to X-ray the whole skeleton, the left wrist and
hand are most commonly used.
Treatment: Levothyroxine (LT4): Start neonates with ~15g/kg/day; adjust by 5g/kg every 2
weeks to a typical dose of 2050g/day. Avoid high TSH levels.

Ans.1. The key is E. Congenital hypothyroidism.


Ans. 2. Points in favour:i) FTT ii) constipation iii) macroglossia iv) prolonged neonatal jaundice.
183. A 3m infant has presented with recurrent infections. He has abnormal facies and CXR shows absent
thymic shadow. What is the most likely dx?
a. Downs syndrome
b. Fragile X syndrome
c. DiGeorge syndrome
d. Marfans syndrome

Q. 1. What is the key?


Q. 2. What are the points in favour?
Ans. 1. The key is C. DiGeorge syndrome.
Ans. 2. Points in favour: i) Early age of onset ii) abnormal facies iii) absent thymic shadow on Chest X-ray iii)
history of recurrent infection [in newborne can be recognized by convulsions from hypocalcaemia due to
malfunctioning parathyroid glands and low level of parathyroid hormones].

DiGeorges syndrome A deletion of chromosome 22q11.2 causes absent thymus,


fits, small parathyroids ( decreased Ca2+), anaemia, lymphopenia, dec growth hormone,
dec T-cell-immunity. It is related to velo-cardiofacial syndrome: characteristic
face, multiple anomalies, eg cleft palate, heart defects, cognitive defects

Management:
Cardiac defects are the usual focus of clinical management.
Hypocalcaemia should be screened for by checking calcium levels three-monthly in infancy and then annually.
Low calcium and high phosphate levels should prompt further testing of parathyroid hormone and vitamin D
levels.
All patients should have baseline immunological testing and annual blood count
Cleft palates may be submucous. In particular, they should be sought if feeding difficulties are encountered in
the neonatal period.
Gastro-oesophageal reflux needs to be managed appropriately with feed thickeners and anti-reflux
medication. Nasogastric tube feeding and occasionally gastrostomy may be needed to deal with feeding
issues.

184. A 30yo man presents with deep penetrating knife wound. He said he had TT when he left school. What
will you do for him now?
a. Human Ig only
b. Human Ig and TT
c. Full course of tetanus vaccine only
d. Human Ig and full course of tetanus vaccine
e. Antibiotic
Ans. The key is B. Human Ig and TT.

185. A 32yo previously healthy woman has developed pain and swelling of both knees and ankles with nodular
rash over her shins. As part of the inv a CXR has been performed. What is the single
most likely CXR appearance?
a. Apical granuloma
b. Bilateral hilar lymphadenopathy
c. Lobar consolidation
d. Pleural effusion
e. Reticular shadowing in the bases
Q. 1. What is the key?
Q. 2. What is the name of this condition? What are the points in favour?
Ans. 1. The key is B. bilateral hilar lymphadenopathy.
Ans. 2. The name is Lofgrens syndrome. It is the triad of i) erythema nodosum ii) bilateral hilar
lymphadenopathy and iii) arthralgia.

Apical granuloma: apical granuloma modified granulation tissue containing elements of chronic inflammation
located adjacent to the root apex of a toothwith infected necrotic pulp.

Lobar consolidation: pneumonia

Sarcoidosis:
Presentation: Lungs are in involved in more than 90% cases of sarcoidosis. There is interstitial lung disease.
The painful skin lesion is erythema nodosum. Also look for Lupus pernio (chronic raised hardened, often
purple lesion) may be seen on the face.
Lofgren syndrome is often a part of sarcoidosis. The triad is i) Erythema nodosum ii) Bilateral hilar
lymphadenopathy iii) Arthralgia
Sarcoidosis is a multisystem disease and can involve any system/organ
Tests: ESR is often raised. Serum ACE enzyme levels are raised in 60% of times
Plain CXR may show bilateral hilar or paratracheal lymphadenopathy. High resolution CT should be done.
There will be restricitve pattern of disease on pulmonary function tests.
Transbronchial biopsy can demonstrate the presence of non-caseating granulomata, giving a more accurate
diagnosis
Bronchioalveolar lavage may also be done

186. A neonates CXR shows double bubble sign. Exam: low set ears, flat occiput. What is the most likely dx?
a. Downs syndrome
b. Fragile X syndrome
c. Turners syndrome
d. DiGeorge syndrome
Q. 1. What is the key?
Q. 2. What double bubble sign indicate?
Ans. 1. The key is A. Downs syndrome.
Ans. 2. Double bubble sign indicate duodenal atresia.

Downs syndrome:
Causes:
Non-disjunction of chromosome >88%
Mosaicism <8%
The robertsonian translocation trisomy 21 is the cause in 4% of Downs syndrome

Presentation:
simian palmar crease, hypotonia, flat face/round head, protruding tongue, broad hands, upward slanted
palpebral fissures and epicanthic folds, speckled irises (Brushfield spots); mental and growth retardation;
pelvic dysplasia, cardiac malformations, short, broad hands, hypoplasia of middle phalanx of (incurving) 5th
finger, intestinal atresia and high arched palate.
.
Associated problems: Duodenal atresia; VSD; patent ductus; AVSD (foramen primum defects, and, later, a low
IQ and a small stature.

Pr enatal diagnosis:
The combined test: combines nuchal translucency (NT) + free -human chorionic gonadotrophin (HCG) +
pregnancy associated plasma protein (PrAP-A or PAPP-A) + the womans age. Used between 10 weeks 3 days
and 13 weeks 6 days. It achieves detection rates of 95% of all aneuploides, 86% trisomy-21, and 100% of
trisomy-18 and trisomy-13

The quadruple test combines maternal -fetoprotein (AFP) + unconjugated estriol + free HCG or total HCG +
inhibin-A + the womans age in the 2nd trimester. Use between 15 weeks + 0 days and 20 weeks + 0 days so
useful for women presenting in the 2nd trimester

The integrated test: It involves NT + PrAP-A in the 1st trimester + the quadruple test in the 2nd trimester. Do
not use 2nd trimester tests for triplets

187. A 19yo boy complains of itching on the site of insect bite. What is the single most appropriate
management?
a. Penicillin oral
b. Doxycycline oral
c. Oral antihistamine
d. Oral ciprofloxacin
e. Reassurance
Ans. The key is C. Oral antihistamine.

This is a case of simple physical utricaria follwing an insect bite which causes itching. For which oral
antihistamine is most suitable.
188. A man presents with scrotal swelling, the swelling is cystic and is non-tender. It is located in the upper
pole of the posterior part of the testis. What is the most likely dx?
a. Epididymal cyst
b. Testicular ca
c. Hydrocele
d. Teratoma
e. Testicular torsion
Ans. The key is A. Epididymal cyst. [the location of upper pole of the posterior part of testis is the common site
for epididymal pathology].

Diagnosing scrotal masses


Cannot get above inguinoscrotal hernia or hydrocele extending proximally
Separate and cystic epididymal cyst
Separate and solid epididymitis/varicocele
Testicular and cystic hydrocele
Testicular and solidtumour, haematocele, granuloma, orchitis, gumma. USS may help.

Epididymal cysts usually develop in adulthood and contain clear or milky (spermatocele)
fluid. They lie above and behind the testis. Remove if symptomatic

189. A young footballer has collapsed during a game. During initial evaluation: RR=14/min,
pulse=88bpm, BP=110/70mmHg. He seems to be sweating and muttering some
incomprehensible words. What is the most imp next step?
a. CT
b. MRI
c. Blood sugar
d. Body temp
e. IV fluids
Ans. The key is C. Blood sugar.

The collapse could be because of hypoglycemia. So we should BSR of the patient first before taking him for CT
scan or any other investigation as it is easy and fast and could give us a cause of collapse that is quickly
reversible.
Even in managing a patient with COMA or seizures. after ABC first thing to do is to check BSR.
The patient here most probably had an attack of SYNCOPE. If in the question there was an option for ECG then
that would have been the right answer.

Syncope:
Causes:
Neurally mediated syncope (NMS) - also called reflex syncope of which vasovagal is the most important cause
of syncope.
Orthostatic hypotension
Cardiac arrythmias
Structural cardiac or cardiopulmonary disease
Substance abuse
Psychogenic

Presentation:
In some forms of syncope there may be a prodromal period with light-headedness, nausea,
sweating, weakness or visual disturbances, but loss of consciousness often occurs without
warning.
Syncope is usually brief, with complete loss of consciousness in reflex syncope not lasting more
than 20 seconds (but may occasionally be up to several minutes). Recovery from syncope is usually
associated with almost immediate restoration of appropriate behaviour and orientation, but there
may be marked fatigue. Retrograde amnesia may occur, especially in older individuals.

Investigations: The initial evaluation of a patient presenting with transient loss of consciousness consists of
careful history, physical examination, including orthostatic blood pressure measurements, and an ECG. Further
investigations :

FBC: acute anaemia will cause syncope, but patients adapt in cases of chronic anaemia.

Fasting blood glucose (hypoglycaemia).

Management: Reassure and if an underlying cause is found treat it.

190. A 45yo waitress complains of pelvic pain which worsens pre-menstrually and on standing and walking.
She also complains of post-coital ache. Select the most likely cause leading to her
symptoms?
a. PID
b. Endometritiosis
c. Pelvic congestion syndrome
d. Adenomyosis
e. Premature ovarian failure
Ans. The key is C. Pelvic congestion syndrome.

PID mostly presents with pelvic pain, fever, spasm of lower abdominal muscles and cervicitis with profuse,
purulent, or bloody vaginal discharge. Heavy menstrual loss suggests endometritis.

Endometriosis can present as given in this question but it will have menstrual abnormalities too and the pain
will not be related to any specific activity.

Pelvic Congestion Syndrome:

Condition is characterised by the presence of dilated pelvic veins associated with stasis
Aetiology
Considered that ovarian dysfunction is responsible for the excessive production of local oestrogen, causing
dilatation and stasis in the pelvic veins, which leads to pelvic pain

Presentation:
women with this condition commonly complain of a dull, aching pain, exacerbated by activities that increase
intra-abdominal pressure; the pain is relieved by lying down.
other clinical features may also be deep dyspareunia, congestive dysmenorrhoea and post-coital ache
condition usually occurs in the reproductive age group, with a mean age of 33 years.

Examination may reveal tenderness that is maximal over the ovaries. Vaginal and cervical examination may
reveal an apparent blue colouration due to congestion of the pelvic veins. The patient may also have varicose
veins of the legs

Investigations for endometriosis and pelvic inflammatory disease must be instigated


Venography is still considered the definitive radiological investigation for women with pelvic congestion
syndrome
Radiological features: dilated uterine and ovarian veins with reduced venous clearance of contrast medium

Management
Medical treatment options include:
Medroxyprogesterone acetate (MPA) suppresses ovarian function and therefore reduces pelvic congestion
and pain, however benefit was not sustained after discontinuing treatment (1)

Gonadorelin analogues goserelin 3.6 mg per month given for 6 months provided an alleviation of symptoms,
an improvement in sexual functioning and a reduction of anxiety and depressive states in women with pelvic
congestion

Other possible treatment options include:


Bilateral ovarian vein ligation
Hysterectomy plus bilateral salpingo-oophrectomy (with post-operative hormone replacement therapy)

191. A 37yo female had a fall with outstretched hand, presented with dinner fork deformity and
tenderness over the right arm. What is the single most associated nerve injury?
a. Axillary nerve
b. Radial nerve
c. Musculocutaneous nerve
d. Median nerve
e. Ulnar nerve
Ans. The key is D. Median nerve. [Median nerve is the nerve injured in Colles fracture].

Distal radial fractures Colles type Fracture: (common in osteoporotic post-menopausal women who fall on an
outstretched hand).
There is dorsal angulation and displacement producing a dinner-fork wrist deformity (the fingers are the
prongs). Avulsion of the ulna styloid process may also occur.
Treatment:
For reduction, Biers block method (= IV regional anaesthesia) is best.

Complications: Median nerve symptoms (should resolve after good reduction); ruptured tendons
(esp. extensor pollicis longus); malunion & nonunion.

192. A mother comes with her 15m child. Which of the following will bother you?
a. Shies away from strangers
b. Can walk but not run
c. Vocabulary consists of only 2 meaningless words
d. She cant make a sentence
e. None
Q. 1. What is the key?
Q. 2. How many words is told clearly by a 15 month old child?
Ans. 1. The key is C. Vocabulary consists of only 2 meaningless words.
Ans. 2. At 15 months the child can clearly say 5 words and his first meaningful clear word he says at 12
months.

Developmental Milestones.

193. A 35yo lady who has been using IUCD for one year now complains of pelvic pain and heavy
painful periods. Select the most likely cause leading to her symptoms?
a. PID
b. Endometriosis
c. Adenomyosis
d. Fibroids
e. Asherman syndrome
Q. 1. What is the key?
Q. 2. What points favour your diagnosis?
Ans. 1. The key is A. PID.
Ans. 2. The given picture may have D/D of PID or fibroid. As IUCD is a risk factor for PID, it is the most likely
diagnosis of given picture.
Fibroids are not related to IUCD. Also in fibroids there will be findings on examination such as mass in lower
abdomen.
194. The dx cells of Hodgkin disease are:
a. T-cells
b. R-S cells
c. B-cells
d. Macrophages
e. Auer rods
Ans. The key is B. R-S cells. [Diagnostic cell in Hodgkins disease is Reed-Sternberg cells].

Hodgkins lymphoma: Lymphomas are disorders caused by malignant proliferations of lymphocytes. These
accumulate in the lymph nodes causing lymphadenopathy.In Hodgkins lymphoma,characteristic cells with
mirror-image nuclei are found, called ReedSternberg cells

Cause: 2 peaks of incidence: young adults and elderly. EBV has a role in causing it.

Symptoms Often presents with enlarged, painless, non-tender, rubbery superficial lymph nodes, typically
cervical (6070%), also axillary or inguinal nodes. Nodes may become matted. 25% have constitutional
symptoms.
Mediastinal lymph node involvement can cause features due to mass effect, eg bronchial or SVC obstruction
or direct extension, eg causing pleural effusions.

Signs Lymph node enlargement. Also, cachexia, anaemia, spleno- or hepatomegaly.

Types:
Tests Tissue diagnosis: Lymph node excision biopsy if possible. Inc ESR or dec Hb show worse prognosis.
Staging is by Ann-Arbor system.
Treatment: Depends on chemotherapy and radiotherapy.

195. A 16yo girl is admitted after taking a paracetamol OD 4 h ago. She has consumed large amounts of
alcohol. Her plasma paracetamol conc is just below the conc that would suggest tx. What should be the tx
option for her?
a. Refer to psychiatry ward
b. Refer to medical ward
c. N-acetylcystine
d. Serum plasma paracetamol
e. No further investigation

Ans. The key is A. Refer to psychiatry ward.

Chronic alcohol consumption is an inducer of P-450 enzyme system while acute alcohol consumption is
inhibitor. Since this lady has consumed large amounts of alcohol recently (acute) the risk of fatal effects of
paracetamol poisoning will be reduced. And all such patients should be referred to the psych ward.

PARACETAMOL POISONING: >150mg/kg or 12 total


PRESENTATION: Hepatic damage shown by deranged LFTs occurs after 24hrs. Patients may develop encaph,
hypoglycemia, ARF
INVESTIGATIONS: Paracetamol levels: 4hrs post ingestion, if time is >4hr or staggered overdose

Any alcohol taken (acute alcohol ingestion will inhibit liver enzymes and may reduce the production of the
toxin NAPQI, whereas chronic alcoholism may increase it)

MANAGEMENT:
If presentation is within the first hour give activated charcoal
All patients who have a timed plasma paracetamol level plotted on or above the line drawn between
100 mg/L at 4 hours and 15 mg/L at 15 hours after ingestion, should receive acetylcysteine.
If time unknown (even in staggered dose) give N-Acetyl cysteine without delay
NAC most effective in the first 8 hrs.
NAC can be given during pregnancy
Beware if the patient is on any P450 enzyme inducer medicines as they increase the toxicity
Refer to ICU if there is fulminant liver failure - those treated with N-acetylcysteine (NAC) to the medical
team and all para-suicides to the psychiatric team.

196. A 64yo woman has been on HRT for 9yrs. She had regular withdrawal bleeds until 3 yrs ago and since
then has been taking a no bleed prep. Recently she noticed a brown vaginal discharge.
Choose the single most appropriate initial inv?
a. Cervical smear
b. High vaginal swab
c. TFT
d. Transvaginal US
Q. 1. What is the key?
Q. 2. Why this test will be done?
Ans. 1. The key is D. Transvaginal US.
Ans. 2. To determine the endometrial thickness!
In a postmenopausal woman with vaginal bleeding, the risk of cancer is approximately 7.3% if her
endometrium is thick (> 5 mm) and < 0.07% if her endometrium is thin ( 5 mm).
In postmenopausal women without vaginal bleeding, the risk of cancer is approximately 6.7% if the
endometrium is thick (> 11 mm) and 0.002% if the endometrium is thin ( 11 mm).
Investigate postmenopausal vaginal bleeding promptly as the cause may be endometrial cancer.

Endometrial Carcinoma:
Most are adenocarcinomas, and are related to excessive exposure to oestrogen unopposed
by progesterone.

Risk Factors: Obesity Unopposed oestrogen Functioning ovarian tumour Family History of breast, ovary,
or colon cancer Nulliparity Late menopause Diabetes mellitus Tamoxifen, tibolone Pelvic irradiation
Polycystic ovaries.

Presentation This is usually as postmenopausal bleeding (PMB). It is initially scanty and occasional ( watery
discharge). Then bleeding gets heavy and frequent. Premenopausal women may have intermenstrual
bleeding, but 30% have only menorrhagia.

Diagnosis: TVUS scan is an appropriate first-line procedure to identify which women with PMB are at higher
risk of endometrial cancer. Endometrial thickness of >5mm warrants biopsy. The definitive diagnosis is made
by uterine sampling or curettage. All parts of the uterine cavity must be sampled; send all material for
histology. Hysteroscopy enables visualization of abnormal endometrium to improve accuracy of sampling.

Staging The tumour is


Stage I in the body of the uterus only.
Stage II in the body and cervix only.
Stage. III advancing beyond the uterus, but not beyond the pelvis.
Stage: IV extending outside the pelvis (eg to bowel and bladder).

Treatment: Stages I and II may be cured by total hysterectomy with bilateral salpingo-
oophorectomy and/or radiotherapy if unfit for surgery. In advanced diseases consider radiotherapy and/or
high dose progesterone which shrinks the tumor.

197. A young girl complains of episodic headaches preceded by fortification spectra. Each episode
last for 2-3 days. During headache pt prefers quiet, dark room. What is the tx of choice for acute
stage?
a. Paracetamol
b. Aspirin
c. Sumatriptan
d. Gabapentin
e. Cafergot
Ans. The key is B. Aspirin. [OHCM, 9th Eition, page-462 where NSAIDS like ketoprophen or dispersible aspirin
900 mg/6 hr are recommended as treatment in acute stage].
Migraine:

Symptoms Classically: Visual or other aura lasting 1530min followed within 1h by unilateral, throbbing
headache. Or: Isolated aura without headache; Episodic severe headaches without aura, often
premenstrual, usually unilateral, with nausea, vomiting photophobia/phonophobia (common migraine).
There may be allodyniaall stimuli produce pain: I cant brush my hair, wear earrings or glasses, or shave, its
so painful.
Signs: None.
Aura: Visual: chaotic cascading, distorting, melting and jumbling of lines, dots, or zigzags, scotomata or
hemianopia; Somatosensory: paresthesia spreading from fingers to face; Motor: dysarthria and ataxia
(basilar migraine), ophthalmoplegia, or hemiparesis; Speech: (8% of auras) dysphasia or paraphasia, eg
phoneme substitution.
Criteria for diagnosis if no aura 5 headaches lasting 472h + nausea/vomiting (or photo/phonophobia) + any
2 of: Unilateral Pulsating Impairs (or worsened by) routine activity.
Partial triggers Seen in 50%: CHOCOLATE or: Chocolate, Hangovers, Orgasms,
Cheese, Oral contraceptives, Lie-ins, Alcohol, Tumult, or Exercise.

Treatment:
Acute:
Step one: simple analgesic with or without anti-emetic. In patients who have tried step 1 and didnt respond
and in patients with moderate-to-severe migraine, move to step three.

Use early in the attack to avoid gastric stasis.


Use soluble aspirin 600-900 mg (not in children) or ibuprofen 400-600 mg. Use prochlorperazine 3
mg buccal tablet if there is nausea and vomiting.

Step two: rectal analgesia and rectal anti-emetic.

Step three: specific anti-migraine drugs Triptans (5HT1-receptor agonists) or ergotamine (the use of
ergotamine is limited by absorption problems and side-effects such as nausea, vomiting and abdominal pain

Triptans are Contra indicated if IHD, coronary spasm, uncontrolled BP, recent lithium, SSRIS,
or ergot use.

Prevention Remove triggers; ensure analgesic rebound headache is not complicating


matters. Drugs eg if frequency equal or >2 a month or not responding to drugs
1st-line: Propranolol, amitriptyline (SE: drowsiness, dry mouth, vision), topiramate (SE: memory) or Ca2+
channel blockers.
2nd-line: Valproate, pizotifen (effective, but unacceptable weight gain in some), gabapentin, pregabalin, ACE-i,
NSAIDS

198. A 60yo pt recovering from a surgery for toxic goiter is found to be hypotensive, cyanosed in the the RR.
Exam: tense neck. There is blood oozing from the drain. What is the most likely dx?
a. Thyroid storm
b. Reactionary hemorrhage
c. Secondary hemorrhage
d. Primary hemorrhage
e. Tracheomalacia
Ans. The key is B. Reactionary haemorrhage. [in the recovery room, cyanosis, hypotension, tense neck, oozing
of blood from drain; all these goes in favour of reactionary haemorrhage].
Primary Haemorrhage: Haemorrhage occurring at the time of Injury/Trauma/Surgery

Reactionary Haemorrhage: Haemorrhage occurring within first 24 hrs following Trauma/Surgery

The causes Reactionary Haemorrhage:

1) Slipping away of Ligatures


2) Dislodgement of Clots
3) Cessation of Reflex vasospasm
4) Normalization of Blood Pressure

Secondary Haemorrhage: Haemorrhage occurring after 7 -14 days after Trauma/Surgery.

The attributed cause is infection and sloughing away of the blood vessels.

The symptoms tell us that the patient is in hypovolemic shock one of the causes of which is haemorrhage
and Since here the patient is still in the recovery room this type of haemorrhage is reactionary.

199. A 33yo man is hit by a car. He loses consciousness but is found to be fine by the paramedics.
When awaiting doctors review in the ED he suddenly becomes comatose. What is the most likely
dx?
a. SAH
b. Subdural hemorrhage
c. Intracerebral hemorrhage
d. Extradural hemorrhage
Ans. The key is D. Extradural haemorrhage. [Age 33 (younger age), considerable head trauma, and lucid
interval (present in bothe extradural and subdural) are the points in favour].

Lucid interval can occur both ins Subdural and extra dural haemorrhage. The difference is that the
presentation of the lucid interval (that is the gain of consciousness and the LOC) in extra dural occurs within
hours or 1-2 days while in subdural it can take days to weeks upto 9 months.

Epidural (extradural) haemorrhage:


Suspect this if, after head injury, conscious level falls or is slow to improve, or
there is a lucid interval. Extradural bleeds are often due to a fractured temporal or
parietal bone causing laceration of the middle meningeal artery and vein, typically
after trauma to a temple just lateral to the eye. Any tear in a dural venous sinus will
also result in an extradural bleed. Blood accumulates between bone and dura.

Presentation: Increasingly severe headache, vomiting, confusion, and fits follow, hemiparesis with brisk
reflexes and an upgoing plantar. If bleeding continues, the ipsilateral pupil dilates, coma deepens, bilateral
limb weakness develops, and breathing becomes deep and irregular (brainstem compression). Death follows a
period of coma and is due to respiratory arrest. Bradycardia and raised blood pressure are late signs.

Tests CT scan shows a haematoma (often biconvex/lens-shaped; the blood forms a more rounded shape
compared with the sickle-shaped subdural haematoma. Skull X-ray may be normal or show fracture lines
crossing the course of the middle meningeal vessels. Skull fracture after trauma greatly increases risk of an
extradural haemorrhage and should lead to prompt CT. Lumbar puncture is contraindicated.
Management Stabilize and transfer urgently for clot evacuation ligation of the bleeding
vessel. Care of the airway in an unconscious patient and measures to decrease ICP often
require intubation and ventilation (+ mannitol IVI
Prognosis Excellent if diagnosis and operation early. Poor if coma, pupil abnormalities, or decerebrate rigidity
are present pre-op.

200. A 77yo male presents with hx of enuresis and change in behavior. Exam: waddling gait. What is the most
likely dx?
a. Subdural hemorrhage
b. Brain tumor
c. Normal pressure hydrocephalus
d. Psychotic depression
Ans. The key is C. Normal pressure hydrocephalus. [age (usually occurs in 60s or 70s), loss of bladder control
(enuresis), waddling gait and behavior change are all features of normal pressure hydrocephalus].

Normal Pressure Hydrocephalus:


describes the condition of ventricular dilatation in the absence of raised CSF pressure on lumbar puncture,
characterised by a triad of gait abnormality, urinary (usually) incontinence and dementia.

Cause: Idiopathic or it may be secondary to:

Subarachnoid haemorrhage. Meningitis. Head injury. Central nervous system (CNS) tumour.

Symptoms: The (gradually progressive) classic triad of symptoms is:

Gait disturbance - this is due to distortion of the corona radiata by the dilated ventricles.
Movements are slow, broad-based and shuffling. The clinical impression is thus one of Parkinson's
disease, except that rigidity and tremor are less marked and there is no response to
carbidopa/levodopa. Gait disturbance is referred to as gait apraxia.
Sphincter disturbance - this is also due to involvement of the sacral nerve supply. Urinary
incontinence is predominant although bowel incontinence can also occur.
Dementia - this is due to distortion of the periventricular limbic system. The prominent features
are memory loss, inattention, inertia and bradyphrenia (slowness of thought). The dementia
progresses less rapidly than that seen with Alzheimer's disease.

Signs
Pyramidal tract signs may be present.
Reflexes may be brisk.
Papilloedema is absent (but there has been found to be an association with glaucoma, so
glaucomatous optic disc changes may be noticed).

Investigations
Neuroimaging - MRI or CT scanning may show ventricular enlargement out of proportion to sulcal
atrophy and periventricular lucency.
CSF: Large-volume lumbar puncture (spinal or CSF tap test) - CSF pressure will be normal, or
intermittently raised.
Intraventricular monitoring

Management:
Medical treatment of NPH includes acetazolamide and repeated lumbar puncture.

Surgical:

The mainstay of treatment is surgical insertion of a CSF shunt. This could be to the peritoneum, the right
atrium or, more recently, via external lumbar drainage.

201. A 29yo teacher is involved in a tragic RTA. After that incident, he has been suffering from
nightmares and avoided driving on the motorway. He has been dx with PTSD. What is the most
appropriate management?
a. CBT
b. Diazepam
c. Citalopram
d. Dosalepin
e. Olanzepin
Ans. The key is A. CBT.

PTSD:
Symptoms: Fearful; horrified; dazed Helpless; numb, detached Emotional responsiveness
Intrusive thoughts Derealization Depersonalization Dissociative amnesia Reliving of events
Avoidance of stimuli Hypervigilance Lack of Concentration Restlessness Autonomic arousal: pulse; BP;
sweating Headaches; abdo pains
Signs: Suspect this if symptoms become chronic, with these
signs (may be delayed years): difficulty modulating arousal; isolated-avoidant modes of living; alcohol abuse;
numb to emotions and relationships; survivor guilt; depression; altered world
view in which fate is seen as untamable, capricious or absurd, and life can yield no meaning
or pleasure.
Treatment: Watchful waiting for mild cases.
For severe cases: CBT or eye movement desensitization and reprocesing is done. Drug treatment is not
recommended but in case it is needed prescribe mirtazepine or paroxetine.

202. A 5yo child presents with fever. He looks pale. His parents say he always feels tired. On exam:
orchidomegaly & splenomegaly. Labs: WBC=1.7, Hgb=7.1, Plt=44. What is the dx?
a. ALL
b. CLL
c. AML
d. CML
e. Hodgkins
Ans. The key is A. ALL. [normally in ALL CBC shows raised WBC, low RBC and low platelet; but it is also possible
to all cell lines to be depressed, as is the presented case].

ALL: This is a malignancy of lymphoid cells, affecting B or T lymphocyte cell lines, arresting
maturation and promoting uncontrolled proliferation of immature blast
cells, with marrow failure and tissue infiltration.
Causes: Genetic susceptibility, environmental factors (ionizing radiations) Downs syndrome.
Commonest cancer of childhood.
Signs and symptoms:
Marrow failure: Anaemia (Hb), infection (WCC), and bleeding (platelets).
Infiltration: Hepatosplenomegaly, lymphadenopathysuperficial or mediastinal, orchidomegaly, CNS
involvementeg cranial nerve palsies, meningism.
INVESTIGATIONS: Characteristic blast cells on blood film and bone marrow
CXR and CT scan to look for mediastinal and abdominal lymphadenopathy.
Lumbar puncture should be performed to look for CNS involvement.
TREATMENT: Blood transfusions, prophylactic antibiotics, IV antibiotics in case of infection. Main stay of
treatment is chemotherapy.
Prognosis Cure rates for children are 7090%; for adults only 40%

203. A 6wk child is brought in with vomiting, constipation and decreased serum K+. What is the dx?
a. Pyloric stenosis
b. Duodenal atresia
c. Hirschsprung disease
d. Achalasia cardia
e. Tracheo-esophageal fistula
Ans. The key is A. Pyloric stenosis. [why not duodenal atresia? Pyloric stenosis is much more commoner than
duodenal atresia; in duodenal atresia the vomitus should contain bile, which is not the case in pyloric
stenosis].
Pyloric stenosis
Symptoms: Presents at 38 weeks)
with vomiting which occurs after feeds and becomes projectile (eg vomiting over far end of cot). Pyloric
stenosis is distinguished from other causes of vomiting by the following:
The vomit does not contain bile, as the obstruction is so high.
No diarrhoea: constipation is likely (occasionally starvation stools).
Even though the patient is ill: he is alert, anxious, and always hungryand possibly malnourished,
dehydrated.
The vomiting is extremely large volume and within minutes of a feed.
Try to palpate the olive-sized pyloric mass
There may be severe water & NaCl deficit. The picture is of hypochloraemic, hypokalaemic
metabolic alkalosis
Imaging: Ultrasound detects early, hard-to-feel pyloric tumours, but is only needed if examination is ve.
Barium studies are never needed.
Management: Correct electrolyte disturbances. Before surgery (Ramstedts pyloromyotomy/ endoscopic
surgery) pass a wide-bore nasogastric tube.
204. A 17 yo girl had an episode of seizure. Contraction of muscles started from around the
interphalangeal joints, which spread to the muscles of wrist and elbow. Choose possible type of
seizure?
a. Grand mal
b. Tonic clonic
c. Myoclonic
d. Absent
Ans. The key is C. Myoclonic. [seizures associated with contraction of specific muscle group is seen in
myoclonic seizers].
Types of seizures:
Primary generalized seizures Simultaneous onset of electrical discharge throughout cortex, with no localizing
features referable to only one hemisphere.
Absence seizures: Brief (10s) pauses, They do not fall but may pause in what they are doing. Their face
often looks pale with a blank expression. They may look dazed, the eyes stare and the eyelids may flutter a
little. Sometimes their head may fall down a little, or their arms may shake once or twice. Each seizure usually
starts and finishes abruptly. The person is not aware of the absence and resumes what they were doing.
Tonicclonic seizures: Loss of consciousness. Limbs stiffen (tonic), then jerk
(clonic). May have one without the other. Post-ictal confusion and drowsiness.
Myoclonic seizures: Sudden jerk of a limb, face or trunk. The patient may be thrown suddenly to the ground,
or have a violently disobedient limb: one patient described it as my flying-saucer epilepsy, as crockery which
happened to be in the hand would take off .
Atonic (akinetic) seizures: Sudden loss of muscle tone causing a fall, no LOC.
Infantile spasms/West syndrome: Peak age: 5 months. Clusters of head nodding (Salaam attack) and arm
jerks, every 330sec. IQ decrease in ~70%. EEG is characteristic (hypsarrythmia).

205. 46yo man, known case of chronic GN presents to OPD. He feels well. BP = 140/90mmHg. Urine dipstick:
protein ++, blood ++ and serum creatinine=106mmol/L. Which medication can prevent the progression of this
dx?
a. ACEi
b. Diuretics
c. Cytotoxic meds
d. Longterm antibiotics
e. Steroids
Ans. The key is A. ACEI. [renal impairment is delayed by ACEI].

206. A 23 yo girl presented with perioral paresthesia and carpopedal spasm 20 mins after a huge argument
with her boyfriend. What is the next step for this pt?
a. SSRI
b. Diazepam
c. Rebreath into a paper bag
d. Propranolol
e. Alprazolam
Q. 1. What is the key?
Q. 2. What is the likely diagnosis?
Ans. 1. The key is C. Rebreathin in paper bag. [hyperventilation causes CO2 washout and respiratory alkalosis.
If you continue breathing and rebreathing in paper bag it will allow CO2 concentration to rise in paper bag and
as you rebreath this again and again you will regain some washed out CO2 and thus relief to this alkalosis].
Ans. 2. The girl may have anxiety disorder when it precipitates leads to hyperventilation syndrome.
Anxiety:
Symptoms: Tension, agitation; feelings of impending doom, trembling; a sense of collapse; insomnia; poor
concentration; goose flesh; butterflies in the stomach; hyperventilation (so tinnitus, tetany, tingling, chest
pains); headaches; sweating; palpitations; poor appetite; nausea;
lump in the throat unrelated to swallowing (globus hystericus); difficulty in getting to sleep; excessive
concern about self and bodily functions; repetitive thoughts and activities
Childrens symptoms: Thumb-sucking; nail-biting; bed-wetting; foodfads.
Causes Genetic predisposition; stress (work, noise, hostile home), events (losing or gaining a spouse or job;
moving house). Others: Faulty learning or secondary gain (a husband forced to stay at home with
agoraphobic wife).
Treatment:
Symptom control: Listening is a good way to reduce anxiety. Explain that headaches are not from a tumour,
and that palpitations are harmless. Regular (non-obsessive!) exercise: Beneficial effects appear to equal
meditation or relaxation.
Meditation: Intensive but time-limited group stress reduction intervention based on mindfulness meditation
can have long-term beneficial effects.
Cognitivebehavioural therapy and relaxation appear to be the best specific measures with 5060% recovering
over 6 months.
Behavioural therapy employs graded exposure to anxiety-provoking stimuli.
Drugs augment psychotherapy: 1 Benzodiazepines (eg diazepam) 2 SSRI eg paroxetine in social anxiety). 3
Azapirones (buspirone, 5HT1A partial agonist; ess addictive/sedating than diazepam, and few withdrawal
issues). 4 Old-style antihistamines (eg hydroxyzine).
5. Beta blockers.6 Others: pregabalin and venlafaxine.
Progressive relaxation training: Teach deep breathing using the diaphragm, and tensing and relaxation of
muscle groups, eg starting with toes and working up the body. Practice is essential.
Hypnosis
207. A 25 yo woman has been feeling anxious and nervous for the last few months. She also
complains of palpitations and tremors. Her symptoms last for a few minutes and are very hard
to control. She tells you that taking alcohol initially helped her relieve her symptoms but now
this effect is wearing off and she has her symptoms even after drinking alcohol. What is the dx?
a. Panic disorder
b. Depression
c. OCD
d. Alcohol addiction
e. GAD
Ans. The key is A. Panic disorder.
Panic Attack:
This condition often co-exists with agoraphobia - the avoidance of exposed situations for fear of panic or
inability to escape
Panic attacks must be associated with >1 month's duration of subsequent, persisting anxiety about recurrence
of the attacks, the consequences of the attacks, or significant behavioural changes associated with them.

A panic attack is defined as a discrete episode of intense subjective fear, where at least four of the
characteristic symptoms, listed below, arise rapidly and peak within 10 minutes of the onset of the attack:
Attacks usually last at least 10 minutes but their duration is variable.
The symptoms must not arise as a result of alcohol or substance misuse, medical conditions or

other psychiatric disorders, in order to satisfy the diagnostic criteria.

Panic disorder manifests as the sudden, spontaneous and unanticipated occurrence of panic attacks, with
variable frequency, from several in a day to just a few per year:
Palpitations, pounding heart or accelerated heart rate.

Sweating.

Trembling or shaking.

Dry mouth.

Feeling short of breath, or a sensation of smothering.

Feeling of choking.

Chest pain or discomfort.

Nausea or abdominal distress.

Feeling dizzy, unsteady, light-headed or faint.

Derealisation or depersonalisation (feeling detached from oneself).

Fear of losing control or 'going crazy'.

Fear of dying.

Numbness or tingling sensations.

Chills or hot flushes.

Signs: No specific signs


Investigations: Just to rule out any physical illness.
Management: Involve the family, find and avoid any triggers. Find out if the symptoms are because of alcohol
use and treat that.
CBT is the first line.
Drugs: Offer an SSRI licensed for this indication first-line unless contra-indicated.

Consider imipramine or clomipramine if there is no improvement after 12 weeks and further

medication is indicated

If there has been an improvement after 12 weeks, continue for 6 months after the optimum dose has been
reached
At the end of treatment, withdraw the SSRI gradually,
Step 3
Reassess the condition and consider alternative treatments.

Step 4
If two interventions have been offered without benefit, consider referral to specialist mental health services.

208. A 2yo child is very naughty. His teacher complains that he is easily distracted. His parents say that he
cant do a particular task for a long time. He sometimes hurts himself and breaks many things. This causes
many troubles at home. What is the dx?
a. ASD
b. Dyslexia
c. ADHD
d. Antisocial personality disorder
e. Oppositional defiant
Ans. The key is C. ADHD (Attention deficit hyperreactive disorder).

Attention deficit & hyperactivity disorder (ADHD)


is the most common neurobehavioral disorder of childhood. It has prevalence of 35% in Western nations
ADHD is commoner in learning-disabled children, and if prenatal cannabis exposure. The core diagnostic
criteria are: impulsivity, inattention and hyperactivity. Not all those with ADD are hyperactive. There is no
diagnostic test
Most parents first note hyperactivity at the toddler stage, Family association is often present. These children
are at increased risk of self harm and suicide.
Management:
1st line treatment for pre-school children and school age children with moderate ADHD/moderate
impairment is parent training/education programmes. Older children may benefit from cognitive behavioural
therapy. Drugs may be useful in school age children if non-drug treatments fail (eg methylphenidate
atomoxetine:
Severe ADHD in school age children methylphenidate and atomoxetine are 1st line
treatments so ensure referral

209. A 79 yo lady who is otherwise well recently started abdominal pain. She is afebrile and
complains that she passed air bubbles during urination. A urethral catheter showed fecal
leakage in the urinary bag. What is the likely pathology?
a. Diuretics
b. CD
c. Rectosigmoid tumor
d. Large bowel perforation
e. UC
Ans. The key is B. CD. [debate came that Crohns disease cannot occur in 79 yrs but this is not the case!
Crohns disease can occur at any age, but is most frequently diagnosed in people ages 15 - 35. About 10% of
patients are children under age 18. [http://www.nytimes.com/health/guides/disease/crohns-disease/risk-
factors.html]. So I think it can occur in this age also and the features support the diagnosis of CD.
NEXT PAGE!
210. A 2 month child with diarrhea and vomiting for 6 days is brought in looking lethargic. What is the
appropriate initial inv?
a. BUE
b. Random blood sugar
c. CBC
d. CXR
e. AXR
Ans. The key is A. BUE.
Diarrhea makes the child dehydrated and loss of electrolytes occur. Which are making the the lethargic so we
need to check blood Urea and electrolyes and correct the electrolyte imbalance.

211. A 72 yo man fell while shopping and hurt his knee. His vitals are fine. He speaks in a low voice
and is very slow to give answers. What is the most probable dx?
a. Alzheimers
b. Vascular demetia
c. TIA
d. Pseudo-dementia
e. Picks dementia
Q. 1. What is the key?
Q. 2. What are the points in favour?
Ans. 1. The key is A. Alzheimers.
Ans. 2. Points in favour: i) age 72 yrs ii) fall iii) loss or slowness of speech.
why not vascular? in vascular: i) confusion ii) disorientation iii)loss of vision
why not pseudodementia? in pseudo i) onset is short and abrupt ii associated depression
why not picks i) dementia and aphasia
Why not TIA? In TIA complete resolution of symptom!! But here symptoms are persistent.
Alzheimers Disease:
This is the leading cause of dementia.
Onset may be from 40yrs (earlier in Downs syndrome, in which AD is inevitable).
Presentation: Suspect Alzheimers in adults with enduring, progressive and global cognitive impairment
(unlike other dementias which may affect certain domains but not others): visuo-spatial skill (gets lost),
memory, verbal abilities and executive function (planning) are all affected
and there is anosognosiaa lack of insight into the problems engendered by the disease, eg
missed appointments, misunderstood conversations or plots of films, and mishandling of money and clerical
work. Later there may be irritability; mood disturbance (depression or euphoria); behavioural change (eg
aggression, wandering, disinhibition); psychosis (hallucinations or delusions); agnosia (may not recognize self
in the mirror). There is no standard natural history. Cognitive impairment is progressive, but non-cognitive
symptoms may come and go over months. Towards the end, often but not invariably, patients become
sedentary, taking little interest in anything.
Associations: environmental and genetic factors both play a role. Accumulation of beta-amyloid
peptide, neurofibrillary tangles, increased numbers of amyloid plaques, and loss of the neurotransmitter
acetylcholinethe hippocampus, amygdala, temporal neocortex and subcortical nuclei (eg nucleus basalis of
Meynert) are most vulnerable. Vascular effects are also important95% of AD patients show evidence of
vascular dementia
Risk factors 1st-degree relative with AD; Downs syndrome; homozygosity for apolipoprotein
e (ApoE) e4 allele;are some of the risk factors
Treatment:
Refer to a specialist memory service.
Acetylcholinesterase inhibitors
Donepezil
Rivastigmine. Patches are also available.
Galantamine
212. A 47 yo man met with a RTA. He has multiple injuries. Pelvic fx is confirmed. He has not passed urine in
the last 4 hrs. What is the next appropriate management for this pt?
a. Urethral catheter
b. Suprapubic catheter
c. IV fluids
d. IV furosemide
e. Insulin
Q. 1. What is the key?
Q. 2. What is the reason of this management?
Ans. 1. The key is B. Suprapubic catheter.
Ans. 2. In pelvic fracture there is chance of urethral rupture and hence displacement of urethral catheter.
Indications of urethral catheterization:
Indications Relieve urinary retention, Monitor urine output in critically ill patients, Collect
uncontaminated urine for diagnosis.
It is contraindicated in urethral injury (eg pelvic fracture) and acute prostatitis.
Suprapubic catheterization: Sterile technique required. Absolutely contraindicated unless there is a large
bladder palpable or visible on ultrasound, because of the risk of bowel perforation. Be wary, particularly if
there is a history of abdominal or pelvic surgery. Suprapubic catheter insertion is high risk and you should be
trained before attempting it, speak to the urologists first!

213. A 49 yo pt presents with right hypochondriac pain. Inv show a big gallstone. What is the most appropriate
management?
a. Lap Cholecystectomy
b. Reassure
c. Low fat diet
d. Ursodeoxycholic acid
e. Emergency laparotomy
Q. 1. What is the key?
Q. 2. Points in favour?
Ans. 1. The key is A. Lap Cholecystectomy.
Ans. 2. i) as symptomatic only reassurence is not appropriate ii) as big ursodyoxycholic acid is less effective iii)
less invasive is preferred so laparoscopic rather than laparotomy.
Gall Stones:
Pigment stones: (<10%) Small, friable, and irregular. Causes: haemolysis.
Cholesterol stones: Large, often solitary. Causes: age, obesity (Admirands triangle: inc risk of stone if dec
lecithin, dec bile salts, inc cholesterol).
Mixed stones: Faceted (calcium salts, pigment, and cholesterol).
Gallstone prevalence: 8% of those over 40yrs. 90% remain asymptomatic.
Risk factors for stones becoming symptomatic: smoking; parity.
Acute cholecystitis follows stone or sludge impaction in the neck of the gallbladder, which may cause
continuous epigastric or RUQ pain (referred to the right shoulder, vomiting, fever, local peritonism, or a GB
mass. If the stone moves to the common bile duct (CBD), obstructive jaundice and cholangitis
may occur
Tests: WCC,
Ultrasound Is the best way to demonstrate stones, being 90-95% sensitive.
it shows a thick-walled, shrunken GB (also seen in chronic disease)
Treatment:
NBM, pain relief, IVI, and antibiotics eg cefuroxime 1.5g/8h IV.
Laparoscopic cholecystectomy is the treatment of choice for all patients fit for GA.
Open surgery is required if there is GB perforation.
Cholecystostomy is also the preferred treatment for acalculous cholecystitis

214. In a man who is neglected and alcohol dependent, which high suicidal risk, which factor can
increase this risk further?
a. Alcohol dependence
b. SSRI
c. Smoking
d. Agoraphobia
e. Court involvement
Ans. The key is A. Alcohol dependence. This is considered a wrong key by previous plabbers and
suggested correct key is B. SSRI.
Since the patient is already alcohol dependent so that cannot FURTHER increase the risk. The correct answer
here will be SSRI.

215. A 71 yo man presents with coarse tremor. He is on some meds. Which one can be the reason for the
tremor?
a. Lithium
b. Diazepam
c. Fluoxetine
d. Imipramine
e. Haloperidol
Ans. The key is A. Lithium. [lithium is associated with tremor].

Lithium:
Indications for use:
Acute manic or hypomanic episodes
Prophylaxis for bipolar disease. Significantly reduces suicidal risks.
Prophylaxis for recurrent depression & schizoaffective disorder.
Augments the effect of anti depressants.
Cluster headache prophylaxis.
Control of intentional self harm or suicidal behavior.

Contra-indications:
Cardiac disease
Significant renal impairment
Addison's disease and patients with low body sodium levels
Untreated hypothyroidism

Avoid in first trimester of pregnancy. Can be used in 2nd and 3rd trimesters if necessary. Avoid during breast
feeding.

During treatment: Check lithium levels (12 hours post dose) at least every three months

Common side-effects can usually be reduced or eliminated by lowering the lithium dose or changing the
dosage schedule:

Abdominal pain
Nausea
Metallic taste in the mouth (usually wears off)
Fine tremor
Thirst, polyuria, impaired urinary concentration - avoid fluid restriction
Weight gain and oedema

216. A young woman complains of diarrhea, abdominal cramps and mouth ulcers. AXR shows
distended transverse colon with goblet cell depletion on rectal biopsy. What is the most
probable dx?
a. CD
b. UC
c. Bowel Ca
d. Bowel obstruction
e. IBS
Q. 1. What is the key?
Q. 2. What are points in favour?
Ans. 1. The key is B. UC.
Ans. 2. In UC there is goblet cell depletion and less mucous production in contrast with CD where there may
be goblet cell hyperplasia and mucous secretion is not reduced.

Refer to the explanation of Question No. 209.


217. After eating a cookie at a garden party, a child began to cough and went blue. The mother also noticed
that there were swollen patches on the skin. What is the dx?
a. Allergic reaction
b. Aspiration of food
c. Cyanotic heart disease
d. Trachea-esophageal fistula
e. Achalasia cardia
Ans. The key is A. Allergic reaction.

This here is anaphylaxis as there are noticable skin changes present.

Urticaria Signs: wheals, rapid onset after taking drug association with angio-oedema /anaphylaxis. It can
result from both immunological and non-immunological mechanisms.
Causes: Drugs:morphine & codeine cause direct mast cell degranulation; penicillins & cefalosporins trigger IgE

responses; NSAIDs; ACEi.


Clinical diagnosis. No investigations required.
Management:
Find the cause and avoid/treat it.

Antihistamines:
Non-sedating H1 antihistamines are the mainstay of treatment
In pregnancy chlorphenamine is often the first choice of antihistamine.

Consider anaphylaxis when there is compatible history of rapid-onset severe allergic-type reaction with
respiratory difficulty and/or hypotension, especially if there are skin changes present and the treatment of
anaphylaxis is IM adrenaline not anti histamine

Treatment:ABCDE, Oxygen, IM Adrenaline. <6yrs 0.15ml, 6-12yrs 0.3ml, >12 yrs 0.5ml 1:1000

218. A 70 yo man presents with balance difficulties, vomiting and nausea. Which of the following is the best
inv?
a. MRI cerebellum
b. CT cerebellum
c. Skull XR
d. LP
e. Blood culture
Ans. The key is A. MRI cerebellum. [posterior fossa lesion MRI is preferred].

219. A 20 yo pt presents with colicky pain which radiates from loin to groin. He complains of similar episodes
in the past. Inv has been done and 7mm stone was found in the ureter. What is the most appropriate
management?
a. Percutaneous nephrolithotomy
b. Open surgery
c. Ureterscopy or laser
d. Conservative tx
e. ESWL
Q. 1. What is the key?
Q. 2. What treatments are recommended for different sized stones?
Ans. 1. The key is E. ESWL.
Ans. 2. Stones < 5mm: pass spontaneously, Increase fluid intake.
Stones 5mm-7mm /pain not resolving: medical expulsive therapy---> Nifedipine or Tamsulosin(and/or
prednisolone).
Stones 7mm-2cm: ESWL or Ureteroscopy using dormia basket.
Stones > 2cm/large/multiple/complex: Percutaneous nephrolithotomy.

Renal Stones (nephrolithiasis): Consist of crystal aggregates.


Site: 1 Pelviureteric junction 2 Pelvic brim 3 Vesicoureteric junction.

Types Calcium oxalate (75%) Magnesium ammonium phosphate (struvite/triple


phosphate; 15%) Also: urate (5%), hydroxyapatite (5%), brushite, cystine (1%), mixed

Presentation: Asymptomatic or:


1 Renal colic: excruciating ureteric spasms loin to
groin (or genitals/inner thigh), with nausea/vomiting. Often cannot lie still (differentiates
from peritonitis).
2 Renal obstruction felt in the loin (like intercostal nerve irritation pain; the latter is not colicky, and is
worsened by specific movements/pressure on a trigger spot).
Obstruction of mid-ureter may mimic appendicitis/diverticulitis.
Obstruction of lower ureter
may lead to symptoms of bladder irritability and pain in scrotum, penile tip, or labia majora.
Obstruction in bladder or urethra causes pelvic pain, dysuria, strangury (desire but inability to void)
interrupted flow. UTI can co-exist (risk if voiding impaired); pyelonephritis (fever, rigors, loin pain, nausea,
vomiting), pyonephrosis (infected hydronephrosis)
3 Haematuria
4 Proteinuria
5 Sterile pyuria
6 Anuria

Tests: BLIs, Mid stream urine, Urine C&S, urinary pH. Urine dipstick: Usually +ve for .blood (90%).
Imaging: Spiral non-contrast CT is superior to and has largely replaced IVU for
imaging stones (99% visible). 80% of stones are visible on KUB XR

Management: Initially: Analgesia, eg diclofenac + IV fluids if unable to tolerate PO; antibiotics (eg cefuroxime
1.5g/8h IV, or gentamicin) if infection.
Stones <5mm in lower ureter: ~9095% pass spontaneously. Increase fluid intake.
Stones >5mm/pain not resolving: Medical expulsive therapy: nifedipine 10mg/8h PO or alpha-blockers
(tamsulosin 0.4mg/d
227) promote expulsion and reduce analgesia
requirements:Most pass within 48h. If not, try extracorporeal shockwave lithotripsy (ESWL) (if <1cm), or
ureteroscopy using a basket.
Percutaneous nephrolithotomy (PCNL): keyhole surgery to remove stones, when large, multiple, or complex.
Open surgery is rare.

Indications for urgent intervention (delay kills glomeruli): Presence of infection


and obstructiona percutaneous nephrostomy or ureteric stent may be needed to relieve obstruction ;
urosepsis; intractable pain or vomiting; impending ARF; obstruction in a solitary kidney; bilateral obstructing
stones.

220. A footballer has been struck in the groin by a kick and a presents with severe pain and mild
swelling in the scrotum. What is the most appropriate next step?
a. USG
b. Doppler
c. Exploratory surgery
d. IV fluids
e. Antibiotics
Ans. The key is C. Exploratory surgery. [To exclude torsion].

Testicular Torsion: It is the twisting of the testes and it impairs the blood flow to the testes.
If in any doubt, surgery is required. If suspected refer immediately to urology.
Symptoms: Sudden onset of pain in one testis, which makes walking uncomfortable.
Pain in the abdomen, nausea, and vomiting are common.
Signs: Inflammation of one testisit is very tender, hot, and swollen. The testis may lie high and transversely.
Torsion may occur at any age but is most common at 1130yrs. With intermittent torsion the pain may have
passed on presentation, but if it was severe, and the lie is horizontal, prophylactic fi xing may be wise.
: The main differential is epididymo-orchitis but with this the patient tends to be older, there may be
symptoms of urinary infection, and more gradual onset of pain.

Tests: Doppler USS may demonstrate lack of blood flow to testis, as may isotope scanning. Only perform if
diagnosis equivocaldo not delay surgical exploration.

Treatment: Ask consent for possible orchidectomy + bilateral fixation (orchidopexy) At surgery expose and
untwist the testis. If its colour looks good, return it to the scrotum and fix both testes to the scrotum.

221. A 47 yo ex-soldier suffers from low mood and anxiety. He cant forget the images he faces
before and has always had flashbacks. He is not able to watch the news because there are
usually some reports about war. What is he suffering from?
a. Depression
b. PTSD
c. Panic attack
d. Agoraphobia
e. GAD
Ans. The key is B. PTSD. [repeated flashbacks and tendency to avoid the thoughts of stressor is diagnostic of
PTSD].

PTSD:
Symptoms: Fearful; horrified; dazed Helpless; numb, detached Emotional responsiveness
Intrusive thoughts Derealization Depersonalization Dissociative amnesia Reliving of events
Avoidance of stimuli Hypervigilance Lack of Concentration Restlessness Autonomic arousal: pulse; BP;
sweating Headaches; abdo pains
Signs: Suspect this if symptoms become chronic, with these
signs (may be delayed years): difficulty modulating arousal; isolated-avoidant modes of living; alcohol abuse;
numb to emotions and relationships; survivor guilt; depression; altered world
view in which fate is seen as untamable, capricious or absurd, and life can yield no meaning
or pleasure.
Treatment: Watchful waiting for mild cases.
For severe cases: CBT or eye movement desensitization and reprocesing is done. Drug treatment is not
recommended but in case it is needed prescribe mirtazepine or paroxetine.

222. A 36 yo woman has recently spent a lot of money on buying clothes. She goes out almost every night with
her friends. She believes that she knows better than her friends, so she should
choose the restaurant for eating out. She gave hx of having low mood at 12 yo. Whats the dx?
a. Mania
b. Depression
c. Bipolar affective disorder
d. Borderline personality disorder
e. Dysthymia
Ans. The key is C. Bipolar affective disorder. [Initial depressive episode (may be before a long) followed by
mania is bipolar affective disorder].

Signs of mania: Mood: Irritability (80%), euphoria (71%), lability (69%).


Cognition: Grandiosity (78%); flight of ideas/racing thoughts (71%); distractibility/ poor concentration (71%);
confusion (25%), many conflicting lines of thought urgently racing in contrary directions; lack of insight.
Behaviour: Rapid speech (98%), hyperactivity (87%), reduced sleep (81%), hypersexuality (57%), extravagance
(55%). Psychotic symptoms: Delusions (48%), hallucinations (15%). Less severe states are termed hypomania.
If depression alternates
with mania, the term bipolar affective disorder is used (esp. if there is a
history of this). During mood swings, risk of suicide is high. Cyclical mood
swings without the more florid features (as above) are termed cyclothymia

Causes Infections, hyperthyroidism; SLE; thrombotic thromocytopenic purpura; stroke; water


dysregulation/hyponatremia;. Drugs: Amphetamines, cocaine, antidepressants (esp. venlafaxine), captopril,
steroids, procyclidine, L-dopa, baclofen.
Bipolar disorder: (Age at onset: <25.) In a 1st attack Ask about: Infections, drug use, and past or family history
of psychiatric disorders. Do: CT of the head, EEG, and screen for drugs/toxins.

Treating acute mania for acute moderate/severe mania: olanzapine 10mg PO(SE: weight gain; inc glucose), or
valproate semisodium, eg 250mg/8h PO
Prophylaxis Those who have bipolar affective disorder after successful treatment of the manic or depressive
episode should have a mood stabilizer for longer-term control. If compliance is good, and U&E, ECG, and T4
normal, give lithium carbonate.

223. A 28 yo female presents with a 3m hx of diarrhea. She complains of abdominal discomfort and passing
stool 20x/day. Exam=febrile. Barium enema shows cobblestone mucosa. What is the
most likely dx?
a. Ameoba
b. Colon Ca
c. GE
d. CD
e. UC
Ans. The key is D. CD. [Hx of diarrhea, abdominal discomfort, and patient being febrile indicate gut
inflammation and cobblestone appearance on barium enema is suggestive of CD].
Refer to the explanation of Q.NO 209

224. A child is brought in with high grade fever, runny nose and bark-like cough. He is also drooling. What is
the most appropriate tx for this child?
a. Corticosteroids
b. Paracetamol
c. Adrenaline nebulizer
d. IV antibiotics
e. Intubation under GA
Ans. The key is E. Intubation under GA. [high fever, bark-like cough, drooling in a child suggest epiglottitis
where urgent intubation is needed to avoid respiratory blockage from epiglottitis].

It is a very tricky question. Symptoms of both croup and acute epiglottitis are mixed here! A confirm diagnosis
cannot be made.
In favour of croup: runny nose and barking cough.
In favour of acute epiglottitis: high grade fever, drooling.

Perhaps in the original exam question more information was provided which couldnot be recalled here in the
question which would have helped in the diagnosis.

Laryngotracheobronchitis/croup: is the leading cause of stridor with a barking cough. More common than
epiglottitis.
Cause: 95% are viral, eg parainfluenza (ribavirin can help, eg in immunodeficiency). Bacteria (klebsiella;
diphtheria) & fungi are rare.
Presentation: If there is cough and no drooling, croup is almost always the diagnosis.
Initially presents with symptoms of URTI. Hoarsness and barking cough develop in a couple of days worse at
night and mild to moderate fever. Stridor is also present. Respiratory distress with marked tachypnoea and
intercostal recession may be noted. Drowsiness, lethargy, and cyanosis despite increasing respiratory distress
should be considered as red flags for impending respiratory failure.
Management: Usually self-limiting; treat at home ( antibiotics). Admit (eg to ITU) if severe. In children, CXR
may show steeple sign of a tapering trachea. Give antibiotics, humidified O2, + nebulized adrenaline (5mL
1:1000, may buy time in severe disease needing ventilating), and dexamethasone 150g/kg PO stat or
budesonide 2mg nebulized.

Acute epiglottitis is rarer than croup but mortality is high: 1% if respiratory distress. Its an emergency as
respiratory arrest can occur.
Presentation: Often, history is short, septicaemia is rapid, and cough is absent. Also: sore throat (100%), fever
(88%), dyspnoea (78%), voice change (75%), dysphagia (76%), tender anterior neck cellulitis (27%),
hoarseness (21%), pharyngitis (20%), anterior neck nodes (9%), drooling (head for ward tongue out), prefers to
sit, refusal to swallow,
Cause: Haemophilus (vaccination has reduced prevalence); Strep pyogenes.
Investigation: Fibre-optic laryngoscopy remains the 'gold standard' for diagnosing epiglottitis
Management: Take to ITU; dont examine throat (causes resp. arrest). Give O2 by mask, Give nebulized
adrenaline, IV dexamethasone, antibiotics, antipyretics until the anesthetist arrives. Definitive management is
intubation

225. A 78yo lady on warfarin for atrial fibrillation lives in a care home. She presents with hx of
progressive confusion for three days. She was also noticed to have bruises on her arms. INR = 7. What is the
most probable dx?
a. Alzheimers
b. Delirium
c. Subdural hemorrhage
d. Vascular dementia
e. Picks dementia
Ans. The key is C. Subdural haemorrhage. [Age 78 yrs, living in a care home where unnoticed trivial injury is
possible (like fall), warfarin and high INR is potential risk factor of subdural haemorrhage suggested by bruises
on arms also].

Subdural Haemorrhage: Consider this very treatable condition in all whose conscious level fluctuates, and also
in those having an evolving stroke, especially if on anticoagulants.
Causes: Bleeding is from bridging veins between cortex and venous sinuses (vulnerable to deceleration injury),
resulting in accumulating haematoma between dura and arachnoid. Most subdurals are from trauma but the
trauma is often forgotten as it was so minor or so long ago (up to 9 months). Elderly are particularly
susceptible. Other risk factors: falls (epileptics, alcoholics); anticoagulation
Symptoms Fluctuating level of consciousness (seen in 35%) insidious physical or intellectual slowing,
sleepiness, headache, personality change, and unsteadiness.
Signs: Raised ICP; seizures. Localizing neurological symptoms (eg unequal pupils, hemiparesis) occur late and
often long after the injury
Imaging: CT/MRI shows clot midline shift (but beware bilateral isodense clots). Look for crescent-shaped
collection of blood over 1 hemisphere. The sickleshape different iates subdural blood from extradural
haemorrhage.
Treatment Irrigation/evacuation, eg via burr twist drill and burr hole craniostomy, can be considered 1st-line;
craniotomy is 2nd-line if the clot has organized. Address causes of the trauma (eg falls due cataract or
arrhythmia; abuse).

226. A 28 yo drug user presents to the ED with collapse and anuria. His serum K+ = 7.5mmol/L. CXR = early
pulmonary edema. What is the next appropriate management?
a. Urgent hemodialysis
b. IV calcium gluconate
c. IV insulin + dextrose
d. Furosemide
e. IV NS 0.9%
Ans. The key is B. IV calcium gluconate. [To correct hyperkalemia to prevent cardiac arrhythmia].
Hyperkaelemia: A plasma potassium >6.5mmol/L is an emergency and needs urgent treatment
The worry is of myocardial hyperexcitability leading to ventricular fibrillation and cardiac arrest.

Concerning signs and symptoms fast irregular pulse, chest pain, weakness, palpitations, and light-headedness.
ECG: tall tented T waves, small P waves, a wide QRS complex (eventually becoming sinusoidal), and ventricular
fibrillation

Artefactual results: If the patient is well, and has none of the above findings, repeat
the test urgently as it may be artefactual

Causes: Oliguric renal failure K+-sparing diuretics Rhabdomyolysis Metabolic acidosis(DM)


Excess K+ therapy Addisons disease Massive blood transfusion Burns Drugs, eg ACE-i,
suxamethonium Artefactual result

Management:
Stabilisation of the cardiac membrane
intravenous calcium gluconate

Short-term shift in potassium from extracellular to intracellular fluid compartment


combined insulin/dextrose infusion
nebulised salbutamol

Removal of potassium from the body


calcium resonium (orally or enema)
loop diuretics
dialysis

227. A 32 yo woman suffers an episode of severe occipital headache with vomiting and loss of
consciousness. She is brought to the hospital where she is found to be conscious and completely alert. Exam:
normal pulse & BP. No abnormal neurological signs. What is the next step in management?
a. Admission for observation
b. CT brain
c. MRI head
d. Reassurance and discharge
e. XR skull
Ans. The key is B. CT brain. [basilar migraine can cause severe headache and LOC. But there occurs no
neurological deficit and recovering from unconsciousness becomes completely alert. But to diagnose basilar
migraine there should at least history of two migraine attacks with aura. As here diagnostic criteria of basilar
migraine is not fulfilled we can not discharge the patient without neuroimaging like CT or MRI].

228. A 25 yo woman was brought to the ED by her boyfriend. She has many superficial lacerations on her
forearm. She is so distressed and constantly says her boyfriend is going to end the relationship. She denies
trying to end her life. What is the most likely dx?
a. Acute psychosis
b. Severe depression
c. Psychotic depression
d. Borderline personality disorder
e. Schizophrenia
Ans. The key is D. Borderline personality disorder. [ Borderline personality disorder: Act impulsively and
develop intense but short-lived emotional attachment to others. They are usually attention seekers but not
suicidal].

Borderline Personality Disorder: There is unstable affect regulation, poor impulse control, and poor
interpersonal relationships/self-image, eg with repeated self-injury, suicidality, and a difficult life-course
trajectory
Associations: ADHD;2 learning difficulties. Genetics and adverse childhood events (eg abuse) are
predispositions. Intervene (and refer) early with specific management plan, addressing work, Dialectical
behaviour therapy, inpatient hospital programmes, and drugs can reduce depression, anxiety, and impulsive
aggression. Eventually, supportive interpersonal dyads are achievable

229. A young woman was brought to the hospital. On exam she has low temperature and tremor.
She says when she closes her eyes, she can see colors. What drug has been used?
a. Amphetamines
b. LSD
c. Cocaine
d. Heroine
e. Ecstasy
Ans. The key is B. LSD.

LSD stands for its chemical name, lysergic acid diethylamide. It is also commonly called acid. Other terms
include blotter, tripper, flash, stars, rainbows.
What are the harmful effects? It can make you frightened and confused. Sometimes you can get "flashbacks"
when you relive the same experience again. People can be more likely to self-harm when they have a bad trip.

230. A lady comes in severe liver disease and hematemesis. Her INR is >10. What should she be given?
a. FFP
b. Steroids
c. Whole blood
d. IV fluids
e. Vit K
Ans. The key is A. FFP.

IN any major bleeding irrespective of INR prothrombin complex concentrate (or FFP) is given and vit.K is also
given! But vit.K takes time to act FFP is a better options.
Situation Management

Major bleeding Stop warfarin


Give intravenous vitamin K 5mg

Prothrombin complex concentrate - if not available then FFP*

INR > 8.0 Stop warfarin

Minor bleeding Give intravenous vitamin K 1-3mg

Repeat dose of vitamin K if INR still too high after 24 hours

Restart warfarin when INR < 5.0

INR > 8.0 Stop warfarin

No bleeding Give vitamin K 1-5mg by mouth, using the intravenous preparation orally

Repeat dose of vitamin K if INR still too high after 24 hours

Restart when INR < 5.0

INR 5.0-8.0 Stop warfarin

Minor bleeding Give intravenous vitamin K 1-3mg

Restart when INR < 5.0

INR 5.0-8.0 Withhold 1 or 2 doses of warfarin

No bleeding Reduce subsequent maintenance dose

231. After eating a cookie at a garden party, a child began to cough and went blue. The mother also noticed
that there were swollen patches on the skin. What is the initial management?
a. OTC antihistamine
b. Oxygen
c. Bronchodilators
d. Epinephrine IM
e. Nebulized epinephrine
Ans. The key is D. Epinephrine IM [anaphylaxis with partially blocked airway].
Already explained in previous questions.
232. A 63 yo female is noted to have left pupil unresponsive to light and is dilated. What is the most probably
dx?
a. Pontine hemorrhage
b. Subdural hemorrhage
c. Cerebellar hemorrhage
d. Extradural hemorrhage
e. Subarachnoid hemorrhage
Ans. The key is D. Extradural hemorrhage. It is a wrong key. In a 63 year old extradural hemorrhage is
extremely unlikely. As no clinical picture is described in question except 3rd nerve palsy E. SAH is more logical
answer!

Sub Arachnoid Haemorrhage


Spontaneous bleeding into the subarachnoid space is often catastrophic
Causes: Rupture of saccular aneurysms (80%); arteriovenous malform ations (AVM; 15%). No cause
is found in <15%.
Risk factors: Smoking, alcohol misuse, inc BP, bleeding disorders, mycotic aneurysm (SBE), perhaps post-
menopausal oestrogen >45yrs old).

Berry aneurysms Common sites: junction of posterior communicating with the internal carotid or of the
anterior communicating with the anterior cerebral artery or bifurcation of the middle cerebral artery. Some
are hereditary. Associations: Polycystic kidneys, coarctation of the aorta, EhlersDanlos syndrome

Symptoms Sudden (usually, but not always, within seconds) devastating typically occipital headacheI
thought Id been kicked in the head. Vomiting, collapse, seizures and coma often follow. Coma/drowsiness
may last for days.
Signs Neck stiffness, Kernigs sign (takes 6h to develop), retinal, subhyaloid and vitreous bleeds (=Tersons
syndrome; it carries a worse prognosis) Focal neurology at presentation may suggest site of aneurysm (eg
pupil changes indicating a IIIrd nerve palsy with a posterior communicating artery aneurysm) or intracerebral
haematoma. Later deficits suggest complications

Tests CT detects >90% of SAH within the 1st 48h. LP if CT ve and no contraindication >12h after headache
onset. CSF in SAH is uniformly bloody early on, and becomes xanthochromic (yellow) after several hours due
to breakdown products of Hb (bilirubin). Finding xanthochromia confirms SAH

Management Refer all proven SAH to neurosurgery immediately.


Re-examine CNS often; chart BP, pupils and GCS. Repeat CT if deteriorating.
Maintain cerebral perfusion by keeping well hydrated, and aim for SBP >160mmHg. Treat BP only if very
severe.
Nimodipine is a Ca2+ antagonist that reduces vasospasm and consequent morbidity from cerebral
ischaemia.
Endovascular coiling is preferred to surgical clipping where possible. Do catheter
or CT angiography to identify single vs multiple aneurysms before intervening.
Intracranial stents and balloon remodelling enable treating wide-necked aneurysms.

Complications Rebleeding is the commonest cause of death, and occurs in 20%,


often in the 1st few days. Cerebral ischaemia due to vasospasm may cause a permanent
CNS deficit, and is the commonest cause of morbidity. Hydrocephalus, due to blockage
of arachnoid granulations, requires a ventricular or lumbar drain. Hyponatraemia
is common but should not be managed with fluid restriction. Seek expert help
233. A 28yo business exec presents at the GP asking for some help because she has been arguing with her
boyfriend frequently. She is worried about her weight, and she thinks she may be fat. She has been on a diet
and lost 7 kgs in the last 2 months on purpose. She is eating less. She used to do a lot of exercise. Now she
says shes feeling down, has some insomnia and feels tired and without energy. She has not showed up at
work. She is worried because recently she got a loan to buy a luxury car. She cant be fired. She complains
about her low mood. She thinks this is weird because she used to be extremely productive. She used to work
showing an excellent performance at the office. She even received compliments from her boss. How, she says
her boyfriend is angry because her apartment is a chaos. Usually she spends a lot of time cleaning it, even
upto 3 AM. She liked it to be perfect, but not its a mess. On exam: BMI=23, no other signs. What is the most
probably dx?
a. Anorexia nervosa
b. Bipolar disease
c. Binge eating disorder
d. Hyperthyroidism
e. Schizophrenia
Ans. The key is B. Bipolar disease.

Bipolar Disease:
It is characterised by episodes of mania (or hypomania) and depression. Either one can occur first and one
may be more dominant than the other but all cases of mania eventually develop depression.
During the manic phase the following may be present:

Grandiose ideas.
Pressure of speech.
Excessive amounts of energy.
Racing thoughts and flight of ideas.
Overactivity.
Needing little sleep or an altered sleep pattern.
Easily distracted - starting many activities and leaving them unfinished.
Bright clothes or unkempt.
Increased appetite.
Sexual disinhibition.
Recklessness with money.

Depressive phase:
In the depressive phase, patients experience low mood with reduced energy. Patients have no joy in daily
activities and have negative thoughts. They lack facial expressions and have poor eye contact and may be
tearful and unkempt. Low mood is worse in the mornings and is disproportionate to the circumstances. There
may be feelings of despair, low self-esteem and guilt for which there may be no clear reason. There may be
weight loss, reduced appetite, altered sleep pattern with early morning wakening and loss of libido.

Diagnosis:

ICD-10 requires at least two episodes in which a person's mood and activity levels are significantly disturbed
(one of which must be mania or hypomania)
Three of the following symptoms confirm mania:

Grandiosity/inflated self-esteem.
Decreased need for sleep.
Pressured speech.
Flight of ideas (rapidly racing thoughts and frequent changing of their train of thought).
Distractibility.
Psychomotor agitation.
Excessive involvement in pleasurable activities without thought for consequences (eg, spending
spree resulting in excessive debts).

There may also be psychotic symptoms - eg, delusions and hallucinations. The manic episode is mixed if there
are associated depressive symptoms

Management: The following are non-pharmacological methods:

Education regarding diagnosis, treatment and side-effects.


Good communication.
Self-help groups.
Support groups.
Self-monitoring of symptoms, side-effects and triggers.
Coping strategies.
Psychological therapy.
Encouragement of engagement in calming activities.
Telephone support.

Psychological therapies have been shown to be beneficial - eg, cognitive behavioural therapy which helps to
identify triggers and how to avoid them.

Pharmacological treatment: For manic attack treatment is same as for mania

If the pt is on antipsychotics and still develops an attack of mania the dose should be increased. Drugs
commonly used are haloperidol, olanzapine, quetiapine and risperidone. If one antipsychotic is ineffective it is
worth changing to a different one.

If ineffective consider adding lithium, if contra indicated add valproate.

Treatment of an acute depressive episode

A risk assessment of suicidal ideation should be made. If it is considered that compulsory hospital admission
would be in the patient's interest, the Mental Health Act or Common Law may need to be invoked

Patients with moderate-to-severe depression should be offered fluoxetine combined with


olanzapine or quetiapine on its own.
If there is no response, lamotrigine on its own can be tried.
If patients are already taking lithium, the level should be checked and the dose increased as
necessary. If this fails, fluoxetine combined with olanzapine or quetiapine can be added.,
During an acute mixed episode antidepressants should be avoided and the aim should be to try to stabilise
patients on anti-manic medication.

234. A woman brought her husband saying she wants the thing on his forehead removed. The
husband is refusing tx saying it improves his thinking. What is the next most appropriate next
step?
a. Assess his mental capacity to refuse tx
b. Remove lesion
c. Refer to ED
d. Mini-mental state exam
e. Refuse surgery and send pt back
Ans. The key is A. Assess his mental capacity to refuse treatment.

235. A 37 yo man who has many convictions and has been imprisoned many times has a hx of many
unsuccessful relationships. He has 2 boys but doesnt contact them. What is the most probable
dx?
a. Borderline personality disorder
b. Schizophrenia
c. Avoidant personality disorder
d. Histrionic personality disorder
e. Antisocial behavior disorder

Ans. The key is E. Antisocial behavior disorder.

Antisocial behavior
Features include:

Unstable interpersonal relationships.


Disregard for the consequences of their behaviour.
A failure to learn from experience.
Egocentricity.
A disregard for the feelings of others.
A wide range of interpersonal and social disturbance.
Comorbid depression and anxiety.
Comorbid alcohol and drug misuse

Diagnostic criteria:
People with antisocial personality disorder have a pervasive pattern of disregard for and violation of the rights
of others and the rules of society. A history of conduct disorder before the age of 15 is a requirement for a
diagnosis of antisocial personality disorder (in the DSM-IV criteria) and includes the following features:

Repeated breaches of the law. They may well have recurrent criminal convictions.
Frequent lying and deception, even when there is no obvious gain.
Physical aggression.
Reckless disregard for safety of self or others.
Utter irresponsibility in work and family environments.
Lack of remorse.

Tests: Toxicology screen and HIV screening should be done

Management: All patients diagnosied in primary setting should be referred to specialist centers.

No drug has UK marketing authorisation specifically for the treatment of antisocial personality disorder.
However, antidepressants and antipsychotics are often used to treat some of the associated problems and
symptoms.

Psychotherapy is at the core of care for personality disorders generally. Cognitive behavioural therapy and
group therapy are perhaps the most widely used and available forms of psychotherapy

Complications:
Suicide
Substance abuse
Accidental injury
Depression
Homicide

236. A 60 yo man has a pathological rib fx. He also complains of recurrent infection. BMA is done. Labs: Ca2+ =
3.9mmol/L and ALP = 127u/L. what type of cell would be found in abdundance in
the marrow smear?
a. Plasma cell
b. Myeloid cell
c. Bence-jones protein
d. Megakaryocytes
e. Reticulocytes
Q. 1. What is the key.
Q. 2. What is the diagnosis?
Q. What are the points in favour of diagnosis?
Ans. 1. The key is A. Plasma cell.
Ans. 2. The diagnosis of multiple myeloma.
Ans. 3. Points in favour: i) age 60 yrs ii) pathological rib fracture (from metastases) iii) recurrent infection (due
to B cell dysfunction (manifested as hypogammaglobulinemia), numerical and functional abnormalities of T
cells, and dysfunction of natural killer cells), iv) raised calcium level.

Multiple Myeloma: PCDs are due to an abnormal proliferation of a single clone of plasma or
lymphoplasmacytic cells leading to secretion of immunoglobulin occuring as monoclonal bands or paraprotein
in urine or serum electrophoresis.
IgG in ~ ; IgA in ~ ; A very few are IgM or IgD. Other Ig levels are low (immunoparesis, causing
susceptibility to infection). In ~ , urine contains Bence Jones proteins.
Symptoms:
Osteolytic bone lesions causing backache, pathological fractures (eg long bones
or ribs) and vertebral collapse. Hypercalcaemia may be symptomatic.
Anaemia, neutropenia, or thrombocytopenia may result from marrow infiltration
by plasma cells, leading to symptoms of anaemia, infection and bleeding.
Recurrent bacterial infections due to immunoparesis, and also because of neutropenia
due to the disease and from chemotherapy.
Renal impairment due to light chain deposition.
Diagnostic criteria:
1 Monoclonal protein band in serum or urine electrophoresis.
2 Plasma cells on marrow biopsy
3 Evidence of end-organ damagefrom myeloma:
Hypercalcaemia Renal insufficiency Anaemia
4 Bone lesions: a skeletal survey afterdiagnosis detects bone disease: Xraysof chest; all of spine; skull; pelvis.

Tests: Rouleaux formation, persistently inc ESR or Polycythemia Vera, inc urea and creatinine, inc Ca2+ (in
~40%), alk phos usually normal unless healing fracture.
Screening test: Serum and urine electrophoresis. Beta2-microglobulin (as a prognostic test).
Imaging: X-rays: lytic punched-out lesions, eg pepper-pot skull, vertebral collapse, fractures or osteoporosis.
CT or MRI may be useful to detect lesions not seen on XR.
Treatment: Symptomatic. For bones give bisphosphonates. For anemia transfusion or erythropoietin. Keep
hydrated to prevent renal failure, dialysis may be needed. Treat infections.
Chemotherapy: If unsuitable for intensive , melphalan + prednisolone is used.

Complications: Hypercalcemia, Renal injury, cord compression, hyperviscosity.

237. A child presents with blue marks on the sclera, short stature and heart murmur. What is the dx?
a. Osteogenesis imperfect
b. Hypopituitarism
c. VSD
d. Achondrogenesis
e. Dwarfism
Ans. The key is A. Osteogenesis imperfecta.
Osteogenesis imperfecta:
is an inherited disorder of type I collagen that
results in fragile, low density bones
Types:
I The mildest and most common form. It is autosomal dominant. Associated with blue sclerae (due to
increased corneal translucency) and 50% have hearing loss. Fractures typically occur before puberty. There is a
normal life expectancy.
II Lethal perinatal form with many fractures and dwarfism; it is recessive.
III Severe formoccurs in about 20%. Recessive. Fractures at birth + progressive spinal and limb deformity,
with resultant short stature; blue or white sclera;Life expectancy is decreased.
IV Moderate form, autosomal dominant. Fragile bones, white sclerae after infancy.

Radiographs: Many fractures, osteoporotic bones with thin cortex, and bowing deformity of long bones.
Histology: Immature unorganized bone with abnormal cortex.
Treatment: Prevent injury. Physio, rehab and occupational therapy are key. Bisphosphonates can be used.
238. A 5month child cant speak but makes sounds. She can hold things with palm, not fingers. Cant sit
independently but can hold her head and sit when propped up against pillows. Hows the
childs development?
a. Normal
b. Delayed speech
c. Delayed sitting
d. Delayed motor development
Ans. The key is A. normal

239. A 27 yo woman has hit her neck in an RTA without complains of tingling or motor loss. What is the next
most appropriate inv?
a. MRI
b. XR
c. CT cervical
d. Diagonal XR
Ans. The key is B. X-ray. [As there is no neurological deficit we can exclude any fracture by x-ray first].[Diagonal
x ray means ,oblique view of cervical spine. By this view we can assess facet joint arthopathy. This doesn't
related to RTA].
240. A young female who has many superficial lacerations was brought into the ED by her boyfriend for
superficially lashing her upper arm. She is adamant and screaming that she is not suicidal but scared her
boyfriend wants to leave her. What is the dx?
a. Acute psychosis
b. Severe depression
c. Obsessive
d. Bipolar
e. Borderline personality
f. Schizophrenia
Ans. The key is acute psychosis.

241. A 22yo woman was brought by her boyfriend with multiple superficial lacerations. There are
scars of old cuts on her forearms. She is distressed because he wants to end the relationship.
She denies suicide. What is the most likely dx?
a. Acute psychosis
b. Borderline personality disorder
c. Severe depression
d. Schizoid personality
e. Psychotic depression
Ans. The key is B. Borderline personality disorder.
242. A 31yo single man lives with his mother. He usually drives to work. He always thinks when the traffic
lights change, his mother is calling him, so he drives back home. What is the dx?
a. OCD
b. GAD
c. Schizophrenia
d. Bipolar
e. Cyclothymia
Ans. The key is C. Schizophrenia. [ delusion of reference - he thinks that the changing traffic lights are giving
message to him].

Schizophrenia is the most common form of psychosis. It is a lifelong, condition, which can take on either a
chronic form or a form with relapsing and remitting episodes of acute illness.

Multiple factors are involved in schizophrenia - eg, genetic, environmental and social.
Risk factors include= family history, intrauterine and perinatal complications,Intrauterine infection,
particularly viral, Abnormal early cognitive/neuromuscular development etc.
Features=
The hallmark symptoms of a psychotic illness are: Delusions,Hallucinations (auditory hallucination like two or
more voices discussing the patient in the third person,thought echo,voices commenting on the patient's
behaviour), Thought disorder (thought insertion,thought withdrawal, thought broadcasting), Lack of insight.

These first Rank or positive symptoms of schizophrenia are absent in other psychotic disorders.

Schizophrenia

NICE published guidelines on the management of schizophrenia in 2009.


Key points:
oral atypical antipsychotics are first-line
Examples of atypical antipsychotics
clozapine, olanzapine,risperidone,quetiapine,amisulpride
adverse effects:
weight gain
clozapine is associated with agranulocytosis
cognitive behavioural therapy should be offered to all patients
close attention should be paid to cardiovascular risk-factor modification due to the high rates of
cardiovascular disease in schizophrenic patients (linked to antipsychotic medication and high smoking
rates).

243. A 56yo woman is known case of pernicious anemia. She refuses to take hydroxycobalamin IM as she is
needle shy. She asks for oral medication. Why will oral meds be not effective?
a. Intrinsic factor def
b. Malabsorption
c. Irritated gastric mucosa
d. Lack of gastric acidity
Ans. The key is A. Intrinsic factor def.

Pernicious Anemia: This is caused by an autoimmune atrophic gastritis, leading to achlorhydria and lack of
gastric intrinsic factor secretion.

Associations Other autoimmune diseases: thyroid disease (~25%), vitiligo, Addisons disease,
hypoparathyroidism. Carcinoma of stomach is ~3-fold more common in pernicious anaemia
Investigations: Hb (30110g/L) Raised MCV Decreased WCC and platelets if severe Decreased Serum
B12 Reticulocytes reduced or normal as production impaired Hypersegmented polymorphs Megaloblasts
in the marrow
Specific tests: 1.Parietal cell antibodies: found in 90% with PA, 2 Intrinsic factor (IF) antibodies:
specific for pernicious anaemia, but lower sensitivity.
Treatment Treat the cause if possible. If a low B12 is due to malabsorption, injections are required. Replenish
stores with hydroxocobalamin (B12) 1mg IM alternate days, for 2wks Maintenance: 1mg IM every 3 months
for life.
If the cause is dietary, then oral B12 can be given after the initial acute course.

244. An 11m baby had an apnea event. The parents are worried that if something like this happens in the
future, how they are to deal. Advise them about infant CPR.
a. Index and middle finger compression
b. Compression with palm of one hand
c. Compression with palm of two hands
d. Compression with rescue breaths 30:2
e. Compression with rescue breaths 15:2
Ans. The key is A. Index and middle finger compression.

How to give chest compressions: Compress lower half of sternum to of the


chests depth; use the heel of one hand (or, in babies, with both your thumbs,
with your hands encircling the thorax) If >8yrs, the adult 2-handed method is
OK. For an infant, 2 fingers are sufficient, in the middle of a line joining the nipples.

How to give the rescue breaths to a child: Ensure head tilt and chin lift.
Pinch the soft part of his nose. Open his mouth a little, but maintain chin up.
Take a breath, and place your lips around his mouth (good seal). Blow steadily
into his mouth over 11.5sec
245. A teacher brings in a child who says she fell down after hitting a table. On probing further, you
decide that it was most probably an absence seizure. What led you to this dx?
a. The child had not eaten since morning
b. The child suddenly went blank and there was up-rolling of eyes
c. The child started moving his fingers uncontrollably before he fell
d. The childs body became rigid and then started to jerk
Ans. The key is B. The child suddenly went blank and there was up-rolling of eyes.

Types of seizures:
Primary generalized seizures Simultaneous onset of electrical discharge throughout cortex, with no localizing
features referable to only one hemisphere.
Absence seizures: Brief (10s) pauses,They do not fall but may pause in what they are doing. Their face often
looks pale with a blank expression. They may look dazed, the eyes stare and the eyelids may flutter a little.
Sometimes their head may fall down a little, or their arms may shake once or twice. Each seizure usually starts
and finishes abruptly. The person is not aware of the absence and resumes what they were doing..
Tonicclonic seizures: Loss of consciousness. Limbs stiffen (tonic), then jerk
(clonic). May have one without the other. Post-ictal confusion and drowsiness.
Myoclonic seizures: Sudden jerk of a limb, face or trunk. The patient may be thrown suddenly to the ground,
or have a violently disobedient limb: one patient described it as my flying-saucer epilepsy, as crockery which
happened to be in the hand would take off .
Atonic (akinetic) seizures: Sudden loss of muscle tone causing a fall, no LOC.
Infantile spasms/West syndrome: Peak age: 5 months. Clusters of head nodding (Salaam attack) and arm
jerks, every 330sec. IQ decrease in ~70%. EEG is characteristic (hypsarrythmia)

246. A man has discharge from his left ear after a fight. Where is the discharge coming from?
a. CSF
b. Inner ear
c. Outer ear
d. Brain
Ans. The key is A. CSF. [probable fracture base of skull]
Ear Discharge & Their Source:
External ear: Inflammation, ie otitis externa produces a scanty watery discharge, as there are no mucinous
glands Blood can result from trauma to the canal. Liquid wax can sometimes leak out.
Middle ear: Mucous discharges are almost always due to middle ear disease.Serosanguinous discharge
suggests a granular mucosa of chronic otitis media. An offensive discharge suggests cholesteatoma.
CSF otorrhoea: CSF leaks may follow trauma: suspect if you see a halo sign on filter paper, or its glucose is
increased , or Beta2 (tau) transferrin is present.
.
247. A 40 yo manic depressive is noted to have high serum levels of lithium and profound
hypokalemia. His GP had started him on anti-HTNs. Choose the single most likely cause?
a. Verapamil
b. Amiodarone
c. Ranitidine
d. Lithium
e. Thiazide
Ans. The key is E. Thiazide. [Thiazide was prescribed for Hpt and when lithium was prescribed its level
increased due to thiazide and thiazide also caused hypokalemia resulting the given picture].

Thiazide Diuretics Side Effects:


A possible increase in blood sugar level.
A possible increase in the level of uric acid. So contra indicated in gout.
It can cause a low blood level of potassium, sodium, and magnesium, and a high level of calcium.
These effects may cause weakness, confusion, and rarely, abnormal heart rhythms to develop.
Other problems, such as:
o Upset stomach.
o Dizziness on standing (due to too low blood pressure).
o Erection problems (impotence) - often reversible on stopping treatment.
o Skin sensitivity to sunlight.

248. A 74yo man presents with weakness in his arm and leg from which he recovered within a few days and
short term memory loss. He has an extensor plantar response. He has similar episodes 2 years ago and
became unable to identify objects and to make proper judgment. What is the most appropriate dx?
a. Alcoholic dementia
b. Picks dementia
c. Huntingtons disease
d. Alzheimers disease
e. Vascular dementia
Ans. The key is E. Vascular dementia. [hemiparesis, memory impairment, extensor plantar reflex, inability to
identify objects, poor judgment are features of vascular dementia].

Vascular dementia: ~25% of all dementias. It represents the cumulative effects of many small strokes, thus
sudden onset and stepwise deterioration is characteristic
Look for evidence of vascular pathology (BP, past strokes, focal CNS signs).

Diagnosis:
Presence of dementia - cognitive decline from higher level of functioning. This can be
demonstrated as memory loss plus impairment in two or more different cognitive domains . This
should be established by clinical examination and neuropsychological testing. Deficits should be
severe enough to interfere with activities of daily living - not secondary effects of the
cerebrovascular event alone.
Cerebrovascular disease, defined by the presence of signs on neurological examination and/or by
brain imaging.
A relationship between the above two disorders inferred by:
o Onset of dementia within three months following a recognised stroke.
o An abrupt deterioration in cognitive functions.
o Fluctuating, stepwise progression of cognitive deficits.

Management: Like other dementias the treatment is symptomatic, addressing the individual's main problems
and supporting the carers. Detecting and addressing cardiovascular risk factors is also very important to try to
slow progression

249. A nurse comes to you saying that she has recently developed the habit of washing her hands after every
15-20 mins. She is unable to conc on her work and takes longer than before to finish tasks as she must
constantly was her hands. What is the most appropriate management?
a. CBT
b. SSRI
c. ECT
d. Antipsychotics
e. Desensitization
Q. 1. What is the key?
Q. 2. What is the diagnosis?
Ans. 1. The key is A. CBT.
Ans. 2. The diagnosis is OCD.

Compulsions are senseless, repeated rituals. Obsessions are stereotyped, purposeless words, ideas, or phrases
that come into the mind.
Repetitive behavior and an urge to do it.
Treatment: CBT is first line. Clomipramine (start with 25mg/day PO) or SSRIs (eg fluoxetine)

250. A 61yo man underwent a surgery in which ileal resection had been done. He complains of
fatigue, headache, and heart racing. Labs: MCV=108fL, Hgb=8.9g/dL. What is the most likely dx?
a. Vit B12 def
b. Iron def
c. Folate def
d. Hemolytic anemia
e. Anemia of chronic disease
Q. 1. What is the key?
Q. 2. What are the points in favour?
Ans. 1. The key is Vit. B12 deficiency.
Ans. 2. Vit B12 is absorbed mostly in ileum. Megaloblastic anaemia.

Sites of minerals and nutrients absorption:


250. A 61yo man underwent a surgery in which ileal resection had been done. He complains of
fatigue, headache, and heart racing. Labs: MCV=108fL, Hgb=8.9g/dL. What is the most likely dx?
a. Vit B12 def
b. Iron def
c. Folate def
d. Hemolytic anemia
e. Anemia of chronic disease
Key : A (Vit B12 def)
Points in favour : typical symptoms of fatigue, headache and palpitations along with favourable age group.
Most important clues lie in Ileal resection (vit b12 absorption occurs in terminal ileum) and secondly
increased MCV(showing macrocytosis).

From symptoms and low HB here we know straight away that the diagnosis here is some kind of anemia. Next
most important thing to look for in anemia questions is the MCV.Normal MCV is between 76-96 fl.
Microcytic (low MCV) = IDA , Thalassemia , Sideroblastic anemias , Anemia of chronic disease.
Normocytic (normal MCV) = Acute blood loss , Anemia of chronic disease, bone marrow failure , Renal failure
, hypothyroidism (or increased mcv), haemolysis (or increased mcv), pregnancy.
Macrocytic (high MCV) = Vit B12 deficiency , Folate deficiency , Alcohol excess or Liver disease,
Myelodysplastic syndromes, Marrow infiltration, hypothyroidism, antifolate drugs.

Next we look for the cause of the anemia in the question which quite obviously in this question is the ileal
resection. Vitamin B12 is absorbed in the terminal ileum once it is attached to the intrinsic factor secreted by
the parietal cells in the stomach. Intrinsic factor is deficient in an an autoimmune disease called Pernicious
anemia. Other causes of Vit B12 deficiency include gastrectomy, gastric resection , atrophic gastritis , long
term H.pylori infection, inadequate intake of vit B 12 (vegan diet), malabsorption, Crohns disease, chronic
tropical sprue, DRUGS ( eg, colchicine, neomycin, metformin, anticonvulsants, long term use of PPIs and H2
receptor blockers).

Ruling out = Except Folate def. other options are easy to rule out. IDA is microcytic and there is no history of
any chronic disease for answer to be D. There is no evidence of haemolysis in the question like jaundice,
retic count and raises LDH etc.
Folate deficiency anemia is mostly due to decreased intake or is pregnancy induced and has nothing to do
with ileum resection.

Topic = Vitamin B12 deficiency and Pernicious anemia


EPI = occurs in all races, peake age is 60, more common in those with blue eyes, early greying, a positive family
history and blood group A. The condition has a female:male ratio of 1.6:1.0.
Presentation = fatigue and lethargy, dyspnoea, faintness, palpitations and headache. Vitamin B12 deficiency
may present with unexplained neurological symptoms - eg, paraesthesia, numbness, cognitive changes or
visual disturbance.
Findings on examination may include pallor, heart failure (if anaemia is severe), lemon tinge to the skin,
glossitis and oral ulceration. Neuropsychiatric features may include irritability, depression, psychosis and
dementia. Neurological features may include subacute combined degeneration of the spinal cord and
peripheral neuropathy.
Investigations = FBC, Blood film , Biochemistry (including serum vit B12 levels and serum Folate
levels),Schilling test and Bone marrow aspiration.
Treatment = For patients with no neurological involvement, treatment is with six injections of
hydroxocobalamin, 1 mg in 1 mL at intervals of between 2-4 days. Subsequently, 1 mg is usually given at
intervals of three months. No NICE guidelines available. Monitory with Vit B12 levels is not recommended so
taper according to severity of symptoms.
For patients with neurological involvement, referral to a haematologist is recommended. Initial treatment is
with hydroxocobalamin 1 mg on alternate days until there is no further improvement, after which 1 mg should
be given every two months for life.
Care should be taken not to give folic acid instead of VitB12 as this may result in fulminant neurological
defcit.
251. A 7yo is brought by his mother who says that he was well at birth but has been suffering from
repeated chest and GI infections since then. She also says that he is not growing well for this age.
What is the likely condition of this child?
a. CF
b. SCID
c. Primary Tcell immunodeficiency
d. Primary Bcell immunodeficiency
e. Malabsorption
Key = A (Cystic fibrosis)
Points in favour = recurrent chest and GI infections and child not growing well.

Epidemiology
CF is the most common inherited disease in white populations.[3]
Prevalence is 1 in 2,500 newborn infants, with calculated carrier frequency of 1 in 25.[4] Just over
10,000 people were recorded as having CF in the 2013 UK CF Registry.[5]
The only risk factor is a family history of the condition.

Pathogenesis
The abnormality in the CFTR gene explains the pathology of CF.

High sodium sweat


Primary secretion of sweat duct is normal but CFTR does not absorb chloride ions, which remain in the lumen
and prevent sodium absorption.

Pancreatic insufficiency
Production of pancreatic enzymes is normal but defects in ion transport produce relative dehydration of
pancreatic secretions, causing their stagnation in the pancreatic ducts.

Biliary disease
Defective ion transfer across the bile duct causes reduced movement of water in the lumen so that bile
becomes concentrated, causing plugging and local damage.

Gastrointestinal disease
Low-volume secretions of increased viscosity, changes in fluid movement across both the small and large
intestine and dehydrated biliary and pancreatic secretions cause intraluminal water deficiency.

Respiratory disease
Dehydration of the airway surfaces reduces mucociliary clearance and favours bacterial colonisation, local
bacterial defences are impaired by local salt concentrations and bacterial adherence is increased by changes in
cell surface glycoproteins.
Increased bacterial colonisation and reduced clearance produce inflammatory lung damage due to an
exuberant neutrophilic response involving mediators such as IL8 and neutrophil elastase.
Presentation
As normal digestive function is possible with <5% pancreatic function, CF can present at any age.
The most common presentation is with respiratory problems - usually recurrent lower respiratory tract
infection (LRTI) with chronic sputum production.
However, immunoreactive trypsinogen (IRT) is now measured on a dried blood spot obtained on the Guthrie
card at day six of life. Samples with abnormally raised IRT levels will undergo CFTR mutation screening. This
was introduced in 2007. This therefore means that clinical presentation of CF will become rarer. However,
screening failures do sometimes occur. Presentation of CF varies with age.
Presentation and diagnosis

Antenatal Amniocentesis/chorionic villus sampling (CVS).


Ultrasound demonstration of bowel perforation/hyperechogenic bowel
(4% cases due to CF).
Perinatal Screening.
Bowel obstruction with meconium ileus (bowel atresia).
Haemorrhagic disease of the newborn.
Prolonged jaundice.
Infancy and childhood Recurrent respiratory infections.
Diarrhoea.
Failure to thrive (thriving does not exclude diagnosis).
Rectal prolapse.
Nasal polyps (in children, nearly always due to CF).
Acute pancreatitis.
Portal hypertension and variceal haemorrhage.
Pseudo-Bartter's syndrome, electrolyte abnormality.
Hypoproteinaemia and oedema.
Adolescence/adulthood Screening.
Recurrent respiratory infections.
Atypical asthma.
Bronchiectasis.
Male infertility with congenital bilateral absence of the vas deferens.
Heat exhaustion/electrolyte disturbance.
Portal hypertension and variceal haemorrhage.

Signs
These may include:
Finger clubbing.
Cough with purulent sputum.
Crackles.
Wheezes (mainly in the upper lobes).
Forced expiratory volume in one second (FEV1) showing obstruction.
Babies diagnosed with CF will usually have no signs or symptoms.

Investigations
Sweat testing confirms the diagnosis and is 98% sensitive. Chloride concentration >60 mmol/L with
sodium concentration lower than that of chloride on two separate occasions.
Molecular genetic testing for CFTR gene.
Sinus X-ray or CT scan - opacification of the sinuses is present in almost all patients with CF.
CXR or CT of thorax.
Lung function testing - spirometry is unreliable before 6 years.
Sputum microbiology - common pathogens include Haemophilus influenzae,Staphylococcus aureus,
Pseudomonas aeruginosa, Burkholderia cepacia, Escherichia coli and Klebsiella pneumoniae.
Various blood tests including FBC, U&Es, fasting glucose, LFTs and vitamin A, D and E levels are usually
performed.
Semen analysis if appropriate.

Management is done by multi disciplinary team and is problem specific as the disease has such a wide
spectrum of abnormalities including multiple systems.

252. A 3yo child has a high temp for 4 days and he had not seen a doctor. Then mother notices rashes
on buccal mucosa and some around the mouth. What is the most appropriate dx?
a. Measles
b. Roseola infectiosum
c. Rubella
d. Chicken pox
e. Impetigo

253. A 70yo lady presents with fever for 3d and confusion. There is no significant PMH. What is the
most probable dx?
a. Delirium
b. Hypoglycemia
c. Alzheimers
d. DKA

Key = A (Delirium)
Points in favour = Old age with acute confusion accompanied by fever for 3 days(causing factor). The fact
that there is no other significant PMH.

Acute/sub-acute confusion in an older patient should be considered delirium until proven otherwise. Almost
possibly any underlying medical condition can be the etiology but most common are infections, drugs and
alcohol withdrawal. Differential may include dementia, Bipolar disorder, depression and functional psychosis.
Depression has its specific set of symptoms such as low mood, sadness, loss of interest etc and although
delirium can present with these features as well along with confusion, for the dx to be depression the
symptoms must be at least 2 weeks long and cause significant impairment.
Bipolar disorder, Dementia and Schizophrenia all have a chronic history with specific symptoms for each one
of them.

Ruling out = DKA is not common in elderly and there is no history of DM.
While hypoglycemia can present with acute confusion but not for 3 days for it to be hypoglycemia
we need more symptoms and an earlier duration of onset.
Alzheimer's has to have a chronic history of dementia and can not present with acute confusion only without
any PMH.
Dx = Delirium
ICD-10 definition : An etiologically nonspecific organic cerebral syndrome characterized by concurrent
disturbances of consciousness and attention, perception, thinking, memory, psychomotor behaviour, emotion,
and the sleep-wake schedule. The duration is variable and the degree of severity ranges from mild to very
severe.

EPI = There is increase in incidence with age reaching to 13.6% in those over 85 years. The prevalence is high
in those with malignancy and HIV and in patients with pre-existing cognitive impairment.

Presentation : The diagnosis of delirium is clinical. The following features may be present:
Usually acute or subacute presentation.
Fluctuating course.
Consciousness is clouded/impaired cognition/disorientation.
Poor concentration.
Memory deficits - predominantly poor short-term memory.
Abnormalities of sleep-wake cycle, including sleeping in the day.
Abnormalities of perception - eg, hallucinations or illusions.
Agitation.
Emotional lability.
Psychotic ideas are common but of short duration and of simple content.
Neurological signs - eg, unsteady gait and tremor.

There are no specific investigations and we can order possibly the whole battery of tests to find the underlying
cause.
Management comprises of supportive and environmental measure along with medical therapy consisting of
antipsychotics such as haloperidol and olanzapine. Then comes the management post-discharge.

Delirium resulting from alcohol discharge is called Delirium tremens and can be treated with
benzodiazepines such as diazepam and chlordiazepoxide.

254. An obese mother suffers from OSAS. Which of the following inv is best for her?
a. ABG
b. Overnight pulse-oximetry
c. Polysomnography
d. EEG

Key = C (Polysomnography)
Points in favour = It is the gold standard investigation for OSAS

In PSG various physiological recordings are taken whilst the patient is asleep overnight. It cosists of at least an
EEG, two elctro-oculograms to measure eye movement and an electromyogram on the chin to monitor muscle
movement.
Overnight pulse-oximetry is a cheaper and more readily available method of diagnosing OSAS but PSG remains
to the best one possible.
ABGs are done to rule out daytime hypoxia or hypercapnia in patients presenting with symptoms of heart
failure. EEG alone is of no diagnostic help.
Dx = Obstructive Sleep Apnea Syndrome
EPI = Incidence is increasing as the incidence of obesity is increasing. Highly prevalent in patients with type 2
DM and related metabolic conditions.

Risk factors include:


Obesity (strongest risk factor).
Male gender.
Middle age (55-59 in men, 60-64 in women).
Smoking.
Sedative drugs.
Excess alcohol consumption.
Family history
Possibly genetic tendency related to jaw morphology.
Obese children - they have a higher prevalence and severity of OSAS.

Presentation : The following may be suggestive OSAS

Impaired concentration.
Snoring.
Unrefreshing sleep.
Choking episodes during sleep.
Witnessed apnoeas.
Restless sleep.
Irritability/personality change.
Nocturia.
Decreased libido.
Excessive daytime sleepiness.

Along with obesity, increased neck circumference and certain craniofacial or pharyngeal abnormalities may be
notes in examination.

Treatment : CPAP remains to be the treatment of choice along with some behaviour modification which
include sleeping posture, smoking and alcohol cessation and weight loss. Oral appliances such as mandibular
advance splits can also be used for mild to moderate OSAS.
Surgery is considered only for patients for whom CPAP and oral appliances have failed.

255. A 28yo business man came to the sexual clinic. He was worried that he has HIV infection. 3 HIV
tests were done and all the results are negative. After a few months, he comes back again and
claims that he has HIV. What is the dx?
a. Somatization
b. Hypochondriac
c. Mancheusens
d. OCD
e. Schizophrenia

Key = B (Hypochondriasis)
Points in favour = Belief/delusion of having a serious/life threatening disease which persists after being
ruled out by investigations.

Somatoform disorders consists of a group psychiatric illnesses in which patients either feel or make up signs
and symptoms without existence of an organic disease. It includes the following disorders :
Somatization disorder : Characterised by occurrence of chronic multiple somatic symptoms for which there is
no physical cause.
Hypochondriasis : Patients have a strong fear or belief that they have a serious often fatal disease that persists
despite appropriate medical reassurance. (like in this case).
Body dysmorphic disorder : A preoccupation with bodily shape or appearance with belief that one is
disfigured in some way.
Dissociative (conversion) disorder : Characterised by loss or distortion of neurological function not fully
explained by organic disease. Previously known as Hysteria.
Somatoform Pain disorder: Severe, persistent pain which cannot be explained by medical condition.

Munchausens syndrome : Severe chronic form of factitious disorder usually older males who travel widely
sometimes in several hospitals in one day. They are convincing enough to persuade doctors to undertake
investigations but no underlying condition is found. Previous similar hospital visits can be traced.

This cannot be Schizophrenia because of absence of specific symptom featuring in schizophrenia.

Management : Cognitive behavioural therapy is the main course in management. Antipsychotics may help but
main line of treatment remains to be reassurance and CBT.

256. A 6wk child presents with progressive cyanosis, poor feeding, tachypnea over the first 2 wks of
life and holosystolic murmur. What is the most appropriate condition?
a. ASD
b. VSD
c. Tricuspid atresia
d. PDA
e. TOF
257. A 29yo woman who was dx to have migraine presents with severe onset of occipital headache.
She lost her consciousness. CT=normal. Neurological exam=normal. What is the most
appropriate management?
a. Repeat CT
b. MRI
c. LP
d. XR
e. No inv required

Key = E (no investigation required)


Points in favour = occipital headache is common in migraine. Normal CT and neuro exam means there is no
need for any further investigations.
Loss of consciousness in patients with migraine is an alarming sign and should prompt for imaging straight
away. A normal CT and no focal neurological deficit on examination reassure that there is no new pathology.
The young age of the patient gives us a clue as well. So there is no need of any intervention in this scenario.

Dx = Migraine
Classification = Migraine with aura, Migraine without aura and Chronic migraine
EPI = Common in women than men (vice versa in children)
First attack often in childhood with over 80% having their first attack before the age of 30. If onset is at
age over 50 other pathology should be sought.
Family history is usually positive.
Severity decreases with advancing age.

Presentation : Typically the headaches last between 4 and 72 hours and have at least two of the following
features:
Unilateral.
Pulsating.
Moderate or severe intensity of pain.
Aggravated by, or resulting in the avoidance of, routine physical activity.
In addition, there is at least one of:
Nausea and vomiting during migraine attacks. These are common symptoms that affect at least 60% of
patients suffering from migraines.
Photophobia and phonophobia, which are also very common.

Aura is highly variable in nature (visual,sensory etc) but tends to be consistent for an individual. Headache
starts maximum within one hour of an aura.

Investigations : Diagnosis is purely clinical. Investigations are only done to rule out secondary causes of
headache or when alarming symptoms like loss of consciousness, seizure. memory loss etc present.

Treatment : Acute attack = Step one = Simple analgesics with or without anti emetics
Step two = Anal analgesics and anal anti emetics
Step three = Triptans or ergotamine ( Patients having moderate to severe migraine
should be moved directly to step 3 ).
Prophylaxis = The NICE guidelines and CKS suggests the following indications for prophylaxis :
Frequent attacks are two or more attacks per month that produce disability lasting for three days or
more.
Medication overuse is a risk when medication is used on more than two days per week on a regular
basis. Overuse needs to be addressed before further treatment can begin.
Prophylaxis should be used when standard analgesia and triptans are either contra-indicated or
ineffective.
Beta blockers, amitriptyline, sodium valproate and Botulinum toxin A all can be used as drugs for
prophylaxis. Identifying and refraining from triggering factor (if any) can also be used to avoid attacks such
as stress, dietary factors like cheese,chocolate,alcohol etc.

258. A 19yo man has been happier and more positive than usual, with more energy than he has ever
felt before for no particular reason. He has been getting more work done at the office today and
has been socializing with his friends as usual. What is the most likely dx?
a. Atypical depression
b. Marked depression
c. Bipolar syndrome
d. Psychosis
e. Hypomania
Key = E
Points in favour = There is elevation of mood and energy, increase in activity but not to severity of Mania
and there is no evidence of delusions or hallucinations.

Hypomania is a milder form of mania. There is elevation of mood and energy , increase in activity and
socializing, inability to concentrate and flight of ideas but without hallucinations or delusions.
Mania on the other hand has more severe symptoms like pressured speech, grandiosity, increase in sexual
activity, insomnia, flight of ideas, psychomotor agitation along with hallucinations and delusions.
For the dx of Bipolar disorder there needs to be a documented episode of depression along with mania
(bipolar 1) or hypomania (bipolar 2).
Depression has opposite symptoms of mania like low mood and energy , low activity and interest etc.
Manic episodes can be treated by atypical antipsychotics like olanzapine and risperidone.
Lithium is used but it has slower onset of action. Lorazepam can be used to sedate and decrease agitation.
Carbamazepine is sometimes used as first line treatment.

259. A 35yo female attempts suicide 10x. There is no hx of psychiatric problems and all neurological
exams are normal. What is the best tx?
a. Problem focused tx
b. CBT
c. Antipsychotic
d. Antidepressant
e. ECT

260. A 57yo man presents with weight loss, tiredness, fever and abdominal discomfort. Exam: spleen
palpable up to the umbilicus. Labs: WBC=127, Hgb=8.7, Plt=138. What is the most likely dx?
a. CML
b. AML
c. CLL
d. AML
e. Polycythemia

Key = A (CML)
Points in favour = Splenomegaly and raised WBC along with typical signs and symptoms.

261. A baby born at 34 weeks with a heart murmur is kept in the incubator for almost 4 weeks. There
is no murmur at discharge. What is the likely cause of this murmur?
a. PDA
b. TOF
c. Aneurysm of sinus of Valsalva
d. Aorto-pulmonary septal defect
e. AVM

Key = A (PDA)
Points in favour = PDA can be found in pre mature babies which closes after birth.

The ductus arteriosus is, in developmental terms, a remnant of the sixth aortic arch and connects the
pulmonary artery to the proximal descending aorta just after the left subclavian artery origin. It is a normal
structure in fetal life.
In utero the lungs are not expanded. Gas exchange occurs at the placenta and only about 10% of the
circulation passes through the lungs. The ductus arteriosus connects the pulmonary artery to the aorta to
shunt most of the blood away from the lungs. After delivery it closes and the blood passes through the opened
lungs. Failure of the ductus arteriosus to close can lead to overloading of the lungs. The shunt is left to right
unless pulmonary hypertension occurs and pulmonary pressure exceeds systemic pressure.
After birth the ductus closes functionally in 12-18 hours and anatomically in 2-3 weeks. If it remains open
beyond three months of life in preterm infants and beyond one year of life in full-term infants it is termed as
persistent patency of ductus arteriosus because the incidence of spontaneous closure beyond these time
limits is very low.
Murmurs in all other cases will persist.

PDA:
History : Patients with a small PDA are usually asymptomatic.A large-shunt PDA may cause lower respiratory
tract infection as well as feeding difficulties and poor growth during infancy, with failure to thrive because of
heart failure.
Examination :
If the pulmonary circulation is markedly overloaded there will be tachycardia, tachypnoea and a wide
pulse pressure.
The precordium is hyperactive and a systolic thrill may be present at the upper left sternal border.
The first heart sound is normal but the second is often obscured by the murmur.
A grade 1 to 4/6 continuous ('machinery') murmur is best audible at the left infraclavicular area or
upper left sternal border.
In the case of a large PDA shunt, a diastolic mitral rumble may be heard because of the high flow rate
across the mitral valve.
Patients with a small PDA do not have the above-mentioned findings.
Peripheral pulses are bounding as the run-off into the pulmonary circulation drops the diastolic
pressure and causes a wide pulse pressure.
Investigations : ECG, CXR and Echocardiography

Management : Indometacin is ineffective in term infants with PDA and should not be used. Medical
management is limited to use of decongestive measures such as diuretics in those with features of heart
failure.
PDA closure is indicated for any symptomatic infant, child or adult (with exclusion of those with fixed
high pulmonary vascular resistance). Closure is also indicated in asymptomatic patients with left heart
volume load. This can be done either by surgery or interventional techniques at any age.[4]
Surgical closure is reserved for patients in whom a non-surgical closure technique is not considered
applicable. In infants with heart failure or pulmonary hypertension, surgery is performed on an urgent
basis. The standard surgical procedure is ligation and division of the ductus through left posterolateral
thoracotomy without cardiopulmonary bypass.[3]
In asymptomatic well infants current practice is to wait until 1 year of age, with regular
echocardiographic evaluation to check for spontaneous closure of the PDA. If the duct is still patent at
1 year of age it can be closed usually by occlusion at cardiac catheterisation (endovascular occlusion).
National Institute for Health and Care Excellence (NICE) guidance has been produced and considers
that current evidence on the safety and efficacy of endovascular occlusion of PDA appears to support
the use of this procedure.[5] The procedure should be performed in units where there are
arrangements for cardiac surgical support in the event of complications. The choice of device depends
largely on the size of PDA. Coils are suitable for closing of small- to medium-sized PDAs while larger
PDAs require other devices such as the Amplatzer patent ductus arteriosus device.
Serious complications of transcatheter closure of PDA are rare and include device embolisation,
femoral artery or vein thrombosis related to vascular access and infection.[4]
Whilst the ductus arteriosus is patent then the risk of endocarditis should be considered (there is no
increased risk of endocarditis once repair is complete). Routine antibiotic prophylaxs is not indicated
but during invasive procedures (eg, urinary or gastrointestinal procedures) involving areas of sepsis,
suitable antibiotics should be given promptly (to cover all the likely organisms, including any known to
cause endocarditis).[6]

262. A 6yo girl who has previously been well presented with a hx of tonic-clonic seizures lasting
4mins. Her mother brought he to the hospital ad she appeaed ell. She is afeile ad didt
lose consciousness during the episode of seizure. She has no neurologic deficit. What is the most
appropriate inv for her?
a. ABG
b. Serum electrolytes
c. ECG
d. Blood glucose
263. A 60yo woman was found by her son. She was confused and had urinary incontinence. She has
recovered fully after 6h with no neurological complaints. What is the most likely dx?
a. Stroke
b. Vestibular insufficiency
c. TIA
d. Intracranial hemorrhage

Key = C (TIA)
Points in favour = Complete recovery in less than 24 hours
A transient ischaemic attack (TIA) is a temporary inadequacy of the circulation in part of the brain (a cerebral
or retinal deficit) that gives a clinical picture similar to a stroke except that it is transient and reversible. Hence,
TIA is a retrospective diagnosis. The duration is no more than 24 hours and a deficit that lasts longer than 24
hours is defined as a stroke. The majority of TIAs last for less than 30 minutes.

Management
Secondary prevention (see below) includes the use of antiplatelet therapy, antihypertensive, and lipid-
modifying treatments, the management of atrial fibrillation if present and the management of any other
underlying or risk factors, including diabetes.

Driving[4]
Group 1 (car or motorcycle)
Must not drive for one month.
No need to notify DVLA after a single TIA.
Multiple TIAs over a short period: require three months free from further attacks before resuming
driving and DVLA should be notified.
Group 2 (lorry or bus)
Licence refused or revoked for one year following a stroke or TIA.
Assessment of the risk of stroke
An ABCD2 score of more than 4 suggests high risk of an early stroke.[5][6]

Scoring System for Risk of Stroke after TIA (ABCD2 Score)

Age Age >60 1

Blood pressure BP>140 systolic and/or >90 diastolic 1

Clinical features Unilateral weakness 2

Speech disturbance without weakness 1

Other 0

Duration of symptoms >60 minutes 2

10-59 minutes 1

<10 minutes 0

Diabetes Presence of diabetes 1

RCP recommendations[1]
All patients with a TIA should be seen by a specialist in neurovascular disease (eg, in a specialist
neurovascular clinic or an acute stroke unit).
People with a suspected TIA should be assessed as soon as possible for their risk of subsequent stroke
by using a validated scoring system such as ABCD2 (as above).
Patients with suspected TIA who are at high risk of stroke (eg, an ABCD2 score of 4 or above) should
receive:
o Aspirin or clopidogrel (each as a 300 mg loading dose and then 75 mg daily) and a statin started
immediately.
o NB: clopidogrel is not licensed for the management of TIA and therefore the National Institute
for Health and Care Excellence (NICE) and the British National Formulary (BNF) recommend
aspirin plus modified-release dipyridamole.[7]
o Specialist assessment and investigation within 24 hours of onset of symptoms.
o Measures for secondary cardiovascular prevention introduced as soon as the diagnosis is
confirmed, including discussion of individual risk factors.
People with crescendo TIA (two or more TIAs in a week), atrial fibrillation or those on anticoagulants
should be treated as being at high risk of stroke even if they may have an ABCD2 score of 3 or below.
Patients with suspected TIA who are at low risk of stroke (eg, an ABCD2 score of 3 or below) should
receive:
o Aspirin or clopidogrel (each as a 300 mg loading dose and then 75 mg daily) and a statin.
o NB: clopidogrel is not licensed for the management of TIA and therefore NICE and the BNF
recommend aspirin plus modified-release dipyridamole.[7]
o Specialist assessment and investigations as soon as possible, but definitely within one week of
onset of symptoms.
o Measures for secondary prevention introduced as soon as the diagnosis is confirmed, including
discussion of individual risk factors.
People who have had a TIA but present late (more than one week after their last symptom has
resolved) should be treated as though they are at a lower risk of stroke.
Patients with TIA in atrial fibrillation should be anticoagulated in the TIA clinic once intracranial
bleeding has been excluded and if there are no other contra-indications.
If the patient is in atrial fibrillation, management of that condition is required. In persistent atrial fibrillation
there is benefit from anticoagulation but there is no evidence of any such benefit in the absence of atrial
fibrillation.
All people with TIA, who after specialist assessment are considered candidates for carotid endarterectomy,
should have carotid imaging conducted urgently to facilitate carotid surgery, which should be undertaken
within seven days of the onset of symptoms.

264. A 34yo woman presents 3 weeks after childbirth. She has had very low mood and has been
suffering from lack of sleep. She also has thought of harming her little baby. What is the most
appropriate management for this pt?
a. ECT
b. CBT
c. IV haloperidol
d. Paroxethine
e. Amitryptiline

Key = A (ECT)
Points in favour = This is postpartum psychosis since the onset is more than 2 weeks after delivery. There is
evidence of depression and psychosis so ECT should be done.
265. A 65yo woman presents with headache. She also complains of dizziness and tinnitus. He has
recently realized she has visual problems. There is hx of burning sensation in fingers and toes.
On exam: splenomegaly, itchy after hot bath. Labs: RBC=87, Hgb=31.9, Plt=796. What is the dx?
a. CML
b. CLL
c. Polycythemia vera
d. Myelofibrosis
e. NHL

Key = C (Polycythemia vera)


Points in favour = Headache, burning finger along with visual problems. High hb
266. A 29yo male brought to ED in conscious state. There is no significant past hx. Which of the
following should be done as the initial inv?
a. CT
b. Blood glucose
c. ABG
d. MRI
e. CBC

Key = B (blood glucose)


Points in favour = Always check for hypoglycemia in unconscious patient first especially young patients with
no significant past history.
Assessing BSR levels is much easier and less invasive than other tests and quicker to rule out.

Diagnostic criteria
The World Health Organization reclassified chronic myeloproliferative diseases as myeloproliferative
neoplasms in 2008.[4] The criteria for the diagnosis of PRV requires two major criteria and one minor
criterion, or the first major criterion and two minor criteria.
Major criteria:
o Haemoglobin of more than 18.5 g/dL in men, 16.5 g/dL in women, or elevated red cell mass
greater than 25% above mean normal predicted value.
o Presence of JAK2 617V F mutation or other functionally similar mutations, such as the exon 12
mutation of JAK2.
Minor criteria
o Bone marrow biopsy showing hypercellularity with prominent erythroid, granulocytic, and
megakaryocytic proliferation.
o Serum erythropoietin level below normal range.
o Endogenous erythroid colony formation in vitro.
Other confirmatory findings no longer required for diagnosis include:
o Oxygen saturation with arterial blood gas greater than 92%.
o Splenomegaly.
o Thrombocytosis (>400,000 platelets/mm3).
o Leukocytosis (>12,000/mm3).
o Leukocyte alkaline phosphatase (>100 units in the absence of fever or infection).

Investigation
Initial blood tests:
o FBC in PCV will show not only elevated Hb and packed cell volume but WCC and platelets will be
elevated too. In secondary polycythaemia only red blood cells are raised.
o Ferritin is often low in primary polycythaemia because of increased demand for iron. In
secondary causes it is usually normal..
Radiology:
o Radioisotopes can be used to measure circulating volumes. Red cells can be labelled with 51Cr
and albumin with 131I. This is expensive, needs skill and is not widely available.
o CT, MRI or ultrasound scanning of the abdomen may show enlargement of the spleen as is
often found in PRV. It should also check for abnormalities of the renal system.
Bone marrow and aspirate:
o Tend to be hypercellular in PRV.
o In the plethoric phase, the blood smear shows normal erythrocytes, variable neutrophilia with
myelocytes, metamyelocytes, and varying degrees of immaturity, basophilia, and increased
platelet counts.
o In the spent phase, the blood smear shows abundant teardrop cells, leukocytosis, and
thrombocytosis.
o Generally the findings are not specific to PRV. The bone marrow can be normal in PRV.
Serum erythropoietin levels are often low in PRV. This can differentiate secondary erythrocytosis and
pseudoerythrocytosis from PRV, but there is overlap in the levels found and it cannot reliably
differentiate.
Cytogenetic studies. Karyotyping can detect fewer than 30% of patients with PRV. An abnormal test is
useful, but a normal test does not exclude PRV.
Clonal assays (using glucose-6-phosphate dehydrogenase (G6PD) markers) are not generally available
for clinical use. Even if it were available it is only of use in female patients.
Research markers include the thrombopoietin receptor MPL expression and the PRV1 mRNA in
granulocytes.[5]

JAK2 testing
With the development of new techniques for detecting the Janus kinase 2 (JAK2) V617F mutation this may
become a clinically useful marker for PRV. It has been recommended as a diagnostic marker.[6][5]
JAK2-positive polycythaemia vera is diagnosed if:[2]
The JAK2 mutation is identified; and
The haematocrit is more than 0.48 in women or more than 0.52 in men, or the red cell mass is 25%
higher than normal.
JAK2-negative polycythaemia vera is diagnosed if:[2]
The JAK2 mutation is not identified; and
The haematocrit is more than 0.56 in women or more than 0.60 in men, or the red cell mass is 25%
higher than normal; and
There is no identifiable secondary cause for polycythaemia; and either
o There is palpable splenomegaly or the presence of an acquired genetic abnormality in the
haematopoietic stem cells or both; or
o Any two of the following clinical features are identified: an abnormally increased platelet count,
an abnormally increased neutrophil count, radiological evidence of splenomegaly, and
abnormally low serum erythropoietin.

Management
The main concern with the management of the disease is the prevention of thrombosis, which is the main
cause of morbidity and mortality. Fibrotic and leukaemic disease also raises mortality and morbidity.
Intermittent long-term phlebotomy to maintain the haematocrit below 45% (lower target level may be
appropriate for women). Phlebotomy may cause progressive and sometimes severe thrombocytosis
and iron deficiency. Splenomegaly and pruritus may persist despite control of the haematocrit by
phlebotomy.[7]
Low-dose aspirin produces a small reduction in thrombotic events, including myocardial infarction and
stroke, whilst not increasing the risk of haemorrhage.[8][9]
If it is not possible to control thrombotic events with phlebotomy alone then myelosuppression must
be considered. However, this is not without risk and increases the risk of leukaemic transformation.
Risks and benefits have to be balanced.
Chemotherapy options include:[2]
o For people younger than 40 years of age: first-line is interferon; second-line is
hydroxycarbamide or anagrelide.
o For people 40-75 years of age: first-line is hydroxycarbamide; second-line is interferon or
anagrelide.
o For people older than 75 years of age: first-line is hydroxycarbamide; second-line is radioactive
phosphorus or busulfan.
Pruritus can be quite disabling:
o Taking baths or showers at lower temperatures and patting the skin dry, to avoid rubbing, may
help.
o Antihistamines, including H2 receptor antagonists (H2RAs), are useful in refractory cases.
o Selective serotonin reuptake inhibitors (SSRIs) - eg, paroxetine or fluoxetine.
Elevated uric acid may require allopurinol.
It may be necessary to consider splenectomy when there is painful splenomegaly or there are repeated
episodes of splenic infarction.

267. A 45yo woman comes with red, swollen and exudating ulcer on the nipple and areola of right
breast with palpable lump under the ulcer. What do you think is nthis skin condition?
a. Inflammatory cells releasing cytokines
b. Infiltration of the lymphatics by the carcinomatous cells
c. Infiltration of the malignant skin cells to the breast tissue

Key = B
Points in favour = This is a case of CA breast in which infiltration of the lymphatics cause the ulceration.
268. A 20yo young lady comes to the GP for advice regarding cervical ca. she is worried as her mother
past away because of this. She would like to know what is the best method of contraception in
her case?
a. POP
b. Barrier method
c. IUCD
d. COCP
e. IUS

Key = B (barrier method)


Points in favour = barrier method can help prevent catching HPV infection which is the main etiology behind
CA cervix. Other methods may provide with better contraception but are not good means of preventing hpv
infections.
269. A 66yo man, an hour after hemicolectomy has an urine output of 40ml. However, an hour after
that, no urine seemed to be draining from the catheter. What is the most appropriate next step?
a. IV fluids
b. Blood transfusion
c. Dialysis
d. IV furosemide
e. Check catheter
Key = E
Points in favour = Always check catheter for any obstruction or other abnormality before iv fluids.
270. A 24yo pt presented with anaphylactic shock. What would be the dose of adrenaline?
a. 0.5ml of 1:1000
b. 0.5ml of 1:10000
c. 1ml of 1:500
d. 5ml of 1:1000
e. 0.05ml of 1:100

Key = A
271. A 44yo woman complains of heavy bleeding per vagina. Transvaginal US was done and normal.
Which of the following would be the most appropriate inv for her?
a. Hysterectomy
b. Endometrial biopsy
c. CBC
d. High vaginal swab
e. Coagulation profile
Key = E (coagulation profile)
Points in favour = After normal vaginal US coagulation profile should be done to rule out systemic causes of
heavy bleeding first. Endometrial biopsy will be needed if ultrasound shows some endometrial abnormality.
Before considering steps like hysterectomy, systemic causes of bleeding must be ruled out by checking
coagulation profile.
CBC and high vaginal swab will not help much in finding the cause of bleeding.
272. A 60yo woman presented to OPD with dysphagia. No hx of weight loss of heartburn. No change
in bowel habits. While doing endoscopy there is some difficulty passing through the LES, but no
other abnormality is noted. What is the single most useful inv?
a. CXR
b. MRI
c. Esophageal biopsy
d. Esophageal manometry
e. Abdominal XR

Key = D
Points in favour = This can be a case of achalasia or esophageal spasms. In both cases manometry is the gold
standard investigation. Another investigation which could have been done before manometry and even
endoscopy is The Barium Swallow.
Remaining inx given in the question will not help is in the diagnosis.

Achalasia is primarily a disorder of motility of the lower oesophageal or cardiac sphincter. The smooth muscle
layer of the oesophagus has impaired peristalsis and failure of the sphincter to relax causes a functional
stenosis or functional oesophageal stricture. Most cases have no known underlying cause, but a small
proportion occurs secondary to other conditions - eg, oesophageal cancer.
It tends to present in adult life and is very rare to present in children.

Presentation
The most common presenting feature is dysphagia. This affects solids more than soft food or liquids.
Regurgitation may occur in 80-90% and some patients learn to induce it to relieve pain.
Chest pain occurs in 25-50%. It occurs after eating and is described as retrosternal. It is more prevalent
in early disease.
Heartburn is common and may be aggravated by treatment.
Loss of weight suggests malignancy (may co-exist).
Nocturnal cough and even inhalation of refluxed contents is a feature of later disease.
Examination is unlikely to be revealing although loss of weight may be noted. Rarely, there may be
signs of an inhalation pneumonia.

Treatment : Calcium channel blockers and nitrates can be used.


Pneumatic dilatation or endoscopic botulinum toxin injection can be used as well.
Heller myotomy remains to be best treatment of choice in patients who are fit for surgery.

273. A 24yo woman presents with deep dyspareunia and severe pain in every cycle. What is the
initial inv?
a. Laparoscopy
b. Pelvic US
c. Hysteroscopy
d. Vaginal Swab

Key = B (Pelvis ultrasound)


Points in favour = To rule out cervical abnormalities, endometriosis. ovarian cysts etc.
274. A 38yo woman, 10d postpartum presents to the GP with hx of passing blood clots per vagina
since yesterday. Exam: BP=90/40mmhg, pulse=110bpm, temp=38C, uterus tender on palpation
and fundus 2cm above umbilicus, blood clots +++. Choose the single most likely dx/
a. Abruption of placenta 2nd to pre-eclampsia
b. Concealed hemorrhage
c. Primary PPH
d. Secondary PPH
e. Retained placenta
f. Scabies
275. A 32yo female with 3 prv 1st trimester miscarriages is dx with antiphospholipid syndrome.
Anticardiolipin
antibodies +ve. She is now 18wks pregnant. What would be the most appropriate
management?
a. Aspirin
b. Aspirin & warfarin
c. Aspirin & heparin
d. Heparin only
e. Warfarin only

Key = C
Points in favour = More than 3 prev miscarriages due to APLS - LMWH plus aspirin throughout pregnancy is
indicated.

Antiphospholipid syndrome (APS) is an autoimmune disorder characterised by arterial and venous thrombosis,
adverse pregnancy outcomes (for mother and fetus), and raised levels of antiphospholipid (aPL) antibodies.

Presentation:
APS has varied clinical features and a range of autoantibodies. Virtually any system can be affected,
including:[1][4][5]
Peripheral artery thrombosis, deep venous thrombosis.
Cerebrovascular disease, sinus thrombosis.
Pregnancy loss: loss at any gestation - recurrent miscarriage or prematurity can be seen in APS.
Pre-eclampsia, intrauterine growth restriction (IUGR).
Pulmonary embolism, pulmonary hypertension.
Livedo reticularis (persistent violaceous, red or blue pattern of the skin of the trunk, arms or legs; it
does not disappear on warming and may consist of regular broken or unbroken circles), purpura, skin
ulceration.
Thrombocytopenia, haemolytic anaemia.
Libman-Sacks endocarditis and cardiac valve disease:
o Usually mitral valve disease or aortic valve disease and usually regurgitation with or without
stenosis.
o Mild mitral regurgitation is very common and is often found with no other pathology. There
may also be vegetations on the heart and valves.
Myocardial infarction.
Retinal thrombosis.
Nephropathy: vascular lesions of the kidneys may result in chronic kidney disease.
Adrenal infarction.
Avascular necrosis of bone.

Investigations
Young adults (50 years old) with ischaemic stroke and women with recurrent pregnancy loss (3 pregnancy
losses) before 10 weeks of gestation should be screened for aPL antibodies.[3]
Levels of aCL, anti-beta2 GPI or lupus anticoagulant (LA) on two occasions at least 12 weeks apart.
FBC; thrombocytopenia, haemolytic anaemia.
Clotting screen.
CT scanning or MRI of the brain (cerebrovascular accident), chest (pulmonary embolism) or abdomen
(Budd-Chiari syndrome).
Doppler ultrasound studies are recommended for possible detection of deep vein thrombosis.
Two-dimensional echocardiography may demonstrate asymptomatic valve thickening, vegetations or
valvular insufficiency.

Management in Pregnancy :
APS in pregnancy may affect both mother and fetus throughout the entire pregnancy and is associated with
high morbidity. Clinical complications are variable and include recurrent miscarriage, stillbirth, IUGR and
pre-eclampsia.
For women with APS with recurrent (3) pregnancy loss, antenatal administration of low molecular
weight heparin combined with low-dose aspirin is recommended throughout pregnancy. Treatment
should begin as soon as pregnancy is confirmed.
For women with APS and a history of pre-eclampsia or IUGR, low-dose aspirin is recommended.
Women wit aPL antibodies should be considered for postpartum thromboprophylaxis.

276. A 23yo presents with vomiting, nausea and dizziness. She says her menstrual period has been
delayed 4 weeks as she was stressed recently. There are no symptoms present. What is the next
appropriate management?
a. Refer to OP psychiatry
b. Refer to OP ENT
c. CT brain
d. Dipstick for B-hCG

Key = D
Points in favour = Test for pregnancy first in case of amenorrhea
e. MRI brain
277. A 16yo girl came to the sexual clinic. She complains of painful and heavy bleeding. She says she
doest a egula le. What is the ost appropriate management?
a. Mini pill
b. Combined pill
c. IUS
d. Anti-prostoglandins
e. Anti-fibrinolytics

278. A 36yo man walks into a bank and demands money claiming he owns the bank. On being denied,
he goes to the police station to report this. What kind of delusions is he suffering from?
a. Delusion of reference
b. Delusion of control
c. Delusion of guilt
d. Delusion of persecution
e. Delusion of grandeur

Key = E
Points in favour = Delusion of grandeur is defined as delusion of exaggerated self worth. Hence the answer.
279. Which method of contraception can cause the risk of ectopic pregnancy?
a. COCP
b. IUCD
c. Mirena
d. POP
Key = B (IUCD)

280. A woman has pernicious anemia. She has been prescribed parenteral vitamin B12 tx but she is
needle phobic. Why is oral tx not preferred for this pt?
a. IM B12 is absorbed more
b. Intrinsic factor deficiency affects oral B12 utilization
c. IM B12 acts faster
d. IM B12 needs lower dosage
e. Pernicious anemia has swallowing difficulties

Key = B
Points in favour = There is def of intrinsic factor dt autoimmune causes in pernicious anemia. VitB12 can not
be absorbed without binding to intrinsic factor.Refer to the explanation of answer to question number 250.
281. An old man comes to the doctor complaining that a part of this body is rotten and he wants it
removed. What is the most likely dx?
a. Guilt
b. Hypochondriasis
. Muhauses
d. Nihilism
e. Capras syndrome
282. A 31yo woman who is 32weeks pregnant attends the antenatal clinic. Labs: Hgb=10.7, MCV=91.
What is the most appropriate management for this pt?
a. Folate supplement
b. Ferrous sulphate 200mg/d PO
c. Iron dextran
d. No tx req
Key = D (no tx required)
Explanation : This is dilutional anemia hence no treatment required.
283. A 47yo man who is a chronic alcoholic with established liver damage, has been brought to the
hospital after an episode of heavy drinking. His is not able to walk straight and is complaining of
double vision and is shouting obscenities and expletives. What is the most likely dx?
a. Korsakoff psychosis
b. Delirium tremens
c. Wernickes encephalopathy
d. Tourettes syndrome
e. Alcohol dependence
Key = C (Wernickes encephalopathy)
Points in favour = Chronic alcoholic - thiamine deficiency - double vision, unable to walk.

In case of delirium tremens , there is history of alcohol consumption around 42-72 hours back. There are
symptoms like hallucination, confusion and severe agitation and sometimes seizures as well.
There is history of vocal or neurological tics for it to be tourettes syndrome.
Korsakoffs syndrome is a late complication of untreated Wernickes. They are both together known as
wernicke-korsakoff syndrome.

Wernicke-korsakoff syndrome:- Wernicke-Korsakoff syndrome (WKS) is a spectrum of disease resulting from


thiamine deficiency, usually related to alcohol abuse.

Presentation :
Vision changes:
o Double vision
o Eye movement abnormalities
o Eyelid drooping
Loss of muscle co-ordination:
o Unsteady, unco-ordinated walking
Loss of memory, which can be profound.
Inability to form new memories.
Hallucinations.
Examination of the nervous system may show polyneuropathy.
Reflexes may be decreased (or of abnormal intensity), or abnormal reflexes may be present.
Gait and co-ordination are abnormal on testing.
Muscles may be weak and may show atrophy.
Eyes show abnormalities of movement - nystagmus, bilateral lateral rectus palsy and conjugate gaze
palsy.
Blood pressure and body temperature may be low.
Pulse may be rapid.
The person may appear cachectic.
Confabulation
Memory loss
Retrograde amnesia

Encephalopathy
At least two of the four following criteria should be present to diagnose encephalopathy:[7]
Dietary deficiencies.
Oculomotor abnormalities.
Cerebellar dysfunction.
Either an altered mental state or mild memory impairment.

Treatment :
Thiamine orally (IM or IV may be used in secondary care) plus vitamin B complex or multivitamins,
which should be given indefinitely. Treatment with thiamine is often started under specialist care,
although when deficiency is suspected, it should be started in primary care.
Offer oral thiamine to harmful or dependent drinkers if either of the following applies:[9]
o They are malnourished (or have a poor diet); prescribe oral thiamine 50 mg per day (as a single
dose) for as long as malnutrition may be present.
o They have decompensated liver disease.
A Cochrane review found there was insufficient evidence from randomised controlled clinical trials to
guide clinicians in the dose, frequency, route or duration of thiamine treatment of WKS due to alcohol
abuse.[10] However, more recent work states that the route of administration and dose depend on the
severity of dependence and overall physical health of the patient.[11]
Although potentially serious allergic adverse reactions may (rarely) occur during, or shortly after,
parenteral administration, the Commission on Human Medicines has recommended that:[12]
o This should not preclude the use of parenteral thiamine in patients where this route of
administration is required, particularly in patients at risk of WKS where treatment with
thiamine is essential.
o IV administration should be by infusion over 30 minutes.
o Facilities for treating anaphylaxis (including resuscitation facilities) should be available when
parenteral thiamine is administered.

284. A 32yo woman of 39wks gestation attends the antenatal day unit feeling very unwell with
sudden onset of epigastric pain a/w nausea and vomiting. Temp 36.7C. Exam: RUQ tenderness.
Bloods: mild anemia, low plts, elevated LFT and hemolysis. What is the most likely dx?
a. Acute fatty liver of pregnancy
b. Acute pyelonephritis
c. Cholecystitis
d. HELLP syndrome
e. Acute hepatitis
Key = D (HELLP syndrome)
Points in favour = hemolysis, elevated LFTs and low platelets
Presentation
HELLP syndrome is a serious form of pre-eclampsia and patients may present at any time in the last
half of pregnancy.
One third of women with HELLP syndrome present shortly after delivery.
Symptoms of HELLP syndrome are usually nonspecific.
Initially, women may report nonspecific symptoms including malaise, fatigue, right upper quadrant or
epigastric pain, nausea, vomiting, or flu-like symptoms.
Hepatomegaly can occur.
Some women may have easy bruising/purpura.
On examination, oedema, hypertension and proteinuria are present.
Tenderness over the liver can occur.

Investigations
There needs to be a high index of clinical suspicion in order to avoid diagnostic delay and improve
outcome.
Haemolysis with fragmented red cells on the blood film
Raised LDH >600 IU/L with a raised bilirubin.
Liver enzymes are raised with an AST or ALT level of >70 IU/L.
Levels of AST or ALT >150 IU/L are associated with increased maternal morbidity and mortality.

Management
The main treatment is to deliver the baby as soon as possible, even if premature, since liver function in
the mother gets worse very quickly.
Problems with the liver can be harmful to both mother and child.
Definitive treatment of HELLP syndrome requires delivery of the fetus and is advised after 34 weeks of
gestation if multisystem disease is present.
There is no clear evidence of any effect of giving corticosteroids on clinical outcomes for women with
HELLP syndrome.[2]
Transfusion of red cells, platelets, fresh frozen plasma and cryoprecipitate or fibrinogen concentrate
are required as indicated clinically and by blood and coagulation tests.
Postpartum HELLP syndrome may be treated with steroids and plasma exchange.
If the fetus is less than 34 weeks of gestation and delivery can be deferred, corticosteroids should be
given.
Blood pressure control is very important.
Women with severe liver damage may need liver transplantation.

285. A 57yo woman presents with dysuria, frequency and urinary incontinence. She complains of
dyspareunia. Urine culture has been done and is sterile. What is the most appropriate step?
a. Oral antibiotics
b. Topical antibiotics
c. Topical estrogen
d. Oral estrogen
e. Oral antibiotics and topical estrogen
Key = C (topical estrogen)
Explanation = The problem here is vaginal dryness for which the age and symptoms are a good clue. Topical
estrogen or HRT can be given to treat vaginal dryness, vaginal discharge and recurrent UTIs in post
menopausal women.
286. A pt came to the ED with severe lower abdominal pain. Vitals: BP=125/85mmHg, Temp=38.9C.
Exam: abdomen rigid, very uncomfortable during par vaginal. She gave a past hx of PID 3 years
ago which was successfully treated with antibiotics. What is the appropriate inv?
a. US
b. Abdomen XR
c. CT
d. High vaginal
e. Endocervical swab
287. A pregnant woman with longterm hx of osteoarthritis came to the antenatal clinic with
complaints of restricted joint movement and severe pain in her affected joints. What is the
choice of drug?
a. Paracetamol
b. Steroid
c. NSAID
d. Paracetamol+dihydrocoiene
e. Pethadine
Key = A (paracetamol)
Explanation = Safest drug in pregnancy is paracetamol among the choices given
288. A 24yo 18wk pregnant lady presents with pain in her lower abdomen for the last 24h. She had
painless vaginal bleeding. Exam: abdomen is tender, os is closed. What is the most probable dx?
a. Threatened miscarriage
b. Inevitable miscarriage
c. Incomplete miscarriage
d. Missed miscarriage
e. Spontaneous miscarriage
Key = A
Points in favour = painless vaginal bleeding , tender abdomen , os closed

Classification of miscarriage is as follows:


Threatened miscarriage: mild symptoms of bleeding. Usually little or no pain. The cervical os is closed.
Inevitable miscarriage: usually presents with heavy bleeding with clots and pain. The cervical os is
open. The pregnancy will not continue and will proceed to incomplete or complete miscarriage.
Incomplete miscarriage: this occurs when the products of conception are partially expelled. Many
incomplete miscarriages can be unrecognised missed miscarriages.
Missed miscarriage: the fetus is dead but retained. The uterus is small for dates. A pregnancy test can
remain positive for several days. It presents with a history of threatened miscarriage and persistent,
dirty brown discharge. Early pregnancy symptoms may have decreased or gone.
Habitual or recurrent miscarriage : three or more consecutive miscarriages.

. A o hild plaig i the gade had a lea ut. She didt hae a aiatios. Also, thee is
no contraindication to vaccinations. Parents were worried about the vaccine side effects. What
will you give?
a. Clean the wound and dress it
b. Give TT only
c. Give DPT only
d. Give DPT and tetanus Ig
e. Give complete DPT vaccine course
290. A 32yo female who has had 3 prv miscarriages in the 1st trimester now comes with vaginal
bleeding at 8wks. US reveals a viable fetus. What would be the most appropriate definitive
management?
a. Admit
b. Aspirin
c. Bed rest 2 weeks
d. Cervical cerclage
e. No tx
Key = B (Aspirin)
Explanation = This is a case of APS probably as evident by 3 prev miscarriages and vaginal bleeding now.
Aspirin should be advised throughout the pregnancy (along with LMWH).
291. A 6yo girl started wetting herself up to 6x/day. What is the most appropriate tx?
a. Sleep alarms
b. Desmopressin
c. Reassure
d. Behavior training
e. Imipramine
Key = A (sleep alarms)
Explanation = Alarm training is a first line treatment for nocturnal enuresis in children and is the most
effective long term strategy. Desmopressin can be used in children above the age of 7 but is indicated in
case of rapid control or when alarms are inappropriate and is usually used second line to alarm training.
Imipramine is another option approved for 6 year olds but is reserved for resistant cases only because of its
side effects. Behavior therapy is often considered inferior to these other confirmed methods of treatment.
Reassurance is for children under the age of 5.
292. A 27yo 34wk pregnant lady presents with headache, epigastric pain and vomiting. Exam:
pulse=115, BP=145/95mmHg, proteinuria ++. She complains of visual disturbance. What is the
best medication for the tx of the BP?
a. 4g MgSO4 in 100ml 0.9%NS in 5mins
b. 2g MgSO4 IV bolus
c. 5mg hydralazine IV
d. Methyldopa 500mg/8h PO
e. No tx
Key = C
Explanation = This is a case of severe pre-eclampsia as evident by pregnancy of more than 20 weeks,
sustained bp of more than 140/90 mmhg and headache, epigastric pain and visual disturbances. Treatment
of choice for Bp control in severe pre-eclampsia and eclampsia is IV hydralazine or labetalol or nifedipine
orally.
MgSO4 is for control of fits not bp.

Management of severe pre-eclampsia[1]


Delivery of the fetus and placenta is the only cure. However, preterm delivery may adversely affect neonatal
outcome, with complications resulting from prematurity and low birth weight.
Blood pressure:
o Antihypertensive treatment should be started in women with a systolic blood pressure over 160
mm Hg or a diastolic blood pressure over 110 mm Hg. In women with other markers of
potentially severe disease, treatment can be considered at lower degrees of hypertension.
o Labetalol (given orally or intravenously), oral nifedipine or intravenous hydralazine are usually
given for the acute management of severe hypertension.
o Atenolol, angiotensin-converting enzyme (ACE) inhibitors, angiotensin-II receptor antagonists
and diuretics should be avoided.
o Antihypertensive medication should be continued after delivery, as dictated by the blood
pressure. It may be necessary to maintain treatment for up to three months, although most
women can have treatment stopped before this.
Prevention of seizures:
Magnesium sulfate should be considered when there is concern about the risk of eclampsia.
In women with less severe disease, the decision is less clear and will depend on individual case
assessment.
Control of seizures:
o Magnesium sulfate is the therapy of choice to control seizures. A loading dose of 4 g is given by
infusion pump over 5-10 minutes, followed by a further infusion of 1 g/hour maintained for 24
hours after the last seizure.
o Recurrent seizures should be treated with either a further bolus of 2 g magnesium sulfate or an
increase in the infusion rate to 1.5 g or 2.0 g/hour.
Fluid balance:
o Fluid restriction is advisable to reduce the risk of fluid overload in the intrapartum and
postpartum periods. Total fluids should usually be limited to 80 ml/hour or 1 ml/kg/hour.
Delivery:
o The decision to deliver should be made once the woman is stable and with appropriate senior
personnel present.
o If the fetus is less than 34 weeks of gestation and delivery can be deferred, corticosteroids
should be given, although after 24 hours the benefits of conservative management should be
reassessed.
o Conservative management at very early gestations may improve the perinatal outcome but
must be carefully balanced with maternal well-being.
o The mode of delivery should be determined after considering the presentation of the fetus and
the fetal condition, together with the likelihood of success ofinduction of labour after
assessment of the cervix.[5]
o The third stage should be managed with 5 units of intramuscular/slow intravenous
Syntocinon. Ergometrine and Syntometrine should not be given for prevention of
haemorrhage, as this can further increase the blood pressure.
o Prophylaxis against thromboembolism should be considered.

Management of eclampsia
Resuscitation:
o The patient should be placed in the left lateral position and the airway secured.
o Oxygen should be administered.
Treatment and prophylaxis of seizures:
o Magnesium sulfate is the anticonvulsant drug of choice.
o Intubation may become necessary in women with repeated seizures in order to protect the
airway and ensure adequate oxygenation.
Treatment of hypertension:
o Reduction of severe hypertension (blood pressure >160/110 mm Hg or mean arterial pressure
>125 mm Hg) is essential to reduce the risk of cerebrovascular accident. Treatment may also
reduce the risk of further seizures.
o Intravenous hydralazine or labetalol are the two most commonly used drugs. Both may
precipitate fetal distress and therefore continuous fetal heart rate monitoring is necessary.
Fluid therapy:
o Close monitoring of fluid intake and urine output is mandatory.
o Pre-loading the circulation with 400-500 ml colloid prior to regional anaesthesia or
vasodilatation with hydralazine may reduce the risk of hypotension and fetal distress.
Delivery:
o The definitive treatment of eclampsia is delivery. Attempts to prolong pregnancy in order to
improve fetal maturity are unlikely to be of value.
o However, it is unsafe to deliver the baby of an unstable mother even if there is fetal distress.
Once seizures are controlled, severe hypertension treated and hypoxia corrected, delivery can
be expedited.
o Vaginal delivery should be considered but Caesarean section is likely to be required in
primigravidae, well before term and with an unfavourable cervix.
o After delivery, high-dependency care should be continued for a minimum of 24 hours.
All patients need careful follow-up and a formal postnatal review to establish if there is chronic hypertension,
proteinuria or liver damage.

Complications
Eclampsia is usually part of a multisystem disorder. Associated complications include haemolysis,
HELLP syndrome (3%), disseminated intravascular coagulation (3%), renal failure (4%) and adult
respiratory distress syndrome (3%).
Pre-eclampsia can progress to eclampsia with epileptic fits and sometimes other neurological
symptoms, including focal motor deficits and cortical blindness.
Cerebrovascular haemorrhage is a complicating factor in 1-2%.

293. A 24yo lady who is 37wk pregnant was brought to the ED. Her husband says a few hours ago she
complained of headache, visual disturbance and abdominal pain. On arrival at the ED she has a
fit. What is the next appropriate management for this pt?
a. 4g MgSO4 in 100ml 0.9%NS in 5mins
b. 2g MgSO4 IV bolus
c. 2g MgSO4 in 500ml NS in 1h
d. 4g MgSO4 IV bolus
e. 10mg diazepam in 500ml 0.9%NS in 1h
Key = A
Explained in the previous question.
. What is the pathologial hage i Baets esophagitis?
a. Squamous to columnar epithelium
b. Columnar to squamous epithelium
c. Dysplasia
d. Metaplasia
e. Hyperplasia
Key = A (squamous to columnar)
295. A 34yo male presents with hx of headache presents with ataxia, nystagmus and vertigo. Where
is the site of the lesion?
a. Auditory canal
b. 8th CN
c. Cerebellum
d. Cerebral hemisphere
e. Brain stem
Key = Cerebellum
Points in favour = ataxia, nystagmus and vertigo
296. A 24yo girl comes to the woman sexual clinic and seeks advice for contraception. She is on
sodium valproate.
a. She at use COCP
b. She can use COCP with extra precaution
c. She can use COCP if anticonvulsant is changed to carbamezapin.
d. She can use COCP with estrogen 50ug and progesterone higher dose
e. She can use COCP
Key = E
Points in favour = Women using anticonvulsants that do not induce live liver enzyme cytochrome - P450 can
use OCPs without any restriction.

Anticonvulsants not inducing liver enzymes = gabapentin, levetiracetam, valproate and vigabatrin.

Anticonvulsants inducing liver enzymes = phenytoin, carbamazepine, barbiturates, primidone, topiramate


and oxcarbazepine. May use depot medroxyprogesterone acetate, copper intrauterine contraceptive devices,
the levonorgestrel-releasing intrauterine system, barrier methods and natural family planning methods.
297. A 27yo lady came to the ED 10 days ago with fever, suprapubic tenderness and vaginal discharge.
PID was dx. She has been on the antibiotics for the last 10days. She presents again with lower
abdominal pain. Temp=39.5C. what is the most appropriate next management?
a. Vaginal swab
b. Endocervical swab
c. US
d. Abdominal XR
e. Laparoscopy
298. An 18yo man complains of fatigue and dyspnea, he has left parasternal heave and systolic thrill
with a harsh pan-systolic murmur at left parasternal edge. What is the most probable dx?
a. TOF
b. ASD
c. VSD
d. PDA
e. TGA
Key = C (VSD)
Points in favour = Age , Left parasternal heave, pan systolic murmur at left parasternal edge.

Transposition of great arteries presents in the infants and not that late in life.
TOF may be left undiagnosed this late but patients present with severe cyanosis and other typical features of
TOF.
ASD has a soft systolic ejection murmur in the pulmonic area and diastolic rumble at left sternal border.

VSD :-

Epidemiology[3]
VSDs are the most common congenital heart defect in children, occurring in 50% of all children with
congenital heart disease and in 20% as an isolated lesion.
The incidence of VSDs has increased significantly with advances in imaging and screening of infants and
ranges from 1.56 to 53.2 per 1,000 live births. The ease with which small muscular VSDs can now be
detected has contributed to this increase in incidence.
In the adult population VSDs are the most common congenital heart defect, excluding bicuspid aortic
valve.

Presentation
How haemodynamically significant a VSD is depends on its size, pressure in the individual ventricles and
pulmonary vascular resistance.[3] The presence of a VSD may not be obvious at birth because of nearly equal
pressures in both the ventricles with little or no shunting of blood. As the pulmonary vascular resistance
drops, the pressure difference between the two ventricles increases and the shunt becomes significant
allowing the defect to become clinically apparent. An exception to this rule is Down's syndrome where the
pulmonary vascular resistance may not fall and the VSD may not become clinically apparent, first presenting
with pulmonary hypertension. All babies with Down's syndrome should therefore be screened for congenital
heart disease no later than 6 weeks of age.[8]
The clinical presentation varies with the severity of the lesion:
With a small VSD, the infant or child is asymptomatic with normal feeding and weight gain and the
lesion may be detected when a murmur is heard at a routine examination.
With a moderate-to-large VSD, although the babies are well at birth, symptoms generally appear by 5
to 6 weeks of age. The main symptom is exercise intolerance and since the only exercise babies do is
feeding, the first impact is on feeding. Feeding tends to slow down and is often associated with
tachypnoea and increased respiratory effort. Babies are able to feed less, and weight gain and growth
are soon affected. Poor weight gain is a good indicator of heart failure in a baby. Recurrent respiratory
infections may also occur.
With very large VSDs the features are similar but more severe. If appropriate management is not
carried out promptly in infants with large VSDs excessive pulmonary blood flow may lead to increase in
pulmonary vascular resistance and pulmonary hypertension. These babies may develop a right to left
shunt with cyanosis or Eisenmenger's syndrome.

Physical signs
Again, these depend on the severity of the lesion with, one exception, the loudness of the murmur. Murmurs
are caused by turbulence of blood flow. There may be more turbulence with a small hole than with a large
defect. The loudness of the murmur gives no indication of the size of the lesion. Even the adage 'the louder
the sound, the smaller the lesion' is untrue.
With a small VSD the infant is well developed and pink. The precordial impulse may be greater than
usual but is usually normal. If it can be heard, the physiological splitting of the second sound is normal
but there is a harsh systolic murmur that is best heard at the left sternal edge, which may obliterate
the second sound. The murmur tends to be throughout systole but, if the defect is in the muscular
portion, it may be shorter as the hole is closed as the muscle contracts.
With a moderate or large VSD there is enhanced apical pulsation as well as a parasternal heave. A
grade 2 to 5/6 systolic murmur is audible at the lower left sternal border. It may be pansystolic or early
systolic. A prominent third sound with a short early mid-diastolic rumble is audible at the apex with a
moderate-to-large shunt (because of increased flow through the mitral valve during diastole). S2 is
loud and single in patients with pulmonary hypertension.
Large defects with no shunts or those with Eisenmenger physiology and right-to-left shunt may have
no murmur.
Investigations = ECG, CXR, ECHOCARDIOGRAPHY and Cardiac catheterization.

Management[10]

Medical management
Management in the infant and child depends on symptoms, with small asymptomatic defects needing
no medical management, and unlikely to need any intervention.
First-line treatment for moderate or large defects affecting feeding and growth is with diuretics for
heart failure and high-energy feeds to improve calorie intake.
Angiotensin-converting enzyme inhibitors are used to reduce afterload which promotes direct systemic
flow from the left ventricle, thus reducing the shunt. Digoxin can also be given for its inotropic effect.
Any patient needing significant medical management should be referred for surgical assessment.

Surgical management
Surgical repair is required if there is uncontrolled heart failure, including poor growth. Even very small
babies may be considered for surgery.
Infundibular defects may be considered for closure even if they are asymptomatic because of their
location.
Development of aortic valve prolapse and aortic regurgitation in perimembranous VSDs may be an
indication for surgical closure.
Most defects are closed nowdays by directly placing a patch from the right ventricular side, usually
with the surgeon working through the tricuspid valve.
Patients with large muscular VSDs which are difficult to see or those with multiple holes (Swiss cheese
septum) presenting as neonates or infants need initial palliation in the form of pulmonary artery
banding followed many months later by corrective surgery and removal of the pulmonary artery band.

Catheter closure
Advances in catheter techniques and devices mean that many muscular and perimembranous VSDs can
now be closed percutaneously. This is in the setting of normal atrioventricular and ventriculoarterial
connections and absence of any atrioventricular or arterial valve override.
Transcatheter techniques are useful because they avoid cardiopulmonary bypass. There are, however,
recognised complications for device closure of perimembranous VSDs, including complete heart block
needing permanent pacemaker.[11]
The National Institute for Health and Care Excellence (NICE) has provided detailed guidance on
indications, efficacy and complications of the procedure.[12]
It is safer to close muscular VSDs using a device but muscular VSDs which are haemodynamically
significant are likely to be seen in only young infants, making catheterisation difficult and challenging.
Hybrid procedures increasingly being used involve insertion of the device in the operation theatre after
surgical exposure of the defect.[13]

299. A young girl presenting with fever, headache, vomiting, neck stiffness and photophobia. She has
no rashes. What is the most appropriate test to confirm dx?
a. Blood culture
b. Blood glucose
c. LP
d. CXR
e. CT
Key = C
Points in favour = This is suspected meningitis. Do Lp to confirm.
300. A 65yo HTN man wakes up in the morning with slurred speech, weakness of the left half of his
body and drooling. Which part of the brain is affected?
a. Left parietal lobe
b. Right internal capsule
c. Right midbrain
d. Left frontal lobe
Key = B

351. A 35yo lady presents with painful ulcers on her vulva, what is the appropriate inv which will lead to the
dx?
a. Anti-HSV antibodies
b. Dark ground microscopy of the ulcer
c. Treponema palladium antibody test
d. Rapid plasma regain test
e. VDRL
Dx genital herpes
Ans. key A. Anti-HSV antibodies. [Genital Herpes may be asymptomatic or may remain dormant for months or
even years. When symptoms occur soon after a person is infected, they tend to be severe. They may start as
multiple small blisters that eventually break open and produce raw, painful sores that scab and heal over
within a few weeks. The blisters and sores may be accompanied by flu-like symptoms with fever and swollen
lymph nodes.
treatment : There are three major drugs commonly used to treat genital herpes symptoms: acyclovir
(Zovirax), famciclovir (Famvir), and valacyclovir(Valtrex). These are all taken in PO. Severe cases may be
treated with the intravenous (IV) drug acyclovir].
options B C D & E are tests for syphilis which presents with single painless ulcer (canchre)

352. A 53yo man presents with a longstanding hx of a 1cm lesion on his arm. It has started
bleeding on touch. What is the most likely dx?
a. Basal cell carcinoma
b. Kaposis sarcoma
c. Malignant melanoma
d. Squamous cell carcinoma
e. Kerathoacanthoma

Ans. D Squamous cell carcinoma. [SSCs Arises in squamous cells. SCCs may occur on all areas of the body
including the mucous membranes and genitals, but are most common in areas frequently exposed to the sun,
such as the rim of the ear, lower lip, face, balding scalp, neck, hands, arms and legs. SCCs often look like scaly
red patches, open sores, elevated growths with a central depression, or warts; they may crust or bleed.
investigation: tissue sample (biopsy) will be examined under a microscope to arrive at a diagnosis.
prognosis : Squamous cell carcinomas detected at an early stage and removed promptly are almost always
curable and cause minimal damage].
basal cell carcinoma is usually on face with inverted margins
malignant melanoma is on sun exposed parts and is dark (black colored) ulcer

353. A 47yo man with hx of IHD complains of chest pain with SOB on exertion over the past few days. ECG
normal, Echo= increased EF and decreased septal wall thickness. What is the most likely dx?
a. Dilated CM
b. Constrictive pericarditis
c. Amyloidosis
d. Subacute endocarditis

Ans. The key is A. Dilated CM.


points in fav: sob, palpitation, dec septal wall thinning
treatment : beta blocker, acei, diuretics

Constrictive pericarditis doesnt fits because it starts with urti has pain on lying flat which is relieved by leaning
forward
ecg shows wide spread st elevation
Amyloid deposition in the heart can cause both diastolic and systolic heart failure. EKG changes may be
present, showing low voltage and conduction abnormalities like atrioventricular block or sinus node
dysfunction. On echocardiography the heart shows restrictive filling pattern, with normal to mildly reduced
ejec fraction

354. An elderly pt who is known to have DM presents to the hospital with drowsiness, tremors and confusion.
What inv should be done to help in further management?
a. Blood sugar
b. ECG
c. Standing and lying BP
d. Fasting blood sugar
e. CT

Ans. The key is A. Blood sugar.since he is known diabetic he may have gotten hypoglycemic d/t his meds

355. A 28yo pregnant woman with polyhydramnios and SOB comes for an anomaly scan at 31 wks. US=
absence of gastric bubble. What is the most likely dx?
a. Duodenal atresia
b. Esophageal atresia
c. Gastrochiasis
d. Exomphalos
e. Diaphragmatic hernia

Ans. The key is B. Oesophageal atresia.

This condition is visible, after about 26 weeks, on an ultrasound. On antenatal USG, the finding of an absent
or small stomach in the setting of polyhydramnios used to be considered suspicious of esophageal atresia.
However, these findings have a low positive predictive value. The upper neck pouch sign is another sign that
helps in the antenatal diagnosis of esophageal atresia and it may be detected soon after birth as the
affected infant will be unable to swallow its own saliva. Also, the newborn can present with gastric
distention, cough, apnea, tachypnea, and cyanosis. In many types of esophageal atresia, a feeding tube will
not pass through the esophagus.
356. A 1m boy has been brought to the ED, conscious but with cool peripheries and has HR=222bpm. He has
been irritable and feeding poorly for 24h. CXR=borderline enlarged heart with clear lung fields. ECG=regular
narrow complex tachycardia, with difficulty identifying p wave. What is the single most appropriate immediate
tx?
a. Administer fluid bolus
b. Administer oxygen
c. Oral beta-blockers
d. Synchronized DC cardio-version
e. Unilateral carotid sinus massage

The key is D. Synchrnized DC cardioversion.


reason: As the patient is in probable hemodynamic instability (suggested by cool peripheries) so we should go
for DC cardioversion.
diagnosis SVT.

357. A 7yo child presented with chronic cough and is also found to be jaundiced on examination.
What is the most likely dx?
a. Congenital diaphragmatic hernia
b. Congenital cystic adenematoid malformation
c. Bronchiolitis
d. RDS
e. Alpha 1 antitrypsin deficiency

The key is E. Alpha 1 antitrypsin deficiency.


REASON. Unexplained liver disease with respiratory symptoms are very suggestive of AATD.
liver disease occurs because of the accumulation AAT in it
where as d/t inability to be transported out of liver AATD causes emphysema hence the resp problems

358. A 35yo construction worker is dx with indirect inguinal hernia. Which statement below best
describes it?
a. Passes through the superficial inguinal ring only
b. Lies above and lateral to the pubic tubercle
c. Does not pass through the superficial inguinal ring
d. Passes through the deep inguinal ring

Ans. The key is D. Passess through the deep inguinal ring.


direct hernia passes directly through the posterior wall of inguinal canal whereas indirect can only do so via
deep ring

359. A woman has numerous painful ulcers on her vulva. What is the cause?
a. Chlamydia
b. Trichomonas
c. Gardenella
d. HSV
e. EBV

Ans. The key is D. HSV. reason has been explained in q 351

360. A 72 yo man has been on warfarin for 2yrs because of past TIA and stroke. What is the most important
complication that we should be careful with?
a. Headache
b. Osteoporosis
c. Ear infection
d. Limb ischemia
e. Diarrhea

Ans. key is wrong


right key is A Headache, as there are chances of SAH or generally ICH

361. A 55yo man has been admitted for elective herniorraphy. Which among the following can be the reason
to delay his surgery?
a. Controlled asthma
b. Controlled atrial fib
c. DVT 2yrs ago
d. Diastolic BP 90mmHg
e. MI 2 months ago

Ans. E SAFER TO DO SURGERY AFTER 6 MONTHS

362. A 65yo known case of liver ca and metastasis presents with gastric reflux and bloatedness. On bone exam
there is osteoporosis. He also has basal consolidation in the left lung. What is the next appropriate step?
a. PPI IV
b. Alendronate
c. IV antibiotics
d. Analgesic
e. PPI PO
IN THIS case reflux is the cause of recurrent pneumonia so both C AND E can be right but to chose single one
E is more appropriate

363. A 66yo man has the following ECG. What is the most appropriate next step in management?
a. Metoprolol
b. Digoxin
c. Carotid sinus massage
d. Adenosine
e. Amiodarone.

Ans. A beta blocker for A FIB


364. A 22yo sexually active male came with 2d hx of fever with pain in scrotal area. Exam: scrotal skin is red
and tender. What is the most appropriate dx?
a. Torsion of testis
b. Orchitis
c. Inguinal hernia
d. Epididymo-orchitis

D Epididymo-orchitis.
In orchitis there should be fever, elevation of testes reduces pain (positive prehn sign), In torsion testis lies at a
higher level. In torsion urinalysis negative but in orchitis it is positive. Orchitis usually occurs in sexually active
man. X

365. A man on warfarin posted for hemicolectomy. As the pt is about to undergo surgery. What
option is the best for him?
a. Continue with warfarin
b. Continue with warfarin and add heparin
c. Stop warfarin and add aspirin
d. Stop warfarin and add heparin
e. Stop warfarin

D Stop warfarin and add heparin


5 DAYS BEFORE SURGERY WARFARIN MUST BE REPLACED BY HEPARIN,
366. A 65yo known alcoholic is brought into hospital with confusion, aggressiveness and
ophthalmoplegia. He is treated with diazepoxide. What other drug would you like to prescribe?
a. Antibiotics
b. Glucose
c. IV fluids
d. Disulfiram
e. Vit B complex

E Vitamin B complex. [confusion and ophthalmoplegia points towards the diagnosis of Wernickes
encephalopathy].
which occurs d/t thiamine def.

367. A 32yo woman has severe right sided abdominal pain radiating into the groin which has lasted for 3h. She
is writhering in pain. She has no abdominal signs. What is the most likely cause of her abdominal pain?
a. Appendicitis
b. Ruptured ectopic pregnancy
c. Salpingitis
d. Ureteric colic
e. Strangulated hernia

D Ureteric colic.
It indicate stone at lower ureter. [i) Pain from upper ureteral stones tends to radiate to the flank and lumbar
areas. ii) Midureteral calculi cause pain that radiates anteriorly and caudally. This midureteral pain in
particular can easily mimic appendicitis on the right or acute diverticulitis on the left. iii) Distal ureteral stones
cause pain that tends to radiate into the groin or testicle in the male or labia majora in the female.

368. A 39yo coal miner who smokes, drinks and has a fam hx of bladder cancer is suffering from BPH. The
most important risk factor for his bladder carcinoma is?
a. Fam hx
b. Smoking
c. Exposure to coal mine
d. BPH

B smoking.
. Risk factors of bladder cancer:
i) Smoking ii) Exposure to chemicals used in dye industry iii) Whites are more likely to develop bladder cancer
iv) Risk increases with age v) More common in men vi) Chronic bladder irritation and infections (urinary
infections, kidney and bladder stones, bladder catheter left in place a long time.) vii) Personal history of
bladder or other urothelial cancer viii) Family history ix) Chemotherapy or radiotherapy x) Pioglitazone for
more than one year and certain herb xi) Arsenic in drinking water xii) Low fluid consumption.

369. A 34yo woman is referred to the endocrine clinic with a hx of thyrotoxicosis. At her 1st
appointment she is found to have a smooth goiter, lid lag and bilateral exophthalmos with puffy
eyelids and conjunctival injection. She wants to discuss the tx of her thyroid prb as she is keen to become
pregnant. What is the most likely tx you would advise?
a. 18m of carbimazole alone
b. 18m of PTU alone
c. A combo od anti-thyroid drug and thyroxine
d. Radioactive iodine
e. Thyroidectomy

B 18m of PTU alone.


Other drug option i.e Carbamazepine is teratogenic [can cause i) spina bifida ii)cardiovascular malformations
ETC . PTU is on the other hand relatively safe in pregnancy.

370. A child living with this stepfather is brought by the mother with multiple bruises, fever and
fractures. What do you suspect?
a. NAI
b. Malnutrition
c. Thrombocytopenia
d. HIV

Ans. The key is A. NAI. [H/O living with stepfather, multiple bruises, fever and fractures are suggestive of NAI].
OTHER possible points can include hx not matching with bruises, wounds which are a day or two older at the
time of presentation.

371. A young man who was held by the police was punched while in custody. He is now cyanosed and
unresponsive. What is the 1st thing you would do?
a. IV fluids
b. Clear airway
c. Turn pt and put in recovery position
d. Give 100% oxygen
e. Intubate and ventilate

B. Clear airway. [ABC protocol].

372. A HTN male loses vision in his left eye. The eye shows hand movement and a light shined in the eye is
seen as a faint light. Fundus exam: flame shaped hemorrhages. The right eye is normal. What is the cause of
this pts unilateral blindness?
a. HTN retinopathy
b. CRA thrombosis
c. CRV thrombosis
d. Background retinopathy
e. Retinal detachment

key is wrong.
right ans is C ( unilateral blindness with flame shaped hemorrhages are characteristic of CRVO). Flame
shaped hemorrhages are seen in HTN and diabetic retinopathy too but they will cause bilateral damage.

373. A mentally retarded child puts a green pea in his ear while eating. The carer confirms this.
Otoscopy shows a green colored object in the ear canal. What is the most appropriate single
best approach to remove this object?
a. By magnet
b. Syringing
c. Under GA
d. By hook
e. By instilling olive oil

C UNDER GA
[Pea is not a magnetic material and hence it cannot be removed by magnet, it will swell up if syringing is
attempted, as hook placement is likely with risk of pushing the pea deeper it is not also suitable in a
mentally retarded child, and olive oil is not of help in case of pea. So to avoid injury it is better to remove
under GA

374. A pt presents with longstanding gastric reflux, dysphagia and chest pain. On barium enema, dilation of
esophagus with tapering end is noted. He was found with Barretts esophagus. He had progressive dysphagia
to solids and then liquids. What is the single most appropriate dx?
a. Achalasia
b. Esophageal spasm
c. GERD
d. Barretts esophagus
e. Esophageal carcinoma
E Oesophageal carcinoma. [there is dilatation in oesophagus which is seen both in achalasia and carcinoma.
Dysphagia to solid initaially is very much suggestive of carcinoma and also barretts change is a clue to
carcinoma]
Progressive dysphagia with h/o barrett esophagus are the key indicators
375. A 48yo lady presents with itching, excoriations, redness, bloody discharge and ulceration around her
nipple. What is the most likely dx?
a. Pagets disease of the breast
b. Fibrocystic dysplasia
c. Breast abscess
d. Duct papilloma
e. Eczema

A Pagets disease of the breast.


TYPICAL manifestation of pagets disease
eczema like rash involving nipple and areola with straw or bloody discharge. Eczema is bilateral.
Also nipple turns inward in advances stages pt complaints of burning sensation at the site of lesion
DX mammography and biopsy
TX surgery + chemo or radio may be needed
376. Pt with widespread ovarian carcinoma has bowel obstruction and severe colic for 2h and was normal in
between severe pain for a few hours. What is the most appropriate management?
a. PCA (morphine)
b. Spasmolytics
c. Palliative colostomy
d. Oral morphine
e. Laxatives

C. Palliative colostomy.
Cancer or chemotherapy induced obstructions are unlikely to respond to conservative management [NBM, IV
fluid, nasogastric suction] and hence only analgesia will not relieve it. So in such cases we have to go for
palliative colostomy.

377. A 70yo man admits to asbestos exposure 20yrs ago and has attempted to quit smoking. He has noted
weight loss and hoarseness of voice. Choose the single most likely type of cancer a.w risk
factors present.
a. Basal cell carcinoma
b. Bronchial carcinoma
c. Esophageal carcinoma
d. Nasopharyngeal carcinoma
e. Oral carcinoma

B. Bronchial carcinoma. [Asbestos exposure is a risk factor for lung cancer and also has a synergistic effect with
cigarette smoke].
#. Conditions related to asbestos exposure: i) Pleural plaques (after a latent period of 20-40 yrs) ii) Pleural
thickening iii) Asbestosis (latent period is typically 15-30 yrs) iv) Mesothelioma (prognosis is very poor) v) Lung
cancer.

378. A 32yo woman had progressive decrease in vision over 3yrs. She is no dx as almost blind. What would be
the mechanism?
a. Cataract
b. Glaucoma
c. Retinopathy
d. Uveitis
e. Keratitis

B. Glaucoma.
. Cataract is unlikely at this age. Nothing in the history suggests retinopathy. Uveitis and iritis doesnt have
such degree of vision loss and iritis and anterior uveitis have pain, redness and photophobia. Open angle
glaucoma is likely cause.

379. A child during operation and immediately after showed glycosuria, but later his urine sugar was normal.
Choose the most probable dx.
a. Pre-diabetic state
b. Normal finding
c. Low renal tubular threshold
d. DM

B Normal finding.
Stress during operation can cause transient hyperglycemia causing glycosuria secondary to stress induced rise
of cortisol which becomes normal after some time.
380. A pt presented with hx of swelling in the region of the sub-mandibular region, which became
more prominent and painful on chewing. He also gave hx of sour taste in the mouth, the area is
tender on palpation. Choose the most probable dx?
a. Chronic recurrent sialadenitis
b. Adenolymphoma
c. Mikuliczs disease
d. Adenoid cystic carcinoma
e. Sub-mandibular abscess
A Chronic recurrent sialadenitis. [pain, swelling, more pain on chewing, tenderness, and submandibular region
suggests diagnosis of submandibular chronic recurrent sialadenitis, usually secondary to sialolithiasis or
stricture].
381. ECG of an 80yo pt of ICH shows saw-tooth like waves, QRS complex of 80ms duration,
ventricular rate=150/min and regular R-R interval. What is the most porbable dx?
a. Atrial fib
b. Atrial flutter
c. SVT
d. Mobitz type1 second degree heart block
e. Sinus tachycardia
B Atrial flutter. [Saw-tooth like waves, normal QRS comples of 80 ms (normal range 70-100 ms), ventricular
rate of 150/min and regular R-R interval is diagnostic of atrial flutter].
FOR AFIB THERE WD BE IRREGULARARLY IRREGULAR RHYTHM

382. A 50 yo woman who was treated for breast cancer 3 yrs ago now presents with increase thirst and
confusion. She has become drowsy now. What is the most likely metabolic abnormality?
a. Hypercalcemia
b. Hyperkalemia
c. Hypoglycemia
d. Hyperglycemia
e. Hypercalcemia.
E HYPERCALCEMIA
Ans. 2. Increased thirst, confusion, drowsiness these are features of hypercalcemia. Any solid organ tumour
can produce hypercalcemia. Here treated Ca breast is the probable cause of hypercalcemia.
383. A 29yo woman presents to her GP with a hx of weight loss, heat intolerance, poor conc and
palpitations. Which of the following is most likely to be a/w dx of thyroiditis a/w viral infection?
a. Bilateral exophthalmos
b. Diffuse, smooth goiter
c. Reduced uptake on thyroid isotope scan
d. Positive thyroid peroxidase antibodies
e. Pretibial myxedema

C. Reduced uptake on thyroid isotope scan.


DX De Quervains or subacute thyroiditis.
. Viral or subacute thyroiditis: diagnostic criteria: i) Features of hyperthyroidism present. ii) Pain thyroid, not
mentioned. iii) Investigations: high esr (60-100) not mentioned, Reduced uptake of radioactive iodine by the
gland.
384. A lady, post-colostomy closure after 4 days comes with fluctuating small swelling in the stoma.
What is the management option for her?
a. Local exploration
b. Exploratory laparotomy
c. Open laparotomy
d. Reassure
A Local exploration.
THERE MUST BE SOME LOCAL WOUND PROBLEM
385. A 65yo female pt was given tamoxifen, which of the following side effect caused by it will
concern you?
a. Fluid retention
b. Vaginal bleeding
c. Loss of apetite
d. Headache and dizziness
e.

B Vaginal bleeding.
. Tamoxifen can promote development of endometrial carcinoma. So vaginal bleeding will be of concern for
us.
386. A 39yo man with acute renal failure presents with palpitations. His ECG shows tall tented T
waves and wide QRS complex. What is the next best step?
a. Dialysis
b. IV calcium chloride
c. IV insulin w/ dextrose
d. Calcium resonium
e. Nebulized salbutamol

B. IV calcium chloride (both IV calcium gluconate or IV calcium chloride can be used) when there is ECG
changes.
DX The ECG changes are suggestive of Hyperkalemia.
At potassium level of >5.5mEq/L occurs tall tented T waves and at potassium level >7mEq/L occurs wide QRS
complex with bizarre QRS morphology.

387. A 54yo pt 7 days after a total hip replacement presents with acute onset breathlessness and
raised JVP. Which of the following inv will be most helpful in leading to a dx?
a. CXR
b. CTPA
c. V/Q scan
d. D-Dimer
e. Doppler US of legs

The key is B. CTPA.


The patient has a +ve two level PE Wells score (if it was negative we should do D-Dimer) and there is no renal
impairment or history suggestive of allergy to contrast media (if these present we should have go for VQ scan)
the investigation of choice is PTCA. NICE guideline.

388. A 7yo girl has been treated with penicillin after sore throat, fever and cough. Then she
develops skin rash and itching. What is the most probable dx?
a. Erythema nodosum
b. Erythema multiforme
c. SJS
d. Erythema marginatum
e. Erythema gangernosum

. The key is B. Erythema multiforme.


Common drugs causing erythma multiforme are: antibiotics (including, sulphonamides, penicillin),
anticonvulsants (phenytoin,barbiturates), aspirin, antituberculoids, and allopurinol.
CLOSELY related option is SJS which would have muco cutaneous rash but in here we hav only cutaneous rash.
Nodosum is in diseases like Tb, sarcoidosis, IBD
Marginatum is the rash of acute rheumatic fever

389. A 60yo man presented with a lump in the left supraclavicular region. His appetite is
decreased and he has lost 5kg recently. What is the most probably dx?
a. Thyroid carcinoma
b. Stomach carcinoma
c. Bronchial carcinoma
d. Mesothelioma
e. Laryngeal carcinoma

The key is B. Stomach carcinoma. [Mentioned lump in the left supraclavicular region is Vershows gland, has
long been regarded as strongly indicative of the presence of cancer in the abdomen, specifically gastric
cancer].
390. A 64yo man has presented to the ED with a stroke. CT shows no hemorrhage. ECG shows atrial fib. He has
been thrombolysed and hes awaiting discharge. What prophylactic regimen is best for him?
a. Warfarin
b. Heparin
c. Aspirin
d. Statins
e. Beta blockers

The key is A. Warfarin. [Atrial fibrillation: post stroke- following a stroke or TIA warfarin should be given as the
anticoagulant of choice. NICE guideline].
391. A 54yo man after a CVA presents with ataxia, intention tremors and slurred speech. Which part of the
brain has been affected by the stroke?
a. Inner ear
b. Brain stem
c. Diencephalon
d. Cerebrum
e. Cerebellum

The key is E. Cerebellum.


i) Ataxia ii) slurred speech or dysarthria iii) dysdiadochokinesia iv) intention tremor v) nystagmus. are the signs
of cerebellar defect

392. A 57yo man with blood group A complains of symptoms of vomiting, tiredness, weight loss and
palpitations. Exam: hepatomegaly, ascites, palpable left supraclavicular mass. What is the most likely dx?
a. Gastric carcinoma
b. Colorectal carcinoma
c. Peptic ulcer disease
d. Atrophic gastritis
e. Krukenberg tumor

Ans. The key is A. Gastric carcinoma. [i) blood group A is associated with gastric cancer ii) vomiting, tiredness,
weight loss are general features of gastric cancer iii) palpitation from anemia of cancer iv) hepatomegaly and
ascites are late features of gastric cancer. v) palpable left supraclavicular mass- is Vershows gland, has long
been regarded as strongly indicative of gastric cancer].

393. A 21yo girl looking unkempt, agitated, malnourished and nervous came to the hospital asking
for painkillers for her abdominal pain. She is sweating, shivering and complains of joint pain.
What can be the substance misuse here?
a. Alcohol
b. Heroin
c. Cocaine
d. LSD
e. Ecstasy

The key is B. Heroin. [agitation, nervousness, abdominal cramp, sweating, shivering and piloerection,
arthralgia these are features of heroin withdrawal].
Also asking for painkillers. Probably looking for morphine.
394. A child presents with increasing jaundice and pale stools. Choose the most appropriate test?
a. US abdomen
b. Sweat test
c. TFT
d. LFT
e. Endomyseal antibodies

The key is A. US abdomen. [This is a picture suggestive of obstructive jaundice. LFT can give clue like much
raised bilirubin, AST and ALT not that high and raised alkaline phosphatase but still USG is diagnostic in case of
obstructive jaundice].

395. A 32yo man presents with hearing loss. AC>BC in the right ear after Rinne test. He also
complains of tinnitus, vertigo and numbness on same half of his face. What is the most
appropriate inv for his condition?
a. Audiometry
b. CT
c. MRI
d. Tympanometry
e. Webers test

The key is C. MRI. [features are suggestive of acoustic neuroma, so MRI is the preferred option]. it involves
basically 8th nerve but 6 7 9 and 10th nerves are also involved with it

396. A 56 yo lady with lung cancer presents with urinary retention, postural hypotension, diminished reflexes
and sluggish pupillary reaction. What is the most likely explanation for her symptoms?
a. Paraneoplastic syndrome
b. Progression of lung cancer
c. Brain metastasis
d. Hyponatremia
e. Spinal cord compression

The key is A. Paraneoplastic syndrome.


s/s are of autonomic neuropathy which occurs in paraneoplastic syndrome

397. An old woman having decreased vision cant see properly at night. She has changed her glasses quite a
few times but to no effect. She has normal pupil and cornea. What is the most likely dx?
a. Cataract
b. Glaucoma
c. Retinal detachment
d. Iritis
e. GCA

key is wrong
correct key is A cataract
old age and progressive weakness supports Cataract
Not glaucoma...as pupil would be mid dilated and sluggish reaction and in acute attack corneal edema

Not RD...as pupil would be yellowish in color and there would be RAPD in massive RD and vision would be
dropped in day and night

Not iritis..as pupil would be constricted and cornea would have precipitation on its back (keratic precipitate)
Not GCA(giant cell arteritis) as vision on it is suddenly dropped to HM up to LP and vision dropped day and
night

398. A pt comes with sudden loss of vision. On fundoscopy the optic disc is normal. What is the
underlying pathology?
a. Iritis
b. Glaucoma
c. Vitreous chamber
d. Retinal detachment

Ans. 1. The Key is D. Retinal detachment.


#Causes of sudden painless loss of vision:
1. Retinal detachment
2. Vitreous haemorrhage
3. Retinal vein occlusion
4. Retinal artery occlusion
5. Optic neuritis
6. Cerebrovascular accident
remember retinal detachment has vision loss as if curtain is coming down

399. A child was woken up from sleep with severe pain in the testis. Exam: tenderness on palpation and only
one testis was normal in size and position. What would be your next step?
a. Analgesia
b. Antibiotics
c. Refer urgently to a surgeon
d. Reassurance
e. Discharge with analgesics

Ans. The key is A. Analgesia. [According to some US sites it is analgesia but no UK site support this!!! So for
Plab exam the more acceptable option is C. Refer urgently to a surgeon].
IN TORSION THE SOONER THE SURGICAL INTERVENTION DONE, THE BETTER THE RESULTS ARE

400. A child suffering from asthma presents with Temp 39C, drooling saliva on to the mothers lap,
and taking oxygen by mask. What sign will indicate that he is deteriorating?
a. Intercostal recession
b. Diffuse wheeze
c. Drowsiness

The key is A. Intercostal recession. [ here intercostals recession and drowsiness both answers are correct.
Hope in exam there will be one correct option]. but to chose among them, better go with A

401. A 12yo boy presents with painful swollen knew after a sudden fall. Which bursa is most likely tobe
affected?
a. Semimembranous bursa
b. Prepatellar bursa
c. Pretibial bursa
d. Suprapatetaller bursa

. The key is B. Prepatellar bursa. [A fall onto the knee can damage the prepatellar bursa. This usually causes
bleeding into the bursa sac causing swollen painful knee. Prepatellar bursitis that is caused by an injury will
usually go away on its own. The body will absorb the blood in the bursa over several weeks, and the bursa
should return to normal. If swelling in the bursa is causing a slow recovery, a needle may be inserted to drain
the blood and speed up the process. There is a slight risk of infection in putting a needle into the bursa].

402. A 61yo man has been referred to the OPD with frequent episodes of breathlessness and chest pain a/w
palpitations. He has a regular pulse rate=60bpm. ECG=sinus rhythm. What is the
most appropriate inv to be done?
a. Cardiac enzymes
b. CXR
c. ECG
d. Echo
e. 24h ECG

The key is E. 24h ECG.


Indications of 24 h ambulatory holter monitoring:

To evaluate chest pain not reproduced with exercise testing

To evaluate other signs and symptoms that may be heart-related, such as fatigue, shortness of breath,
dizziness, or fainting

To identify arrhythmias or palpitations

To assess risk for future heart-related events in certain conditions, such as idiopathic hypertrophic
cardiomyopathy, post-heart attack with weakness of the left side of the heart, or Wolff-Parkinson-White
syndrome

To assess the function of an implanted pacemaker

To determine the effectiveness of therapy for complex arrhythmias

403. A woman dx with Ca Breast presents now with urinary freq. which part of the brain is the
metastasis spread to?
a. Brain stem
b. Pons
c. Medulla
d. Diencephalon
e. Cerebral cortex

The key is D. Diencephalon. [diencephalon is made up of four distinct components: i) the thalamus ii) the
subthalamus iii) the hypothalamus and iv) the epithalamus. Among these the hypothalamus has crucial role in
causing urinary frequency].

404. A man is very depressed and miserable after his wifes death. He sees no point in living now that his wife
is not around and apologises for his existence. He refuses any help offered. His son has brought him to the ED.
The son can.t deal with the father any more. What is the most
appropriate next step?
a. Voluntary admission to psychiatry ward
b. Compulsory admission under MHA
c. Refer to social services
d. Alternate housing
e. ECT

Ans. The key is B. Compulsory admission under MHA. [This patient is refusing any help offered! And his son
cannot deal with him anymore! In this situation voluntary admission to psychiatry ward is not possible and the
option of choice is compulsory admission under MHA].

405. A 31yo man has epistaxis 10 days following polypectomy. What is the most likely dx?
a. Nasal infection
b. Coagulation disorder
c. Carcinoma

The key is A. Nasal infection.


HEMORRHAGE AFTER 7 TO 14 DAYS IS SECONDARY HEMORRHAGE [Infection is one of the most important
cause of secondary hemorrhage].

406. A woman had an MI. She was breathless and is put on oxygen mask and GTN, her chest
pain has improved. Her HR=40bpm. ECG shows ST elevation in leads I, II, III. What is your next step?
a. LMWH
b. Streptokinase
c. Angiography
d. Continue current management
e. None

Ans. The key is B. Streptokinase


algorithm for st elevation MI
angioplasty/thrombolysis
b blocker
acei
clopidogrel

407. A 67yo male presents with polyuria and nocturia. His BMI=33, urine culture = negative for
nitrates. What is the next dx inv?
a. PSA
b. Urea, creat and electrolytes
c. MSU culture and sensitivity
d. Acid fast urine test
e. Blood sugar

The key is E. Blood sugar. [Age at presentation and class1 obesity favours the diagnosis of type2 DM].
since culture is -ve for nitrates, so uti is ruled out

408. A pt from Africa comes with nodular patch on the shin which is reddish brown. What is the
most probable dx?
a. Lupus vulgaris
b. Erythema nodosum
c. Pyoderma gangrenosum
d. Erythema marginatum
e. Solar keratosis

The key is B. Erythema nodosum. [Causes of erythema nodosum: MOST COMMON CAUSES- i) streptococcal
infection ii) sarcoidosis. Other causes- tuberculosis, mycoplasma pneumonia, infectious mononucleosis, drugs-
sulfa related drug, OCP, oestrogen; Behcets disease, CD, UC; lymphoma, leukemia and some others].
#Nodes are mostly on anterior aspect of shin

409. A 29yo lady came to the ED with complaints of palpitations that have been there for the past 4 days and
also feeling warmer than usual. Exam: HR=154bpm, irregular rhythm. What is the tx for her condition?
a. Amiadarone
b. Beta blockers
c. Adenosine
d. Verapamil
e. Flecainide

The key is B. Beta blockers [the probable arrhythymia is AF secondary to thyrotoxicosis(heat intolerance). So
to rapid control the symptoms of thyrotoxicosis Beta blocker should be used].

410. A T2DM is undergoing a gastric surgery. What is the most appropriate pre-op management?
a. Start him in IV insulin and glucose and K+ just before surgery
b. Stop his oral hypoglycemic on the day of the procesure
c. Continue regular oral hypoglycemic
d. Stop oral hypoglycemic the prv night and start IV insulin with glucose and K+ before
surgery
e. Change to short acting oral hypoglycemic

The key is D. Stop oral hypoglycemic the prv night and start IV insulin with glucose and K+ before surgery.
411. A 19yo boy is brought by his mother with complaint of lack of interest and no social interactions. He has
no friends, he doesnt talk much, his only interest is in collecting cars/vehicles having around 2000 toy cars.
What is the most appropriate dx?
a. Borderline personality disorder
b. Depression
c. Schizoaffective disorder
d. Autistic spectrum disorder
The key is D. Autistic spectrum disorder.
Autism spectrum disorders affect three different areas of a child's life:
Social interaction
Communication -- both verbal and nonverbal
Behaviors and interests
In some children, a loss of language is the major impairment. In others, unusual behaviors (like spending hours
lining up toys) seem to be the dominant factors.
412. A 45yo man who is diabetic and HTN but poorly compliant has chronic SOB, develops severe SOB and
chest pain. Pain is sharp, increased by breathing and relieved by sitting forward. What is the single most
appropriate dx?
a. MI
b. Pericarditis
c. Lung cancer
d. Good pastures syndrome
e. Progressive massive fibrosis
The key is B. Pericarditis. [Nature of pain i.e. sharp pain increased by breathing and relieved by sitting forward
is suggestive of pericarditis].
Nature of pericardial pain: the most common symptom is sharp, stabbing chest pain behind the sternum or in
the left side of your chest. However, some people with acute pericarditis describe their chest pain as dull, achy
or pressure-like instead, and of varying intensity.
The pain of acute pericarditis may radiate to your left shoulder and neck. It often intensifies when you cough,
lie down or inhale deeply. Sitting up and leaning forward can often ease the pain.
Ecg widespread st elevation
Tx: ansaid
413. A 6m boy has been brought to ED following an apneic episode at home. He is now completely well but his
parents are anxious as his cousin died of SIDS at a similar age. The parents ask for guidance on BLS for a baby
of his age. What is the single most recommended technique for cardiac compressions?
a. All fingers of both hands
b. All fingers of one hand
c. Heel of one hand
d. Heel of both hand
e. Index and middle fingertips of one hand
The key is E. Index and middle fingertips of one hand.

414. A 70yo man had a right hemicolectomy for cecal carcinoma 6days ago. He now has abdominal distension
and recurrent vomiting. He has not opened his bowels since surgery. There are no bowel sounds. WBC=9,
Temp=37.3C. What is the single most appropriate next management?
a. Antibiotic therapy IV
b. Glycerine suppository
c. Laparotomy
d. NG tube suction and IV fluids
e. TPN

The key is D. NG tube suction and IV fluids. [The patient has developed paralytic ileus which should be treated
conservatively].
s/s of paralytic ileus
1. diffuse abd pain
2. constipation
3. abd distension
4. nausea vomitis may contain bile
INV : abd x ray errect+ serum electrolytes
TX : conservative
1. npo
2. ng +iv fluids

215. A 60yo man with a 4y hx of thirst, urinary freq and weight loss presents with a deep painless
ulcer on the heel. What is the most appropriate inv?
a. Arteriography
b. Venography
c. Blood sugar
d. Biopsy for malignant melanoma
e. Biopsy for pyoderma

The key is C. Blood sugar. [The patient probably developed diabetic foot].
the next step wd be doppler scan to assess the vascular status

416. A 16yo boy presents with rash on his buttocks and extensor surface following a sore throat.
What is the most probable dx?
a. Measles
b. Bullous-pemphigoid
c. Rubella
d. ITP
e. HSP

it's a wrong key


right ans is E
# In HSP rash typically found in buttocks, legs and feets and may also appear on the arms, face and trunk.
in ITP it mostly occurs in lower legs. #HSP usually follow a sore throat and ITP follow viral infection like flue or
URTI.
# HSP is a vasculitis while ITP is deficiency of platelets from more destruction in spleen which is immune
mediated].

417. A 34yo man with a white patch on the margin of the mid-third of the tongue. Which is the single most
appropriate LN involved?
a. External iliac LN
b. Pre-aortic LN
c. Aortic LN
d. Inguinal LN
e. Iliac LN
f. Submental LN
g. Submandibular LN
h. Deep cervical LN

The key is G. Submandibular LN.

418. A 50yo lady presents to ED with sudden severe chest pain radiating to both shoulder and
accompanying SOB. Exam: cold peripheries and paraparesis. What is the single most appropriate
dx?
a. MI
b. Aortic dissection
c. Pulmonary embolism
d. Good pastures syndrome
e. Motor neuron disease
The key is B. Aortic dissection. [Usual management for type A dissection is surgery and for type B is
conservative].
Dissecting aortic aneurysm. pt history of chest pain & interscapular back pain indicate dissecting aneurysm in
the descending thoracic aorta can causes interference with the blood supply to the anterior spinal artery and
causes the infarction of the ant aspect of the spinal artery that is anterior spinal artery syndrome and
paraparesis. Treatment- Type A: Immediately IV labetalol for control of HTN to reduces the extension of
dissection then for surgical Mx but Type-B for only conservative Mx.
STANFORD CLASSIFICATION
1. TYPE A : INVOLVING ASCENDING AORTA
2. TYPE B: DOES NOT INVOLVE ASCENDING AORTA

419. A 54yo myopic develops flashes of light and then sudden loss of vision. That is the single most
appropriate tx?
a. Panretinal photocoagulation
b. Peripheral iridectomy
c. Scleral buckling
d. Spectacles
e. Surgical extraction of lens

The key is C. Scleral buckling.


DX: RETINAL DETACHMENT

420. A 40yo chronic alcoholic who lives alone, brought in the ED having been found confused at
home after a fall. He complains of a headache and gradually worsening confusion. What is the
most likely dx?
a. Head injury
b. Hypoglycemia
c. Extradural hematoma
d. Subdural hematoma
e. Delirium

The key is D. Subdural hematoma. [subdural hematoma may be acute or chronic. In chronic symptoms may
not be apparent for several days or weeks. Symptoms of subdural hematomas are: fluctuating level of
consciousness, insidious physical or intellectual slowing, sleepiness, headache, personality change and
unsteadiness.
TX: SURGERY e.g. via barr twist drill and burr hole craniostomy 1st line. Craniotomy if the clot organized 2nd
line].
MOST COMMON IN OLD PEOPLE AND DRUNKS WITH H/O FREQUENT FALLS

421. A 54yo man with alcohol dependence has tremor and sweating 3days into a hosp admission for a fx
femur. He is apprehensive and fearful. What is the single most appropriate tx?
a. Acamprossate
b. Chlordiazepoxide
c. Lorazepam
d. Lofexidine
e. Procyclidine

Ans. The key is B. Chlordiazepoxide. [This is a case of alcohol withdrawal syndrome. Chlordiazepoxide when
used in alcohol withdrawal it is important not to drink alcohol while taking Chlordiazepoxide.
Chlordiazepoxide should only be used at the lowest possible dose and for a maximum of up to four weeks.
This will reduce the risks of developing tolerance, dependence and withdrawal].
422. A 5yo child complains of sore throat and earache. He is pyrexial. Exam: tonsils enlarged and
hyperemic, exudes pus when pressed upon. What is the single most relevant dx?
a. IM
b. Acute follicular tonsillitis
c. Scarlet fever
d. Agranulocytosis
e. Acute OM

Ans. The key is B. Acute follicular tonsillitis. [Tonsillitis is usually caused by a viral infection or, less commonly,
a bacterial infection. The given case is a bacterial tonsillitis (probably caused by group A streptococcus).
There are four main signs that tonsillitis is caused by a bacterial infection rather than a viral infection. They
are:
a high temperature
white pus-filled spots on the tonsils
no cough
swollen and tender lymph nodes (glands).

423. A man with a fam hx of panic disorder is brought to the hosp with palpitations, tremors,
sweating and muscles tightness on 3 occasions in the last 6 wks. He doesnt complain of
headache and his BP is WNL. What is the single most appropriate long-term tx for him?
a. Diazepam
b. Olanzapine
c. Haloperidol
d. Fluoxetine
e. Alprazolam

Ans. The key is D. Fluoxetine. [Recommended treatment for panic disorder is i) CBT ii) Medication (SSRIs or
TCA). NICE recommends a total of seven to 14 hours of CBT to be completed within a four month period.
Treatment will usually involve having a weekly one to two hour session. When drug is prescribed usually a SSRI
is preferred. Antidepressants can take two to four weeks before becoming effective].

424. A 28yo man presents with rapid pounding in the chest. He is completely conscious throughout. The ECG
was taken (SVT). What is the 1st med to be used to manage this condition?
a. Amiodarone
b. Adenosine
c. Lidocaine
d. Verapamil
e. Metoprolol

Ans. The key is B. Adenosine. [Management of SVT: i) vagal manoeuvres (carotid sinus message, valsalva
manoeuvre) transiently increase AV-block, and unmask the underlying atrial rhythm. If unsuccessful then the
first medicine used in SVT is adenosine, which causes transient AV block and works by i) transiently slowing
ventricles to show the underlying atrial rhythm ii) cardioverting a junctional tachycardia to sinus rhythm.
OHCM].

425. A 56yo woman who is depressed after her husband died of cancer 3m ago was given
amitryptaline. Her sleep has improved and she now wants to stop medication but she still
speaks about her husband. How would you manage her?
a. CBT
b. Continue amitryptaline
c. Psychoanalysis
d. Bereavement counselling
e. Antipsychotic

Ans. The key is B. Continue amitriptyline. [depression is important feature of bereavement. Patient may pass
sleepless nights. As this patients sleep has improved it indicate he has good response to antidepressant and as
he still speaks about her husband there is chance to deterioration of her depression if antidepressant is
stopped. For depressive episodes antidepressants should be continued for at least 6-9 months

351. A 35yo lady presents with painful ulcers on her vulva, what is the appropriate inv which will lead to the
dx?
a. Anti-HSV antibodies
b. Dark ground microscopy of the ulcer
c. Treponema palladium antibody test
d. Rapid plasma regain test
e. VDRL
Dx genital herpes
Ans. key A. Anti-HSV antibodies. [Genital Herpes may be asymptomatic or may remain dormant for months or
even years. When symptoms occur soon after a person is infected, they tend to be severe. They may start as
multiple small blisters that eventually break open and produce raw, painful sores that scab and heal over
within a few weeks. The blisters and sores may be accompanied by flu-like symptoms with fever and swollen
lymph nodes.
treatment : There are three major drugs commonly used to treat genital herpes symptoms: acyclovir
(Zovirax), famciclovir (Famvir), and valacyclovir(Valtrex). These are all taken in PO. Severe cases may be
treated with the intravenous (IV) drug acyclovir].
options B C D & E are tests for syphilis which presents with single painless ulcer (canchre)

352. A 53yo man presents with a longstanding hx of a 1cm lesion on his arm. It has started
bleeding on touch. What is the most likely dx?
a. Basal cell carcinoma
b. Kaposis sarcoma
c. Malignant melanoma
d. Squamous cell carcinoma
e. Kerathoacanthoma

Ans. D Squamous cell carcinoma. [SSCs Arises in squamous cells. SCCs may occur on all areas of the body
including the mucous membranes and genitals, but are most common in areas frequently exposed to the sun,
such as the rim of the ear, lower lip, face, balding scalp, neck, hands, arms and legs. SCCs often look like scaly
red patches, open sores, elevated growths with a central depression, or warts; they may crust or bleed.
investigation: tissue sample (biopsy) will be examined under a microscope to arrive at a diagnosis.
prognosis : Squamous cell carcinomas detected at an early stage and removed promptly are almost always
curable and cause minimal damage].
basal cell carcinoma is usually on face with inverted margins
malignant melanoma is on sun exposed parts and is dark (black colored) ulcer
353. A 47yo man with hx of IHD complains of chest pain with SOB on exertion over the past few days. ECG
normal, Echo= increased EF and decreased septal wall thickness. What is the most likely dx?
a. Dilated CM
b. Constrictive pericarditis
c. Amyloidosis
d. Subacute endocarditis

Ans. The key is A. Dilated CM.


points in fav: sob, palpitation, dec septal wall thinning
treatment : beta blocker, acei, diuretics

Constrictive pericarditis doesnt fits because it starts with urti has pain on lying flat which is relieved by leaning
forward
ecg shows wide spread st elevation
Amyloid deposition in the heart can cause both diastolic and systolic heart failure. EKG changes may be
present, showing low voltage and conduction abnormalities like atrioventricular block or sinus node
dysfunction. On echocardiography the heart shows restrictive filling pattern, with normal to mildly reduced
ejec fraction

354. An elderly pt who is known to have DM presents to the hospital with drowsiness, tremors and confusion.
What inv should be done to help in further management?
a. Blood sugar
b. ECG
c. Standing and lying BP
d. Fasting blood sugar
e. CT

Ans. The key is A. Blood sugar.since he is known diabetic he may have gotten hypoglycemic d/t his meds

355. A 28yo pregnant woman with polyhydramnios and SOB comes for an anomaly scan at 31 wks. US=
absence of gastric bubble. What is the most likely dx?
a. Duodenal atresia
b. Esophageal atresia
c. Gastrochiasis
d. Exomphalos
e. Diaphragmatic hernia

Ans. The key is B. Oesophageal atresia.

This condition is visible, after about 26 weeks, on an ultrasound. On antenatal USG, the finding of an absent
or small stomach in the setting of polyhydramnios used to be considered suspicious of esophageal atresia.
However, these findings have a low positive predictive value. The upper neck pouch sign is another sign that
helps in the antenatal diagnosis of esophageal atresia and it may be detected soon after birth as the
affected infant will be unable to swallow its own saliva. Also, the newborn can present with gastric
distention, cough, apnea, tachypnea, and cyanosis. In many types of esophageal atresia, a feeding tube will
not pass through the esophagus.
356. A 1m boy has been brought to the ED, conscious but with cool peripheries and has HR=222bpm. He has
been irritable and feeding poorly for 24h. CXR=borderline enlarged heart with clear lung fields. ECG=regular
narrow complex tachycardia, with difficulty identifying p wave. What is the single most appropriate immediate
tx?
a. Administer fluid bolus
b. Administer oxygen
c. Oral beta-blockers
d. Synchronized DC cardio-version
e. Unilateral carotid sinus massage

The key is D. Synchrnized DC cardioversion.


reason: As the patient is in probable hemodynamic instability (suggested by cool peripheries) so we should go
for DC cardioversion.
diagnosis SVT.

357. A 7yo child presented with chronic cough and is also found to be jaundiced on examination.
What is the most likely dx?
a. Congenital diaphragmatic hernia
b. Congenital cystic adenematoid malformation
c. Bronchiolitis
d. RDS
e. Alpha 1 antitrypsin deficiency

The key is E. Alpha 1 antitrypsin deficiency.


REASON. Unexplained liver disease with respiratory symptoms are very suggestive of AATD.
liver disease occurs because of the accumulation AAT in it
where as d/t inability to be transported out of liver AATD causes emphysema hence the resp problems

358. A 35yo construction worker is dx with indirect inguinal hernia. Which statement below best
describes it?
a. Passes through the superficial inguinal ring only
b. Lies above and lateral to the pubic tubercle
c. Does not pass through the superficial inguinal ring
d. Passes through the deep inguinal ring

Ans. The key is D. Passess through the deep inguinal ring.


direct hernia passes forectly through the posterior wall of inguinal canal whereas indirect can only do so via
deep ring

359. A woman has numerous painful ulcers on her vulva. What is the cause?
a. Chlamydia
b. Trichomonas
c. Gardenella
d. HSV
e. EBV

Ans. The key is D. HSV. reason has been explained in q 351


360. A 72 yo man has been on warfarin for 2yrs because of past TIA and stroke. What is the most important
complication that we should be careful with?
a. Headache
b. Osteoporosis
c. Ear infection
d. Limb ischemia
e. Diarrhea

Ans. key is wrong


right key is A Headache, as there are chances of SAH or generally ICH

361. A 55yo man has been admitted for elective herniorraphy. Which among the following can be the reason
to delay his surgery?
a. Controlled asthma
b. Controlled atrial fib
c. DVT 2yrs ago
d. Diastolic BP 90mmHg
e. MI 2 months ago

Ans. E SAFER TO DO SURGERY AFTER 6 MONTHS

362. A 65yo known case of liver ca and metastasis presents with gastric reflux and bloatedness. On bone exam
there is osteoporosis. He also has basal consolidation in the left lung. What is the next appropriate step?
a. PPI IV
b. Alendronate
c. IV antibiotics
d. Analgesic
e. PPI PO
IN THIS case reflux is the cause of recurrent pneumonia so both C AND E can be right but to chose single one
E is more appropriate

363. A 66yo man has the following ECG. What is the most appropriate next step in management?
a. Metoprolol
b. Digoxin
c. Carotid sinus massage
d. Adenosine
e. Amiodarone.

Ans. A beta blocker for A FIB

364. A 22yo sexually active male came with 2d hx of fever with pain in scrotal area. Exam: scrotal skin is red
and tender. What is the most appropriate dx?
a. Torsion of testis
b. Orchitis
c. Inguinal hernia
d. Epididymo-orchitis
D Epididymo-orchitis.
In orchitis there should be fever, elevation of testes reduces pain (positive prehn sign), In torsion testis lies at a
higher level. In torsion urinalysis negative but in orchitis it is positive. Orchitis usually occurs in sexually active
man. X

365. A man on warfarin posted for hemicolectomy. As the pt is about to undergo surgery. What
option is the best for him?
a. Continue with warfarin
b. Continue with warfarin and add heparin
c. Stop warfarin and add aspirin
d. Stop warfarin and add heparin
e. Stop warfarin

D Stop warfarin and add heparin


5 DAYS BEFORE SURGERY WARFARIN MUST BE REPLACED BY HEPARIN,
366. A 65yo known alcoholic is brought into hospital with confusion, aggressiveness and
ophthalmoplegia. He is treated with diazepoxide. What other drug would you like to prescribe?
a. Antibiotics
b. Glucose
c. IV fluids
d. Disulfiram
e. Vit B complex

E Vitamin B complex. [confusion and ophthalmoplegia points towards the diagnosis of Wernickes
encephalopathy].
which occurs d/t thiamine def.

367. A 32yo woman has severe right sided abdominal pain radiating into the groin which has lasted for 3h. She
is writhering in pain. She has no abdominal signs. What is the most likely cause of her abdominal pain?
a. Appendicitis
b. Ruptured ectopic pregnancy
c. Salpingitis
d. Ureteric colic
e. Strangulated hernia

D Ureteric colic.
It indicate stone at lower ureter. [i) Pain from upper ureteral stones tends to radiate to the flank and lumbar
areas. ii) Midureteral calculi cause pain that radiates anteriorly and caudally. This midureteral pain in
particular can easily mimic appendicitis on the right or acute diverticulitis on the left. iii) Distal ureteral stones
cause pain that tends to radiate into the groin or testicle in the male or labia majora in the female.

368. A 39yo coal miner who smokes, drinks and has a fam hx of bladder cancer is suffering from BPH. The
most important risk factor for his bladder carcinoma is?
a. Fam hx
b. Smoking
c. Exposure to coal mine
d. BPH
B smoking.
. Risk factors of bladder cancer:
i) Smoking ii) Exposure to chemicals used in dye industry iii) Whites are more likely to develop bladder cancer
iv) Risk increases with age v) More common in men vi) Chronic bladder irritation and infections (urinary
infections, kidney and bladder stones, bladder catheter left in place a long time.) vii) Personal history of
bladder or other urothelial cancer viii) Family history ix) Chemotherapy or radiotherapy x) Pioglitazone for
more than one year and certain herb xi) Arsenic in drinking water xii) Low fluid consumption.

369. A 34yo woman is referred to the endocrine clinic with a hx of thyrotoxicosis. At her 1st
appointment she is found to have a smooth goiter, lid lag and bilateral exophthalmos with puffy
eyelids and conjunctival injection. She wants to discuss the tx of her thyroid prb as she is keen to become
pregnant. What is the most likely tx you would advise?
a. 18m of carbimazole alone
b. 18m of PTU alone
c. A combo od anti-thyroid drug and thyroxine
d. Radioactive iodine
e. Thyroidectomy

B 18m of PTU alone.


Other drug option i.e Carbamazepine is teratogenic [can cause i) spina bifida ii)cardiovascular malformations
ETC . PTU is on the other hand relatively safe in pregnancy.

370. A child living with this stepfather is brought by the mother with multiple bruises, fever and
fractures. What do you suspect?
a. NAI
b. Malnutrition
c. Thrombocytopenia
d. HIV

Ans. The key is A. NAI. [H/O living with stepfather, multiple bruises, fever and fractures are suggestive of NAI].
OTHER possible points can include hx not matching with bruises, wounds which are a day or two older at the
time of presentation.

371. A young man who was held by the police was punched while in custody. He is now cyanosed and
unresponsive. What is the 1st thing you would do?
a. IV fluids
b. Clear airway
c. Turn pt and put in recovery position
d. Give 100% oxygen
e. Intubate and ventilate

B. Clear airway. [ABC protocol].


372. A HTN male loses vision in his left eye. The eye shows hand movement and a light shined in the eye is
seen as a faint light. Fundus exam: flame shaped hemorrhages. The right eye is normal. What is the cause of
this pts unilateral blindness?
a. HTN retinopathy
b. CRA thrombosis
c. CRV thrombosis
d. Background retinopathy
e. Retinal detachment

key is wrong
right ans is C ( unilateral blindness with flameshaped hemorrhages are characteristic of CRVO)

373. A mentally retarded child puts a green pea in his ear while eating. The carer confirms this.
Otoscopy shows a green colored object in the ear canal. What is the most appropriate single
best approach to remove this object?
a. By magnet
b. Syringing
c. Under GA
d. By hook
e. By instilling olive oil

C UNDER GA
since child is retard, if he wasnt then the ans would be D
olive oil is for insects

374. A pt presents with longstanding gastric reflux, dysphagia and chest pain. On barium enema, dilation of
esophagus with tapering end is noted. He was found with Barretts esophagus. He had progressive dysphagia
to solids and then liquids. What is the single most appropriate dx?
a. Achalasia
b. Esophageal spasm
c. GERD
d. Barretts esophagus
e. Esophageal carcinoma
E Oesophageal carcinoma. [there is dilatation in oesophagus which is seen both in achalasia and carcinoma.
Dysphagia to solid initaially is very much suggestive of carcinoma and also barretts change is a clue to
carcinoma]
Progressive dysphagia with h/o barrett esophagus are the key indicators

375. A 48yo lady presents with itching, excoriations, redness, bloody discharge and ulceration around her
nipple. What is the most likely dx?
a. Pagets disease of the breast
b. Fibrocystic dysplasia
c. Breast abscess
d. Duct papilloma
e. Eczema

A Pagets disease of the breast.


TYPICAL manifestation of pagets disease
eczema like rash involving nipple and areola with straw or bloody discharge
Also nipple turns inward in advances stages pt complaints of burning sensation at the site of lesion
DX mammography and biopsy
TX surgery + chemo or radio may be needed
376. Pt with widespread ovarian carcinoma has bowel obstruction and severe colic for 2h and was normal in
between severe pain for a few hours. What is the most appropriate management?
a. PCA (morphine)
b. Spasmolytics
c. Palliative colostomy
d. Oral morphine
e. Laxatives

C. Palliative colostomy.
Cancer or chemotherapy induced obstructions are unlikely to respond to conservative management [NBM, IV
fluid, nasogastric suction] and hence only analgesia will not relieve it. So in such cases we have to go for
palliative colostomy.

377. A 70yo man admits to asbestos exposure 20yrs ago and has attempted to quit smoking. He has noted
weight loss and hoarseness of voice. Choose the single most likely type of cancer a.w risk
factors present.
a. Basal cell carcinoma
b. Bronchial carcinoma
c. Esophageal carcinoma
d. Nasopharyngeal carcinoma
e. Oral carcinoma

B. Bronchial carcinoma. [Asbestos exposure is a risk factor for lung cancer and also has a synergistic effect with
cigarette smoke].
#. Conditions related to asbestos exposure: i) Pleural plaques (after a latent period of 20-40 yrs) ii) Pleural
thickening iii) Asbestosis (latent period is typically 15-30 yrs) iv) Mesothelioma (prognosis is very poor) v) Lung
cancer.

378. A 32yo woman had progressive decrease in vision over 3yrs. She is no dx as almost blind. What would be
the mechanism?
a. Cataract
b. Glaucoma
c. Retinopathy
d. Uveitis
e. Keratitis

B. Glaucoma.
. Cataract is unlikely at this age. Nothing in the history suggests retinopathy. Uveitis and iritis doesnt have
such degree of vision loss and iritis and anterior uveitis have pain, redness and photophobia. Open angle
glaucoma is likely cause.
379. A child during operation and immediately after showed glycosuria, but later his urine sugar was normal.
Choose the most probable dx.
a. Pre-diabetic state
b. Normal finding
c. Low renal tubular threshold
d. DM

B Normal finding.
Stress during operation can cause transient hyperglycemia causing glycosuria secondary to stress induced rise
of cortisole which becomes normal after some time.
380. A pt presented with hx of swelling in the region of the sub-mandibular region, which became
more prominent and painful on chewing. He also gave hx of sour taste in the mouth, the area is
tender on palpation. Choose the most probable dx?
a. Chronic recurrent sialadenitis
b. Adenolymphoma
c. Mikuliczs disease
d. Adenoid cystic carcinoma
e. Sub-mandibular abscess
A Chronic recurrent sialadenitis. [pain, swelling, more pain on chewing, tenderness, and submandibular region
suggests diagnosis of submandibular chronic recurrent sialadenitis, usually secondary to sialolithiasis or
stricture].
381. ECG of an 80yo pt of ICH shows saw-tooth like waves, QRS complex of 80ms duration,
ventricular rate=150/min and regular R-R interval. What is the most porbable dx?
a. Atrial fib
b. Atrial flutter
c. SVT
d. Mobitz type1 second degree heart block
e. Sinus tachycardia
B Atrial flutter. [Saw-tooth like waves, normal QRS comples of 80 ms (normal range 70-100 ms), ventricular
rate of 150/min and regular R-R interval is diagnostic of atrial flutter].
FOR AFIB THERE WD BE IRREGULARARLY IRREGULAR RHYTHM

382. A 50 yo woman who was treated for breast cancer 3 yrs ago now presents with increase thirst and
confusion. She has become drowsy now. What is the most likely metabolic abnormality?
a. Hypercalcemia
b. Hyperkalemia
c. Hypoglycemia
d. Hyperglycemia
e. Hypercalcemia.
E HYPERCALCEMIA
Ans. 2. Increased thirst, confusion, drowsiness these are features of hypercalcemia. Any solid organ tumour
can produce hypercalcemia. Here treated Ca breast is the probable cause of hypercalcemia.
383. A 29yo woman presents to her GP with a hx of weight loss, heat intolerance, poor conc and
palpitations. Which of the following is most likely to be a/w dx of thyroiditis a/w viral infection?
a. Bilateral exophthalmos
b. Diffuse, smooth goiter
c. Reduced uptake on thyroid isotope scan
d. Positive thyroid peroxidase antibodies
e. Pretibial myxedema
C. Reduced uptake on thyroid isotope scan.
DX De Quervains or subacute thyroiditis.
. Viral or subacute thyroiditis: diagnostic criteria: i) Features of hyperthyroidism present. ii) Pain thyroid, not
mentioned. iii) Investigations: high esr (60-100) not mentioned, Reduced uptake of radioactive iodine by the
gland.
384. A lady, post-colostomy closure after 4days comes with fluctuating small swelling in the stoma.
What is the management option for her?
a. Local exploration
b. Exploratory laparotomy
c. Open laparotomy
d. Reassure
A Local exploration.
THERE MUST BE SOME LOCAL WOUND PROBLEM
385. A 65yo female pt was given tamoxifen, which of the following side effect caused by it will
concern you?
a. Fluid retention
b. Vaginal bleeding
c. Loss of apetite
d. Headache and dizziness
e.

B Vaginal bleeding.
. Tamoxifen can promote development of endometrial carcinoma. So vaginal bleeding will be of concern for
us.
386. A 39yo man with acute renal failure presents with palpitations. His ECG shows tall tented T
waves and wide QRS complex. What is the next best step?
a. Dialysis
b. IV calcium chloride
c. IV insulin w/ dextrose
d. Calcium resonium
e. Nebulized salbutamol

B. IV calcium chloride (both IV calcium gluconate or IV calcium chloride can be used) when there is ECG
changes.
DX The ECG changes are suggestive of Hyperkalemia.
At potassium level of >5.5mEq/L occurs tall tented T waves and at potassium level >7mEq/L occurs wide QRS
complex with bizarre QRS morphology.

387. A 54yo pt 7 days after a total hip replacement presents with acute onset breathlessness and
raised JVP. Which of the following inv will be most helpful in leading to a dx?
a. CXR
b. CTPA
c. V/Q scan
d. D-Dimer
e. Doppler US of legs
The key is B. CTPA.
The patient has a +ve two level PE Wells score (if it was negative we should do D-Dimer) and there is no renal
impairment or history suggestive of allergy to contrast media (if these present we should have go for VQ scan)
the investigation of choice is PTCA. NICE guideline.

388. A 7yo girl has been treated with penicillin after sore throat, fever and cough. Then she
develops skin rash and itching. What is the most probable dx?
a. Erythema nodosum
b. Erythema multiforme
c. SJS
d. Erythema marginatum
e. Erythema gangernosum

. The key is B. Erythema multiforme.


Common drugs causing erythma multiforme are: antibiotics (including, sulphonamides, penicillin),
anticonvulsants (phenytoin,barbiturates), aspirin, antituberculoids, and allopurinol.
CLOSELY related option is SJS which would have muco cutaneous rash but in here we hav only cutaneous rash

389. A 60yo man presented with a lump in the left supraclavicular region. His appetite is
decreased and he has lost 5kg recently. What is the most probably dx?
a. Thyroid carcinoma
b. Stomach carcinoma
c. Bronchial carcinoma
d. Mesothelioma
e. Laryngeal carcinoma

The key is B. Stomach carcinoma. [Mentioned lump in the left supraclavicular region is Vershows gland, has
long been regarded as strongly indicative of the presence of cancer in the abdomen, specifically gastric
cancer].

390. A 64yo man has presented to the ED with a stroke. CT shows no hemorrhage. ECG shows atrial fib. He has
been thrombolysed and hes awaiting discharge. What prophylactic regimen is best for him?
a. Warfarin
b. Heparin
c. Aspirin
d. Statins
e. Beta blockers

The key is A. Warfarine. [Atrial fibrillation: post stroke- following a stroke or TIA warfarine should be given as
the anticoagulant of choice. NICE guideline].
391. A 54yo man after a CVA presents with ataxia, intention tremors and slurred speech. Which part of the
brain has been affected by the stroke?
a. Inner ear
b. Brain stem
c. Diencephalon
d. Cerebrum
e. Cerebellum

The key is E. Cerebellum.


i) Ataxia ii) slurred speech or dysarthria iii) dysdiodokokinesis iv) intention tremor v) nystagmus. are the signs
of cerebellar defect

292. A 57yo man with blood group A complains of symptoms of vomiting, tiredness, weight loss and
palpitations. Exam: hepatomegaly, ascites, palpable left supraclavicular mass. What is the most likely dx?
a. Gastric carcinoma
b. Colorectal carcinoma
c. Peptic ulcer disease
d. Atrophic gastritic
e. Krukenburg tumor

Ans. The key is A. Gastric carcinoma. [i) blood group A is associated with gastric cancer ii) vomiting, tiredness,
weight loss are general features of gastric cancer iii) palpitation from anemia of cancer iv) hepatomegaly and
ascites are late features of gastric cancer. v) palpable left supraclavicular mass- is Vershows gland, has long
been regarded as strongly indicative of gastric cancer].

293. A 21yo girl looking unkempt, agitated, malnourished and nervous came to the hospital asking
for painkillers for her abdominal pain. She is sweating, shivering and complains of joint pain.
What can be the substance misuse here?
a. Alcohol
b. Heroin
c. Cocaine
d. LSD
e. Ecstasy

The key is B. Heroin. [agitation, nervousness, abdominal cramp, sweating, shivering and piloerection,
arthralgia these are features of heroin withdrawal].

394. A child presents with increasing jaundice and pale stools. Choose the most appropriate test?
a. US abdomen
b. Sweat test
c. TFT
d. LFT
e. Endomyseal antibodies

The key is A. US abdomen. [This is a picture suggestive of obstructive jaundice. LFT can give clue like much
raised bilirubin, AST and ALT not that high and raised alkaline phosphatase but still USG is diagnostic in case of
obstructive jaundice].

395. A 32yo man presents with hearing loss. AC>BC in the right ear after Rhine test. He also
complains of tinnitus, vertigo and numbness on same half of his face. What is the most
appropriate inv for his condition?
a. Audiometry
b. CT
c. MRI
d. Tympanometry
e. Webers test

The key is C. MRI. [features are suggestive of acaustic neuroma, so MRI is the preferred option]. it involves
basically 8th nerve but 6 7 9 and 10th nerves are also involved with it

396. A 56 yo lady with lung cancer presents with urinary retention, postural hypotension, diminished reflexes
and sluggish pupillary reaction. What is the most likely explanation for her symptoms?
a. Paraneoplastic syndrome
b. Progression of lung cancer
c. Brain metastasis
d. Hyponatremia
e. Spinal cord compression

The key is A. Paraneoplastic syndrome.


s/s are of autonomic neuropathy which occurs in paraneoplastic syndrome

397. An old woman having decreased vision cant see properly at night. She has changed her glasses quite a
few times but to no effect. She has normal pupil and cornea. What is the most likely dx?
a. Cataract
b. Glaucoma
c. Retinal detachment
d. Iritis
e. GCA

key is wrong
correct key is A cataract
old age and progressive weakness supports Cataract

398. A pt comes with sudden loss of vision. On fundoscopy the optic disc is normal. What is the
underlying pathology?
a. Iritis
b. Glaucoma
c. Vitreous chamber
d. Retinal detachment

Ans. 1. The Key is D. Retinal detachment.


#Causes of sudden painless loss of vision:
1. Retinal detachment
2. Vitreous haemorrhage
3. Retinal vein occlusion
4. Retinal artery occlusion
5. Optic neuritis
6. Cerebrovascular accident
remember retinal detachment has vision loss as if curtain is coming down
399. A child was woken up from sleep with severe pain in the testis. Exam: tenderness on palpation and only
one testis was normal in size and position. What would be your next step?
a. Analgesia
b. Antibiotics
c. Refer urgently to a surgeon
d. Reassurance
e. Discharge with analgesics

Ans. The key is A. Analgesia. [According to some US sites it is analgesia but no UK site support this!!! So for
Plab exam the more acceptable option is C. Refer urgently to a surgeon].
IN TORSION THE SOONER THE SURGICAL INTERVENTION DONE, THE BETTER THE RESULTS ARE

400. A child suffering from asthma presents with Temp 39C, drooling saliva on to the mothers lap,
and taking oxygen by mask. What sign will indicate that he is deteriorating?
a. Intercostal recession
b. Diffuse wheeze
c. Drowsiness

The key is A. Intercostal recession. [ here intercostals recession and drowsiness both answers are correct.
Hope in exam there will be one correct option]. but to chose among them, better go with A

401. A 12yo boy presents with painful swollen knew after a sudden fall. Which bursa is most likely tobe
affected?
a. Semimembranous bursa
b. Prepatellar bursa
c. Pretibial bursa
d. Suprapatetaller bursa

. The key is B. Prepatellar bursa. [A fall onto the knee can damage the prepatellar bursa. This usually causes
bleeding into the bursa sac causing swellen painful knee. Prepatellar bursitis that is caused by an injury will
usually go away on its own. The body will absorb the blood in the bursa over several weeks, and the bursa
should return to normal. If swelling in the bursa is causing a slow recovery, a needle may be inserted to drain
the blood and speed up the process. There is a slight risk of infection in putting a needle into the bursa].

402. A 61yo man has been referred to the OPD with frequent episodes of breathlessness and chest pain a/w
palpitations. He has a regular pulse rate=60bpm. ECG=sinus rhythm. What is the
most appropriate inv to be done?
a. Cardiac enzymes
b. CXR
c. ECG
d. Echo
e. 24h ECG

The key is E. 24h ECG.


Indications of 24 h ambulatory holter monitoring:

To evaluate chest pain not reproduced with exercise testing


To evaluate other signs and symptoms that may be heart-related, such as fatigue, shortness of breath,
dizziness, or fainting

To identify arrhythmias or palpitations

To assess risk for future heart-related events in certain conditions, such as idiopathic hypertrophic
cardiomyopathy, post-heart attack with weakness of the left side of the heart, or Wolff-Parkinson-White
syndrome

To assess the function of an implanted pacemaker

To determine the effectiveness of therapy for complex arrhythmias

403. A woman dx with Ca Breast presents now with urinary freq. which part of the brain is the
metastasis spread to?
a. Brain stem
b. Pons
c. Medulla
d. Diencephalon
e. Cerebral cortex

The key is D. Diencephalon. [diencephalon is made up of four distinct components: i) the thalamus ii) the
subthalamus iii) the hypothalamus and iv) the epithalamus. Among these the hypothalamus has crucial role in
causing urinary frequency].

404. A man is very depressed and miserable after his wifes death. He sees no point in living now that his wife
is not around and apologises for his existence. He refuses any help offered. His son has brought him to the ED.
The son can.t deal with the father any more. What is the most
appropriate next step?
a. Voluntary admission to psychiatry ward
b. Compulsory admission under MHA
c. Refer to social services
d. Alternate housing
e. ECT

Ans. The key is B. Compulsory admission under MHA. [This patient is refusing any help offered! And his son
cannot deal with him anymore! In this situation voluntary admission to psychiatry ward is not possible and the
option of choice is compulsory admission under MHA].

405. A 31yo man has epistaxis 10 days following polypectomy. What is the most likely dx?
a. Nasal infection
b. Coagulation disorder
c. Carcinoma

The key is A. Nasal infection.


HEMORRHAGE AFTER 5 TO 7 DAYS IS SECONDARY HEMORRHAGE [Infection is one of the most important
cause of secondary hemorrhage].

406. A woman had an MI. She was breathless and is put on oxygen mask and GTN, her chest
pain has improved. Her HR=40bpm. ECG shows ST elevation in leads I, II, III. What is your next step?
a. LMWH
b. Streptokinase
c. Angiography
d. Continue current management
e. None

Ans. The key is B. Streptokinase


algorithm for st elevation MI
angioplasty/thrombolysis
b blocker
acei
clopidogrel

407. A 67yo male presents with polyuria and nocturia. His BMI=33, urine culture = negative for
nitrates. What is the next dx inv?
a. PSA
b. Urea, creat and electrolytes
c. MSU culture and sensitivity
d. Acid fast urine test
e. Blood sugar

The key is E. Blood sugar. [Age at presentation and class1 obesity favours the diagnosis of type2 DM].
since culture is -ve for nitrates, so uti is ruled out

408. A pt from Africa comes with nodular patch on the shin which is reddish brown. What is the
most probable dx?
a. Lupus vulgaris
b. Erythema nodosum
c. Pyoderma gangrenosum
d. Erythema marginatum
e. Solar keratosis

The key is B. Erythema nodosum. [Causes of erythema nodosum: MOST COMMON CAUSES- i) streptococcal
infection ii) sarcoidosis. Other causes- tuberculosis, mycoplasma pneumonia, infectious mononucleosis, drugs-
sulfa related drug, OCP, oestrogen; Behcets disease, CD, UC; lymphoma, leukemia and some others].
#Nodes are mostly on anterior aspect of shin

409. A 29yo lady came to the ED with complaints of palpitations that have been there for the past 4 days and
also feeling warmer than usual. Exam: HR=154bpm, irregular rhythm. What is the tx for her condition?
a. Amiadarone
b. Beta blockers
c. Adenosine
d. Verapamil
e. Flecainide

The key is B. Beta blockers [the probable arrhythymia is AF secondary to thyrotoxicosis(heat intolerance). So
to rapid control the symptoms of thyrotoxicosis Beta blocker should be used].
410. A T2DM is undergoing a gastric surgery. What is the most appropriate pre-op management?
a. Start him in IV insulin and glucose and K+ just before surgery
b. Stop his oral hypoglycemic on the day of the procesure
c. Continue regular oral hypoglycemic
d. Stop oral hypoglycemic the prv night and start IV insulin with glucose and K+ before
surgery
e. Change to short acting oral hypoglycemic

The key is D. Stop oral hypoglycemic the prv night and start IV insulin with glucose and K+ before surgery.
411. A 19yo boy is brought by his mother with complaint of lack of interest and no social interactions. He has
no friends, he doesnt talk much, his only interest is in collecting cars/vehicles having around 2000 toy cars.
What is the most appropriate dx?
a. Borderline personality disorder
b. Depression
c. Schizoaffective disorder
d. Autistic spectrum disorder
The key is D. Autistic spectrum disorder.
Autism spectrum disorders affect three different areas of a child's life:
Social interaction
Communication -- both verbal and nonverbal
Behaviors and interests
In some children, a loss of language is the major impairment. In others, unusual behaviors (like spending hours
lining up toys) seem to be the dominant factors.
412. A 45yo man who is diabetic and HTN but poorly compliant has chronic SOB, develops severe SOB and
chest pain. Pain is sharp, increased by breathing and relieved by sitting forward. What is the single most
appropriate dx?
a. MI
b. Pericarditis
c. Lung cancer
d. Good pastures syndrome
e. Progressive massive fibrosis
The key is B. Pericarditis. [Nature of pain i.e. sharp pain increased by breathing and relieved by sitting forward
is suggestive of pericarditis].
Nature of pericardial pain: the most common symptom is sharp, stabbing chest pain behind the sternum or in
the left side of your chest. However, some people with acute pericarditis describe their chest pain as dull, achy
or pressure-like instead, and of varying intensity.
The pain of acute pericarditis may radiate to your left shoulder and neck. It often intensifies when you cough,
lie down or inhale deeply. Sitting up and leaning forward can often ease the pain.
Ecg widespread st elevation
Tx: ansaid
413. A 6m boy has been brought to ED following an apneic episode at home. He is now completely well but his
parents are anxious as his cousin died of SIDS at a similar age. The parents ask for guidance on BLS for a baby
of his age. What is the single most recommended technique for cardiac compressions?
a. All fingers of both hands
b. All fingers of one hand
c. Heel of one hand
d. Heel of both hand
e. Index and middle fingertips of one hand
The key is E. Index and middle fingertips of one hand.

414. A 70yo man had a right hemicolectomy for ceacal carcinoma 6days ago. He now has abdominal distension
and recurrent vomiting. He has not opened his bowels since surgery. There are no bowel sounds. WBC=9,
Temp=37.3C. What is the single most appropriate next management?
a. Antibiotic therapy IV
b. Glycerine suppository
c. Laparotomy
d. NG tube suction and IV fluids
e. TPN

The key is D. NG tube suction and IV fluids. [The patient has developed paralytic ileus which should be treated
conservatively].
s/s of paralytic ileus
1. diffuse abd pain
2. constipation
3. abd distension
4. nausea vomitis may contain bile
INV : abd x ray errect+ serum electrolytes
TX : conservative
1. npo
2. ng +iv fluids

215. A 60yo man with a 4y hx of thirst, urinary freq and weight loss presents with a deep painless
ulcer on the heel. What is the most appropriate inv?
a. Ateriography
b. Venography
c. Blood sugar
d. Biopsy for malignant melanoma
e. Biopsy for pyoderma

The key is C. Blood sugar. [The patient probably developed diabetic foot].
the next step wd be doppler scan to assess the vascular status

416. A 16yo boy presents with rash on his buttocks and extensor surface following a sore throat.
What is the most probable dx?
a. Measles
b. Bullous-pemphigoig
c. Rubella
d. ITP
e. HSP

its a wrong key


right ans is E
# In HSP rash typically found in buttocks, legs and feets and may also appear on the arms, face and trunk.
in ITP it mostly occurs in lower legs. #HSP usually follow a sorethroat and ITP follow viral infection like flue or
URTI.
# HSP is a vasculitis while ITP is deficiency of platelets from more destruction in spleen which is immune
mediated].

417. A 34yo man with a white patch on the margin of the mid-third of the tongue. Which is the single most
appropriate LN involved?
a. External iliac LN
b. Pre-aortic LN
c. Aortic LN
d. Inguinal LN
e. Iliac LN
f. Submental LN
g. Submandibular LN
h. Deep cervical LN

The key is G. Submandibular LN.

418. A 50yo lady presents to ED with sudden severe chest pain radiating to both shoulder and
accompanying SOB. Exam: cold peripheries and paraparesis. What is the single most appropriate
dx?
a. MI
b. Aortic dissection
c. Pulmonary embolism
d. Good pastures syndrome
e. Motor neuron disease

The key is B. Aortic dissection. [Usual management for type A dissection is surgery and for type B is
conservative].
STANFORD CLASSIFICATION
1. TYPE A : INVOLVING ASCENDING AORTA
2. TYPE B: DOESNOT INVOLVE ASCENDING AORTA

419. A 54yo myopic develops flashes of light and then sudden loss of vision. That is the single most
appropriate tx?
a. Pan retinal photo coagulation
b. Peripheral iridectomy
c. Scleral buckling
d. Spectacles
e. Surgical extraction of lens

The key is C. Scleral buckling.


DX: RETINAL DETACHMENT

420. A 40yo chronic alcoholic who lives alone, brought in the ED having been found confused at
home after a fall. He complains of a headache and gradually worsening confusion. What is the
most likely dx?
a. Head injury
b. Hypoglycemia
c. Extradural hematoma
d. Subdural hematoma
e. Delirium

The key is D. Subdural hematoma. [subdural hematoma may be acute or chronic. In chronic symptoms may
not be apparent for several days or weeks. Symptoms of subdural hematomas are: fluctuating level of
consciousness, insidious physical or intellectual slowing, sleepiness, headache, personality change and
unsteadiness.
TX: SURGERY e.g. via barr twist drill and barr hole craniostomy 1st line. Craniotomy if the clot organized 2nd
line].
MOST COMMON IN OLD PEOPLE AND DRUNKS WITH H/O FREQUENT FALLS

421. A 54yo man with alcohol dependence has tremor and sweating 3days into a hosp admission for a fx
femur. He is apprehensive and fearful. What is the single most appropriate tx?
a. Acamprossate
b. Chlordiazepoxide
c. Lorazepam
d. Lofexidine
e. Procyclidine

Ans. The key is B. Chlordiazepoxide. [This is a case of alcohol withdrawal syndrome. Chlordiazepoxide when
used in alcohol withdrawal it is important not to drink alcohol while taking Chlordiazepoxide.
Chlordiazepoxide should only be used at the lowest possible dose and for a maximum of up to four weeks.
This will reduce the risks of developing tolerance, dependence and withdrawal].
422. A 5yo child complains of sore throat and earache. He is pyrexial. Exam: tonsils enlarged and
hyperemic, exudes pus when pressed upon. What is the single most relevant dx?
a. IM
b. Acute follicular tonsillitis
c. Scarlet fever
d. Agranulocytosis
e. Acute OM

Ans. The key is B. Acute follicular tonsillitis. [Tonsillitis is usually caused by a viral infection or, less commonly,
a bacterial infection. The given case is a bacterial tonsillitis (probably caused by group A streptococcus).
There are four main signs that tonsillitis is caused by a bacterial infection rather than a viral infection. They
are:
a high temperature
white pus-filled spots on the tonsils
no cough
swollen and tender lymph nodes (glands).

423. A man with a fam hx of panic disorder is brought to the hosp with palpitations, tremors,
sweating and muscles tightness on 3 occasions in the last 6 wks. He doesnt complain of
headache and his BP is WNL. What is the single most appropriate long-term tx for him?
a. Diazepam
b. Olanzapine
c. Haloperidol
d. Fluoxetine
e. Alprazolam
Ans. The key is D. Fluoxetine. [Recommended treatment for panic disorder is i) CBT ii) Medication (SSRIs or
TCA). NICE recommends a total of seven to 14 hours of CBT to be completed within a four month period.
Treatment will usually involve having a weekly one to two hour session. When drug is prescribed usually a SSRI
is preferred. Antidepressants can take two to four weeks before becoming effective].

424. A 28yo man presents with rapid pounding in the chest. He is completely conscious throughout. The ECG
was taken (SVT). What is the 1st med to be used to manage this condition?
a. Amiodarone
b. Adenosine
c. Lidocaine
d. Verapamil
e. Metoprolol

Ans. The key is B. Adenosine. [Management of SVT: i) vagal manoeuvres (carotid sinus message, valsalva
manoeuvre) transiently increase AV-block, and unmask the underlying atrial rhythm. If unsuccessful then the
first medicine used in SVT is adenosine, which causes transient AV block and works by i) transiently slowing
ventricles to show the underlying atrial rhythm ii) cardioverting a junctional tachycardia to sinus rhythm.
OHCM].

425. A 56yo woman who is depressed after her husband died of cancer 3m ago was given
amitryptaline. Her sleep has improved and she now wants to stop medication but she still
speaks about her husband. How would you manage her?
a. CBT
b. Continue amitryptaline
c. Psychoanalysis
d. Bereavement counselling
e. Antipsychotic

Ans. The key is B. Continue amitriptyline. [depression is important feature of bereavement. Patient may pass
sleepless nights. As this patients sleep has improved it indicate he has good response to antidepressant and as
he still speaks about her husband there is chance to deterioration of her depression if antidepressant is
stopped. For depressive episodes antidepressants should be continued for at least 6-9 months

426. A 64yo man presents with a hx of left sided hemiparesis and slurred speech. He was absolutely fine 6h
after the episode. What is the most appropriate prophylactic regimen?
a. Aspirin 300mg for 2 weeks followed by aspirin 75mg
b. Aspirin 300mg for 2 weeks followed by aspirin 75mg and dipyridamole 200mg
c. Clopidogrel 75mg
d. Dipyridamole 200mg
e. Aspirin 300mg for 2 weeks

KEY- B
Dx- TIA.
What is TIA?
Inadequate circulation in part of the brain, gives a picture similar to stroke but duration < 24 hours.
Common in old age. Men > women. ^ in black race.
Important risk factors- HTN, smoking, DM, Hyperlipidemia, Heart disease. Management is by: Antiplatelets,
anti HTN, lipid modifying ttt, AF ttt and any risk factors like DM.
Treatment: Aspirin + dypiridamole (each as 300mg loading then 75mg daily) + statin. [NICE guidelines]

427. A 63yo lady with a BMI=32 comes to the ED with complaints of pigmentation on her legs. Exam: dilated
veins could be seen on the lateral side of her ankle. Which of the following is involved?
a. Short saphenous vein
b. Long saphenous vein
c. Deep venous system
d. Popliteal veins
e. Saphano-femoral junction

KEY- A
Short saphenous vein- lateral side
Long saphenous vein- medial side
*Long saphenous vein is the vessel of choice used for autotransplantation in coronary artery bypass. It is also a
common site for varicose vein formation.

428. A 55yo man presents with hx of weight loss and tenesmus. He is dx with rectal carcinoma. Which risk
factors help to develop rectal carcinoma except following?
a. Smoking
b. Family hx
c. Polyp
d. Prv carcinoma
e. High fat diet
f. High fibre diet

KEY- F
All options except High fiber diet are risk factors for developing rectal carcinoma.
*Other risk factors for Rectal Carcinoma are:
-IBD
-Nulliparity and early menopause
-Diet rich in meat and fat, poor in folate and Calcium
-Sedentary lifestyle, obesity, smoking and high alcohol intake.
-Diabetes
-Radiation and asbestos exposure

429. A pt presents with a painful, sticky red eye with a congested conjunctiva. What is the most suitable tx?
a. Antibiotic PO
b. Antihistamine PO
c. Antibiotic drops
d. Steroid drops
e. IBS
KEY- C
Dx- Bacterial Conjunctivitis.
Painful eye, usually bilateral. Smearing of vision on waking up.
Mild photophobia. If severe, indicates corneal involvement or adenoviral conjunctivitis.
Thick yellowish-white mucopurulent discharge. Visual acuity is normal
Symptoms- Red eye, difficult to open in the morning, glued together by discharge. Presence of follicles on the
conjunctiva- More likely viral conjunctivitis.

Treatment:
Topical broad spectrum antibiotics. Drug of choice is chloramphenicol drops. If pregnant, intolerant to
chloramphenicol or history of aplastic anemia or blood dyscrasia, use fusidic acid.

430. A 45yo woman complains of pain in her hands precipitated by exposure to the cold weather. She is
breathlessness on walking. When she is eating, she can feel food suddenly sticking to the gullet. It seems to be
in the middle of the esophagus but she cant localize exactly where it sticks. It is usually relieved with a drink
of water. Choose the single most likely cause of dysphagia from the options?
a. Esophageal carcinoma
b. Systemic sclerosis
c. SLE
d. Pharyngeal carcinoma
e. Globus hystericus

KEY-B
We can rule out option A and D simply because she presents with systemic complaints, and these two will
cause only local signs.
*Globus hystericus is when a patient feels like they have a lump in their throat, when infact they dont.
Examination is completely normal.
*This leaves SLE and systemic sclerosis. In SLE, there is the condition mentioned in this question (Raynauds
phenomenon) but NO DYSPHAGIA.
->Systemic sclerosis (SS) is classified into 2 types- Limited cutaneous SS (70%) and Diffuse cutaneous SS (30%)
according to extent of skin involvement.
-Limited SS formerly called CREST syndrome
Calcinosis
Raynauds phenomenon- cardinal sign, early and very common presentation.
Esophageal dysmotility
Sclerodactyly
Telangiectasia

431. A 3yo child brought to the ED with a swelling over the left arm. XR shows multiple callus formation in
the ribs. Exam: bruises on child's back. What is the most appropriate next step?
a. Check child protection register
b. Coagulation profile
c. Skeletal survey
d. Serum calcium
e. DEXA scan

KEY- C
Dx- This is a case of Non accidental injury (NAI) i.e. child abuse.
*The clinchers are the multiple calluses in the ribs and the bruises on the childs back, denoting repeated
trauma. The injuries are often multiple, frequent or of different ages. Abusers almost always go to the ED and
not their family GP since the chances of meeting the same ER doctor twice is less, hence the chance of
someone detecting the abuse is less.
*Initial investigations include FBC, clotting screen, skeletal survey (X-ray series to detect any other injuries),
brain imaging and retinal exam if there is head injury, and sexual health test. Next, check child protection
register.
*Other options:
-Checking child protection register is not done until confirmation or suspicion is made (not initial step).
-Serum calcium has no benefit here (serum sodium is sometimes checked if Salt poisoning is suspected)
-DEXA scan has no role here since it is used for diagnosis and follow up of osteoporosis.

432. A 35yo woman has had bruising and petechiae for a week. She has also had recent menorrhagia but is
otherwise well. Blood: Hgb=11.1, WBC=6.3, Plt=14. What is the single most likely dx?
a. Acute leukemia
b. Aplastic anemia
c. HIV infection
d. ITP
e. SLE

KEY- D
*Patient only presents with petechiae and menorrhagia, but is othwerwise well. Hence all other options are
unlikely. Also aplastic anaemia will result in pancytopenia, but WBCs and Hb is normal here.
*What is ITP? Immune thrombocytopenic purpura.
-Autoimmune, destruction or decreased reduction of platelets. Hence decreased platelets.
-Classified into primary (isolated) or secondary (in association with other disease).
>Secondary ITP causes:
_Autoimmune disorders (Antiphospholipid AB syndrome, SLE)
_Viral ( CMV, VZ, HepC, HIV)<-- Most common in children at around 6 years.
_H.pylori
_Drugs
>Presentation:-
Petichae, epistaxis, hematuria or menorrhagia. Rarely intracranial bleeds.
>Investigations:
FBC, peripheral blood smear. Screen for HIV, HepC and other underlying cause.
>Treatment:
-Only if symptomatic.
-Avoid NSAIDs and aspirin.
-First line tt is Prednisolone for 3 weeks, then taper off, IVIG and give IV anti-D in Rh +ve and non-
splenectomised people.
-Second line Splenectomy. Complications- infection, bleeding, thrombosis, relapse.
-Refractory ITP- Romiplostim and Eltrombopag (thrombopoetin receptor agonists)

433. A 30yo man complains of episodes of hearing music and sometimes threatening voices within a couple
of hours of heavy drinking. What is the most likely dx?
a. Delirium tremens
b. Wernickes encephalopathy
c. Korsakoffs psychosis
d. Alcohol hallucinosis
e. Temporal lobe dysfunction

KEY- D
*Alcohol withdrawal presents in the following stages:
-Minor withdrawal symptoms- [Appear 6-12 hours after alcohol has stopped.] Insomnia, tremors, mild anxiety,
mild agitation or restlessness, nausea, vomiting, headache, excessive sweating, palpitations, anorexia,
depression and craving.

-Alcohol hallucinosis- Visual, auditory or tactile hallucinations that can occur either during acute intoxication
or withdrawal. During withdrawal, they [occur 12-24 hours after alcohol has stopped.]

-Withdrawal seizures are generalized tonic-clonic seizures that [appear 24-48 hours after alcohol has stopped.]

-Delirium tremens appears [48-72 hours after alcohol has stopped]. Altered mental status in the form of
confusion, delusions, severe agitation and hallucinations. Seizures can occur. Examination might reveal
stigmata of chronic alcoholic liver disease.
>Investigation: FBC, LFTs, clotting, ABG to look for metabolic acidosis, Glucose, blood alcohol levels, U&E,
creatinine, amylase, CPK and blood culture. CXR to check for aspiration pneumonia. CT scan if seizures or
evidence of head trauma. ECG-arrhythmia.
>Management of alcohol withdrawal-
-ABC
-Treat hypoglycemia
-Sedation: Benzodiazepine (chlordiazepoxide). Alternative- diazepam.
-Carbamezapine or Mg if history of withdrawal seizures.
-IV Thiamine to prevent or treat Wernickes encephalopathy that might lead to korsakoff syndrome.

*Wernickes encephalopathy- Triad of ataxia, ophthalmoplegia and mental confusion). If left untreated, leads
to Korsakoffs syndrome (Wernickes plus confabulation, antero or retrograde amnesia and telescoping of
events)
>Investigations: FBC (^MCV), LFTs, Glucose, U&E (^Na, ^Ca, ^Uricaemia), ABG (^Carbia and Hypoxia), Serum
thiamine (low).

434. A pt had TIA which he recovered from. He has a hx of stroke and exam shows HR in sinus rhythm. He is
already on aspirin 75mg and anti-HTN drugs. What other action should be taken?
a. Add clopidogrel only
b. Increase dose of aspirin to 300mg
c. Add warfarin
d. Add clopidogrel and statin
e. Add statin only
KEY- D
TIA Prophylaxis: Aspirin, clopidogrel and statin.
TIA ttt: Aspirin and dypiridamole.
435. A 40yo woman suddenly collapsed and died. At the post-mortem autopsy, it was found that there a
bleed from a berry aneurysm from the circle of Willis. In which space did the bleeding occur?
a. Subarachnoid
b. Subdural
c. Extradural
d. Subparietal
e. Brain ventricles
KEY- A.
Berry (or saccular) aneurysms are found in the circle of willis which is found in the subarachnoid space. They
are the most common form of cerebral aneurysms. They present with sudden severe headache and gold
standard for diagnosis is CT. Gold standard for treatment is surgical clipping, done after restoration of
respiration and reduction of ICP. Berry aneurysms are often associated with APCKD.
436. A schizophrenic pt hears people only when he is about to fall asleep. What is the most likely dx?
a. Hypnopompic hallucinations
b. Hyponogogic hallucinations
c. Hippocampal hallucinations
d. Delirious hallucinations
e. Auditory hallucinations
KEY- B
Hypnopompic hallucinations- While waking up.
Hyponogogic hallucinations- While falling asleep.
Hippocampal hallucination- Photographic, animated or film-like clarity of people, animals, faces, flowers,
insects etc.
Auditory hallucinations- hearing voices that arent present.

437. A pt who came from India presents with cough, fever and enlarged cervical LN. Exam: caseating
granulomata found in LN. What is the most appropriate dx?
a. Lymphoma
b. TB adenitis
c. Thyroid carcinoma
d. Goiter
e. Thyroid cyst
KEY- B
Points in favour- Traveling to India, cough, LN and caseating granulomata, which is unique for TB.

438. A 44yo man comes with hx of early morning headaches and vomiting. CT brain shows ring enhancing
lesions. What is the single most appropriate option?
a. CMV
b. Streptococcus
c. Toxoplasmosis
d. NHL
e. Pneumocystis jerovii
KEY- C
*Causes of ring enhancing lesions on CT brain:
-Brain abscess
-Primary or secondary tumour
-CNS lymphoma
-CNS toxoplasmosis
-Nocardia infection.
>Out of the options, toxoplasmosis is the right answer, and it is commonly found in HIV patients. TREATMENT
with pyrimethamine/sulfadiazine and folinic acid. OR clindamycin if intolerant FOR 4-6 WEEKS.
If immunocompromised, PROPHYLAXIS with Trimethoprim+sulfamethoxazole.

439. A 72yo man is found to be not breathing in the CCU with the following rhythm. What is the most likely
dx?
a. SVT
b. VT
c. VF
d. Atrial fib
e. Atrial flutter
KEY- C
VFib- Chaotic depolarisation of ventricles. Atrial rate 60-100. Ventricular rate 400-600. Irregular. Ttt by
immediate defibrillation
VTach- Sequence of 3 or more ventricular beats. Atrial rate 60-100. Ventricular rate 110-250. Regular. Can
progress to VFib and cardiac arrest. Ttt if pulse present, cardioversion. If pulseless, defibrillation.

440. A 65yo man with difficulty in swallowing presents with an aspiration pneumonia. He has a bovine
cough and fasciculating tongue. Sometimes as he swallows food it comes back through his nose. Choose the
single most likely cause of dysphagia from the given option?
a. Bulbar palsy
b. Esophageal carcinoma
c. Pharyngeal pouch
d. Pseudobulbar palsy
e. Systemic sclerosis
KEY- A
Bulbar palsy - Relates to medulla. Affection of lower cranial nerves (VII-XII). Dysphagia, dysphonia, dysarthria,
tremulous lips, FASICULATIONS.
Pseudobulbar palsy - Affection of corticobulbar tracts. Dysphagia, dysphonia. Donald duck speech, unable to
protrude tongue. NO FASICULATIONS

441. A 16yo teenager was brought to the ED after being stabbed on the upper right side of his back. Erect
CXR revealed homogenous opacity on the lower right lung, trachea was centrally placed. What is the most
probable explanation for the XR findings?
a. Pneumothorax
b. Hemothorax
c. Pneumonia
d. Tension pneumothorax
e. Empyema
KEY- B
Sharp stabbing wound- Hemothorax. Clincher- homogenous opacity; not seen with pneumothorax. Also since
trachea is not displaced from the centre, it is simple, not tension hemothorax.
Treatment- Chest drain insertion in the 5th intercostal space, mid-axillary line.
For tension hemo/pneumothorax, needle thoracostomy insertion in the 2nd intercostal space, mid-clavicular
line.

442. A 55yo woman complains of retrosternal chest pain and dysphagia which is intermittent and
unpredictable. The food suddenly sticks in the middle of the chest, but she can clear it with a drink of water
and then finish the meal without any further problem. A barium meal shows a corkscrew esophagus. What is
the single most likely dysphagia?
a. Esophageal candidiasis
b. Esophageal carcinoma
c. Esophageal spasm
d. Pharyngeal pouch
e. Plummer-vinson syndrome
KEY- C.
**Esophageal spasm- Oesophageal motility disorder. Dysphagia, regurgitation and chest pain. corkscrew
oesophagus on Barium swallow X-ray. Ttt- Nitroglycerin, CCB, PPI. Botulinum toxin, balloon dilatation.

**Plummer vinson syndrome- triad of iron deficiency, esophegeal webs and dysphagia. Premalignant -
squamous cell carcinoma of oesophegus. Also presents with cheilitis, koilonychia, glossitis and splenomegaly.
Patient complains of burning sensation in tongue and oral mucosa. Ttt is iron supplementation and endoscopic
dilation for webs
**Oesophageal candidiasis- Immunocompromised like HIV or renal transplant. Odynophagia, with oral thrush.
maybe weight loss. Ttt fluconazole for atleast 21 days or atleast 14 days after disappearance of symptoms.
**Oesophageal carcinoma- Dysphagia to colod foods then later to liquids. Weight loss, hoarseness of voice(if
involving the recurrent laryngeal nerve), hematemesis, hemoptysis, nausea and vomiting.
Risk factors- smoking and unhealthy diet.
Diagnosis- Endoscopy and biopsy.
Treatment- Surgery, radio and chemotherapy depending on stage.
**Pharyngeal pouch (Zenckers diverticulum)- Common above 70. M:F is 5:1.
Presentation: Dysphagia, regurgitation, aspiration, chronic cough and weight loss.Neck lump that gurgles on
palpation. Halitosis from food decaying in the pouch. Investigation: Barium swallow shows residual contrast
pool within the pouch. Aspiration from the pouch might cause inhalation pneumonia. Ttt cricopharyngeal
myotomy.

443. A 38yo female presents with sudden loss of vision but fundoscopy is normal. She a similar episode
about 1 y ago which resolved completely within 3m. Exam: mild weakness of right upper limb and exaggerated
reflexes. What is the single most appropriate tx?
a. Pan retinal photo coagulation
b. Pilocarpine eye drops
c. Corticosteroids
d. Peripheral iridectomy
e. Surgical extraction of lens

KEY- C
> This is a case of optic neuritis caused by Multiple sclerosis. Steroids are the answer here. They are given
during acute symptomatic attacks of MS. During relapse or remission, disease modifying agents like
interferons are given.
> Pan retinal photocoagulation is done for diabetic retinopathy where parts on the retina are burned in order
to reduce the Oxygen demand.
Lens extraction is done mainly for cataract to remove the opacified lens that disturbs the vision
> Peripheral iridectomy is done by making a hole in the iris for open angle glaucoma in order to provide an
alternative drainage for the fluid accumulating inside the eye, thus decreasing the IOP.
> Pilocarpine is a parasympathomimetic given for open angle glaucoma in order to contract the ciliary muscles
and to open the trabecular meshwork, allowing increased outflow of the aqueous humour
>Surgical extraction of the lens is done for cataract where the opacified lens that disturbs the vision is
removed

444. A 15yo boy presents with a limp and pain in the knee. Exam: leg is externally rotated and 2cm shorter.
There is limitation of flexion, abduction and medial rotation. As the hip is flexed external rotation is increased.
Choose the most likely dx?
a. Juvenile rheumatoid arthritis
b. Osgood-schlatter disease
c. Reactive arthritis
d. Slipped femoral epiphysis
e. Transient synovitis of the hip
KEY- D
> Slipped femoral epiphysis- Fracture through the growth plate (physis), which results in slippage of the
overlying end of the femur (epiphysis). Symptoms include gradual, progressive onset of thigh or knee pain
with a painful limp. Hip motion will be limited, particularly internal rotation.
> Osgood Schlatter disease- Inflammation of the patellar ligament at the tibial tuberosity. Painful lump just
below the knee, often seen in young adolescents. Risk factors- overuse (especially in sports involving running,
jumping and quick changes of direction) & adolescent growth spurts.
> Reactive arthritis or Reiter's syndrome- Autoimmune reaction to an infection somewhere else in the body.
Triad- arthritis, uveitis, urethritis\cervicitis

445. A 64yo woman has difficulty moving her right shoulder on recovering from surgery of the posterior
triangle of her neck. What is the single most appropriate option?
a. Accessory nerve
b. Glossopharyngeal nerve
c. Hypoglossal nerve
d. Vagus nerve
e. Vestibule-cochlear nerve
KEY- A
446. A 37yo man with an ulcer on the medial malleolus. Which of the following LN is involved?

a. External iliac LN
b. Pre-aortic LN
c. Aortic LN
d. Inguinal LN
e. Iliac LN
f. Submental LN
g. Submandibular LN
h. Deep cervical LN
447. A pt presents with weight loss of 5kgs despite good appetite. He also complains of palpitations,
sweating and diarrhea. He has a lump in front of his neck which moves on swallowing. What is the most
appropriate dx?
a. Lymphoma
b. TB adenitis
c. Thyroid Ca
d. Goiter
e. Thyroid cyst
KEY- D
Typical symptoms of hyperthyroidism- Weight loss, palpitations, sweating, diarrhoea. Goiter lump moves with
swallowing.
Thyroglossal cyst moves upwards on tongue protrusion
Thyroid cancer usually presents as a painless, hard and FIXED thyroid mass enlarging rapidly over a period of a
few weeks.

448. A 76yo woman has become tired and confused following an influenza like illness. She is also breathless
with signs of consolidation of the left lung base. What is the most likely dx?
a. Drug toxicity
b. Delirium tremens
c. Infection toxicity
d. Hypoglycemia
e. Electrolyte imbalance

KEY- C
Infection toxicity is also called Toxic shock syndrome. It is the case here because of the history of preceding
flu-like illness which points towards toxins (enterotoxin type B) from Staphylococcus aureus]. There is also
consolidation of the lung which is most probably due to the Staph pneumonia.
Delirium tremens is due to alcohol withdrawal and it usually occurs at around day 3 of cessation of alcohol
intake.
No other choice fits this scenario.

449. A young pt is complaining of vertigo whenever she moves sideways on the bed while lying supine.
What would be the most appropriate next step?
a. Head roll test
b. Reassure
c. Advice on posture
d. Carotid Doppler
e. CT
KEY- A
Dx? Benign Paroxysmal Positional Vertigo (BPPV)- Most common cause of vertigo. Vertigo triggered by change
in head position. Might be accompanied by nausea and nystagmus. Less commonly, vomiting and syncope.
Diagnosis: Dix-Hallpike and Head roll test.
Management: Epley and Semont Maneuver.

450. A 32yo man has OCD. What is the best tx?


a. CBT
b. SSRI
c. TCA
d. MAO inhibitors
e. Reassure
KEY- A
OCD is treated initially with individual CBT (Cognitive Behavioural therapy) plus exposure and response
prevention. If symptoms become severe or do not improve, SSRIs like fluoxetine or Citalopram etc are
introduced. Recent studies have shown that there is no superiority of one over the other (CBT over SSRIs), but
CBT remains the initial management plan, This question is quite deficient, and the original key is B. SSRI, but
Im sure in the exam, it will be more detailed; but this is how OCD is managed. Reference: Patient.co.uk. Link-
http://patient.info/doctor/obsessive-compulsive-disorder-pro
451. A 65yo woman says she died 3m ago and is very distressed that nobody has buried her. When she is
outdoors, she hears people say that she is evil and needs to be punished. What is the most likely explanation
for her symptoms?
a. Schizophrenia
b. Mania
c. Psychotic depression
d. Hysteria
e. Toxic confusional state
KEY- C
Psychotic depression consists of a major depressive episode plus psychotic symptoms like hallucinations or
delusions (in this case nihilistic delusions).
Toxic confusional state can be eliminated since there is no history of infection.

452. A 50yo woman presents following a fall. She reports pain and weakness in her hands for several
months , stiff legs, swallowing difficulties, and has bilateral wasting of the small muscles of her hands. Reflexes
in the upper limbs are absent. Tongue fasciculations are present and both legs show increased tone, pyramidal
weakness and hyper-reflexia with extensor plantars. Pain and temp sensation are impaired in the upper limbs.
What is the most likely dx?
a. MS
b. MND
c. Syringobulbia
d. Syringomyelia
e. Myasthenia gravis
KEY- C
In MS, there are characteristic relapse and remission which is absent here.
MND is purely motor, there is no sensory deficit; In myasthenia gravis there is muscular weakness without
atrophy.
Syringomyelia is a condition in which there is fluid-filled tubular cyst (syrinx) within the central, usually
cervical, spinal cord. The syrinx can elongate, enlarge and expand into the grey and white matter and, as it
does so, it compresses the nervous tissue of the corticospinal and spinothalamic tracts and the anterior horn
cells. This leads to various neurological symptoms and signs, including pain, paralysis, stiffness and weakness
in the back, shoulders and extremities. It may also cause loss of extreme temperature sensation, particularly in
the hands, and a cape-like loss of pain and temperature sensation along the back and arms.
** If the syrinx extends into the brainstem, syringobulbia results. This may affect one or more cranial nerves,
resulting in facial palsies. Sensory and motor nerve pathways may be affected by interruption and/or
compression of nerves.

453. Which of the following formulas is used for calculating fluids for burn pts?
a. 4 x weight(lbs) x area of burn = ml of fluids
b. 4 x weight(kgs) x area of burn = L of fluids
c. 4 x weight(kgs) x area of burn = ml of fluids
d. 4 x weight(lbs) x area of burn = L of fluids
e. 4.5 x weight(kgs) x area of burn = dL of fluids
KEY- C
>Burns are injuries caused by thermal, chemical, electrical or radiation energy.
Start with ABCs. Establish the time of the injury- from the time the injury happened, not from the time the
patient presents. Give strong analgesia. Rule out Non accidental injury. Avoid hypothermia.
>Fluid Requirements = Body area burned(%) x Wt (kg) x 4mL. This is called Parkland formula. Give 1/2 of total
requirements in 1st 8 hours, then give 2nd half over next 16 hours. Area of body burn is calculated by addition
of percentage of burn in each area, by rule of 9s:
9% head and neck, 9% each upper limb, 18% each lower limb, 18% front of trunk, 18% back of trunk, 1%
Palmar surface of the hand, including fingers, 1% Perineum
454. A 65yo male presents with dyspnea and palpitations. Exam: pulse=170bpm, BP=120/80mmHg. Carotid
massage has been done as first instance. What is the next step of the management?
a. Adenosine
b. Amilodipine
c. DC cardioversion
d. Lidocaine
e. Beta blocker
KEY- A
Likely diagnosis SVT. Initially, vagal manoeuvres, if fails iv adenosine.
Vagal manoeuvres (carotid sinus massage, Valsalva manoeuvre) transiently increase AV block, and may
unmask an underlying atrial rhythm.
If unsuccessful, give adenosine, which causes transient AV block.

455. A 48yo farmer presented with fever, malaise, cough and SOB. Exam: tachypnea, coarse end-inspiratory
crackles and wheeze throughout, cyanosis. Also complaint severe weight loss. His CXR shows fluffy nodular
shadowing and there is PMN leukocytosis. What is the single most appropriate dx?
a. Ankylosing spondylitis
b. Churg-strauss syndrome
c. Cryptogenic organizing
d. Extrinsic allergic alveolitis
e. Progressive massive fibrosis
KEY- D
Dx- Farmers lung/ Hypersensitivity penumonitis/ Extrinsic allergic penumonitis.
It is diffuse granulomatous inflammation of the lung in patients who are allergic to organic antigens present in
dust particles. On chest X-ray, diffuse nodular opacities are seen.

456. A 35yo lady is admitted with pyrexia, weight loss, diarrhea and her skin is lemon yellow in color. CBC =
high MCV. What is the most probably dx?
a. Aplastic anemia
b. Pernicious anemia
c. Leukemia
d. ITP
e. Lymphoma
KEY- B
Clincher- High MCV. It may be graves with pernicious anemia. Lemon yellow pallor occurs in pernicious
anemia. Hyperthyroidism may cause persistently raised body temperature

457. A 72yo woman who had a repair of strangulated femoral hernia 2 days ago becomes noisy, aggressive
and confused. She is febrile, CBC normal apart from raised MCV. What is the most likely dx?
a. Electrolyte imbalance
b. Delirium tremens
c. Wernickes encephalopathy
d. Infection toxicity
e. Hypoglycemia
KEY- B
Delirium tremens occurs after alcohol withdrawal, usually 3 to 4 days after cessation of alcohol. Altered
mental status in the form of confusion, delusions, severe agitation and hallucinations. Seizures can occur.
Examination might reveal stigmata of chronic alcoholic liver disease. Alcohol also typically raises MCV.
Wernickes encephalopathy- Triad of ataxia, ophthalmoplegia and mental confusion). If left untreated, leads to
Korsakoffs syndrome (Wernickes plus confabulation, antero or retrograde amnesia and telescoping of
events)
Electrolyte imbalance may cause confusion but not aggressiveness.
Infection toxicity will cause high fever, low BP, rash etc which is absent here.
Hypoglycemia can occur with alcohol intake but it does not present this way. It presents with sweating, pallor,
shakiness etc.

458. An old lady had UTI and was treated with antibiotics. She then developed diarrhea. What is the single
most likely tx?
a. Co-amoxiclav
b. Piperacillin + tazobactam
c. Ceftriaxone
d. Vancomycin
KEY- D
This is a case of pseudomembraneous colitis. It is caused by Clostridium difficile. It occurs after use of
antibiotics. Treated with Vancomycin or Metronidazole.

459. A 56yo man has symptoms of sleep apnea and daytime headaches and somnolence. Spirometry shows
a decreased tidal volume and vital capacity. What is the single most appropriate dx?

a. Ankylosing spondylitis
b. Churg-strauss syndrome
c. Good pasture syndrome
d. Motor neuron disease
e. Progressive massive fibrosis
f. Spinal cord compression
KEY- D
Involvement of respiratory muscles in Motor Neuron Disease is associated with poor respiration causing sleep
apnoea.

460. A 55yo man presents with mild headache. He has changed his spectacles thrice in 1 yr. there is mild
cupping present in the disc and sickle shaped scotoma present in both eyes. What is the single most
appropriate tx?
a. Pan retinal photo coagulation
b. Pilocarpine eye drops
c. Corticosteroids
d. Scleral buckling
e. Analgesics alone
KEY- B
> Sickle-shaped scotoma or siedel sign is often seen in glaucoma. That along with the fact that he keeps
changing his spectacles denotes that this is a case of progressive open angle glaucoma. It can also present with
nausea, vomiting, headache and ocular pain. Treated with Carbonic anhydrase inhibitors like acetazolamide,
Miotic agents (parasympathomimetics) such as pilocarpine, Alpha2-adrenergic agonists like brimonidine, or
Prostaglandin analogs like latanoprost.
> Pan retinal photocoagulation is done for diabetic retinopathy where parts on the retina are burned in order
to reduce the Oxygen demand.
> Scleral buckling is done for retinal detachment to put the retina back in place.
461. A 55yo woman was found collapsed at home, paramedics revived her but in the ambulance she had a
cardiac arrest and couldnt be saved. The paramedics report tells that the woman was immobile lately due to
hip pain and that they found ulcers on the medial side of ankle. She had DM and was on anti-diabetics. What is
the cause of her death?
a. Acute MI
b. DKA
c. Pulmonary embolism
d. Acute pericarditis
e. Cardiac tamponade
KEY- C
This is a case of collapse due to PE following DVT caused by the patients immobilization due to hip pain.
Cardiac tamponade- Triad of hypotension, distended engorged neck veins, and muffled JVP.
Pericarditis- Chest pain worse with inspiration and lying down, relieved by lying forward.
No history supporting DKA or MI.

462. An 18yo previously well student is in his 1st year at uni. He has been brought to the ED in an agitated,
deluded and disoriented state. What is the most probable reason for his condition?
a. Drug toxicity
b. Delirium tremens
c. Infection toxicity
d. Electrolyte imbalance
e. Head injury
KEY- A
Clinchers are teenage, and 1st year of university, where students tend to experiment with drugs.
Infection toxicity can be ruled out due to lack of any signs of infection like fever. Lack of history of trauma rules
out head injury, and delirium tremens is due to alcohol withdrawal.

463. A young adult presents to the ED after a motorcycle crash. The pt has bruises around the left orbital
area. GCS=13, examination notes alcoholic breath. Shortly afterwards, his GCS drops to 7. What is the single
most important initial assessment test?
a. MRI brain
b. CT brain
c. CXR
d. CT angio brain
e. Head XR
KEY- B
This is a typical case of Epidural hematoma. It is usually due to trauma, and has a period of lucidity before
collapse. Due to the sudden drop in GCS, CT brain should be immediately done.

464. A 30yo female attends OPD with a fever and dry cough. She says that she had headache, myalgia and
joint pain like one week ago. Exam: pulse=100bpm, temp=37.5C. CXR: bilateral patchy consolidation. What is
the single most likely causative organism?
a. Pneumococcal pneumonia
b. Legionella
c. Mycoplasma
d. Klebsiella
e. Chlamydia pneumonia
KEY- C
> Mycoplasma pneumonia- Atypical pneumonia. Slow onset, dry cough, pleuritic pain, myalgia, arthralgia,
malaise.
> Legionella- history of travel and stay in hotel- atypical symptoms plus GI manifestations.
> Klebsiella- commonly associated with alcohol.

465. A 46yo man is being investigated for indigestion. Jejunal biopsy shows deposition of macrophages
containing PAS (Periodic acid-schiff) +ve granules. What is the most likely dx?
a. Bacterial overgrowth
b. Celiac disease
c. Tropical sprue
d. Whipples disease
e. Small bowel lymphoma
KEY- D
Periodic acid-schiff positive granules containing macrophages in jejunal biopsy is diagnostic of whipples
disease.
Coeliac disease is gluten sensitivity.
466. A 32yo woman of 38wks gestation complains of feeling unwell with fever, rigors and abdominal pains.
The pain was initially located in the abdomen and was a/w urinary freq and dysuria. The pain has now become
more generalized specifically radiating to the right loin. She says that she has felt occasional uterine
tightening. CTG is reassuring. Select the most likely dx?
a. Acute fatty liver of pregnancy
b. Acute pyelonephritis
c. Round ligament stretching
d. Cholecystitis
e. UTI
KEY- B
This is a case of UTI followed by ascending infection leading to pyelonephritis. Fever, rigors and abdominal
pain are typical symptoms.
467. A 32yo pt presents with cervical lymphadenopathy and splenomegaly. What is the single most
appropriate option?
a. Hemophilus
b. Streptococcus
c. Toxoplasmosis
d. NHL
e. Pneumocystis jerovcii
KEY- D
Non Hodgkins lymphoma is the only option here that will have both lymphadenopathy and splenomegaly
(although splenomegaly is not a common presentation). Pneumocystis jerovici and Toxoplasmosis are
common in HIV patients.

468. A 62yo man who was admitted for surgery 3days ago suddenly becomes confused. His attn span is
reduced. He is restless and physically aggressive and picks at his bed sheets. What single aspect of the pts hx
recovered in his notes is most likely to aid in making the dx?
a. Alcohol consumption
b. Head trauma
c. Hx of anxiety
d. Prescribed med
e. Obvious cognitive impairment
KEY- A
This is a typical case of Delirium tremens. It appears [48-72 hours after alcohol has stopped]. Altered mental
status in the form of confusion, delusions, severe agitation and hallucinations. We should ask alcohol history.
Examination might reveal stigmata of chronic alcoholic liver disease.

469. A 10yo girl presents with pallor and features of renal failure. She has hematuria as well as proteinuria.
The serum urea and creat are elevated. These symptoms started after an episode of bloody diarrhea 4 days
ago. What is the most probable dx?
a. TTP
b. HUS
c. ITP
d. HSP
e. ARF

KEY- B
Haemolytic Uraemic Syndrome (HUS) is a triad of Haemolytic anaemia, thrombocytopaenia and Renal failure.
It is said to be caused most commonly by E.coli O:157H7 which binds to endothelial receptors in the GIT, Renal
and central nervous system. Symptoms [ abdominal pain, pallor due to anaemia, hematuria and proteinuria,
features of renal failure like- nausea/vomiting, swelling of face, hand, feet or entire body etc. and elevated
urea and creatinine etc.] start around two weeks after an episode of bloody diarrhea. The diarrheoa is
charactised to get bloody after 1-3 days. This scenario is typical for HUS. It is also known to be precipitated by
strept pneumonia and some drugs like cyclosporin and tacrolimus.

470. A 40yo woman has had intermittent tension, dizziness and anxiety for 4 months. Each episode usually
resolves after a few hours. She said she takes alcohol to make her calm. She is in a loving relationship and has
no probs at work or home. What is the next step in her management?
a. Collateral info
b. CT brain
c. CBC
d. LFT
e. TFT
KEY- A Collateral info. Likely diagnosis is panic disorder. Collateral info from family, friends & other peers
should be asked to find out the cause for her anxiety.

471. A 45yo IV drug abuser is brought into the ED with complaint of fever, shivering, malaise, SOB and
productive cough. Exam: temp=39C, pulse=110bpm, BP=100/70mmHg. Inv: CXR=bilateral cavitating
bronchopneumonia. What is the single most likely causative organism?
a. Mycoplasma
b. Staphylococcus
c. Chlamydia pneumonia
d. Pseudomonas
e. PCP
KEY- B
Staphylococcus and PCP are common in IV drug abusers. Both are also recognized cause of cavitating
pneumonia. This case is with productive cough which goes more with staphylococcus as PCP is not productive,
but is rather associated with dry cough.
Mycoplasma pneumonia- Atypical pneumonia. Slow onset, dry cough, pleuritic pain, myalgia, arthralgia,
malaise.
472. A 71yo woman looks disheveled, unkempt and sad with poor eye contact. She has recently lost her
husband. Which of the following describes her condition?
a. Anxiety
b. Hallucination
c. Mania
d. High mood
e. Low mood

KEY- E
Dx- Depression. Disheveled and unkempt because she doesnt take care of herself, plus the loss of her
husband, points towards depression.

473. A 62yo male comes to the GP complaining of double vision while climbing downstairs. Which of the
following nerve is most likely involved?
a. Abducens nerve
b. Trochlear nerve
c. Oculomotor nerve
d. Optic nerve
e. Trigeminal nerve

KEY- B
This is a lesion in the Trochlear nerve affecting the Superior oblique muscle.
All extrinsic muscles of the eye are supplied by the Oculomotor nerve except the Lateral rectus by the
Abducens nerve and the Superior oblique by the trochlear (mnemonic LAST).
Oculomotor nerve affection causes palsy of inferior rectus, medial rectus and superior rectus manifesting as
double vision in multiple gaze. But trochlear involving superior oblique only causes diplopia in downgaze only.

474. L1 level, what is the most appropriate landmark?


a. Mcburneys point
b. Stellate ganglion
c. Deep inguinal ring
d. Termination of the spinal cord
e. Transpyloric plane
KEY- E
475. A 32yo woman presents to the ED with headache and vomiting. She was decorating her ceiling that
morning when the headache began, felt mainly occipital with neck pain. Some 2hs later she felt nauseated,
vomited and was unable to walk. She also noticed that her voice had altered. She takes no reg meds and has
no significant PMH. Exam: acuity, field and fundi are normal. She has upbeat nystagmus in all directions of
gaze with normal facial muscles and tongue movements. Her uvulas deviated to the right and her speech is
slurred. Limb exam: left arm past-pointing and dysdiadochokinesia with reduced pin prick sensation in her
right arm and leg. Although power is normal, she cant walk as she feels too unsteady. Where is the most likely
site of lesion?
a. Right medial medulla
b. Left medial pons
c. Left cerebellar hemisphere
d. Right lateral medulla
e. Left lateral medulla
KEY- E
Lateral medullary syndrome affects:
-Contralateral spinothalamic tract (loss of pain and temperature on the opposite side of the body)
-Ipsilateral Sympathetic tract- Horners syndrome.
-Ipsilateral Spinal trigeminal nucleus (loss of pain,temperature and corneal reflex on same side of the face)
-Nucleus ambigous- Dysphagia and Dysarthria
-Inferior cerebellar peduncle- Ataxia
Ipsilateral Cranial nerves- IX, X and XI (dysphagia, loss of gag reflex, palate paralysis)
Cause- Occlusion of PICA (posterior inferior cerebellar artery)
Medial medullary syndrome affects:
Contralateral corticospinal tract/pyramids- weakness of arms and legs opposite side.
Contralateral Medial lemniscus/dorsal column- loss of proprioception and vibration.
Ipsilateral hypoglossal nerve- weakness of tongue on the same side.
[Hypoglossal nerve affection manifests as protrusion of the tongue to the side of the weakness while at rest, it
deviates to the contralateral side)
Cause- Occlusion of Anterior spinal artery.

476. A 28yo female presents with 1 wk hx of jaundice and 2d hx of altered sleep pattern and moods. She
was dx with hypothyroidism for which she is receiving thyroxine. TFT showed increased TSH. PT=70s. What is
the most probable dx?
a. Acute on chronic liver failure
b. Hyper-acute liver failure
c. Autoimmune hepatitis
d. Acute liver failure
e. Drug induced hepatitis
KEY- C
Autoimmune hepatitis may present as acute hepatitis, chronic hepatitis, or well-established cirrhosis.
Autoimmune hepatitis rarely presents as fulminant hepatic failure. One third may present as acute hepatitis
marked by fever, hepatic tenderness and jaundice. Non specific features are anorexia, weight loss and
behavioural change (here altered sleep pattern and moods). There may be coagulopathy (here PT=70s.)
leading to epistaxis, gum bleeding etc. Presence of other autoimmune disease like hypothyroidism supports
the diagnosis of autoimmune hepatitis.

477. A 55yo man has a chronic cough and sputum, night sweats and weight loss. What is the single most
likely causative organism?
a. Coagulase +ve cocci in sputum
b. Gram -ve diplococci in sputum
c. Gram +ve diplococci in sputum
d. Pneumocystis carinii in sputum
e. Sputum staining for mycobacterium tuberculosis
KEY- E
Classic features of TB- Chronic cough and sputum, night sweats and weight loss. Organism is Acid fast bacilli
mycobacterium tuberculosis.

478. A 20yo pregnant 32wks by date presents to the antenatal clinic with hx of painless vaginal bleeding
after intercourse. Exam: P/A soft and relaxed, uterus=dates, CTG=reactive. Choose the single most likely dx?
a. Abruption of placenta 2nd to pre-eclampsia
b. Antepartum hemorrhage
c. Placenta previa
d. Preterm labor
e. Placenta percreta
KEY- C
Clincher- painless bleeding, typical presentation of placenta previa. Uterus is soft and relaxed and theres no
pain, so we rule out placental abruption.

479. A 30yo man presents to the ED with difficulty breathing. He has returned from India. Exam: throat
reveals grey membranes on the tonsils and uvula. He has mild pyrexia. What is the single most relevant dx?
a. Diphtheria
b. IM
c. Acute follicular tonsillitis
d. Scarlet fever
e. Agranulocytosis
KEY- A
Clinchers- History of travel to India, and greyish membrane.
Infectious mononucleosis will typically present in a teenager, with enlarged cervical lymphadenopathy and
fever.
Acute follicular tonsillitis as the name suggests, will have follicles on the tonsils.
Scarlet fever presents with rash and strawberry tongue

480. A 23yo man comes to the ED with a hx of drug misuse. He recognizes that he has a prb and is willing to
see a psychiatrist. Which of the following terms best describes this situation?
a. Judgement
b. Thought insertion
c. Thought block
d. Mood
e. Insight
KEY- E
Insight is the patient's awareness and understanding of the origins and meaning of his attitudes, feelings, and
behavior and of his disturbing symptoms, basically, he is aware that he has a problem.

481. A pt with hodgkins lymphoma who is under tx develops high fever. His blood results show WBC <2800
and has a chest infection. Choose the most likely tx?
a. Co-amoxiclav
b. Piperacillin+tazobactam
c. Erythromycin
d. Piperacillin+Co-amoxiclav
e. Penicillin+tazobactam
KEY- B
This patient with Hodgkins lymphoma has a severe infection and his WBC count is very low, so he needs to be
covered with broad spectrum antibiotics, hence piperacillin and tazobactam.

482. A 25yo woman presents with urinary freq, dysuria and fever. Urine microscopy shows 20-50 RBC and
10-20 WBC in each field. What is the most probable dx?
a. Schistosmiasis
b. Kidney trauma
c. Ureteric calculus
d. Bladder calculi
e. Cystitis
KEY- E
Clincher- WBCs.
These are typical Symptoms of UTI- fever, frequency, dysuria. Urine microscopy here (hematuria and presence
of WBCs) indicate cystitis.
Schistosomiasis can present with hematuria and fever but also with additional symptoms like diarrhea,
abdominal pain, hepatosplenomegaly, cough and history of travel would likely be given.
Kidney trauma can present with hematuria, but not dysuria, frequency and fever.
Calculi can present with severe lower abdominal and back pain, difficult urination, frequency, fever, dysuria
and haematuria. The pain, that comes in waves, may also be associated with nausea, vomiting and chills.
WBCs though, will not be present.

483. A 65yo presents with dyspareunia after sex. She in menopause. She complains of bleeding after sex.
What is the most probably dx?
a. Cervical ca
b. Endometrial ca
c. Ovarian ca
d. Breast ca
e. Vaginal ca
KEY- B
RULE- Postmenopausal bleeding, be it post coital or not, is Endometrial cancer unless proven otherwise.
Clinchers- Post menopausal, and age 65.
Cervical cancer is common in women aged 25-34 years, while 90% of women with endometrial cancer are over
50 years of age. [Source- patient.co.uk]

484. A 45yo man underwent an emergency splenectomy following a fall from his bicycle. He smokes 5
cigarettes/day. Post-op, despite mobile, he develops swinging pyrexia and a swollen painful left calf. His CXR
shows lung atelectasis and abdominal US demonstrates a small subphrenic collection. What is the single most
likely risk factor for DVT in this pt?
a. Immobility
b. Intraperitoneal hemorrhage
c. Smoking
d. Splenectomy
e. Sub-phrenic collection
KEY- D
Since it is stated that patient is mobile. option A can be eliminated. Option B and E are not known to
predispose to Thromboembolism. This leaves C and D, and splenectomy is a stronger link to DVT due to it
being:
1. A recent major surgery(within 12 weeks), which is a big risk factor for DVT.
2. The surgery itself- Vascular events after splenectomy are likely multifactorial, probably resulting from
some combination of hypercoagulability, platelet activation, disturbance and activation of the
endothelium, and altered lipid profiles. The spleen's primary phagocytic function is to remove
infectious organisms, other insoluble cellular debris, and senescent or abnormal red cells and platelets.
This filtration function results from the blood moving slowly through the splenic sinusoids in the red
pulp lined with macrophages actively ingesting that which does not easily pass around them. Absence
of this extremely sensitive filter may permit particulate matter and damaged cells to persist in the
bloodstream, therefore perturbing and activating the vascular endothelium leading to a shift in
vascular homeostasis toward enhanced coagulation.

485. A 6m baby had LOC after which he had jerky movement of hands and feet. What is the most probable
dx?
a. Infantile spasm
b. Absence
c. Partial simple seizure
d. Atonic seizure
e. Partial complex
KEY- E
> Generalised- Entire body is involved.
> Focal/Partial- Not the entire body is involved.
> Complex- Loss of consciousness.
> Simple- No loss of consciousness.
> Absence seizures- The person has a brief loss of consciousness (an absence) for a few seconds. They do not
fall but may pause in what they are doing. Their face often looks pale with a blank expression. They may look
dazed, the eyes stare and the eyelids may flutter a little. Sometimes their head may fall down a little, or their
arms may shake once or twice. Each seizure usually starts and finishes abruptly. The person is not aware of the
absence and resumes what they were doing.
> Infantile spasm (West Syndrome) occurs in the first year of life (3-8 months) and is not associated with LOC.
Its nature is more generalized rather than the focal nature described here. Infantile spasm can also be called
salaam spasms, because the appearance of the seizures is like a bowing forwards or backwards movement.

486. A 24yo primigravida who is 30wk pregnant presents to the labor ward with a hx of constant abdominal
pain for the last few hours. She also gives a hx of having lost a cupful of fresh blood per vagina before the pain
started. Abdominal exam: irritable uterus, CTG=reactive. Choose the single most likely dx?
a. Abruption of placenta 2nd to pre-eclampsia
b. Antepartum hemorrhage
c. Placenta previa
d. Vasa previa
e. Revealed hemorrhage
KEY- B
Presentation indicates abruption of the placenta, but not confirmed yet. Generally bleeding during this time is
given a general diagnosis of antepartum haemorrhage. There is no history or features suggestive of of
hypertension or pre-eclampsia so A is not the choice. Abruption can be either concealed or revealed
abruption.
Placenta praevia is painless bleeding.
487. A 62yo lady presents with right sided headache and loss of vision. What is the single most inv?
a. ESR
b. BUE
c. CT head
d. XR orbit
e. IOP
KEY- A
> This is most probably Giant cell arteritis/Temporal arteritis. It is common in females and elderly people and
should always be considered in cases of new-onset headache in patients 50 years of age or older. Initial
investigation is ESR which will be raised (>40mm/hr), and confirmatory diagnosis is temporal artery biopsy.
Patient should be started on steroids immediately if GCA is suspected, even if diagnosis is not confirmed, as
delay in treatment might lead to blindness due to occlusion of the ophthalmic artery.
> IOP (Intraocular pressure) is used to investigate glaucoma.

488. A 24yo man asks his GP for a sick note from work. He says that feels down, is lethargic and has stopped
enjoying playing the piccolo (his main hobby). He was admitted to the psychiatry ward last year following an
episode of overspending, promiscuity and distractibility. What is the most probable dx?
a. Psychosis
b. Cyclothymia
c. Bipolar affective disorder
d. Seasonal affective disorder
KEY- C
> Features of Mania (overspending, promiscuity and distractibility) plus Depression (low mood, lethargy and
anhedonia) denote Bipolar Disorder.
> Cyclothymia is a mild form of Bipolar disorder [Mild depression without somatic symptoms + Hypomania]
that often goes unnoticed.

489. A 42yo female who is obese comes with severe upper abdominal pain with a temp=37.8C. She has 5
children. What is the most probable dx?
a. Ectopic pregnancy
b. Ovarian torsion
c. Hepatitis
d. Biliary colic
e. Cholecystitis
KEY- E
> This is cholecystitis, or non-alcoholic steatohepatitis. The 5 Fs of cholecystitis are- Fat
Female
Fare
Forty
Fertile.
> Ovarian torsion and ectopic pregnancy will have lower abdominal pain.

490. A child has just recovered from meningitis. What inv will you do before discharge?
a. CT scan
b. EEG
c. Blood culture
d. Repeat LP
e. Hearing test
KEY- E
** Patient is already recovering from meningitis, so none of options A, B, C or D are indicated.
** Since hearing loss is the most common complication of meningitis, people recovering from the condition
will usually have a hearing test. The test should be carried out before you're discharged, or within 4 weeks of
being well enough to have the test. Children and young people should discuss the results of their hearing test
with a paediatrician between 4 and 6 weeks after being discharged from hospital. In cases where hearing is
severely affected, cochlear implants may be needed. [NICE Guidelines and NHS].
> Complications of Meningitis:
*Immediate: septic shock, DIC, coma with loss of protective airway reflexes, cerebral oedema and raised ICP,
septic arthritis, pericardial effusion and haemolytic anaemia (H. influenzae).
Subdural effusions: reported in 40% of children aged 1-18 months with bacterial meningitis.
Syndrome of inappropriate antidiuretic hormone secretion (SIADH).
Seizures
*Delayed: decreased hearing, or deafness; other cranial nerve dysfunction, multiple seizures, focal paralysis,
subdural effusions, hydrocephalus, intellectual deficits, ataxia, blindness, Waterhouse-Friderichsen syndrome
and peripheral gangrene.

491. A primiparous woman with no prv infection with herpes zoster is 18wk pregnant. She had recent
contact with a young 21yo pt having widespread chicken pox. What is the most suitable management for the
pregnant lady?
a. Acyclovir PO
b. Acyclovir IV +IVIG
c. Acyclovir IV
d. Reassure
e. IVIG
KEY- E
*If the pregnant woman is not immune to VZV and she has had a significant exposure, she should be offered
varicella-zoster immunoglobulin (VZIG) as soon as possible. VZIG is effective when given up to 10 days after
contact (in the case of continuous exposures, this is defined as 10 days from the appearance of the rash in the
index case).
*If she had no previous infection and develops a rash (got infected) and comes within 24 hour of development
of rash- acyclovir is given. [MRCOG Guideline].

492. A 40yo woman presents to the GP with low mood. Of note, she has an increased appetite and has
gone up 2 dress sizes. She also complains that she cant get out of bed until the afternoon. What is the most
likely dx?
a. Pseudo depression
b. Moderate depression
c. Severe depression
d. Dysthymia
e. Atypical depression
KEY- E
Atypical depression is a subtype of major depression or dysthymic disorder that involves several specific
symptoms, including increased appetite or weight gain, hypersomnia, marked fatigue or weakness, moods
that are strongly reactive to environmental circumstances, and feeling extremely sensitive to rejection, or
feeling of being weighed down, paralyzed, or "leaden.

493. An 8yo boy is clinically obese. As a baby he was floppy and difficult to feed. He now has learning
difficulties and is constantly eating despite measures by his parents to hide food out of reach. What is the
most probable dx?
a. Cushings syndrome
b. Congenital hypothyroidism
c. Prader Willi syndrome
d. Lawrence moon biedel syndrome
e. Downs syndrome
KEY- C
Prader Willi syndrome- congenital disorder caused by defect in gene on chromosome 15. Characterized by
hypotonia at birth, feeding difficulties, poor growth and delayed development. At the beginning of childhood,
they present with obsessive eating and obesity, learning difficulties, behavioural problems and compulsive
behavior such as picking on skin. Distinctive facial features like triangular mouth, unusually fair skin and light-
coloured hair, almond shaped eyes and short forehead.

494. A 20yo lady is suffering from fever and loss of appetite. She has been dx with toxoplasmosis. What is
the tx?
a. Pyrimethamine
b. Pyrimethamine + sulfadiazine
c. Clindamycin
d. Spiramycin
e. Trimethoprim + sulfamethoxazole
KEY- B
Toxoplasmosis:
TREATMENT with pyrimethamine/sulfadiazine and folinic acid. OR clindamycin if intolerant FOR 4-6 WEEKS.
If immunocompromised, PROPHYLAXIS with Trimethoprim+sulfamethoxazole.

495. A 68yo woman has a sudden onset of pain and loss of hearing in her left ear and unsteadiness when
walking. There are small lesions visible on her palate and left external auditory meatus. What is the single
most likely dx?
a. Acute mastoiditis
b. Cholesteatoma
c. Herpes zoster infection
d. Oropharyngeal malignancy
e. OM with infusion
KEY- C
Clincher- lesions which are probably vesicles.
Herpes zoster oticus (Ramsay Hunt syndrome) occurs when latent varicella zoster virus reactivates in the
geniculate ganglion of the 7th cranial nerve.
Symptoms: Painful vesicular rash on the auditory canal on drum, pinna, tongue, palate or iris with ipsilateral
facial palsy, loss of taste, vertigo, tinnitus, deafness, dry mouth and eyes. OHCM 9th edition, page 505.

496. A 45yo woman has been dx with Giant Cell A and is being treated with steroids. What is the other drug
that can be added to this?
a. ACEi
b. Beta blockers
c. Aspirin
d. Interferons
e. IVIG
KEY- C
For GCA, along with Steroids, 3 other medications are required:
1). Low-dose aspirin: Start aspirin 75 mg daily unless there are contra-indications - eg, active peptic ulceration
or a bleeding disorder. Low-dose aspirin has been shown to decrease the rate of visual loss and strokes in
patients with GCA.
2). Start gastroprotection with a proton pump inhibitor in view of added risk of peptic ulceration with high-
dose steroids and aspirin.
3). Start Osteoporosis prophylaxis with bisphosphonates since patient in on long-term steroid treatment.

497. A 17yo man has acute pain and earache on the right side of his face. Temp=38.4C and has extensive
pre-auricular swelling on the right, tender on palpation bilaterally. What is the single most likely dx?
a. Acute mastoiditis
b. Acute otitis externa
c. Acute OM
d. Mumps
e. OM with effusion
KEY- D
> Mumps- Prodromal malaise, fever, painful parotid swelling, becoming bilateral in 70%. OHCS 9th edition,
page 142.
> Otitis externa typically presents after swimming, and involves only the external ear. Treated with topical
antibiotic drops (aminoglycosides) and acetic acid 2% ear drops.
> Otitis media will involve the tympanic membrane and the pain will be in the ear, not pre-auricular. Treated
with analgesics and antipyretics for adults, and antibiotics for children.

498. An ECG of an elderly lady who collapsed in the ED shows rapid ventricular rate of 220 bpm,
QRS=140ms. What is the most probable dx?
a. Atrial fibrillation
b. VT
c. SVT
d. Mobitz type1 2nd degree heart block
e. Sinus tachycardia
KEY- B
VTach- Sequence of 3 or more ventricular beats. Atrial rate 60-100. Ventricular rate 110-250. Wide QRS
complex (>120ms). Regular. Can progress to VFib and cardiac arrest. Ttt- if pulse present, cardioversion. If
pulseless, defibrillation.
SVT and AFib will have narrow QRS complexes.
499. A pt presents with purple papular lesions on his face and upper trunk measuring 1-2 cm across. They
arent painful or itchy. What is the single most likely dx?
a. Kaposis sarcoma
b. Hairy leukoplakia
c. Cryptosporidium
d. CMV infection
e. Cryptococcal infection
KEY- A
Kaposis sarcoma is a spindle-cell tumour derived from capillary endothelial cells or from fibrous tissue, caused
by human herpes virus. It presents as non painful purple papules ( to 1 cm) or plaques on skin and mucosa
(any organ). It is not itchy, and it metastasizes to nodes. Associated with AIDS infection. OHCM 9th edition,
page 716.
500. A 6yo boy is clinically obese, his BMI >95th centile. He has no other medical prbs, examination is
unremarkable. His mother says that she has tried everything to help him lose weight. What is the most
probable dx?
a. Cushings syndrome
b. Congenital hypothyroidism
c. Downs syndrome
d. Lawrence moon biedel syndrome
e. Primary obesity
KEY- E
Features support primary childhood obesity. No other associated signs or symptoms except obesity. Its not
cushing (No moon face, pigmentation, hyperglycaemia) etc. Its not congenital hypothyroidism, (No weight
loss despite increased appetite), not Down syndrome (No features of Down) or Lawrence moon biedel
syndromes (No learning difficulties).

501. A 20yo boy is brought by his parents suspecting that he has taken some drug. He is agitated,
irritated and cant sleep. Exam: perforated nasal septum. Which of the following is the most
likely to be responsible for his symptoms?
a. Heroine
b. Cocaine
c. Ecstasy/MDMA/amphetamine
d. Alcohol
e. Opioids

B. Cocaine
perforated nasal septum
Heroine: pinpoint pupils, dec consciousness, bradycardia, resp depression, hypoxia. antidote: naloxone
Ecstasy/MDMA/amphetamine: agitation, anxiety, confusion, ataxia, tachycardia, hypertension,
hyponatraemia, hyperthermia, rhabdomyolysis
Mechanism of action
cocaine blocks the uptake of dopamine, noradrenaline and serotonin
The use of cocaine is associated with a wide variety of adverse effects:

Cardiovascular effects
myocardial infarction
both tachycardia and bradycardia may occur
hypertension
QRS widening and QT prolongation
aortic dissection

Neurological effects
seizures
mydriasis
hypertonia
hyperreflexia

Psychiatric effects
agitation
psychosis
hallucinations

Others
hyperthermia
metabolic acidosis
rhabdomyolysis

Management of cocaine toxicity


in general benzodiazipines are generally first-line for most cocaine related problems
chest pain: benzodiazipines + glyceryl trinitrate. If myocardial infarction develops then primary
percutaneous coronary intervention
hypertension: benzodiazipines + sodium nitroprusside
the use of beta-blockers in cocaine-induced cardiovascular problems is a controversial issue. The
American Heart Association issued a statement in 2008 warning against the use of beta-blockers (due
to the risk of unopposed alpha-mediated coronary vasospasm) but many cardiologists since have
questioned whether this is valid. If a reasonable alternative is given in an exam it is probably wise to
choose it.

502. For a pt presenting with Parkinsons disease which of the following drugs is most useful in the
management of the tremor?
a. Apomorphine
b. Cabergoline
c. Selegiline
d. Amantadine
e. Benzhexol

e. Benzhexol
Antimuscarinics
block cholinergic receptors
now used more to treat drug-induced parkinsonism rather than idiopathic Parkinson's disease
help tremor and rigidity
e.g. procyclidine, benzotropine, trihexyphenidyl (benzhexol)

503. A 26yo woman has become aware of increasing right sided hearing deficiency since her recent pregnancy.
Her eardrums are normal. Her hearing tests show: BC-normal. Weber test lateralizes to the right ear. What is
the single most likely dx?
a. Encephalopathy
b. Functional hearing loss
c. Tympano-sclerosis
d. Otosclerosis
e. Sensorineural deafness

key is D. Otosclerosis. [There are no features of encephalopathy. As Weber test is lateralized it is unlikely to be
functional hearing loss. In tympanosclerosis ear drum becomes chalky white. So as the ear drum is normal it is
not tympanosclerosis. Weber test is lateralized to right and deafness is also on the right. So it not
sensorineural deafness but conductive deafness which makes otosclerosis as the most likely diagnosis.

Rinne's test
air conduction (AC) is normally better than bone conduction (BC)
if BC > AC then conductive deafness

Weber's test
in unilateral sensorineural deafness, sound is localised to the unaffected side
in unilateral conductive deafness, sound is localised to the affected side

504. A 58yo T1DM on anti-HTN therapy for 13yrs developed central chest pain for 45 mins while
driving a/w cold sweating and dyspnea. What is the single most appropriate dx?
a. MI
b. Pericarditis
c. Pulmonary embolism
d. Costochondritis
e. Pneumothorax

a. MI
characteristic central or epigastric chest pain radiating to the arms, shoulders, neck, or jaw.
The pain is described as substernal pressure, squeezing, aching, burning, or even sharp pain.
Radiation to the left arm or neck is common.
Chest pain may be associated with sweating, nausea, vomiting, dyspnoea, fatigue, and/or
palpitations.

Pericarditis chest pain: may be pleuritic. Is often relieved by sitting forwards


Pulmonary embolism pleuritic chest pain, dyspnoea and haemoptysis
Costochondritis Chest wall pain with a history of repeated minor trauma or unaccustomed activity (eg,
painting, moving furniture) is common

505. A man was brought to the ED from a shopping mall after collapsing there. He is conscious and answering
questions now. His ECG shows irregular rhythm. Your choice of inv:
a. CT
b. MRI
c. 24h ECG
d. Echo
d. Echo
The man had a syncopial attack ..the most valvular cause for it is aortic stenosis which needs an Echo to
diagnose it or if there is any other valvular lesion or ventricular dysfunction
go for echo just to exclude any structural abnormalities.
holter- ecg is already said to be irregular. we already know that there is an rhythm problem so no use to
holter.
If in history something indicated towards TIA or stroke then CT or MRI would be considered.

506. A 10yo boy is clinically obese and the shortest in his class. He had a renal transplant last year and his
mother is worried that he is being bullied. What is the most probable dx?
a. Cushings syndrome
b. Congenital hypothyroidism
c. Pseudocushings syndrome
d. Lawrence moon biedel syndrome
e. Downs syndrome

a. Cushings syndrome
he's on steroids post-renal transplant, Oral steroids is the chief cause of Cushing's syndrome (OHCM, 8th, page
124).
Laurance-moon synd. Night blindness due to retinitis pigmentosa, polydactyly are important features
(OHCS/8th/648). With no emphasis on more common features, Oral-steroid induced (post renal transplant)
Cushing makes more sense.
Congenital hypothyroidism Feeding difficulties, Somnolence, Lethargy, Low frequency of crying, Constipation

Downs syndrome he is clinically obese not conganital case,down syndrome has cardaic problem and
characteristic facial feature and mentalyy retarded so it cant be option,these features are same for lawrence
moon but ptnt are mentally retarded whereas kid is studying in normal school rather than special one
Pseudocushings syndromeit is mainly an idiopathic condition.Some frequently occurring illnesses can induce
a phenotype that largely overlaps with Cushing syndrome and is accompanied by hypercorticolism

507. A 45yo man had cancer of head of pancreas which has been removed. He has a hx of
longstanding heartburn. He now comes with rigid abdomen which is tender, temp 37.5C,
BP=90/70mmHg, pulse=120bpm. What is the next step of the inv?
a. CT abdomen
b. XR abdomen
c. MRI abdomen
d. US abdomen
e. Endoscopy

b. XR abdomen
Long standing Heart burn - peptic ulcer disease resulting into hollow viscous perforation leading to gas
under diaphragm in x-ray abdomen!
where are all those sign of acute pancreatitis in this case except hypotension and tachycardia which can occur
with perotinitis.suppose its pancreatitis ,do u think cT is nxt step after presentation? Nxt step must be serum
amylase and lipase and cT is most accurate.

Laparoscopy has become a routine procedure in the management of acute abdominal disease

508. A 50yo man presents to the ED with acute back pain radiating down to his legs. Pain which is usually
relieved by lying down and exacerbated by long walks and prolong sitting. What inv
would be the best option?
a. MRI
b. CT spine
c. XR spine
d. Dual energy XR abruptiometry
e. Serum paraprotein electrophoresis

a. MRI
Diagnosis most likely Prolapsed Intervertebral Disc. Sudden onset acute back pain radiating down the leg, and
it is relieved on lying down and exacerbated by prolonged walks and on coughing and moving the back.
Investigation done is MR Spine to look for prolapsed disc and nerve root compression.
Never think of CT in case of spinal cord compression
mri... better for visualisin soft tissue.. ct better if bony detail is desired.. this is lumbar degenerative disc
disease most likely due to a herniated nucleus pulposus at l4/5 or l5/s1

Lumbosacral disc herniation: (patient.co.uk)


If there is nerve entrapment in the lumbosacral spine, this leads to symptoms of sciatica which
include:
o Unilateral leg pain that radiates below the knee to the foot/toes.
o The leg pain being more severe than the back pain.
o Numbness, paraesthesia, weakness and/or loss of tendon reflexes, which may be
present and are found in the same distribution and only in one nerve root distribution.
o A positive straight leg raising test (there is greater leg pain and/or more nerve
compression symptoms on raising the leg).
o Pain which is usually relieved by lying down and exacerbated by long walks and
prolonged sitting.
o MRI is very sensitive in showing disc herniations

Management
Simple analgesics as first line
Pain due to a herniated lumbosacral disc may settle within six weeks. If it does not, or there are red flag signs
such as the possibility of cauda equina syndrome, referral to an orthopaedic or neurosurgeon should be
considered.
509. What is the most appropriate antibiotic to treat uncomplicated chlamydial infection in a 21yo
female who isnt pregnant?
a. Erythromycin
b. Ciprofloxacin
c. Metronidazole
d. Cefixime
e. Doxycycline

e. Doxycycline

Chlamydia is the most prevalent sexually transmitted infection in the UK.


Management
doxycycline (7 day course) or azithromycin (single dose). The 2009 SIGN guidelines suggest
azithromycin should be used first-line due to potentially poor compliance with a 7 day course of
doxycycline
if pregnant then erythromycin or amoxicillin may be used.

Potential complications
epididymitis
pelvic inflammatory disease
endometritis
increased incidence of ectopic pregnancies
infertility
reactive arthritis
perihepatitis (Fitz-Hugh-Curtis syndrome)

510. A 45yo manual worker presented with a 2h hx of chest pain radiating to his left arm. His ECG is normal.
What is the single most appropriate inv?
a. Cardiac enzymes
b. CXR
c. CT
d. ECG
e. V/Q scan

a. Cardiac enzymes
to rule out NSTEMI.
Non-ST-elevation ACS (NSTE-ACS): patients present with acute chest pain but without persistent ST-segment
elevation. The ECG shows persistent or transient ST-segment depression or T-wave inversion, flat T waves,
pseudo-normalisation of T waves, or no ECG changes at presentation.
Management:
All patients should receive
aspirin 300mg
nitrates or morphine to relieve chest pain if required

Antithrombin treatment. Fondaparinux should be offered to patients who are not at a high risk of bleeding
and who are not having angiography within the next 24 hours. If angiography is likely within 24 hours or a
patients creatinine is > 265 mol/l unfractionated heparin should be given.
Clopidogrel 300mg should be given to all patients and continued for 12 months.

Intravenous glycoprotein IIb/IIIa receptor antagonists (eptifibatide or tirofiban) should be given to patients
who have an intermediate or higher risk of adverse cardiovascular events (predicted 6-month mortality above
3.0%), and who are scheduled to undergo angiography within 96 hours of hospital admission.

Coronary angiography should be considered within 96 hours of first admission to hospital to patients who
have a predicted 6-month mortality above 3.0%. It should also be performed as soon as possible in patients
who are clinically unstable.

511. A 26yo woman had bipolar disorder for 10yrs and is on Lithium for it. She is symptom free for
the past 4 years. She is now planning her pregnancy and wants to know whether she should
continue taking lithium. What is the single most appropriate advice?
a. Continue lithium at the same dose and stop when pregnancy is confirmed
b. Continue lithium during pregnancy and stop when breast feeding
c. Reduce lithium dosage but continue throughout pregnancy
d. Reduce lithium gradually and stop when pregnancy is confirmed
e. Switch to sodium valproate

d. Reduce lithium gradually and stop when pregnancy is confirmed


symptom free for last 4 years. Lithium is teratogenic.
Adverse effects
nausea/vomiting, diarrhoea
fine tremor
polyuria (secondary to nephrogenic diabetes insipidus)
thyroid enlargement, may lead to hypothyroidism
ECG: T wave flattening/inversion
weight gain

Monitoring of patients on lithium therapy


inadequate monitoring of patients taking lithium is common - NICE and the National Patient Safety
Agency (NPSA) have issued guidance to try and address this. As a result it is often an exam hot topic
lithium blood level should 'normally' be checked every 3 months. Levels should be taken 12 hours post-
dose
thyroid and renal function should be checked every 6 months
Pregnancy: avoid in first the trimester (teratogenic). Only use in the second and third trimester if
considered essential, ie a severe risk to the patient, and monitor levels closely, as dose requirements
may alter.
Breast-feeding: avoid, as present in milk, and there is risk of toxicity in an infant. Bottle-feeding is
advisable.

Withdrawal
Abrupt withdrawal (both because of poor compliance or rapid change in dose) can precipitate relapse.
Withdraw lithium slowly over several weeks, watching for relapse.
512. A pt presents with dysphagia and pain on swallowing. He has sore mouth and soreness in the corners of
the mouth. What is the single most likely dx/
a. Kaposis sarcoma
b. Molluscum contagiosum
c. CMV infection
d. Candida infection
e. Toxoplasma abscess

d. Candida infection
Pain on swallowing is classic for candida.
Kaposi's sarcoma
caused by HHV-8 (human herpes virus 8)
presents as purple papules or plaques on the skin or mucosa (e.g. gastrointestinal and respiratory
tract)
skin lesions may later ulcerate
respiratory involvement may cause massive haemoptysis and pleural effusion
radiotherapy + resection

Molluscum contagiosum
The majority of cases occur in children (often in children with atopic eczema), with the maximum incidence in
preschool children aged 14 years.characteristic pinkish or pearly white papules with a central umbilication,
which are up to 5 mm in diameter. Lesions appear in clusters in areas anywhere on the body (except the palms
of the hands and the soles of the feet).

CMV infection
Infection is worldwide and usually asymptomatic. The most common disease manifestation is gastrointestinal
disease. CMV pneumonia is the most serious complication, but has become less common with prevention
strategies for at-risk patients. Rare manifestations include retinitis and encephalitis.

ToxoplasmaThis is asymptomatic in most cases.Toxoplasmic chorioretinitis.Toxoplasmic


encephalitis.Pneumonitis.Multiorgan involvement with respiratory failure and shock.

513. A 30yo lady has epistaxis for 30mins. Her Hgb is normal, MCV normal, WBC normal,
PT/APTT/Bleeding time are normal. Where is the defect?
a. Plts
b. Coagulation factor
c. Sepsis
d. Anatomical
e. RBC

d. Anatomical
All labs normal.

Trauma to the nose (the most common cause) - especially nose picking! Insertion of foreign bodies
and excessive nose blowing may also be seen as trauma. The latter is likely to occur with a cold
when the nasal mucosa is congested. Sinusitis causes nasal congestion.
Disorders of platelet function - thrombocytopenia and other causes of abnormal platelets, including
splenomegaly and leukaemia. Waldenstrm's macroglobulinaemia may present with nosebleeds.
Idiopathic thrombocytopenic purpura (ITP) can occur in children and young adults.
Drugs - aspirin and anticoagulants.
Disorders of platelets are more likely to be a problem than clotting factor deficiency.
Abnormalities of blood vessels in the elderly arteriosclerotic vessels prolong bleeding. Hereditary
haemorrhagic telangiectasia (Osler-Rendu-Weber syndrome) causes recurrent epistaxis from nasal
telangiectases.
Malignancy of the nose may present with bleeding - juvenile angiofibroma is a highly vascular benign
tumour that typically presents in adolescent males.
Cocaine use - if the septum looks sloughed or atrophic ask about use of cocaine.
Other conditions - Wegener's granulomatosis and pyogenic granuloma can present as an epistaxis.

514. Midpoint between the suprasternal notch and pubic symphysis. What is the single most
appropriate landmark?
a. Fundus of the gallbladder
b. Mcburneys point
c. Stellate ganglion
d. Deep inguinal ring
e. Transpyloric plane

e. Transpyloric plane
An upper transverse, the transpyloric, halfway between the jugular notch and the upper border of the
symphysis pubis; this indicates the margin of the transpyloric plane, which in most cases cuts through the
pylorus, the tips of the ninth costal cartilages and the lower border of the first lumbar vertebra;

Fundus located at the tip of the 9th costal cartilage


Mcburneys point one-third of the distance from the anterior superior iliac spine to the umbilicus
Stellate ganglion located at the level of C7 (7th cervical vertebrae), anterior to the transverse process of C7,
superior to the neck of the first rib, and just below the subclavian artery
Deep inguinal ring immediately above the midpoint of the inguinal ligament (midway between the anterior
superior iliac spine and the pubic tubercle)

Structures crossed
The transpyloric plane is clinically notable because it passes through several important abdominal structures.
These include:
lumbar vertebra 1 and hence passes just before the end of the spinal cord in adults.
the fundus of the gallbladder
the end of the spinal cord
the Neck of pancreas
the origin of the superior mesenteric artery from the abdominal aorta and termination of the superior
mesenteric vein at the hepatic portal vein
the left and right colic flexure
hilum of the kidney on the left
hilum of the kidney on the right
the root of the transverse mesocolon
duodenojejunal flexure
the 1st part of the duodenum
the upper part of conus medullaris
the spleen
the pylorus of the stomach which will lie at this level approximately 5 cm to the right of the midline.
cisterna chyli (which drains into the thoracic duct)

515. Tip of the 9th costal cartilage. What is the single most appropriate landmark?
a. Fundus of the gallbladder
b. Deep inguinal ring
c. Termination of the spinal cord
d. Transpyloric plane
e. Vena cava opening in the diaphragm

a. Fundus of the gallbladder


spinal cord around the L1/L2 vertebral level, forming a structure known as the conus medullaris.

Apertures through the diaphragm

1. Vena caval hiatus (vena caval foramen)


at the level of T8 and transmits the IVC and occasionally the phrenic nerve.
2. Esophageal hiatus
at the level of T10 and transmits the esophagus and vagus nerves.
3. Aortic hiatus
at the level of T12 and transmits the aorta, thoracic duct, azygos vein, an occasionally greater
splanchnic nerve.

516. A child complains of RIF pain and diarrhea. On colonoscopy, granular transmural ulcers are seen near the
ileo-cecal junction. What should be the management?
a. Sulfasalazine
b. Oaracetamol
c. Ibuprofen
d. Metronidazole

a. Sulfasalazine
Crohns disease-transmural ulcers
metronidazole is often used for isolated peri anal disease
Remission
glucocorticoids (oral, topical or intravenous) are generally used to induce remission.
5-ASA drugs (e.g. mesalazine) are used second-line to glucocorticoids but are not as effective
stopping smoking is a priority (remember: smoking makes Crohn's worse, but may help ulcerative colitis)

Maintenance
azathioprine or mercaptopurine is used first-line to maintain remission
methotrexate is used second-line
517. A 60yo woman presents with acute onset of bone and back pain following a rough journey in a car. Exam:
tenderness at mid-thoracic vertebra with spasm, she feels better once she bends
forward. What is the single most probable dx?
a. Osteoporotic fx verterbra
b. Myofacial pain
c. Whiplash injury
d. MI
e. Pancreatitis

b. Myofacial pain
Myofascial pain syndrome typically occurs after a muscle has been contracted repetitively.
Only myofascial pain/muscle sprain relieves on change of position.
Whiplash is only for cervical vertebrae
osteoporotic fx has dull pain and persistent or even worse on movement.
After repeated contraction of a muscle there occur spasm of the muscle and often the pain may felt in some
other part (referred pain). Here rough journey and associated spasm is clincher. In vertebral fracture you will
find neurological features which is absent here.
Spinal Stenosis pain also Relieved by sitting down, leaning forwards and crouching down

518. A 70yo woman presents with recurrent episodes of parotid swelling. She complains of difficulty in talking
and speaking and her eyes feel gritty on waking in the morning. What is the single most likely dx?
a. C1 esterase deficiency
b. Crohns disease
c. Mumps
d. Sarcoidosis
e. Sjogrens syndrome

e. Sjogrens syndrome
main symptoms of xerophthalmia (dry eyes), xerostomia (dry mouth) and enlargement of the parotid glands.
Difficulty eating dry food, typically cracker biscuits.
Difficulty with dentures.
Complaint of the tongue sticking to the roof of the mouth.
Speaking for long periods of time causes hoarseness.
Oral candidiasis and angular cheilitis.
Dry eyes tend to cause a gritty sensation. There is a predisposition to blepharitis and the eyes may be sticky in
the morning.
There may be recurrent parotitis, usually bilateral. Glands are usually enlarged but this is not often the
presenting feature.
Dryness of the mucosa of the trachea and bronchi may present as a dry cough.
Dryness of the pharynx and oesophagus may cause difficulty in swallowing, and lack of saliva and
secretions may predispose to gastro-oesophageal reflux.
There can be dry skin and vaginal dryness causing dyspareunia
Disease of the pancreas can lead to malabsorption and even acute pancreatitis or chronic pancreatitis
but a more likely cause of elevated serum amylase is parotitis.
Fatigue is a common feature.
About 20% have Raynaud's phenomenon.

Associated diseases
There may be a number of associated autoimmune conditions, such as the variant of scleroderma: calcinosis,
Raynaud's phenomenon, (o)esophageal motility disorder, sclerodactyly and telangiectasia (CREST). There may
be joint pain, swelling and fatigue rrecurrent miscarriage with antiphospholipid syndrome.

Investigations
Rheumatoid factor
Antinuclear antibodies
Schirmer test

519. A 39yo woman has not had her period for 10months. She feels well but is anxious as her mother had an
early menopause. Choose the single most appropriate initial inv?
a. Serum estradiol conc.
b. Serum FSH/LH
c. Serum progesterone conc.
d. None
e. Transvaginal US

b. Serum FSH/LH
Premature Menopause (ovarian faililure)shoud be ruled out.So FSH and LH(very high)
Premature Ovarian Failure (menopause before 40 years of age).
climacteric symptoms: hot flushes, night sweats
infertility
secondary amenorrhoea
raised FSH, LH levels
serum estradiol reflects primarily the activity of the ovaries. useful in the detection of baseline estrogen in
women with amenorrhea or menstrual dysfunction, and to detect the state of hypoestrogenicity and
menopause.
Serum progesterone: indicates if failure to ovulate
7 days prior to expected next period

520. A 50yo man with DM suddenly develops persistent crushing central chest pain radiating to the neck.
What is the single most appropriate dx?
a. Angina
b. Costochondritis (tietzs disease)
c. Dissecting aneurysm
d. MI
e. Pulmonary embolism

c. Dissecting aneurysm
Pain can radiate to back (classically described in questions) or to the neck as well. MI is an important
differential but usually MI in diabetics is silent one.
Angina does not radiate, costochondritis mostly have localised pain
In aortic dissection, pain is abrupt in onset and maximal at the time of onset. In contrast, the pain associated
with acute myocardial infarction starts slowly and gains in intensity with time. It is usually more oppressive
and dull.
Although tearing is the classical description, the pain is described as sharp more often than tearing, ripping, or
stabbing.
Investigations
Often the first problem is to distinguish aortic dissection from myocardial infarction. Both conditions may exist
if the dissection involves the coronary ostium. For this reason, the electrocardiogram (ECG) is very important.
Best: MRI

521. A 22yo man has rushed into the ED asking for help. He describes recurrent episodes of
fearfulness, palpitations, faintness, hyperventilation, dryness of the mouth with peri-oral
tingling and cramping of the hands. His symptoms last 5-10 mins and have worsened since their
onset 3months ago. He is worried he may be having a heart attack. An ECG shows sinus
tachycardia. What is the single most appropriate immediate intervention?
a. High flow oxygen
b. IV sedation
c. Rebreathe into a paper bag
d. Refer for anxiety management course
e. Refer for urgent cardiology opinion

c. Rebreathe into a paper bag

Characteristic symptoms experienced during panic attacks


Panic disorder manifests as the sudden, spontaneous and unanticipated occurrence of panic attacks, with
variable frequency, from several in a day to just a few per year:
Palpitations, pounding heart or accelerated heart rate.
Sweating.
Trembling or shaking.
Dry mouth.
Feeling short of breath, or a sensation of smothering.
Feeling of choking.
Chest pain or discomfort.
Nausea or abdominal distress.
Feeling dizzy, unsteady, light-headed or faint.
Derealisation or depersonalisation (feeling detached from oneself).
Fear of losing control or 'going crazy'.
Fear of dying.
Numbness or tingling sensations.
Chills or hot flushes.

522. An 8yo boy has longstanding asthma. He has admitted with a severe episode and is tired and drowsy. He
has not improved on oxygen, inhaled B2 agonist and IV hydrocortisone. CXR shows bilateral hyperinflation. He
is too breathless to use a peakflow meter and is O2 sat <90%. What is the single most appropriate inv?
a. CBG
b. CXR
c. CT chest
d. Pulse oximetry
e. Spirometry
a. CBG
cbg=capillary blood gas, abg= arterial blood gas any one can be used.
ABG to assess the severity to guide oxygen therapy and to decide if intubation is needed or not.

Clinical assessment of the severity of an acute asthma attack in those aged over 2 years
Acute severe Life-threatening
Unable to complete sentences in one breath. Silent chest.
Unable to feed or talk. Cyanosis.
Pulse >125 in those aged over 5 years or >140 in 2- to 5-year- Poor respiratory effort.
olds. Hypotension.
Respiratory rate >30 in those aged over 5 and >40 in 2- to 5-year- Exhaustion.
olds. Confusion.
Coma.

523. A man was operated for colorectal ca. His pain is relieved with morphine 60mg bd PO but now he cant
swallow medications. What will be the next regimen of analgesic administration?
a. Oxycodone
b. Fentanyl patch
c. Morphine 60mg IV/d
d. Morphine 240mg IV/d

b. Fentanyl patch

A regular 4-hourly starting dose for opioid-naive patients is usually 5-10 mg morphine.
Once pain relief is at a satisfactory and stable level, sustained-release preparations can be
substituted to allow od or bd dosing
Any breakthrough pain not associated with unusual activity should be treated with morphine elixir
or ordinary tablets at 1/6 total daily dose.
the breakthrough dose of morphine is one-sixth the daily dose of morphine
When increasing the dose of opioids the next dose should be increased by 30-50%.
If vomiting, dysphagia or increasing weakness prevent patients from taking oral morphine then
usual practice is to convert to a subcutaneous infusion of opioid via a device such as a syringe
driver.( in whom oral opioids are not suitable and analgesic requirements are unstable) Injection
site should be changed every 2-3 days.
An alternative to both oral morphine and subcutaneous diamorphine in patients with stable pain is
transdermal fentanyl or buprenorphine patches. (in whom oral opioids are not suitable and analgesic
requirements are stable) They can be useful in ambulatory patients where the following exist:
-Problems with the oral route.
-Intractable constipation or subacute obstruction.
-Morphine intolerance.
Consider if agitated confusion is due to opioid toxicity rather than uncontrolled pain before giving
further opioids. Consider switching to an alternative strong opioid. Alternatives include
hydromorphone, methadone and oxycodone
Consider changing the route of administration - eg where gastrointestinal absorption is poor, consider
switching to skin patches.
opioids should be used with caution in patients with chronic kidney disease. Alfentanil, buprenorphine
and fentanyl are preferred

Oral to oral route conversions[2]

Converting Converting to: Divide 24-hour dose of current opioid by figure below to
calculate initial 24-hour dose of new opioid
from:
(new opioid)

(current
opioid)

oral codeine oral morphine Divide by 10

oraltramadol oral morphine Divide by 5

oral morphine oral oxycodone Divide by 2

oral morphine oralhydromorphone Divide by 7.5

524. Just above the mid-inguinal point. What is the single most appropriate landmark?
a. Femoral artery pulse felt
b. Mcburneys point
c. Stellate ganglion
d. Deep inguinal ring
e. Transpyloric plane
d. Deep inguinal ring

525. 5th ICS in the ant axillary line. What is the single most appropriate landmark?
a. Apex beat
b. Chest drain insertion
c. Stellate ganglion
d. Transpyloric plane
e. Vena cava opening into the diaphragm

b. Chest drain insertion


Surface anatomy of Apex beat is left 5th ICS midclavicular line.

526. A 34yo man with MS has taken an OD of 100 tablets of paracetamol with intent to end his life. He has
been brought to the ED for tx but is refusing all intervention.
a. Assessment
b. Evaluate pts capacity to refuse tx
c. Establish if pt has a prv mental illness

b. Evaluate pts capacity to refuse tx

Urgent treatment
Consent not needed when urgent treatment is required:

To save the patient's life.


To prevent a serious deterioration in the patient's condition, so long as the treatment is not
irreversible.
To alleviate serious suffering so long as the treatment is neither irreversible nor hazardous.
To prevent the patient from behaving violently or being a danger to self or others so long as the
treatment is neither irreversible nor hazardous, and represents the minimum interference
necessary.

527. A 23yo woman with painless vaginal bleeding at 36wks pregnancy otherwise seems to be
normal. What should be done next?
a. Vaginal US
b. Abdominal US
c. Vaginal exam
d. Reassurance

b. Abdominal US
to assess fetal being and check placenta previa
vaginal US is more accurate but not initial when bleeding.
No PV until no PP

Placenta previa
Painless bleeding starting after the 28th week (although spotting may occur earlier) is usually the main sign.

Acute bleeding
Admit the patient to hospital.

DO NOT PERFORM A VAGINAL EXAMINATION, as this may start torrential bleeding in the presence of
placenta praevia.

Blood loss is assessed and cross-matched for possible transfusion.


Resuscitation if indicated; the mother is the priority and should be stabilised prior to any
assessment of the fetus.
Appropriate surgical intervention may be required:
o In severe bleeding the baby is delivered urgently whatever its gestational age.
o Hysterectomy should also be considered in severe cases.
If immediate delivery is not likely, maternal steroids may be indicated in order to promote fetal
lung development and reduce the risk of respiratory distress syndrome and intraventricular
haemorrhage.

528. A 29yo lady admitted with hx of repeated UTI now developed hematuria with loin pain. What is the most
probable dx?
a. Acute pyelonephritis
b. Chronic pyelonephritis
c. UTI
d. Bladder stone

a. Acute pyelonephritis

Presentation
Onset is usually rapid with symptoms appearing over a day or two. There is unilateral or bilateral loin pain,
suprapubic or back pain. Fever is variable but can be high enough to produce rigors. Malaise, nausea,
vomiting, anorexia and occasionally diarrhoea occur. There may or may not be accompanying lower urinary
tract symptoms with frequency, dysuria, gross haematuria or hesitancy. Gross haematuria occurs in 30-40% of
young women. The patient looks ill and there is commonly pain on firm palpation of one or both kidneys and
moderate suprapubic tenderness without guarding.
Investigation of choice:
Contrast-enhanced helical/spiral CT (CECT) scan is the best investigation in adults
In children, the choice is between ultrasound and CT scanning. CT is more sensitive but the exposure to
radiation may make ultrasound a safer option.
Treatment: ciprofloxacin for seven to ten days
529. A 45yo chronic smoker attends the OPD with complaints of persistent cough and copious
amount of purulent sputum. He had hx of measles in the past. Exam: finger clubbing and
inspiratory crepitations on auscultation. What is the single most likely dx/
a. Interstitial lung disease
b. Bronchiectasis
c. Asthma
d. COPD
e. Sarcoidosis

b. Bronchiectasis
Bronchiectasis -(clubbing is not present in pure COPD)
Plus all the signs and symptoms are characteristic of bronchiectasis.
- Copious and purulent sputum
- Finger clubbing
- Post infective occurrence (as can be noted with pertussis, measles, recurrent childhood bronchiolitis etc)
That being said, he probably has underlying COPD owing to the history of chronic smoking, which makes him
susceptible to repeated viral infections of the respiratory tract and consequently bronchiectasis.
If COPD ever presents with finger clubbing, we investigate for underlying bronchogenic Ca or bronchiectasis.

Interstitial lung disease The most common symptom of all forms of interstitial lung disease is shortness of
breath. dry nonproductive cough.
Asthma wheeze, breathlessness, chest tightness and cough, particularly if:
symptoms worse at night and in the early morning
symptoms in response to exercise, allergen exposure and cold air
symptoms after taking aspirin or beta blockers
COPD patients over 35 years of age who are smokers or ex-smokers and have symptoms such as exertional
breathlessness, chronic cough or regular sputum production.
Bronchiectasis
Bronchiectasis describes a permanent dilatation of the airways secondary to chronic infection or
inflammation.
Post-infection - eg, childhood respiratory viral infections (measles, pertussis, influenza, respiratory
syncytial virus), tuberculosis, bacterial pneumonia. Infection is the most common cause.
persistent daily expectoration of large volumes of purulent sputum.
dyspnoea, chest pain and haemoptysis. Bronchiectasis may progress to respiratory failure and cor
pulmonale.
Coarse crackles are the most common finding
The gold standard for diagnosis is HRCT of the chest.
first-line treatment is amoxicillin 500 mg three times a day or clarithromycin 500 mg twice daily
All children and all adults up to the age of 40, presenting with bronchiectasis, should have
investigations for cystic fibrosis.

530. A 68yo man has had malaise for 5 days and fever for 2 days. He has cough and there is dullness to
percussion at the left lung base. What is the single most appropriate inv?
a. Bronchoscopy
b. CXR
c. CT
d. MRI
e. V/Q scan

b. CXR
If a person is sufficiently sick to require hospitalization, a chest radiograph is recommended.
A CT scan can give additional information in indeterminate cases.
Pneumonia
Presentation
Symptoms: cough, purulent sputum which may be blood-stained or rust-coloured, breathlessness,
fever, malaise.
Diagnosis is unlikely if there are no focal chest signs and heart rate, respiratory rate and temperature
are normal.
The elderly may present with mainly systemic complaints of malaise, fatigue, anorexia and myalgia.
Signs: tachypnoea, bronchial breathing, crepitations, pleural rub, dullness with percussion.

CURB-65 criteria of severe pneumonia


Confusion (abbreviated mental test score <= 8/10)
Urea > 7 mmol/L
Respiratory rate >= 30 / min
BP: systolic <= 90 or diastolic <= 60 mmHg
age >= 65 years
Management
low or moderate severity CAP: oral amoxicillin. A macrolide should be added for patients admitted to
hospital
high severity CAP: intravenous co-amoxiclav + clarithromycin OR cefuroxime + clarithromycin OR
cefotaxime + clarithromycin

531. A 5yo child was admitted with hx of feeling tired and lethargic all the time, bleeding gums and
sore throat since the last 3months. Exam: hepatosplenomegaly. What is the most probable dx?
a. ALL
b. AML
c. CML
d. CLL
e. Lymphoma

a. ALL
Acute lymphoblastic leukaemia (ALL) is the most common type in children, primarily affecting 2 to 8 year-olds.
Approximately 2/3s of AML patients are over 60.
hepatosplenomegaly in both. lymphadenopathy in ALL.
AML presents with bleeding gums, but bleeding tendency in ALL too. ALL also has gum hypertrophy.

ALL
Symptoms
Fatigue, dizziness and palpitations
bone and joint pain
Recurrent and severe infections (oral, throat, skin, perianal infections commonly)
Fever
Left upper quadrant fullness and early satiety due to splenomegaly (10-20%)
Haemorrhagic or thrombotic complications due to thrombocytopenia or disseminated intravascular
coagulopathy (DIC) - for example, menorrhagia, frequent nosebleeds, spontaneous bruising
Signs
Pallor
Tachycardia and a flow murmur
Nonspecific signs of infection
Petechiae,purpura or ecchymoses
hepatosplenomegaly
Lymphadenopathy
Gum hypertrophy
Cranial nerve palsy (especially III, IV, VI and VIII) in mature B-cell ALL

532. A 65yo man presents with back pain. Exam: splenomegaly and anemia. Blood: WBC=22,
Hgb=10.9, Plt=100, ESR=25. He has been found to have Philadelphia chromosome. What is the
single most likely dx?
a. ALL
b. AML
c. CML
d. CLL
e. Lymphoma

c. CML
CML is characterised by a consistent cytogenetic abnormality - a reciprocal translocation between the long
arms of chromosomes 22 and 9, t(9;22). The result is a shortened chromosome 22, known as the Ph
chromosome.

Symptoms
Fatigue.
Night sweats.
Weight loss.
Abdominal fullness or abdominal distension.
Left upper quadrant pain due to splenic infarction.

Signs
Splenomegaly - the most common physical finding
Hepatomegaly.
Enlarged lymph nodes are also a possibility.
Anaemia
Easy bruising.
Fever.
Gout due to rapid cell turnover.
Hyperviscosity syndrome due to leukocytosis - visual disturbance (fundoscopy may show papilloedema,
venous obstruction and retinal haemorrhages), priapism,cerebrovascular accident (CVA), confusion.
FBC:
o Leukocytosis is common.
o Differential shows granulocytes at all stages of development and increased numbers of
eosinophils and basophils.
o Platelets may be elevated, decreased or normal levels.
o A mild-to-moderate, usually normochromic and normocytic, anaemia is common.
Peripheral blood smear - all stages of maturation seen; often resembles a bone marrow aspiration.

533. A 24yo woman has 8wk amenorrhea, right sided pelvic pain and vaginal bleeding. She is
apyrexial. Peritonism is elicited in the RIF. Vaginal exam reveals right sided cervical excitation.
What is the most probable dx?
a. Ectopic pregnancy
b. Salpingitis
c. Endometriosis
d. Ovarian torsion
e. Ovarian tumor

a. Ectopic pregnancy
amenorrea, pelvic pain n vaginal bleeding clinical triad of ectopic pregnancy
cervical excitation is sign in ectopic pregnancy and PID
cervical motion tenderness which is when bilateral we suspect PID n when unilateral ectopic most likely
since the pt is apyrexial this rules out PID

The most common symptoms are


Abdominal pain.
Pelvic pain.
Amenorrhoea or missed period.

Examination
There may be some tenderness in the suprapubic region.
Peritonism and signs of an acute abdomen may occur.
Women with a positive pregnancy test and any of the following need to be referred immediately to
hospital:
o Pain and abdominal tenderness.
o Pelvic tenderness.
o Cervical motion tenderness.
Vaginal bleeding (with or without clots).
The most accurate method to detect a tubal pregnancy is transvaginal ultrasound.
Human chorionic gonadotrophin (hCG) levels are performed in women with pregnancy of unknown location
who are clinically stable.
534. A 64 yo woman has been treated for breast cancer with tamoxifen. What other drug should be added to
her tx regime?
a. Bisphosphonates
b. Calcium
c. Vit D
d. Calcitonin
e. Phosphate binders

a. Bisphosphonates
bisphosphonates reduce the risk of bone metastasis in cancers and is normally taken as adjuvant therapy in
many types of tumours including breast cancer
Plus it prevents bone resorption
breast cancer can easily metastasise to bones and that in turn will lead to bone lysis so maybe thats why we
need bisphosphonates.
since the patient has been treated with TAMOXIFEN , we conclude the fact that she has a metastatic disease.
According to BNF , the use of BISPHOSPHONATE in patients with metastatic breast cancer may reduce pain
and prevent skeletal complications of bone metastases.
The other choices like calcitonin and VIT D are of little value in postmenopausal women with metastases.

Bisphosphonates
Bisphosphonates decrease demineralisation in bone. They inhibit osteoclasts.

Clinical uses
prevention and treatment of osteoporosis
hypercalcaemia
Paget's disease
pain from bone metatases

Adverse effects
oesophagitis, oesophageal ulcers
osteonecrosis of the jaw
increased risk of atypical stress fractures of the proximal femoral shaft.

535. A 6yo woman with regular menses and her 28yo partner comes to the GP surgery complaining of primary
infertility for 2yrs. What would be the single best investigation to see whether she is ovulating or not?
a. Basal body temp estimation
b. Cervical smear
c. Day2 LH and FSH
d. Day21 progesterone
e. Endometrial biopsy

d. Day21 progesterone
To check for whether it is ovulatiry or anovulatiry cycles. Progesterone level inceases and peaks 5 to six days
post ovulation. Which is 21 day progesterone levels
Infertility affects around 1 in 7 couples. Around 84% of couples who have regular sex will conceive within 1
year, and 92% within 2 years

Causes
male factor 30%
unexplained 20%
ovulation failure 20%
tubal damage 15%
other causes 15%

Basic investigations
semen analysis
serum progesterone 7 days prior to expected next period (day 21 of 28 day cycle)
indicates ovulation.

Interpretation of serum progestogen

Level Interpretation

< 16 nmol/l Repeat, if consistently low refer to specialist

16 - 30 nmol/l Repeat

> 30 nmol/l Indicates ovulation

Key counselling points


folic acid
aim for BMI 20-25
advise regular sexual intercourse every 2 to 3 days
smoking/drinking advice

536. A 10yo boy who takes regular high dose inhaled steroids for his longstanding asthma has been advised to
use bronchodilators to control his acute attacks. His parents are unsure when should he use his
bronchodilator. What is the single most appropriate inv?
a. CXR
b. None
c. Peak flow rate diary
d. Pulse oximetry
e. Spirometry

c. Peak flow rate diary


spirometry is to make the diagnosis while Peak flow rate is for monitoring
PEF decides the severity of asthma attack..so when PEF will decreas during day he could take bronchodilato
A peak flow diary is a useful way for you to write down and record your peak flow scores on a regular basis.

A peak flow is a measurement of how much air you can blow out of your lungs in a set amount of time. Peak
flow scores help you to see when your asthma is getting better of worse.

537. A woman presented with blurred vision and intermittent clumsiness for 3m. Reflexes are brisk in her arm
and optic disc is pale. What is the single most appropriate test to confirm dx?
a. CSF analysis
b. CT
c. MRI
d. EEG
e. EMG

c. MRI
multiple sclerosis, investigation of choice is gadolinium enhanced mri.

Multiple sclerosis: features


3 times more common in women
most commonly diagnosed in people aged 20-40 years

Patient's with multiple sclerosis (MS) may present with non-specific features, for example around 75% of
patients have significant lethargy.

Visual
optic neuritis: common presenting feature
optic atrophy
Uhthoff's phenomenon: worsening of vision following rise in body temperature
internuclear ophthalmoplegia

Sensory
pins/needles
numbness
trigeminal neuralgia
Lhermitte's syndrome: paraesthesiae in limbs on neck flexion

Motor
spastic weakness: most commonly seen in the legs

Cerebellar
ataxia: more often seen during an acute relapse than as a presenting symptom
tremor

Others
urinary incontinence
sexual dysfunction
intellectual deterioration

538. A 63yo man presents after having a seizure. Exam: alert, orientated, inattention on the left side and
hyperreflexia of the arm. What is the most probable dx?
a. Cerebral tumor
b. Pituitary adenoma
c. Cerebellar abscess
d. Huntingtons chorea
e. Parkinsonism

a. Cerebral tumor
Inattention or neglect is a feature of parietal lobe lesion. If lesion is on right side there will be left sided
inattention. That is patient is unaware of his left side and he when shaves do it only to right half of face, during
eating eats only from the right half of plate and can not drive as he only aware of his right side and totally
unaware of left side of the road
nothing else fit thats the best expl.. pituitary will give bitemporal vision loss cerebellar signs are nystagmus
ataxia etc. Chorea is repeated movements.. its focal ant lobe lesion.
Seizure n hyperflexia (exaggerated reflexes) are indicators of space occupied lesion (SOL)
Because the pt has got upper motor neuron signs.
Its not pitutry because no signs of optic n compression and its not bilateral.
Its not cerebellar dis other wise he should have ataxia, pass pointing rombergism.
Its not chorea cuz no symptoms of chorea.
Not parkinson because no hypokinesia, tremor
seizure causing neuro deficit in elderly... first D/D should b cerebral tumour unless specified otherwise..

539. A 40yo man with a 25y hx of smoking presents with progressive hoarseness of voice, difficulty swallowing
and episodes of hemoptysis. He mentioned that he used to be a regular cannabis user. What is the single most
likely dx?
a. Nasopharyngeal cancer
b. Pharyngeal carcinoma
c. Sinus squamous cell carcinoma
d. Squamous cell laryngeal cancer
e. Hypopharyngeal tumor

d. Squamous cell laryngeal cancer


Hoarseness of voice is localizing the problem to the larynx

The symptoms of cancer of the pharynx differ according to the type:


Oropharynx: common symptoms are a persistent sore throat, a lump in the mouth or throat, pain in
the ear.
Hypopharynx: problems with swallowing and ear pain are common symptoms and hoarseness is not
uncommon.
Nasopharynx: most likely to cause a lump in the neck but may also cause nasal obstruction, deafness
and postnasal discharge.

Laryngeal Cancer
Smoking is the main avoidable risk factor for laryngeal cancer
Chronic hoarseness is the most common early symptom.
urgent CXR to decide where to refer
Flexible laryngoscopy is the best way to inspect the larynx
staging include CT and/or MRI scans.

540. A 30yo lady complains of intermittent diarrhea, chronic abdominal and pelvic pain and
tenesmus. Sometimes she notices blood in her stool. Select the most likely cause leading to her
symptoms?
a. Inflammatory bowel disease
b. Diverticulosis
c. Irritable bowel disease
d. Adenomyosis
e. UTI

a. Inflammatory bowel disease


classic features.

Diverticulosis is defined as the presence of diverticula which are asymptomatic.


The diagnosis of IBS should be considered if the patient has had the following for at least 6 months:
abdominal pain, and/or
bloating, and/or
change in bowel habit
Adenomyosis is the invasion of the myometrium by endometrial tissue. Extrauterine endometrial
tissue causes inflammation, pain and the formation of adhesions. Clinically its significance is as a cause
of chronic pelvic pain, dyspareunia and female infertility.

541. A 50yo lady with weak limbs when examined was found to have burn marks on finger tips,
wasted and weak hands with diminished reflexes. She also has weak spastic legs and dissociated
sensory loss. What is the dx?
a. MS
b. Syringomyelia
c. MND
d. Guillian-barre
e. Freidriechs ataxia

b. Syringomyelia
weakness and wasting of muscle, diminished or loss of tendon reflex, loss of pain and temperature sense,
experience of pain on touching skin, there are burn marks on finger due to loss of temperature sense, these all
points towards syringomyelia
MS- causes UMNL with brisk reflex not causes dissociated sensory loss. most common cause of dissociated
sensory loss - syringomyelia,& Brown-sequard syndrome.
motor+sensory-syringomyelia
motor+tongue-syringobulbia
motor only MND

Friedreich's ataxia is a progressive neurodegenerative disorder, typically with onset before 20 years of age.
Signs and symptoms include progressive ataxia, ascending weakness and ascending loss of vibration and joint
position senses, pes cavus, scoliosis, cardiomyopathy, and cardiac arrhythmias.

If the syrinx extends into the brainstem, syringobulbia results. This may affect one or more cranial nerves,
resulting in facial palsies.
Syringomyelia is more common in men

Sensory features
Pain and temperature sensation are lost due to spinothalamic tract damage.
Classically, the sensation loss is experienced in a shawl-like distribution over the arms, shoulders and
upper body.
Dysaesthesia (pain experienced when the skin is touched) is common.
Light touch, vibration and position senses in the feet are affected as the syrinx enlarges into the dorsal
columns.
Painless ulcers on the hands.
Motor features
These begin to occur as the syrinx extends and damages the lower motor neurons of the anterior horn
cells.
Muscle wasting and weakness begins in the hands and then affects the forearms and shoulders.
Tendon reflexes are lost.
Claw hand may be present.
There may be respiratory muscle involvement.

Autonomic features
Bladder, bowel and sexual dysfunction can occur.
Horner's syndrome may be present.
Investigations
MRI is now primarily used for diagnosis
CT scanning is better at showing abnormalities of bony spinal canal, whilst MRI scanning is better at
showing soft tissue.
Plain X-rays may show a widened cervical canal.
Lumbar puncture is best avoided because of risk of herniation.

542. A 23yo woman is being followed up 6wks after a surgical procedure to evacuate the uterus
following a miscarriage. The histology has shown changes consistent with a hydatidiform mole.
What is the single most appropriate inv in this case?
a. Abdominal US
b. Maternal karyotype
c. Paternal blood group
d. Serum B-HCG
e. Transvaginal US
d. Serum B-HCG

Complete hydatidiform mole

Occurs when an empty egg is fertilized by a single sperm that then duplicates its own DNA, hence the all 46
chromosomes are of paternal origin

Features
bleeding in first or early second trimester
exaggerated symptoms of pregnancy e.g. hyperemesis
uterus large for dates
very high serum levels of human chorionic gonadotropin (hCG)
hypertension and hyperthyroidism* may be seen

Management
urgent referral to specialist centre - evacuation of the uterus is performed
effective contraception is recommended to avoid pregnancy in the next 12 months

Around 2-3% go on to develop choriocarcinoma

In a partial mole a normal haploid egg may be fertilized by two sperms, or by one sperm with duplication of
the paternal chromosomes. Therefore the DNA is both maternal and paternal in origin. Usually triploid - e.g.
69 XXX or 69 XXY. Fetal parts may be seen

*hCG can mimic thyroid-stimulating hormone (TSH)

543. A 67yo man with hx of weight loss complains of hoarseness of voice. CT reveals opacity in the right upper
mediastinum. He denied any hx of difficulty breathing. What is the single most
appropriate inv?
a. Laryngoscopy
b. Bronchoscopy
c. LN biopsy
d. Bronchoalveolar lavage
e. Barium swallow

c. LN biopsy
Hoarseness + weight loss.. suspicion of malignancy.

hoarseness due to Malignancy


Laryngeal cancer - smoking is major risk factor.
Other neck or chest tumours - eg, lung cancer, lymphoma, thyroid cancer.

Hoarseness persisting for >3 weeks requires investigation to exclude malignancy:


Carcinomas of larynx and lung must be considered, so CXR and/or laryngoscopy are indicated.
National Institute for Health and Care Excellence (NICE) guidance on suspected cancer states that for
patients with hoarseness persisting for >3 weeks, particularly smokers aged 50 years and heavy
drinkers:
o Arrange urgent CXR.
o Refer patients with positive findings urgently to a team specialising in the management of lung
cancer.
o Refer patients with a negative finding urgently to a team specialising in head and neck cancer.

544. A 52yo man whose voice became hoarse following thyroid surgery 1 wk ago shows no
improvement. Which anatomical site is most likely affected?
a. Bilateral recurrent laryngeal nerve
b. Unilateral recurrent laryngeal nerve
c. Unilateral external laryngeal nerve
d. Bilateral external laryngeal nerve
e. Vocal cords

b. Unilateral recurrent laryngeal nerve


bilateral injury of the RLN leads to aphonia.
In unilateral damage, the patient voice is still preserved but it's harsh ( hoarse ) due to unilateral paralysis of
the vocal cords.
Direct injury to the vocal cords is unlikely in thyroid procedures since the larynx isn't opened.
The external laryngeal nerves are more frequently damaged than the RLN , but they cause only minor changes
in voice quality ( pitch changes).
Bilateral rln palsy will cause emergency airway obstruction and stridor
Vocal cord inj should be transient and improving
External laryngeal inj doesnt cause hoarseness
B due to close relation of the inferior thyroid artery to the recurrent laryngeal nerve the clamping of artery
during surgery might accidentally injured the nerve causing hoarseness of voice if bilaterally affected the
nerve it will most likely causing acute respiratory distress

545. A 73yo male presents with a 12m hx of falls. His relatives have also noticed rather strange
behavior of late and more recently he has had episodes of enuresis. Exam: disorientation to
time and place, broad-based, clumsy gait. What is the most probable dx?
a. Dementia
b. Pituitary adenoma
c. CVD
d. Syringomyelia
e. Normal pressure hydrocephalus

e. Normal pressure hydrocephalus


clincher: Gait, dementia and enuresis
the wet, wobbly and wacky grandpa
It is a reversible cause of dementia. Seen in elderly patients. Secondary to reduced CSF absorption at the
arachnoid villi.
Classical triad of features:-
1. Urinary incontinence
2. Dementia
3. Bradyphrenia
4. Gait abnormality
Imaging:
Hydrocephalus with an enlarged 4th ventricle
Management:
Ventriculoperitoneal shunting

one in which person talks vulgar things---> frontotemporal dementia


One in which dementia fluctuates---> lewy body dementia
One asso with parslysis---> vascular dementia
One with stepwise deterioration---> Alzheimer's disease

546. A 75yo nursing home resident complains of headache, confusion and impaired vision for 4days. She has
multiple bruises on her head. What is the most likely cause of confusion in this pt/
a. Alcohol intoxication
b. Infection
c. Subdural hematoma
d. Hypoglycemia
e. Hyponatremia

c. Subdural hematoma
multiple bruises on her head

Subdural Bleeding into the outermost meningeal layer. Most commonly occur around the frontal
haematoma and parietal lobes.

Risk factors include old age, alcoholism and anticoagulation.

Slower onset of symptoms than a epidural haematoma.

547. A 50yo woman returned by air to the UK from Australia. 3days later she presented with sharp chest pain
and breathlessness. Her CXR and ECG are normal. What is the single most
appropriate inv?
a. Bronchoscopy
b. Cardiac enzymes
c. CT
d. MRI
e. Pulse oximetry
f. V/Q scan
g. CTPA

g. CTPA
Long flight and sharp chest pain along with breathlessness points towards PE
As per NICE guidelines the most appropriate investigation is CTPA if ur suspecting PE. V/Q scan is preferred in
only few situation like pregnancy, Ckd patients, or ctpa n/a. U have to do wells scoring of the patient and if its
> 4, u straight away do CTpa .. Dont even wait for d dimer. Definitely CTPA in this case

medical student textbook triad of pleuritic chest pain, dyspnoea and haemoptysis.
computed tomographic pulmonary angiography (CTPA) is now the recommended gold standard
Management:
Low molecular weight heparin (LMWH) or fondaparinux should be given initially after a PE is diagnosed.An
exception to this is for patients with a massive PE where thrombolysis is being considered. In such a situation
unfractionated heparin should be used.
a vitamin K antagonist (i.e. warfarin) should be given within 24 hours of the diagnosis
the LMWH or fondaparinux should be continued for at least 5 days or until the international
normalised ratio (INR) is 2.0 or above for at least 24 hours, whichever is longer, i.e. LMWH or
fondaparinux is given at the same time as warfarin until the INR is in the therapeutic range
warfarin should be continued for at least 3 months.
NICE advise extending warfarin beyond 3 months for patients with unprovokedPE.
for patients with active cancer NICE recommend using LMWH for 6 months

Thrombolysis
thrombolysis is now recommended as the first-line treatment for massive PE where there is circulatory
failure (e.g. hypotension). Other invasive approaches should be considered where appropriate facilities
exist

548. A tall thin young man has sudden pain in the chest and becomes breathless while crying. What is the
single most appropriate inv?
a. Cardiac enzymes
b. CXR
c. CT
d. ECG
e. V/Q scan

b. CXR
spontaneous pneumothorax .most often in young thin male ..due to rupture of bullous emphysema
patient may have marfans syndrome or alpha-1 antitrypsin deficiency

Secondary pneumothorax

Recommendations include:
if the patient is > 50 years old and the rim of air is > 2cm and/or the patient is short of breath then a
chest drain should be inserted.
otherwise aspiration should be attempted if the rim of air is between 1-2cm. If aspiration fails (i.e.
pneumothorax is still greater then 1cm) a chest drain should be inserted. All patients should be
admitted for at least 24 hours
if the pneumothorax is less the 1cm then the BTS guidelines suggest giving oxygen and admitting for 24
hours

549. A 21yo woman has had several sudden onset episodes of palpitations, sweating, nausea and
overwhelming fear. On one occasion she was woken from sleep and feared she was going insane. There is no
prv psychiatric disorder. What is the most probable dx?
a. Pheochromocytoma
b. Panic disorder
c. GAD
d. Phobia
e. Acute stress disorder

b. Panic disorder
panic attack is MORE likely... it can be pheochromocytoma but "overwhelming fear" makes panic attack
look more fitting... It cant be phobia, because the attacks are just random without any known trigger

Pheochromocytoma pressure symptoms

Treatment in primary care


NICE recommend either cognitive behavioural therapy or drug treatment
SSRIs are first-line. If contraindicated or no response after 12 weeks then imipramine or clomipramine
should be offered

550. A 55yo woman with a persistent cough and hx of smoking develops left sided chest pain
exacerbated by deep breathing with fever and localized crackles. What is the single most
appropriate dx?
a. Dissecting aneurysm
b. Pericarditis
c. Pneumonia
d. Pneumothorax
e. Pulmonary embolism

c. Pneumonia

dissec aneurysm will have a sharp pain radiating to the back.


pericarditis has similiar features , but i dont see a cardiac cause.
pneumothorax is seen in smokers , as a complication to COPD , but it wont have fever and above features. (
reduced air entry is seen in it, with hyper resonance on percussin ).
pulm embloism also seems unlikely , should have travel history .
looks like pneumonia then - pain on inspiration ( pleural pain ), fever , cough, and crackles !

Klebsiella pneumoniae is classically in alcoholics

Streptococcus pneumoniae (pneumococcus) is the most common cause of community-acquired pneumonia

Characteristic features of pneumococcal pneumonia


rapid onset
high fever
pleuritic chest pain
herpes labialis

Management

CURB-65 criteria of severe pneumonia


Confusion (abbreviated mental test score <= 8/10)
Urea > 7 mmol/L
Respiratory rate >= 30 / min
BP: systolic <= 90 or diastolic <= 60 mmHg
age >= 65 years

Patients with 3 or more (out of 5) of the above criteria are regarded as having a severe pneumonia

The British Thoracic Society published guidelines in 2009:


low or moderate severity CAP: oral amoxicillin. A macrolide should be added for patients admitted to
hospital
high severity CAP: intravenous co-amoxiclav + clarithromycin OR cefuroxime + clarithromycin OR
cefotaxime + clarithromycin

551. A 40yo woman complains of dysphagia for both solids and liquids. She sometimes suffers from
severe retrosternal chest pain. Barium swallow reveals a dilated esophagus which tapers to a
fine distal end. What is the best management strategy?
a. Reassurance
b. Antispasmodics
c. Dilatation of the LES
d. Endoscopic diverticulectomy
e. Barium swallow

c. Dilatation of the LES


Achalasia typically presents in middle-age and is equally common in men and women
Investigations
manometry: excessive LOS tone which doesn't relax on swallowing - considered most important
diagnostic test
barium swallow shows grossly expanded oesophagus, fluid level, 'bird's beak' appearance. This is in
contrast to the rat's tail appearance of carcinoma of the oesophagus
CXR: wide mediastinum, fluid level
Gold standard - Manometery

Treatment
intra-sphincteric injection of botulinum toxin
Heller cardiomyotomy for fit young patients.
balloon dilation for old unwell patients.

Complications : Aspiration pneumonia, perforation, GERD, Oesophagus CA.

552. A 38yo female G4 at 32wks of pregnancy presented with thick white marks on the inside of her mouth for
3wks. Her mouth including her tongue appeared inflamed on examination. She
smokes 20 cigarettes/day despite advice to quit. She attends her ANC regularly. What is the
most probable dx?
a. Lichen planus
b. Aphthous ulcer
c. Smoking
d. Candidiasis
e. Leukoplakia
d. Candidiasis
pregnancy is one of the risk factors, as well as smoking.
Oral fluconazole 50 mg/day for 7 days.

Prevention of oral candidiasis


Patients taking oral/inhaled steroids - good inhaler technique, spacer device, rinse mouth with water
after use.
Denture wearers - thorough cleaning of dentures, leave them out at night, ensure they fit correctly.
Smoking cessation.

Lichen planus is a flat topped violaceous skin lesion not associated with smoking.
Classically, white slightly raised lesions with a trabecular, lacy appearance on the inside of the cheeks.Can be
precipitated by trauma (Kbner's phenomenon).Topical steroids are considered to be the first-line treatment
for oral lichen planus. Topical immunomodulators (eg, imiquimod) may be useful as second-line treatment in
severe oral lichen planus.

Aphthous ulcer clearly defined, painful, shallow rounded ulcers not associated with systemic disease. They are
not infective.

Leukoplakia This is a white patch adhering to oral mucosa that cannot be removed by rubbing. It is usually a
diagnosis of exclusion.

553. A 69yo woman has had a stroke. Her left upper and lower limbs are paralyzed and she is having difficulty
in speaking. Which anatomical site is most likely affected?
a. Hippocampus
b. Cerebellum
c. Internal capsule
d. Thalamus
e. Brain stem

c. Internal capsule
Internal capsule. Its lacunar infarct. Internal capsule has both corticospinal and corticobulbar fibers. Infarct
results in hemiparesis/ hemiplegia with dysphagia/ dysarthria
Right middle cerebral artery territory is affected that irrigates internal capsule structure and broca area too
by exclusion method for me:
A would've caused memory deficit.
B some typical signs.
D sensory loss ipsilat.
E at least 2 cranial nerves involved
554. A 72yo man brought to the ED with onset of paraplegia following a trivial fall. He was treated for prostatic
malignancy in the past. What is the single most probable dx?
a. Pagets disease
b. Osteoporotic fx of vertebre
c. Secondary
d. Multiple myeloma
e. Spondylosis

c. Secondary
Bones weakened from metastatic cancer may break (fracture). The fracture can happen with a fall or injury,
but a weak bone can also break during everyday activities. These fractures often cause sudden, severe pain.
The pain may keep you from moving much at all. In some cases, a fracture is the first sign of bone metastasis.
The most common sites of fractures are the long bones of the arms and legs and the bones of the spine.
Sudden pain in the middle of the back, for example, is a common symptom of a bone in the spine breaking and
collapsing from cancer.
c.a prostate led to vertebral mets(most common bone secondary in c.a prostate),that lead to fracture on
trivial injury and paraplegia due to spinal cord compression

Multiple myeloma can present with a wide variety of symptoms including hypercalcaemia, anaemia, renal
impairment and bone pain.
Spondylosis inflammatory back pain and enthesitis (inflammation at the site of bone insertion of ligaments
and tendons) or arthritis with radiological findings.

Pagets disease most common complaints are bone pain and/or deformity.pathological fractures elevated
serum alkaline phosphatase or characteristic abnormality on X-ray.

555. A 14yo girl has developed an itchy, scaly patch on her scalp. She had a similar patch that cleared
spontaneously 2yrs ago. Her aunt has a similar undiagnosed rash on the extensor aspects of her elbows and
knees. What is the single most likely dx?
a. Eczema
b. Fungal infection
c. Impetigo
d. Lichen planus
e. Psoriasis

e. Psoriasis
HINT: Someone with psoriasis may have other family members with the same problem. Psoriasis is a skin
condition that tends to flare up from time to time
Psoriasis. Rash Always on extensors.
Eczema on flexors.
onset below age 2 years, flexural involvement, generally dry skin, other atopic disease
Impetigo 'golden', crusted skin lesions typically found around the mouth, very contagious
Lichen planus itchy, papular rash most common on the palms, soles, genitalia and flexor surfaces of arms,
'white-lace' pattern on the surface (Wickham's striae)

Psoriasis
presents with red, scaly patches on the skin although it is now recognised that patients with psoriasis are at
increased risk of arthritis and cardiovascular disease.
plaque psoriasis: the most common sub-type resulting in the typical well demarcated red, scaly patches
affecting the extensor surfaces, sacrum and scalp
flexural psoriasis: in contrast to plaque psoriasis the skin is smooth
guttate psoriasis: transient psoriatic rash frequently triggered by a streptococcal infection. Multiple red,
teardrop lesions appear on the body
pustular psoriasis: commonly occurs on the palms and soles

Management of chronic plaque psoriasis


regular emollients may help to reduce scale loss and reduce pruritus
first-line: NICE recommend a potent corticosteroid applied once daily plus vitamin D analogue applied
once daily (applied separately, one in the morning and the other in the evening) for up to 4 weeks as
initial treatment
second-line: if no improvement after 8 weeks then offer a vitamin D analogue twice daily
third-line: if no improvement after 8-12 weeks then offer either: a potent corticosteroid applied twice
daily for up to 4 weeks or a coal tar preparation applied once or twice daily
short-acting dithranol can also be used

556. A pt after transurethral prostatic biopsy. What electrolyte imbalance can he develop?
a. Hyperkalemia
b. Hyponatremia
c. Hypocalcemia
d. Hypernatremia
e. Hypercalcemia

b. Hyponatremia
Transurethral Resection of the Prostate (TURP) Syndrome is a rare but potentially life-threatening
complication of a transurethral resection of the prostate procedure. It occurs as a consequence of the
absorption into the prostatic venous sinuses of the fluids used to irrigate the bladder during the operation.
Symptoms and signs are varied and unpredictable, and result from fluid overload and disturbed electrolyte
balance and hyponatraemia. Treatment is largely supportive and relies on removal of the underlying cause,
and organ and physiological support. Preoperative prevention strategies are extremely important.

557. A 28yo woman has been admitted at 38wks gestation. Her BP=190/120mmHg and proteinuria +++.
Immediately following admission she has a grand-mal seizure. What is the single most
appropriate initial management?
a. Diazepam IV
b. Fetal CTG
c. Hydralazine IV
d. Immediate delivery
e. Magnesium sulphate IV

e. Magnesium sulphate IV
Eclampsia is defined as the occurrence of one or more convulsions superimposed on pre-eclampsia.

Management of eclampsia
Resuscitation
Treatment and prophylaxis of seizures:
o Magnesium sulfate is the anticonvulsant drug of choice.
o Intubation may become necessary in women with repeated seizures
Treatment of hypertension:
o Reduction of severe hypertension (blood pressure >160/110 mm Hg or mean arterial
pressure >125 mm Hg) is essential to reduce the risk of cerebrovascular accident.
Treatment may also reduce the risk of further seizures.
o Intravenous hydralazine or labetalol are the two most commonly used drugs. Both may
precipitate fetal distress and therefore continuous fetal heart rate monitoring is
necessary.
Fluid therapy:
o Close monitoring of fluid intake and urine output is mandatory.
Delivery:
o The definitive treatment of eclampsia is delivery.
o However, it is unsafe to deliver the baby of an unstable mother even if there is fetal
distress. Once seizures are controlled, severe hypertension treated and hypoxia
corrected, delivery can be expedited.
o Vaginal delivery should be considered but Caesarean section is likely to be required in
primigravidae, well before term and with an unfavourable cervix.
o After delivery, high-dependency care should be continued for a minimum of 24 hours

558. A 27yo woman had pre-eclampsia and was delivered by C-section. She is now complaining of RUQ pain
different from wound pain. What inv will you do immediately?
a. Coagulation profile
b. LFT
c. Liver US
d. MRCP
e. None

b. LFT

HELLP syndrome

Presentation
HELLP syndrome is a serious form of pre-eclampsia and patients may present at any time in the last
half of pregnancy.
One third of women with HELLP syndrome present shortly after delivery.
Initially, women may report nonspecific symptoms including malaise, fatigue, right upper quadrant or
epigastric pain, nausea, vomiting, or flu-like symptoms.
Hepatomegaly can occur.
Some women may have easy bruising/purpura.
On examination, oedema, hypertension and proteinuria are present.
Tenderness over the liver can occur.

Investigations
There needs to be a high index of clinical suspicion in order to avoid diagnostic delay and improve
outcome.
Haemolysis with fragmented red cells on the blood film
Raised LDH >600 IU/L with a raised bilirubin.
Liver enzymes are raised with an AST or ALT level of >70 IU/L.

Definitive treatment of HELLP syndrome requires delivery of the fetus and is advised after 34 weeks of
gestation if multisystem disease is present.

559. A 10yo girl has been referred for assessment of hearing as she is finding difficulty in hearing her teacher
in the class. Her hearing tests show: BC normal, symmetrical AC threshold reduced
bilaterally, weber test shows no lateralization. What is the single most likely dx?
a. Chronic perforation of tympanic membrane
b. Chronic secretory OM with effusion
c. Congenital sensorineural deficit
d. Otosclerosis
e. Presbycusis

b. Chronic secretory OM with effusion


B/L conductive deafness
glue ear/ OM e effusion
Bc normal means no sn deafness .. there is conductive deafness .. otosclerosis has cd but it usually appears in
3rd decade of life n associated with tinnitus .. perforation on both sides is uncommon .. so we're left with
csom with effusion which is most common cause of cd in school going age
Glue Ear/ otitis media with effusion: recurrent ear infections, poor speech development, and failing
performances at school, typically in children between the ages of 2 and decreasing with advancement of age..
.Causes conductive hearing loss.
The clincher also is 'child finding difficulty in hearing in classroom/turning up the volume of Tv'

Chronic suppurative otitis media (CSOM) is a chronic inflammation of the middle ear and mastoid cavity.
Clinical features are recurrent otorrhoea through a tympanic perforation, with conductive hearing loss of
varying severity.

CSOM presents with a chronically draining ear (>2 weeks), with a possible history of recurrent AOM,
traumatic perforation, or insertion of grommets.
The otorrhea should occur without otalgia or fever.
Fever, vertigo and otalgia should prompt urgent referral to exclude intratemporal or intracranial
complications.
Hearing loss is common in the affected ear

Treatment options include:


grommet insertion - to allow air to pass through into the middle ear and hence do the job normally
done by the Eustachian tube. The majority stop functioning after about 10 months
adenoidectomy

560. A thin 18yo girl has bilateral parotid swelling with thickened calluses on the dorsum of her hand. What is
the single most likely dx?
a. Bulimia nervosa
b. C1 esterase deficiency
c. Crohns disease
d. Mumps
e. Sarcoidosis

a. Bulimia nervosa
Clincher for a is calluses over dorsum; (chronic inducing vomiting) ,parotid swelling
Thickened calluses at back of hand (Russel's sign -tooth mark on finger for induced vomiting) +parotid
enlargment Bulimia
C1 esterase deficiency autosomal dominant condition associated with low plasma levels of the C1 inhibitor
(C1-INH) protein. attacks may be proceeded by painful macular rash
painless, non-pruritic swelling of subcutaneous/submucosal tissues
may affect upper airways, skin or abdominal organs (can occasionally present as abdominal pain due to
visceral oedema)
Mumps can be asymptomatic.fever, headache, malaise, myalgia and anorexia, can precede parotitis.Parotitis
is usually bilateral although it can be unilateral.

Sarcoidosis
acute: erythema nodosum, bilateral hilar lymphadenopathy, swinging fever, polyarthralgia
insidious: dyspnoea, non-productive cough, malaise, weight loss
skin: lupus pernio(chronic raised hardened, often purple lesion)
hypercalcaemia
Heerfordt's syndrome (inflammation of submaxillary/parotid glands with uveitis and facial nerve
palsy) may accompany constitutional presentation.

Bulimia nervosa
Presentation
The history often dates back to adolescence.
o Regular binge eating.
o Attempts to counteract the binges - eg, vomiting, using laxatives, diuretics, dietary restriction
and excessive exercise.
o Preoccupation with weight, body shape, and body image.
o low self-esteem, and self-harm.
o Periods may be irregular.
Physical examination is usually normal and is mainly aimed at excluding medical complications such as
dehydration or dysrhythmias (induced by hypokalaemia).
o Examination must include height and weight (and calculation of the BMI) and blood pressure.
o Salivary glands (especially the parotid) may be swollen.
o There may be oedema if there has been laxative or diuretic abuse.
o Russell's sign may be present (calluses form on the back of the hand, caused by repeated
abrasion against teeth during inducement of vomiting).

561. A 48yo presents with severe chest pain since the last 40mins. In the ED he is given oxygen, GTN,
morphine. ECG=ST elevation. Bloods=increased troponin levels. What is the next step of
management?
a. Beta blockers
b. Percutaneous angiography
c. Anticoagulant & heparin
d. Clopidogrel
e. Aspirin

b. Percutaneous angiography
Technically speaking trops are raised which means 2-3 hours have passed. PCI is indicated within 120 mins.
It should be E
PCI is more appropriate as the clinch is the time 40 mins that's why i went for B.
I would go for B
In case of ST elevation MI.. mx include aspirin at first usually given by GP or paramedic then morphine with
metoclopramide. GTN not used routinely except in case of HTN or severe LVF. Next step is PCI if available
within 120mins of hospital contact. If not available .fibrinolysis done if no CI and later proceed for PCI.
As here time frame of 4o mins mentioned..i guess its PCI..B
Its B. ST elevation MI means that the thrombus clot has already been stabilized and occluded the vessel...that
is why we give t-PA (thrombolytic) in ST elevation MI only... aim is to break down the thrombus, not stop it
from forming...aspirin only stops it from forming it doesn't break it down... smile emoticon also raised
troponin levels mean that the patient has a previous infarct at most 10 days before (since it raised in only 40
minutes, not 4 hours), having previous infarct means has 2 or 3 vessel disease and not 1 vessel disease, he is
high risk patient and should have angioplasty as soon as possible...answer is B, angiography is done right
before angioplasty

Acute Myocardial Infarction

Pre-hospital management
first line management is MONA (Morphine, O2, Nitrates and Aspirin)
Sublingual glyceryl trinitrate and intravenous morphine + metoclopramide should be given to help relieve the
symptoms.
Aspirin 300mg should be given to all patients (unless contraindicated)
Pre-hospital thrombolysis is indicated if the time from the initial call to arrival at hospital is likely to be over 30
minutes.

Primary percutaneous coronary intervention (PCI)


Door (or diagnosis) to treatment time should be less than 90 minutes, or less than 60 minutes if the hospital is
PCI ready and symptoms started within 120 minutes
If they cannot be transferred to a larger hospital for PCI within 120 minutes then fibrinolysis should be given. If
the patient's ECG taken 90 minutes after fibrinolysis failed to show resolution of the ST elevation then they
would then require transfer for PCI.
PCI should be considered if there is an ST elevation acute coronary syndrome, if symptoms started up to 12
hours previously

562. A 34yo female presents with a foul smelling discharge. What set of organisms are we looking for to be
treated here?
a. Chlamydia, gonorrhea
b. Chlamydia, gardenella
c. Chlamydia, gonorrhea, gardenella
d. Gonorrhea, gardenella
e. Gardenella only

e. Gardenella only
Chlamydia is usually asymptomatic (no odour) and generally goes with gonorrhea. BV will give the grey fish-
smelling discharge
Bacterial vaginosis and Trichomonas vaginalis give foul smelling discharge. In BV its grey white fishy and in TV
it can be greenish frothy fihy alongwith vulvovginitis i-e strawberry cervix. The discharge of Chlamydia and
Gonorrhea is not foul smelling but gives dysuria.

Bacterial vaginosis (BV) describes an overgrowth of predominately anaerobic organisms such as Gardnerella
vaginalis.
Amsel's criteria for diagnosis of BV - 3 of the following 4 points should be present
thin, white homogenous discharge
clue cells on microscopy: stippled vaginal epithelial cells
vaginal pH > 4.5
positive whiff test (addition of potassium hydroxide results in fishy odour)

Management
oral metronidazole for 5-7 days

563. A 6wk formula fed baby boy is found at the child health surveillance to be deeply jaundiced. His weight
gain is poor and his stools are pale. What is the most likely dx?
a. Galactosemia
b. Biliary atresia
c. G6PD deficiency
d. Rh incompatibility
e. Congenital viral infection

b. Biliary atresia
pale stools , dark urine- biliary atresia
pale stools, straw urine - galactosemia
galactosemia presents with vomitting,diarrhea,failure to thrive and jaundice...progressive n deep jaundice is
biliary atresia

Biliary atresia presents shortly after birth, with persistent jaundice, pale stools and dark urine. All term infants
who remain jaundiced after 14 days (and preterm infants after 21 days) should be investigated.

Galactosemia There is often feeding difficulty, with vomiting and failure to gain weight, with poor growth in
the first few weeks of life.

G6PD deficiency
neonatal jaundice is often seen
intravascular haemolysis
gallstones are common
splenomegaly may be present
Heinz bodies on blood films

564. A 45yo man with colon cancer now develops increased thirst, increased frequency in urination and
weight loss. His fasting blood glucose=9mmol/L. what is the most appropriate management?
a. Oral hypoglycemic
b. Insulin long acting
c. Short acting insulin before meal
d. IV insulin
e. Subcutaneous insulin

a. Oral hypoglycemic
colon cancer is assoc with hyperinsulinemia or insulin resistance..so oral hypoglycemics preferred
because oral hypoglycemic (metformin) has anticancerogenic effect.
A.. first line treatment dont get confused by colon cancer..
Metformin is the first drug of choice for the management of type 2 diabetes. It has two main antidiabetic
mechanisms of action, both of which have also been implicated as anticarcinogenic mechanisms. Firstly,
metformin inhibits hepatic glucose production through an LKB1/AMP-activated protein kinasemediated
mechanism which has been shown to adversely affect the survival of cancer cell lines. Secondly, metformin
improves insulin sensitivity in peripheral tissues reducing hyperinsulinemia. Insulin resistance and
hyperinsulinemia have been associated with increased risk of several types of neoplasm and specifically with
colorectal cancer.

565. A 34yo man from Zimbabwe is admitted with abdominal pain to the ED. An AXR reveals bladder
calcification. What is the most likely cause?
a. Schistosoma mansoni
b. Sarcoidosis
c. Leishmaniasis
d. TB
e. Schistosoma haematobium

e. Schistosoma haematobium
Schistosoma Hematobium (Bilhaarziasis). CA urinary bladder and vesicolithiasis are the two main concern here

S. haematobium causes urinary schistosomiasis, and is the most prevalent and widespread species in Africa
and the Middle East.
Schistosomiasis is associated with anaemia, chronic pain, diarrhoea, exercise intolerance, and malnutrition.
The first sign may be swimmer's itch
Fever.
Hepatosplenomegaly.
Right upper quadrant pain or tenderness.
Urticaria may be seen occasionally.
Lymphadenopathy.

Praziquantel is the drug of choice


Oxamniquine is the only alternative

Complications:
renal stones
increased risk of squamous cell carcinoma of bladder that has been noticed especially in Egypt. It is
possible that the infestation and the carcinogens in tobacco smoke have a synergistic effect.
Hydronephrosis
renal failure may occur
iron-deficiency anaemia
Portal hypertension

566. A 6yo came with full thickness burn. He is crying continuously. What is the next step of
management?
a. Refer to burn unit
b. IV fluid stat
c. Antibiotic
d. Analgesia
e. Dressing

d. Analgesia
In NHS,, making comfortable to patient is vital. Here question ask for initial management, hence analgesia is
the most here then after treat accordingly, either refer to burn unit or give if fluids using parklands.
D. Then iv fluids then refer to burn unit.

Referral to secondary care


all deep dermal and full-thickness burns.
superficial dermal burns of more than 10% TBSA in adults, or more than 5% TBSA in children
superficial dermal burns involving the face, hands, feet, perineum, genitalia, or any flexure, or
circumferential burns of the limbs, torso, or neck
any inhalation injury
any electrical or chemical burn injury
suspicion of non-accidental injury

Management of burns
initial first aid as above
review referral criteria to ensure can be managed in primary care
superficial epidermal: symptomatic relief - analgesia, emollients etc
superficial dermal: cleanse wound, leave blister intact, non-adherent dressing, avoid topical creams,
review in 24 hours

567. A 78yo nursing home resident is revived due to the development of an intensely itchy rash.
Exam: white linear lesions are seen on the wrists and elbows and red papules are present on the
penis. What is the most appropriate management?
a. Topical permethrin
b. Referral to GUM clinic
c. Topical betnovate
d. Topical ketoconazole
e. Topical selenium sulfide hyoscine

a. Topical permethrin
Red papule on penis typical with wrist and elbow lesion goes with Scabies, topical permethrin once wk and
repeat if symptoms remain.
white linear lesions

Features
widespread pruritus
linear burrows on the side of fingers, interdigital webs and flexor aspects of the wrist
Nodules may develop. These occur particularly at the elbows, anterior axillary folds, penis, and
scrotum.
in infants the face and scalp may also be affected
secondary features are seen due to scratching: excoriation, infection
Management
permethrin 5% is first-line
malathion 0.5% is second-line
give appropriate guidance on use
pruritus persists for up to 4-6 weeks post eradication

568. A 4yo has earache and fever. Has taken paracetamol several times. Now its noticed that he
increases the TV volume. His preschool hearing test shows symmetric loss of 40db. What is the
most likely dx?
a. OM with effusion
b. Otitis externa
c. Cholesteatoma
d. CSOM
e. Tonsillitis

a. OM with effusion
see Q. 559

569. A pt presents with gradual onset of headache, neck stiffness, photophobia and fluctuating LOC. CSF
shows lymphocytosis but no organism on gram stain. CT brain is normal. What is the single most likely dx?
a. Hairy leukoplakia
b. TB
c. CMV infection
d. Candida infection
e. Cryptococcal infection
b. TB
TB as there is lymphocytosis and no organism on gram staining Zn staining or AFB can detect mycobacterium
TB

Viral meningitis may be clinically indistinguishable from bacterial meningitis but features may be more mild
and complications (eg, focal neurological deficits) less frequent. Any person presenting with suspected
meningitis should therefore be managed as having bacterial meningitis until proved otherwise.
classic triad of fever, neck stiffness and a change in mental status was present in only 44% of adults presenting
with community-acquired acute bacterial meningitis. However, 95% had at least two of the four symptoms of
headache, fever, neck stiffness and altered mental status.
Most patients with viral meningitis present with subacute neurological symptoms developing over 1-7 days.
Chronic symptoms lasting longer than one week suggest meningitis caused by some viruses as well as TB,
syphilis or fungi.

Bacterial Viral Tuberculous

Appearance Cloudy Clear/cloudy Slight cloudy, fibrin web

Glucose Low (< 1/2 plasma) 60-80% of plasma glucose* Low (< 1/2 plasma)
Protein High (> 1 g/l) Normal/raised High (> 1 g/l)

White cells 10 - 5,000 polymorphs/mm 15 - 1,000 lymphocytes/mm 10 - 1,000 lymphocytes/mm

570. An 18m boy has been brought to the ED because he has been refusing to move his left arm and crying
more than usual for the past 24h. He has recently been looked after by his mothers new bf while she
attended college. Assessment shows multiple bruises and a fx of the left humerus
which is put in plaster. What is the single most appropriate next step?
a. Admit under care of pediatrician
b. Discharge with painkillers
c. Follow up in fx clinic
d. Follow up in pediatric OPD
e. Follow up with GP

a. Admit under care of pediatrician


Non accidental injury
The most common manifestation of abuse is bruising
An estimated 15-25 % of pediatric burns are the result of abuse.
Fractures are the second most common manifestation of child abuse after soft tissue injuries.
Any fracture in a young child should be concerning, especially if the child is not ambulating.
Abusive head trauma, also known as shaken baby syndrome, is the most common cause of child abuse death,
usually occurring during the first year of life.

571. A 74yo female presents with headache and neck stiffness to the ED. Following a LP the pt was started on
IV ceftriaxone. CSF culture = listeria monocytogenes. What is the appropriate tx?
a. Add IV amoxicillin
b. Change to IV amoxicillin + gentamicin
c. Add IV ciprofloxacin
d. Add IV co-amoxiclav
e. Continue IV ceftriaxone as mono-therapy

b. Change to IV amoxicillin + gentamicin


Meningitis caused by meningococci
Intravenous ceftriaxone for at least seven days is usually used..
Prevention of secondary case of meningococcal meningitis is usually with rifampicin or ciprofloxacin.
Meningitis caused by pneumococci
Vancomycin and a third-generation cephalosporin (either cefotaxime or ceftriaxone)
Benzylpenicillin may be given if the organism is penicillin-sensitive but penicillin resistance is becoming
an increasing problem.
Meningitis caused by H. influenzae type b
Children aged 3 months and older and young people - intravenous ceftriaxone for 10 days.
Meningitis caused by group B streptococci
This mainly occurs in babies between the ages of 7-90 days. Intravenous cefotaxime for at least 14 days
should be given.
Meningitis caused by listeriosis
For children under the age of 3 months, intravenous amoxicillin or ampicillin for 21 days in total, plus
gentamicin for at least the first seven days.

572. A pt presents with fever, dry cough and breathlessness. He is tachypneic but chest is clear.
Oxygen saturation is normal at rest but drops on exercise. What is the single most likely dx?
a. CMV infection
b. Candida infection
c. Pneumocystis carinii infection
d. Cryptococcal infection
e. Toxoplasma abscess

c. Pneumocystis carinii infection


This kind of history about oxygen desaturation on exercise is typical for PCP. Dry cough along with that
supports that.
patient is immunocompromised. Mostly in HIV patients we see that pt becomes breathless after a walk or
exercise. Also fever with dry cough is there. Most likely pathogen is PCP.
CMV affects retina n brain
Toxoplasmosis..brain
Candida. .mouth and esophagus
Cryptococcus..meningitis

Whilst the organism Pneumocystis carinii is now referred to as Pneumocystis jiroveci, the term Pneumocystis
carinii pneumonia (PCP) is still in common use

PCP is the most common opportunistic infection in AIDS


all patients with a CD4 count < 200/mm should receive PCP prophylaxis

Features:
dyspnoea
dry cough
fever
very few chest signs

Pneumothorax is a common complication of PCP.

Extrapulmonary manifestations are rare (1-2% of cases), may cause


hepatosplenomegaly
lymphadenopathy
choroid lesions

Investigation
CXR: typically shows bilateral interstitial pulmonary infiltrates but can present with other x-ray findings e.g.
lobar consolidation. May be normal
exercise-induced desaturation
sputum often fails to show PCP, bronchoalveolar lavage (BAL) often needed to demonstrate PCP (silver stain
shows characteristic cysts)
Management
co-trimoxazole
IV pentamidine in severe cases
steroids if hypoxic (if pO2 < 9.3kPa then steroids reduce risk of respiratory failure by 50% and death by a third)

573. A 14yo boy fell and hit his head in the playground school. He didnt lose consciousness. He has
swelling and tenderness of the right cheek with a subconjuctival hemorrhage on his right eye.
What is the most appropriate initial inv?
a. CT brain
b. EEG
c. MRI
d. Skull XR
e. Facial XR

e. Facial XR
there's no indication of CT scan here- he is conscious and has not vomited

The difference between Skull and facial x ray is view. Skull PA view is done in prone position for seeing Skull
bones. Facial is simply reverse i.e AP view done in supine position and gives more clear view of facial bones.
Suspected injury to facial bones is a CONTRAINDICATION for PA view as patient cannot be asked to lie down in
prone position/ or to lean forward with face down.
A facial or sinus X-ray may be done to:
Find problems of the sinuses of the face and nose, such as sinusitisor abnormal growths (polyps or
tumors).
Find fractures of the facial bones and nose.
Check the bones around the eye (orbital cavity).
Check the sinuses before surgery.
Check for metal objects around the eyes before a magnetic resonance imaging (MRI) test.
Look for the cause of pain in the face.

574. A 15m child is due for his MMR vaccine. There is a fam hx of egg allergy. He is febrile with acute OM.
What is the single most appropriate action?
a. Defer immunization for 2wks
b. Dont give vaccine
c. Give half dose of vaccine
d. Give paracetamol with future doses of the same vaccine
e. Proceed with standard immunization schedule

a. Defer immunization for 2wks


egg allergy is not contraindication for MMR ...therefore if pt is febrile then wait for the next two weeks until
he is afebrile and give the normal dose of immunization
Children in the UK receive two doses of the Measles, Mumps and Rubella (MMR) vaccine before entry to
primary school. This currently occurs at 12-15 months and 3-4 years as part of the routine immunisation
schedule
Contraindications to MMR
severe immunosuppression
Acute illness
allergy to neomycin
children who have received another live vaccine by injection within 4 weeks
pregnancy should be avoided for at least 1 month following vaccination
immunoglobulin therapy within the past 3 months (there may be no immune response to the measles
vaccine if antibodies are present)

Adverse effects
malaise, fever and rash may occur after the first dose of MMR. This typically occurs after 5-10 days and
lasts around 2-3 days

Note that the following are NOT contra-indications:


Family history of any adverse reactions following immunisation.
Previous history of infection with pertussis, measles, rubella or mumps.
Contact with an infectious disease.
Asthma, eczema, hay fever or rhinitis.
Treatment with antibiotics or locally acting (eg, topical or inhaled) steroids.
The child's mother being pregnant.
The child being breast-fed.
History of jaundice after birth.
Being over the age recommended in the immunisation schedule.
'Replacement' corticosteroids.
Allergy to eggs
Neurological conditions are not a contra-indication although, if the condition is poorly controlled (eg,
epilepsy), immunisation should be deferred.
MMR should ideally be given at the same time as other live vaccines, such as BCG. However, if live
vaccines cannot be administered simultaneously, a four-week interval is recommended.

575. A 33yo lady with Hodgkins lymphoma presents with temp=40C, left sided abdominal pain and
lymphadenitis. Blood was taken for test. What will you do next?
a. Wait for blood test
b. Start broad spectrum IV antibiotics
c. Oral antibiotics
d. CBC
e. Monitor pyrexia

b. Start broad spectrum IV antibiotics


the patient has an immune compromising disease ,you cant wait until you get lab results or give oral
antibiotics, you shuld give systemic antibiotic to treat any possible infectons
Chemotherapy causes imunosuppresion so increased chance of infections,as in this case temp 40,and
lymphadenitis so broad spectrum antibiotics

576. A 40yo man with marked weight loss over the preceding 6m has bilateral white, vertically
corrugated lesion on the lateral surfaces of the tongue. What is the single most likely dx?
a. C1 esterase deficiency
b. Crohns disease
c. HIV disease
d. Sarcoidosis
e. Sjogrens syndrome

c. HIV disease

'Hairy' leukoplakia
This is associated with Epstein-Barr virus (EBV) and occurs mostly in people with HIV, both
immunocompromised and immunocompetent.
The natural history of hairy leukoplakia is variable. Lesions may frequently appear and disappear
spontaneously. Hairy leukoplakia is often asymptomatic and many patients are unaware of its presence. Some
patients with hairy leukoplakia do experience symptoms including mild pain, dysaesthesia, alteration of taste
and the psychological impact of its unsightly cosmetic appearance.
Systemic antiviral therapy, which usually achieves resolution of the lesion within 1-2 weeks of therapy.
Topical therapy with podophyllin resin 25% solution, which usually achieves resolution after 1-2
treatment applications.
Topical therapy with retinoic acid (tretinoin), which has been reported to resolve hairy leukoplakia.
Ablative therapy, which can also be considered for small hairy leukoplakia lesions. Cryotherapy has
been reported as successful but is not widely used.

577. A 3m baby was miserable and cried for 2h following his 1st routine immunization with DTP, HiB and
meningitis. What is the single most appropriate action?
a. Defer immunization for 2wks
b. Dont give vaccine
c. Give half dose of vaccine
d. Give paracetamol with future doses of the same vaccine
e. Proceed with standard immunization schedule

e. Proceed with standard immunization schedule

General contraindications to immunisation


confirmed anaphylactic reaction to a previous dose of a vaccine containing the same antigens
confirmed anaphylactic reaction to another component contained in the relevant vaccine (e.g. egg
protein)

Situations where vaccines should be delayed


febrile illness/intercurrent infection

Contraindications to live vaccines


pregnancy
immunosuppression

Specific vaccines
DTP: vaccination should be deferred in children with an evolving or unstable neurological condition

Not contraindications to immunisation


asthma or eczema
history of seizures (if associated with fever then advice should be given regarding antipyretics)
breastfed child
previous history of natural pertussis, measles, mumps or rubella infection
history of neonatal jaundice
family history of autism
neurological conditions such as Down's or cerebral palsy
low birth weight or prematurity
patients on replacement steroids e.g. (CAH)

578. A 65yo man with HTN develops gingival hyperplasia. What is the single most likely dx?
a. ACEi
b. Beta blockers
c. Crohns disease
d. Nifedipine
e. Sarcoidosis

d. Nifedipine
Side effect of CCB
also due to cyclosporin, phenytoin, AML.

579. A 65yo woman is undergoing coronary angiography. What measure will protect her kidneys
from contrast?
a. Furosemide
b. Dextrose
c. 0.45% saline
d. 0.9% saline

d. 0.9% saline
post contrast nephropathy due to contrast induced or cholesterol embolisation. adequately hydrated pt prior
to procedure reduces the complications.

580. An 83yo woman who is a resident in a nursing home is admitted to hospital with a 4d hx of
diarrhea. She has had no weight loss or change in appetite. She has been on analgesics for 3wks
for her back pain. She is in obvious discomfort. On rectal exam: fecal impaction. What is the
single most appropriate immediate management?
a. Codeine phosphate for pain relief
b. High fiber diet
c. Oral laxative
d. Phosphate enema
e. Urinary catheterization

d. Phosphate enema
Codiene
Laxative
Fiber will increase gut motility
Where as
Phosphate enema will act locally
Helpful in clearing
Fecal impaction too

Bulk producers:
Increase faecal mass, which stimulates peristalsis.
They must be taken with plenty of fluid
Contra-indications: difficulty in swallowing; intestinal obstruction; colonic atony; faecal impaction.
Stool softeners:
Side-effects can include: anal seepage, lipoid pneumonia, malabsorption of fat-soluble vitamins
Stimulants:
Increase intestinal motility and should not be used in intestinal obstruction.
Prolonged use should be avoided, as it may cause colonic atony and hypokalaemia (but there are no
good, long-term follow-up studies).
Osmotic agents:
Retain fluid in the bowel.
Enemas and suppositories - useful additional treatment.

581. A 26yo woman being treated for a carcinoma of the bronchus with steroids presents with
vomiting, abdominal pain and sudden falls in the morning. What is the most specific cause for
her symptoms?
a. Steroid side effects
b. Postural hypotension
c. Adrenal insufficiency
d. Conns disease
e. Cushings disease

c. Adrenal insufficiency

Streoids causing suppression of acth. In turn causing mineralcorticoid deficiency so adrenal insufficiency
high dose sterods suppresss adrenals...cause hyponatraemia..hypotension..hypoglycaemia..hyperkalaemia
Exogenous steroids can suppress the pituitary adrenal axis leading to adrenal insufficiency. Symptoms include
weakness, anorexia, dizzy,
Faints,nausea,vomiting,abd pain
Steroid does not cause vomit or falls so A can be excluded.
Postural hypotension does not cause abd. Pain so B excluded .
Conn's syndrome is hyperaldosteronism which would cause hypernatremia and hypokalemia and
hypertension.
So D excluded
Cushing causes hypertension so E Excluded
C is the right answer cuz adrenal insufficiency due to prolonged steroid intake would cause addison
syndrome which is hyponatremia hypotension abdominal pain .

582. A 78yo woman presents with unilateral headache and pain on chewing. ESR=70mm/hr. She is on oral
steroids. What is the appropriate additional tx?
a. Bisphosphonates
b. HRT
c. ACEi
d. IFN
e. IV steroids

a. Bisphosphonates
She getting treated for temporal arteritis, therefore steroid will cause osteoporosis. So additional therapy is A
It appears GCA. We first do ESR and start steroids. If symptoms are not resolved then we up the dose of
steroids.
Additional treatment would be bisphosphonate to reduce risk of osteoporosis.

Bisphosphonates
Bisphosphonates decrease demineralisation in bone. They inhibit osteoclasts.

Clinical uses
prevention and treatment of osteoporosis
hypercalcaemia
Paget's disease
pain from bone metatases

Adverse effects
oesophagitis, oesophageal ulcers
osteonecrosis of the jaw
increased risk of atypical stress fractures of the proximal femoral shaft.

583. A 30yo man is suffering from fever, rash and photophobia. Doctors are suspecting he is suffering from
meningitis. Which is the best medication for this condition?
a. Ampicilling
b. Cefotaxime
c. Tetracycline
d. Acyclovir
e. Dexamethasone

b. Cefotaxime

Initial 'blind' therapy


Children 3 months and older and young people should be given intravenous ceftriaxone as empirical
treatment before identification of the causative organism. If calcium-containing infusions are required
at the same time, cefotaxime is preferable.

584. A 15yo girl was admitted with anemia, chest infection and thrombocytopenia. She was treated and her
symptoms had regressed. She was brought again with fever and the same symptoms a few days later. She also
seems to have features of meningitis. What is the most likely dx?
a. AML
b. ALL
c. Aplastic anemia
d. CML
e. CLL
b. ALL
Young, anemia, thrombocytopenia, recurrent infectionswith/without cns involvement and testicular swelling...
always go for All..if not treated completed can appear again...in aplastic anemia the cell count of all cell types
is low with a mention of some predisposing factor..like drugs, radiation or a dry tap of bone marrow.
Patients with ALL frequently have meningeal leukaemia at the time of relapse (50-75% at one year in the
absence of CNS prophylaxis) and a few have meningeal disease at diagnosis (<10%).

ALL
Symptoms
Fatigue, dizziness and palpitations
bone and joint pain
Recurrent and severe infections (oral, throat, skin, perianal infections commonly)
Fever
Left upper quadrant fullness and early satiety due to splenomegaly (10-20%)
Haemorrhagic or thrombotic complications due to thrombocytopenia or disseminated intravascular
coagulopathy (DIC) - for example, menorrhagia, frequent nosebleeds, spontaneous bruising
Signs
Pallor
Tachycardia and a flow murmur
Nonspecific signs of infection
Petechiae,purpura or ecchymoses
hepatosplenomegaly
Lymphadenopathy
Gum hypertrophy
Cranial nerve palsy (especially III, IV, VI and VIII) in mature B-cell ALL

585. A pt was admitted to the ED after a head injury. When examined on arrival his GCS=15 and then at night
his GCS deteriorated to 12. What investigation should be done?
a. CT head
b. XR skull
c. IV mannitol
d. Drill a burr hole
e. Shift to OR

a. CT head

In patients with normal or near-normal GCS and who are alert


Haemodynamic status - pulse rate, blood pressure, fluid status.
Neurological assessment - full history and examination, make notes of pupil size and reaction to light.
Look for other possible injuries and any other relevant examination.

In patients with reduced GCS


Resuscitate but make a quick assessment of GCS and pupils. The priority is to get the patient to
hospital and CT scanned within the first hour after injury.
Selection of adults for CT scan

CT scan of the brain within one hour (with a written radiology report within one hour of the scan being
undertaken):
Glasgow Coma Scale (GCS) <13 when first assessed or GCS <15 two hours after injury
Suspected open or depressed skull fracture
Signs of base of skull fracture*
Post-traumatic seizure
Focal neurological deficit
>1 episode of vomiting
All patients with a coagulopathy or on oral anticoagulants should have a CT brain scan within eight hours
of the injury, provided there are no other identified risk factors, as listed above.

586. A 4yo boy who prv had normal hearing, has a mild earache relieved by paracetamol. He has
been noticed to turn up the vol on the TV. He has bilateral dull tympanic membranes. His preschool hearing
test shows symmetrical loss of 40dB. What is the single most likely dx?
a. Acute otitis externa
b. Acute OM
c. Ear wax
d. Foreign body
e. OM with effusion

e. OM with effusion
Glue Ear/ otitis media with effusion: recurrent ear infections, poor speech development, and failing
performances at school, typically in children between the ages of 2 and decreasing with advancement of age..
.Causes conductive hearing loss.
The clincher also is 'child finding difficulty in hearing in classroom/turning up the volume of Tv'

Chronic suppurative otitis media (CSOM) is a chronic inflammation of the middle ear and mastoid cavity.
Clinical features are recurrent otorrhoea through a tympanic perforation, with conductive hearing loss of
varying severity.

CSOM presents with a chronically draining ear (>2 weeks), with a possible history of recurrent AOM,
traumatic perforation, or insertion of grommets.
The otorrhea should occur without otalgia or fever.
Fever, vertigo and otalgia should prompt urgent referral to exclude intratemporal or intracranial
complications.
Hearing loss is common in the affected ear

Treatment options include:


grommet insertion - to allow air to pass through into the middle ear and hence do the job normally
done by the Eustachian tube. The majority stop functioning after about 10 months
adenoidectomy
587. An 18yo man presents to his GP with thirst and polyuria. Some 6m ago he had a significant head injury as
the result of a RTA. He is referred to the local endocrine clinic. Which of the following results would be the
most useful in confirming the dx of diabetes insipidus after a water
deprivation test (without additional desmopressin)?
a. Plasma sodium of 126mmol/l
b. Plasma sodium of 150mmol/l
c. Plasma osmolality of 335mosmol/kg and urine osmolality of 700mosmol/kg
d. Plasma osmolality of 280mosmol/kg and urine osmolality of 700mosmol/kg
e. Plasma osmolality of 335mosmol/kg and urine osmolality of 200mosmol/kg

e. Plasma osmolality of 335mosmol/kg and urine osmolality of 200mosmol/kg


normal osmolality of plasma is somewhere around 285 to 295. Since water is being lost plasma will be more
concentrated hence osmolality more than 300. Urines normal osmolality is more than 600. Since urine is
dilute osmolality is much lower..so E is correct
In DI: urine osmolarity is low while plasma osmolarity is high.
In contrast, in SIADH, urine osmolarity is high while plasma osmolarity is low.

Classification of causes of diabetes insipidus on basis of water deprivation and DDAVP response

Urine osmolality after fluid Urine osmolality after DDAVP Likely diagnosis
deprivation (mOsm/kg) (mOsm/kg)
<300 >800 CDI
<300 <300 NDI
>800 >800 Primary/psychogenic polydipsia
<300 >800 Partial CDI or NDI or PP or
diuretic abuse

588. A 75yo man has left-sided earache and discomfort when he swallows. There is ulceration at the back of
his tongue and he has a palpable non-tender cervical mass. What is the single most likely dx?
a. Acute mastoiditis
b. Dental abscess
c. Herpes zoster infection
d. Oropharyngeal malignancy
e. Tonsillitis

d. Oropharyngeal malignancy
Nontender lymphadenopathy usu suggests neoplasia.And she has ulcer near to the root of the tongue.out of
the given option closest diagnosis seems to be orppharyngeal Carcinoma which is option D.

The symptoms of cancer of the pharynx differ according to the type:


Oropharynx: common symptoms are a persistent sore throat, a lump in the mouth or throat, pain in
the ear.
Hypopharynx: problems with swallowing and ear pain are common symptoms and hoarseness is not
uncommon.
Nasopharynx: most likely to cause a lump in the neck but may also cause nasal obstruction, deafness
and postnasal discharge.

589. A 42yo man has been tired and sleepy for the last few weeks in the morning. His work has starte getting
affected as he feels sleepy in the meetings. His BMI=36. What is the single most likely dx?
a. Idiopathic hypersomnia
b. Narcolepsy
c. Chest hyperventilation syndrome
d. OSAS
e. REM-related sleep disorder

d. OSAS
Obstructive sleep apnea syndrome
Risk factors include:
Obesity (strongest risk factor).
Male gender.
Middle age (55-59 in men, 60-64 in women).
Smoking.
Sedative drugs.
Excess alcohol consumption.

All patients with OSA causing excessive daytime or awake time sleepiness need to cease driving until
satisfactory control of symptoms has been attained.

Narcolepsy tetrad of classic symptoms: excessive daytime sleepiness (EDS), cataplexy, hypnagogic
hallucinations and sleep paralysis.

590. A 35yo pregnant woman has been having tingling and numbness of her thumb, index and
middle fingers for a while. She has been treated with local steroids but it hasnt helped her
much and now she has planned to undergo a surgical procedure. Which of the following
structures will be incised?
a. Flexor digitorum profundus
b. Transverse carpel ligament
c. Palmar aponeurosis
d. Extensor retinaculum

b. Transverse carpel ligament


carpal tunnel syndrome
in pregnancy, if carpal tunnel syndrome occurs: you first go for topical steroids.. then splint.. and then surgery:
and you incise the transverse carpal lig.
carpal tunnel, an anatomical compartment bounded by the bones of the carpus and the flexor retinaculum.

CTS is characterised by tingling, numbness, or pain in the distribution of the median nerve (the thumb, index,
and middle fingers, and medial half the ring finger on the palmar aspect) that is often worse at night and
causes wakening.
Positive Phalen's test: flexing the wrist for 60 seconds causes pain or paraesthesia in the median nerve
distribution.
Positive Tinel's sign: tapping lightly over the median nerve at the wrist causes a distal paraesthesia in
the median nerve distribution.
Positive carpal tunnel compression test: pressure over the proximal edge of the carpal ligament
(proximal wrist crease) with thumbs causes paraesthesia to develop or increase in the median nerve
distribution.

Electroneurography (ENG) - This is the gold standard investigation for CTS.

591. A 58yo pt presents with altered bowel habits and bleeding per rectum. Exam and sigmoidoscopy showed
an ulcer. What is the single most likely dx?
a. Colorectal carcinoma
b. Celiac disease
c. Crohn's disease
d. UC
e. IBS

a. Colorectal carcinoma
Older patient. Altered bowel habits and bleeding PR is Ca unless proven otherwise.
Alter bowl habbit+ bleeding are a red flag for CA colon>>>> colonoscopy >>>> one ulcer >>> CA. UC Usually
there is no alteration in bowel habit, usually boodly diarrhoea >>> colonoscopy >>>> multiple ulceration
third most common cancer in the UK
second most common cause of cancer death in the UK.

Presentation:
Right colon cancers: weight loss, anaemia, occult bleeding, mass in right iliac fossa, disease more likely
to be advanced at presentation.
Left colon cancers: often colicky pain, rectal bleeding, bowel obstruction, tenesmus, mass in left iliac
fossa, early change in bowel habit, less advanced disease at presentation.
The most common presenting symptoms and signs of cancer or large polyps are rectal bleeding,
persisting change in bowel habit and anaemia.
Colonoscopy is the gold standard for diagnosis of colorectal cancer.

592. A mother is concerned that her 18m son has a vocabulary of ten words but cant form a
sentence. What is the best management strategy?
a. Arrange hearing test
b. Assess developmental milestones
c. Reassurance
d. Refer to speech therapist
e. MRI brain

c. Reassurance

Age Milestone

3 months Quietens to parents voice


Turns towards sound
Squeals

6 months Double syllables 'adah', 'erleh'

9 months Says 'mama' and 'dada'


Understands 'no'

12 months Knows and responds to own name

12-15 months Knows about 2-6 words (Refer at 18 months)


Understands simple commands - 'give it to mummy'

2 years Combine two words


Points to parts of the body

2 years Vocabulary of 200 words

3 years Talks in short sentences (e.g. 3-5 words)


Asks 'what' and 'who' questions
Identifies colours
Counts to 10 (little appreciation of numbers though)

4 years Asks 'why', 'when' and 'how' questions

593. A 55yo man has weight loss, dyspnea and syncope. He smokes 20 cigarettes/day. Inv confirms squamous
cell carcinoma in the left bronchus. What is the single most likely biochemical
abnormality to be a/w the condition?
a. Hypercalcemia
b. Hyperkalemia
c. Hypoernatremia
d. Hypocalcemia
e. Hypomagnesium

a. Hypercalcemia
Squamous cell ca causes hypercalcemia...whereas small cell ca causes SIADH
paraneoplastic syndrome causing ectopic production of parathyroid hormone-related protein (PTHrP),
resulting in "hypercalcemia"

There are three main subtypes of non-small cell lung cancer:

Squamous cell cancer


typically central
associated with parathyroid hormone-related protein (PTHrP) secretion hypercalcaemia
strongly associated with finger clubbing

Adenocarcinoma
typically peripheral
most common type of lung cancer in non-smokers, although the majority of patients who develop lung
adenocarcinoma are smokers

Large cell lung carcinoma


typically peripheral
anaplastic, poorly differentiated tumours with a poor prognosis
may secrete -hCG

594. A 72yo man presents with intermittent difficulty in swallowing with regurgitation of stale food
materials. Sometimes he wakes up at night with a feeling of suffocation. Choose the single most
likely cause of dysphagia?
a. Benign structure
b. Esophageal carcinoma
c. Esophageal spasm
d. Pharyngeal pouch
e. Systemic sclerosis

d. Pharyngeal pouch

Pharyngeal More common in older men


pouch Represents a posteromedial herniation between thyropharyngeus and cricopharyngeus
muscles
Usually not seen but if large then a midline lump in the neck that gurgles on palpation
Typical symptoms are dysphagia, regurgitation, aspiration and chronic cough. Halitosis may
occasionally be seen

595. A 9m child is brought to the ED with an irreducible firm swelling which descended into the left
groin when the child has been crying. Exam: both testicles are palpable in the scrotum. What is
the most appropriate management strategy?
a. Reassurance
b. Emergency herniotomy
c. Elective herniotomy
d. Emergency herniotomy + orchidopexy
e. Elective herniotomy + orchidopexy

c. Elective herniotomy
As the swelling descended into the left scrotum when the child was crying so probably it is no more
irreducible
So elective herniotomy

strangulated hernia:
Examination reveals a firm lump in the groin of a crying child, which may extend into the scrotum. The child
may have vomited but is usually well.
Paediatric surgeons will undertake repair soon after diagnosis, regardless of age or weight, in healthy
full-term infant boys with asymptomatic reducible inguinal hernias. Emergency surgery is twenty times
more likely to cause complications than an elective procedure.
Immediate surgery is not always necessary in a case of strangulation: four out of five can be reduced
manually.
Tachycardia, fever or signs or obstructions are indications for surgery.

596. A 37yo woman was admitted for femur fx repair after a RTA. On the 4th post-op day she became
confused and starts picking on her bed sheets and complains of seeing spiders all over. What is the most likely
dx?
a. Delirium tremens
b. Wernickes encephalopathy
c. Korsakoffs psychosis
d. Psychotic depression

a. Delirium tremens
Delirium tremens usually begins 24-72 hours after alcohol consumption has been reduced or stopped.
The symptoms/signs differ from usual withdrawal symptoms in that there are signs of altered mental
status. These can include:
o Hallucinations (auditory, visual, or olfactory).
o Confusion.
o Delusions.
o Severe agitation.
Seizures can also occur.
Examination may reveal signs of chronic alcohol abuse/stigmata of chronic liver disease. There may
also be:
o Tachycardia.
o Hyperthermia and excessive sweating.
o Hypertension.
o Tachypnoea.
o Tremor.
o Mydriasis.
o Ataxia.
o Altered mental status.
o Cardiovascular collapse.

Withdrawal symptoms:
Symptoms typically present about eight hours after a significant fall in blood alcohol levels. They peak
on day 2 and, by day 4 or 5, the symptoms have usually improved significantly.
Minor withdrawal symptoms (can appear 6-12 hours after alcohol has stopped)
Alcoholic hallucinosis (can appear 12-24 hours after alcohol has stopped):[6]
o Includes visual, auditory or tactile hallucinations.
Withdrawal seizures (can appear 24-48 hours after alcohol has stopped):
o These are generalised tonic-clonic seizures.
Alcohol withdrawal delirium or 'delirium tremens' (can appear 48-72 hours after alcohol has stopped)
597. A 36yo pt came with diarrhea, bleeding, weight loss and fistula. What is the single most likely dx?
a. Celiac disease
b. Crohns disease
c. UC
d. IBS

b. Crohns disease
fistula due to transmural involvement, weight loss in crohns.

598. A 72yo woman who is taking look diuretics is suffering from palpitations and muscle weakness.
What is the electrolyte imbalance found?
a. Na+ 130mmol/l, K+ 2.5mmol/l
b. Na+ 130mmol/l, K+ 5.5mmol/l
c. Na+ 140mmol/l, K+ 4.5mmol/l
d. Na+ 150mmol/l, K+ 3.5mmol/l
e. None

a. Na+ 130mmol/l, K+ 2.5mmol/l


Hypokalaemia can occur with loop or thiazide diuretics.

599. A 60yo diabetic pt on anti-diabetic medication developed diarrhea. What is the most likely cause for his
diarrhea?
a. Autonomic neuropathy
b. Infective
c. Celiac disease
d. Crohns disease

a. Autonomic neuropathy
In general people no autonomic neuropathy usually. Once a diabetic develop autonomic neuropathy he gets
diarrhea off and on due to his autonomic neuropathy and it becomes the most comon cause for his diarrhea.
So most likely cause in this case would be Autonomic neuropathy.
Autonomic neuropathy{GI tract} causes dysfunctional motility / secretions /absorption. ..leads to
gasteroparesis(damage to vagus nerve ), ch: diarrhea(neuronal damage to small intestine)
,,/constipation(colon nerves damage )...
>diarrhea can be due to metformin] it has very common GI side effects ..

Autonomic neuropathy
Risk factors include hypertension and dyslipidaemia. It is more common in females.
May present with:
o Cardiac autonomic neuropathy, which has been linked to:[5]
Resting tachycardia, postural hypotension, orthostatic bradycardia and orthostatic
tachycardia.
Exercise intolerance.
Decreased hypoxia-induced respiratory drive.
Increased incidence of asymptomatic myocardial ischaemia, myocardial infarction,
decreased rate of survival after myocardial infarction.
Congestive heart failure.
o Genitourinary:
Impotence, retrograde ejaculation, urinary hesitancy, overflow incontinence.
At least 25% of men with diabetes have problems with sexual function.
There is often no association with glycaemic control, duration or severity of diabetes.
Risk factors for erectile dysfunction include increasing age, alcohol, initial glycaemic
control, intermittent claudication and retinopathy.
o Gastrointestinal:
Nausea and vomiting.
Abdominal distension.
Dysphagia.
Diarrhoea.
o Gustatory sweating, anhidrosis.

600. Which artery runs in the anterior inter-ventricular groove?


a. Acute marginal branch
b. Left ant descending artery
c. Septal branches
d. Circumflex artery
e. Right coronary artery

b. Left ant descending artery

601. A mother presents with her 12m daughter. The child has no meaningful words, is unable to sit unaided
and cant play with her toys. She doesnt laugh and has poor interaction with her
siblings. What is the best management strategy?
a. Arrange hearing test
b. Assess developmental milestones
c. Reassure
d. Refer to speech therapist
e. MRI brain

b. Assess developmental milestones


1st we will do assessment of all developmental milestones thn will go for ct or mri
602. A pt presents with progressive visual deterioration. Exam: large, multiple cotton wool spots in
both eyes. What is the single most likely dx?
a. Kaposis sarcoma
b. Cryptosporidium
c. CMV infection
d. Pneumocystis carinii infection
e. Cryptococcal infection

c. CMV infection

Retinitis:
Retinitis is the most common manifestation of CMV disease in patients who are HIV positive.
It presents with decreased visual acuity, floaters, and loss of visual fields on one side.
Ophthalmological examination shows yellow-white areas with perivascular exudates. Haemorrhage is
present. Lesions may appear at the periphery of the fundus, but they progress centrally.
It begins as a unilateral disease, but in many cases it progresses to bilateral involvement. It may be
accompanied by systemic CMV disease.
Ganciclovir has been used to treat retinitis, but it only slows the progression of the disease. The
optimal treatment is using ganciclovir implants in the vitreous, accompanied by intravenous ganciclovir
therapy.
Oral ganciclovir may be used for prophylaxis of CMV retinitis. It should not be used for treatment.

603. A 53yo had a dental extraction after which he recently had a mitral valve prolapse, high temp of 39C,
cardiac failure and new cardiac murmur. What is the single most likely dx?
a. Atheroma
b. Congenital
c. Regeneration
d. Infection
e. Neoplastic

d. Infection
infective endocarditis?

604. A 12yo boy with a hx of fall on an outstretched hand was brought to the ED with swelling and
pain around the elbow. His radial nerve was affected. What is the type of fx?
a. Angulated fx
b. Epiphyseal fx
c. Compound fx
d. Spiral fx

d. Spiral fx

Humeral shaft fractures

Complications
Radial nerve injury: occurs in 11.8% of fractures. It is most common in distal third fractures.It is more
common in transverse or spiral fracture. Spontaneous recovery occurs in 70.7% treated conservatively.
Initial expectant treatment may avoid unnecessary operations.
Brachial artery injury.
Non-union.

605. A 32yo lady complains that she hears everyone saying that she is an evil person. What type of
hallucinations is she suffering from?
a. 2nd person auditory hallucinations
b. 3rd person auditory hallucinations
c. Echo de la pense
d. Gedankenlautwerden

b. 3rd person auditory hallucinations


She hears everyone
talking ABOUT her. So it's third person hallucination. If she had been hearing
everyone talking TO her, it would've been second person hallucination.

606. A 65yo woman had an excision of colonic tumor 3yrs ago. Now she is losing weight and feels
lethargic. Exam: pale but no abdominal findings. What is the most appropriate inv?
a. CA 125
b. CA 153
c. CA 199
d. CEA
e. AFP
d. CEA

607. A 46yo African-Caribbean man is found to have BP=160/90mmHg on 3 separate occasions. What is the
best initial tx?
a. ACEi
b. Beta-blockers
c. ARBs
d. None
e. CCB

e. CCB

Step 1 treatment
patients < 55-years-old: ACE inhibitor (A)
patients > 55-years-old or of Afro-Caribbean origin: calcium channel blocker

Step 2 treatment
ACE inhibitor + calcium channel blocker (A + C)

Step 3 treatment
add a thiazide diuretic (D, i.e. A + C + D)
NICE now advocate using either chlorthalidone (12.5-25.0 mg once daily) or indapamide (1.5 mg
modified-release once daily or 2.5 mg once daily) in preference to a conventional thiazide diuretic such
as bendroflumethiazide

608. A 39yo woman will undergo tubal sterilization and she wants to know the failure rate of this
type of sterilization.
a. 1:50
b. 1:200
c. 1:500
d. 1:1000
e. 1:5000

b. 1:200
Male sterilisation - vasectomy
failure rate: 1 per 2,000*
semen analysis needs to be performed twice following a vasectomy before a man can have
unprotected sex (usually at 16 and 20 weeks)
Female sterilisation
failure rate: 1 per 200*

609. Which of the following reflexes and innervating spinal nerves are correctly paired?
a. Anal reflex S1
b. Ankle jerk L5
c. Biceps jerk C7 & C8
d. Knee jerk L3 & L4
e. Triceps jerk T1
d. Knee jerk L3 & L4
bicep: C5-C6
Tricep: C6-C7
Ankle: S1

610. A 62yo man with rheumatoid arthritis struck his hand against a door. He subsequently found
that although he could extend the interphalangeal joint of his right thumb, the MCP joint of the
thumb remained flex. What is the single most likely tendon to have been damaged?
a. Extensor carpi ulnaris
b. Extensor digitorum
c. Extensor indicis
d. Extensor pollicis brevis
e. Extensor pollicis longus

d. Extensor pollicis brevis


Remember pollicis is for thumb. Extensir pollicis brevis is inserted at the base of proximal phalanx of thumb .
Extensor pollucis longus at the interphalangeal joint of thumb. As she is unable to extend MCP joint so brevus
tendon is ruptured.
( Make it simple.. from longus remember it would be long and go more distal to get inserted at the IP joint.
And brevis ends short at MCP joint)

611. A 68yo lady complains of falls to the ground without any warning, maintains consciousness and no
confusion. She says this has occurred at number of times. What is the dx?
a. Stokes Adams attack
b. Hypoglycemia
c. Vasovagal syncope
d. Drop attacks
e. Epilepsy

d. Drop attacks
Sudden falling to ones knees without LOC and without warning. Recovery is immediate
brief LOC in SAA without warning,while vv syncope- triggers with loc
because she maintains consciousness. Stokes Adams and Vaso-vagal--> brief loss of consciousness. Stokes
Adams patient gets pale before attack. Vaso-vagal mostly in young girls
age is not for epilepsy , other than a,b,c preceding particular s/s with brief loc in a&b ,
Adams is associated Av heart block bradyarrhythmia,hypoglycemia has hx of dm or sweating
tachycardia,vasovagal is suddenly change in position and epilepsy is hx and initial convulsive and postictal
phase

A classic Stokes-Adams attack is a collapse without warning, associated with loss of consciousness for
a few seconds. Typically, complete (third-degree) heart block is seen on the ECG during an attack (but
other ECG abnormalities such as tachy-brady syndrome have been reported)
Syncope is a transient loss of consciousness caused by transient global cerebral hypoperfusion
characterised by rapid onset, short duration, and spontaneous complete recovery
Hypoglycaemia
Shaking and trembling
Sweating, pins and needles in the lips and tongue
Hunger, palpitations
Headache (occasionally), double vision, difficulty in concentrating
Slurring of speech, confusion, change of behaviour
Stupor, coma

612. A 50yo man complains of being pursued by the police for a crime he denies committing. He has poor
concentration and impaired short-term memory. He admits to drinking large amounts of
alcohol for the last 20yrs. What is the most probable dx?
a. Dementia
b. Hallucination
c. Wernickes encephalopathy
d. Schizophrenia
e. Korsakoff psychosis

e. Korsakoff psychosis
ConfabulationFalsification of memory in clear consciousness - very characteristic of the syndrome.
Anterograde amnesia is the main feature of the syndrome.
Retrograde amnesiaTelescoping of events is characteristic - eg, the patient says something happened recently
when it took place many years ago.

Wernicke-Korsakoff syndrome (WKS) is a spectrum of disease resulting from thiamine deficiency, usually
related to alcohol abuse.

Wernicke's encephalopathy: a classic triad of symptoms (mental confusion, ataxia and ophthalmoplegia)

Korsakoff's syndrome: late manifestation of the condition, where Wernicke's encephalopathy has not been
adequately treated

Serum thiamine levels (vitamin B1) levels may be low.

613. A pt with prv hx of HTN, the membranes have ruptured and the cervix is 3cm dilated. 4h later on
examination showed that the cervix was still 3cm dilated. What is the single most appropriate
management for her labor?
a. Repeat vaginal examination in 4h
b. CTG
c. C-section
d. External rotation
e. IV syntocin drip

e. IV syntocin drip
Prolonged labour first line is to augment labour, either artficial rupture of membranes followed by IV syntocin.
In this case patient membranes have ruptured spontaneously and there is no contraindication to syntocin. So
augment labour first if patient still had poos progress after augmenting labour then C section.

Active management of labour has been modified significantly over time but the core principles remain:
Early diagnosis following strict criteria, by a senior midwife.
Vaginal examination hourly for three hours, then every two hours, at least. This allows the rate of
progress to be plotted on a partogram.
Amniotomy one hour after admission.
Augmentation with Syntocinon if not dilating at rate of 1 cm/hour.

614. A 6yo girl has had 2 short episodes of cough and wheeze over the last 12m. These 2 acute
episodes responded quickly to bronchodilator, she has no symptoms or abnormal physical signs.
She has slight eczema and her mother has asthma. What is the single most appropriate inv?
a. CXR
b. Peak flow rate diary
c. Pulse oximetry
d. Spirometry
e. Sweat test

d. Spirometry
Diagnostic test for asthma is Spirometry. And if it was a known asthmatic case then our investigation wud be
pefr....if the ques mentioned something abt known asthma on beta agonist and or steroid...yt having
symptoms and exacerbations provided she is taking inhaler properly...best advice wud be to maintain a pef
diary so tht we can adjust treatment options.
Spirometry
If FEV1/ FBV > 80% ( Restrictive lung disease)
If FEV1/FVC < 80% ( obstructive lung disease)
If reversible > 12% after brochochodilator ( reversibilty test) its asthma otherwise copd

Spirometry
With older children with an intermediate probability of asthma, diagnostic tests such as PEFR and forced
expiratory volume in one second (FEV1) can provide objective measures of airways obstruction but these may
be normal between episodes of bronchospasm and provide poor discrimination with other conditions that
also cause airways obstruction.Spirometry is usually possible from about 5 years old, although there is wide
variation, and is dependent on the child's co-operation and comprehension of the task.
Where there is evidence of airways obstruction, looking for changes in PEFR or FEV1 10 minutes after the use
of a bronchodilator (reversibility usually taken as >12% subsequent improvement in lung function). Also, look
for response to a treatment trial over a defined time period, as this adds further weight to the diagnosis of
asthma.

615. A 45yo man had recently started taking anti-HTN therapy. 6m later his RBS=14mmol/l. Which
single drug is most likely to have caused this?
a. Amlodipine
b. Bendroflumethiazide
c. Doxazosin
d. Losartan
e. Ramipril

b. Bendroflumethiazide
Pt was not diabetic, but he develops DM after taking medication...Thiazide and B-blockers increases risk of DM
OHCM-134
The connection between diuretics and hyperglycemia involves intracellular K+ levels. Intracellular K+ is
involved in the secretion of a lot of hormones...including insulin. Some diuretics cause hypokalemia, (decrease
inK+) like thiazides. This inhibits insulin secretion and can lead to hyperglycemia.
Common adverse effects
dehydration
postural hypotension
hyponatraemia, hypokalaemia, hypercalcaemia
gout
impaired glucose tolerance
impotence
Amlodipine Flushing, headache, ankle swelling
Doxazosin postural hypotension, drowsiness, dyspnoea, cough
Losartan Like ACE inhibitors they should be used with caution in patients with renovascular disease. Side-
effects include hypotension and hyperkalaemia.

Ramipril cough, angioedema, hyperkalaemia, first-dose hypotension.

616. A 27yo waitress has pelvic pain, dysmenorrhea and increasingly heavy periods. She also
complains of dyspareunia. There is generalized pelvic tenderness without peritonism. Pelvic US
is normal. What is the most likely dx?
a. Endometriosis
b. Uterine fibroid
c. Pelvic congestion syndrome
d. PID
e. Tubal pregnancy

c. Pelvic congestion syndrome (or A?)


Here, Profession , Waitress, is the clue , prolonged standing is the risk factor for pelvic congestion syndrome!
It is pelvic venous congestion, dilated pelvic veins cause cyclic dragging pain, worse menstrually and after
prolonged standing,walking. Dyspareunia. Air hostess, waiters.
Also pelvis us free suggest absent of endometriosis plus waitress !! But if didn't mentioned pelvic us it will be
typically endometriosis

Ultrasound examination may be useful (US). This imaging test uses sound waves to detect the abnormal veins.
It is good at showing the blood flow through the veins and is non-invasive. However, sometimes the veins in
the pelvis are difficult to see through the abdomen, therefore a special ultrasound where a small probe is
placed into the vagina to see the veins, called a transvaginal ultrasound may be required.

Duplex ultrasound scanning =golden standard diagnosis

617. A 14yo girl is clinically obese. She has not started her periods yet and has severe acne. Among
her inv, a high insulin level is found. What is the most probable dx?
a. Cushings syndrome
b. Graves disease
c. Acquired hypothyroidism
d. PCOS
e. Addisons disease
d. PCOS
14 yr...primary amenorrhea, severe acne ,high insulin. ...All favours PCOS.
obesity hyperandrogenism(acne) hyperinsulinemia and amenorrhea
P' AM + Insulin Resistance. Both Cushing n PCOS have hyperglycaemia but in Cushing's $, it's due to high
glucocorticoids lvl, not coz of Insulin Resistance.
Features
subfertility and infertility
menstrual disturbances: oligomenorrhea and amenorrhoea
hirsutism, acne (due to hyperandrogenism)
obesity
acanthosis nigricans (due to insulin resistance)

Investigations
pelvic ultrasound: multiple cysts on the ovaries
FSH, LH, prolactin, TSH, and testosterone are useful investigations: raised LH:FSH ratio is a 'classical'
feature but is no longer thought to be useful in diagnosis. Prolactin may be normal or mildly elevated.
Testosterone may be normal or mildly elevated - however, if markedly raised consider other causes
check for impaired glucose tolerance

618. An 18yo girl with primary amenorrhea complains of severe abdominal pain every 4-8weeks
which is now getting worse. Exam: lower abdominal mass is felt. What is the most probable dx?
a. Ectopic pregnancy
b. Ovarian carcinoma
c. Hematometrium
d. Biliary colic
e. Renal carcinoma

c. Hematometrium
hematometrium, may be septate vagina
She has cyclical bleed every month as scenario tells. Examination shows lower abdmonal mass likely blood
accumulation everytime. Cause is likely imperforate hymen or transvaginal septum

619. A 14yo boy with asthma suddenly developed chest pain and increasing breathlessness during a
game of football. When seen in the ED he was not cyanosed. He has reduced breath sounds on
the right side. His oxygen saturation is 94% on air. What is the single most appropriate inv?
a. Capillary blood gases
b. CXR
c. CT chest
d. Exercise challenge
e. MRI chest

b. CXR
spontaneous pneumothorax
young pt, sudden cp, sob, decreased breath sounds -> spont pneumothorax on rt. side ... CXR- if >2cm...do
aspiration
if the rim of air is < 2cm and the patient is not short of breath then discharge should be considered
otherwise aspiration should be attempted
if this fails (defined as > 2 cm or still short of breath) then a chest drain should be inserted

620. A 36yo woman was recently admitted to a psychiatric ward. She believes that the staff and
other pts know exactly what she is thinking all the time. What is the most likely symptom this pt
is suffering from?
a. Thought insertion
b. Thought withdrawal
c. Thought block
d. Though broadcasting
e. Hallucination

d. Though broadcasting

Thought insertion, removal or interruption - delusions about external control of thought

Thought broadcasting - the delusion that others can hear one's thoughts

thought withdrawal is the delusional belief that thoughts have been 'taken out' of the patient's mind, and the
patient has no power over this. It often accompanies thought blocking.

Thought blocking is a thought condition usually caused by a mental health condition such as schizophrenia.
During thought blocking, a person stops speaking suddenly and without explanation in the middle of a
sentence.

hallucination is a perception in the absence of external stimulus that has qualities of real perception.

621. A 60yo woman is admitted to the hospital after a fall. She is noted to have poor eye contact.
When asked how she is feeling, she admits to feeling low in mood and losing enjoyment in all
her usual hobbies. She has also found it difficult to concentrate, feels that she is not good at
anything, feels guilty over minor issues and feels very negative about the future. What is the
most likely dx?
a. Mild depression
b. Moderate depression
c. Severe depression
d. Psychosis
e. Seasonal depression

a. Mild depression

NICE use the DSM-IV criteria to grade depression:


1. Depressed mood most of the day, nearly every day
2. Markedly diminished interest or pleasure in all, or almost all, activities most of the day, nearly every
day
3. Significant weight loss or weight gain when not dieting or decrease or increase in appetite nearly
every day
4. Insomnia or hypersomnia nearly every day
5. Psychomotor agitation or retardation nearly every day
6. Fatigue or loss of energy nearly every day
7. Feelings of worthlessness or excessive or inappropriate guilt nearly every day
8. Diminished ability to think or concentrate, or indecisiveness nearly every day
9. Recurrent thoughts of death, recurrent suicidal ideation without a specific plan, or a suicide attempt
or a specific plan for committing suicide

Subthreshold depressive
symptoms Fewer than 5 symptoms

Mild depression Few, if any, symptoms in excess of the 5 required to make the diagnosis, and
symptoms result in only minor functional impairment

Moderate depression Symptoms or functional impairment are between 'mild' and 'severe'

Severe depression Most symptoms, and the symptoms markedly interfere with functioning. Can
occur with or without psychotic symptoms

622. A 70yo woman lives in a nursing home following a stroke has developed reddish scaly rash on
her trunk. She has many scratch marks on her limbs and trunk with scaling lesions on her hands
and feet. What is the single most appropriate initial tx?
a. Aqueous cream
b. Chlorphenaramine
c. Coal tar
d. 1% hydrocortisone ointment
e. Permethrin

e. Permethrin
Nursing home, multiple scratch marks :scabies

Features
widespread pruritus
linear burrows on the side of fingers, interdigital webs and flexor aspects of the wrist
Nodules may develop. These occur particularly at the elbows, anterior axillary folds, penis, and
scrotum.
in infants the face and scalp may also be affected
secondary features are seen due to scratching: excoriation, infection

Management
permethrin 5% is first-line
malathion 0.5% is second-line
give appropriate guidance on use (see below)
pruritus persists for up to 4-6 weeks post eradication

623. A 16yo boy following a RTA was brought to the ED with a swelling and deformity in his right
thigh. Exam: airway is patent and is found to have a pulseless leg. Which structure is involved in
this fx?
a. Femoral artery
b. Posterior tibial artery
c. Common peroneal nerve
d. Dorsalis pedis

a. Femoral artery

624. A man sat cross-legged for about 30mins. After this he was unable to dorsiflex his left foot and
had loss of sensation in the web space between the big toe and the 2nd toe. He also has sensory
loss on the same side of the foot after 2h. Which of the following was affected?
a. Femoral nerve
b. Sural nerve
c. Peroneal nerve
d. Sciatic nerve

c. Peroneal nerve
Chronic peroneal neuropathy can result from, among other conditions, bed rest of long duration, hyperflexion
of the knee, peripheral neuropathy, pressure in obstetric stirrups, and conditioning in ballet dancers. The most
common cause is habitual leg crossing that compresses the common peroneal nerve as it crosses around the
head of the fibula.Transient trauma to the nerve can result from peroneal strike.
Damage to this nerve typically results in foot drop, where dorsiflexion of the foot is compromised and the foot
drags (the toe points) during walking; and in sensory loss to the dorsal surface of the foot and portions of the
anterior, lower-lateral leg.
625. A 25yo woman is presenting with diarrhea and abdominal bloating over the last 4m. Exam: she
has blistering rash over her elbows. Biochemistry: low serum albumin, calcium and folate conc.
On jejunal biopsy, there is shortening of the villi and lymphocytosis. What is the most likely dx?
a. Celiac disease
b. Whipples disease
c. Crohns disease
d. Tropical sprue
e. Giardiasis
f. Cystic fibrosis

a. Celiac disease
Patient with chronic diarrhoea, villus atrophy, lymphocytosis, Dermatitis herpetiformis. All with Celiac disease.
Coeliac Disease ( Endoscopic small bowel biopsy- subtotal villus atrophy and lymphocytic infiltration )
The blistering rash is dermatitis herpetiformis..associated with coeliac

Coeliac disease is caused by sensitivity to the protein gluten. Repeated exposure leads to villous atrophy which
in turn causes malabsorption. Conditions associated with coeliac disease include dermatitis herpetiformis (a
vesicular, pruritic skin eruption) and autoimmune disorders (type 1 diabetes mellitus and autoimmune
hepatitis).

Signs and symptoms Conditions

Chronic or intermittent diarrhoea Autoimmune thyroid disease


Failure to thrive or faltering growth (in children) Dermatitis herpetiformis
Persistent or unexplained gastrointestinal Irritable bowel syndrome
symptoms including nausea and vomiting Type 1 diabetes
Prolonged fatigue ('tired all the time') First-degree relatives (parents, siblings or
Recurrent abdominal pain, cramping or children) with coeliac disease
distension
Sudden or unexpected weight loss
Unexplained iron-deficiency anaemia, or other
unspecified anaemia

Complications
anaemia: iron, folate and vitamin B12 deficiency (folate deficiency is more common than vitamin B12
deficiency in coeliac disease)
hyposplenism
osteoporosis, osteomalacia
lactose intolerance
enteropathy-associated T-cell lymphoma of small intestine
subfertility, unfavourable pregnancy outcomes

Immunology
tissue transglutaminase (TTG) antibodies (IgA) are first-choice according to NICE
endomyseal antibody (IgA)
anti-gliadin antibody (IgA or IgG) tests are not recommended by NICE
anti-casein antibodies are also found in some patients

Jejunal biopsy
villous atrophy
crypt hyperplasia
increase in intraepithelial lymphocytes
lamina propria infiltration with lymphocytes

Whipples disease diarrhea, steatorrhea, abdominal pain, weight loss, migratory arthropathy, fever, and
neurological symptoms. duodenal endoscopy, which reveals PAS-positive macrophages

626. A 19yo man presents for the 1st time with a firm and unshakable belief that he is being followed
by terrorists who are plotting against him. What is the single best term for this mans condition?
a. Delusion of persecution
b. Delusion of grandeur
c. Delusion of control
d. Delusion of reference
e. Delusion of nihilism

a. Delusion of persecution
most common types of delusions, centering around a person's fixed, false belief that others aim to obstruct,
harm, or kill him/her.

Delusion of grandeur fixed, false belief that one possesses superior qualities such as genius, fame,
omnipotence, or wealth.

Delusion of control false belief that another person, group of people, or external force controls one's general
thoughts, feelings, impulses, or behavior.

Delusion of reference A neutral event is believed to have a special and personal meaning. For example, a
person with schizophrenia might believe a billboard or a celebrity is sending a message meant specifically for
them.

Delusion of nihilism the delusion that things (or everything, including the self) don't exist. a sense that
everything is unreal.

627. A 19yo female is brought in by her parents. They are concerned about her BMI which is 12. She
is satisfied with it. What is the next step?
a. Psychiatric referral for admission
b. Family counselling
c. Social service
d. Start antidepo
e. Medical admission

e. Medical admission
bmi s 12..so next step s medical admission.
The defining clinical features are:
Refusal to maintain a normal body weight for age and height.
Weight below 85% of predicted. This means in adults a body mass index (BMI) below 17.5 kg/m2.
Having a dread of gaining weight.
Disturbance in the way weight or shape is experienced, resulting in over-evaluation of size.
Amenorrhoea for three months or longer
fatigue, hypothermia, hypotension, peripheral oedema, gaunt face, lanugo hair, scanty pubic hair,
acrocyanosis (hands or feet are red or purple), and bradycardia
Enhanced weight loss by over-exercise, diuretics, laxatives and self-induced vomiting
need for urgent referral and appropriate medical intervention

Nutrition: BMI below 14; weight loss more than 0.5 kg per week.
Circulation: systolic BP below 90; diastolic BP below 70; postural drop greater than 10 mm Hg.
Squat test: unable to get up without using arms for balance or leverage.
Core temperature below 35C.
Blood tests: low potassium, sodium, magnesium or phosphate. Raised urea or LFTs. Low albumin or
glucose.
ECG: pulse rate below 50; prolonged QT interval.

628. A lady who works at a nursing home presents with itching. Exam: linear tracks on the wrist. She
says that 2d ago she had come in contact with a nursing home inmate with similar symptoms.
What is the mechanism of itching?
a. Infection
b. Destruction of keratinocytes
c. Allergic reaction
d. Immunosuppression
e. None
c. Allergic reaction
Scabies. pruritis due to allergic reaction.

629. A teacher had a respiratory infection for which she was prescribed antibiotics. After the
antibiotic course when she rejoined school, she lost her voice completely. What is the single
most appropriate dx?
a. Recurrent laryngeal nerve palsy
b. Angioedema
c. Laryngeal obstruction by medication
d. Laryngitis
e. Functional dysphonia/vocal cords

e. Functional dysphonia/vocal cords

Can't be recurrent laryngeal nerve because major reasons are it's either trauma via surgery for thyroid or any
neck surgery, tumour of neck,tumour superior vena cava, tumour of mediastinal,metastasis etc.

can't b angioedema because it presents acutely with in minutes and usually superficial ie visible sites though
also involve deeper respiratory structures. Drug's notorious to cause angioedema are A.C.E.i and in rare cases
A.R.B.s and very rarely antibiotics antifungals and can be any drug acute reaction.
Can't be laryngeal obstruction because it will eventually block your respiratory intake ie breathing which this
patient seems to having no problem with.

Similarly Laryngitis is an infection for which doctors do prescribe antibiotics but main treatment is voice rest
and gargles and further question says after use of antibiotics in laryngitis voice hoarseness or loss is pre
treatment mostly.

Antibiotics rarely cause voice loss.

Where no organic cause is found - a diagnosis of exclusion.


A common cause of hoarseness. There are various forms (below).
Infections

Acute laryngitis (common), often with upper respiratory infection. Usually viral (may have secondary infection
with staphylococci or streptococci).
Other infections - fungal or tuberculous.
Benign laryngeal conditions

Voice overuse - common.


Benign lesions of the vocal cords - eg, nodules (singer's nodes), polyps and papillomas.

630. A 43yo lady is admitted with pyrexia, arthropathy, breathlessness and syncope. She was recently
dx with pulmonary emboli. There is an early diastolic sound and a mid-diastolic rumble. Her JVP
is elevated with prominent a-waves. What is the most likely cause?
a. Mitral regurgitation
b. Ventricular ectopics
c. Pulmonary regurgitation
d. Atrial myxoma
e. Complete heart block

d. Atrial myxoma
Atrial Myxoma presents mostly with signs and symptoms of mitral stenosis if in left atrium, so there's mid
diastolic murmur and the other early diastolic sound is called tumour plop which is characteristic to the impact
of the tumour on the mitral valve in diastole as the valves in systole were closed and holding the tumour
above but when they open the Myxoma falls on the valve producing early diastolic plop.On top we have extra
cardiac symptoms

depending upon location,if myxoma obstructs the valve, then its diastolic rumble...if leaflets r damaged, then
its a systolic rumble(due to regurgitation)
Jugular venous pressure may be elevated, and a prominent A wave may be present.
A loud S1 is caused by a delay in mitral valve closure due to the prolapse of the tumor into the mitral valve
orifice (mimicking mitral stenosis).
P2 may be delayed. Its intensity may be normal or increased, depending on the presence of pulmonary
hypertension.
In many cases, an early diastolic sound, called a tumor plop, is heard. This sound is produced by the impact of
the tumor against the endocardial wall or when its excursion is halted.
An S3 or S4 may be audible.
A diastolic atrial rumble may be heard if the tumor is obstructing the mitral valve.
If there is valve damage from the tumor, mitral regurgitation may cause a systolic murmur at the apex.
A right atrial tumor may cause a diastolic rumble or holosystolic murmur due to tricuspid regurgitation.
General examination may reveal fever, cyanosis, digital clubbing, rash, or petechiae.

631. A 28yo man presents with a maculopapular rash over his trunk and palms. He also has
numerous mouth ulcers. He had a penile ulcer which healed 2wks ago. What will you do to
confirm the dx?
a. PCR for treponemal and non-treponemal antibiodies
b. Dark ground microscopy from mouth ulcer
c. Blood culture for treponema
d. Dengue fever

a. PCR for treponemal and nontreponemal antibodies


because it has now progressed to secondary syphilis and the investigation of choice is PCR
Dark ground microscopy is done from the chancre fluid and not from mouth ulcers
stage 1/ primary= dark field microscopy. 2= treponeme specific and non- antibodies, treponemes r seen in the
lesions too. Late secondary= organisms can no longer be seen but the AB tests are still +. tertiary= look for FTA
and TPHA antibodies in CSF.
PCR is the best for confirmation of any diagnosis??

Management
benzylpenicillin
alternatives: doxycycline
the Jarisch-Herxheimer reaction is sometimes seen following treatment. Fever, rash, tachycardia after
first dose of antibiotic. It is thought to be due to the release of endotoxins following bacterial death
and typically occurs within a few hours of treatment.

632. A 34yo man complains of arthralgia, abdominal pain and vomiting, a facial rash that is worse in
the summer and hematuria. Urea and creatinine are slightly elevated with urinalysis
demonstrating red cell casts. PMH is remarkable for childhood eczema. Which inv is most likely
to lead to a dx?
a. US KUB
b. Joint aspiration
c. Auto antibodies
d. IVU
e. Renal biopsy

c. Autoantibodies
HSP mostly in children after a viral infection and with a palpable purpura on buttocks and extensor surfaces. Si
can't be HSP.
SLE yes I agree a lot of presentations going in favour of SLE. arthralgias rash photosensitivity renal
involvement. I don't recall abdominal pain and vomiting in SLE.
SLE - Facial Rash, worsen by sunlight, arthralgia, Nephritis, Other immune problems ( eczema )
General features
fatigue
fever
mouth ulcers
lymphadenopathy
Skin
malar (butterfly) rash: spares nasolabial folds
discoid rash: scaly, erythematous, well demarcated rash in sun-exposed areas. Lesions may progress to
become pigmented and hyperkeratotic before becoming atrophic
photosensitivity
Raynaud's phenomenon
livedo reticularis
non-scarring alopecia

Musculoskeletal
arthralgia
non-erosive arthritis

Cardiovascular
myocarditis

Respiratory
pleurisy
fibrosing alveolitis

Renal
proteinuria
glomerulonephritis (diffuse proliferative glomerulonephritis is the most common type)

Neuropsychiatric
anxiety and depression
psychosis
seizures

Autoantibodies:
ANA: screening test with a sensitivity of 95% but not diagnostic in the absence of clinical features.
Anti-dsDNA: high specificity but sensitivity is only 70%.

633. A 56yo woman has had severe abdominal pain for 24h radiating to her back and is accompanied
by nausea and vomiting. She appears to be tachycardic and in shock. She was found to have
gallstones, 2yrs ago. What is the most likely inv to confirm dx?
a. US abdomen
b. LFT
c. Serum lipase
d. Angiography
e. CT abdomen

c. Serum lipase
lipase is done for diagnosing pancreatiitis.... CT is done to find the complications of pancreatitis..
Serum amylase 3 or more times normal is the traditional way of diagnosing acute pancreatitis. However,
lipase levels are more sensitive and more specific.

634. A 32yo female with axillary freckles and caf au lait spots wants to know the cahnces of her child
also having similar condition.
a. 1:2
b. 1:4
c. No genetic link
d. 1:16
e. Depends on the genetic make up of the partner

a. 1:2 (or E)?


neurofibromatosis..autosomal dominant..so 1:2
if her partner has also same condition than there 75 %chances while he is normal than 50 % chances we
cant tell her for sure that there is 50 % of getting her baby affected when he husband is diseased. So i think
option E is correct answer in such scenerio
635. A 40yo man has pain, redness and swelling over the nasal end of his right lower eyelid. The eye is watery
with some purulent discharge. The redness extends on to the nasal peri-orbital area
and mucoid discharge can be expressed from the lacrimal punctum. What is the single most
appropriate clinical dx?
a. Acute conjunctivitis
b. Acute dacrocystitis
c. Acut iritis
d. Retrobulbar neuritis
e. Scleritis

b. Acute dacryocystitis

Dacryocystitis is infection of the lacrimal sac

Features
watering eye (epiphora)
swelling and erythema at the inner canthus of the eye

Management is with systemic antibiotics. Intravenous antibiotics are indicated if there is associated periorbital
cellulitis

636. A 60yo lady has severe chest pain. ECG shows changes of inferior wall MI. ECG also shows
progressive prolongation of PR interval until a QRS complex is dropped. What is the most
probable dx?
a. Atrial fibrillation
b. VT
c. SVT
d. Mobitz type I 2nd degree heart block
e. Mobitz type II 2nd degree heart block

d. Mobitz type I 2nd degree heart block

First degree heart block


PR interval > 0.2 seconds

Second degree heart block


type 1 (Mobitz I, Wenckebach): progressive prolongation of the PR interval until a dropped beat occurs
type 2 (Mobitz II): PR interval is constant but the P wave is often not followed by a QRS complex

Third degree (complete) heart block


there is no association between the P waves and QRS complexes

637. A 52yo woman speaks rapidly without any pause and ignores interruptions. She doesnt even
pause to take enough breaths. What term best describes this kind of speech?
a. Flight of ideas
b. Brocas aphasia
c. Wernickes aphasia
d. Pressure of speech
e. Verbal dysphasia

d. Pressure of speech

Flight of ideas a nearly continuous flow of rapid speech that jumps from topic to topic
Brocas aphasia When a stroke injures the frontal regions of the left hemisphere, different kinds of language
problems can occur. This part of the brain is important for putting words together to form complete
sentences. Injury to the left frontal area can lead to what is called Brocas aphasia.
Wernickes aphasia People with serious comprehension difficulties have what is called Wernickes aphasia.

638. A 30yo woman has been feeling low and having difficulty in concentrating since her mother
passed away 2m ago. She feels lethargic and tends to have breathlessness and tremors from
time to time. What is the most likely dx?
a. Adjustment disorder
b. PTSD
c. Panic disorder
d. GAD
e. Bereavement

a. Adjustment disorder
Adjustment disorder is a short-term condition that occurs when a person has great difficulty coping with, or
adjusting to, a particular source of stress, such as a major life change, loss, or event.
Unlike major depression, however, an adjustment disorder doesn't involve as many of the physical and
emotional symptoms of clinical depression (such as changes in sleep, appetite and energy) or high levels of
severity (such as suicidal thinking or behavior).
adj disorder starts within 3 m of the stress and does not last more than 6 m while bereavement starts w the
stress and does not last more than 2 months

It is not PTSD as in aetiology death of near one is not included and given case doesn't have the diagnostic
feature of repeated memory flashbacks or dream. Panic disorder does not occur in response to any external or
internal stress! So it is not panic disorder. Similarly GAD is chronic anxiety which is not directly related to any
object or situation. Which also does not explain death as a point in its favour! Beyond 2 months bereavement
is considered to be either pathological bereavement or major depression. So given time period of 2 months
indicates it is no more normal bereavement. So by exclusion I think it is a case of adjustment disorder (of
which death of near one is considered as an etiologic factor).
Bereavement is the time spent adjusting to loss. It has four stages accepting that your loss really happened
experiencing the pain that comes with grief
trying to adjust to life without the person who died
putting less emotional energy into your grief and finding a new place to put it i.e. moving on.
Since this woman is not able to adjust to the loss and is having physical symptoms I think that's the reason it's
adjustment disorder.

639. A 32yo man on psychiatric medications complains of inability to ejaculate. Which drug is most
likely to cause these symptoms?
a. Lithium
b. Haloperidol
c. Chlorpromazine
d. Fluoxetine
e. Clozapine

d. Fluoxetine
SSRI's ( fluoxetine ) cause sexual dysfunction
gastrointestinal symptoms are the most common side-effect

640. A 4yo boy is brought by his parents with complains of wetting his bed at night and whenever he
gets excited. What would be the most appropriate management for this child?
a. Desmopressin
b. Oxybutynin
c. Behavioural therapy
d. Tamsulosin
e. Restrict fluid intake

c. Behavioural therapy
Children below 7yrs : sleep alarms or behavioural therapy
Children above 7yrs : Desmopressin
Alarm training is a first-line treatment for nocturnal enuresis and is the most effective long-term strategy
Desmopressin should be offered first-line to children aged over 7 where rapid control is needed or an alarm is
inappropriate. Otherwise it should be used second-line after an alarm has been tried. It may be used in
children aged 5-7 if treatment is required under the same circumstances.
641. A 34yo DM pt is undergoing contrast radiography. What measure should be taken to prevent
renal damage with contrast dye?
a. Reduce contrast dye
b. Plenty of fluids
c. NSAIDS
d. ACEi
e. IV dextrose

b. Plenty of fluids

642. A 75yo woman presents to the breast clinic having noticed that she has had a blood stained
discharge from the left nipple, together with dry skin over the left areola. Exam: blood stained
discharge with dry flaky skin noted on the left areola. The nipple was noted to be ulcerated. Wht
is the most appropriate inv?
a. FNAC
b. MRI
c. Punch biopsy
d. Open biopsy
e. Stereotactic biopsy

c. Punch biopsy
Pagets disease
A punch biopsy is when the doctor removes a small circle of skin tissue to biopsy. You might have this type of
biopsy if your doctor thinks you could have inflammatory breast cancer or Paget's disease of the nipple.

643. A 50yo man presents with low mood, poor concentration, anhedonia and insomnia. He has had
2 episodes of increased activity, promiscuity and aggressive behavior in the past. He was arrest
8m ago for trying to rob a bank claiming it as his own. Which drug is most likely to benefit him?
a. Haloperidol
b. Citalopram
c. Desipramine
d. Carbamazepine
e. Ethosuximide

d. Carbamazepine
bipolar disorder
Lithium should be considered first-line, with the addition of valproate if ineffective.
Valproate or olanzapine should be considered for patients intolerant of lithium or who are not
prepared to undergo regular monitoring.
If symptoms still continue then the patient should be referred to a mental health specialist.
Medications that might be used in this situation are lamotrigine (especially in bipolar II disorder) or
carbamazepine.
If medication is stopped, patients should be made aware of early warning symptoms of recurrence.
Medication should be tailed off gradually

644. A 25yo woman complains of dizziness, nausea, vomiting, visual disturbances and anxiety which
keep coming from time to time. Most of the attacks are a/w sudden change in posture. What is
the most likely dx?
a. Panic disorder
b. Carotid sinus syncope
c. BPPV
d. Vertebrobasilar insufficiency
e. Postural hypotension

c. BPPV

vertigo triggered by change in head position (e.g. rolling over in bed or gazing upwards)
may be associated with nausea
each episode typically lasts 10-20 seconds
positive Dix-Hallpike manoeuvre

BPPV has a good prognosis and usually resolves spontaneously after a few weeks to months. Symptomatic
relief may be gained by:
Epley manoeuvre (successful in around 80% of cases)
teaching the patient exercises they can do themselves at home, for example Brandt-Daroff exercises

Medication is often prescribed (e.g. Betahistine) but it tends to be of limited value.

645. A 56yo man was recently put on anti-HTN meds and recent biochemistry on 2 occasions showed:
Na+=132, K+=7.6, Urea=11.3, Creat=112. Which of the following drugs is responsible for this
result?
a. Amlodipine
b. Bendroflumethiazide
c. Doxazosin
d. Atenolol
e. Ramipril

e. Ramipril
no Angiotensin II >> no Aldosterone >> hyponatremia, hyperkalemia.
Side-effects:
cough: occurs in around 15% of patients and may occur up to a year after starting treatment. Thought
to be due to increased bradykinin levels
angioedema: may occur up to a year after starting treatment
hyperkalaemia
first-dose hypotension: more common in patients taking diuretics

Cautions and contraindications


pregnancy and breastfeeding - avoid
renovascular disease - significant renal impairment may occur in patients who have undiagnosed
bilateral renal artery stenosis
aortic stenosis - may result in hypotension
patients receiving high-dose diuretic therapy (more than 80 mg of furosemide a day) - significantly
increases the risk of hypotension
hereditary or idiopathic angioedema
646. A 46yo woman has offensive yellow discharge from one nipple. She had a hx of breast abscess
3yrs ago. What is the possible dx?
a. Duct papilloma
b. Duct ectasia
c. Duct fistula
d. Breast cancer

c. Duct fistula
as there is H/O abscesses which might have led to fistula formation. Clincher here is previous H/O abscess
In cancer n papilloma discharge is blood stained mainly and ectasia it is greenish clear discharge from multiple
ducts

Breast cancer Characteristically a hard, irregular lump. There may be associated nipple inversion or skin
tethering

Paget's disease of the breast - intraductal carcinoma associated with a reddening and thickening (may
resemble eczematous changes) of the skin/areola
Mammary duct ectasia Dilatation of the large breast ducts
Most common around the menopause
May present with a tender lump around the areola +/- a green nipple discharge
If ruptures may cause local inflammation, sometimes referred to as 'plasma cell mastitis'
Duct papilloma Local areas of epithelial proliferation in large mammary ducts
Hyperplastic lesions rather than malignant or premalignant
May present with blood stained discharge
Breast abscess More common in lactating women
Red, hot tender swelling

647. A 35yo woman undergoing tx for TB presents with malar rash, photosensitivity and hematuria.
What is the single most likely positive antibody?
a. Anti Ds DNA
b. Anti Sm
c. Anti Histone
d. Anti La
e. Anti centromere

c. Anti Histone
drug induced lupus.
Most common causes
procainamide
hydralazine

Less common causes


isoniazid
minocycline
phenytoin

648. A 6wk child with profuse projectile vomiting. What is the first thing you will do?
a. US
b. Check serum K+ level
c. ABG
d. NG tube
e. IV fluids

b. Check serum K+ level


vomiting causes metabolic alkalosis.initially hypokalemia and later hyponatremia
This seems a case of Pyloric stenosis
Definitive diagnosis is done by US showing olive like
But on blood tests it shows low blood levels of potassium and chloride in association with an increased blood
pH and high blood bicarbonate level due to loss of stomach acid (which contains hydrochloric acid) from
persistent vomiting.
6 weeks is the typical age for pyloric stenosis presentation

649. A 55yo woman who attends the clinic has recently been dx with a depressive episode. She
complains of unintentionally waking early in the morning, a recent disinterest in sex and a loss
of appetite, losing 5kg weight in the last month. She feels that her mood is worse at the
beginning of the day. What is the most likely dx for this pt?
a. Mild depression
b. Moderate depression
c. Severe depression
d. Low mood
e. Pseudo depression

b. Moderate depression
Physical symptoms like weight loss and early morning insomnia makes it moderate as opposed to mild

Pseudodepression
"A condition of personality following frontal lobe lesion in which apathy, indifference and a loss of initiative
are apparent symptoms but are not accompanied by a sense of depression in the patient."

Q.621 for more

650. An employer sent his worker to the ED after having hit his head on a machine. Exam: normal. What is the
single most likely inv you would do?
a. Skull XR
b. CT head
c. MRI head
d. Reassure

a. Skull XR

CT head only if
Loss of consciousness more than 5 minutes,
More than two episodes of vomiting,
Signs of base of skull fracture ( peri orbital haematoma, bleeding or CSF leak from nose or ears),
Fits,
Double vision,
Headache not easing off with paracetamol or ibuprofen,
Unusual drowsiness/lOW GCS,
If none of the above discharge with head injury advise which means if any of the above develop in next 24 he
pt should come back to A&E,
This is current NICE head injury guideline.

651. A lady with fam hx of ovarian carcinoma has a pelvis US that fails to reveal any abnormality.
What is the single most appropriate inv?
a. Pelvic CT
b. CA 125
c. CA 153
d. Laparoscopy
e. MRI

b. CA 125
Pelvic usg is not as sensitive as ca 125. So even if pelvic usg fails to detect a small lesion, it can still be detected
by a rise in ca 125.

652. A 10yo boy is taken to his GP by his parents with behavioural prbs. He attends a special school
due to inappropriate behavior and during the interview with his parents the boy barks at
infrequent episodes and shouts expletives. What is the most likely dx?
a. Asperger syndrome
b. Cotard syndrome
c. Rett syndrome
d. Ekbom syndrome
e. Tourettes syndrome

e. Tourettes syndrome
tourette syndrome (multiple motor tics + vocal / phonetic tics)

Asperger syndrome: autism spectrum disorder (autism: a mental condition, present from early childhood,
characterized by great difficulty in communicating and forming relationships with other people and in using
language and abstract concepts.) The main difference from classic autism is a lack of delayed or retarded
cognition and language. Those with AS are also more likely to seek social interaction and share activities and
friendships.
In classic autism, children tend to be spotted earlier (18-30 months) because of impaired communication. In
Asperger's syndrome (AS), the diagnosis comes later - usually at school entry, when socialisation becomes
necessary. Many people with AS may learn to mask their problems. They may present as patients with no
serious mental health problem, but who are anxious, lonely, have a poor employment record and just don't
seem to fit in.

Cotard syndrome: afflicted person holds the delusion that he or she is dead, either figuratively or literally.

Rett syndrome: small feet ,hands & head ,no speech no walking, repeated hand movement. Onset occurs
between 6 and 18 months of age. deceleration of the rate of head growth

ekbom syndrome: delusional parasitosis delusional belief that they are infested with parasites, whereas in
reality no such parasites are present.
Wittmaack-Ekbom syndrome: a synonym of restless legs syndrome

Tourettes syndrome Tics can be defined as sudden, purposeless, repetitive, non-rhythmic, stereotyped
movements or vocalisations - eg, eye twitching or blinking. Examples of vocal tics are throat clearing, grunting
and barking.

Other features that may be seen in Tourette's syndrome


Echolalia - involuntary copying of other's' words.
Palilalia - repeating one's own words.
Coprolalia - compulsory saying of dirty words, which is pathognomonic of the syndrome and is seen in
about 10% of patients.
Copropraxia - making obscene gestures.
Echopraxia - involuntary copying of other's movements.
Difficulty concentrating or easily distracted.

653. A 52yo male presents with sudden complete loss of vision from right eye. He also had been
complaining of right sided headaches which would come up more on chewing. On fundoscopy,
the retina was pale and a cherry red spot could be seen in the macular region. What caused this
vision loss?
a. CRAO
b. CRVO
c. Branch RAO
d. Branch RVO
e. Circumciliary vein occlusion

a. CRAO
pale optic disc, cherry red spot on macula

The most common causes of a sudden painless loss of vision are as follows:
ischaemic optic neuropathy (e.g. temporal arteritis or atherosclerosis)
occlusion of central retinal vein
occlusion of central retinal artery
vitreous haemorrhage
retinal detachment

Ischaemic optic neuropathy


may be due to arteritis (e.g. temporal arteritis) or atherosclerosis (e.g. hypertensive, diabetic older
patient)
due to occlusion of the short posterior ciliary arteries, causing damage to the optic nerve
altitudinal field defects are seen

Central retinal vein occlusion


incidence increases with age, more common than arterial occlusion
causes: glaucoma, polycythaemia, hypertension
severe retinal haemorrhages are usually seen on fundoscopy
Central retinal artery occlusion
due to thromboembolism (from atherosclerosis) or arteritis (e.g. temporal arteritis)
features include afferent pupillary defect, 'cherry red' spot on a pale retina

Vitreous haemorrhage
causes: diabetes, bleeding disorders
features may include sudden visual loss, dark spots

Retinal detachment
features of vitreous detachment, which may precede retinal detachment, include flashes of light or
floaters

654. A 48yo woman presents with left-sided severe headache. She also has a red, watering eye and complains
of seeing colored haloes in her vision. What is the most appropriate next step?
a. Measure IOP
b. Relieve pain with aspirin
c. 100% oxygen
d. CT
e. Relieve pain with sumatriptan

a. Measure IOP
Red watery eye point to cluster headache but that's occur in young males...all other points to glaucoma
haloes seen in glaucoma. Measure iop

Features
severe pain: may be ocular or headache
decreased visual acuity
symptoms worse with mydriasis (e.g. watching TV in a dark room)
hard, red eye
haloes around lights
semi-dilated non-reacting pupil
corneal oedema results in dull or hazy cornea
systemic upset may be seen, such as nausea and vomiting and even abdominal pain

Management
urgent referral to an ophthalmologist
management options include reducing aqueous secretions with acetazolamide and inducing pupillary
constriction with topical pilocarpine

655. A 31yo woman presents with 7-10days following childbirth, with loss of feeling for the child, loss
of appetite, sleep disturbance and intrusive and unpleasant thoughts of harming the baby. What
is the best tx for this pt?
a. Fluoxetine
b. Haloperidol
c. CBT
d. Reassurance
e. ECT

e. ECT
Antipsychotics are given taking into account the breastfeeding factor. Usually lithium is given but ECT is better
way to go. Why we need ECT? As this is psychosis and there is danger to both mother and child we need to
control the situation rapidly with high intensity psychological intervention

Postpartum psychosis is a psychiatric emergency. It requires urgent assessment, referral, and usually
admission, ideally to a specialist mother and baby unit.[8]

Management is primarily pharmacological, using the same guidance as for other causes of psychosis.
Medication would normally involve an antipsychotic and/or mood stabilising drug. However, choice of
medication must take breastfeeding into account. Mothers requiring lithium treatment should be encouraged
not to breast-feed, due to potential toxicity in the infant. Most antipsychotics are excreted in the breast milk,
although there is little evidence of it causing problems. Where they are prescribed to breast-feeding women,
the baby should be monitored for side-effects. Clozapine is associated with agranulocytosis and should not be
given to breast-feeding women. Electroconvulsive therapy (ECT) may also be considered in some cases

656. A 56yo male pt presents with intermittent vertigo, tinnitus and hearing loss. What is the best
drug tx for this pt?
a. Buccal prochlorperazine
b. Oral flupenphenazine
c. TCA
d. Gentamicin patch on the round window
e. No med tx available

a. Buccal prochlorperazine
meiners disease-t/t oral percholperazine

Features
recurrent episodes of vertigo, tinnitus and hearing loss (sensorineural). Vertigo is usually the
prominent symptom
a sensation of aural fullness or pressure is now recognised as being common
other features include nystagmus and a positive Romberg test
episodes last minutes to hours
typically symptoms are unilateral but bilateral symptoms may develop after a number of years

Natural history
symptoms resolve in the majority of patients after 5-10 years
the majority of patients will be left with a degree of hearing loss
psychological distress is common

Management
ENT assessment is required to confirm the diagnosis
patients should inform the DVLA. The current advice is to cease driving until satisfactory control of
symptoms is achieved
acute attacks: buccal or intramuscular prochlorperazine. Admission is sometimes required
prevention: betahistine may be of benefit

657. An 82yo woman has developed painful rash on one side of her forehead and ant scalp. Lesions
have also affected her cornea. What is the single most appropriate option?
a. Accessory nerve
b. Facial nerve
c. Olfactory nerve
d. Optic nerve
e. Trigeminal nerve

e. Trigeminal nerve
Corneal involvement. Forehead n scalp. All are area of supply of trigeminal

658. A 24yo woman presents with episodes of peri-oral tingling and carpo pedal spasms every time
she has to give a public talk. This also happens to her before interviews, exams and after
arguments. What is the best management strategy for this pt?
a. Diazepam
b. Rebreathe in a paper bag
c. Desensitization
d. Buspirone
e. Propranolol

b. Rebreathe in a paper bag??? many confusing answers


She has co2 washout which results in hypocalcemia hence the Peri oral tingling and carpo pedal spasms
Desensitization is for phobias...like arachnophobia
buspirone is for smoking cessation
For acute attack.. Rebreathe into paper bag
For prophylaxis just like when a pt has to give a public talk or appear in an interview..beta blocker.
.propanolol.
Best Mx is CBT. Desensitization
If CBT doesn't help we go for medical Mx.. SSRI

questions mentions all the events she is worried about and get symptoms from.
What I think is that this is panic disorder which starts at the time public speaking and ends after its over. As
opposed to GAD which is persistent and long lasting.
And if we go according to management of panic attack. Step 1 is education
Step 2 is cbt or ssri.
So i think in this case cbt is the best answer.
If it would be that the patient needs something to calm down while an exam then propranolol would be best. I
mean for one time event.
And if it would be acute and first time episode then rebreathing would be way to go.

659. A 32yo woman P3 of 39wks gestation reports having spontaneous ROM 4days ago. She didnt attend the
delivery suite as she knew that would happen and had already decided on a home
birth. Today she feels very hot and sweaty. She thought that she was starting to have labour
pains but she describes the pain as more constant. Exam: uterus is tender throughout. Blood
tests show raised CRP and WBC. Select the most likely dx?
a. Round ligament stretching
b. Chorioamnionitis
c. Uterine rupture
d. Labor
e. DIC

b. Chorioamnionitis

The characteristic clinical signs and symptoms of chorioamnionitis include the following:
Maternal fever (intrapartum temperature >100.4F or >37.8C): Most frequently observed sign
Significant maternal tachycardia (>120 beats/min)
Fetal tachycardia (>160-180 beats/min)
Purulent or foul-smelling amniotic fluid or vaginal discharge
Uterine tenderness
Maternal leukocytosis (total blood leukocyte count >15,000-18,000 cells/L)

The standard drug treatment in the mother with chorioamnionitis includes ampicillin and an aminoglycoside
(ie, usually gentamicin), although clindamycin may be added for anaerobic pathogens. Clindamycin may also
be used if the mother is allergic to penicillin

660. A 63yo man continues to experience chest pain and has a temp of 37.8C 2 days after an acute
MI. His ECG shows widespread ST elevation with upward concavity. What is the single most
likely explanation for the abnormal inv?
a. Acute pericarditis
b. Cardiac tamponade
c. Atrial thrombus
d. Left ventricular aneurysm
e. Dressler syndrome

a. Acute pericarditis
Pericarditis in the first 48 hours following a transmural MI is common

Features
chest pain: may be pleuritic. Is often relieved by sitting forwards
other symptoms include non-productive cough, dyspnoea and flu-like symptoms
pericardial rub
tachypnoea
tachycardia

Causes
viral infections (Coxsackie)
tuberculosis
uraemia (causes 'fibrinous' pericarditis)
trauma
post-myocardial infarction, Dressler's syndrome
connective tissue disease
hypothyroidism
ECG changes
widespread 'saddle-shaped' ST elevation
PR depression: most specific ECG marker for pericarditis

Dressler's syndrome tends to occur around 2-6 weeks following a MI. The underlying pathophysiology is
thought to be an autoimmune reaction against antigenic proteins formed as the myocardium recovers. It is
characterised by a combination of fever, pleuritic pain, pericardial effusion and a raised ESR. It is treated
with NSAIDs.

661. A 55yo man presents with an ulcer of the scrotum. Which of the following LN is involved?
a. External iliac LN
b. Pre-aortic LN
c. Aortic LN
d. Inguinal LN
e. Iliac LN
f. Submental LN
g. Submandibular LN
h. Deep cervical LN

D. Inguinal LN
662. A 35yo woman has butterfly rash on her face and she suffers symmetrical joint pains on knee
and elbow, ESR is raised. What is the most discriminative inv for dx?
a. Anti DNA antibodies
b. Anti Jo1 antibodies
c. Anti nuclear antibodies
d. Anti centromere antibodies
e. Anti la antibodies

a. Anti DNA antibodies


in diagnostic critera for SLE,we have four antibody.Among them most specific is anti double stranded DNA
antibody.So answer is a.

Immunology
99% are ANA positive
20% are rheumatoid factor positive
anti-dsDNA: highly specific (> 99%), but less sensitive (70%)
anti-Smith: most specific (> 99%), sensitivity (30%)
also: anti-U1 RNP, SS-A (anti-Ro) and SS-B (anti-La)

Monitoring
ESR: during active disease the CRP is characteristically normal - a raised CRP may indicate underlying
infection
complement levels (C3, C4) are low during active disease (formation of complexes leads to
consumption of complement)
anti-dsDNA titres can be used for disease monitoring (but note not present in all patients)

663. Pt had a fight following which he developed bleeding, ringing and hearing loss from one ear.
What is the inv of choice?
a. CT
b. XR skull
c. Otoscopy
d. MRI vestibule
e. Coagulation study

a. CT
CT scan to rule out basilar skull fracture esp when there is history of fight, bleeding from ear
(hemotympanum)
If bleeding and tinnitus is present it is almost certain that the patient has a traumatic perforation...
So now our main aim is to rule out fracture base of skull which can be best done by a CT scan.

Selection of adults for CT scan


CT scan of the brain within one hour (with a written radiology report within one hour of the scan being
undertaken):
Glasgow Coma Scale (GCS) <13 when first assessed or GCS <15 two hours after injury
Suspected open or depressed skull fracture
Signs of base of skull fracture*
Post-traumatic seizure
Focal neurological deficit
>1 episode of vomiting
All patients with a coagulopathy or on oral anticoagulants should have a CT brain scan within eight hours
of the injury, provided there are no other identified risk factors, as listed above. Again, a written radiology
report should be available within one hour of the scan being undertaken.

664. A 35yo IVDA on penicillin and flucloxacillin for cellulitis now presents with jaundice, pale stools
and dark urine. What is the single most likely dx?
a. Hep A
b. Cholestatic jaundice
c. Chronic active hepatitis
d. Primary biliary cirrhosis
e. Hep B

b. Cholestatic jaundice
Cholestatic jaundice associated with flucloxacillin therapy.
Although it's a vague scenario, the diagnosis of drug induced hepatic injury ( DIHI ) requires these attributes 1)
history of drug exposure 2) exclusion of other causes of hepatic injury , like hepatitis etc 3) improvement after
stopping the suspected drug 4) exacerbation/recurrence after the use of offending drug
Pale stool = cholestasis

665. A 79yo woman has been dx with T2DM. Her BMI=22. RBS are 8 and 10mmol/l. Her
BP=130/80mmHg. Her fasting cholesterol=5.7mmol/l. She is currently symptom-free but has
microalbuminuria. What is the single most appropriate drug management?
a. ACEi and glibenclamide
b. ACEi and metformin
c. Statin and ACEi
d. Statin and glibenclamide
e. Statin and metformin

c. Statin and ACEi


Statins with acei to decrease cholesterol and prevent protinurea.
We are giving ACE I because of microalbuminuria. And Statins because of high cholesterol. You never start off
with oral hypoglycemics straightaway. Her BMI is ok. Lifestyle modifications n exercise would be advised
if microalbuminuria is present which is a microvascular complication- nephropathy, v add ACEIs/sartan to
inhibit renin angiotensin system to protect kidneys even if BP is normal ! plus statin here for raised chol levels

ACE inhibitors are also used to treat diabetic nephropathy and have a role in secondary prevention of
ischaemic heart disease.

People with IFG should then be offered an oral glucose tolerance test to rule out a diagnosis of diabetes. A
result below 11.1 mmol/l but above 7.8 mmol/l indicates that the person doesn't have diabetes but does have
IGT.'

The diagnosis of type 2 diabetes mellitus can be made by either a plasma glucose or a HbA1c sample.
Diagnostic criteria vary according to whether the patient is symptomatic (polyuria, polydipsia etc) or not.

If the patient is symptomatic:


fasting glucose greater than or equal to 7.0 mmol/l
random glucose greater than or equal to 11.1 mmol/l (or after 75g oral glucose tolerance test)

666. A 68yo woman is unable to extend the IP joint of her right thumb 7wks following a fx of the right radius.
Other finger and thumb movements are normal. What is the single most likely tendon to be damaged?
a. Abductor pollicis longus
b. Extensor pollicis brevis
c. Extensor pollicis longus
d. Flexor digitorum profundus
e. Flexor pollicis longus

C. Extensor pollicis longus


Extensor pollicis brevis is inserted at the proximal phalanx of thumb meaning at MCP joint. Its function is to
extend MCP of thumb. However extensor pollicis longus is inserted at distal phalanx of thumb so it causes
extension at Interphalangeal joint and also at MCP joint of thumb.

667. A mother presents her 6m son who is vocalizing. She has noticed that he doesnt respond to
loud noises. His motor milestones are normal. What is the best management strategy?
a. Arrange hearing test
b. Assess development milestones
c. Reassure
d. Refer to speech therapist
e. MRI brain

a. Arrange hearing test


3 months Quietens to parents voice
Turns towards sound
Squeals

668. A 39yo man presents to the ED with persistent cough, sputum and dyspnea. He gave a hx of
smoking 20 cigarettes/d for the last 10 years. Pt was given oxygen in ambulance but he is not
improving. What is the next step?
a. Prednisolone
b. Salbutamol
c. Check ABG
d. CXR
e. ECG

c. Check ABG
to assess hypoxia and co2 levels. it seems like this man has a chest infection. Salbutamol would be more
appropriate if they mentioned wheezy chest
COPD,with type two respiratory failure, patient is not getting better O2, so ABGs to proceed further to confirm
dx and treat.

>> whenever you initiate or change oxygen therapy, do an ABG within the next hour or sooner, if the patient is
deteriorating.

NICE guidelines from 2010 recommend the following:


increase frequency of bronchodilator use and consider giving via a nebuliser
give prednisolone 30 mg daily for 7-14 days
it is common practice for all patients with an exacerbation of COPD to receive antibiotics. NICE do not
support this approach. They recommend giving oral antibiotics 'if sputum is purulent or there are
clinical signs of pneumonia'

669. A 66yo woman has been brought to the hospital on a number of occasions with a hx of loss of memory.
Her PMH is significant for a MI 6yrs ago. It is noted that she has a step wise decline of
her cognitive functions. What is the most likely dx?
a. Alzhemiers
b. Vascular dementia
c. Picks dementia
d. Huntingtons disease
e. Lewy body dementia

b. Vascular dementia
step-wise is a clincher for vascular
History of multiple loss of memory is vascular dementia. She also has vascular problems indicates by MI. Step
wise decline is characteeistic of it as well.
Step laddar detoriation of cognitive function is characteristic feature of vascular dementia which is also
supported by the h/o MI, a vascular condition.
Alzheimers ussually has an insidious onset
Step wise deterorition in memory >h/o multiple TIAs> vascular
Gradual >alZihemer
Fluctuating > features of parkinsonism> lewy body dementia
Associted with depression >pseudo dementia
Strong family history>hungtington's
Impaired cognition> memory is usually normal> pick's dementia

670. A 55yo man returns for routine follow up 6wks after a MI. He gets breathless when walking
uphill. His ECG shows ST elevation in leads V1, V2, V3 and V4. What is the single most likely
explanation for the abnormal investigation?
a. Heart block
b. Right ventricular strain
c. Atrial thrombus
d. Left ventricular aneurysm
e. Dresslers syndrome

d. Left ventricular aneurysm


S t elevation 6 weeks after mi...with no other major symptoms... Also a continuous St elevation and a history
of mi points towards an aneurysm
persistent ST elevation after few months of acute MI,D/D-1.ventricular aneurysm 2.underkinetic wall motion
disorder. N.B -contrary to sounding fatal,there is neither predisposition nor any association of cardiac rupture
in ventricular aneurysm.
Ventricular aneurysm occurs at the site of previous STEMI. V1-V4 involvement indicates previous anteroseptal
MI ( caused by LAD obstruction ) . This localized involvement of leads almost excludes Dressler's syndrome
where pericarditis causes ST elevation in all but aVR leads.

Left ventricular aneurysm


The ischaemic damage sustained may weaken the myocardium resulting in aneurysm formation. This is
typically associated with persistent ST elevation and left ventricular failure. Thrombus may form within the
aneurysm increasing the risk of stroke. Patients are therefore anticoagulated.

Dressler's syndrome tends to occur around 2-6 weeks following a MI. The underlying pathophysiology is
thought to be an autoimmune reaction against antigenic proteins formed as the myocardium recovers. It is
characterised by a combination of fever, pleuritic pain, pericardial effusion and a raised ESR. It is treated with
NSAIDs.

671. A 4m girl has severe FTT and increasing jaundice which was 1st noticed at 1wk of age. She has an
enlarged liver and scratches on her skin. Her parents have been unable to seek medical care.
What is the most likely dx?
a. Biliary atresia
b. G6PD deficiency
c. Hep B
d. Spherocytosis

a. Biliary atresia
gradual increasing of jaundice with obstructive jaundice indicate this. so USG to confirm this.G6PD def is
hereditary disease of XLR inheritance and always presented when aggravating factor present (some
medication) and otherwise asymptomatic. congenital spherocytosis not presented with this early stage upto
development of gallstone due to haemolysis. hepB not possible due to it causes hepatocellular picture.

Galactosemia: Poor weight gain, poor feeding, irritable, jaundice.


B. Atresia: Jaundice noticed usually at 1 wk, progressive, pale stools, dark urine.
G6PD Def: Jaundice comes usu after/ during illness, off and on.

672. A 76yo man suddenly collapsed and died. At post mortem exam, a retroperitoneal hematoma
due to ruptured aortic aneurysm was noted. What is the most likely underlying cause of the
aortic aneurysm?
a. Atheroma
b. Cystic medial necrosis
c. Dissecting aneurysm
d. Polyarteritis nodosa
e. Syphilis

a. Atheroma

risk factors include:

Severe atherosclerotic damage of the aortic wall; however, new evidence suggests this is not the
only factor, and aneurysmal disease is probably a distinct arterial pathology.
Family history - there are probably strong genetic factors. About 15% of first-degree relatives of a
patient with an AAA, mainly men, will develop an aneurysm.[6]
Tobacco smoking is an important factor.
Male sex.
Increasing age.
Hypertension.
Chronic obstructive pulmonary disease.
Hyperlipidaemia.
In population-based studies, people with diabetes have a lower incidence of aneurysms than
people without diabetes.

673. A 33yo male came to the hospital with complaint of occasional left sided chest pain that lasts
<30mins, following exercise, which relieves upon taking rest. What is the most probable dx?
a. Unstable angina
b. Decubitus angina
c. Stable angina
d. Coronary spasm
e. MI

c. Stable angina

Stable angina is when the pain is precipitated by predictable factors - usually exercise.
Unstable angina: angina occurs at any time and should be considered and managed as a form of
acute coronary syndrome.
Decubitus angina occurs when the patient lies down. It is usually a complication of cardiac failure due to the
strain on the heart resulting from the increased intravascular volume.
Patients usually have severe coronary artery disease.

A full 12-lead ECG may show some ischaemic changes but a normal ECG does not rule out a diagnosis of
angina. Changes on a resting 12-lead ECG that are consistent with CAD include:
Pathological Q waves.
Left bundle branch block (LBBB).
ST-segment and T-wave abnormalities (eg flattening or inversion).

when an attack of angina occurs, they should:


Stop what they are doing and rest.
Use glyceryl trinitrate (GTN) spray or tablets as instructed.
Take a second dose of GTN after 5 minutes if the pain has not eased.
Take a third dose of GTN after a further 5 minutes if the pain has still not eased.
Call 999/112/911 for an ambulance if the pain has not eased after another 5 minutes (ie 15
minutes after onset of pain), or earlier if the pain is intensifying or the person is unwell.

Medication
all patients should receive aspirin and a statin in the absence of any contraindication
sublingual glyceryl trinitrate to abort angina attacks
NICE recommend using either a beta-blocker or a calcium channel blocker first-line based on
'comorbidities, contraindications and the person's preference'
if a calcium channel blocker is used as monotherapy a rate-limiting one such as verapamil or diltiazem
should be used. If used in combination with a beta-blocker then use a long-acting dihydropyridine
calcium-channel blocker (e.g. modified-release nifedipine). Remember that beta-blockers should not
be prescribed concurrently with verapamil (risk of complete heart block)
if there is a poor response to initial treatment then medication should be increased to the maximum
tolerated dose (e.g. for atenolol 100mg od)
if a patient is still symptomatic after monotherapy with a beta-blocker add a calcium channel blocker
and vice versa
if a patient is on monotherapy and cannot tolerate the addition of a calcium channel blocker or a beta-
blocker then consider one of the following drugs: a long-acting nitrate, ivabradine, nicorandil or
ranolazine
if a patient is taking both a beta-blocker and a calcium-channel blocker then only add a third drug
whilst a patient is awaiting assessment for PCI or CABG

674. A 42yo overweight smoker comes with heavy periods. A scan reveals a normal uterus. She
would like a long term tx with minimal side effects that would offer tx for the menorrhagia and
provide contraception. She is unsure whether she would like more children. She is adamant that
she doesnt want surgery as she is terrified of the prospect. Select the best management for her
menorrhagia?
a. COCP
b. GrH analogues
c. IU/systemic progesterone
d. NSAIDs
e. Copper containing IUCD

c. IU/systemic progesterone
IUS. mirena coil. it provides relief of the menorrhagia and provides long term contraception for up to 5 years.
As pt is smoker COCP is not suitable and patient has menorrhagia. In this case C is better option i think.
Copper containing IUCD is not indicated in unexplained/undiagnosed vaginanl bleeding.
Gnrh analogues mostly used for menorehagia n not contracep

Menorrhagia was previously defined as total blood loss > 80 ml per menses, but it is obviously difficult to
quantify.

Does not require contraception


either mefenamic acid 500 mg tds (particularly if there is dysmenorrhoea as well) or tranexamic acid 1
g tds. Both are started on the first day of the period
if no improvement then try other drug whilst awaiting referral

Requires contraception, options include


intrauterine system (Mirena) should be considered first-line
combined oral contraceptive pill
long-acting progestogens

Norethisterone 5 mg tds can be used as a short-term option to rapidly stop heavy menstrual bleeding.

675. A 10yo male child was brought by his mother complaining that her child watches TV at very high volumes,
doesnt like to play outside and instead has become more sincere with reading. She
also says that her son doesnt respond to her. What do you expect to see on otoscopy?
a. Flamingo pink tympanic membrane
b. Attic perforation
c. Bluish grey tympanic membrane with air fluid levels
d. Inflamed tympanic membrane with cart wheel appearance of vessels
e. Red and inflamed tympanic membrane

c. Bluish grey tympanic membrane with air fluid levels


serous otitis media or OM with effusion...often first presenting symptom is watching tv at high volume
676. A lady underwent debulking surgery for ovarian carcinoma. Soon after the surgery she presents
with signs of intestinal obstruction. What is the single most appropriate inv?
a. Pelvic CT
b. CA 125
c. Laparotomy
d. Laparoscopy
e. Abdominal US

c. Laparotomy?
Abdominal CT then Laparotomy, while abdominal CT is not mentioned here so I will choose laparotomy.
since the patient has stage 4 cancer, further management with the patient is focused on palliative care. with
intestinal obstruction, laparotomy would be able to locate the level of obstruction, and place a colostomy/
ileostomy to relieve the obstruction
Laparotomy.
Or CECT abdomen which is not in the choice.
Better never open without imaging.
In Ca Ovary, Omental cake is one of the biggest challenge which leads to recurrent obstruction.
And also loculated ascites.

677. A 45yo woman undergoing tx for RA for the last 5yrs presents with dizziness, easy fatigabiliy and lack of
energy. A blood film shows MCV 106. What could be the most probable reason for her
anemia?
a. Steroids
b. Chronic disease
c. NSAIDs
d. Methotrexate
e. Sulfasalazine

d. Methotrexate
due to def of folate megaloblastic anemia.as methotrexate is an anti folate drug
methotrexate cause many side effects including macrocytic anemia, liver & pulmonary toxicity, BM supp.
,stomatitis, rash, nausea and alopecia ..
Anaemia of chronic disease is either normocytic normochronic or micro hypo

-MTX & sulfasalazine can both give rise to high-MCV anemia, MTX by anti-folate effect & salfasalazine by
hemolytic process which result in high reticulocyte count (which are large cells) and hence high MCV
values...but statistically speaking, MTX anemia & myelosuppression is more common that salfasalazine-
induced hemolytic anemia (about 10 times).

-NSAID & steroids can cause microcytic anemia (low MCV, low ferritin & iron,high TIBC and high soluble
transferrin receptor) due to GI bleeding and iron deficiency.

-Long standing RA can cause also anemia of chonic disease (low MCV,high ferritin, low iron,low TIBC and low
soluble transferrin receptor).

-Miscellaneous causes of anemia in RA includes: GI bleeding due to vasculitis, Felty syndrome, non-hodgkin
lymphoma (T-large granular lymphocyte lymphoma), all of which are microcytic (i.e low MCV).

Antifolate drugs:
With uncertain mechanism of action (e,g anticonvulsants and possibly alcohol and nitrofurantoin)
Causing malabsorption of folate (eg, cholestyramine, sulfasalazine, methotrexate)
Trimethoprim may exacerbate pre-existing folate deficiency but does not cause megaloblastic
anaemia

678. A 62yo man who has recently had flu-like illness has woken to find his speech altered.
Movement of his eyelids and lips are weak on the right side. Exam: normal. Which anatomical
site is most likely to be affected?
a. Facial nerve
b. Hypoglossal nerve
c. Oculomotor nerve
d. Trigeminal nerve
e. Glossopharyngeal nerve

a. Facial nerve
bell's palsy

679. A 5yo girl has had an URTI for 3 days and has been treated with paracetamol by her mother. For the last
12h she has been hot and irritable with severe pain in her right ear. What is the most
likely dx?
a. Herpes zoster infection
b. Impacted ear wax
c. Mumps
d. OM
e. Perforation of eardrum

d. OM
AOM is a condition in which there is inflammation of the middle ear, frequently in association with an upper
respiratory tract infection (URTI). It commonly presents with:

Pain
Malaise
Irritability
Fever
Vomiting
The fever is often very high and may be associated with febrile convulsions.

A well-recognised complication is that a child who is screaming and in a great deal of pain finally settles and
the ear starts to discharge green pus. The eardrum has burst, releasing the pressure and relieving the pain.

The Health Protection Agency recommends a five-day course of amoxicillin or erythromycin for patients
allergic to penicillin

680. A 35yo man has a temp=39C, cough with purulent sputum and right sided chest pain on
inspiration. He has herpes labialis. What is the single most likely causative organism?
a. Coagulase +ve cocci in sputum
b. Gram -ve diplococci in sputum
c. Gram +ve diplococci in sputum
d. Pneumocystis carinii in sputum
e. Serology for legionella

c. Gram +ve diplococci in sputum


Pneumococcus is the most common , and presents with herpes labialis apart from other symptoms
Herpes labialis is a benign viral infection. If this is associated with a pneumonia, it is usually a pneumococcal
pneumonia. It is a self limiting condition that RESOLVES in 10-15 days.
which is Streptococcus pneumoniae (Pneumococcus) which is coagulase & catalase (-)ve, alpha hemolytic
Gram (+)ve diplococci (Inulin, bile fermentation & optochin sensitivity distinguishes it from Viridans group
which is quite similar and is out of the scope of the question).

The key to answer is again "statistical" one (i.e the most common cause of community-acquired pneumonia in
this age group).

The herpes labialis is often associated with streptococcal pneumonia which is another key feature.

Pneumocystis carinii would cause a noticeable dyspnea in an immunosuppressed individual or known case of
HIV with CD4 count less than 200.

Legionella would cause diarrhea, altered level of consciousness & SIADH (hyponatremia) in an elderly.

Coagulase (+)ve is staphylococcal group in which S.aureus genus is responsible for a pneumonia following
influenza in a debilitated elderly patient which is multilobular, pneumatocele, abscesses and necrotizing with
dissemination, endocarditis and osteomyelitis.

681. A 27yo female was brought to the ED by her friend from a movie theatre. She complains of
sudden severe pain in the eye followed by vomiting and also was seeing colored halos. She gives
a past hx of recurrent headaches which used to resolve spontaneously. Exam: fixed, dilated
ovoid pupil seen. What is the first inv?
a. CT head
b. MRI orbits
c. Blood culture and sensitivity
d. Toxicology screen
e. Applanation tonometry

e. Applanation tonometry
With Halos with eye pain acute angle closure glaucoma should be rule out. So option should be E. Here in
theatre(usually dark) , her pupil have dilated and aggravated the symptom of glaucoma.

movie theatre --->lights off ----> pupil fully dilated----> angle closed--->obstruction of fluid flow from anterior
chamber---> increased pressure in anterior chamber---> AACG

Diagnosis is Acute Angle Closure glaucoma. It is based on the finding of two symptoms of ocular pain,
nausea/vomiting, and a history of intermittent blurring of vision with haloes and at least three signs of the
following: IOP greater than 21 mm Hg, conjunctival injection, corneal epithelial oedema, mid-dilated non-
reactive pupil and shallower chamber in the presence of occlusion.

Features
severe pain: may be ocular or headache
decreased visual acuity
symptoms worse with mydriasis (e.g. watching TV in a dark room)
hard, red eye
haloes around lights
semi-dilated non-reacting pupil
corneal oedema results in dull or hazy cornea
systemic upset may be seen, such as nausea and vomiting and even abdominal pain

Management
urgent referral to an ophthalmologist
management options include reducing aqueous secretions with acetazolamide and inducing pupillary
constriction with topical pilocarpine

682. An 82yo male suddenly becomes unconscious and fell down. He recovered completely within
minutes. What is the best inv you to to dx the case?
a. ECG
b. EEG
c. Blood glucose level
d. CT
e. CXR

a. ECG
in old age arrhythmias are common like stokes adam..so ecg
Sudden loss of consciousness with sudden gain is always cardiac until proved otherwise (except in diabetics )
The immediate recovery is the trick. If it was hypoglycemia or head injury, the patient would not wake up
quickly. The only condition where the patient gets loss of conscious and recover quickly is in cardiac conditions
If hypoglycemia recovery without glucose would be difficult. There would be some other features like
palpitation sweating etc.
For exam purposes.. A pt in hypo won't recover until glucose is administered

A classic Stokes-Adams attack is a collapse without warning, associated with loss of consciousness for a few
seconds. Typically, complete (third-degree) heart block is seen on the ECG during an attack (but other ECG
abnormalities such as tachy-brady syndrome have been reported)
683. A child admitted with progressive muscle weakness and frequent falls. What is the most
probable dx?
a. Duchennes MD
b. Beckers MD
c. Polymyositis
d. Dermatomyositis
e. Polymyalgia rheumatic

a. Duchennes MD
Duchenne is an x linked recessive disorder where muscle degeneration occurs and present at a very early age (
before 6 yrs usually) with proximal limb and pelvis muscle weakness..later progressing to the arms etc.
Beckers is same as duchenne but a much milder form so present at a later age.
Duchene ...starts early in life 5-6 years.
B...starts later after 10 years
b is almost same but less severe,c and d are inflammatory but there is rash involved in d..and e is entirely
different related to giant cell arteritis

There is progressive proximal muscular dystrophy with characteristic pseudohypertrophy of the calves. All
patients have symptoms by age 3 years, but diagnosis is often delayed. Presenting features are:

Motor milestones delayed


Inability to run - waddling gait when attempting to do so
Other gait signs - no spring in the step, cannot hop or jump; toe walking; falls
Gower's sign - 'climbing up legs' using the hands when rising from the floor
Hypertrophy of calf muscles (and possibly other muscles too, including the deltoid, quadriceps,
tongue and masseters)

Non-locomotor presenting symptoms:

Speech delay or global developmental delay


Failure to thrive
Abnormal LFTs (raised AST or ALT)
Anaesthetic complications - eg, myoglobinuria, rhabdomyolysis or malignant hyperthermia after
certain anaesthetics
Fatigue - this is common

The initial investigation is serum CK


The precise diagnosis is best achieved by a combination of:
Genetic analysis - can identify most (but not all) of the DMD mutations.
Muscle biopsy - with assay for dystrophin protein.
Clinical observation of muscle strength and function.

684. A 56yo man presents to the ED with chest pain. The following ECG was taken. What is the most likely dx?
a. Anterior MI
b. Inferior MI
c. Lateral MI
d. Posterior MI
e. NSTEMI

a. Anterior MI

ECG changes Coronary artery

Anteroseptal V1-V4 Left anterior descending

Inferior II, III, aVF Right coronary

Anterolateral V4-6, I, aVL Left anterior descending or left circumflex

Lateral I, aVL +/- V5-6 Left circumflex

Posterior Tall R waves V1-2 Usually left circumflex, also right coronary
685. A schizophrenic says: life is unfair. I like fairs. Fairs have food. It must be good. What term
describes this pts speech?
a. Neologism
b. Flight of ideas
c. Brocas aphasia
d. Wernickes aphasia
e. Clang association

e. Clang association
rhytmic speech is Clang association
association of words based upon sound rather than concepts
Neologism: is the name for a relatively new or isolated term, word, or phrase that may be in the process of
entering common use, but that has not yet been accepted into mainstream language

686. A man comes to the ED with hx of pulsatile swelling in the abdomen, he has hx of HTN and exam:
pulse=120bpm, BP=70/40mmHg. He is restless and in shock. What emergency management should be done
on priority basis?
a. Urgent abdominal CT
b. Urgent abdominal US
c. IV fluids 0.9% NS crystalloids to bring SBP to 90mmHg
d. IV fluids 0.9% NS crystalloids to bring SBP to 120mmHg
e. Dopamine inj

c. IV fluids 0.9% NS crystalloids to bring SBP to 90 mmHg


Aortic aneurism rupture with hypovolemic shock.. first step is to give crystalloids for resus to bring the systolic
bp up to a min of 90
our target sbp should be <100 here bcoz he's hypertensive and the autoregulatory mech in chronic HTN wl
maintain higher bp once sbp is corrected to 90...again if sbp is corrected to higher levels by crystalloids, it may
increase preload and predispose to MI, CVA in AAA rupture

C, then A, then surgery

687. A 5yo boy has cough and swelling at the knee after falling on the ground with rashes on the
buttocks which are non-blanching. PT=13, APTT=71, Hgb=11, WBC=8, Plt=200. Choose the most likely dx?
a. NAI
b. Hemophilia
c. HSP
d. Osler weber rendu syndrome
e. Von-Willebrand disease

b. Hemophilia
APTT is prolonged
haemarthrosis is a feature of hemophilia
there is also prolonged in aptt which indicates that there is something wrong with internal pathway of
coagulation process. This is true from hemophilia because the factor 8 deficiency is affecting the internal
pathway
HSP purpura is also non blanchable though mostly present on buttocks and limbs..... but here APTT IS
RAISED...in HSP the cause is vasculitis due IGA deficiency....so diagnosis is Haemophilia.aided further by
joint swelling. In hsp it is arthritis and arthralgia.
Labs of HSP :
ESR raised ( as it is vasculitis )
IgA raised
Proteinuria
Nothing happens to PT , APTT
As they are significantly deranged in this scenario suggesting Haemophilia.

Haemophilia A is due to a deficiency of factor VIII whilst in haemophilia B (Christmas disease) there is a lack of
factor IX

Features
haemarthrosis, haematomas
prolonged bleeding after surgery or trauma

Blood tests
prolonged APTT
bleeding time, thrombin time, prothrombin time normal

688. A 45yo woman presents with discharge from the left nipple. The discharge is brownish-green
and foul smelling. What is the most likely dx?
a. Duct papilloma
b. Intra-ductal papilloma
c. Duct ectasia
d. Mammary duct fistula
e. Breast abscess

c. Duct ectasia
brownish-green
and foul smelling

689. A 10yo boy presents with generalized swelling. This has been present for 4days and included
swollen ankles and puffiness of the face. It started a few days after he had a mild cold with
runny nose. His only PMH was eczema. Urine analysis: hematuria, proteinuria 10g/24h, creat
60umol/l and albumin=15g/l. What is the single most likely dx?
a. IgA nephropathy
b. HSP
c. Minimal change nephropathy
d. Wilsons disease
e. Cardiac failure

a. IgA nephropathy
the typical presentation of nephritic syndrome as well as we know the patient upper respiratory tract infection
then it lead to immune response
no hematuria in minimal change
HSP presents typically with the following JAKS joints abdomen kidney and skin
arthritis, abdominal pain, haematuria and petechiae
In hsp there is purpura on extensor surfaces, abdominal pain and nephritis
iga nephropathy after 2-3 days of cold infection, post strep glomerulonephritis after 1-12wks of URTI

690. A 28yo man complains of vertigo, nausea and vomiting for more than 30 mins and tinnitus,
hearing loss in the left ear. What is the tx for this pt?
a. Buccal prochlorperazine
b. Metachlorpromide
c. Cyclazine
d. Cotrimazole
e. Ondansetron

a. Buccal prochlorperazine
1st line-if vomiting
or betahistine
or chlorthalidone (OHCS)

691. A 16yo girl has been unwell for 5days with malaise, headache and dry cough. She has a few
crackles in her chest. Her CXR shows patchy consolidation in the lower lobes. What is the single
most likely causative organism?
a. Cold agglutinins
b. Gram ve diplococci in sputum
c. Gram +ve diplococcic in sputum
d. Serology for legionella
e. Sputum staining for mycobacterium TB

a. Cold agglutinins
Mycoplasma
malaise, headache, dry cough n patchy consolidation all go i favour of mycoplasma....legionella shows bibasal
consolidation
patchy consolidation and the features go in favour of mycoplasma and since mycoplasma can casue
autoimmune hemolytic anemia we go for testing cold agglutinins
dry cough is either mycoplasma or legionella (patchy consolidation : mycoplasma +no history of travel ) so cold
agglutination . treatment clarythro or doxy

Mycoplasma pneumoniae is a cause of atypical pneumonia which often affects younger patients. It is
associated with a number of characteristic complications such as erythema multiforme and cold autoimmune
haemolytic anaemia.

Features
the disease typically has a prolonged and gradual onset
flu-like symptoms classically precede a dry cough
bilateral consolidation on x-ray
complications may occur as below

Complications
cold agglutins (IgM) may cause an haemolytic anaemia, thrombocytopenia
erythema multiforme, erythema nodosum
meningoencephalitis, Guillain-Barre syndrome
bullous myringitis: painful vesicles on the tympanic membrane
pericarditis/myocarditis
gastrointestinal: hepatitis, pancreatitis
renal: acute glomerulonephritis

Investigations
diagnosis is generally by Mycoplasma serology
positive cold agglutination test

Management
erythromycin/clarithromycin
tetracyclines such as doxycycline are an alternative

692. A child with increasing jaundice and pale stools. Choose the appropriate test:
a. Endomyseal antibodies
b. Sweat test
c. TFT
d. LFT
e. US

e. US
Biliary atresia. Us abdomen
US abdomen. [This is a picture suggestive of obstructive jaundice. LFT can give clue like much raised bilirubin,
AST and ALT not that high and raised alkaline phosphatase but still USG is diagnostic in case of obstructive
jaundice].
clinically pale stool, dark urine and itching is suggestive of obstructive jaundice, high bilirubin, slight rise in ast
and alt but alp is very high. bilirubin is predominantly conjugated

693. A 73yo woman with RA is unable to extend the fingers of her right hand at the MCP joint and IP joints
following a fall. What is the single most likely tendon to have been damaged?
a. Extensor carpi radialis
b. Extensor carpi ulnaris
c. Extensor digitorum
d. Extensor indicis
e. Flexor digitorum profundum

c. Extensor digitorum
Extensor digitorum makes an extensor expansion and is inserted in lateral four fingers on the dorsal aspect. It
gives off 3 slips. Main attachment is at dorsal side of the base of proximal phalanx. Middle slip is attached on
the sides middle phalanx and lateral slips at the sides of distal phalanx. Pt is unable to extend both the MCP
and IP joints of all fingers

694. You are called to see a 20yo woman 2h post-LSCS. She has not passed urine since her operation. She
denied any urinary symptoms preoperatively. Exam: appears unwell, temp=37.5C,
BP=94/73mmHg, pulse=116bpm, sat=97%. Her abdomen is distended with tenderness in the left
flank and suprapubic region. Bowel sounds are not audible. Choose the most appropriate post Csection
complication for this lady?
a. UTI
b. Urinary tract injury
c. Pleurisy
d. Acute pyelonephritis
e. Paralytic ileus

b. Urinary tract injury


it is one of the common complication of CS. here patient does not have urination. and all the gut symptoms is
because of release of urine from injured urether to the abdomen

695. A 58yo man has a headache and confusion of 3 days after slipping and hitting his head in the
garden. What is the most appropriate initial inv?
a. XR skull
b. XR face
c. CT brain
d. MRI brain
e. EEG

c. CT brain

Selection of adults for CT scan


CT scan of the brain within one hour (with a written radiology report within one hour of the scan being
undertaken):
Glasgow Coma Scale (GCS) <13 when first assessed or GCS <15 two hours after injury
Suspected open or depressed skull fracture
Signs of base of skull fracture*
Post-traumatic seizure
Focal neurological deficit
>1 episode of vomiting
All patients with a coagulopathy or on oral anticoagulants should have a CT brain scan within eight hours of
the injury, provided there are no other identified risk factors, as listed above. Again, a written radiology report
should be available within one hour of the scan being undertaken.
696. A 4yo boy has a cough and arthritis followed by rash on legs which are non-blanching on glass test. No hx
of fever. PT=13, APTT=31, Hgb=12, WBC=6.5, Plt=300. Whats the most likely dx?
a. Meningitis septicemia
b. Hemophilia
c. HSP
d. ITP
e. TTP

c. HSP
HSP henoch scholene pupura rashes on buttock plus arthritis
and age grp in children
Triad of upper respiratory or git infection with arthritis and non blanching rash.
aptt normal is HSP,APTT RAISED IS HEMOPHILIA
HSP,actually 4 systems involves-GIT,RENAL,SKIN AND JOINTS
triad of purpura, arthritis n abd pain mostly precipitated by a RTI ..purpura involving extensor surface of legs
and buttocks... kidney involvement later mostly presenting as hematuria
differentiate btw itp n hsp?
1.HSP is vasculitis with immune complex deposition in vessels /ITP is due to antibodies against platelet 2. HSP
normal or raised plt count /ITP low plt count 3.HSP specific distribution of rash, joint, kidney , GIT involved /ITP
bleeding through most orifices n other mucocutaneous sites... 4. HSP usually precipitated by infection /ITP
precipitated by infection, trauma etc purpura in ITP generalised

Features
palpable purpuric rash (with localized oedema) over buttocks and extensor surfaces of arms and legs
abdominal pain
polyarthritis
features of IgA nephropathy may occur e.g. haematuria, renal failure

Treatment
analgesia for arthralgia
treatment of nephropathy is generally supportive. There is inconsistent evidence for the use of steroids
and immunosuppressants

697. A 72yo man presents to the ED with chest pain. The following ECG was taken What is the most likely dx?
a. Anterior MI
b. Inferior MI
c. Lateral MI
d. Posterior MI
e. NSTEMI

b. Inferior MI

698. A young man has diarrhea, loss of weight and flatulence for 3 days. What is the most imp tx?
a. Metronidazole
b. Fluconazole
c. Vancomycin
d. Amoxicillin

a. Metronidazole
only suitable option for diarrhea

699. A 6yo child presented with drooling of saliva and severe stridor. He is febrile and sick looking. XR Neck in
extension shows a thumb sign. Choose the single most likely dx.
a. Croup
b. URTI
c. Diphtheria
d. Acute epiglottitis

d. Acute epiglottitis
most common in children between the ages of 2 and 8

The most common symptoms


Sore throat.
Odynophagia (painful swallowing).
Drooling (inability to swallow secretions).
Fever.
Anterior neck tenderness over the hyoid bone.

Fibre-optic laryngoscopy remains the 'gold standard' for diagnosing epiglottitis


radiograph of the neck may show the 'thumbprint sign'.

700. A mother presents with her 3yo son who has indistinct nasal speech. He snores at night and has restless
sleep. He is tired by day. What is the best management strategy?
a. Arrange hearing test
b. Assess development milestones
c. Refer to ENT surgeon
d. Refer to speech therapist
e. MRI brain

c. Refer to ENT surgeon


The symptoms points to adenoid hypertrophy so for confirmation refer to ENT surgeon
Swollen adenoids may not cause any symptoms or problems. However, symptoms may develop in some cases,
especially if the adenoids become very large. Possible symptoms include the following:
Breathing through the nose may be noisy or rattly. This may get worse and cause difficulty breathing
through the nose. The child then mainly mouth breathes.
A constantly runny nose.
Snoring at night. In severe cases sleep may be disrupted by the blocked nose and there is difficulty with
breathing.
Swollen adenoids may block the entrance of the Eustachian tube. This is the tube that goes from the
back of the nose to the middle ear. It normally allows air to get into the middle ear. If this tube is
blocked it may contribute to the formation of glue ear

701-730: Wajiha
731-765: Sonya Hashmi
766-790: Samrah
790-800: Murk Memon
701. A 17yo boy while playing football got a kick and now he is complaining of severe pain and swelling of
the left side of his scrotum. What inv is the most appropriate to dx?
a. Needle aspiration of scrotum
b. US scrotum
c. MSU
d. Surgical exploration of scrotum
e. Urine test for hematuria

Ans: D
Painful swelling after trauma in teenage boys indicates torsion of testis which is a surgical emergency.
If the blood supply to the testis is cut off for more than 6 hours can lead to permanent damage.
More accurately torsion of testis is called torsion of spermatic cord which causes occlusion of testicular blood
vessels
Occurs in one in 4,000 males under 25 years of age
Left side is more common
In patients who present within 24 hours of torsion, the preferred option is exploration of the scrotum,
detorsion and orchidopexy if the testis is viable.

US scrotum with Doppler will help in detection of presence/absence of intratesticular blood flow for the early
identification of testicular torsion

Urinalysis may be helpful in borderline cases, to exclude urine infection and epididymitis

702. A 50yo man has had hoarseness of voice and drooping eyelid for 2m. a mass is palpable in the right
supraclavicular fossa. He smokes 20 cigarettes/day for the last 30yrs. What is the most likely dx?
a. Carcinoma larynx
b. Carcinoma thyroid
c. Carcinoma right bronchus
d. Mesothelioma
e. Pancoast tumor

Drooping of eyelid points towards horners syndrome. Pancoast tumor can also lead to hoarsness of voice by
compressing the recurrent laryngeal nerve. There could be weakness of small muscles of the hand by
involvement of the C8-T1 of the brachial plexus.

703. An 84yo man got surgical pain which is well controlled by oral morphine 60mg BD. However, now this
pt is unable to swallow. What is the most appropriate next step?
a. Morphine 60mg state
b. Morphine 60mg TDS
c. Oxycodone 10mg OD
d. Morphine 60mg IV
e. Fentanyl patches
Fentanyl patches are used to relieve severe pain in people who are expected to need pain medication around
the clock for a long time and who cannot be treated with other medications. Fentanyl is in a class of
medications called opiate (narcotic) analgesics. It works by changing the way the brain and nervous system
respond to pain

Since the patient is pain free and we know dose of morphine to be given we can switch to fentanyl patches.
704. A 19yo man has exercise induced asthma. This has prv been controlled using a salbutamol inhaler as
req, but he now gets attacks with exercise. What is the single most appropriate tx?
a. Regular salbutamol
b. Regular salbutamol and budesonide
c. Sodium cromoglycate
d. Oral steroid
e. Inhaled steroid

Answer here should be B or E. Inhaled salbutamol + inhaled ICS are required.

Exercise-induced asthma
For most, exercise-induced asthma indicates poorly controlled asthma and will require regular inhaled steroid
treatment beyond the anticipatory use of a bronchodilator when preparing for sport. Where exercise poses a
particular problem and patients are already on inhaled corticosteroids, consider the addition of long-acting
beta agonists, leukotriene inhibitors, chromones, oral beta2 agonists or theophyllines.

705. A 3yo boy has a sudden onset of fever, vomiting and bilateral face swelling. Few days earlier the GP
saw him for bilateral parotid pain and gave analgesics. What is the most appropriate next step?
a. Analgesic
b. Antibiotic
c. Biopsy
d. Immediate surgery
e. Reassurance

This is a case of mumps. It can be unilateral or bilateral. Can cause distortion of the face due to swelling. Pain
is usually around the angle of the jaw. Fever can be as high as 39.5 and it can cause dryness of the mouth due
to blockage of the salivary ducts. No specific treatment is required. Just keep the patient hydrated and treat
symptomatically.

706. A 75yo man with adenocarcinoma of the prostate which has spread outside the capsule of the gland
has ARF. What is the most appropriate next inv?
a. MRI spine
b. Radionuclide bone scan
c. Trans rectal US
d. US pelvis
e. US KUB

This is to look for renal status and metastatic invasion.

707. A 57yo male presents with sudden onset severe abdominal pain and rigidity against a 4d background
of LIF pain and pyrexia. He has no PM/SHx of note and isnt on any meds. What is the most likely dx?
a. Intussusception
b. Ischemic colon
c. Sigmoid volvulus
d. Perforated diverticulum
e. Perforated meckels diverticulum
It is a case of peritonitis.
Pain in LIF points to one of the following:
Gastroenteritis (will have more of a generalized abdominal pain)
Constipation (Will not be this serious unless there is obstruction which will be apyrexial)
Sigmoid volvulus (again it will cause intestinal obstruction)
Hernia (may lead to obstruction or strangulation)
Diverticulitis (90% have pain LIF)
Appendicitis (mostly RIF veryyy rarely pain is in LIF.)

Meckels diverticulum mimics appendicitis and in most of the cases it is asymptomatic.


This is perforated diverticulitis as there is evidence of peritonitis because of severe abdominal pain and rigidity
and pyrexia.

708. A 46yo woman has weight gain, sensitivity to cold, pulse=50bpm, heart is enlarged with murmur. What
is the single most likely dx?
a. Hypothyroidism
b. Hyperthyroidism
c. Cushings syndrome
d. Addisons disease
e. Pheochromocytoma

Typical presentation of hypothyroidism.


Hyperthyroidisim has opposite symptoms.
Cushings has moon face, hypertension, weight gain csn occur, abdominal striae.
Addisons has pigmentation, hyponatremia, hypotension, hyperkalemia
Pheochromocytoma presents with episodic headaches and HTN.

709. An alcoholic who has completely given up drinking hears voices. What is the most appropriate tx?
a. Olanzapine
b. Diazepam
c. Acamprosate
d. Disulfiram
e. Haloperidol

This is psychosis. The patient has auditory hallucinations. Diazepam is used in acute alcohol withdrawal.
Acamprosate is to reduce cravings and maintain remission. Disulfiram is a deterrent. Olanzapine is preferred
over haloperidol as it is an atypical antipsychotic which has fewer side effects.

710. A 6yo boy has completed an induction course of chemo for ALL. He has an enlarged left scrotum. What
is the most appropriate next step?
a. Herniotomy
b. CT abdomen
c. Biopsy
d. Immediate surgery
e. Reassurance
711. A 32yo miner is rescued after being trapped under a fallwn rock for 4h. After applying a bladder
catether, 15-20ml of reddish brown urine was obtained. HR=120bpm, SBP=100mmHg. What would be the
next appropriate step?
a. Dopamine IV
b. Fluid challenge
c. Furosemide IV
d. 20% Mannitol IV
e. Antibiotics

The patient has developed rhabdomyolysis (trapped under a fallen rock, reddish brown urine due to
myoglobin in urine) in which there is a great danger of acute kidney failure as is the case in this patient due to
decreased urine output so we have to give this patient IV fluids to avoid further renal compromise.

712. A 60yo man has had spontaneous painful swelling of his right knee for 3days. 5days prv he had an
inguinal hernia repaired as a day case. He takes bendroflumethiazide 2.5mg daily. He is apyrexial. What is the
single most appropriate diagnostic inv?
a. Blood culture
b. CRP
c. D-dimer
d. XR knee
e. Serum uric acid

There is a suspicion of gout for which serum uric acid needs to be checked. The patient takes a thiazide
diuretic which predisposes to gout.

713. A 27yo woman with anxiety and weight loss has tachycardia, tremor and mild proptosis. What single
mechanism accounts for her weight loss?
a. Deficiency in thyroid hormone
b. Increased level of calcitonin
c. Increased metabolic rate
d. Insulin resistance
e. Reduced caloric intake

The patient has thyrotoxicosis (increased T3 & T4) which are catabolic enzyme increasing the metabolic rate
leading to the weight loss. The symptoms given point towards Graves disease.

714. A man with carcinoma and multiple metastasis presents with intractable nausea and vomiting. He has
become drowsy and confused. What is the most appropriate management?
a. Dexamethasone IM
b. Dexamethasone PO
c. Ondansetron IM
d. Ondansetron PO
e. Morphine oral

Ondansetron is a 5HT-3 blocker mostly used for chemotherapy induced vomiting. Dexa and morphine have no
role in this case. PO route cannot be used as the patient is vomiting.
715. A 19yo man presents with weight loss, increasing thirst and increasing frequency of going to the
washroom. His father, grandfather and 2 sisters have been dx with DM. What is the most likely type of DM this
pt suffers from?
a. IDDM
b. NIDDM
c. LADA
d. MODY
e. DKA

Diabetes at such a young age with a positive family history of early diabetes always think of Maturity onset
diabetes of the young. It has autosomal dominant mode of inheritance.
716. A 42yo woman with a PMH of severe headache treated in the ED presents with signs and symptoms of
renal failure. She has been seen by her GP for HTN and abdominal pain with OP inv pending. Which inv is most
likely to lead to a dx?
a. US KUB
b. CT brain
c. IVU
d. Renal artery Doppler
e. Renal biopsy

Loin pain, gross hematuria, nocturia, HTN all can be the features of polycystic kidney disease. Headaches could
be because of HTN. or it could also be renal artery stenosis for which US has to be done.

717. In perforation of a post gastric ulcer, where will the fluid accumulate in the peritoneal cavity?
a. Left paracolic gutter
b. Pelvic cavity
c. Lesser sac
d. Under the diaphragm
e. Right paracolic gutter

Behind the stomach is the lesser sac. All other are irrelevant here.

718. A 62yo male is brought to the ED by his daughter because of his persistent lying. He is a known
alcoholic and has been admitted recently with delirium tremens. On questioning, he denies any problem with
memory. He knows his name and address and states that was at the betting shop this morning, but his
daughter interjects calling him a liar explaining that he was at her home. What is the most likely dx?
a. Ganser syndrome
b. Cotard syndrome
c. Wernickes encephalopathy
d. Korsakoff psychosis
e. Alcohol withdrawal

Answer should be D.

Symptoms of Korsakoff syndrome


Vision changes:
o Double vision
o Eye movement abnormalities
o Eyelid drooping
Loss of muscle co-ordination:
o Unsteady, unco-ordinated walking
Loss of memory, which can be profound.
Inability to form new memories.
Hallucinations.

719. A 70yo man presented with muscle weakness and inability to climb the stairs. Inv: CPK raised, ESR 15.
What is the most likely dx?
a. Polymyositis
b. Polymyalgia rheumatic
c. Reactive arthritis
d. RS
e. Duchennes MD

Proximal muscle weakness raised CPK point towards the diagnosis. Anti Jo antibodies are raised in
polymyositis. Polymyalgia rheumatica presents with shoulder girlde ache, morning stiffness. Duchene will not
present at the age of 70 yrs it presents in childhood.

720. A 65yo known alcoholic is brought into the hospital with confusion, aggressiveness and
ophthalmoplegia. He is treated with diazepoxide. What other drug would you like to prescribe?
a. Antibiotics
b. Glucose
c. IV fluids
d. Disulfiram
e. Vit B complex

An alcoholic who presents with the triad of ophthalmoplegia, ataxia and confusion is wernickes syndrome. It
occurs due to vitamin B1 deficieny (thiamine def).

721. A pt suffering from schizophrenia laughs while talking about his fathers death. Which term best
describes his condition?
a. Depression
b. Flat affect
c. Emotional liability
d. Incongruent affect
e. Clang association

Flat affect, which is also called blunted affect, is one of the negative symptoms of schizophrenia. A person
with negative symptoms lacks a normal range of feelings and behaviors
Emotional liability also known as pseudobulbar affect is characterized by involuntary crying or uncontrollable
episodes of crying and/or laughing, or other emotional displays.
Symptoms are said to be mood-congruent if they are consistent with a patient's mood or mental disorder.
Conversely, they are said to be mood-incongruent if they are inconsistent with their primary mood.

clanging refers to a mode of speech characterized by association of words based upon sound rather than
concepts
722. A 72yo man has been on warfarin for 2yrs because of past TIAs and strokes. What is the most imp
complication the pt should be careful with?
a. Headache
b. Osteoporosis
c. Ear infection
d. Limb ischemia
e. Diarrhea

The answer should either be headache or diarrhea. diarrhea is one of the direct adverse effects of warfarin.
Since the patient is on warfarin there is a chance of intracranial bleed which can present as a headache so it
could be an indirect effect of warfarin use. Warfarin has nothing to do with osteoporosis.

723. A 24yo woman is afraid to leave her house as whenever she goes out, she tends to have SOB and
sweating. She has stopped going out except with her husband. What is the most likely dx?
a. Social phobia
b. Claustrophobia
c. Depression
d. Panic disorder
e. Agoraphobia

Social phobia is when the person is afraid to go to places where he/she may be judged, commented on or
criticized by others like parties.
Claustrophobia is fear of a tight space, small room etc.
Agoraphobia is fear of large open spaces.

724. A pt on HTN drugs develops hyperkalemia. Which anti-HTN is likely to cause it?
a. Ramipril
b. Lorsartan
c. Thiazide
d. Nifedipine
e. Furosemide

Just an adverse effect of ace inhibitors. Diuretics cause hypokalemia.

725. A young man develops itching worse at night and following bathing. Exam: greysish white linear rash
can be seen on the wrist and periumbilical area. What is the dx?
a. Scabies
b. Polycythemia
c. Urticarial
d. Atopic eczema
e. Lichen planus
Widespread itching, worse at night and when the person is warm. Skin signs can vary.

726. A 40yo lady who has been a smoker since she was a teenager has the following blood result: Hgb=19.
What hormone should you check?
a. Aldosterone
b. Cortisol
c. Erythropoietin
d. T4
e. TSH

The person has polycythemia which means increased RBC mass. Erythropoietin is responsible for the
production of RBCs. No other mentioned hormone here has anything to do with RBC production.

727. A 25yo man presents with hoarseness of voice. He has swollen vocal cords. His BMI=32 and he smokes
20-25 cigarettes/day. What would you advise him?
a. Stop smoking
b. Lose weight

There are several reasons for hoarsness of voice on of which is CA larynx. Smoking is a big risk factor for it.

728. A 64yo male was admitted to the medical ward with complaint of diarrhea, abdominal pain and weight
loss for few months. Exam: clubbing, perianal skin tags and abdominal tenderness. Colonscopy reveals
transmural granulomatous inflammation involving ileocecal junction. He was dx with what?
a. CD
b. UC
c. Bowel cancer
d. Gastric cancer
e. IBD

Peri anal tag, transmural granulomatous disease disease take the diagnosis towards crohns disease.

729. A pt presents with hemoptysis 7d after tonsillectomy. What is the next step?
a. Packing
b. Oral antibiotics and discharge
c. Admit and IV antibiotics
d. Return to OT and explore
e. Ice cream and cold fluids

This is a case of secondary hemorrhage (mostly between 5-14 days) the most common cause of which is
infection.

730. A 55yo man presents with HTN. He complains of headache and visual disturbances. He also reports
itching after a hot bath and burning sensation in finger and toes. His face is flushed red. PE: mild
splenomegaly. Inv: Hgb=20g/dl, WBC=20, plt=500, EPO normal. What is the likely dx?
a. Myelofibrosis
b. Polycythemia rubra vera
c. Essential thrombocythemia
d. CML
e. CLL

Increased haemoglobin, flushing, itching, burning sensation, visual disturbance (amaurosis fugax) all are
symptoms of polycythemia rubra vera.
731. An old man having T2DM with increased skin tanning, heart failure and having high ferritin
(hemochromatosis) level is refusing tx. Where is the first site of cancer?
a. Testes
b. Adrenal
c. Liver
d. Pancreas

It is a case of hemochromatosis so the primary site is liver. It can lead to HCC.

732. A 60yo DM lady presents with severe peri-anal pain and swelling. Whats the cause?
a. Anal carcinoma
b. Anal fissure
c. Hemorrhoid
d. Anal abscess

Q1. What is the key?


Q2. What is the supporting evidence?

A1. The answer is anal abscess.


A2. The lady is 60 years old and presents with perianal pain and swelling. Firstly it is important to think of an
infection in case of diabetics.
An anorectal abscess is a collection of pus in the anal or rectal region. It may be caused by infection of anal
fissure/STD/blockage of anal glands.

The following anatomical types have been identified.


Perianal abscess: the most common (60%) - caused by direct extension of sepsis in the intersphincteric
plane caudal to the perianal skin.
Ischiorectal abscess: (20%) - results from extension of sepsis through the external sphincter into the
ischiorectal space.
Intersphincteric abscess: (5%) - depending on the effort made to find them, sepsis confined to the
intersphincteric space.
Supralevator abscess: (4%) - produces horseshoe abscess track.
Postanal abscess: posteriorly based below the level of the ano-coccygeal ligament.
High Risk group includes diabetics, immunocompromised, anal sex, patients of inflammatory bowel disease.
Age group 20-60 with mean age 40, M:F=2:1
SYMPTOMS: pain, lump, nodule, hardened area in perianal region, tenderness at edge of anus, fever,
constipation or pain associated with bowel movements.
Pain is usually constant, throbbing and worse on sitting down.
INVESTIGATIONS: Digital rectal examination is sometimes enough to diagnose.
PROCTOSIGMOIDOSCOPY: to rule out associated diseases.
MRI: for assessment of location of fistular tracts and their openings, location of deep abscesses, peri-rectal
space, any damage to anal sphincter.
TRANSPERINEAL USG: may help to locate deep abscesses or an associated pathology.
SCREENING for STD, inflammatory bowel disease, diverticular disease or lower GI malignancy.
TREATMENT:
prompt surgical drainage
pain relief
Antibiotics only if diabetic or immunosuppression.
COMPLICATIONS: systemic infection, recurrence, scarring, fistula in ano in 30% of patients, reduces with early
surgery.

To differentiate from anal carcinoma, it may also present with anal swelling and pain, but there is usually
associated bleeding and faecal incontinence.
Anal fistula usually presents with excruciating pain while defecating like shreds of glass, and may be associated
with rectal bleeding usually bright red and not mixed with stools.
For haemorrhoids, bright red, painless rectal bleeding with defecation is usually the most common symptom.

733. A woman is sad, fatigues and she is eating more and also has sleeping disturbance and hears the voice of
her husband who died 3yrs ago. What is the dx?
a. OCD
b. Psychotic depression
c. Grieving
d. Severe depression

Q1. What is the key?


Q2. Why is it so.

Q1. The key is Psychotic depression.


Q2. Why..

Depression refers to both negative affect (low mood) and/or absence of positive affect (loss of interest and
pleasure in most activities) and is usually accompanied by an assortment of emotional, cognitive, physical and
behavioural symptoms.It is currently ranked the third most prevalent moderate and severe disabling condition
globally by the World Health Organization (WHO).

Classification:
(NICE) guidance uses the Diagnostic and Statistical Manual Fourth Edition (DSM-IV) classification.

To diagnose major depression, this requires at least one of the core symptoms:
Persistent sadness or low mood nearly every day.
Loss of interests or pleasure in most activities.

Plus some of the following symptoms:


Fatigue or loss of energy.
Worthlessness, excessive or inappropriate guilt.
Recurrent thoughts of death, suicidal thoughts, or actual suicide attempts.
Diminished ability to think/concentrate or increased indecision.
Psychomotor agitation or retardation.
Insomnia/hypersomnia.
Changes in appetite and/or weight loss.
Symptoms should have been present persistently for at least two weeks and must have caused clinically
significant distress and impairment.
Rule out other organic/physical factor like substance abuse, or chronic illness.

DSM-5 was published in 2013. It proposes the following changes to the classification of depressive
disorders:
Persistent depressive disorder - this term is proposed to encompass both chronic major depressive
disorder and dysthymia.
Removal of the major depression bereavement exclusion - the diagnosis of major depression was
excluded in people who had recently been bereaved. This has been removed, leaving more leeway
for clinical judgement.
A new category of mixed anxiety/depressive disorder.

The NICE guidelines encourage a case-finding approach with at-risk groups (individuals with a past history of
depression or a chronic health problem with associated functional impairment) using a two question
approach:
During the past month, have you:
o Felt low, depressed or hopeless?
o Had little interest or pleasure in doing things?
ASSESSMENT:
The Patient Health Questionnaire (PHQ-9)
The Hospital Anxiety and Depression (HAD) Scale
Beck's Depression Inventory
Full history and examination +mental state examination
enquire about delusions/hallucinations and suicidal ideas.
Rule out organic causes of depression like hypothyroidism/drug side effects
Medications that may cause depressed mood include:
Centrally acting antihypertensives (eg, methyldopa).
Lipid-soluble beta-blockers (eg, propranolol).
Benzodiazepines or other central nervous system depressants.
Progesterone contraceptives, especially medroxyprogesterone injection.

Differential diagnosis
Bipolar disorder.
Schizophrenia (depression may co-exist).
Dementia may occasionally present as depression and vice versa.
Seasonal affective disorder.
Dysthymia (recently classified by DSM-5 as a subtype of persistent depressive disorder) is a chronic
depressive state of more than two years in duration.
Other psychiatric conditions may co-exist with depression (eg, generalised anxiety disorder, panic
disorder, obsessive-compulsive disorder, personality disorders).
Bereavement: depressive symptoms begin within 2-3 weeks of a death (uncomplicated bereavement
and major depression share many symptoms but active suicidal thoughts, psychotic symptoms and
profound guilt are rare with uncomplicated bereavement).

INVESTIGATIONS:
Rule out organic causes
Blood tests may include blood glucose, U&Es, LFTs, TFTs, calcium levels, FBC and inflammatory markers, HIV
OR SYPHILIS SEROLOGY, drug screening.

MANAGEMENT:
Consider watchful waiting, assessing again normally within two weeks.
Guided self-help based on cognitive behavioural therapy (CBT) principles
Antidepressants are not recommended for the initial treatment of mild depression, because the
risk:benefit ratio is poor. However they are recommended in moderate to severe depression.
Selective serotonin reuptake inhibitors (SSRIs) are used as first-line antidepressants in routine care
because they are as effective as tricyclic antidepressants and less likely to be discontinued because of
side-effects; also because they are less toxic in overdose.
Citalopram, fluoxetine, paroxetine, or sertraline have equal efficacy however a recent meta-analysis
suggested that escitalopram had the highest probability of remission and is the most effective and
cost-effective pharmacological treatment in a primary care setting
Fluoxetine is the antidepressant of choice for children and young people.
Where a patient has concurrent physical health problems, citalopram or sertraline may be preferred,
as they have less risk of significant drug interactions
Treatments such as dosulepin, phenelzine, combined antidepressants and lithium augmentation of
antidepressants should be initiated only by specialist mental healthcare professionals.
Electroconvulsive therapy (ECT) may be used to gain fast and short-term improvement of severe
symptoms after all other treatment options have failed, or when the situation is thought to be life-
threatening.

What is Psychosis;
Psychosis is a severe mental disorder in which there is extreme impairment of ability to think clearly, respond
with appropriate emotion, communicate effectively, understand reality and behave appropriately.
Psychosis occurs in a number of serious mental illnesses and not just schizophrenia, eg depression, bipolar
disorder (manic-depressive illness), puerperal psychosis and sometimes with drug and alcohol abuse
Disabling symptoms include delusions and hallucinations:
A delusion is a false, fixed, strange, or irrational belief that is firmly held. The belief is not normally
accepted by other members of the same culture or group. There are delusions of paranoia (plots
against them), delusions of grandeur (exaggerated ideas of importance or identity) and somatic
delusions (false belief in having a terminal illness).
An hallucination is sensory perception (seeing, hearing, feeling, smelling) without an appropriate
stimulus, like hearing voices when no one is talking. Not all hallucination suggests psychosis.
So to conclude we can see that the patient is the question was suffering from complicated bereavement
associated with psychotic symptoms.
734. A 40yo teetotaler woman is recovering from a hysterectomy 2days ago. At night she becomes agitated
and complains of seeing animals and children walking around the ward. What is the most likely dx?
a. Delirium tremens
b. Toxic confusional state
c. Hysteria
d. Mania
e. Drug induced personality disorder

The key says toxic confusional state.

Confusion and agitated behaviour are common complications after operation,


especially in elderly patients. It is customary to prescribe sedation, instead of regarding confusion as a
symptom whose causes must be , diagnosed, and preferably anticipated and prevented.
This is the most common psychosis seen in hospital with 20% of elderly patients having some form of delirium.
Also know as toxic confusional state.

A simple scheme of management is outlined, with special emphasis on common conditions.Often it is a


symptom of an acute underlying disorder which needs early treatment. It may in turn cause its own
complications, such as injury, disturbance of healing tissues or of equipment, or pneumonia due to mistaken
over-use of sedation in treatment. The turmoil caused by a confused patient is distressing to other patients,
and creates hard work for nurses and junior medical staff.

CAUSES OF POST OPERATIVE CONFUSION:


1. Common:
Cerebrovascular disease.
Drugs, delirium tremens.
Chest infection or atelectasis.
Renal infection.
Abdominal sepsis, superficial or deep.
Over-full bladder or rectum.
2. Less common:
Anaemia (especially vitamin B12 deficiency), unrecognized blood loss, and other forms of anoxia.
Hypothyroidism.
Hepatic or renal failure.
Subdural haematoma.
3. Psychological factors:
Sensory distortion by bombardment or deprivation.
Sleep disturbance and loss.
Depression, anxiety, schizophrenia.
4. Aggravation by noxious stimuli:
Fear, discomfort, pain, thirst, hunger.
5. Rare causes:
Hypoglycaemia, fat embolism, hypernatraemia.
Drugs BZDs, opiates, tricyclics, alcohol, L-Dopa

Signs
Disordered thinking incoherent, slow, irrational, jumbled, rambling often worse at night
Euphoric, fearful, angry or depressed
Language impaired reduced speech, repetetive or disruptive
Illusions/delusion/hallucinations think tactile or visual (auditory more suggests psychosis)
Reversal of sleep patterns drowsy by day > awake & vigilant at night
Inattention
Unaware/disorientated doesnt know time of day, name or address
Memory deficit may be amnesic

TREATMENT:
Management of the confused postoperative patient may be quite easy or very difficult, depending on the
underlying causes of the condition. Most often there is a single predominating factor which is temporary, such
as retention of urine, or pneumonia with congestive cardiac failure, to be treated along accepted lines.

Sedative drugs are best withdrawn unless there is uncontrollable agitation, when the most useful are
diazepam (Valium), paraldehyde, or chlorpromazine.
identify and treat underlying cause
careful nursing and rehydration
nurse in quiet, single room, with window
review current drug therapy
if disruptive can use haloperidol
can persist for 1-2 week

As the clue given is the patient being post-op , so we have to consider acute post op toxic confusional state as
the most likely diagnosed compared to others and since the patient is non alcoholic there is no chance of
delirium tremens.

735. A woman with a hx of drug abuse and increased alcohol intake, now comes for help and she is
concerned about her problem. What is the most appropriate management option?
a. Voluntary admission
b. Psychiatry team
c. Mental health team
d. Psychiatry voluntary admission
Key: The important point in this scenario is that the patient is willing for treatment. NICE has published
guidelines for drug misuse and treatment and for every patient these protocols need to be followed. The main
points from the guidelines regarding assessment and decision making as as follows. The only problem with this
key was that psychiatry team and mental health team seem to be the same thing and usually patients are sent
to organizations such as alcoholics anonymous etc who have specially designed plans for management so they
are neither psychiatric teams, nor mental health teams. Anyway have a look at the basic framework of the
management plan and then decide about answer according the patients scenario.

Treatment provision: key points


The needs of drug misusers should be assessed in terms of their health, social functioning and criminal
involvement.
All GPs have a duty to provide basic medical services to people who are dependent on opioids and they
should screen patients for drug misuse.
A good initial assessment is essential:
o This may involve a multidisciplinary team.
o Good assessment is vital to the continuing care of the patient.
o It can enable the patient to become engaged in treatment and may begin a process of change
even before a full assessment is completed.
Confirmation of drug taking should be gained (through history, examination and drug testing).
Any risks to their children should be assessed and child protection services involved as appropriate.
Emergency or acute problems should be treated (for example, access to clean needles and equipment).
Testing for blood-borne infections should be arranged as appropriate.
A physical and psychological health assessment should be carried out.
Any ongoing criminal involvement or offences should be determined.
The drug misuser's expectations and desire to change should be assessed.
The degree of dependence and need for substitute medication should be assessed.
An individual care and treatment plan should be drawn up and reviewed regularly.
A named person should manage and deliver an individual's care (eg the GP or drugs worker). They may
be known as the 'keyworker'.
If detoxification and/or substitute prescribing are requested, after an initial assessment, GPs can refer
to local specialist community drug services and there are usually locally agreed shared care guidelines.
A care plan between the drug misuser and the service provider can then be drawn up.
A GP may have a special clinical interest in the management of substance misuse in primary care and
may be able to take more responsibility in the treatment of patients, particularly in complex cases.
Drug testing can help to monitor compliance and treatment outcome.

Psychosocial components of treatment: key points


A keyworker with a good therapeutic alliance is best placed to provide psychosocial assessment and
support.
Counselling, cognitive behavioural therapy and supportive help (for example with housing and
benefits) are examples of psychosocial treatment strategies.
If the keyworker does not have the full range of competencies to deliver psychosocial interventions,
other professionals may be involved.
Common social problems among drug misusers include housing, employment and financial difficulties.
Criminal convictions are also common.
Mental health problems such as depression and anxiety can co-exist with drug misuse.
In cannabis, hallucinogen and stimulant abuse (including cocaine), psychosocial interventions are the
main treatment.
For opioid, alcohol and polydrug misuse, they can be used in conjunction with drug treatment.
Patients should also be advised about support groups such as Alcoholics Anonymous and Narcotics
Anonymous.
Some patients find that self-help approaches work for them and these should be discussed.
In other countries, couple- and family-based interventions and contingency management have been
found to be helpful. These approaches are not commonly used in the UK at present but they should be
considered (provided the appropriate training has been received).
The NICE guidelines support a number of formal psychosocial treatments and detail the evidence that
supports them. These include: brief motivational interventions, self-help groups and contingency
management (eg, incentives contingent on each presentation of a negative drug test).

736. A 28yo woman who is 32 wks pregnant in her 3rd pregnancy is diagnosed as a case of placental
abruption. After all the effective measures, she is still bleeding. What is the underlying pathology?
a. Clotting factor problem
b. Clausers syndrome
c. Platelet problem
d. Succiturate lobe
e. Villamentous insertion of placenta

Ans: Placental abruption has been defined as the complete or partial separation of a normally implanted
placenta from its uterine site before the delivery of the fetus. This definition differentiates this process from
placenta previa, in which the placenta is implanted in an abnormal anatomical position covering the internal
cervical os.

Grade I: Mild. This group accounts for 40% of all cases; it includes antepartum hemorrhage of uncertain cause.
There may be slight vaginal bleeding and uterine irritability. Maternal blood pressure usually is normal, and
there is no maternal coagulopathy or fetal distress. The diagnosis of this class of abruptio placentae is
confirmed on postpartum detection of a small retroplacental clot.
Grade II: Intermediate. This accounts for 45% of all cases. This diagnosis is based on the classic features of
abruptio placentae with uterine hypertonicity, but the fetus still is alive. There is a greater amount of vaginal
bleeding (mild to moderate), hypofibrinogenemia, and fetal distress. Blood pressure is maintained, but the
pulse rate may be elevated and postural blood volume deficits may be present.
Grade III: Severe. This accounts for 15% of all cases. In such cases, the fetus is always dead. Usually, heavy
vaginal bleeding occurs, although in some cases this may be concealed. Maternal hypotension,
hypofibrinogenemia, and thrombocytopenia are present, along with a tetanic uterus. This type is further
subdivided into grade IIIA, in which overt coagulopathy is not present, and grade IIIB, when an overt
coagulopathy results.2
This grading system may be helpful in establishing a therapeutic plan.

Clinicians and investigators have observed the presence of a bleeding diathesis accompanying some cases of
premature separation of the placenta. In addition to its association with severe placental abruption, acquired
hypofibrinogenemia also has been reported in cases of amniotic fluid embolism, long-standing fetal death in
utero, septic abortion, eclampsia, and delayed postpartum hemorrhage.

The phenomenon of consumption coagulopathy leads to patient injury because of two problems: the bleeding
diathesis caused by diminished coagulation factors and elevated FDP and the localized tissue necrosis in target
organs because of fibrin deposition in small blood vessels. As a protective mechanism, the fibrinolytic system
is activated secondarily, and dissolution of the fibrin clots by plasmin may protect the local tissue from anoxia.
This delicate balance between fibrin deposition and degradation is present in the body at these times. If this
secondary fibrinolysis and fibrinogenolysis become excessive, the decreased coagulation factors resulting from
consumption are augmented by destruction of the same factors by plasmin, and the hemorrhagic situation is
made worse. The degradation products of fibrin and fibrinogen also interfere with fibrinogen conversion to
fibrin. Fibrinolytic problems in abruptio placentae are mostly secondary, with progressive activation of the
system occurring only after an initial phase of intravascular coagulation.

The complications of hypofibrinogenemia have been detected in one third to one half of patients having signs
and symptoms of severe placental separation. Most patients with abruptio placentae have the milder form
and exhibit no clinical difficulties with the clotting mechanism.

So as it says in the mcq that all other measures to stop the bleeding have failed then we should start thinking
on the lines of clotting factor problems. Even in the most severe cases, clinically evident coagulopathy usually
resolves by 12 hours after delivery.

737. An old woman having decreased vision cant see properly at night. She has changed her glasses quite a
few times but with no effect. She has normal pupils and cornea. What is the most likely dx?
a. Cataract
b. Glaucoma
c. Retinal detachment
d. Iritis
e. GCA

Vision loss among the elderly is a major health care problem. Approximately one person in three has some
form of vision-reducing eye disease by the age of 65. The most common causes of vision loss among the
elderly are age-related macular degeneration, glaucoma, cataract and diabetic retinopathy. Age-related
macular degeneration is characterized by the loss of central vision. Primary open-angle glaucoma results in
optic nerve damage and visual field loss. Because this condition may initially be asymptomatic, regular
screening examinations are recommended for elderly patients. Cataract is a common cause of vision
impairment among the elderly, but surgery is often effective in restoring vision. Diabetic retinopathy may be
observed in the elderly at the time of diagnosis or during the first few years of diabetes. Patients should
undergo eye examinations with dilation when diabetes is diagnosed and annually thereafter.

Many causes of gradual visual loss can be diagnosed on history and examination alone with only the most
basic additional investigations. For any patient with a gradual loss in visual acuity ,the following protocol
should be followed to reach to an initial diagnosis.

Assessment

History
A routine history is mandatory and will often guide you to a possible cause. Specifically ask about:
The nature of the problem:
o Unilateral versus bilateral.
o Blurred vision: whether this is the whole field, close, distance or both.
o Restricted visual field: often noted following difficulties in driving, knocking into things at the
periphery of vision
o Distorted rather than blurred vision (eg, dent in printed words, door/window frames, objects
appearing smaller or larger). If so, check with an Amsler grid (see under 'Further reading &
references', below). Distortion of straight lines indicates serious macular pathology and needs
urgent referral.
o Bits of visual field missing altogether: central versus peripheral; establish what the remainder of
the vision is like.

Progression: ask whether there has been a slow and steady decline; whether there have been step-
wise drops in visual acuity, or whether the problem has been intermittent. If intermittent, think of
transient ischaemic attacks (TIAs) or impending acute angle-closure glaucoma.
Ask whether there have been any associated factors. For example, pain (very important diagnostically,
see 'Painful loss', below), redness, or visual phenomena - eg, haloes, flashes of light, new floaters.
These symptoms usually merit an urgent referral.
Ask whether there are any precipitating factors. Specifically ask about changes over the course of a day
and whether vision is better in the day or at night.

Examination
Important points to note on examination are:
The visual acuity of both eyes. Note whether this improves using a pinhole.
The red reflex: a media opacity (appears black against the red reflex) suggests a corneal, lens or
vitreous problem. To localise the site of the opacity with respects to the pupil (lens):
o Slowly shift the direction of your ophthalmoscope light.
o Look at the direction in which the opacity appears to move in relationship to the pupillary
(central) axis.
o If there is no 'movement' of the opacity, it lies within the pupil (lens).
o If the opacity 'moves' in the same direction, it is anterior to the lens (cornea).
o If the opacity 'moves' in the opposite direction, it is posterior to the pupil (posterior lens or
vitreous).
o If the media is clear, it is more likely to be a retinal or optic nerve disorder.
o If there is a normal red reflex, take a good look at the fundus.
o Do a functional testing of visual field, pupils, optic nerve and macula.
o Use an Amsler grid to look for distortion of straight lines - if present, this indicates serious
macular pathology and needs urgent referral

Interpreting the findings - some clinical patterns


Loss of vision in one eye - implies the problem is in the eye itself or in the optic nerve before it reaches
the optic chiasm.
Loss of vision in both temporal fields (bitemporal hemianopia) - occurs from lesions compressing the
optic chiasm (eg, pituitary tumour or craniopharyngioma).
Loss of left or right visual field (homonymous hemianopia) - implies the lesion is somewhere between
the optic chiasm and the occiput.
Chronic glaucoma tends to cause tunnel vision (loss of peripheral visual field).
Macular degeneration causes loss of central vision (central scotoma) and may cause distortion of
straight lines.
Cataracts - the patient often complains of glare in dark conditions (and so difficulty in driving at night)
and may complain that colours appear more dull than they used to. There may be an abnormal red
reflex and, in advanced cases, the cataract may be visible to the naked eye (this is increasingly rare
these days). Other aspects of the examination should be normal unless there is concurrent pathology.
Refer routinely.

738. A 53yo man was admitted to the hospital for inv of hemoptysis. 3 days after admission he developed
alternating state of consciousness, ataxic gait and some visual problems. What is the most appropriate
management of this pt?
a. Acamprosate
b. Chlordiazepoxide
c. Diazepam
d. High potent vitamins
e. Disulfiram

Ans: As we can see from this question that it is not important to find out the reason for haemoptysis , the
reason may be any, he was admitted for investigations and then around 3 days later he started having some
symptoms. In a hospital setting, if a patient starts having symptoms like these, one should always think about
alcohol withdrawal.

Alcohol withdrawal syndrome is a potentially life-threatening condition that can occur in people who have
been drinking heavily for weeks, months, or years and then either stop or significantly reduce their alcohol
consumption.When heavy drinkers suddenly stop or significantly reduce their alcohol consumption, the
neurotransmitters previously suppressed by alcohol are no longer suppressed. They rebound, resulting in a
phenomenon known as brain hyperexcitability. So, the effects associated with alcohol withdrawal -- anxiety,
irritability, agitation, tremors, seizures, and DTs -

Presentation
This may be in a number of different ways:
A patient may present in acute alcohol withdrawal.
A patient may be admitted to hospital for another reason and thus an unplanned alcohol withdrawal
may be precipitated. Alcohol-use disorders can complicate the assessment and treatment of other
medical and psychiatric problems.
A patient may present wishing to abstain from alcohol but be seen as at risk of acute alcohol
withdrawal.

Alcohol withdrawal symptoms can begin as early as two hours after the last drink, persist for weeks, and range
from mild anxiety and shakiness to severe complications, such as seizures and delirium tremens (also called
DTs).
Withdrawal symptoms:
Symptoms typically present about eight hours after a significant fall in blood alcohol levels. They peak
on day 2 and, by day 4 or 5, the symptoms have usually improved significantly.
Minor withdrawal symptoms (can appear 6-12 hours after alcohol has stopped):
o Insomnia and fatigue.
o Tremor.
o Mild anxiety/feeling nervous.
o Mild restlessness/agitation.
o Nausea and vomiting.
o Headache.
o Excessive sweating.
o Palpitations.
o Anorexia.
o Depression.
o Craving for alcohol.
Alcoholic hallucinosis (can appear 12-24 hours after alcohol has stopped):
o Includes visual, auditory or tactile hallucinations.
Withdrawal seizures (can appear 24-48 hours after alcohol has stopped):
o These are generalised tonic-clonic seizures.
Alcohol withdrawal delirium or 'delirium tremens' (can appear 48-72 hours after alcohol has stopped)

Delirium tremens
This is a medical emergency. A hyperadrenergic state is present.

Clinical features
Delirium tremens usually begins 24-72 hours after alcohol consumption has been reduced or stopped.
The symptoms/signs differ from usual withdrawal symptoms in that there are signs of altered mental
status. These can include:
o Hallucinations (auditory, visual, or olfactory).
o Confusion.
o Delusions.
o Severe agitation.
Seizures can also occur.
Examination may reveal signs of chronic alcohol abuse/stigmata of chronic liver disease. There may
also be:
o Tachycardia.
o Hyperthermia and excessive sweating.
o Hypertension.
o Tachypnoea.
o Tremor.
o Mydriasis.
o Ataxia.
o Altered mental status.
o Cardiovascular collapse.
MANAGEMENT:
The goals of treatment are threefold: reducing immediate withdrawal symptoms, preventing complications,
and beginning long-term therapy to promote alcohol abstinence.
Prescription drugs of choice include benzodiazepines, such as diazepam (Valium), chlordiazepoxide (Librium),
lorazepam (Ativan), and oxazepam (Serax). Such medications can help control the shakiness, anxiety, and
confusion associated with alcohol withdrawal and reduce the risk of withdrawal seizures and DTs. In patients
with mild to moderate symptoms, the anticonvulsant drug carbamazepine (Tegretol) may be an effective
alternative to benzodiazepines, because it is not sedating and has low potential for abuse.
Thiamine
Thiamine deficiency is common in people who are alcohol-dependent, due to their poor diet, the
presence of gastritis which can affect its absorption and also the fact that it is a coenzyme in alcohol
metabolism.
Deficiency can cause Wernicke's encephalopathy, which if left untreated, can lead to Korsakoff's
syndrome.
Oral thiamine is poorly absorbed in dependent drinkers. For this reason, all those undergoing
detoxification in the community should be considered for admission for parenteral high-potency B
complex vitamins (Pabrinex) as prophylactic treatment. However, because of the risk of anaphylaxis,
resuscitation facilities need to be available at the time of administration. The risk of anaphylaxis is
lower if the drug is given intramuscularly (IM).
As prophylactic treatment, one pair of ampoules of Pabrinex should be given IM or intravenously (IV)
once a day for three to five days. A pair of ampoules contains 250 mg of thiamine.
MANAGEMENT OF DT:
This should be in a hospital setting. Intensive care may be needed for very unwell patients.
It should first include assessment and management of 'Airway, Breathing and Circulation (ABC)' .
Any hypoglycaemia should be treated.
Sedation with benzodiazepines is suggested. Diazepam has a rapid onset of action.
Addition of barbiturates may also be necessary in those refractory to benzodiazepine treatment and
may reduce the need for mechanical ventilation in very unwell patients in the intensive care unit.
Patients with delirium tremens may also have Wernicke's encephalopathy and should be treated for
both conditions
o At least two pairs of ampoules of Pabrinex (500 mg thiamine) should be given IV three times
daily for three days.
o If the patient does not respond, treatment should be discontinued.
o If signs or symptoms respond to treatment, continue with two ampoules of Pabrinex once
daily for five days or for as long as improvement continues.
Magnesium may also protect against seizures and arrhythmias

Disulfiram is used to treat chronic alcoholism. It causes unpleasant effects when even small amounts of
alcohol are consumed. These effects include flushing of the face, headache, nausea, vomiting, chest pain,
weakness, blurred vision, mental confusion, sweating, choking, breathing difficulty, and anxiety. These effects
begin about 10 minutes after alcohol enters the body and last for 1 hour or more. Disulfiram is not a cure for
alcoholism, but discourages drinking.

Acamprosate, is a medication used along with counselling in the long term treatment of alcohol dependence.
Acamprosate is thought to stabilize the chemical balance in the brain that would otherwise be disrupted by
alcohol withdrawal. Reports indicate that acamprosate works to best advantage in combination with
psychosocial support and can help facilitate reduced consumption as well as full abstinence.

739. A pt underwent hip surgery. Later he presents with SOB and chest pain. What is the dx?
a. Pulmonary embolism
b. MI
c. Tension pneumothorax
d. Fat embolism
e. None

Pulmonary embolism is the sudden blockage of a major artery in the lung, usually by a blood clot. In most
cases, the clots are small and are not deadly, but they can damage the lung. But if the clot is large and stops
blood flow to the lung, it can be deadly. Quick treatment could save your life or reduce the risk of future
problems.
The most common symptoms are:
Sudden shortness of breath.
Sharp chest pain that is worse when you cough or take a deep breath.
A cough that brings up pink, foamy mucus.
In most cases, pulmonary embolism is caused by a blood clot in the leg that breaks loose and travels to the
lungs. A blood clot in a vein close to the skin is not likely to cause problems. But having blood clots in deep
veins (DVT) can lead to pulmonary embolism. More than 300,000 people each year have DVT or a pulmonary
embolism.Other things can block an artery, such as tumors, air bubbles, amniotic fluid, or fat that is released
into the blood vessels when a bone is broken. But these are rare.
Other things that can increase your risk include:
Being inactive for long periods. This can happen when you have to stay in bed after surgery or a serious
illness, or when you sit for a long time on a flight or car trip.
Recent surgery that involved the legs, hips, belly, or brain.
Some diseases, such as cancer, heart failure, stroke, or a severe infection.
Pregnancy and childbirth (especially after C section).
Taking Oral contraceptive pills or hormone therapy.
Smoking.
So the scenario in this mcq is most likely pulmonary embolism.

740. A 25yo man presents with hx of breathlessness. A transthoracic echo reveals a patent foramen ovale.
What diagnostic inv would you do for a patent foramen ovale?
a. Transesophageal echo
b. Bubble echo
c. Transthoracic echo
d. ECG

Patent foramen ovale

Patent foramen ovale (PFO) is a hole between the left and right atria of the heart. This hole exists in everyone
before birth, but usually closes shortly after being born. PFO is what the hole is called when it fails to close
naturally after a baby is born.

This hole allows blood to bypass the fetal lungs, which cannot work until they are exposed to air. When a
newborn enters the world and takes its first breath, the foramen ovale closes, and within a few months it has
sealed completely in about 75 percent of us. When it remains open, it is called a patent foramen ovale. For the
vast majority of the millions of people with a PFO, it is not a problem, even though blood is leaking from the
right atrium to the left. Problems can arise when that blood contains a blood clot.

Diagnosis of PFO
Patent Foramen Ovale can only be detected by specialized testing. If suspected, your doctor may order:
Echocardiogram
Transesophageal echo (TEE) an ultrasound test used to visualize the heart and defect, where an
imaging probe with a camera is placed into the esophagus
Bubble study In some cases during an echo or TEE, the doctor may inject agitated saline through an
IV in your arm with bubbles. During the test, the doctor can watch the echo to see if bubbles pass from
the right to the left side of the heart to reveal the PFO.
741. A 25yo woman with a hx of several episodes of depression is brought to the ED after she was found with
several empty bottles of her meds. She complains of coarse tremor, nausea and vomiting. Which of the
following drugs is likely to have caused her symptoms?
a. Fluoxetine
b. Amitryptilline
c. Lithium
d. Phenelzine
e. Olanzapine

742. A 23yo man feels anxious and agitated when faced with stress. He has an interview in 3days and would
like some help in relieving his symptoms. What is the most appropriate management?
a. SSRI
b. CBT
c. Propranolol
d. Diazepam

743. An 8yo boy dx with asthma is on salbutamol and beclomethasone. However, he wakes up at night due to
his symptoms. What is the next appropriate management?
a. LABA
b. High dose steroid
c. Aminophylline
d. Oral prednisolone
e. Sodium cromoglycate

744. A woman presents with a hx of poisoning 10x with different substances. There are no obvious signs of
depression or suicidal behavior. What is the best preventive step?
a. Open access to ED
b. 24h help line
c. CBT
d. Anti-depressants
e. Insight into problem

Is 745. A boy was rushed to the ED unconscious after he had taken methadone belonging to the sister.
He was given naloxone and he regained consciousness. After a while he started getting drowsy again.
What is responsible for his present drop in level of consciousness?
a. Naloxone is absorbed faster than methadone
b. Methadone is absorbed faster than naloxone
c. He has also taken another substance apart from methadone
d. The methadone had already caused some brain damage
e. Naloxone is eliminated faster than methadone
746. A 24yo male on remand in prison for murder is referred by the prison doctor. He is noted to be
behaving oddly whilst in prison and complains of seeing things. He has a prv hx of IV drug abuse. On
questioning he provides inappropriate but approximate answers to all questions stating that Bill Clinton is the
prime minister of England. What is the prisoner suffering from?
a. Capgras syndrome
b. Cotard syndrome
c. Ganser syndrome
d. Ekbom syndrome
e. Tourettes syndrome
747. A 32yo lady has recently become more active, sleeps less and bought a house and 2 new cars. What is
the most likely dx?
a. Bipolar disorder
b. Mania
c. Hypomania
d. Schizophrenia
748. The body of a 65yo man who was treated for TB and bronchitis was seen at autopsy. His legs were
swollen and his liver showed signs of a transudate fluid. What was the cause of the transudate?
a. Liver cirrhosis
b. Alcoholic liver disease
c. Cardiac failure
d. Budd-chiari syndrome
e. TB
749. A 60yo woman has tiredness. She has noticed that her skin looks permenantly tanned and she
describes dizziness on standing up. What is the single most likely electrolyte pattern to be found?
a. Na+=120mmol/L, K+=5.9mmol/L
b. Na+=125mmol/L, K+=2.9mmol/L
c. Na+=140mmol/L, K+=4.5mmol/L
d. Na+=150mmol/L, K+=3.5mmol/L
e. Na+=150mmol/L, K+=5.9mmol/L
750. A 20yo girl with amenorrhea and BMI of 14 still thinks she has to lose weight. What is the most likely
dx?
a. Anorexia nervosa
b. Bulimia nervosa
c. OCD
d. Depression
e. Body dysmorphic disorder
751. A guy who has several convictions and has been imprisoned several times, breaks up with his family
and doesnt contact his children. What type of personality disorder is this?
a. Borderline
b. Antisocial
c. Schizotypal
d. Schizoid
e. Criminal

Cluster A
They tend to be odd or eccentric:
Paranoid personality disorder They display pervasive distrust and suspicion. Common beliefs include:
Others are exploiting or deceiving them.
Friends and associates are untrustworthy.
Information confided to others will be used maliciously.
There is hidden meaning in remarks or events others perceive as benign.
The spouse or partner is unfaithful. Pathological jealousy is sometimes called the Othello syndrome.
Schizoid personality disorder This type of personality disorder is less common in clinical settings:
They are detached from others and have little desire for close relationships.
There is little pleasure in activities.
They appear indifferent to praise or criticism and often seem cold or aloof.
Schizotypal personality disorder They show marked eccentricities of thought, perception and behaviour.
Typical examples include:
Ideas of reference (believing that public messages are directed personally at them).
Odd beliefs or magical thinking.
Vague, circumstantial, or stereotyped speech.
Excessive social anxiety that does not diminish with familiarity.
Idiosyncratic perceptual experiences or bodily illusions.

Cluster B
They tend to be dramatic and emotional:

Antisocial personality disorder See separate article Antisocial Personality Disorder and National Institute for
Health and Clinical Excellence (NICE) guidance.[5] People with antisocial personality disorder have a pervasive
pattern of disregard for and violation of the rights of others and the rules of society. Onset must occur by age
15 years.( Criminal behavior is central to its definition)

Borderline personality disorder See separate article Borderline Personality Disorder and NICE guidance.[6] The
important feature of borderline personality disorder is a pervasive pattern of unstable and intense
interpersonal relationships, self-perception and moods. Impulses are poorly controlled. At times they may
appear psychotic because of the intensity of their distortions. It is a commonly overused diagnosis in DSM-IV.

Histrionic personality disorder


They also display excessive emotion and attention-seeking behaviour.
They are quite dramatic and often sexually provocative or seductive.
Their emotions are labile.
In clinical settings, their tendency to vague and impressionistic speech is often apparent.
Narcissistic personality disorder Narcissistic patients are grandiose and require admiration from others.[7]
Features include:
Exaggeration of their own abilities or achievements.
Sense of entitlement.
Exploitation of others.
Lack of empathy.
Envy of others.
An arrogant, haughty attitude.

Cluster C
They tend to be anxious and fearful:

Avoidant personality disorder


They are generally very shy.
There is a pattern of social inhibition, feelings of inadequacy and hypersensitivity to rejection.
Unlike patients with schizoid personality disorder, they do desire relationships but are paralysed, by
their fear and sensitivity, into social isolation.
Dependent personality disorder Many people exhibit dependent behaviours and traits but people with
dependent personality disorder have an excessive need to be cared for and that results in submissive and
clinging behaviour, regardless of consequences. Diagnosis requires at least five of the following features:
Difficulty making decisions without guidance and reassurance.
Need for others to assume responsibility for most major areas of the person's life.
Difficulty expressing disagreement with others.
Difficulty initiating activities because of lack of confidence.
Excessive measures to obtain nurture and support.
Discomfort or helplessness when alone.
Urgent seeking for another relationship when one has ended.
Unrealistic preoccupation with fears of being left to self-fend.
Obsessive-compulsive personality disorder
People with obsessive-compulsive personality disorder are markedly preoccupied with orderliness,
perfectionism and control.
They lack flexibility or openness.
Their preoccupations interfere with efficient function despite their focus on tasks.
They are often scrupulous and inflexible about matters of morality, ethics and values, to a point
beyond cultural norms.
They are often 'stingy' as well as stubborn.

752. A 17yo lady presents with a worm in her ear. She is very agitated and anxious. What is the next step?
a. Remove under GA
b. Suction
c. Alcohol drops
d. Forceps
753. A 20yo male smoker is noted to have intense rubor of the feet and absent foot pulse. Exam:
amputated right 2nd toe. What is the most probable dx?
a. Intermittent claudication
b. Cardiovascular syphilis
c. Buergers disease
d. Embolism
e. Acute limb ischemia
754. A young lady after a heavy bout of drinking last night comes to the ED with dizziness, abdominal pain,
vomiting blood with cool peripheries. After initial resuscitation, oxygen and fluids, she still continues to bleed
with pulse=130bpm and BP=85/58mmHg. What would be your next best management?
a. Clotting screen
b. US
c. CT
d. Endoscopy
e. Omeprazole
755. A 12yo boy complains of acute development of purpura on the dependent areas of his body 2wks after
a URTI. The purpura doesnt blanch on pressure, tests reveal: Hgb=12, plts=50, WBC=5. Bleeding time=10mins,
APTT=40s, PT=1.02. What is the most likely dx?
a. ITP
b. TTP
c. Von Willebrands disease
d. Hemophilia A
e. Hemophilia B
756. A woman presents with hx of falls, becomes pale and clumsy. She is hypertensive and takes atenolol,
bendroflumethiazide and amlodipine. What inv is needed?
a. 24h ECG
b. 24h BP monitoring
c. ECG
d. Echo
e. CT head
757. A 43yo woman has been feeling lethargic and tired. Her BP=160/90mmHg. Bloods: Na+=140mmol/L,
K+=3.1mmol/L. What is the most likely dx?
a. Cushings syndrome
b. Conns syndrome
c. Hyperparathyroidism
d. Renal disease
e. Pheochromocytoma
Consider Conn's syndrome if hypertension, hypokalemia or alkalosis in patient not on diuretics.
Causes of hypertension with hypokalemia
1. Essential HTN with diuretic use
2. Conns
3. Cushings
4. Pheochromocytoma
5. Malignant hypertension

758. A 2yo child aspirated a foreign object which was removed at the hosp. the parents are now asking how
to remove it if that ever happens at home. What do you advise?
a. Hemlich maneuver
b. Bring to the hospital
c. Turn the child on his back and give thumps
d. CPR
e. Remove manually by fingers
759. A 28yo pregnant lady presents with severe lower abdominal pain with excessive per vaginal bleeding
at 34wks gestation. What should be the initial inv of choice?
a. Coagulation profile
b. US abdomen
c. CT pelvis
d. D-dimer
e. Kleiuber test
760. A 3yo child with severe diarrhea and vomiting, looks lethargic, has sunken eyes and a feeble cry. What
is the choice of fluids?
a. 0.9%NS
b. 0.9%NS + 5%Dextrose
c. 0.45%NS + 5%Dextrose
d. 0.45%NS
761. A lady with depression has a bag full of meds. She now presents with coarse tremors. Which drug
caused her symptoms?
a. Lithium
b. Thyroxine
c. Amitriptyline
d. Sodium valproate
e. Tetrabenazine
762. A 38yo man has had a liver biopsy as part of inv for abnormal LFTs. The p athologist report states:
special stains demonstrate the presence of a very large amount of iron pigment within the hepatocytes. What
single condition is identified by the pathologists report?
a. Alpha 1 antitrypsin deficiency
b. Hemangioma
c. Hemochromatosis
d. Hemosiderosis
e. Wilsons disease
The term "hemosiderosis" is used to denote a relatively benign accumulation of iron. The term
"hemochromatosis" is used when organ dysfunction occurs..A Prussian blue iron stain demonstrates the blue
granules of hemosiderin in hepatocytes and Kupffer cells. Hence i think pathologist will describe it as
hemosiderosis whereas clinicians after ruling out the organ dysfunction termed it as a hemochromatosis.. SO i
think answer here would be D ( hemosiderosis)because question is asking about pathologist report.

763. A 27yo man presents to the ED after a RTA where his foot was stuck under a truck for several hours.
He now has swelling on the foot. Exam: foot tenderness, loss of sense in the space between the 3 rd metatarsal
and big toe and his dorsalis pedis is not felt. What is the most likely dx?
a. Compartment syndrome
b. Arterial rupture
c. Arterial embolus
d. DVT
e. Fibular fx
764. You are a FY doctor in the ED when a mother brings her 2yo son to you with a 1h hx of noisy breathing.
She state that although he had mild coryza over the last week, he was improving and so they had gone to a
childrens picnic with nursery friends. Another parent had found him coughing and spluttering, and ever since
his breathing has remained noisy. Though he appears well in the ED, his current observations demonstrate a
raised RR and sat=91% on air. What is the most likely dx?
a. Anaphylaxis
b. Croup
c. Foreign body aspiration
d. Epiglottitis
765. A pt taking doxycycline complains of nausea, indigestion, abdominal pain and vomiting. What will you
advise?
a. Take it after meals
b. Take it before meals
c. Stop the drug
d. Take antiacids
e. Take antiemetic

766. A 49yo man lost his job and now is homeless. He was found wandering in the park. He is muttering that
some people are after him. Alcohol was tested and it was negative. What will your next step be?
a. Thiamine
b. Neuropsycho analysis
c. Mini mental state
d. CT head
e. MRI head
ans: Middle aged man, presenting with persecutory delusions. Age is not in favour of schizophrenia. There are
stressors present, loss of job and homeless. Substance abuse ruled out for alcohol. Not other drugs ruled out.
So go for neuropsychiatric analysis to find out cause. Ct would have been indicated after trauma, mri for focal
symptoms. Thiamine in alcohol intoxication.Minimental is specific for cognition and wouldnt point towards
diagnosis.
767. A pt with SNHL and loss of corneal reflex on the left side. What is the most definitive inv?
a. CT of internal auditory meatus
b. Nuclear imaging of brain
c. MRI of internal auditory meatus
d. Radio isotope scan
e. XR skull
Unilateral sensorineural hearing loss and loss of corneal reflex indicate involvement of facial, trigeminal and
vestibulocochlear nerve which is common in acoustic neuroma. MRI of internal auditory meatus and CP angle
would show the tumour.
Acoustic Neuroma:
Presentation: progressive ipsilateral tinnitus, sensorineural deafness. Big tumours may cause ipsilateral
cerebellar signs or raised ICP. 5, 6, 7 cranial nerves at risk.
Investigations: MRI
TREatment: surgery

768. A middle aged man complains of a node which has been growing on his nose for several months. Now
its firm with central depression. It is 0.6cm in size. What is the single most likely dx?
a. Basal cell carcinoma
b. Squamous cell carcinoma
c. LN
d. Melanoma
e. Kaposis sarcoma
f. Keratoacanthoma
g. Molluscumcontagiosum
Basal Cell carcinoma presents with pearly nodule with rolled telangiectatic edge on the face. Lesions on trunk
can appear as red scaly plaques with raised smooth edge.
Causes: uv exposure
Treatment: excision and radiotherapy
769. A 45yo woman presents with rotational vertigo, nausea and vomiting, especially on moving her head.
She also had a similar episode 2yrs back. These episodes typically follow an event of runny nose, cold, cough
and fever. What is the most probable dx?
a. Acoustic neuroma
b. Menieres disease
c. Labyrinthitis
d. BPPV
e. Vestibular neuronitis
above scenario is that of vestibular neuronitis.
Symptoms of rotational vertigo nausea and vomiting followed an event of viral illness.
Acoustic neuroma, would have some other cranial nerve involvement.
Menieres is characterised by spisodes of vertigo, sensorineural hearing loss, sense of aural fullness and
tinnitus.
BPPV is characterised by vertigo and dizziness related to head movement.
Labyrinthitis resembles vestibular neuronitis but with additional hearing loss.
Presentation: nausea, vomiting and vertigo .moving head aggravates symptoms.
Nystagmus
Investigations: clinical diagnosis.
MRI ct to rule out other causes
Differntials: BPPV, labyrinthitis, Meniers
Management: reassurance. Bed rest.If severe, antiemetics.Prochlorperazine.
770. A 65yo man with cancer of middle 1/3 of the esophagus presents with dysphagia. What is the most
immediate management?
a. Chemotherapy
b. Radiotherapy
c. Stenting
d. Gastrostomy
e. TPN
Squamous cell carcinoma of the esophagus.Presenting with dysphagia.
Treatment: for early esophageal cancer, endoscopic resection or submucosal dissection
Endoscopic Esophagectomy TOC for most advanced
Stenting is the first line approach to assist swallowing.
Radio chemo may be of used primarily in reducing bulk of tumour,
Nutritional status: use liguid feeds or PEG tubes.
771. A 1yo child is brought to ED. He woke up in the middle of the night crying severely. What initial
measure should be taken for this child?
a. Refer to surgeon
b. Discharge with advice
c. Analgesia
d. Antibiotic
information provided seems insufficient.
However, an infant can only express underlying pain by crying. So immediate treatment could be analgesia to
relax and baby and proceed with further examination to find out the cause.
772. A 30yo lady was playing volleyball when her hand got injured with the ball. The right hand is not
swollen and there is tenderness under the root of the thumb. XR is normal. What is the most appropriate next
management?
a. Arm sling for 1 wk
b. Raise had for 2d
c. Repeat XR
d. Full arm cast
773. A 33yo female presents with sudden severe colicky abdominal pain in her RIF. A mobile mass is felt on
examination. What is the most likely dx?
a. PID
b. Appendicitis
c. Ovarian torsion
d. Constipation
Severe colicky abdominal pain in rif could be ureteric colic or ovarian torsion.
Mobile mass and female gender indicates its ovarian torsion
Ovarian Torsion: Usually happens when ovaries enlarge via cysts. There may be adnexal tenderness.
Ultrasound: torsion may be intermittent presenting with intermittent pain.
Treatment: initially by laparoscopy and possible oophoropexy.
If severe vascular compromise or peritonitis: salpingo oophorectomy.
774. A middle aged male is feeling unwell after a recent MI. The recent ECG shows prolonged QRS complex
and tented T wave. Na+=136mmol/l, K+=6.2mmol/l, urea=5mmol/l. What is the most appropriate
management?
a. Calcium gluconate
b. Calcium resonium
c. Calcium with vit D supplement
d. Vit D
e. Calcium
Ecg abnormalities and a raised potassium level needs immediate treatment of hyperkalemia.
Symptoms: fast irregular pulse, chest pain, palpitations or weakness
Ecg: tall tented T waves, small P waves, wide QRS, vfib
Causes: oliguric renal failure, potassium sparing diuretics, rhabdomyolysis, metabolic acidosis, burn
Treatment:
Urgent or Non urgent
Urgent: 10 ml calcium gluconate over 2 minutes
Insulin+ glucose
Nebulise with salbutamol
Calcium resonium
Dialysis
775. A 68yo man with DM and HTN was noted to have cholesterol level of 3.4mmol/l. he was also noted to
have microalbuminuria. What is the best drug to add to his regimen?
a. ACEi
b. Statin
c. Amylodipine
d. Biguanides
Cholesterol levels are normal.
In hypertension and diabetes, acei are the drugs employed to slow down renal progression of disease. They
are the DOC for proteinuria.
776. A child playing with toys suddenly develops breathlessness and stridor. Which inv will lead to the dx?
a. Laryngoscopy
b. CXR
c. Peak flow meter
d. ABG
scenario of foreign body intake by the child. He has developed breathlessness and stridor indicating airway
obstruction. So laryngoscopy will help in visualising and extraction of the foreign body.
777. Anxious parents ask you for resus technique for their 3yo. What do you tell them?
a. 5 compression: 1 breath
b. 5 compression: 2 breaths
c. 15 compression: 2 breaths with nose pinched
d. 15 compression: 2 breaths without nose pinched
e. 30 breaths: 2 compressions
778. A 56yo man complains of increased vol of sputum with specks of blood and chest pain. He has ahx of
DVT. Exam: clubbing. What is the cause of blood in his sputum?
a. Pulmonary thrombosis
b. Bronchial carcinoma
c. Bronchiectasis
d. Pulmonary TB
Bronchiectasis
permanent dilatation and thickeningof airways characterised by cough, copious purulent sputum, recurrent
infection
signs: coarse crackles, often in lower zones.
Ronchi
Wheeze
Inv:
CXR
HRCT gold standard
Sputum microbiology
Differentials: COPD, Asthma, TB, pneumonia, sinusitis
Treatment: chest physio, postural drainage
Stop smoking
Antibiotics
Amoxicillin is first line
Cipro in pseudomonas patients for 14 days
779. A 32yo female has ahx of SOB and fever. Pre-broncho-dilation test was done and it was 2/3.5 and post-
bronchodilator was 3/3.7. The pt was dx of eczema and TB. What is the possible dx?
a. COPD
b. Asthma
c. Pneumonia
d. Bronchiectasis
Asthma:
Shortness of breath and fever alongwith eczema hx. Points toward asthma
Precipitants: cold exercise nsaid b blockers smoking infection
Associated with acid reflux and other atopic disease
Diagnosis:
Fev1 is reduced more than fvc and ratio is less than 75: obstructive defect
Greather than 15% improvent in fev1 with bronchodilators as in this scenario.
Management
Step 1: short acting b agonist inhaler, salbutamol
If not controlled or night symptoms, use more than twice a week
Step two: add low dose inhaled corticosteroid
Step 3 add long acting b agonist. If little improvement inc dose of inhaled corticosteroid. If no improvement,
stop LABA. And add leukotriene receptor antagonist
Step 4: increase inhaled corticosteroid or oral t5heophylline
Step 5 oral corticosteroid
780. A 2yo male pt was brought by his mother with a swelling in the right side of his neck extending from
the angle of the mouth to the middle 1/3 of the sternocleidomastoid muscle. The swelling was on the
anterolateral side of the sternocleidomastoid and was brilliantly transilluminant. What is the likely dx?
a. Lymphangioma
b. Branchial cyst
c. Thyroglossal cyst
d. Ranula
e. Thyroid swelling
Branchial Cyst:
Commonly presents as painless mass in the neck of a child. Discharge maybe associated if present with sinus
tract. Smooth non tender fluctuant masses along the lower one thord of anterior border of SCM
Differentials: lymphadenopathy
Carotid body tumour
Cystic hygroma
Ectopic thyroid
Investigation
Ultrasound
Ct
Surgical excision
781. A 50yo newly dx with HTN complains of urinary freq and dysuria. The urinalysis reveals presence of
white cells and protein. Choose the single most appropriate tx?
a. Imipramine
b. Adjust diuretics
c. Vaginal estrogen
d. Trimethoprim
bacterial growth of 10 power 5 org/ml. in MSU
urethritis, cystitis,
upperuti: pyelonephritis
e.coli main organism, strep, proteus
PC: frequency, urgency, dysuria, hematuria, suprapubic pain.
Inv: urine dipstick
MSU
Urine CS
Rx: trimethoprim or nitrofurantoin
2nd line: co amoxiclav
782. A boy injured his ear during a rugby match. He reported it being painful. Exam: red and tender pinna.
Tympanic membrane was normal. What would be the next appropriate step?
a. Topical gentamicin
b. Oral flucloxacillin
c. IV flucloxacillin
d. Refer to ENT specialist
e. No further intervention needed
scenario most probably of perichondrial hematoma.
If not surgically explored and aspirated can lead to malformed ear or cauliflower ear which is cosmetically
undesirable. Common in boxers
783. A 2yo girl prv well presents with ahx of vomiting and diarrhea for 4hrs. What is the most suitable
indication for IV fluid administration?
a. Capillary refill time >4s
b. HR >90bpm
c. Increased RR
d. Stool >10x/d
e. Weight of child = 10kgs
784. A 44yo woman with breast cancer had an extensive removal and LN clearance. She needs an
adjunctive tx. Her mother had cancer when she was 65. Which of the following factors will be against the tx?
a. Famhx
b. Tumor grading
c. LN involvement
d. Her age
adjunctivetx is given in LN involvement. Tumor grading should also be done to stage it.
785. A 45yo man presents with hearing loss and tinnitus in the right ear. Exam: weber test lateralizes to the
left. Audiometry: AC > BC in both ears. What is the next best inv?
a. CT
b. MRI brain
c. Angiogram
d. Otoscopy
Unilateral sensorineural hearing loss and loss of corneal reflex indicate involvement of facial, trigeminal and
vestibulocochlear nerve which is common in acoustic neuroma. MRI of internal auditory meatus and CP angle
would show the tumour.
Acoustic Neuroma:
Presentation: progressive ipsilateral tinnitus, sensorineural deafness. Big tumours may cause ipsilateral
cerebellar signs or raised ICP. 5, 6, 7 cranial nerves at risk.
Investigations: MRI
TREatment: surgery
786. A 74yo lady called an ambulance for an acute chest pain. She has ahx of DM and HTN, and is a heavy
smoker. Paramedics mentioned that she was overweight and recently immobile because of a hip pain. She
collapsed and died in the ambulance. What is the most likely cause of death?
a. Pulmonary embolism
b. MI
c. Stroke
d. Cardiac arrhythmia
e. Cardiac failure
acute onset of chest pain and hx of immobility point towards pul embolism.
Causes: DVT
After long bone fracture
Amniotic fluid
Air embolism
Immobility

Risk factors for venous thromboembolism[3]

Major risk factors: relative risk of 5-20 Minor risk factors: relative risk of 2-4

Cardiovascular:
Surgery:
Congenital heart disease.

Congestive cardiac failure.


Major abdominal/pelvic surgery
Hypertension.
or hip/knee replacement
Paralytic stroke.
(risk lower if prophylaxis used).
Oestrogens:
Postoperative intensive care.
Pregnancy (but see major risk factors for
Obstetrics:
late pregnancy and puerperium).
Late pregnancy. Combined oral contraceptive.

Puerperium. Hormone replacement therapy.

Caesarean section. Haematological:

Lower limb problems: Thrombotic disorders (a detailed list is

Fracture. available)

Varicose veins - previous Consider this in cases of PE aged <40 years,

varicose vein surgery; recurrent VTE or a positive family history.

superficial thrombophlebitis; varicose veins Myeloproliferative disorders.

per se are not a risk factor. Renal:

Malignancy: Nephrotic syndrome.

Abdominal/pelvic. Chronic dialysis.

Advanced/metastatic. Paroxysmal nocturnal haemoglobinuria.

Reduced mobility: Miscellaneous:

Hospitalisation. Chronic obstructive pulmonary disease

Institutional care. (COPD).

Previous proven VTE: Neurological disability.

Intravenous (IV) drug use Occult malignancy.

(could be major or Long-distance sedentary travel.

minor risk factor: Obesity.

no data on relative risk). Other chronic diseases: inflammatory bowel


disease, Behet's disease.
Other:

Major trauma.

Spinal cord injury.

Central venous lines.


Dyspnoea.
Pleuritic chest pain, retrosternal chest pain.

Cough and haemoptysis.

Any chest symptoms in a patient with symptoms suggesting a deep vein thrombosis (DVT).

In severe cases, right heart failure causes dizziness or syncope.

Signs include:

Tachypnoea, tachycardia.

Hypoxia, which may cause anxiety, restlessness, agitation and impaired consciousness.

Pyrexia.

Elevated jugular venous pressure.

Gallop heart rhythm, a widely split second heart sound, tricuspidregurgitant murmur.

Pleural rub.

Systemic hypotension and cardiac arrest

Offer patients in whom PE is suspected and with a likely two-level PE Wells' score either an immediate

computed tomography pulmonary angiogram (CTPA) or immediate interim parenteral anticoagulant

therapy followed by a CTPA, if a CTPA cannot be carried out immediately. Consider a proximal leg vein

ultrasound scan if the CTPA is negative and DVT is suspected.

Offer patients in whom PE is suspected and with an unlikely two-level PE Wells' score a D-dimer test

and, if the result is positive, offer either an immediate CTPA or immediate interim parenteral

anticoagulant therapy followed by a CTPA, if a CTPA cannot be carried out immediately.

For patients who have an allergy to contrast media, or who have renal impairment, or whose risk from

irradiation is high:

o Assess the suitability of a ventilation/perfusion single-photon emission computed tomography

(V/Q SPECT) scan or, if a V/Q SPECT scan is not available, a V/Q planar scan, as an alternative to

CTPA.
o If offering a V/Q SPECT or planar scan that will not be available immediately, offer immediate

interim parenteral anticoagulant therapy.

Diagnose PE and treat patients with a positive CTPA or in whom PE is identified with a V/Q SPECT or

planar scan.

Consider alternative diagnoses in the following two groups of patients:

Patients with an unlikely two-level PE Wells' score and either a negative D-dimer test, or a positive D-

dimer test and a negative CTPA.

Patients with a likely two-level PE Wells' score and both a negative CTPA and no suspected DVT.

nitial resuscitation
Oxygen 100%.

Obtain IV access, monitor closely, start baseline investigations.

Give analgesia if necessary (eg, morphine).

Assess circulation: suspect massive PE if systolic BP is <90 mm Hg or there is a fall of 40 mm Hg, for

15 minutes, not due to other causes.

Anticoagulation therapy[4]
Offer a choice of low molecular weight heparin (LMWH) or fondaparinux to patients with confirmed

PE, with the following exceptions:

For patients with severe renal impairment or established chronic kidney disease (estimated

glomerular filtration rate (eGFR) <30 ml/min/1.73 m2) offer unfractionated heparin (UFH) with dose

adjustments based on the activated partial thromboplastin time (aPTT) or LMWH with dose

adjustments based on an anti-Xa assay.

For patients with an increased risk of bleeding, consider UFH.

For patients with PE and haemodynamic instability, offer UFH and consider thrombolytic

therapy.
Start the LMWH, fondaparinux or UFH as soon as possible and continue it for at least five days or

until the international normalised ratio (INR) is 2 or above for at least 24 hours, whichever is longer.

Offer LMWH to patients with active cancer and confirmed PE, and continue the LMWH for six

months. At six months, assess the risks and benefits of continuing anticoagulation.

Offer a vitamin K antagonist (VKA) to patients with confirmed PE within 24 hours of diagnosis and

continue the VKA for three months. At three months, assess the risks and benefits of continuing VKA

treatment.

Offer a VKA beyond three months to patients with an unprovoked PE, taking into account the

patient's risk of VTE recurrence and whether they are at increased risk of bleeding.

Rivaroxaban:[7]

Rivaroxaban is recommended by NICE as an option for treating PE and preventing recurrent

DVT and PE in adults.

The duration of treatment recommended depends on bleeding risk and other clinical criteria.

Short-term treatment (at least three months) is recommended for people with transient risk

factors such as recent surgery and trauma. Longer treatment is recommended for people with

permanent risk factors, or idiopathic (unprovoked) DVT or PE.

nitial resuscitation
Oxygen 100%.

Obtain IV access, monitor closely, start baseline investigations.

Give analgesia if necessary (eg, morphine).

Assess circulation: suspect massive PE if systolic BP is <90 mm Hg or there is a fall of 40 mm Hg, for

15 minutes, not due to other causes.

Anticoagulation therapy[4]
Offer a choice of low molecular weight heparin (LMWH) or fondaparinux to patients with confirmed

PE, with the following exceptions:


For patients with severe renal impairment or established chronic kidney disease (estimated

glomerular filtration rate (eGFR) <30 ml/min/1.73 m2) offer unfractionated heparin (UFH) with dose

adjustments based on the activated partial thromboplastin time (aPTT) or LMWH with dose

adjustments based on an anti-Xa assay.

For patients with an increased risk of bleeding, consider UFH.

For patients with PE and haemodynamic instability, offer UFH and consider thrombolytic

therapy.

Start the LMWH, fondaparinux or UFH as soon as possible and continue it for at least five days or

until the international normalised ratio (INR) is 2 or above for at least 24 hours, whichever is longer.

Offer LMWH to patients with active cancer and confirmed PE, and continue the LMWH for six

months. At six months, assess the risks and benefits of continuing anticoagulation.

Offer a vitamin K antagonist (VKA) to patients with confirmed PE within 24 hours of diagnosis and

continue the VKA for three months. At three months, assess the risks and benefits of continuing VKA

treatment.

Offer a VKA beyond three months to patients with an unprovoked PE, taking into account the

patient's risk of VTE recurrence and whether they are at increased risk of bleeding.

Rivaroxaban:[7]

Rivaroxaban is recommended by NICE as an option for treating PE and preventing recurrent

DVT and PE in adults.

The duration of treatment recommended depends on bleeding risk and other clinical criteria.

Short-term treatment (at least three months) is recommended for people with transient risk

factors such as recent surgery and trauma. Longer treatment is recommended for people with

permanent risk factors, or idiopathic (unprovoked) DVT or PE.

787. A 77yo woman suffered diarrhea 4d ago. What would be her blood gas result?
a. pH =7.2, PaCO2 =8
b. pH =7.4, PaCO2 =1.5
c. pH =7.4, PaCO2 =2.6
d. pH =7.4, PaCO2 =2.8
Diarrhea is a cause of metabolic acidosis. In the initial states there will be respiratory compensation leading to
normalisation of ph and low carbondioxide by washing out carbondioxide. With time, the kidney will also save
bicarbonate and bring ph back to normal. In this question, b, c, d all are suitable but best is option B.
788. A man presents with abdominal pain, vomiting, pulse=120bpm, BP=90/60mmHg and a rigid abdomen.
His chest is clear. What is the immediate management?
a. Call radiology
b. Admit to medical ward
c. Urgent admission to ITU
patient presenting with acute abdomen. Peritonitis. First thing would be to shift him to ICU and stabilise.
789. A 60yo man presents with weight loss and Hgb=6. Hx reveals that he has abdominal pain and diarrhea
for the past 6m. What is the most appropriate inv?
a. Barium enema
b. Colonoscopy
c. Sigmoidoscopy
presenting features are consistent with Colonic carcinoma.

Risk factors
Family history of colorectal neoplasia: carcinoma; adenoma under the age of 60 years.[3]

Past history of colorectal neoplasm: carcinoma, adenoma.

Inflammatory bowel disease: ulcerative colitis, Crohn's colitis.

Polyposis syndromes: familial adenomatous polyposis (Gardner's syndrome), Turcot's syndrome,

attenuated adenomatous polyposis coli, flat adenoma syndrome, hamartomatous polyposis syndromes

(Peutz-Jeghers syndrome, juvenile polyposis syndrome, Cowden's syndrome).

Hereditary non-polyposis colorectal cancer (HNPCC).

Hormonal factors: nulliparity, late age at first pregnancy, early menopause.

Diet: rich in meat and fat; poor in fibre, folate and calcium.

Sedentary lifestyle, obesity, smoking, high alcohol intake.

Diabetes mellitus.

Previous irradiation, occupational hazards - eg, asbestos exposure.

History of small bowel cancer, endometrial cancer, breast cancer or ovarian cancer.

Presentation
The presentation depends on the site of the cancer:
Right colon cancers: weight loss, anaemia, occult bleeding, mass in right iliac fossa, disease

more likely to be advanced at presentation.

Left colon cancers: often colicky pain, rectal bleeding, bowel obstruction, tenesmus, mass in

left iliac fossa, early change in bowel habit, less advanced disease at presentation.

The most common presenting symptoms and signs of cancer or large polyps are rectal bleeding,
persisting change in bowel habit and anaemia.
All patients with symptoms suspicious of colorectal cancer must have a thorough abdominal
examination and rectal examination.
Inv:
Colonoscopy and biopsy
Ct for staging
Rx
Surgery
Chemo radio for advanced cases
790. A 35yo primigravida post C-section complains of inability to void. She denies dysuria but complains of
fullness. She was treated with an epidural for analgesia. What is the single most appropriate inv?
a. MSU
b. US abdomen
c. US KUB
d. Serum calcium

Answer= C. US KUB. This is the case of postoperative urinary retention (the risk factors for it include operation
itself and spinal anesthesia) so US KUB should be done to measure the urinary volume of bladder. it will guide
in the treatment plan. if the urine volume is <400ml observe the patient and if the urine volume is > 600ml
catheterize the patient.

791. A 35yo primigravida post C-section complains of inability to void. She denies dysuria but complains of
fullness. She was treated with an epidural for analgesia. What is the single most appropriate inv?
a. MSU
b. US abdomen
c. US KUB
d. Serum calcium
Answer= C. US KUB. This is the case of postoperative urinary retention (the risk factors for it include operation
itself and spinal anesthesia) so US KUB should be done to measure the urinary volume of bladder. it will guide
in the treatment plan. if the urine volume is <400ml observe the patient and if the urine volume is > 600ml
catheterize the patient.

792. A female pt with HTN and serum K+=2.7mmol/l. which hormone would you be looking for?
a. Aldosterone
b. Cortisol
c. Thyrotoxin
d. Renin
e. Testosterone
Answer= A. Aldosterone. hypertension with hypokalemia points towards the diagnosis of primary
hyperaldosteronism so aldosterone levels will be high in this patient.

PRIMARY HYPERALDOSTERONISM
Primary hyperaldosteronism was previously thought to be most commonly caused by an adrenal adenoma,
termed Conn's syndrome. However, recent studies have shown that bilateral idiopathic adrenal hyperplasia is
the cause in up to 70% of cases. Differentiating between the two is important as this determines treatment.
Adrenal carcinoma is an extremely rare cause of primary hyperaldosteronism

Features
hypertension
hypokalaemia (e.g. muscle weakness)
alkalosis

Investigations
high serum aldosterone
low serum renin
high-resolution CT abdomen
adrenal vein sampling

Management
adrenal adenoma: surgery
bilateral adrenocortical hyperplasia: aldosterone antagonist e.g. spironolactone

793. A 25yo male with a hx of frequent binge drinking presents 4h after having had a take away meal
following a nights heavy drinking. He complains of nausea and has vomited on several occasions. After the last
vomiting episode, he vomited approximately a cupful of blood. On admission, he smells of alcohol,
pulse=100bpm, BP=140/80mmHg. He has some tenderness in the epigastrium. What is your dx?
a. Gastric carcinoma
b. Mallory-weiss tear
c. Esophageal carcinoma
d. Esophageal varices
e. Esophageal varices
f. Peptic ulceration
B. Mallory-weiss tear. Alcoholic patient with the complains of several episodes of vomiting followed by
hematemesis all point towards the diagnosis of mallory weiss tear.
MalloryWeiss tear
Persistent vomiting/retching causes haematemesis via an oesophageal mucosal tear

Aetiology:
Haematemesis due to a Mallory Weiss tear usually occurs after a prolonged or forceful bout of retching,
vomiting, coughing, straining or even hiccupping.
Risk factors:
excessive alcohol consumption.
conditions predisposing to retching and vomiting: e.g gastroenteritis, hyperemesis gravidarum,bulimia, renal
disease,hepatitis, migraine, raised icp etc.

Presentation:
The classic presentation is of haematemesis following a bout of retching or vomiting. However, a tear may
occur after a single vomit.
Other symptoms include melaena, light-headedness, dizziness, or syncope, and features associated with the
initial cause of the vomiting - eg, abdominal pain

Diagnosis:
Endoscopy is the primary diagnostic investigation.

Treatment:
Resuscitation is a priority - maintain airway, provide high-flow oxygen, correct fluid losses.
MWS usually follows a benign course but occasionally endoscopic treatment is required to stop bleeding.

794. A young boy presented with bilateral periorbital edema, ankle swelling and increase in body weight.
What is the most likely dx?
a. Chronic heart failure
b. Nephrotic syndrome
c. Renal failure
d. Acute heart failure
e. Glomerulonephritis
Answer: B. Nephrotic syndrome.
NEPHROTIC SYNDROME
The nephrotic syndrome is a triad of:
Proteinuria >3.5g/24h (ACR >250 mg/mmol)
Hypoalbuminemia (<25g/L, usually much lower)
Oedema
Severe hyperlipidaemia (total cholesterol >10 mmol/L) is often present
causes:
It can be due to primary renal disease or secondary to a number of systemic disorders.
Primary causes: Minimal change disease, membranous nephropathy, focal segmental glomerulosclerosis
(FSGS), mesangiocapillary GN (MCGN).
Secondary causes: Hepatitis B/C (usually membranous, hep C can cause MCGN),
SLE (class V lupus nephritis causes a membranous pattern), diabetic nephropathy,
amyloidosis, paraneoplastic (usually membranous pattern) or drug related (again
usually membranousNSAIDS, penicillamine, anti-TNF, gold).
Treatment:
(In children the peak incidence is between 2 and 5 years of age. Around 80% of cases in children are due to a
condition called minimal change glomerulonephritis. The condition generally carries a good prognosis with
around 90% of cases responding to high-dose oral steroids.)
Reduce oedema: Loop diuretics, eg furosemide are used, often high doses are
needed.
Reduce proteinuria: ACE-i or ARB should be started in all patients.
Reduce risk of complications
Treat underlying condition.
795. A 53yo man with prv hx of COPD presents with breathlessness and purulent sputum. O2 stat=85% on air.
ABG: PaO2=7.6, PaCOS=7. What is the appropriate management for his condition?
a. 24% oxygen
b. Mechanical ventilation
c. 100% oxygen
d. Nebulized salbutamol
Answer= A. 24% oxygen. this is acute exacerbation of copd so 24% oxygen is given first.
796. A 34yo man was involved in a RTA and whilst in the ambulance his GCS deteriorated and RR increased
from 30-48. What is the most appropriate management for this pt?
a. IV fluid
b. Needle thoracocentesis
c. 100% oxygen
d. Portable XR
Answer= C. 100% oxygen. RTA case so we will follow ABC protocol.
The ATLS programme uses the ABCDE mantra.
It prioritizes direct treatment according to the most life-threatening injury identified and avoids delay.
Remember to act immediately
Primary survey
A=Airway + O2 + cervical spine. Approach the patient with arms ready to immobilize
Assess the airway; jaw thrust can be used to help maintain patency. Give 100% 02 to all patients.
B=Breathing + ventilation. Check air entry with auscultation; also auscultate
the heart; inspect, palpate and percuss the chest wall for further evidence of
injury. Check RR.
C=Circulation + haemorrhage control. Check GCS/AVPU, skin perfusion, BP &
pulse. Control any visible haemorrhage with local pressure, and consider possible sources of occult
haemorrhage if no source identified but the patient is shocked, ie examine abdo, pelvis, femoral.
Get 2L of warmed Ringers lactate solution/Hartmanns solution/0.9% saline running stat via 2 separate points
of venous access

D=Disability. Check GCS (if not already done), pupillary reflexes, gross evidence of a lateralizing injury or
spinal cord level.

E=Exposure. Check and maintain body temperature using rewarming


methods. Totally undress the patient, cutting all clothes off if necessary.

Adjuncts to the primary survey can add life-saving information: CXR, lateral C Spine X-ray and pelvic X-ray .
urinary catheter to accurately assess urine output (exclude urethral injury first);
NGT insertion, O2 sats and then ABG to accurately
assess oxygenation.

Secondary survey:
Now the patient is stabilized. More focused imaging can take
place. Includes more focused examination (eg PR exam, otoscopy) and tests (eg
limb x-ray, full C-spine series).

797. A 44yo lady who has PCKD is concerned because her 38yo brother has just died of an intracranial
insult. She knows he was not hypertensive. What was the most likely cause of her brothers death?
a. Subdural hematoma
b. Subarachnoid hemorrhage
c. Cerebral infarct
d. Epidural hematoma
Answer: B. subarachnoid hemorrhage. subarachnoid hemorrhage is mainly due to rupture of berry aneurysm (
which is associated with adult polycystic kidney disease, ehler danlos syndrome and coarctation of aorta)

SUBARACHNOID HEMORRHAGE
Causes
85% are due to rupture of berry aneurysms (conditions associated with berry aneurysms include adult
polycystic kidney disease, Ehlers-Danlos syndrome and coarctation of the aorta)
AV malformations
trauma
tumours

Investigations
CT: negative in 5%
lumbar puncture: done after 12 hrs (allowing time for xanthochromia to develop)

Complications
rebleeding (in 30%)
obstructive hydrocephalus (due to blood in ventricles)
vasospasm leading to cerebral ischaemia

Management
neurosurgical opinion: no clear evidence over early surgical intervention against delayed intervention
post-operative nimodipine (e.g. 60mg / 4 hrly, if BP allows) has been shown to reduce the severity of
neurological deficits but doesn't reduce rebleeding.

798. A 36yo male dx with glioblastoma since last 5m has cerebral edema and is on dexamethasone. He has
diarrhea and vomiting for the last 3 days. He has been suffering from repeated falls today. What could be the
possible cause for his falls?
a. Adrenal insufficiency
b. Dehydration
c. Dexamethasone therapy
d. Raised ICP
Answer: A. Adrenal insufficiency. diarrhea, vomiting, falls (d/t postural hypotension) all point towards
diagnosis. It is because of the steroid resistance and these exogenous steroids would have caused atrophy of
adrenal glands.

Adrenal insufficiency is a condition in which there is destruction of the adrenal cortex and subsequent
reduction in the output of adrenal hormones, ie glucocorticoids (cortisol) and/or mineralocorticoids
(aldosterone). There are two types of adrenal insufficiency:

Primary insufficiency (Addison's disease) - there is an inability of the adrenal glands to produce enough
steroid hormones. The most common cause for this in the developed world is autoimmune disease.
Secondary insufficiency - there is inadequate pituitary or hypothalamic stimulation of the adrenal glands.
Presentation:
Often diagnosed late: lean, tanned, tired, tearful weakness, anorexia,dizzy, faints, fl u-like
myalgias/arthralgias. Mood: depression, psychosis, low self esteem.
GI: nausea/vomiting, abdominal pain, diarrhoea/constipation. Think of Addisons in all with unexplained
abdominal pain or vomiting. Pigmented palmar creases & buccal mucosa (ACTH; cross-reacts with melanin
receptors). Postural hypotension.Vitiligo. Signs of critical deterioration: Shock (BP, tachycardia),T, coma.

investigations:
In a patient with suspected Addison's disease the definite investigation is a ACTH stimulation test (short
Synacthen test).If a ACTH stimulation test is not readily available (e.g. in primary care) then sending a 9 am
serum cortisol can be useful:

> 500 nmol/l makes Addison's very unlikely


< 100 nmol/l is definitely abnormal
100-500 nmol/l should prompt a ACTH stimulation test to be performed

Associated electrolyte abnormalities are seen in around one-third of undiagnosed patients:

hyperkalaemia
hyponatraemia
hypoglycaemia
metabolic acidosis
management:
Patients who have Addison's disease are usually given both glucocorticoid and mineralocorticoid replacement
therapy.
This usually means that patients take a combination of:
hydrocortisone: usually given in 2 or 3 divided doses. Patients typically require 20-30 mg per day, with the
majority given in the morning dose
fludrocortisone

799. A 2yo child is brought by his mother. The mother had hearing impairment in her early childhood and is
now concerned about the child. What inv would you do?
a. Audiometry
b. Distraction testing
c. Scratch test
d. Tuning fork
Answer: A. Audiometry. Family history of deafness so audiometry should be done.
800. An 8yo child presents with recurrent abdominal pain, occasional headaches but maintains a good
appetite. Exam: normal. CBC, BUE, etc are normal. What would you do for her next?
a. US abdomen
b. CT head
c. Reassure
d. Analgesics
Answer: C reassure. This is the case of abdominal migraine. Reassurance is the most important treatment in it.

Abdominal migraine. This presents typically as recurrent bouts of generalised abdominal pain associated with
nausea and vomiting but no headache, followed by sleep and recovery. Typical migraines may occur
separately. pain is mostly periumbilical and lasts 1 to 4 hrs sometimes interfering with normal physical activity
and routine.

Diagnosis: clinical. assess triggers like sleep pattern, missing meals etc.
children with abdominal migraine have tendency to develop migraine when they grow up.

Treatment: Reassurance is the most important treatment.


For headaches in children paracetamol can be given.

Muniba / Samar: 814-845


Chishti: 856-865
Hamza: 866-875
Wajiha: 876-950

801. A 78yo gentleman suddenly collapsed. His HR=120bpm, BP=70/40mmHg. Exam: pulsatile mass in
abdomen. What is the most appropriate dx?
a. Aortic aneurysm
b. Mesenteric cyst
c. Umbilical hernia

Key is A aortic aneurysm.


Clincher: Pulsatile mass in abdomen is most likely to be an aortic aneurysm.
Its actually a burst aortic aneurysm which can be predicted by collapsed, tachycardic patient presenting with
hypotension.
ABDOMINAL AORTIC ANEURYSM:

An aneurysm is a permanent and irreversible dilatation of a blood vessel by at least 50% of the normal
expected diameter.
An aneurysm is caused by degradation of the elastic lamellae, a leukocytic infiltrate, enhanced proteolysis and
smooth muscle cell loss. The dilatation affects all three layers of the arterial wall. A false aneurysm
(pseudoaneurysm) is caused by blood leaking through the arterial wall but contained by the adventitia or
surrounding perivascular tissue.

The 'normal' diameter of the abdominal aorta is approximately 2 cm; it increases with age. An abdominal
aneurysm is usually defined as an aortic diameter of 3 cm or greater. Most abdominal aortic aneurysms (AAAs)
arise from below the level of the renal arteries.

Presentation

Unruptured AAA
Most patients with unruptured AAA have no symptoms.
AAA may be an incidental finding on clinical examination, or on scans - ultrasound, CT or MRI. It
may sometimes be visible on a plain X-ray film.
Possible symptoms and signs are:
o Pain in the back, abdomen, loin or groin:
This may be due to pressure on nearby structures. Back pain may be due to
erosion of the vertebral bodies.
NB: severe lumbar pain of recent onset may indicate impending rupture.
o The patient or doctor may find a pulsatile abdominal swelling
o Distal embolisation may produce features of limb ischaemia. The appearance of micro-
embolic lower limb infarcts in a patient with easily palpable pedal pulses suggests a
popliteal or abdominal aneurysm.
o Ureterohydronephrosis can also occur with AAA.
o Inflammation or retroperitoneal fibrosis can complicate AAA and may cause symptoms
- eg, back pain, weight loss and symptoms related to entrapment of adjacent
structures.

Ruptured AAA
NB: the diagnosis may not be obvious. Ruptured AAA should be considered in any patient with
hypotension and atypical abdominal symptoms. Similarly, abdominal pain in a patient with a
known aneurysm or pulsatile mass must be considered as a possible ruptured or rapidly expanding
aneurysm and treated accordingly.
Ruptured AAA may present with:
o Pain in the abdomen, back or loin - the pain may be sudden and severe.
o Syncope, shock or collapse:
The degree of shock varies according to the site of rupture and whether it
is contained - eg, rupture into the peritoneal cavity is usually dramatic, with
death before reaching hospital; whereas rupture into the retroperitoneal
space may be contained initially by a temporary seal forming.

802. A woman prv in good health presents with sudden onset of severe occipital headache and vomiting.
Her only physical sign on exam is a stiff neck. Choose the most likely dx.

a. Subarachnoid hemorrhage
b. Subdural hematoma
c. Cerebellar hemorrhage
d. Migraine
e. Cerebral embolus

Key is A
Any Severe headache of Sudden onset in previously well patient is Subarachnoid hemorrhage unless proven
otherwise.
It may also be described as worst headache of my life.

Subdural hematoma usually presents with history of falls in an alcoholic or elderly patient and may also shows
signs of confusion.

Cerebellar hemorrhage may present with signs and symptoms of cerebellar lesion like lack of balance ,
nystagmus e.t.c.

Migraine presents with unilateral severe throbbing pain with photophobia and phonophobia. It lasts for about
4-72 hours and may occur along with nausea and vomiting and aura.

Cerebral embolus rarely cause headache and presents with focal neurological deficits.

SUBARACHNOID HEMORRHAGE:

Presentation
The most characteristic feature is a sudden explosive headache. This may last a few seconds or even a fraction
of a second.

Sudden explosive headache may be the only symptom in a third of patients.


Of patients who present with a sudden explosive headache as the only symptoms, around 10%
have SAH.
It is difficult to suspect SAH without sudden headache, but with seizure or confusional state - there are
many other more common causes of these presentations, but SAH should be on the list of differential
diagnoses.
Vomiting may occur; although this does not distinguish it from other causes of headache.
Seizures, occur in about 7%. When they do, they are highly suggestive of a haemorrhage.
1-2% of patients with SAH present with an acute confusional state.
Neck stiffness and other signs of meningism may be present, although it usually presents around six
hours after onset of SAH.
Trauma may be confused with SAH if trauma has also occurred. Patients may also cause a motor
vehicle accident as a result of SAH. SAH needs to be on the differential diagnostic list for patients with
altered consciousness, headache or seizure after trauma, particularly if there is disproportionate
headache or neck stiffness.
SAH following head injury causes headache, decreased level of consciousness and hemiparesis. SAH is a
frequent occurrence in traumatic brain injury, and carries a poor prognosis if it is associated with
deterioration in the level of consciousness.

Investigations

CT scanning
If SAH is suspected, CT scanning (without contrast) is the first line in investigation because of the
characteristically hyperdense appearance of blood in the basal cisterns.

Lumbar puncture
CT is negative in 2% of patients with SAH.
If the CT scan is negative but the history is suggestive, lumbar puncture should be undertaken,
providing the scan shows no contra-indications. Around 3% of patients with a negative CT scan will
prove, on lumbar puncture, to have had an SAH.
Lumbar puncture to remove a CSF sample from the lumbar sac should ideally take place over 12
hours after the onset of the headache because if there are red cells in the CSF, sufficient lysis will
have taken place during that time for bilirubin and oxyhaemoglobin to have formed

Angiography
Further investigation should follow immediately acute SAH is confirmed.
After an SAH is confirmed, its origin needs to be determined. If the bleeding is likely to have
originated from an aneurysm, the choice is between cerebral angiography (injecting radiocontrast
through a catheter to the brain arteries) and CT angiography (visualising blood vessels with
radiocontrast on a CT scan) to identify aneurysms. Catheter angiography also offers the possibility
of coiling an aneurysm.

803. A 34yo housemaid presents with headaches in the back of her head for several days and pain on
flexing her neck. What is the most likely cause?
a. Subdural hemorrhage
b. Cervical spondylosis
c. Subarachnoid hemorrhage
d. Meningitis
e. Cluster headache

Key is B
Clincher is 34 year old housemaid which shows that patient has to work with flexed neck for longer time of the
day.
pain on flexion of neck shows nerve root irritation due to compression,herniation or osteophytic outgrowth.

Subarachnoid hemorrhage is usually sudden in onset and does not prevail for several days.

Migraine is a unilateral throbbing pain and is not associated with neck flexion

Cluster headache presents with unilateral headache with unilateral autonomic signs like rhinorrhea,
lacrimation and ptosis

CERVICAL SPONDYLOSIS:
Cervical spondylosis is chronic cervical disc degeneration with herniation of disc material, calcification and
osteophytic outgrowths. As with simple back pain, it is multifactorial in origin, reflecting poor posture, muscle
strain, sporting and occupational activities as well as psychological factors.
Cervical spondylosis undoubtedly contributes to this burden, but may also cause:
Radiculopathy due to compression, stretching or angulation of the cervical nerve roots.
Myelopathy due to compression, compromised blood supply or recurring minor trauma to the
cord.

Symptoms
Cervical pain worsened by movement.
Referred pain (occiput, between the shoulder blades, upper limbs).
Retro-orbital or temporal pain (from C1 to C2).
Cervical stiffness - reversible or irreversible.
Vague numbness, tingling or weakness in upper limbs.
Poor balance.

Signs
Limited range of movement (forward flexion, backward extension, lateral flexion, and rotation to
both sides).
Minor neurological changes like inverted supinator jerks (unless complicated by myelopathy or
radiculopathy).
Poorly localised tenderness.

Investigations
Most patients do not need further investigation and the diagnosis is made on clinical grounds alone.

Plain X-ray of cervical spine showing formation of osteophytes, narrowing of disc spaces with
encroachment of intervertebral foraminae. This is not diagnostic as these findings are common in
normal middle-aged patients.
Patients with neurological abnormality will need magnetic resonance imaging (MRI) of the cervical
spine at an early stage, particularly if they have progressive myelopathy, radiculopathy or
intractable pain.

804. A 40yo man complains of thirst and lethargy. His BP=140/90mmHg, corrected Ca2+=3.7mmol/l. What
is the most appropriate management at this stage?
a. IV fluids
b. Prednisolone
c. IV hydrocortisone
d. Calcium prep

KEY is A
This is Hypercalcemia since CA2+ levels are increased (normal range 2.25-2.5mmol/l).
Increasing the circulating volume with 0.9% saline, helping to increase the urinary output of calcium

Presentation

Presentation of Hypercalcaemia

At levels <2.8 mmol/L At levels <3.5 mmol/L At levels >3.5 mmol/L


All of the previous plus: All of the previous plus:
Polyuria and polydipsia Abdominal pain
Dyspepsia - due to calcium- Muscle weakness Vomiting
regulated release of gastrin Constipation Dehydration
Depression Anorexia and Lethargy
Mild cognitive impairment nausea Cardiac arrhythmias,
Fatigue shortened QT interval
Coma
Pancreatitis
If the hypercalcaemia is long-standing, calcium may be deposited in soft tissues or may result in stone
formation - eg, nephrocalcinosis, nephrolithiasis or chondrocalcinosis.

Acute hypercalcaemia[11]
Treatment should be initiated in hospital on the advice of a specialist and should include:

Increasing the circulating volume with 0.9% saline, helping to increase the urinary output of
calcium.
A loop diuretic such as furosemide. This is occasionally used where there is fluid overload but it
does not reduce serum calcium .
After rehydration, bisphosphonates (which act by reducing bone turnover) should be administered
intravenously. Pamidronate and zolendronic acid are commonly used. Salmon calcitonin may also
be given. It has fewer side-effects than bisphosphonates but is less effective in reducing
hypercalcaemia.[12][13]
Glucocorticoids are useful for hypercalcaemia due to vitamin D toxicity, sarcoidosis and
lymphoma.[13]
Gallium was identified as a useful drug when it was found that patients with malignancy having
gallium scans did not develop hypercalcaemia. It may be given intravenously to patients with
malignant hypercalcaemia who do not respond to bisphosphonates.[14]
Cinacalcet hydrochloride is a calcimimetic (= mimicking the action of calcium) agent that
effectively reduces parathyroid levels in patients with secondary hyperparathyroidism.[12]
A new experimental approach to malignancy-associated hypercalcaemia involves the blockade of
receptor activator of nuclear factor kappa-B ligand, usually abbreviated as RANKL. RANKL is a key
element in the differentiation, function and survival of osteoclasts, which plays an essential role in
removing calcium ions from the bone in response to PTH stimulation.[15] Denosumab, a human
monoclonal antibody that acts in this manner, is licensed for the prevention of osteoporotic
fractures but is also useful for patients with persistent or relapsed hypercalcaemia of
malignancy.[16]
Haemodialysis or peritoneal dialysis may be relevant in patients with severe hypercalcaemia
secondary to renal failure.

805. A 75yo man on digoxin develops weakness in the right upper and lower limbs which resolves within a
few hours. What is the most definitive inv for this condition?
a. Carotid Doppler
b. Angiography
c. CT head
d. Digoxin level

KEY is B its a wrong key.


Correct Key is A.
This is a case of TIA since focal neurological symptoms vanished within few hours of onset.
Also supported by use of digoxin which means patient already had some arrhythmias or heart failure or there
can be digoxin induced heart failure due to which the chance of thromboembolism were increased and TIA
occurred.
One of the major complication of these emboli is Carotid artery stenosis which leads to decreased perfusion of
the cerebral cortex and presentation with stroke or TIA.
Carotid Doppler is the most definitive investigation for this condition.

806. A 35yo man presents with balance problems, headache, SNHL and loss of coreneal reflex on the left
side. What is the most definitive inv?
a. CT scan of internal auditory meatus
b. Nuclear imagine of the brain
c. MRI of internal auditory meatus
d. MRI brain

KEY is D.
Diagnosis- Acoustic Neuroma.
Clincher: balance problem, headache and unilateral SNHL presenting origin from Cerbellopontine angle.
Loss of Corneal reflex shows involvment of trigeminal nerve.

Investigation of Choice for Diagnosing Acoustic Neuroma is MRI scan.

ACOUSTIC NEUROMA:
Acoustic neuromas are tumours of the vestibulocochlear nerve (eighth cranial nerve), arising from the
Schwann cells of the nerve sheath. Most arise from the vestibular portion and only a few arise from the
cochlear (auditory) division

Presentation

Any unilateral sensorineural hearing loss should be considered as caused by an acoustic neuroma until proven
otherwise.

Consider the diagnosis of acoustic neuroma in patients with:

Unilateral or asymmetrical hearing loss or tinnitus, whether progressive or acute in onset.


Impaired facial sensation.

As the tumour spreads, there is an increase in hearing loss and disequilibrium, and symptoms due to
compression of other structures may occur:

Facial pain or numbness due to involvement of the trigeminal nerve.


Facial weakness is uncommon despite the tumour pressing on the facial nerve.
Earache.
Ataxia due to cerebellar compression.
Severe brainstem compression can produce hydrocephalus with visual loss and persistent
headache and even decreased level of consciousness.
Balance problems without other explanation.

807. A 52yo man has a painful, red, photophobic right eye with slightly blurred vision and watering for 3
days. He has had no such episodes in the past. On slit lamp examination there are cells and flare in the ant
chamber and pupil is sluggish to react. What is the single most appropriate clinical dx?
a. Acute close-angle glaucoma
b. Acute conjunctivitis
c. Acute dacrocystitis
d. Acute iritis
e. Corneal foreign body
KEY is B
This is a clear Picture of Acute Iritis. Iritis forms the part of anterior Uveitis.

Anterior uveitis is the term for inflammation which affects the eye's front (anterior) part of the uveal tract.
This can include the iris (iritis) or the iris and the ciliary body (iridocyclitis). It is the most common type of
uveitis

SYMPTOMS AND SIGNS:


this usually affects one eye. The common symptoms are eye pain (usually felt as a dull ache in and around the
eye), redness of your eye, and photophobia (which means you do not like bright light). You may develop
blurred vision or even some visual loss (usually temporary). You may develop headaches and notice that the
pupil of the affected eye may change shape slightly. The pupil may not react to light (normally becomes
smaller) or it may lose its smooth round shape. Your eye may become watery. The symptoms tend to develop
over a few hours or days.

FINDINGS on SLIT LAMP EXAMINATION:


The diagnosis of iritis is confirmed by examining the eye with a slit lamp (a special microscope designed for eye
exams). Your ophthalmologist can see cells (whiteblood cells) and flare (particles of protein) in the fluid that is
produced in the eye.
Two other physical exam findings aid your eye doctor in diagnosing iritis. They include:

Topical anesthetics do not relieve the pain associated with iritis.


Shining light in the normal, unaffected eye causes pain in the affected eye if iritis is present. This is
because shining light in one eye causes both pupils to constrict. Movement of the affected iris causes
pain

808. A 55yo lady with swelling on the abdomen below the umbilicus on the right side. What is the single
most appropriate lymph node?

a. External iliac LN
b. Pre-aortic LN
c. Aortic LN
d. Inguinal LN
e. Iliac LN
f. Submental LN
g. Submandibular LN
h. Deep cervical LN

KEY is D
swelling below the umbilicus in a 55 year old lady is more likely to be of ovarian origin.
809. A 66yo male presents with painful swallowing. What is the most likely dx?
a. Neisseria meningitides
b. Cryptococcus neoformans
c. Candida albicans
d. Isospora belli
e. Mycobacterium avium

KEY is C
Candida infections are more common in old age, diabetics and in immunocompromised
Isospora belli causes an intestinal infection mostly in HIV positive patients.

MAC can cause fevers, diarrhea, malabsorption, as well as loss of appetite and weight loss, and can
disseminate to the bone marrow

Cryptococcus mainly cause fungal meningitis and encephalitis in immunocompromised and HIV positive.

Oesophageal candidiasis

Presentation
Dysphagia, pain on on swallowing food or fluids and/or retrosternal pain, usually with
oropharyngeal candidiasis.
This combination of symptoms is predictive of oesophageal candidiasis.
It is most often associated with treatment of haematopoietic or lymphatic malignancies.
In HIV-positive patients, it is an AIDS-defining illness.

Diagnosis
A therapeutic trial of fluconazole for patients is useful; most patients will respond within 7 days of
treatment.
Definitive diagnosis is by endoscopy.

Management of oesophageal candidiasis


Consider admission to hospital - oesophageal candidiasis is a life-threatening infection.
Treat for 14-21 days; the following treatment options are suggested:
o First-line treatment options:
Oral fluconazole (200-400 mg daily).
Intravenous (IV) fluconazole, an echinocandin - eg, caspofungin, or
amphotericin.[10]
o Second-line drugs are oral itraconazole oral solution, IV posaconazole or IV
voriconazole.
For AIDS patients, HAART is advised to prevent recurrence.

810. A 45yo lady complains of expressible galactorrhea, decreased libido and amenorrhea, weight gain and
easy fatigue. Her serum prolactin levels are 1100 um/l. what is the likely cause of hyperprolactinemia?
a. Hypothyroidism
b. Stress
c. Pregnancy
d. Prolactin secreting pituitary tumor
e. PCOS

KEY is D
Prolactin levels greater than 150-200ng/L are almost always due to a prolactin secreting tumour.
CAUSES OF HYPERPROLACTINEMIA:
'Physiological' causes:

Pregnancy.
Puerperium.
Breast stimulation (including suckling a child).
Stress - physical (including excessive exercise) or psychological - including venepuncture.
Non-fasting sample.
Macroprolactinaemia:
o This refers to prolactin of high molecular mass, mostly complexes of monomeric
prolactin with immunoglobulins (prolactin auto-antibody complexes). These larger
molecules have no bioactivity and a prolonged clearance rate similar to that of
immunoglobulins. Depending on the immunoassay used, macroprolactinaemia may
account for 25% of laboratory documented hyperprolactinaemia. Consider this cause
in an asymptomatic patient with hyperprolactinaemia and consult laboratory staff (see
'Investigations', below).

Intracranial causes:

Pituitary tumours:
o Abnormally high levels of prolactin may be caused by a prolactin-secreting pituitary
tumour or by a non-secreting pituitary tumour that prevents dopamine (prolactin
release-inhibiting hormone) from reaching normal prolactin-producing cells.
o Prolactinomas:
Microprolactinomas (the most common, approximately 90%).
Macroprolactinomas (>10 mm size - approximately 10%).
Pituitary or hypothalamic tumour compressing the pituitary stalk,
egcraniopharyngioma.
Prolactinomas occur in about 20% of patients with multiple endocrine
neoplasia type 1.[4]
Head injury (eg due to disruption of the pituitary stalk).
Brain surgery and radiotherapy.
Post-ictal - within hours of a seizure.

Endocrine and metabolic causes:

Hypothyroidism (due to increased synthesis of TRH).


Cushing's syndrome.
Chronic renal failure.
Severe liver disease.
Polycystic ovarian syndrome.
Coeliac disease (possibly).[5]

Drugs:

Dopamine receptor antagonists, eg domperidone, metoclopramide, neuroleptics.


Dopamine-depleting agents, eg methyldopa.
Antidepressants, e.g tricyclic antidepressants, monoamine-oxidase inhibitors, serotonin reuptake
inhibitors.
Verapamil.
Opiates.
Protease inhibitors.
Bezafibrate.
Omeprazole.
H2-receptor antagonists.
Oestrogens, anti-androgens.
Cyproheptadine.
Cocaine.

Other causes

Chest wall surgery or trauma.


Sarcoidosis.
Langerhan's cell histiocytosis.
Idiopathic - a diagnosis of exclusion.

811. A 42yo female had sudden severe headache and vomiting. She took paracetamol and then collapsed.
What is the most likely dx?
a. SAH
b. Viral encephalitis
c. Meningitis
d. Anaphylaxis

Key is A
any sudden severe headache is subarachnoid hemorrhage unless proven otherwise.

812. Parents of a 3m baby are worried about cot death. What advice would you give?
a. Lay on the back with feet towards head end
b. Lay on the back with feet towards feet end
c. Lay on side
d. Lay on stomach

KEY is A
Cot death is more properly known as sudden infant death syndrome (SIDS). It is the term used to describe the
sudden death of a baby in its sleep where no cause or reason can be found.

Research has shown that the risk of cot death can be reduced. The things you can do fall into four main
categories:

Sleeping - learn the best way for your baby to sleep.


Smoking - create a smoke-free home for your baby.
Symptoms of illness - see a doctor if your baby appears unwell.
Consider breast-feeding.

SLEEPING
The best way for your baby to sleep is as follows. Make sure that any childminders or babysitters know this
too.

Lay your baby on their back to sleep


In other words, do not lay him or her on their front or side. Cot death is less common in babies who sleep on
their back. It is important to put babies on their backs for daytime naps and any other times of sleep, in
addition to the longer night-time sleep.

813. A child born at 36wks developed difficulty in breathing with intercoastal recession and nasal flaring.
His temp is normal but his mother had PROM 48h ago. What is the most likely inv that will lead to tx?
a. Blood culture
b. CXR
c. Stool culture
d. Sputum culture

Key is B
This is Infant Respiratory Distress Syndrome.
since the child is afebrile so this is not Bronchiolitis plus the symptoms appeared right after the delivery.

IRDS:
Infant respiratory distress syndrome (IRDS) is caused by the inadequate production of surfactant in the lungs

Risk factors
Premature delivery.
Male infants.
Infants delivered via caesarean section without maternal labour.
Hypothermia.
Perinatal asphyxia.
Maternal diabetes.
Multiple pregnancy.
Family history of IRDS.
Secondary surfactant deficiency may occur in infants as a result of:[1]

Intrapartum asphyxia.
Pulmonary infection - eg, group B beta-haemolytic streptococcal pneumonia.
Pulmonary haemorrhage.
Meconium aspiration pneumonia.
Oxygen toxicity along with pressure or volume trauma to the lungs.
Congenital diaphragmatic hernia and pulmonary hypoplasia.

Presentation
Usually preterm delivery.
Presents very soon after birth with respiratory distress: tachypnoea, expiratory grunting, subcostal
and intercostal retractions, diminished breath sounds, cyanosis and nasal flaring.

Investigations
Blood gases: respiratory and metabolic acidosis along with hypoxia. Metabolic acidosis results
from poor tissue perfusion.
Pulse oximetry is used as a non-invasive tool to monitor oxygen saturation, which should be
maintained at 85-93%.
CXR.
Monitor FBC, electrolytes, glucose, renal and liver function.
Echocardiogram: diagnosing patent ductus arteriosus (PDA), determine the direction and degree of
shunting, making the diagnosis of pulmonary hypertension and excluding structural heart disease.
Cultures to rule out sepsis.
May rapidly progress to fatigue, apnoea and hypoxia.

814. A 68yo man has had increasing dysphagia for solid food for 3m and has lost 5kgs in weight. What single
inv is most likely to lead to a def dx?
a. Barium swallow
b. CXR
c. CT chest
d. Endoscopy and biopsy
e. Video-fluoroscopy
815. A 24yo male was trying to move his wardrobe but it fell on his thigh. It was there for a very long time
before someone was able to remove it. When he was seen in ED he had casts in his urine but no RBCs. Other
inv showed hypocalcemia and high serum creatinine. What is the cause for his renal failure?
a. Acetylcholine
b. Myoglobin
c. Myotroponin
d. Acetyl acetate

Answer: B
Rhabdomyolysis can be defined as a clinical syndrome associated with the breakdown of skeletal muscle fibres
and myocyte cell membranes, leading to release of muscle contents into the circulation, resulting in multiple
complications, including hyperkalaemia. It is a medical emergency and can lead to cardiac arrest if not
promptly treated. Myoglobinuria is an early feature of rhabdomyolysis, but it may be cleared within a few
hours despite other sequelae continuing.
Pathophysiology:
Myocyte function under normal circumstances is maintained by adenosine triphosphate-dependent channels
which ensure effective cell ion levels and play a role in calcium efflux from myocytes.[1] Damage to the
myocyte membrane (eg, due to trauma or lack of energy for the cell membrane channels) causes an increase
in the amount of calcium in the cell, which leads to apoptosis through various proteolytic enzymes.[1][2] This
leads to the muscle necrosis and the release of various substances into the circulation - eg, myoglobin,
potassium, phosphate, creatine kinase (CK) and urate.[3]
Myoglobin is a skeletal muscle protein involved in metabolism and myoglobinaemia usually occurs before a
rise in CK in rhabdomyolysis. Any myoglobin that reaches the circulation will be filtered by the kidneys and can
lead to acute kidney injury through either direct toxicity or precipitation, or both.[3] This process is facilitated
by an acidotic environment and hypovolaemia.[1] Myoglobin can appear in the urine (myoglobinuria) causing
'tea-coloured' urine with a positive urine dipstick for blood. The latter can cause confusion with haematuria
and haemoglobinuria.

816. Anatomical structure to be pierced during surgery midline port during gallstone removal.

a. External iliac muscle


b. Cricoid cartilage
c. Linea alba
d. Rectus sheath muscle
e. Duramater
f. 1st tracheal cartilage
g. Conjoined tendon
h. Intercostal muscles

Answer: C
In lap. Choly open technique incision is made for entering the port either in infraumblical or transumblical
region which leads to piercing of linea Alba.

817. A 48yo man has continuous ant chest pain which is worse on inspiration and has temp of 37.5C since
4wks after a MI. His ESR=45mm/h. What is the single most likely explanation for the abnormal inv?
a. Acute pericarditis
b. Cardiac tamponade
c. Atrial thrombus
d. Left ventricular aneurysm
e. Dressler syndrome

Answer: E
Dresslers syndrome was first described in 1956. It is characterised by pleuritic chest pain, low-grade fever and
pericarditis (autopsy shows localised fibrinous pericarditis), which may be accompanied by pericardial
effusion. It tends to follow a benign clinical course. It is thought to be immune-mediated (antiheart antibodies
may be present). The reported incidence has been declining in recent years.[1]
Risk factors
If a person has had a previous episode, it is more likely to recur. It seems more likely to occur after a large
infarct.
Presentation
It usually presents two to five weeks after the initial episode, with pain and fever that may suggest further
infarction.
The pain is the main symptom, often in the left shoulder, often pleuritic, and worse on lying down.
There may be malaise, fever and dyspnoea.
Rarely, it may cause cardiac tamponade or acute pneumonitis.
A pericardial friction rub may be heard. The typical sound of pericarditis is described as like the sound of boots
walking over fresh snow.
Differential diagnosis
The pain may initially suggest a further episode of angina or myocardial infarction. Pleuritic chest pain may
also suggest pneumonia or pulmonary embolism.

Investigations
FBC will show leukocytosis, sometimes with eosinophilia and an elevated ESR.
Serology may show heart autoantibodies.
ECG may show ST elevation in most leads without reciprocal ST depression, typical of pericardial effusion.
Echocardiography shows pericardial effusion.
MRI scan may show an effusion and, more recently, has been shown to reveal pericardial involvement.[8]
CXR shows pleural effusions in 83%, parenchymal opacities in 74%, and an enlarged cardiac silhouette in 49%.

Management
Aspirin may be given in large doses.
Other non-steroidal anti-inflammatory drugs (NSAIDs) or corticosteroids may be used, especially if there are
severe and recurrent symptoms.
Steroids are particularly valuable where severe symptoms have required pericardiocentesis, and when
infection has been excluded.
In resistant or recurrent cases, colchicine may be useful.
If there is significant pericardial effusion then pericardiocentesis, involving aspiration of the fluid, may be
required to relieve the constriction on the heart.

dressler occurs after > 2 weeks @ least. before that it can be pericarditis. Dressler & Aneurysm occurs after 2
weeks. wherever there is pleurisy after 2 weeks of MI, most probably Dressler. Autoimmune kind of disease.
only steroids will help.

818. An 8yo child swallowed 12 tablets of paracetamol 4h ago. Serum paracetamol levels when tested were
at critical level. What would you do next?
a. Activated charcoal
b. IV N-acetylcysteine
c. Gastric lavage
d. Observation only
Answer: B
NAC is believed to work by a number of protective mechanisms. It acts as a precursor for glutathione,
promoting normal conjugation of any remaining paracetamol, and also supplies thiols that function as
antioxidants. It is virtually 100% effective in preventing liver damage when given within eight hours of
ingestion.[3] After eight hours, efficacy decreases sharply.
The initial dose of acetylcysteine should be given as an infusion over 60 minutes. This should reduce the
number of dose-related adverse effects. The infusion should be in 5% glucose, with 0.9% sodium chloride as
an alternative. There are now NO specific contra-indications to acetylcysteine use. Even if there is a previously
reported reaction, the benefits of treatment outweigh the risks.

Specific weight-related dosing tables are available to guide the health professional.[5] Children receive the
same doses and treatment as adults, but with a reduced quantity of intravenous fluid, as fluid overload is a
potential risk.

A full treatment course comprises three consecutive doses, administered sequentially, with no break between
infusions.

819. A pt dx with DVT is taking warfarin. What is his cut off INR limit?
a. <1
b. 1-2
c. 2-3
d. 3-4

Answer: C
Target INR range and duration of treatment
In most situations the INR target is 2.5 (target range 2.0 3.0). This range is appropriate for the prophylaxis or
treatment of venous thromboembolism and reduction of the risk of systemic embolism for people with atrial
fibrillation and valvular heart disease.5 In some situations higher ranges are more appropriate. The target INR
may vary depending on individual clinical situations. The target INR for mechanical prosthetic valves is
dependent on the type of valve replacement used.6

The duration of warfarin therapy for a provoked DVT or PE is 13 weeks. For unprovoked DVT or PE the
duration again is 13 weeks, but for individual patients within their clinical context, the indefinite use of
warfarin may be appropriate.5 For atrial fibrillation, cardiomyopathy and valvular heart disease (selected
cases) an indefinite period of warfarin treatment is recommended.

820. Inserting a drain in the mid-axillary line. What is the single most appropriate anatomical structure?
a. External iliac muscle
b. Linea alba
c. Rectus sheath muscle
d. Conjoined tendon
e. Intercostal muscles

Answer: E

821. A 29yo man presents with hx of cough, weight loss and night sweats. Exam: pansystolic murmur. What
is the most appropriate dx of underlying cause?
a. Malaria
b. HSP
c. HIV
d. Dengue fever

Answer: C
Patients with HIV are likely to have heart valve disease (tricuspid mc in drug abusers) leading to pansystolic
murmur. All other options do not have direct relation to valvular disease.

822. A 15yo boy presents with recurrent breathlessness and wheeze especially after exercise. What is the
most diagnostic inv?
a. CXR
b. Lung function test
c. PEFR
d. CT scan
Answer: B

What is wheezing?
Wheezing is a whistling sound that occurs during breathing when the airways are narrowed. Commonly the
sound is more prominent when you breathe out than when you breathe in (although not always). The sound is
caused by air that is forced through airways that are narrower than normal. Narrowed airways can be due to:

Bronchospasm. This means that the muscle within the lining of the airways contracts. This has an effect of
narrowing (constricting) the airways.
Swelling of the lining of the airways.
A lot of secretions (mucus, etc) in the airways.
An inhaled object. For example, if you inhale a peanut (which is not uncommon in children).
An abnormal growth in the airway, such as a tumour (cancer).

Wheeze with breathing difficulty


Breathing difficulty can mean breathlessness, fast breathing or difficulty getting your breath. If you have any
breathing difficulty and do not already have clear instruction as to what to do then see a doctor immediately.
(Some people with asthma have pre-arranged action plans as agreed with their doctor. For example, if they
have wheeze and become breathless then they have a plan of which medication to take. However, if you do
not have such a plan and become breathless, you need to see a doctor immediately. In particular, children
with any breathing difficulty need to be seen as soon as possible by a doctor.)

What are the causes of wheezing?


There are many possible causes. Below is a brief overview of the more common and important causes.

Asthma
Asthma is a condition that affects the smaller airways (bronchioles). From time to time the airways narrow
(constrict) due to bronchospasm (described earlier). Asthma can start at any age, but it most commonly starts
in childhood. At least 1 in 10 children, and 1 in 20 adults, have asthma. The common symptoms are cough and
wheeze. You may also become breathless, and develop a feeling of chest tightness. Symptoms can range from
mild to severe between different people, and at different times in the same person. Each episode of
symptoms may last just an hour or so, or persist for days or weeks unless treated.

Chronic obstructive pulmonary disease (COPD)


COPD is a general term which includes the conditions chronic bronchitis and emphysema. Chronic means
persistent. Bronchitis is inflammation of the airways of the lungs (bronchi). Emphysema is damage to the
smaller airways and air sacs (alveoli) of the lungs. Pulmonary means 'affecting the lungs'. Chronic bronchitis
and emphysema commonly occur together. The term COPD is used to describe airflow obstruction due to
chronic bronchitis, emphysema, or both. The main cause of COPD is smoking and it mainly affects older
people.

Bronchiectasis
Bronchiectasis is an abnormal widening of one or more airways. Extra mucus is made in the abnormal airways,
which is prone to infection. The main symptom is a cough which produces a lot of sputum. You may also cough
up some blood from the abnormal inflamed airway and become wheezy. There are various underlying causes
of bronchiectasis. For example, a previous severe lung infection, some inherited conditions, and some other
conditions that can damage part of an airway.

Bronchiolitis
Bronchiolitis is an infection of the small airways of the lung (the bronchioles). It is a common condition of
babies. Most affected babies are not seriously ill, and make a full recovery. Sometimes it becomes more
serious and hospital care may be needed. It is usually caused by a virus called the respiratory syncytial virus
(RSV). Bronchiolitis in the UK usually occurs in the winter months (November to March).

Other infections
An infection anywhere in the respiratory tract may trigger wheezing. For example, acute bronchitis (infection
of the bronchi) is common in adults and may cause some wheezing in addition to other symptoms. A viral
respiratory infection is a common cause in young children.

823. A 34yo man was walking along the countryside when an insect bit him. After which he started to
complain of an annular rash spreading upwards.

a. Penicillin PO
b. Doxycycline PO
c. Flucloxacillin PO
d. Gentamicin PO
e. Ciprofloxacin PO
f. Antihistamine PO
g. Antihistamine IV
h. Corticosteroid IV
i. Corticosteroid IM
j. Adrenaline IM
k. Adrenaline IV
l. Atropine IV
m. Reassurance
Answer: B
Lyme disease can be contracted from a deer tick (Ixodes) bite containing Borrelia species of bacteria.
Erythema migrans is the pathognomonic skin finding associated with Lyme disease and is often described as a
bull's-eye or target rash. It typically is an annular, erythematous plaque with central clearing. Lyme disease can
be associated with myalgias and arthralgias, fever, anorexia and nausea, fatigue, and regional
lymphadenopathy, but the rash may be the only finding at presentation.

Lesions are usually 5 to 68 cm, although they can vary in size.1 They appear three to 30 days after the tick bite,
but most commonly within seven to 14 days. The classic lesion occurs in approximately 80% of cases.2 Lyme
disease has been reported in all 50 states but is endemic in the Northeast and in parts of Minnesota,
Wisconsin, and northern California.3 Most cases occur from May to September when the Ixodes tick is in the
nymph stage.
There are three distinct clinical stages of Lyme disease. The early localized stage (three to 30 days after tick
exposure) includes the influenza-like symptoms of fever, fatigue, arthralgias, and myalgias. The early
disseminated stage (days to weeks after tick exposure) includes multiple lesions, neurologic symptoms
(palsies, radiculopathy, or peripheral neuropathy), or cardiac symptoms (myocarditis and varying degrees of
atrioventricular block). The late stage (months to years after tick exposure) includes arthritis, primarily
affecting the knee, and possible cognitive disturbances. Treatment should be initiated promptly to avoid
progression to late stages of the disease.

Erythema multiforme is a hypersensitivity reaction to medication use or an infection, such as herpes simplex
virus infection.4,5 It typically manifests as papules or plaques with erythematous borders. The target or iris
lesions typically appear on the palms, soles, elbows, or knees.

Granuloma annulare is a benign, self-limited, annular eruption that does not require treatment. It presents as
skin-colored plaques or papules on distal portions of the extremities, specifically the hands, wrists, and feet. It
is idiopathic and occurs in adults and children, although it is most common between 40 and 50 years of age. It
is more common in women than in men.6

Nummular eczema is an idiopathic papulovesicular dermatitis commonly associated with asthma and atopic
dermatitis. It typically manifests as coin-shaped lesions and is most prominent in cold or dry months.

Rheumatic fever is an inflammatory disease that can develop after group A streptococcal infection and is
associated with erythema marginatum. This annular rash is typically slightly elevated, mildly erythematous,
and nonpruritic, and is primarily found on extensor surfaces of extremities, sparing the face.

Antibiotics for lyme disease

Lyme disease is curable with several types of antiboitics including common antibiotics like doxycycline,
amoxicillin and azitromycin, which can be taken orally. Sometimes the antibiotic is administered intravenous
(IV), particularly when the drug of choice is ceftriaxone (Rocephin), which cannot be taken orally.

Other antibiotics used include: minocycline, tetracycline, cefuroxim, claritromycin. Some doctors also
experiment with other types of antibiotics or combinations of antibiotics. Many of those treatment have not
been studied, though.

824. A 60yo man was brought in by his wife complaining of ataxia, urinary incontinence and erectile
dysfunction. He also complains of rigidity and slowing of movement with a pill rolling tremor of the hands.
What is the likely dx?
a. Parkinsons disease
b. Idiopathic parkinsons disease
c. Shy-drager syndrome
d. Huntingtons disease

KEY is C
SHY DRAGER SYNDROME:
Shy-Drager Syndrome (Multiple System Atrophy) (SDS)(MSA) is a a rare degenerative condition resulting from
degeneration of certain nerve cells in the brain and spinal cord. Body functions controlled by these areas of
the brain and spinal cord do not function normally in sufferers. These functions include the autonomic or
involuntary nervous system (which controls blood pressure, heart rate, and bladder function) and the motor
system (which controls balance and muscle movement).
Depending upon which part of the brain is affected first, MSA may appear in different ways. Sometimes it
presents with low blood pressure on standing, urinary bladder problems, or difficulties with balance and
movement that resemble Parkinson's disease. Early symptoms often include impotence and urinary
incontinence. Most patients complain of severe constipation and later develop rectal incontinence.
Orthostatic hypotension an excessive drop in blood pressure when the patient stands up causing light-
headedness or dizziness - is a universal feature of SDS. It is worse after meals and early in the day. When
orthostatic hypotension becomes severe (i.e. blood pressure of less than 70/40) blackouts may result. When
the patient falls to a prone position, the blood pressure returns toward normal and the patient will regain
consciousness.
Many patients also complain of dry mouth and dry skin, and because of abnormal sweating have problems
with relation of body temperature. The hands and feet will be cold.

Major features supporting diagnosis of probable MSA

System Feature Notes

Autonomic Severe (symptomatic or otherwise) Blood pressure fall by 30 mm Hg systolic


orthostatic hypotension. Commonly and 15 mm Hg diastolic within three
associated symptoms include light- minutes of standing from a previous
headedness, dizziness, weakness of legs, three-minute supine position. Associated
fatigue and syncope. Postprandial supine hypertension is common, and is
hypotension may be a major feature. aggravated by medication used to reduce
orthostatic hypotension.

Urinary incontinence or Urinary dysfunction is the


Urogenital incomplete emptying. most frequent initial
Erectile dysfunction. complaint in women.
Erectile dysfunction is the
most frequent initial
complaint in men.

Bradykinesia.
Extrapyramidal Rigidity. Check that postural instability is not
tract Postural instability. caused by primary visual, vestibular,
Tremor (but not classic pill cerebellar, or proprioceptive dysfunction.
rolling).

Gait/limb ataxia.
Cerebellar Ataxic dysarthria.
function Oculomotor dysfunction
(sustained gaze-evoked
nystagmus).

825. A 67yo man being managed for a malignancy develops neutropenic fever. He has been commenced on
Ticacillin, Tazobactam and Gentamicin. He has also recently commenced on Meropenem but on the 3 rd day his
temp still remains >39C. 2 blood tests and urine cultures show no organism. Inv: Hgb=104g/dl, WBC=<0.5,
Plt=15. What will you do next?
a. Continue IV antibiotics and add oral antifungals
b. Continue antibiotics and add IV antifungals
c. Stop antibiotics
d. Continue only present antibiotics

826. A young girl who is known to have T1DM presented with drowsiness and deep breathing. Her sugar
level=20. Her BP=120/80 mmHg and her mucous membranes are dry. What would be the next appropriate
step?
a. Serum urea
b. Blood culture
c. CT
d. HbA1c
e. ABG
827. Removal of a glioma, which single most appropriate anatomical structure will be pierced?
a. Cricoid cartilage
b. Rectus sheath muscle
c. Dura Mater
d. Conjoined tendon
e. Intercostal muscles
828. A child is not breathing and intubation failed. At what anatomical site should the incision be made?

a. External iliac muscle


b. Cricoid cartilage
c. Linea alba
d. Rectus sheath muscle
e. Duramater
f. 1st tracheal cartilage
g. Conjoined tendon
h. Intercostal muscles
829. A 10yo child who presented with fx of the radius which was treated with a plaster cast, complains of
pain. Exam: limb is swollen but warm and well perfused, pulses are present. What should you do next?
a. Send for repeat XR
b. Remove cast
c. Replace cast with more padding
d. Give analgesic

830. A 32yo man has been to Thailand and returned with cervical lymphadenopathy and fever. What
is he most likely suffering from?
a. HIV
b. EBV
c. Typhoid
d. Measles
key : b
reason : clinical pic fits EBV ( fever + lymphadenopathy )
synonms : glandular fever / infectious mononucleosis
Presentation :
Low-grade fever, fatigue and prolonged malaise.
Lymphadenopathy, especially neck glands
Sore throat; tonsillar enlargement is common, classically exudative and may be massive; palatal
petechiae and uvular oedema.
Fine macular non-pruritic rash
Later signs include:
Mild hepatomegaly and splenomegaly (splenic enlargement returns to normal or near normal
usually within three weeks after the clinical presentation) with tenderness over the spleen.
Jaundice occurs in fewer than 10% of young adults, but in as many as 30% of infected elderly
patients.
Investigations :
Heterophile antibodies >>> Following IM caused by EBV, 70-90% of patients produce IM heterophile
antibodies (antibodies against an antigen produced in one species that react against antigens from other
species). The heterophile antibodies are not specific for the virus. These antibodies can be detected by :
Paul-Bunnell test
Monospot test
EBV-specific antibodies
Management :
Patients are traditionally advised to avoid contact sports for three weeks because of the risk of splenic rupture
Avoid alcohol for the duration of the illnes
Advise paracetamol for analgesia and control of fever.
No specific antiviral therapy is available.
Short courses of corticosteroids are beneficial for haemolytic anaemia, central nervous system involvement or
extreme tonsillar enlargement.
831. A 6yo child presents with edema and mild proteinuria. No hematuria. What is the most likely dx?
a. PSGN
b. Membranous GN
c. Minimal change GN
d. RPGN
key : c
reason : age 6 ys + mild proteinuria
Minimal change disease
Light microscopy is virtually normal but electron microscopy shows widespread fusion of the epithelial
cell foot processes on the outside of the glomerular basement membrane. Immunofluorescence is usually
negative.
Most often presents in children aged between 2 and 4 years. Accounts for 90% of cases of nephrotic
syndrome in children and about 20% of cases in adults.
Clinical features: nephrotic syndrome with selective proteinuria; normal renal function, normal blood
pressure, normal complement levels; increased risk of infections, especially urinary tract infections and
pneumococcal peritonitis (therefore give prophylactic penicillin if oedematous).
Associated with atopy in children, especially those who are HLA-DR7-positive.
May also be related to underlying Hodgkin's disease in adults.
Usually responds to a course of high-dose prednisolone but relapse is frequent.
Relapsing disease may go into remission following treatment with prednisolone and
cyclophosphamide or ciclosporin.
One third of patients have one episode, one third develop occasional relapses and one third have
frequent relapses which stop before adulthood.
Minimal change disease does not progress to end-stage chronic kidney disease.
Membranous nephropathy
Widespread thickening of the glomerular basement membrane occurs.
Immunofluorescence reveals granular deposits of immunoglobulin and complement.
Although most cases are idiopathic, it may also be secondary to SLE, hepatitis B, malignancy or the use
of gold or penicillamine.
It is more common in men.
It is the most common cause of nephrotic syndrome in adults. It may present with proteinuria or
nephritic syndrome, hypertension. Haematuria is rare.
The idiopathic form may respond to a treatment regimen involving alternate months of corticosteroids
with chlorambucil or cyclophosphamide, or to ciclosporin.
It progresses to end-stage kidney disease in 30-50% of patients. The remainder with idiopathic
membranous nephropathy has a complete or partial spontaneous remission of nephrotic syndrome with
stable renal function.
832. An 80yo woman suffering from RA presents with severe epigastric pain and vomiting. She also
complains of shoulder tip pain. What is the single most discriminatory inv?
a. US Abdomen
b. Sigmoidoscopy
c. Colonscopy
d. Barium meal
e. Upper GI endoscopy
f. Erect CXR
key : f
reason : acute abdomen + diaphragmatic irritation >>>> gut perforation mostly peptic ulcer because of RA
drugs
Indications of plain abdominal x-ray
Renal colic:
A 'KUB' picture is requested. This is a large film that is designed to take in the kidneys, ureters and
bladder.
About 90% of renal stones are radio-opaque. Uric acid stones may be missed.
False positives may occur from phleboliths that are most common in the pelvic veins. False
negatives may arise, especially if stones are small.
Calcification may represent gallstones but only a minority of gallstones are radio-opaque. Gallstones
become more frequent with age and are often asymptomatic.[4]
Doctors in A&E tend to be poor at identifying stones on plain films but, if urinalysis is negative, the
diagnosis is unlikely to be renal colic.
Intestinal obstruction:
Erect and supine films are used to confirm the diagnosis.
Obstruction of the small bowel shows a ladder-like series of small bowel loops but this also occurs
with an obstruction of the proximal colon. Fluid levels in the bowel can be seen in upright views.
Distended loops may be absent if obstruction is at the upper jejunum.
Obstruction of the large bowel is more gradual in onset than small bowel obstruction. The colon is in
the more peripheral part of the film and distension may be very marked.
Fluid levels will also be seen in paralytic ileus when bowel sounds will be reduced or absent rather
than loud and tinkling as in obstruction.
In an erect film, a fluid level in the stomach is normal as may be a level in the caecum. Multiple fluid
levels and distension of the bowel are abnormal.
Perforation of the intestine:
If the bowel has been perforated and a significant amount of gas has been released it will show as a
translucency under the diaphragm on an erect film.
Gas will also be found under the diaphragm for some time after laparotomy or laparoscopy.
Appendicitis
An appendicolith may be apparent in an inflamed appendix in 15% of cases but as a diagnostic point
in the management of appendicitis, the plain X-ray is of very limited value.[6]
It may be of value in infants.
Intussusception:
Intussusception occurs in adults and children.
A plain abdominal X-ray may show some characteristic gas patterns.[8]
A sensitivity and specificity of 90% adds to this rather difficult diagnosis but ultrasound is vastly
superior.[9]
Detection of swallowed foreign bodies:
Plain X-ray will detect the presence of radiopaque foreign bodies.
A plain abdominal X-ray will show 90% of cases of 'body packing' (internal concealment of drugs to
avoid detection) but there will be false positives in 3%.
833. A 44yo man went on holiday to Sudan 5wks ago. He now presents with red urine and fever.
Exam: hepatomegaly. What is the most likely dx?
a. Malaria
b. Brucellosis
c. Leptospirosis
d. Schistosomiasis
key : d
Schistosomiasis
Presentation
Infection can be acute or chronic.
Physical findings vary with the stage of illness, worm burden, worm location, and organs involved.
Schistosomiasis is associated with anaemia, chronic pain, diarrhoea, exercise intolerance, and
malnutrition.[2]

Acute syndrome (Katayama syndrome)


The acute reaction is due to the sudden release of highly antigenic eggs.
The most common acute syndrome is Katayama fever. It usually occurs in children or young adults
with no past exposure to the disease and is most likely with S. japonicum.
As travellers present several weeks after contact with infested water, it is necessary to obtain a careful
travel history, including drinking water sources and activities such as swimming.
Symptoms:
Most acute infections are asymptomatic.
The first sign may be swimmer's itch in which there is an urticarial response for a few days after the
parasite has penetrated the skin.
Malaise.
Arthralgia or myalgia.
Cough.
Diarrhoea.
Right upper quadrant pain.
Signs:
Fever.
Hepatosplenomegaly.
Right upper quadrant pain or tenderness.
Urticaria may be seen occasionally.
Lymphadenopathy.
Initial invasion of skin and infection with non-human species may cause itching and rash.
Chronic disease
Chronic schistosomiasis can present months to years after exposure, making diagnosis difficult.
It is endemic in poor, rural areas.
Many patients have not had an acute syndrome.
Symptoms may be few or mild. They may be nonspecific or reflect the site of egg production in the
mesentery or bladder wall, the extent of damage to liver or spleen, the degree of lung involvement, and
possibly other sites including the central nervous system (CNS).
Symptoms:
Bloody diarrhoea.
Abdominal pain, right upper quadrant pain, cramps.
Haematemesis, which can occur from oesophageal varices with portal hypertension.
Haematuria, dysuria:
The first feature may be frequency of micturition.
Initially, haematuria is only terminal but, as it becomes more severe, the blood produces red urine
throughout the stream.
There is proteinuria.
Pulmonary hypertension may produce:
Fatigue.
Dyspnoea on exertion.
Cough.
Atypical chest pain.
Hepatosplenomegaly.
Signs:
Abdominal tenderness.
Ascites with portal hypertension.
Seizures and/or altered mental state (with cerebral infection).
Investigations
Microscopic examination of stool or urine is the gold standard for diagnosis but requires the adult
worms to be producing eggs.
Serology can diagnose less advanced infections
FBC shows eosinophilia and anaemia.
Renal function may be impaired if the urinary tract is obstructed.
Management
Praziquantel is the drug of choice in most cases.
The World Health Organization (WHO) believes that praziquantel is safe in pregnancy, lactation and in
children under the age of 24 months.
Oxamniquine is the only alternative
In acute Katayama fever, corticosteroids are very important to subdue the hypersensitivity reaction.
Corticosteroids and anticonvulsants may be needed as adjuvants to praziquantel in
neuroschistosomiasis.
Surgical
Endoscopy and sclerotherapy can treat oesophageal varices.
A ventriculoperitoneal shunt and corticosteroids are required to treat hydrocephalus and raised
intracranial pressure in cerebral schistosomiasis.
Complications
Urinary tract
Secondary bacterial infection and renal stones may occur.
There is an increased risk of squamous cell carcinoma of bladder that has been noticed especially in
Egypt. It is possible that the infestation and the carcinogens in tobacco smoke have a synergistic effect.
Hydronephrosis may occur but will reverse if the disease is treated, suggesting that the renal
parenchyma is compressed but not destroyed and renal function is not markedly impaired.
Schistosomal nephropathy leading to renal failure may occur.
Female urogenital schistosomiasis may be a risk factor for HIV infection.
Alimentary canal
Gastrointestinal complications include gastrointestinal bleeding, gastrointestinal obstruction,
malabsorption and malnutrition.
Lesions tend to bleed and there is loss of blood and protein, causing iron-deficiency anaemia and
hypoproteinaemia. These lesions are mostly in the colon and rectum.
Fibrosis of the liver occurs, producing portal hypertension. S. mansoni infection invariably results in
liver fibrosis.
Portal hypertension can produce oesophageal varices that may bleed, and ascites.
Portocaval shunting predisposes to pulmonary infestation and problems of pulmonary hypertension.
Co-infection with hepatitis, HIV, and malaria can increase the risk of hepatocellular carcinoma and
increase the risk of mortality.
Other complications
Chronic septicaemic salmonellosis (prolonged fever with enlargement of the liver and spleen) may
occur in schistosoma-infected individuals who are co-infected with salmonella.
Pulmonary hypertension.
Cor pulmonale.
Neuroschistosomiasis (includes increased intracranial pressure, myelopathy and radiculopathy).
834. A 32yo homosexual comes with hx of weight loss. Fundoscopy reveals retinal hemorrhages.
What is the single most appropriate option?
a. Mycobacterium avium
b. CMV
c. Hemophilus influenze
d. NHL
e. Pneumocystic jerovici
key : B
reason : homosexual +wt loss >>> HIV >>> CMV
Cytomegalovirus and HIV infection
CMV can cause very serious infection in HIV infection.
Retinitis:
Retinitis is the most common manifestation of CMV disease in patients who are HIV positive.
It presents with decreased visual acuity, floaters, and loss of visual fields on one side.
Ophthalmological examination shows yellow-white areas with perivascular exudates. Haemorrhage
is present. Lesions may appear at the periphery of the fundus, but they progress centrally.
It begins as a unilateral disease, but in many cases it progresses to bilateral involvement. It may be
accompanied by systemic CMV disease.
Ganciclovir has been used to treat retinitis, but it only slows the progression of the disease. The
optimal treatment is using ganciclovir implants in the vitreous, accompanied by intravenous ganciclovir
therapy.
Oral ganciclovir may be used for prophylaxis of CMV retinitis. It should not be used for treatment.
CMV pneumonia in patients who are HIV positive is uncommon. The reason for this is unknown.
Gastrointestinal tract:
In the upper gastrointestinal tract, CMV has been isolated from oesophageal, gastric and duodenal
ulcers. Patients with oesophageal disease may present with painful dysphagia.
In the lower gastrointestinal tract, patients with CMV may present with diarrhoea due to colitis.
CMV may cause disease in the peripheral and central nervous system.
835. A 30yo man comes with hx of early morning back pain and stiffness. Exam: red eyes. What is the
single most appropriate option?
a. Iris
b. Ciliary body
c. Cornea
d. Conjunctivitis
e. Sclera
key : A
reason : male pt with early morning back pain and stiffness >> AS >>>uvitits
Presentation
AS usually presents before the age of 30 years.
Most patients have mild chronic disease or intermittent flares with periods of remission.
Systemic features are common. Fever and weight loss may occur during periods of active disease.
Fatigue is also prominent.
Morning stiffness is characteristic.
Inflammatory back pain which Often improves with moderate physical activity.
The spinal disease starts in the sacroiliac joints (bilateral lumbosacral region) and may be felt as
diffuse nonspecific buttock pain.
Peripheral enthesitis:
Occurs in approximately a third of patients.
Common sites - behind the heel (Achilles tendonitis), the heel pad (plantar fasciitis) and the tibial
tuberosity.
Lesions tend to be painful, especially in the morning. There may be associated swelling of the tendon
or ligament insertion.
Peripheral arthritis:
Also occurs in about a third of patients.
Joint involvement is usually asymmetric, involving the hips, shoulder girdle (glenohumeral,
acromioclavicular, and sternoclavicular joints), joints of the chest wall (costovertebral joints, costosternal
junctions) and symphysis pubis.
Other peripheral joints are less often and less severely affected, usually as asymmetrical oligoarthritis.
In children, AS tends to commence with arthritis prior to spinal disease developing.
Temporomandibular joints are occasionally involved.
Extra-articular manifestations of AS
Eye involvement
Acute anterior uveitis occurs in 20-30% of patients. Of all patients presenting with acute anterior
uveitis, a third to a half have or will go on to develop AS.
Acute anterior uveitis presents with an acutely painful red eye and severe photophobia and requires
emergency treatment to prevent visual loss.
Cardiovascular involvement
This occurs in <10% of patients, usually in those with severe long-standing disease.
Aortitis of the ascending aorta may lead to distortion of the aortic ring, causing aortic regurgitation.
Fibrosis of the conduction system may result in various degrees of atrioventricular block, including
complete heart block.
Pulmonary involvement
Restrictive lung disease may occur in patients in later stages, with costovertebral and costosternal
involvement limiting chest expansion.
Pulmonary fibrosis of the upper lobes.
Renal involvement
Amyloidosis is a very rare complication in patients with severe, active and long-standing disease and
may cause renal dysfunction with proteinuria and renal insufficiency or chronic kidney disease.
Immunoglobulin A (IgA) nephropathy is another association.
Neurological involvement
This usually occurs secondary to fractures of a fused spine.
Also, patients with AS are prone to atlanto-axial subluxation, which may lead to cervical myelopathy.
Cauda equina syndrome may occur in patients with severe long-standing disease.
Metabolic bone disease
Osteopenia and osteoporosis may occur in patients with long-standing spondylitis, further increasing
risk of fracture.
Diagnosis
The British Society for Rheumatology recommends that the modified New York criteria be used to diagnose
AS.
Clinical criteria:
Low back pain, for more than three months; improved by exercise, not relieved by rest.
Limitation of lumbar spine motion in both the sagittal and the frontal planes.
Limitation of chest expansion relative to normal values for age and sex.
Radiological criterion: sacroiliitis on X-ray.
Definite AS is diagnosed if the radiological criterion is present plus at least one clinical criterion and
probable AS if three clinical criteria are present, or if the radiological criterion is present but no clinical criteria
are present.
investigations :
X-rays are the most helpful imaging modality in established disease, although they may be normal in early
disease.
MRI scanning may be useful in identifying early sacroiliitis. MRI of the sacroiliac joints is more sensitive than
either plain X-ray or CT scan in demonstrating sacroiliitis
Musculoskeletal ultrasound scanning can help in diagnosing enthesitis.
836. A 70yo woman with longstanding anxiety is seen in the OPD. She complains of her heart skipping
a beat quite often. This particularly occurs when she is trying to get to sleep. The palpitations are never
sustained. What is the most likely rhythm disturbance?
a. SVT
b. VF
c. VT
d. V-ectopics
e. A-fib
837. A 17yo has acute pain around his right eye, pain on one side of his face and ear ache too. What is
the single most dx?
a. Ear wax
b. Ear foreign body
c. Dental abscess
d. Cellulitis
e. Herpes zoster
key : e
reason : acute onset + unilatral with sensory nerve distribution
THIS PT IS IN the pre eruptive phase >>> no blisters yet
838. A 12yo boy presented with itching in his hands. Exam: skin is dry and red. His mother is asthmatic
and older brother has hay fever. What is the single most likely causative factor?
a. Dermatitis herpitiformis
b. Scabies
c. Eczema
d. Uremia
e. Drug induced
key : c
Eczema
Diagnostic criteria
Must have an itchy skin condition (or report of scratching or rubbing in a child) plus three or more of
the following:
History of itchiness in skin creases such as folds of the elbows, behind the knees, fronts of ankles, or
around the neck (or the cheeks in children aged 18 months or under).
History of asthma or hay fever (or history of atopic disease in a first-degree relative in children aged
under 4 years).
General dry skin in the preceding year.
Visible flexural eczema (or eczema affecting the cheeks or forehead and outer limbs in children aged
under 4 years).
Onset in the first two years of life (not always diagnostic in children aged under 4 years).
If it does not itch it is very unlikely to be eczema.
management :
emolient + topical steroids
839. A 45yo man presented with pruritic purple papules on the flexor surface of his wrist and some
white lacy markings on his buccal mucosa. What is the single most likely causative factor?
a. ALL
b. Lymphoma
c. Polycythemia
d. IDA
e. Lichen planus
key : e
Lichen planus
Presentation
Onset is usually acute, affecting the flexor surfaces of the wrists, forearms and legs.
The typical lesion is an intensely itchy 2-5 mm red or violet shiny flat-topped papule with white streaks
('Wickham's striae').
Mucous membranes are commonly affected >>> Classically, white slightly raised lesions with a trabecular, lacy
appearance on the inside of the cheeks
840. A known DM was admitted with sudden LOC. What is the initial inv?
a. CT scan
b. RBS
c. MRI
d. ECG
e. ABG
key : B
reason : diabetic pt >>> hypogycemia is common >>> RBS is the initial inv
841. A 36yo lady comes with hx of early morning stiffness of her small joints and with red and painful
eye. What is the single most appropriate option?
a. Iris
b. Ciliary body
c. Cornea
d. Conjunctivitis
e. Sclera
f. Lichen planus
key : E
reason :female pt with early morning stiffness of her small joints >>> RA >>> scleritis
Scleritis
Scleritis often appears in association with other inflammatory diseases such as rheumatoid arthritis and
granulomatosis - the histopathological changes are characteristic of a chronic granulomatous disease.
842. A 23yo man comes with 2d hx of sticky greenish discharge from the eyes with redness. What is
the single most appropriate option?
a. Iris
b. Ciliary body
c. Cornea
d. Conjunctivitis
e. Sclera
key : D
Bacterial conjunctivitis
Presentation
Symptoms
Discomfort - burning or gritty but not sharp.
Pain is minimal; significant pain suggests a more serious diagnosis.
Vision is usually normal, although 'smearing', particularly on waking, is common.
Discharge tends to be thick rather than watery.
There may be mild photophobia. Significant photophobia suggests severe adenoviral conjunctivitis or
corneal involvement.
History
Ask about contact lens wear: establish whether this could be (or lead to) a problem of the (vulnerable)
cornea.
Time course: onset, duration - in chronic cases consider venereal disease in people at a sexually active
age.
Use of over-the-counter medication: consider whether this could be a reaction to previously
administered drops or ointment.
Social aspect: establish whether anybody else has had it (family, school, work). Determine whether
there are concerns about working during the course of the illness.
Findings
'Red eye' with uniform engorgement of all the conjunctival blood vessels.
Bacterial conjunctivitis may often be distinguished from other types of conjunctivitis by the presence of
a yellow-white mucopurulent discharge.
Eyes may be difficult to open in the morning, glued together by discharge.
There is also usually a papillary reaction (small bumps on the palpebral conjunctiva, appearing like a
fine velvety surface). The presence of follicles is more likely to indicate viral conjunctivitis.
Bacterial conjunctivitis is usually bilateral (but often sequential).
Check visual acuity - this should be normal, other than the mild and temporary blur secondary to the
discharge which can be blinked or wiped away.
843. A pt was admitted with erectile dysfunction, reduced facial hair and galactorrhea. What is the
most probable dx?
a. Hyperprolactinemia
b. Cushings syndrome
c. Pheochromocytoma
d. Hyperthyroidism
e. Hypoparathyroidism
key : A
reason : galactorrhea + with erectile dysfunction + reduced facial hair
hyperprolactinaemia
Presentation
Women:
Common symptoms of are amenorrhoea, oligomenorrhoea and galactorrhoea.
They may also have infertility, hirsutism and reduced libido.
Men:
The hormonal effects of raised prolactin levels are subtle and develop slowly.
Endocrine symptoms are reduced libido, reduced beard growth and erectile dysfunction.
Children:
Growth failure and delayed puberty are possible presentations in children.
Symptoms due to tumour size (usually macroprolactinomas):
Headache.
Visual disturbances (classically, a bitemporal hemianopia (lateral visual fields) or upper temporal
quadrantanopia).
Cranial nerve palsies.
Symptoms and signs of hypopituitarism.
Rarely, cerebrospinal fluid (CSF) leak or secondary meningitis.
Investigations
Initial investigations
TFTs.
Exclude pregnancy.
Basal serum prolactin:
If prolactin is mildly elevated (eg 400-1000 mU/L, normal range <400 mU/L), it should be repeated
before referral.
Dynamic prolactin stimulation tests, such as the TRH test, are not required. Measurement of serum
prolactin on three separate occasions (at least two hours after rising and when the patient is rested) is
sufficient.
A prolactin level >5000 mU/L usually indicates a true prolactinoma.
Further investigations
Visual field testing.
Pituitary imaging (preferably MRI).
Assessment of pituitary function
844. A 32yo man has been repeatedly admitted to hospital for what was described as anxiety or panic
attacks and palpitations. On occasions he is found to be tremulous and hypertensive. A persistent weight loss
is noted. What is the most probable dx?
a. Hyperthyroidism
b. Panic attacks
c. Pheochromocytoma
d. Cushings disease
e. GAD
key : c
reason :male young age + anxiety + palpitation +wt loss +HTN
Pheochromocytoma
signs and Symptoms
Headache
Profuse sweating
Palpitations
Tremor
Nausea
Weakness
Anxiety
Sense of doom
Epigastric pain
Flank pain
Constipation
Weight loss
Hypertension but it may be paroxysmal in 50%.
Postural hypotension.
symptoms are intermittent and may vary from once a month to several times a day with duration from
seconds to hours.
Investigations
Blood tests
Blood glucose is often raised.
Calcium may be elevated.
Haemoglobin is elevated
Plasma catecholamines and plasma metanephrines (the o-methylated metabolites of catecholamines)
have both been used in diagnosis
Urine
24-hour urine collection is required for creatinine , total catecholamines, vanillylmandelic acid (VMA)
and metanephrines.
Imaging
After biochemical confirmation of a tumour, imaging is necessary to locate it.[9]
90% of phaeochromocytomas are in the adrenal glands and 98% within the abdomen. Common
locations for extra-adrenal phaeochromocytomas include close to the origin of the inferior mesenteric artery,
bladder wall, heart, mediastinum and carotid and glomus jugulare tumours.
MRI can locate all tumours within the adrenals.
CT is less sensitive and detects around 85-95% of tumours in excess of 1 cm in diameter.
management :
Surgical resection of the tumour is the treatment of choice and usually results in cure of the hypertension. Pre-
operative treatment with alpha-blockers and beta-blockers is required to control blood pressure and prevent
intraoperative hypertensive crises.
845. A 35yo man with T1DM is dehydrated with BP of 90/50mmHg. What is the single most
appropriate initial inv?
a. ABG
b. CBC
c. HbA1c
d. LFT
e. Serum Urea
key: a
reason : T1DM suspected to have DKA>>> ABG will show acidosis
DKA
DKA is characterised by hyperglycaemia, acidosis, and ketonaemia
DKA is normally seen in type 1 diabetics and may be a presenting feature of undiagnosed type 1 diabetes,
particularly in children. However DKA may rarely occur in type 2 diabetics
Check capillary blood glucose and blood gases promptly. If these suggest diabetic ketoacidosis (DKA) then
immediately begin resuscitation and management.
management:
Immediate resuscitation as required
Correct dehydration >>> 0.9% sodium chloride solution is the recommended fluid of choice
Insulin therapy >>> A fixed-rate IV insulin infusion calculated on 0.1 units/ per kilogram infusion is
recommended.
Metabolic treatment
Treat any precipitating illness

846. In OGTT what is the glucose venous plasma level 2h after glucose intake which indicates impaired
glucose tolerance?
a. >11.1mmol/l
b. Between 7.8-11.0mmol/l
c. Between 8.0-10.9mmol/l
d. Between 10.0-11.0mmol/l
e. Between 7.1-11.0mmol/l

ANSWER is B.
847. A young man who has no PMH presented with jaundice, low Hgb, retics 8% and other indices WNL but
occasional spherocytes were seen on blood film. What is the single most appropriate inv?
a. G6PD enzyme assay
b. Direct coombs test
c. Repeat blood film
d. Indirect coombs test
e. BMA

Normal retic count is 1%. Increased reticulocyte count always indicates hemolysis. Since there are spherocytes
on the blood film it means this patient either has autoimmune hemolytic anemia or hereditary spherocytosis.
Direct coombs test will be positive in autoimmune hemolytic anemia. For HS Flow cytometric analysis of eosin-
5-maleimide (EMA) binding to red cells, and cryohaemolysis test have replaced osmotic fragility tests.

848. A 22yo man came to the hosp after an injury in his hand while playing basketball. Exam: avulsion of
extensor tendon from the distal phalanx. What is the single most probable deformity?
a. Dinner fork deformity
b. Game keeper thumb
c. Mallet finger
d. Gun-stock deformity
e. Garden spade deformity

A Colles' fracture (dinner fork deformity) is a fracture of the distal radius in the forearm with dorsal and
radial displacement of the wrist and hand.

Game keepers thumb: This is injury to the ulnar collateral ligament (UCL) of the MCP joint (on the medial side
of the thumb) due to forced abduction of the MCP.

Mallet Finger: There is avulsion of the extensor tendon causing the finger to be stuck in moderate flexion.

Gunstock deformity: Also known as cubitus varus. Cubitus varus (varus means a deformity of a limb in which
part of it is deviated towards the midline of the body) is a common deformity in which the extended forearm
is deviated towards midline of the body .
Garden spade deformity: This is the Smiths fracture. Reverse of colles. The definition is a fracture of the distal
radius, with or without ulnar involvement, that has volar (anterior) displacement of the distal fragments

So the ANSWER here is C.

849. A 28yo man is inv for recurrent lower back pain. A dx of Ankylosing Spondylitis is suspected. Which of
the following inv is most useful?
a. ESR
b. XR sacro-iliac joints
c. HLA B27
d. XR thoracic spine
e. CT lumbar spine
Tests: Diagnosis is clinical, supported by imaging (MRI is most sensitive and better at detecting early disease).
Sacroiliitis is the earliest X-ray feature, but may appear late.
In later stages, calcification of ligaments with ankylosis lead to a bamboo spine appearance.
Also: FBC (normocytic anaemia), ESR, CRP, HLA B27+ve (not diagnostic)
Treatment: Exercise, NSAIDs, TNF blockers etanercept, adalimumab and golimumab are indicated in severe
active AS if NSAIDS fail

850. A 4yo girl is taken by her mother to the ED and complains of feeling unwell, urinary urgency and
temp=39C. What is the single next best inv?
a. Catheter catch of urine
b. Clean catch of urine
c. US
d. IVU
e. Suprapubic catch of urine
ANSWER B.
Recommended way of getting a urine sample: Dipstick all ward urines. If nitrites or WCC +ve, get a clean
catch (or a suprapubic aspirate or catheter sample; bag urines have many false positives from vulvitis or
balanitis). Wash the genitals gently with water, and tap repeatedly in cycles of 1min with 2 fingers just above
the pubis, 1h after a feed, and wait for a clean voided urine (CVU) sample, avoiding the streams 1st part

851. A 2yo girl presents with a 4d hx of fever that started with a cough. Her RR=45bpm, sat=94%,
temp=38.9C, capillary refill time=1s. There are crepitations at the left base on auscultation. Urine shows
negative dipstick. What is the single inv most likely to lead to dx?
a. Blood for C&S
b. ESR
c. CXR
d. Urine for C&S
e. CSF analysis

ANSWER C.
The main presenting symptoms here are Cough, fever and tachypnea. Which means some respiratory problem
is present. A cause of fever has been ruled out by giving the negative urine test which rules out UTI. Plus there
are crepitations at the base of lung. So we will first do a chest X-ray to look for the cause possibly pnuemonia.
Blood C & S takes times. ESR is raised in sooo many diseases and will not point towards any specific diagnosis.
Urine dipstick is negative so no need to culture that. No signs of neurological involvement so no need to do
CSF analysis.

852. A 3yo girl presents with fever for 2d. She is drowsy and had a seizure causing twitching of the right side
of the body for 4mins. Her RR=30 bpm, sat=90%, temp=38.9C, capillary refill time=2s. Urine negative on
dipstick. What is the single inv most likely to lead to dx?
a. Blood for C&S
b. ESR
c. CXR
d. Urine for C&S
e. CSF analysis

This question is similar to the one given above. But here the patient is drowsy and had a seizure which shows
CNS involvement so we would go for CSF analysis to rule out meningitis most probably.
853. A 6m boy is admitted with persistent irritability. He is lethargic and is not feeding as well as usual. His
RR=30bpm, sat=97%, temp=38.0C, capillary refill time=2s. Urine reveals leucocytes on dipstick. What is the
single inv most likely to lead to dx?
a. Blood for C&S
b. ESR
c. CXR
d. Urine for C&S
e. CSF analysis

Again a similiar sort of a scenario. Only abnormal sign is the temperature with symptoms of irritability,
lethargy and not taking feed. Here the urine analysis reveals leucocytosis pointing towards a possible diagnosis
of UTI so we go for urine culture and sensitivity to know about the particular organism and the specific
antibiotic for it. All other inv here will be useless until UTI has been ruled out.
854. A 3yo boy presents with a 1d hx of being unwell. He appears shocked and has 3h old rash made up of
urticarial and purpural spots. His RR=30bpm, sat=94%, temp=39C, capillary refill time=1s. Urine is clean on
dipstick. What is the single inv most likely to lead to dx?
a. Blood for C&S
b. ESR
c. CXR
d. Urine for C&S
e. CSF analysis

Patient with fever and rash could lead to meningitis. In a previous question when meningitis was suspected we
went for CSF analysis but here since there is a rash LP is contraindicated so we go for Blood C & S.
Contraindications of LP: DIC; purpura or brain herniation is near (odd posture or breathing; glascow coma
scale <13; dilated pupils, dolls eye reflexes, BP, pulse, papilloedema.

855. A child is dx with VUR. What would you tell his parents?
a. Requires antibiotic prophylaxis
b. Most will require surgery
c. Most will have kidney scarring by 5yo
d. Nothing can be done
e. Reassure

Well according to the current guidelines there is little benefit from surgery in patients with VUR. OHCS 9th ed.
page 175 states Surgical correction of moderate reflux is unlikely to be beneficial, and in minor reflux is
likely to be harmful
And for prophylaxis it is recommended that no prophylaxis shall be given after the first episode of UTI but
antibiotic prophylaxis is recommended in patients with recurrent UTIs. So the correct answer here should be
A.

856. A 2yo child presents with severe vomiting. Exam: mass felt in abdomen. What inv is most appropriate?
a. US
b. XR
c. CT
d. CBC

Pyloric stenosis dd intussusseption


Presentation
Typical presentation is onset of vomiting at 2-8 weeks of age (late presentation up to 6 months can
occur but is very rare)
o Vomiting: non-bilious, often but not always projectile and usually 30-60 minutes after a
feed, with the baby remaining hungry.
o Vomiting increases in frequency over several days.
o Vomiting also increases in intensity until it becomes projectile.
o Slight haematemesis may occur.
Persistent hunger, weight loss, dehydration, lethargy, and infrequent or absent bowel movements
may be seen.
Stomach wall peristalsis may be visible.
An enlarged pylorus, classically described as an 'olive', may be palpated in the right upper
quadrant or epigastrium of the abdomen:
o The 'olive' is best palpated at the start of a feed but is often missed.
o With the infant supine and the examiner on the child's left side, gently palpate the liver
edge near the xiphoid process.
o Then displace the liver superiorly; downward palpation should reveal the pyloric olive
just on, or to the right of, the midline.
o It should be possible to roll the pylorus beneath the examining finger.

Investigations
Serum electrolytes (for correction of imbalances before surgical repair); there is often metabolic
alkalosis with severe potassium depletion. However biochemical disturbances are now much less
common with earlier diagnosis.[8]
Ultrasound is reliable and easily performed and has replaced barium studies as the main
investigation.[9] There is a normal variation of pylorus muscle measurements with age and
gestation but ultrasound has a very high sensitivity and specificity.[10]

857. A 13yo girl complains of a 2d hx of hoarseness of voice a/w dry cough. She feels feverish. On direct
laryngoscopy, her vocal cords are grossly edematous. What is the single most appropriate inv?
a. None req
b. Sputum for AFB
c. Laryngoscopy
d. Bronchoscopy
e. XR cervical spine
acute laryngitis probably due to viralinfection no need of any further investigation

Acute laryngitis
Investigations are rarely helpful in primary care. A swab for microbiological analysis may be
contributory if excessive exudate is present.
Clinicians with the skill to perform indirect laryngoscopy will typically find redness and small
dilated vasculature on the inflamed vocal folds.
858. A 7yo girl is brought by her mother with bright red staining of her underpants. She also gives a hx that
her daughter recently started taking horse riding lessons. What is the single most appropriate next action?
a. Local exam
b. Exam under GA
c. Continue regular child care
d. Inform child protection services
e. Coag profile

it will be extremely difficult to examine a child unless under GA. We can't assume that the problem is
superficial so we might need to have a look deeper to exclude any serious problem caused the mother to bring
her child to hospital.

859. A 7d baby whose birth weight was 3.5kg and now is 3kg. What is the most appropriate next step?
a. Check child protection register
b. Nutritional assessment
c. Skeletal survey
d. Continue regular child care
e. Inform police
it's normal to lose weight .5 kg during initial period of life and baby will regain weight double the birth weight
by 2-3 month
860. A 6yo child fell on his nose 2d ago. His parents have now brought him with difficulty in breathing.
Exam: fever, nasal bones are straight. What is the single most likely dx?
a. Nasal polyp
b. Septal hematoma
c. Septal abscess
d. Deviated nasal septum
e. Fx nose
Fever= abscess.

Hematomas are not associated with fever unless they are infected.

861. A 12yo pt came to the OPD with complains of fever, malaise, weight loss, anorexia and productive
cough. Exam: temp=39C, pulse=100bpm. His mother says that he has a hx of recurrent chest infections and he
is not thriving well. What is the single most likely causative organism?
a. Pneumococcal pneumonia
b. Staphylococcus
c. Mycobacterium TB
d. Pseudomonas
e. PCP
Pseudomonas common infective agent in CF
862. A 3yo child brought by his mother. Exam: bruises on the buttocks. Mother also gives hx of runny nose
2wks ago. What is the single most appropriate next action?
a. Check child protection register
b. Coag profile
c. Skeletal survey
d. Continue regular child care
e. Inform police
This is HSP.
Answer should be B.
NAI also presents this way with bruise fractures multiple callus formation but the history is a bit different like
step father came to emergency department.
But again most of the options are given for NAI.

863. A 4yo is brought to the ED by ambulance. His mother reports that he has been unwell with a sore
throat for 8h. He is sitting on his mothers knee and is tolerating an oxygen mask but looks unwell. He has
constant noisy breathing and he is drooling saliva. His temp=39C. What is the most imp dx?
a. Acute asthma
b. Bronchiolitis
c. Croup
d. Epiglottitis
e. Tonsillitis
In epiglotittis there is always drooling of saliva

864. A pt with terminal cancer is being treated with chemo develops tingling and numbness of the
fingertips of both arms. What is the single most likely cause of the symptoms?
a. Bone mets to cervical vertebrae
b. Post-chemo neuropathy
c. Hyponatremia
d. Hypocalcemia
865. An 80yo man has a permanent catheter. Catheter specimen urine found lots of e-coli. What is the
single most appropriate management as he wants to attend his daughters wedding next week?
a. Change the catheter
b. Prolonged antibiotics
c. Bladder wash
d. Repeat MSU after wedding
e. Reassure
Catheter has become infected due to presence of e coli if it is left aside it will lead to catheter related UTI
866. A 35yo male typist who suffered a scaphoid fx was treated with a scaphoid cast. After 2wks when the
cast was removed for a review XR, it was found that he had problems in moving the thumb, index and middle
fingers. What would you suggest as the management for the recent prb?
a. Release of flexor retinaculum
b. Release of common flexor sheath
c. Release of palmar sheath
d. Ulnar nerve release
e. Fasciotomy
867. A pt on insulin is booked in for a hernia operation. What is the most appropriate management of
insulin?
a. Give insulin and saline pre-op
b. Stop insulin for the duration of the op
c. Give IV insulin + dextrose + saline pre-op
d. Give insulin as usual pre-op
e. None
868. A 35yo male who recently had an appendicectomy has got severe pain in his right big toe. Joint is red
and swollen. He consumes 30 units of alcohol/week. What is the most probable dx?
a. Rhabdomyosarcoma
b. Osteoarthritis
c. Gout
d. Pseudogout
e. Arthritis
869. A 25yo male who recently noticed change in his shoe size, he is also constipated, has a preference to
hot weather, his skin is dry, has severe pain in wrist joint. Joint is red and swollen. What is the most probable
dx?
a. Chondro-sarcoma
b. Lipo-sarcoma
c. Gout
d. Pseudogout
e. Ankylosing spondylitis
870. A 45yo woman had her visual acuity checked at her local optician. 12h later she presents to the ED
with severe pain and redness in her eye. What is the single most appropriate option?
a. Iris
b. Ciliary body
c. Ant chamber
d. Post chamber
e. Cornea
871. A 75yo man who has DM and HTN experiences acute monocular blindness which resolves after 1h.
What is the most likely dx?
a. GCA
b. Optic neuritis
c. Lacunar infarct
d. Pontine hemorrhage
e. Amaurosis fugax
872. A 26yo presents with prolonged constipation, blood on side of stool and very painful defecation. PR
exam: very painful. What is the single most likely dx?
a. Ca Colon
b. UC
c. CD
d. Anal fissure
e. Constipation
873. A 35yo man with painless left testicular enlargement for the past 6m which is increasing in size and 3x
larger than the right side. There is no tenderness or redness. What is the most likely dx?
a. Testicular tumor
b. Hydrocele
c. Epididymal cyst
d. Epididymo-orchitis
e. Reassure
874. A middle aged man who has had a hx of chronic sinusitis, nasal obstruction and blood stained nasal
discharge. He now presents with cheek swelling, epiphora, ptosis, diplopia, maxillary pain. What is the single
most likely dx?
a. Nasopharyngeal ca
b. Pharyngeal ca
c. Sinus squamous cell ca
d. Squamous cell laryngeal ca
e. Hypopharyngeal tumor
875. A 60yo man with a long hx of smoking and alcohol presents with nasal obstruction, epistaxis, diplopia,
otalgia and conductive deafness. What is the single most likely dx?
a. Nasopharyngeal ca
b. Pharyngeal ca
c. Sinus squamous cell ca
d. Squamous cell laryngeal ca
e. Hypopharyngeal tumor
876. A 60yo is on tx for IHD, HTN and hyperlipidemia. During the night he complains of wheeze and SOB.
Which of the following meds is responsible for that?
a. Amlodipine
b. Atenolol
c. Ramipril
d. Simvastatin
e. Bendroflumethiazide

Key : B
Clincher : wheeze and SOB which is a very common side effect of Atenolol, a beta blocker which are
contraindicated in asthma as it causes severe bronchospasm leading to wheeze and SOB

SOB is less common with calcium channel blockers, ACE inhibitors and anti hyperlipidemics. Thiazide diuretics
do not cause SOB

BETA BLOCKERS

Beta-blockers are medicines that are used to treat a variety of conditions. Their full correct name is beta-
adrenoceptor blocking medicines. There are several types of beta-blocker. They include acebutolol, atenolol,
bisoprolol, carvedilol, celiprolol, labetalol, metoprolol, nadolol, nebivolol, oxprenolol, pindolol, sotalol,
propranolol and timolol.

Beta-blockers work by blocking the transmission of certain nerve impulses. The ends of some nerves release a
chemical (neurotransmitter) called noradrenaline when the nerve is stimulated. This chemical then stimulates
beta-adrenergic receptors. These receptors are tiny structures which occur on cells in various parts of the
body including the heart, brain, and blood vessels. When these receptors are stimulated, they cause various
effects. For example, nerve impulses to the heart can stimulate beta-adrenergic receptors on heart cells. This
causes an increase in the force and rate of the heartbeat.

The beta-adrenergic receptors are also stimulated by adrenaline (epinephrine), a hormone which circulates in
the bloodstream. Adrenaline is made in the adrenal gland.

The beta-blocker medicine 'sits' on beta-adrenergic receptors and stops (blocks) the receptor from being
stimulated. So, for example, if beta-adrenergic receptors in the heart are blocked, the force and rate of the
heartbeat are reduced.

Indications :
prevent angina pains
reduce risk of further MI
control certain arrythmias
treat heart failure
lower bkood pressure
glucoma
anxiety
overactive thyroid gland
migraine
Common side effects include :
Blurred vision
cold hands or feet
confusion
difficult or labored breathing
dizziness, faintness, or lightheadedness
shortness of breath
sweating
tightness in chest
unusual tiredness or weakness
wheezing

Contraindications :
asthma
uncontrolled heart failure
bradycardia
hypotension
certain problems with rythm of heart (source: patient.co.uk and mayo clinic)

877. A 15yo boy who complains of pain in his leg which has settled with aspirin. What is the most probable
dx?
a. Leiomyosarcoma
b. Liposarcoma
c. Painful hip
d. Exostosis
e. Osteoid osteoma

Key : e
Clincher : age and pain settling with aspirin

Leiomyosarcoma is rare occurs over 50yrs of age


Liposarcoma occurs in adults
Exostosis is bone growth commonly in ribs and is not relieved by aspirin

OSTEOID OSTEOMA

Benign bone forming tumors in children particularly adolescents


has a characteristic lucent nidus <2 cm and surrounding solid periosteal reaction
tumor cells proliferate rapidly producing high amounts of PGE2
it is a very vascular tumor, allowing drugs to penetrate
aspirin inhibits PGE2 synthesis providing analgesia

878. A 20yo fit man suddenly developed severe lower back pain as getting up from bed. What is the single
most probable dx?
a. Pagets disease
b. Multiple myeloma
c. PID
d. AS
e. Spondylosis

Key : c
Clincher : age and sudden onset of pain

In pagets disease the bone forming process becomes fast and out of control. Commonly, there are no
symptoms and it is diagnosed by chance when x ray is done for another reason.
Myeloma is a cancer that affects cells in the bone marrow, called plasma cells. As the cancerous plasma cells
fill the bone marrow, you are not able to make enough normal blood cells. The symptoms and problems which
develop are mainly due to the uncontrolled production of plasma cells in the bone marrow, and the excess
amount of antibody (paraprotein) that the plasma cells make.
AS is a persistent (chronic) arthritic (rheumatic) disease of unknown cause. It mainly affects the spine and the
sacroiliac joints. The main symptom is back pain. The pain usually starts in the lower back. You may think of it
as just mild backache at first. It typically becomes gradually worse over several months.
Spondylosis literally means stiffening or fixation of the bony building blocks of the spine (vertebrae) as the
result of a disease process. Spondylosis refers to degenerative changes in the spine such as bone spurs and
degenerating intervertebral discs. Many people with spondylosis on X-ray do not have any symptoms. In fact,
lumbar spondylosis (spondylosis in the low back) is present in 27%-37% of people without symptoms.

PID ( prolapsed intervertebral disc) (source : patient.co.uk)

A 'slipped' (prolapsed) disc often causes severe lower back pain. The disc often presses on a nerve root which
can cause pain and other symptoms in a leg. In most cases, the symptoms ease off gradually over several
weeks. Bouts of back pain are very common. However, less than 1 in 20 cases of sudden-onset (acute) back
pain are due to a 'slipped' (prolapsed) disc. The most common age to develop a prolapsed disc is between 30
and 50 years. Twice as many men as women are affected.
symptoms of a prolapsed disc

Back pain : The pain is often severe and usually comes on suddenly. The pain is usually eased by lying down
flat and is often made worse if you move your back, cough or sneeze.

Nerve root pain (usually sciatica) : Nerve root pain is pain that occurs because a nerve coming from the spinal
cord is pressed on (trapped) by a 'slipped' (prolapsed) disc, or is irritated by the inflammation caused by the
prolapsed disc.

The irritation or pressure on the nerve next to the spine may also cause pins and needles, numbness or
weakness in part of a buttock, leg or foot. The exact site and type of symptoms depend on which nerve is
affected.

Your doctor will normally be able to diagnose a 'slipped' (prolapsed) disc from the symptoms and by examining
you. (It is the common cause of sudden back pain with nerve root symptoms.) In most cases, no tests are
needed, as the symptoms often settle within a few weeks. Tests such as X-rays or scans may be advised if
symptoms persist. In particular, an MRI scan can show the site and size of a prolapsed disc

TREATMENT :
Exercise and keep going

Painkillers, if needed should be taken regularly

Some people visit a physiotherapist, chiropractor or osteopath for manipulation and/or other
physical treatments.

surgery may be considered if the symptoms have not settled after about six weeks or so.

879. A 60yo man brought to the ED with fx hip, he is deaf and has bilateral pedal edema. What is the single
most probable dx?
a. Pagets disease
b. Osteoporotic fx vertebra
c. Secondary
d. Multiple myeloma
e. Spondylosis

Key : a
Clincher : bilateral pedal edema and deafness

Osteoporosis, multiple myeloma and spondylosis do not cause pedal edema as their complication.

PAGET'S DISEASE

In Paget's disease, the bone-making process (bone turnover) becomes faster and out of control. Affected areas
of bone form new bone material in an abnormal way. The newly formed bone is thicker than normal and the
bone may become wider. However, the bone is not made properly and it is weaker than normal bone. This can
cause deformity of the affected bone, particularly in weight-bearing bones such as the leg bones, which may
bend. Affected bone is also more likely to break (fracture).

In affected areas of bone there are abnormal osteoclasts which are bigger than normal and more in number.
They dissolve more than normal bone. As a response to this, the osteoblasts increase in activity to make new
bone material. But this increase in bone turnover leads to badly structured areas of bone that are wrongly
woven.

It mainly affects people aged over 50. The disease affects three men for every two women.

Symptoms :

One or more bones may be affected. The bones most commonly affected are:
The pelvis
The thigh bone in the upper leg (femur)
The bones (vertebrae) of the spine
The skull
The shin bone (tibia).

In many cases there are no symptoms


More than 9 in 10 people with Paget's disease have no symptoms.
The disease is commonly found by chance when an X-ray is taken for another reason.

Pain
Deformity
Fractures
(Affected bones are more liable to break (fracture). A fracture after a minor fall or injury may be the first
indication that Paget's disease has developed.)
Nerve compression
Joint inflammation (arthritis)

Complications :

Heart failure
Vascular steal syndrome
(If the skull is extensively affected, the extra blood flow needed to supply the enlarged skull may be 'stolen'
from the blood supply going to the brain. This may result in you becoming tired and listless. )
Bone cancer
Hypercalcemia
Gout

Investigations :

Xray
Bone scan
Blood tests ( alkaline phosphatase rise with increase in bone turnover)
Biopsy, if the diagnosis is uncertain

Treatment :

No treatment is required if pt. is asymptomatic


Treatment is advised when there is pain, bone deformity or complication
Treatment is also advised if paget's disease is in a site where it may cause problem. (At base of the skull it can
compress the
Ear nerve and lead to deafness)

Bisphosphonates given for several weeks or months reduce the abnormal bone turnover
Painkillers
Calcium and vitamin D
Walking sticks and shoe raises
Calcitonin
Surgery

880. An 80yo lady presents with pain on left 6th rib for a week. It is non-tender on examination. What is your
most likely dx?
a. Herpes zoster
b. Costochondritis
c. Bone degeneration
d. Thoracic vertebra compression
Key : c
Clincher : age and non tender

Herpes zoster ... there's no vesicular eruption that occurs in 2-3days


Costochondritis .... it is tender to touch
Thoracic vertebra compression ... this will cause limb weakness and bowel involvement

So in an 80yrs old its common to have bone degenerative disease, which includes both osteopenia and
osteoporosis.
Both men and women start having age-related bone loss at about age 50, but bone loss can be accelerated in
individuals who didn't develop maximum peak bone mass. There are two levels of bone loss that can occur
and are associated with an increased risk for fractures:

Osteopenia: Osteopenia, which is loss of bone mineral density (BMD), is the warning siren that the bones are
thinning. This phase begins when existing bone breaks down faster than the body can replace it. Preventing
transition into osteoporosis takes the combination of exercise, calcium, and possibly some medications
prescribed by your doctor.

Osteoporosis: Osteoporosis Latin for porous bone takes years to develop as bones slowly lose minerals,
density, and structure, which makes them weaker. If left untreated, osteoporosis can lead to stooped posture,
loss of height, and broken bones.The good news is that not everyone ends up with osteoporosis and there are
tests to determine how dense your bones are.

Women's bones are smaller and less dense than men's, and women are four times more likely than men to
suffer from osteoporosis. This is because men in their 50s don't experience the rapid loss of bone mass that
women do in the years following menopause. By age 65 or 70, however, men and women are losing bone
mass at the same rate, and the absorption of calcium decreases in both sexes. Excessive bone loss causes
bone to become fragile and more likely to fracture.

881. A 68yo DM, HTN with a 45pack/year smoking hx, has left sided chest pain increased with breathing.
Exam: myosis on left side and wasting of small muscles of left hand. What is the single most appropriate dx?
a. Costochondritis
b. Lung cancer
c. Good pastures syndrome
d. MND
e. Progressive massive fibrosis

Key : b
Clincher : age, long h/o smoking, myosis, wasting of small muscles of hand

Costochondritis ... chest pain is close to sternum mostly 4,5,6th ribs. There is tenderness
Good pasture's syndrome ... Goodpasture's Syndrome is an uncommon autoimmune disease that affects both
the kidneys and the lungs. Doesn't cause myosis or wasting of muscles
MND ... Motor neurone disease (MND) causes a progressive weakness of many of the muscles in the body.
The main types of MND are:
Amyotrophic lateral sclerosis (ALS), Progressive bulbar palsy (PBP), Progressive muscular atrophy (PMA) &
Primary lateral sclerosis (PLS).
PANCOAST TUMOR (Lung cancer)

Classically caused by an apical (superior pulmonary sulcus) malignant neoplasm of the lung. The neoplasm is
usually bronchogenic in origin (most commonly squamous cell carcinoma, sometimes adenocarcinoma and
large-cell carcinoma).

This syndrome results from the invasion of a number of structures and tissues around the thoracic inlet and
may be characterised by:An ipsilateral invasion of the cervical sympathetic plexus leading to Horner's
syndrome (miosis, enophthalmos, ptosis; in 14-50% of patients). Ipsilateral reflex sympathetic dystrophy may
occur.Shoulder and arm pain (brachial plexus invasion C8-T2) leading to wasting of the intrinsic hand muscles
and paraesthesiae in the medial side of the arm.Less commonly, unilateral recurrent laryngeal nerve palsy
producing unilateral vocal cord paralysis (hoarse voice bovine cough), and/or phrenic nerve
involvement.There may be arm oedema secondary to the compression of blood vessels.Superior vena cava
syndrome may also occur.

This is similar to other lung cancer emphasising imaging, including CXR and CT scan of the lungs and abdomen,
and also possibly positron emission tomography (PET). MRI is the imaging of choice to assess structures at the
thoracic inlet prior to surgery. Brain CT or MRI are required, as it is the most common site of metastases. Good
biopsy results are usually achieved by percutaneous methods.

Originally, Pancoast's tumour was fatal due to involvement of vital structures at the thoracic inlet. This has
improved with multimodality treatment, including induction chemoradiotherapy (usually cisplatin-based)
followed by resection. Resection may involve a wedge resection or a lobectomy.

882. A 34yo man had a cold 2d back. He now presents with right sided facial pain. What is the single most
likely dx?
a. Maxillary sinus
b. Ethmoid sinus
c. Septal hematoma
d. Septal abscess
e. Allergic rhinitis
Key : a
Clincher : age, h/o cold, facial pain

Ethmoid sinus ... it is more common in children


Septal hematoma ... collection of blood within the septum of the nose, it doesnt cause facial pain
Septal abcess ... a serious condition that is caused by bacteria. Trauma to the nose, or even nasal surgery, can
leave the patient prone to develop a nasal abscess, which is basically a pocket filled with blood (haematoma)
which has become affected by bacteria. Symptoms include nasal blockage, pain, redness over the nasal bridge,
difficulty in breathing and fever.
Allergic rhinitis ... doesnt cause facial pain

MAXILLARY SINUS (source : patient.co.uk)

The sinuses are small, air-filled spaces inside the cheekbones and forehead. They make some mucus which
drains into the nose through small channels.
Sinusitis means inflammation of a sinus. Most bouts of sinusitis are caused by an infection. The cheekbone
(maxillary) sinuses are the most commonly affected.
Acute sinusitis means that the infection develops quickly (over a few days) and lasts a short time. Many cases
of acute sinusitis last a week or so but it is not unusual for it to last 2-3 weeks.
Chronic sinusitis means that a sinusitis becomes persistent and lasts for longer than 12 weeks. Chronic sinusitis
is uncommon.

Risk factors :
Cold/flu
Dental infection
Allergic rhinitis
Nasal polyp
Facial injury or surgery
Foreign body
Smoking
Asthma
Pregnancy
Poor immune system

Symptoms :
The most typical symptoms are headache and pain in the face, with the latter increasing when the patient
bends down, lifts something, coughs etc., that is, when pressure increases in the sinuses. In case of acute
maxillary sinusitis pain is much stronger than in the case of the chronic type.
Blocked/runny nose
Toothache
Fever
Headache
Cough
Tiredness

Treatment :

Antibiotics are needed only :If your symptoms are severe or if you are very unwell.If you have another illness
such as cystic fibrosis, heart problems or a weakened immune system.If your symptoms are not settling within
seven days, or are worsening.

Treatment to relieve symptoms :


Painkillers
Decongestant nasal spray or drop
Keeping hydrated
Warm face packs
Saline nasal drops
Steam inhalation

883. A 29yo man with hx of asthma comes with post nasal discharge and bilateral painless nasal blockage.
What is the single most likely dx?
a. Nasal polyp
b. Septal hematoma
c. Septal abscess
d. Atopic rhinitis
e. Allergic rhinitis
Key : a
Clincher : h/o asthma

Septal hematoma and septal abcess are not associated with asthma
Atopic and allergic rhinitis will have watery discharge but no PND

NASAL POLY (source:patient.co.uk)

Nasal polyps are soft fleshy swellings that grow inside the nose. They may be yellowish, grey or pink in colour.
They are common and non-cancerous (benign). Nasal polyps can vary greatly in size. There may be only one
but sometimes several grow like a small bunch of grapes on a stem.

Certain conditions make nose inflammation and polyps more likely. These include asthma, an allergy to
aspirin, cystic fibrosis and some rare conditions of the nose (such as allergic fungal sinusitis and Churg-Strauss
syndrome).

Symptoms :

blocked feeling in the nose.


difficult to breathe through your nose.
Watering from the nose (rhinorrhoea) is common.
A postnasal drip may occur.

An ENT surgeon can usually diagnose nasal polyps based on your symptoms and on examination of your nose.

Occasionally a CT scan or MRI scan may be needed. These scans may show more detail about where the
polyps are and what effects they might have had on other parts of the face, sinuses and skull.

Treatment :
Steroid nose drops
Steroids tablets
Surgery ( polypectomy / endoscopic sinus surgery)

884. A 24yo man has been found unconscious in an alleyway with a RR=6bpm and HR=60 bpm. His pupils
are constricted. What is the best tx?
a. Methadone
b. Naloxone
c. Naltrexone
d. Thiamine
e. Glucose
Key : b
Clincher : miosis, respiratory depression

Methadone is used for severe refractory pain


Naltrexone is used for opioid dependence
Thiamine deficiency, which is found in a large number of alcoholics, is an important contributor to alcohol
related brain damage of all kinds. Depending on the level of thiamine deficiency, symptoms can vary greatly.
There are two primary types of thiamine deficiency: wet beriberi and dry beriberi.
Glucose is given in hypoglycemia which will not present with constricted pupils

OPIOID POISONING ( morphine toxicity here )

Opiate poisoning can occur at any time from birth (when pethidine given to the mother in labour may
suppress ventilation) to terminal care. The outcome can range from minor adverse effects such as constipation
to death from respiratory depression.

Symptoms

Opiate poisoning may be a chronic problem, in which case the main complaint will be of constipation. There
may be nausea, vomiting or just loss of appetite. There may be sedation and craving for the next dose.Acute
toxicity presents with drowsiness that will be more severe if there is also alcohol involved, or involvement of
other sedatives. There may be nausea or vomiting.

Signs

Respiratory depression may be apparent. Hypotension and tachycardia are possible. There are usually
pinpoint pupils but this sign may be absent if other drugs are involved.The 'post-mortem sole incision' sign has
been identified. This is an incision made in the sole by an acquaintance in the belief that the subsequent blood
loss will reduce the likelihood of death in an individual who has taken an accidental overdose of an opiate.

Management

Do not delay establishing a clear airway, adequate ventilation and oxygenation if consciousness is impaired.
Give naloxone intravenously (IV) (0.4-2 mg for an adult and 0.01 mg/kg body weight for children) if coma or
respiratory depression is present.
Give intramuscularly (IM) if no vein is available. Repeat the dose if there is no response within two minutes.
Naloxone is a competitive antagonist and large doses (4 mg) may be required in a severely poisoned patient.

885. A 23yo female presents with back pain and early morning stiffness, also complaining of eye problem
and her sister has a similar condition. What is the single most probable dx?
a. Pagets disease
b. PID
c. Myofacial pain
d. AS
e. Spondylosis

Key : d
Clincher : back pain, early morning stiffness, eye problem

Paget's disease ... pain is mostly worse at night


PID ... no stiffness
Myofascial pain ... Myofascial pain syndrome is a chronic pain disorder. In myofascial pain syndrome, pressure
on sensitive points in your muscles (trigger points) causes pain in seemingly unrelated parts of your body. This
is called referred pain.
Spondylosis ... Spondylosis literally means stiffening or fixation of the vertebrae as the result of a disease
process. There is no treatment to reverse the process of spondylosis, because it is a degenerative process. The
treatments of spondylosis target the back pain and neck pain that spondylosis can cause.

ANKYLOSING SPONDYLITIS

AS is a persistent (chronic) arthritic (rheumatic) disease of unknown cause. It mainly affects the spine and the
sacroiliac joints. Sometimes other joints and other parts of the body are affected.
AS most commonly begins between 20 and 30 years of age, but sometimes first develops in children and older
adults. It is three times more common in men than in women. There may be a family history with two or more
members of a family being affected.
There is a strong hereditary (genetic) tendency. For example, there is a strong association with a gene called
HLA-B27.

Symptoms :

Back pain
Rest does not make it better. In fact, the pain may wake you from sleep. Instead, exercise and movement
usually ease the pain. The pain tends to be worse first thing in the morning. Lying in bed after waking is often
uncomfortable. The pain tends to ease as the day goes on.
Stiffness in the lower spine
Inflammation of tendons and ligaments
Inflammation of part of the eye (uveitis)

Until recently, the X-ray changes were the only way to confidently confirm AS. More recently, an MRI scan of
the sacroiliac joints has been used to confirm the diagnosis at an earlier stage. An MRI scan can give a much
more detailed view of a joint than a traditional X-ray picture and can detect inflammation in the sacroiliac
joints.

Symptoms can vary in severity and usually wax and wane. Flare-ups of inflammation which cause periods of
worse pain and stiffness tend to occur from time to time. If joints outside your spine are affected, they tend to
flare up at the same time as back symptoms. The number of flare-ups that occur, how severe they are, and
how long they last, can vary greatly from person to person.

886. A 63yo female with a hx of osteoporosis suddenly falls on her outstretched hand while shopping. XR
shows fx at distal radius with backward shift of the distal fragment. What is the single most probable
deformity?
a. Dinner fork deformity
b. Coxavara
c. Mallet finger
d. Cubitus valgus
e. Garden spade deformity

Key : a
Clincher : backward shift of distal fragment

Coxavara ... a deformity of the hip


Mallet finger ... an injury to the thin tendon that straightens the end joint of a finger or thumb, also known as
"baseball finger".
Cubitus valgus ... a deformity in which the forearm is angled away from the body to a greater degree than
normal when fully extended.
Garden spade deformity ... fracture at distal radius with forward shift of the distal fragment

Dinner fork deformity

Fracture at distal radius with backward shift of the distal fragment caused due to sudden fall on an
outstretched hand.

887. A 60yo man presents with severe colicky pain from his right flank radiating to his groin. His urinalysis
reveals trace blood cells. What is the single most discriminatory inv?
a. US abdomen
b. XR KUB
c. Colonoscopy
d. Upper GI endoscopy
e. Laproscopy

Key : a ... corrected : b


Clincher : severe colicky pain, flank pain, traces blood cells in urinalysis

Although xray KUB does not show uric acid stones. Around 80% of renal stones are calcium and are seen on
xrays. Plus, in this patient, pain is radiating from loin to groin as in uretric stones which are not visualised by
US. Colicky pain, flank pain and blood cells in urine point towards renal pathology. When a renal pathology is
suspected, US KUB is done, not US abdomen.
Colonoscopy, upper GI endoscopy and laproscopy have different indications.

888. A 45yo man has been admitted for an elective hernia surgery. 3d later he presents with agitation,
sweating, aggressiveness, and complains of seeing snakes on the hosp wall. Chlordiazepoxide has been started
for this pt. What is the most appropriate next step?
a. Diazepam
b. Acamprosate
c. Disulfiram
d. Thiamine
e. Methadone

Key : d
Clincher : postop patient, agitation, aggressiveness, mental confusion

Diazepam ... indicated in anxiety, acute alcohol withdrawl, muscular spasm, convulsive disorders
Acamprosate ... for treatment of alcohol dependence
Disulfiram ... an alcohol abuse deterrent
Methadone ... used for severe refractory pain

Thiamine deficiency has been observed in an appreciable portion of post op patients receiving parenteral
alimentation with glucose but without supplementary thiamine. Thiamine deficiency is of 2 types : wet
beriberi and dry beriberi
The symptoms of dry beriberi are:decreased muscle function, particularly in the lower legs tingling or loss of
feeling in the feet and hands pain mental confusion difficulty speaking vomiting involuntary eye movement
paralysis.
889. A woman with a prv hx of pain at the left wrist following a fall 4m ago for which she didnt seek any tx
now presented with pain in the same wrist below the thumb and the pain is aggravated whenever she holds
her baby. What is the cause?
a. Fx radial head
b. Scaphoid fx
c. Carpal tunnel syndrome
d. Colles fx
e. Ulnar fx

Key : b
Clincher : pain below thumb, aggravated by movement.

Fx radial head ... pain and swelling in elbow joint


Carpal tunnel syndrome ... pain, numbness or tingling in wrist joint, forearm
Colles fracture ... is another name of dinner fork deformity.
Ulnar fracture ... a broken ulna will cause instant pain, tenderness, swelling, painful elbow movement.

SCAPHOID FRACTURE (source : http://www.assh.org/handcare/hand-arm-injuries/scaphoid-fracture)

The scaphoid is one of eight small bones that make up the carpal bones of the wrist. It connects two rows of
these bones - the proximal row and the distal row. This connection puts it at extra risk for injury.
A fracture of the scaphoid bone usually occurs from a fall onto the outstretched hand.
Pain (with or without swelling or bruising at the thumb side of the wrist) can be noticed within days
following a fall.
Scaphoid fractures are usually diagnosed by an x-ray of the wrist; however, x-rays do not always show
scaphoid fractures.
If the fracture is non-displaced (bone has not moved out of place at the fracture), it usually can be successfully
treated with a cast. Although the fracture may heal in as little as six weeks, it may take longer for some
patients. If the fracture is in a certain part of the bone or if the fracture is at all displaced (bone ends have
shifted), surgery might be the best option.

890. A 29yo man was involved in an RTA. He presents with distended neck veins, clear breath sounds and a
trachea which is in the midline. His RR=34bpm, BP=60/0mmHg. What is the most likely dx?
a. Simple pneumothorax
b. Tension pneumothorax
c. Cardiac tamponade
d. Pericarditis

Key : c
Clincher : RTA, clear breath sounds, trachea in midline

Simple pneumothorax ... respiratory distress, distant or absent breath sounds


Tension pneumothorax ... absent breath sounds, trachea shifts away from affected side
Pericarditis ... pain and high grade fever

CARDIAC TEMPONADE (source : patient.co.uk)

Cardiac tamponade is caused by the accumulation of blood, fluid, pus, clots, or gas in the pericardial space,
resulting in reduced ventricular filling and subsequent haemodynamic compromise. Cardiac tamponade is a
medical emergency.

Cardiac tamponade related to trauma or HIV is more common in young adults. Tamponade due to malignancy
and/or chronic kidney injury occurs more frequently in elderly individuals.

Signs

Distended neck veins, hypotension, tachycardia, tachypnoea and hepatomegaly.Muffled heart sounds.
Increased jugular venous pressure (JVP). Cyanosis and pulmonary oedema may occur.

Transthoracic echocardiogram is the investigation of choice.


Patients should be monitored in an intensive care unit. Pericardiocentesis (echocardiography-guided being the
procedure of choice) is the definitive treatment but may be hazardous and not relieve symptoms in cases of
small effusions associated with constrictive pericarditis - eg, malignancy, autoimmune conditions and viral
infection.Oxygen.Volume expansion to maintain adequate intravascular volume - small boluses work best.[7]
Improve venous return: bed rest with leg elevation.Positive inotropic drugs: eg, dobutamine.

891. An elderly woman is found anemia. As part of her exam, she had a barium enema which reveals a mass
lesion in the ascending colon. What is the single most appropriate dx?
a. Sigmoid volvulus
b. Anal fissure
c. Sigmoid carcinoma
d. Cecal carcinoma
e. Diverticular disease

Key : d
Clincher :

Sigmoid volvulus ... more common in men, presents with sudden colicky lower abdominal pain with distension
Anal fissure ... An anal fissure is a small tear of the skin around the anus. Often an anal fissure will bleed a
little. You may notice blood after you pass faeces. The blood is usually bright red, and a small amount may be
seen on the toilet paper or coating your faeces.
Sigmoid carcinoma ... mass lesion would have been on left side.
Diverticular disease ... lower abdominal pain, usually left-sided.

COLON CANCER

Cancer of colon and rectum can exhibit itself in several ways. If you have any of these symptoms, seek
immediate medical help. You may notice bleeding from your rectum or blood mixed with your stool.People
commonly attribute all rectal bleeding to hemorrhoids, thus preventing early diagnosis owing to lack of
concern over "bleeding hemorrhoids." New onset of bright red blood in the stool always deserves an
evaluation. Blood in the stool may be less evident, and is sometimes invisible, or causes a black or tarry
stool.Rectal bleeding may be hidden and chronic and may only show up as an iron deficiency anemia.It may be
associated with fatigue and pale skin due to the anemia.

If the tumor gets large enough, it may completely or partially block your colon. You may notice the following
symptoms of bowel obstruction:
Abdominal distension
Abdominal pain is rare in colon cancer.
persistent nausea or vomiting
Unexplained weight loss
Change in frequency or character of stool. Small-caliber (narrow) or ribbon-like stools
Sensation of incomplete evacuation after a bowel movement
Rectal pain: Pain rarely occurs with colon cancer and usually indicates a bulky tumor in the rectum that may
invade surrounding tissue.

892. A 55yo male after gastrectomy developed anemia. His MCV=106fl. Exam: loss of proprioception and
vibration sense. What is the most likely dx?
a. IDA
b. Folate def
c. Vit B12 def
d. Anemia of chronic disease

Key : c
Clincher : gastrectomy, anemia, mcv, loss of proprioception and vibration sense

IDA ... mcv is decreased


Anemia of ch. Disease ... mcv is decreased
Folate def. Anemia ... mcv is increased but it doesn't affect proprioception and vibration sense

VITAMIN B12 DEFICIENCY ANEMIA (source : patient.co.uk)

Vitamin B12 is present in meat and animal protein foods. Absorption occurs in the terminal ileum and requires
intrinsic factor (IF), a secretion of gastric mucosal (parietal) cells, for transport across the intestinal mucosa. In
pernicious anaemia, IF production is deficient. It is believed to be an autoimmune disease.Helicobacter pylori
infection has been mooted to be an initiating factor, with subsequent autoimmune changes affecting the
gastric mucosa. Genetic susceptibility to this process has been suspected.[1]

Causes
Pernicious anaemia accounts for 80% of cases of megaloblastic anaemia due to impaired absorption of vitamin
B12.
Other causes of vitamin B12 deficiency include:
Gastric causes: gastrectomy, gastric resection, atrophic gastritis, H. pylori infection or congenital IF deficiency
or abnormality.I
Inadequate dietary intake of vitamin B12 - eg, a vegan diet.
Intestinal causes - eg, malabsorption, ileal resection, Crohn's disease affecting the ileum, chronic tropical
sprue, HIV and any radiotherapy causing irradiation of the ileum.
Drugs - eg, colchicine, neomycin, metformin, anticonvulsants.
Long-term use of drugs that affect gastric acid production (eg, H2 receptor antagonists, proton pump
inhibitors) can worsen deficiency because gastric acid is needed to release vitamin B12 bound to proteins in
food.

Presentation
Symptoms of anaemia may include fatigue and lethargy, dyspnoea, faintness, palpitations and headache.
Vitamin B12 deficiency may present with unexplained neurological symptoms - eg, paraesthesia, numbness,
cognitive changes or visual disturbance.

Peripheral loss of vibratory sense and position are early indications of central nervous system (CNS)
involvement, accompanied by reflex loss and mild-to-moderate weakness. Later stages may be characterised
by spasticity, Babinski's responses and ataxia.

Management
For patients with neurological involvement, referral to a haematologist is recommended. Initial treatment is
with hydroxocobalamin 1 mg on alternate days until there is no further improvement, after which 1 mg should
be given every two months for life.

893. A 26yo male has been operated for abdominal trauma and splenectomy was done. On the 3 rd post-op
day the pt developed acute abdominal pain and distention in the upper abdominal area with hypotension. On
insertion of ryle's tubes, 2L of coffee ground fluid was aspirated. What is the most probable dx?
a. Acute gastric dilatation
b. Reactionary hemorrhage
c. Subphrenic abscess
d. DVT
e. Left lower lobe atelectasis

Key : a
Clincher : splenectomy, 3rd postop day, upper abdominal distension,

Reactionary hemorrhage ... occurs during first 48hrs


Subphrenic abscess ... takes longer to develop
DVT ... causes pain in leg normally
Left lower lobe atelectasis ... can occur secondary to any surgery but does not cause upper abdominal
distention

ACUTE GASTRIC DILATATION is one of the post splenectomy complications which if not treated promptly leads
to death.

894. A 50yo man presented with increased breathlessness at rest. He is currently on furosemide, digoxin
and isosorbide mononitrate. What drug is going to help him?
a. Ramipril
b. Bendroflumethiazide
c. Atenolol
d. Amlodipine
e. Diltiazem

Key : b
Clincher : increased breathlessness at rest, on furosemide already

Ramipril ... ACE inhibitor to treat high blood pressure decreasing the risk of heart attack and stroke
Atenolol ... beta blocker to treat angina, hypertension and prevent heart attack
Amlodipine ... calcium channel blocker used to treat hypertension, angina and other conditions caused by
coronary artery disease
Diltiazem ... calcium channel blocker used to treat hypertension, angina and certain heart rhythm disorders.

895. A 31yo man underwent an operation where his hand was hanging outside the table. After the
operation he had wrist drop and sensory loss over the dorsum of his hand. Which nerve was injured?
a. Radial
b. Ulnar
c. Median
d. Axillary
e. Brachial

Key : a
Clincher : wrist drop, sensory loss over dorsum of hand

The radial nerve and its branches provide motor innervation to the dorsal arm muscles (the triceps brachii and
the anconeus) and the extrinsic extensors of the wrists and hands; it also provides cutaneous sensory
innervation to most of the back of the hand.

896. What is the mode of spread of chickenpox?


a. Airborne
b. Close contact
c. Fecal-oral
d. Blood
e. Vector

897. A 64yo man presents with ipsilateral vertigo, tinnitus and left side hearing loss. Exam: Renne test +ve
and Webers lateralizes to the right ear. What is the most appropriate inv?
a. CT
b. MRI brain
c. XR
d. Audiometry
e. None
f. Caloric testing

Key : b
Clincher : vertigo, tinnitus, hearing loss

MRI has largely superseded CT scanning as the investigation of choice for suspected acoustic neuroma.
(Patient.co.uk)

ACOUSTIC NEUROMA (source : nhs )

An acoustic neuroma is a benign (non-cancerous) growth, or tumour, in the brain. It's also known as a
vestibular schwannoma.An acoustic neuroma grows on the vestibulocochlear nerve, which helps control
hearing and balance. This nerve runs alongside the facial nerve, which carries information from the brain to
the face muscles.

A small acoustic neuroma can lead to hearing loss or tinnitus, vertigo


A large acoustic neuroma can cause headaches with blurred vision, numbness or pain on one side of the face,
problems with limb co-ordination on one side of the body.

The cause of most acoustic neuromas is unknown, but a small number of cases (about 5%) are caused by a
rare, inherited condition called neurofibromatosis type 2.Acoustic neuromas grow from the cells that cover
the vestibulocochlear nerve, called Schwann cells.
The three types of test you may have are :
Hearing tests, MRI, CT

Acoustic neuromas tend to grow slowly and don't spread to other parts of the brain.They can be so small and
grow so slowly that they may not cause any symptoms or problems. In such cases, the acoustic neuroma may
just be monitored to avoid risks associated with surgery.In rare cases, the tumour can grow large enough to
press on the brain. However, most acoustic neuromas can be treated before they reach this stage, either with
surgery to remove the tumour or radiotherapy to destroy it.

898. A 67yo man presents to the ED with pain in his left groin. He suddenly collapses and his is not able to
move or lift his leg. He is on alendronate. What is the dx?
a. Fx of neck of femur
b. Post hip dislocation
c. Fx of shaft of femur
d. Pelvic base fx
e. Peripheral vascular disease

Key : a
Clincher : age, pain in groin, unable to move leg, on alendronate

Alendronate being a bisphosphonate is used in osteoporosis in which fracture of neck of femur is very
common.

899. A young male met with a RTA and is suspected to have a femur fx. His BP is 90/60mmHg. What is the
next immediate action?
a. XR
b. IV fluids
c. Put leg splint
d. Send bloods for inv
e. US

Key : c
Clincher : RTA, femur fracture, low BP

In case of traumatic hemorrhagic shock, primary goal is to stop bleeding.

Shaft of femur fractures (source : patient.co.uk)

These are caused by a high-energy injury, such as road traffic accidents, unless pathological fracture in a
patient with osteoporosis or metastatic disease.There are often associated injuries to the hip, pelvis, knee and
other parts of the body.

Diagnosis

Deformity, shortening, external rotation and abduction at the hip on the affected side.

Management
Initial management:
Assess vital functions and any associated injuries.
Resuscitate and treat life-threatening injuries as necessary.
Splint fractures (Thomas' splint or equivalent traction splint)
X-rays of the femur.
Blood tests, including blood for cross-matching.
Obtain intravenous access and start fluid replacement.
Peripheral sensation and pulses should be closely monitored.
Analgesia: adequate intravenous analgesia. Femoral nerve block is usually effective.

900. A 70yo pt presents with cough and SOB. He stopped smoking cigarettes 2yrs ago but has a 50yr
smoking hx before quitting. CXR=consolidation and bilateral hilar lymphadenopathy. What is the best inv for
this pt?
a. LN biopsy
b. Pleural fluid cytology
c. CT
d. MRI
e. US

Key : a
Clincher : age, long h/o smoking, cough n SOB, consolidation and bihilar lymphadenopathy

Pleural fluid cytology ... only gives suspicion of malignancy


CT ... not diagnostic but helps in staging
MRI ... CT is superior
US ... US guided biopsy is done for definitive diagnosis

LN biopsy ... done for definitive diagnosis

If the CT scan shows the cancer is at an early stage and you are fit to be treated you may be asked to have
another type of scan. This is called a positron emission tomography-CT (PET-CT) scan. This shows up areas of
active cancer and whether it has spread to the lymph glands in the chest. If the cancer has spread to these
glands, you will be offered a biopsy. (Source: patient.co.uk

901. A 27yo pt met with a RTA. While the NGT is passing, bowel sounds are heard in the chest. CXR shows
NGT curled. What is the dx?
a. Diaphragm rupture
b. Aortic rupture
c. Splenic rupture
d. Bowel rupture
e. Liver rupture

Key : a
Clincher : RTA, bowel sounds in chest, NGT curled

An acquired diaphragmatic hernia (ADH) is usually the result of a blunt or penetrating injury. Traffic accidents
and falls cause the majority of blunt injuries.
Symptoms :
Difficulty Breathing
Blue Discoloration of the Skin
Tachypnea
Tachycardia
Diminished or Absent Breath Sounds
Bowel Sounds in the Chest Area
Less-Full Abdomen

Diagnosis :
X-ray
ultrasound scan
CT scan
arterial blood gases (ABG)

902. A 62yo man dx with T2DM with BMI=33. Lifestyle modifications have failed to control blood sugar.
Labs: urea=3.6mmol/l, creatinine=89mmol/l. what is the next appropriate management?
a. Biguanide
b. Sulfonylurea
c. Insulin
d. Glitazone
e. Sulfonylurea receptor binder

Key : a
Clincher : BMI = 33, failure of lifestlye modifications

If BMI is >25 and life style modifications have failed, start biguanides
If BMI is < 25, start sulfonylurea

903. A pt presents with progressive dyspnea. He complains of cough, wheeze and a table spoonful of
mucopurulent sputum for the last 18m. Spirometry has been done. FEV1/FVC=2.3/3.6. After taking
salbutamol, the ratio=2.4/3.7. What is the most likely dx?
a. Chronic bronchitis
b. Asthma
c. Bronchiectasis
d. Lung fibrosis
e. Sarcoidosis

Key : a
Clincher : table spoonful of mucopurulent sputum, FEV1/FVC ratio being <75%

Asthma presents with dyspnea, wheeze and cough with sputum. FEV1/FVC ratio is<75%. COPD is more likely
to cause an ongoing cough with sputum than asthma.
Bronchiectasis ... There is copious amount of sputum. FEV1/FVC is being <75%
Lung fibrosis and sarcoidosis present with dry cough. FEV1/FVC ratio being >75%

Chronic bronchitis presents with cough, sputum, dypnoea and wheeze. Shows obstructive pattern in lung
function tests with FEV1/FVC ratio being <70%
904. A 62yo man presents with cough, breathlessness and wheeze. 24% O2, salbutamol and hydrocortisone
were given. The symptoms havent improved and so nebulized bronchodilator was repeated and IV
aminophylline was given. ABG: pH=7.31, RR=32. What is the next appropriate management?
a. Nasal IPPV
b. Intubation and ventilation
c. LABA
d. Toxapram
e. Amoxicillin PO

Key : a
Clincher : failure after IV aminophylline, pH and RR

If there is no response after IV aminophylline, consider nasal IPPV if RR >30 & pH <735. It is delivered by nasal
mask and a flow generator.
Intubation and ventilation is done if nasal IVVP fails.
(Source : OHCM Pg # 823 8th edition)

905. A young girl returns from holidays in Spain. She complains of discharge from her ear and complains of
tragal tenderness. Exam: tympanic membrane normal. Aural toilet has been done. What is the next
appropriate med?
a. Antibiotic PO
b. Antibiotic IV
c. Steroid PO
d. Steroid drop
e. Antibiotic drop with steroid

Key : e
Clincher : holiday, tragal tenderness

This girl has recently been on vacations. Swimming can cause acute otitis externa with normal tympanic
membrane, tender tragus. It is treated with topical antibiotics. Steroids can be added.

906. A 23yo man sprained his right ankle 6wks ago while playing football. He was tx with a below knee
walking cast. On removal of the cast, the pt noted to have right foot drop. He has weakness of extensors of the
ankle and toes and diminished pin prick sensation over the dorsum of the foot. The ankle jerk is present and
plantar reflex is flexor. What is the most likely cause of the foot drop?
a. Compression of common peroneal nerve
b. Compression of the tibial nerve
c. Compression of the S1 nerve root
d. Rupture of Achilles tendom
e. Tx of the medial collateral lig of the ankle

Key : a
Clincher : right foot drop, wkness of extensorsof ankle and toes, diminished pin prick sensationover the
dorsum of foot

Causes of unilateral foot drop (source: OHCM Pg # 471 8th edition)

DM
common peroneal nerve palsy
Stroke
Prolapsed disc
MS

907. A young man was knocked down during a fight in the waiting room of the ED. He is now unconscious
and unresponsive. What is the 1st thing you would do?
a. Turn pt and put in recovery position
b. Put airway
c. Endotracheal intubation
d. Assess GCS
e. Start CPR

Key : b
Clincher : brought to ED unconcious, unresponsive after a fight

ABC
Airway Breathing Circulation

908. A 52yo man underwent a hemicolectomy. After a few days he complains of left ventricular pain and
fever. ECHO has been done and shows a systolic murmur. What is the next appropriate inv?
a. CT
b. US
c. CXR
d. Blood culture
e. LFT

909. A 19yo man has exercised induced asthma and is using a salbutamol inhaler as req and
beclomethasone 400ug BD. He complains that he has to wake up at night for his inhaler. What is the single
most appropriate tx?
a. Beclo
b. Regular salbutamol and budesonide
c. Sodium cromoglycate
d. Oral steroid
e. Inhaled steroid
Key : c
Clincher : night symptoms of asthma

Usually a long acting beta agonist is needed to cover night symptoms but its not in options. Others dont make
much sense. In essence we want undisturbed sleep. Sodium chromoglycate is the next best option.

910. Pt with a long hx of smoking is now suffering from bronchial ca. histology reveals there are sheets of
large polygonal or giant MNC. What is the most likely dx?
a. Squamous cell ca
b. Small cell ca
c. Adenocarcinoma
d. Large cell ca
e. Oat cell ca
Key : d
Clincher : hitology reveals sheets of large polygonal or giant MNC

Large cell carcinoma ( lung)

It is a poorly differentiated malignant epithelial tumor. Consists of sheets of large polygonal or giant MNC. One
histological variation is large call neuroendocrine carcinoma

911. A 27yo man presents with chest pain and respiratory distress. Exam: tachycardia, hypotension and
neck vein distension. Trachea is deviated to the left side, breathing sounds on right side are absent and
diminished on left side. What is the next appropriate management?
a. CXR
b. Right side aspiration (16G)
c. Left side aspiration (16G)
d. Right side drain with a small tube (12F)
e. Left side drain with a small tube (12F)

Key : b
Clincher : chest pain, resp.distress, hypotension, neck vein distension, deviated trachea, absent breath sounds

Its a case of tension pneumothorax

Indications of simple aspiration include : PSP ( any size) , Small SSP in pts. <50yrs
Needle aspiration (14to16G) is as effective as large bore (>20F) chest drain and is associated with less
hospitalisation and less stay.
Following failed needle aspiration, a small bore (<14F) chest drain insertion is recommended.

912. A 16wk pregnant pt who was exposed to a child with chickenpox came to GP for help. She was tested
ve for varicella antibody. What is the next most imp step in management?
a. Reassurance
b. Ig
c. Ig + vaccine
d. Vaccine only
e. Acyclovir

Key : b
Clincher : 16 wk pregnant, exposed to child with chicken pox, tested -ve for varicella antibody

Ig + vaccine or only vaccine are contraindicated during pregnancy


Acyclovir ... there are no signs of infection

Ig is given because there is only history of exposure and there are no signs of infection yet.

913. A 68yo woman dx with T2DM and BMI=33. Lab: GFR=29, urea=13, creatinine=390mmol/L. what is the
next appropriate management?
a. Biguanide
b. Sulfonylurea
c. Insulin
d. Glitazone
e. Sulfonylurea receptor binder

Key : c
Clincher : BMI = 33, Creatinine = 390 mmol/L

Biguanides ... cant be given if creatinine is > 150mmol/L


Sulfonylurea / sulfonylurea receptor binder ... is given when BMI is < 25
Glitazone ... 2nd line drug which also causes wt.gainby water retention

Insulin indications :
HBA1C > 7.5
DKA
when OHA dont control
Pregnancy
Increased BUN

914. A 5yo boy was brought to GP with high temp and many vesicles on his back. What is the most
appropriate management?
a. Topic acyclovir
b. Oral acyclovir
c. Oral antibiotics
d. Topical steroids
e. None

Key : e
Clincher : age

no treatment is required in children from 28days old to 12years old in chicken pox.

915. A woman came with the complaint of pain in her right arm when she abducts it. She has recently
moved to a new house. There is no hx of trauma. Wht is the likely cause of her pain?
a. Rupture of the long head of biceps
b. Sprain of the acromio-clavicular ligament
c. Tendinitis of the abductor sheat
d. Supraspinatus tendinitis
e. Shoulder dislocation

Key : d
Clincher : pain in her arm when she abducts it

Supraspinatous muscle initiates abduction of arm upto 15 after which deltoid continues the abduction.

916. An 83yo man with longstanding COPD has become progressively breathless over the last 2yrs. He is on
salbutamol, ipratropium, salmetarol, beclomethasone and theophylline. His FEV1<30%. What is the next
appropriate management?
a. Lung transplant
b. Trial of CPAP
c. Trial of non-invasive ventilation
d. Assessment for long term O2 therapy
e. Short course of O2 therapy

Key : d
Clincher : every option has been tried, still has SOB, FEV1 is less than 30%

Lung transplant has no role in old age

when treatment has failed and patient still has SOB ... long term O2 therapy. Indications of LTO2T include :
FEV1 less than 30%
polycythemia
cyanosis
periphral edema
raised JVP
O2 saturation less than 92%

917. A 49yo man complains of fullness in his left ear, recurrent vomiting and tinnitus. What is the most
appropriate med?
a. Buccal prochlorperazine
b. Oral chlorpheniramine
c. Oral flupenphenazine
d. Buccal midazolam
e. IV rantidine

Key : a
Clincher : age, fullness in one ear, vomitting, tinnitus

MENIERES DISEASE (source : nhs)

Its a case of menieres disease.


Mnire's disease is a rare disorder that affects the inner ear. It can cause vertigo, tinnitus, hearing loss, and a
feeling of pressure deep inside the ear.
Mnire's disease often progresses through different stages. In the early stages, most people have sudden and
unpredictable attacks of vertigo, accompanied by nausea, vomiting and dizziness. During the later stages, the
episodes of vertigo tend to occur less frequently and sometimes stop altogether over time. However, the
tinnitus and hearing loss often become worse and you may be left with permanent balance and hearing
problems.

Mnire's disease most commonly affects people aged 20-60 and it's thought to be slightly more common in
women than men.

During an attack of Mnire's disease, you may be prescribed medication to treat the symptoms of vertigo,
nausea and vomiting.This is usually prochlorperazine or an antihistamine. If these work, you may be given a
supply to keep, so you can take them quickly during an attack.

If you experience vomiting during your attacks, you can take a type of prochlorperazine called Buccastem. This
comes as a tablet that you place between your gums and your cheek, on the inside of your mouth. The tablet
dissolves and is absorbed into your body.
918. A man had a soft mass on his mandible. Mass is freely mobile and has started growing progressively
over the past 6m. The mass still moves freely. What is the best inv for this pt?
a. FNAC
b. CT
c. XR
d. MRI
e. ESR

Key : a
Clincher : soft freely mobile mass on mandible

Source OHCM Pg 597 8thed.

salivary gland tumours

80% are in the parotid, 80% of these are pleomorphic adenomas, 80% of these are in the superficial lobe.
Deflectionof ear outwards is a classical signs. Remove any salivary swelling for assessment ifresent for
>1month. 7th nerve palsy means malignancy.

Investigations (source patient.co.uk)

Ultrasound is the usual initial means to assess superficial lesions. Ultrasound is more limited at visualising the
deep lobe of the parotid and some minor salivary glands depending on location.Ultrasound-guided fine-needle
aspiration (FNA) cytology is used to obtain cytological confirmation. CT-guided biopsy can also be used.If deep
tissue extension is suspected or malignancy confirmed on cytology, an MRI or CT scan is used to evaluate
tumour bulk, local invasion and perineural spread.

919. A 63yo man has been brought to the hosp after collapsing during a wedding. His ECG is below. What is
the most likely dx?
a. VT
b. A-fib
c. VF
d. A-flutter
e. SVT

920. A 75yo war veteran complains of loss of appetite and says he has lost weight over the past few
months. He says that he has passed some blood in his urine, however, he had no pain. A recent report shows
that PSA >5.5ng/ml. how will you manage this pt?
a. Radical prostatectomy
b. TURP
c. Cryosurgery
d. Brachytherapy
e. Irradiation
1st choice should be radical prostatectomy as it seems not to be beyond localy advanced disease fro its
presentation. TURP is used to relief pressure symptom which is not yet in this patient and it does not cure the
cancer. For cryosurgery (killing cancer cell by freezing them) it is not widely available and its long-term
effectiveness has not yet been conclusively proven. Irradiation- If you consider surgery don't go for irradiation
first as following irradiation surgical complications are much higher. Brachytherapy is also a kind of
radiotherapy which is given locally (by placing radioactive seeds or needles in prostate).

921. A 19yo boy comes to the ED with pain, swelling and tenderness 2cm distal to Listers tubercle of
radius. Exam: proximal pressure on the extended thumb and index finger is painful. XR: no fx. What is the next
appropriate management for the pt?
a. Immobilization with cast
b. Repeat XR
c. MRI
d. Surgery
e. None

Key : a
Clincher : swelling and tenderness 2cm distal to lister's tubercle, xray no fx

Scaphoid fracture

Only a few cases ojf scaphoid fracture are seen on xray. Treatment is to immobilize with a scaphoid cast for 8
to 10days then repeat xray if it still does not show and symptoms persist, go for a CT scan.

922. A 71yo man with a hx of 50yrs of smoking presents with cough, hemoptysis, dyspnea and chest pain.
He also has anorexia and weight loss. The dx of lung cancer has been stabilized. Which electrolyte abnormality
can be seen?
a. Hyperkalemia
b. Hypocalcemia
c. Hyponatremia
d. Hypernatremia
e. Hypomagnesemia

Key : c
Clincher : lung cancer

Syndrome of inappropriate ADH secretion (SIADH)[8][9]

Inappropriate ADH secretion from posterior pituitary or from ectopic source despite low serum osmolality.

Major diagnostic features

Hyponatraemia.Plasma hypo-osmolality proportional to hyponatraemia.Inappropriately elevated urine


osmolality (>100 mOsmol/kg) commonly > plasma osmolarity.Persistent urine [Na+] >40 mmol/L with normal
salt intake.Euvolaemia.Normal thyroid and adrenal function.Extra features include an elevated ADH level and
low blood uric acid level.

Causes (not exhaustive)

Neurological: tumour, trauma, infection, Guillain-Barr syndrome, multiple sclerosis, systemic lupus
erythematosus, intracranial haemorrhage, sinus thrombosis, AIDS, porphyria.Pulmonary: lung small-cell
cancer, mesothelioma, pneumonia, abscess, cystic fibrosis, asthma, tuberculosis, positive-pressure ventilation.
Other malignancy: oropharyngeal, stomach, pancreas, leukaemia, lymphoma, thymoma, and genitourinary
tract cancers.
Drugs: chlorpropramide, carbamazepine, selective serotonin reuptake inhibitor (SSRI) antidepressants, tricyclic
antidepressants, lithium, MDMA/ecstasy, tramadol, haloperidol, vincristine, desmopressin, fluphenazine.
Miscellaneous: idiopathic, hereditary, pain, postoperative, stress, endurance exercise and marathon runners,
dermatomal herpes zoster.

923. A 56yo man who is hypertensive recently underwent a change in meds. 2days later he developed
wheezing. Which drug can cause this?
a. Atenolol
b. Ramipril
c. Bendroflumethiazide
d. Verapamil
e. Furosemide

Key : a
Clincher : hypertensive, new meds, wheeze

Beta blockers cause bronchoconstriction leading to wheeze and SOB. it is contraindicated in asthma.

924. A 33yo man has a temp=38.5C, cough and chest pain on the right side on inspiration. He also has
purulent sputum. What is the most likely organism to cause pneumonia in this pt?
a. Gram +ve diplococci
b. Coagulase +ve cocci
c. PCP cold agglutinins
d. AFB
e. Gram ve diplococci

Key : c ....... corrected : a


Clincher : no specific pointers here

The most common cause of CAP strept.pneumonia which is a gram +ve diplococci

925. A young mans arm was caught in a machine. XR showed no fx but arm is very swollen. What is
the best tx?
a. Plaster cast
b. Wide splint with upward position
c. Analgesics
d. Antibiotics
e. Tetanus prophylaxis

b. Wide splint with upward position


Swollen arm is held high for swelling to settle.

Splint
Noncircumferential
Acute and definitive treatment of select fractures
Soft tissue injuries (sprains, tendons)
Acute management of injuries awaiting orthopedic intervention
Allows for acute swelling
Decreased risk of complications
Faster and easier application
Commercial splints available and appropriate for select injuries
May be static (preventing motion) or dynamic (functional; assisting with controlled motion)
Lack of compliance
Increased range of motion at injury site
Not useful for definitive care of unstable or potentially unstable fractures
Cast
Circumferential
Definitive management of simple, complex, unstable, or potentially unstable fractures
Severe, non acute soft tissue injuries unable to be managed with splinting
More effective immobilization
Higher risk of complications
More technically difficult to apply

926. A child was brought in to ED by his parents for taking his grand-dads meds. There is an extra
systole in the ECG. Which drug was taken?
a. Digoxin
b. Amitriptyline
c. Atenolol
d. Ramipril
e. Bendroflumethiazide
a. Digoxin
In digoxin toxicity, the finding of frequent premature ventricular beats (PVCs) is the most common and the
earliest dysrhythmia. Sinus bradycardia is also very common. In addition, depressed conduction is a
predominant feature of digoxin toxicity. Other ECG changes that suggest digoxin toxicity include bigeminal and
trigeminal rhythms, ventricular bigeminy, and bidirectional ventricular tachycardia

Features
generally unwell, lethargy, nausea & vomiting, anorexia, confusion, yellow-green vision
arrhythmias (e.g. AV block, bradycardia)
Management
Digibind
correct arrhythmias
monitor potassium

Amitriptyline
Sinus tachycardia is the most common ECG finding
Widening of the PR, QRS, and QTc intervals; nonspecific ST-segment and T-wave changes

Atenolol
bradycardia (slow heartbeat), severe hypotension with shock, acute heart failure, hypoglycemia and
bronchospastic reactions.

Ramipril
hypotension, tachycardia, hyperkalaemia, and acute renal failure.
927. A 5yo child came from Ghana 6wks ago. 2d ago he developed fever, vomiting and neck stiffness.
He had taken malaria prophylaxis and had no rash. What is the dx?
a. Cerebral abscess
b. Cerebral malaria
c. Meningococcal meningitis
d. SAH
e. Cerebral tumor
f. Pneumonia

b. Cerebral malaria
Malaria prophylaxis doesn't provide full protection against all subtypes of malarial parasites. Secondly
meningococcal meningitis is fever with a rash. Here pt has no rash.
abscess have typical swinging fever pattern.
Suspect malaria in any patient has travelled to endemic areas within one year (patient com). Antimalarial
prophylaxis don't give full immunity and infection can be happened (BNF).
a-e: focal signs
c- rash
d- sudden
f-cough

Consider malaria in every febrile patient returning from a malaria-endemic area within the last year, especially
in the previous three months, regardless of whether they have taken chemoprophylaxis, as prompt
recognition and appropriate treatment will improve prognosis and prevent deaths.

928. A HTN pt on bendroflumethiazide 2.5mg/d has come for his routine checkup. Exam:
BP=145/85mmHg. Lab: K+=5.9, Na+=137. What is the most appropriate management for this pt?
a. Stop meds
b. Continue same dose
c. Increase the dose
d. Decrease the dose
e. Repeat the blood test

e. Repeat the blood test


Repeat the test as thiazides cause hypokalemia

Common adverse effects


dehydration
postural hypotension
hyponatraemia, hypokalaemia, hypercalcaemia
gout
impaired glucose tolerance
impotence

929. A 65yo man presents with significant weight loss and complains of cough, SOB and chest pain. Exam: left
pupil constricted, drooping of left eyelid. What is the most likely dx?
a. Pancoast tumor
b. Thoracic outlet syndrome
c. Cervical rib
d. Pneumonia
e. Bronchogenic ca

a. Pancoast tumor
Classically caused by an apical (superior pulmonary sulcus) malignant neoplasm of the lung. The neoplasm is
usually bronchogenic in origin (most commonly squamous cell carcinoma, sometimes adenocarcinoma and
large-cell carcinoma).
An ipsilateral invasion of the cervical sympathetic plexus leading to Horner's syndrome (miosis,
enophthalmos, ptosis; in 14-50% of patients).
Ipsilateral reflex sympathetic dystrophy may occur.
Shoulder and arm pain (brachial plexus invasion C8-T2) leading to wasting of the intrinsic hand muscles
and paraesthesiae in the medial side of the arm.
Less commonly, unilateral recurrent laryngeal nerve palsy producing unilateral vocal cord paralysis
(hoarse voice bovine cough), and/or phrenic nerve involvement.
There may be arm oedema secondary to the compression of blood vessels.

930. A 4yo boy presents with fever, sore throat and lymphadenopathy. The dx of tonsillitis has been made. He
had 3 episodes last yr. What is the most appropriate management for this pt?
a. Tonsillectomy
b. Paracetamol/ibuprofen
c. Oral penicillin V
d. IV penicillin
e. None

b. Paracetamol/ibuprofen

Management
paracetamol or ibuprofen for pain relief
antibiotics are not routinely indicated

NICE indications for antibiotics


features of marked systemic upset secondary to the acute sore throat
unilateral peritonsillitis
a history of rheumatic fever
an increased risk from acute infection (such as a child with diabetes mellitus or immunodeficiency)
patients with acute sore throat/acute pharyngitis/acute tonsillitis when 3 or more Centor criteria are
present

The Centor criteria* are as follows:


presence of tonsillar exudate
tender anterior cervical lymphadenopathy or lymphadenitis
history of fever
absence of cough

If antibiotics are indicated then either phenoxymethylpenicillin or erythromycin (if the patient is
penicillin allergic) should be given. Either a 7 or 10 day course should be given

Complications of tonsillitis include:


otitis media
quinsy - peritonsillar abscess
rheumatic fever and glomerulonephritis very rarely

The indications for tonsillectomy are controversial. NICE recommend that surgery should be considered only if
the person meets all of the following criteria
sore throats are due to tonsillitis (i.e. not recurrent upper respiratory tract infections)
the person has five or more episodes of sore throat per year
symptoms have been occurring for at least a year
the episodes of sore throat are disabling and prevent normal functioning

931. A pt had passed a 4mm stone in his urine. He has a 3mm stone in the renal pelvis found on US.
What is the management?
a. ESWL
b. None
c. Dormier basket
d. Surgery
e. PCNL

b. None
If the stone is <5mm in lower ureter ,it will pass spontaneously..
if it is >5mm ---> medical therapy ( nifedipine ,alpha blocker ) if not passed , go for ESWL ( If < 1cm ) or dormia
basket...
PCNL ---> when stone is large ,multiple or complex..
Percutaneous nephrostomy ---> presence or infection or obstruction , to safe the kidney from reflux damage
and save the person from Sepsis ( if pus collected )

932. A 4yo boy presents with fever, severe ear ache, vomiting and anorexia. He also has mod
tonsillitis. Exam: tympanic membrane bulging. He came to the GP a few days ago and was dx
with URTI. What is the most appropriate dx?
a. OE
b. Acute OM
c. Serous otitis
d. Chronic suppurative OM
e. Mastoiditis

b. Acute OM
Complications of tonsillitis include:
otitis media
quinsy - peritonsillar abscess
rheumatic fever and glomerulonephritis very rarely
antibiotics are not routinely recommended. NICE recommends however that they should be considered in the
following situations:
children younger than 2 years with bilateral acute otitis media
children with otorrhoea who have acute otitis media

933. A 3yo girl presents with complaints of sudden right facial weakness and numbness and pain
around her ear. There are no symptoms. What is the most appropriate dx?
a. SAH
b. Bells palsy
c. Stroke
d. TIA
e. Subdural hemorrhage

b. Bells palsy

Bell's palsy may be defined as an acute, unilateral, idiopathic, facial nerve paralysis. The aetiology is unknown
although the role of the herpes simplex virus has been investigated previously. The peak incidence is 20-40
years and the condition is more common in pregnant women.

Features
lower motor neuron facial nerve palsy - forehead affected*
patients may also notice post-auricular pain (may precede paralysis), altered taste, dry eyes,
hyperacusis

Management
in the past a variety of treatment options have been proposed including no treatment, prednisolone
only and a combination of aciclovir and prednisolone
following a National Institute for Health randomised controlled trial it is now recommended that
prednisolone 1mg/kg for 10 days should be prescribed for patients within 72 hours of onset of Bell's
palsy. Adding in aciclovir gives no additional benefit
eye care is important - prescription of artificial tears and eye lubricants should be considered

934. A 6yo boy fell in the playground and has been holding his forearm complaining of pain. Exam: no sign of
deformity or swelling. However, there is minimal tenderness on exam. What is the dx?
a. Fx mid radius
b. Fx mid ulnar
c. Fx neck of humerus
d. Fx shaft of humerus
e. Green stick fx of distal radius

e. Green stick fx of distal radius


Greenstick fractures common in children

Paediatric both-bone forearm fractures


Fractures may be of greenstick type (incomplete) or complete. A greenstick fracture can occur in one bone
with a complete fracture in the other. Complete fractures may be undisplaced, minimally displaced or
overriding. Fractures of the proximal third are relatively rare. Middle third fractures account for about 18% of
both-bone fractures and distal third fractures for about 75%.[6]
Mechanism of injury: usually an indirect injury following a fall on to an outstretched hand. Occasionally
caused by a direct trauma.
Presentation: pain, swelling and deformity at the fracture site.
Investigation: X-rays of the wrist, elbow and whole forearm should be taken.
Management: unlike the management of these fractures in adults, conservative management is still
the first line of treatment for paediatric forearm fractures, especially in children less than 10 years old.
935. A 62yo man has been smoking about 15 cigarettes/day for 45yrs, and has been working as a
builder since he was 24yo. He presents with chest pain, SOB, weight loss. CXR shows bilateral
fibrosis and left side pleural effusion. What is the best inv that will lead to dx?
a. CXR
b. Pleural fluid aspiration of cytology
c. MRI
d. Pleural biopsy
e. CT

d. Pleural biopsy
Only biopsy confirms carcinoma
Ct is the next step not the best step towards the diagnosis. The best work up leading to diagnosis should b
pleural biopsy,
Asbestosis predisposing to mesothelioma and therefore pleural biopsy
Pleural fluid: straw coloured or blood stained. Cytological analysis occasionally leads to the diagnosis but a
pleural biopsy is usually required.

936. During a basketball match, one of the players suddenly collapsed to the ground with coughing and SOB.
What is the inv of choice?
a. CXR
b. CT
c. MRI
d. V/Q scan
e. CTPA

a. CXR
case of spontaneous pneumothorax...xray chest ...first
Basketball player... tall height.... more chances of apical subpleural blebs... its SPONTANEOUS
PNEUMOTHORAX.. so CXR is best

937. A 57yo man having HTN on oral anti-HTN. However, he is finding it difficult to mobilize as he
feels dizzy whenever he tries to get up. What is the most appropriate inv for him?
a. Ambulatory BP
b. ECG
c. MRI
d. CXR
e. CT

a. Ambulatory BP

Who should be referred for ambulatory blood pressure monitoring?


The National Institute for Health and Care Excellence (NICE) recommends that if a clinic blood pressure
is 140/90 mmHg or higher, ABPM should be offered to confirm the diagnosis of hypertension. If a
person is unable to tolerate ABPM, home blood pressure monitoring (HBPM) is a suitable alternative to
confirm the diagnosis of hypertension.[5]
Poorly controlled hypertension - eg, suspected drug resistance.
Patients who have developed target organ damage despite control of blood pressure.
Patients who develop hypertension during pregnancy.
High-risk patients - eg, those with diabetes mellitus, those with cerebrovascular disease, and kidney
transplant recipients.
Suspicion of white coat hypertension - high blood pressure readings in clinic which are normal at home.
Suspicion of reversed white coat hypertension, ie blood pressure readings are normal in clinic but
raised in the patient's own environment.
Postural hypotension.
Elderly patients with systolic hypertension.
938. A 33yo female complains of diplopia on upright gaze. Exam: ptosis can be seen. There are no other
complains or any significant PMH. What is the most appropriate inv for him?
a. Ophthalmoscopy
b. Visual field test
c. TFT
d. CT
e. Checking red reflex

d. CT
ptosis can be due to neurological causes of muscle weakness in this case , but there is associated diplopia so
its better to exclude any nerve lesion through ct.
3rd nerve palsy

939. A tall rugby player was hit in the chest by a player of the opponent team. He developed
breathlessness and his face went blue and purple. You have been called to look at him, how will
you manage him?
a. Insert a needle in the 2nd ICS in the mid-clavicular line
b. Insert a needle in the 5th ICS in the mid-axillary line
c. Intubate the pt
d. Start CPR
e. Give oxygen

a. Insert a needle in the 2nd ICS in the mid-clavicular line


Tension pneumothorax

940. A young woman fell and hit her knee. Exam: valgus test +ve. What ligament was most probably
injured?
a. Ant cruciate
b. Medial collateral
c. Lateral collateral
d. Post cruciate
e. Meniscus

b. Medial collateral
Medial collateral: Valgus stress test
Lateral collateral: Varus stress test
Anterior Cruciate: Anterior drawer test, Lachmans test, pivot shift test.
Posterior cruciate: Posterior drawer test, posterior sag test.

941. A 75yo man comes in complaining of difficulty in passing urine, poor stream and dribbling at the end of
voiding and anorexia. US shows bilateral hydronephrosis. What is the cause of these
findings?
a. BPH
b. Renal stones
c. Bladder stones
d. Prostatic ca
e. UTI

d. Prostatic ca
bph is common and BOO signs are present with it earlier but here the anorexia is given which points to
carcinoma

bladder outlet obstruction: hesitancy, urinary retention


haematuria, haematospermia
pain: back, perineal or testicular
digital rectal examination: asymmetrical, hard, nodular enlargement with loss of median sulcus

942. 2h after an appendectomy, a pt complains of a rapid HR and fever. He says there is also
abdominal pain and pain in the shoulder area. What is happening to this pt?
a. Intra-abdominal bleeding
b. Anastomotic leak
c. Sepsis
d. Intestinal obstruction

a. Intra-abdominal bleeding
rapid HR due to haeg, shoulder pain due to diaphragmatic irritation following intraabdominal bleeding and
fever for tissue reaction following surgery
following appendicitis, gut anastomosis not needed and gut anastomosis is needed following large or small gut
ischaemic necrosis or gangrene.
Sepsis in 2 hours very unlikely
post op h'age : primary- immediate or as a continuation of intraop bleed. reactionary : within 24hrs.
secondary: upto 10 days.

Mx: 1. fluid replacement - crystalloids upto 1000ml bolus and then maintain. 2.direct compression to control
superficial bleeding if any. 3. cross match blood. 4. emergency surgery

943. A 50yo man presents with the complaints of recurrent UTI and occasional blood in the urine.
Some unusual cells have been seen in urine on routine exam. Which os the following inv would
you like to carry out now?
a. Cystoscopy
b. Urine C&S
c. XR KUB
d. US
e. CBC

d. US
the unusual cells may indicate malignancy , which in turn can be the cause of recurrent UTI .US followed by
cystoscopy seems to be the appropriate approach.
unusual cell points towards possible malignancy! may be bladder cancer or renal cell carcinoma. So we have
two options, for bladder cystoscopy and for renal cell ca US. Before going to more invasive procedure we can
think first noninvasive procedure. So US is more logical i think. for uti we can do c/s but that is not the major
issue.

944. A 28yo drug user presents to ED collapsed and anuria. His serum K+=7.5mmol/l. CXR shows early
pulmonary edema. What is the next appropriate management for this pt?
a. Urgent hemodialysis
b. IV calcium gluconate
c. IV insulin + dextrose
d. Furosemide
e. IV 0.9% NS

b. IV calcium gluconate
cardioprotective
Stabilisation of the cardiac membrane
intravenous calcium gluconate

Short-term shift in potassium from extracellular to intracellular fluid compartment


combined insulin/dextrose infusion
nebulised salbutamol

Removal of potassium from the body


calcium resonium (orally or enema)
loop diuretics
dialysis
945. DM man feels hot, painful lump near the anal region. What is the most probable dx?
a. Anal fissure
b. Abscess
c. Hematoma
d. Wart
e. External hemorrhoids

b. Abscess
Perianal abscess: the most common (60%)
High-risk groups include those with diabetes, immunocompromised patients, people who engage in receptive
anal sex and patients with inflammatory bowel disease.
Symptoms include painful, hardened tissue in the perianal area, discharge of pus from the rectum, a lump or
nodule, tenderness at the edge of the anus, fever, constipation or pain associated with bowel movements.
Prompt surgical drainage.
Medication for pain relief.
Antibiotics are usually not necessary unless there is associated diabetes or immunosuppression.

946. A 65yo lady with T1DM for the last 20y comes with a tender lump near the anal opening. She
says she also has a fever. What tx should she get?
a. I&D + antibiotics
b. IV antibiotics
c. C&S of aspirate from swelling
d. Painkillers
e. Cautery of swelling

a. I&D + antibiotics
see q 245

947. An 80yo DM lady presents with redness and swelling over her right foot. It is tender to touch,
warm and glossy. What are the complications this pt might develop?
a. Meningitis
b. Sepsis
c. Ulcer
d. Gangrene

d. Gangrene
Cellulitis to wet gangrene
In clinical setting in diabetics it's usually cellulitis then gangrene then sepsis.

Complications of cellulitis
Complications are uncommon but may include:
Abscess formation.
Gangrene.
Thrombophlebitis/lymphangitis.
Chronic leg oedema (a late complication which may predispose to further episodes of infection).
Less common complications (occurring in <1%) include:
Necrotising fasciitis.
Osteomyelitis.
Compartment syndrome.
Acute glomerulonephritis.
Endocarditis.
Septicaemia.
Streptococcal toxic shock syndrome.

Flucloxacillin 500 mg four times daily (in adults) is usually given as first-line
Severe cellulitis should be treated with intravenous benzylpenicillin + flucloxacillin.

948. After surgery a pts left leg has become swollen and tender. The diameter of the calf has
increased and passive movements cause pain. What is the most probable dx?
a. DVT
b. Lymphedema
c. Peripheral vascular disease
d. Hematoma
e. Superficial thrombophlebitis

a. DVT
because the human body is under hypercoagulable state after any operation, and because of the diameter of
calf has increased
Its not specified the site of surgery. Surgery is a highly stressful state and also leads to immobilization both
risks for dvt. Clinically there is swelling and tenderness- moses sign. Pain on passive movements- homan sign.

If a DVT is 'likely' (2 points or more)


a proximal leg vein ultrasound scan should be carried out within 4 hours and, if the result is negative, a
D-dimer test
if a proximal leg vein ultrasound scan cannot be carried out within 4 hours a D-dimer test should be
performed and low-molecular weight heparin administered whilst waiting for the proximal leg vein
ultrasound scan (which should be performed within 24 hours)

If a DVT is 'unlikely' (1 point or less)


perform a D-dimer test and if it is positive arrange:
a proximal leg vein ultrasound scan within 4 hours
if a proximal leg vein ultrasound scan cannot be carried out within 4 hours low-molecular weight
heparin should be administered whilst waiting for the proximal leg vein ultrasound scan (which should
be performed within 24 hours)

Management

Low molecular weight heparin (LMWH) or fondaparinux should be given initially after a DVT is diagnosed.
a vitamin K antagonist (i.e. warfarin) should be given within 24 hours of the diagnosis
the LMWH or fondaparinux should be continued for at least 5 days or until the international
normalised ratio (INR) is 2.0 or above for at least 24 hours, whichever is longer, i.e. LMWH or
fondaparinux is given at the same time as warfarin until the INR is in the therapeutic range
warfarin should be continued for at least 3 months. At 3 months, NICE advise that clinicians should
'assess the risks and benefits of extending treatment'
949. 2h after an appendectomy, a pt complains of a rapid HR and fever. He says there is also
abdominal pain and pain in the shoulder area. What is the first step in the management?
a. Maintain IV access and give IV fluids
b. Start IV antibiotics
c. Insert NGT for intestinal decompression
d. Cross match blood
e. Emergency exploratory laparotomy

a. Maintain IV access and give IV fluids


Q942

950. A pregnant woman presents with knee pain on movements. The pain becomes worse at the end of the
day. Radiology shows decreased joint space. Labs: CRP=12. What is the 1st line med?
a. Paracetamol
b. NSAIDs
c. Oral steroid
d. Intra articular steroid
e. DMARDs

a. Paracetamol
Osteoarthritis... First line is paracetamol
safe in pregnancy
second-line treatment is oral NSAIDs/COX-2 inhibitors, opioids, capsaicin cream and intra-articular
corticosteroids.

951. A 68yo man presents with muscle weakness. He is not able to climb stairs. He also complains of mild
breathlessness. He says that he sometimes feels difficulty in swallowing food. Labs: ALP=216, AST=49, ALT=43,
CK=417, ESR=16. What is the most likely dx?
a. Polymyositis
b. Polymyalgia rheumatic
c. Muscular dystrophy
d. Esophageal carcinoma
e. Osteoarthritis

Ans : A
Reason : Patients with Polymyositis have affected proximal muscle groups, which presents itself with difficulty
standing up from a chair , inability to climb stairs, lifting objects and combing hair. Fatigue, myalgia, and
muscle cramps. fine muscular movements are affected late in the disease, dysphagia, no rash.
Dermatomyositis presents with similar features along with skin manifestations, blue purple heliotrope rash ,
shawl appearance ( over shoulders and back).fever, arthralgia, malaise and weight loss,tachyarrhythmias and
dilated cardiomyopathy,git ulcers and infections.
Epidemiology: Polymyositis 30 and 60 years smaller peak at about 15 years of age.
Dermatomyositis Peak age of Onset Adults 50. Peak onset age children 5-10
INVESTIGATIONS : CK, SGPT, SGOT, LDH,anti-Jo-1 antibodies in polymyositis ( Lung symptoms also present)
diagnosis is established by electromyography (EMG) and is confirmed by muscle biopsy. In polymyositis it is
the definitive test.

positive antinuclear antibody (ANA) finding is common in patients with dermatomyositis. Anti-Mi-2 antibodies
are specific for dermatomyositis.EMG and muscle biopsy can be diagnostic.
D/D. SLE, Sojrens, RA, Scleroderma

Treatment
STEP1 Steroid, STEP2 Azathioprine , STEP3 Cyclophosphamide , IV immunoglobulins for resistant
dermatomyositis
FOR LUNG DISEASE combination regimen including ciclosporin A or tacrolimus with cyclophosphamide +
steroid.

952. A 67yo builder presents with a persistent nodular lesion on upper part of pinna with some
telangiectasia around the lesion. What is the dx?
a. Basal cell
b. Squamous cell
c. Keratocanthoma
d. Actinic keratosis
e. Bowens disease

Ans: A
Reason: presentation of basal cell CA is over sun exposed areas, lesions small pearly white and raised with
telangiectasia.rodent ulcer has an indurated edge and ulcerated centre.
types: Nodular=Solitary, shiny, red nodule with large telangiectatic vessels.Commonly on the face.
Superficial=Often multiple, usually on the upper trunk and shoulders, Erythematous well-demarcated
scaly plaques.
Morphoeic=Also known as sclerosing or infiltrative BCC.Usually found in mid-facial sites.More aggressive
and have poorly defined borders
Pigmented=Brown, blue or greyish lesion.Nodular or superficial histology.Seen more often in individuals
with dark skin.
Basosquamous
Mixed BCC and squamous cell carcinoma (SCC)

Treatment : Surgery : mohs micrographic surgery


Curettage and cautery/electrodesiccation.Not recommended for recurrent, large, morphoeic tumours or
tumours on the face
Cryotherapy/cryosurgery .Cryotherapy is well established for treating small low-risk lesions, including
superficial BCCs.Histology is not available unless an incisional biopsy is taken first.
Imiquimod 5% cream:Topical Imiquimod appears to be effective in the treatment of primary small superficial
BCCs
Radiotherapy.The best indications for radiotherapy are BCC with incomplete excision, recurrent BCC, nodular
BCC of the head and neck under 2 cm and BCC with invasion of bone or cartilage

953. A 68yo pt wakes up with slurred speech and right sided weakness. CT shows cerebral infarct. What is
the most appropriate tx?
a. Aspirin
b. Alteplase
c. Warfarin
d. Clopidogrel
e. Dipyridamole

ANS: B
Reason : If a patient with ischemic stroke presents within 4.5 hours of ischemic episode thrombolysis can be
tried. Do a CT 24 hrs Post thrombolysis.after that patients are prescribed Aspirin 300 mg for 2 weeks.(
ischemic stroke is a gradual process and episodes might take place during sleep)
CI:
major infarct or hemorrhage
mild deficit
Recent Birth,Surgery,Trauma
Past CNS bleed
Seizures at presentation
platelets <100
BP >220/130

954. A 73yo man who is recovering from surgery on the left carotid artery in his neck. He has slurred
speech. On protrusion of his tongue, the tip deviated to the left. What is the single most appropriate option?
a. Accessory nerve
b. Facial nerve
c. Glossopharyngeal nerve
d. Hypoglossal nerve
e. Vagus nerve
Ans: D

Reason: Ask the patient to protrude his/her tongue and note any deviation. A fluttering motion called
fibrillation rather than fasciculation may be seen with an LMN lesion.
If the tongue deviates to one side when protruded, this suggests a hypoglossal nerve lesion. If it is an LMN
lesion, the protruded tongue will deviate towards the side of the lesion. With a UMN lesion, the tongue will
deviate away from the side of the lesion
Causes of a single XII lesion: rare. Polio,syringomyelia tuberculosis, median branch thrombosis of the
vertebral artery.

955. A 24yo woman known to be suffering from panic disorder presents to the hospital with tingling and
numbness in her fingers. ABG: pH=7.52, PCO2=2.2kPa, PO2=11kPa, Bicarb=20. What is the most likely
condition?
a. Acute metabolic alkalosis
b. Acute resp alkalosis
c. Compensated resp alkalosis
d. Compensated metabolic acidosis
e. Acute metabolic acidosis

Ans:B
Reason: Acute respiratory alkalosis due to hyperventilation which causes CO2 washout.a secondary
hypocalcemia results with shifting of serum Ca to unionized bound form.
S/S Dizziness.Perioral tingling.Weakness.Tinnitus.Palpitations.Feeling of choking or
suffocation.Wheezing.Sweating.Loss of consciousness (uncommon)
Treatment: rebreathing into paper bag (not in physical disease)

A panic attack is defined as a discrete episode of intense subjective fear, where at least four of the
characteristic symptoms, listed below, arise rapidly and peak within 10 minutes of the onset of the attack:
Attacks usually last at least 10 minutes but their duration is variable.
The symptoms must not arise as a result of alcohol or substance misuse, medical conditions or other
psychiatric disorders, in order to satisfy the diagnostic criteria
S/S
Palpitations, pounding heart or accelerated heart rate.Sweating.Trembling.Dry mouth.Feeling short of
breath.Feeling of choking.Chest pain or discomfort.Nausea or abdominal distress.Feeling dizzy, unsteady, light-
headed or faint. Derealisation .Fear of losing control or 'going crazy'.Fear of dying.Numbness or tingling
sensations.Chills or hot flushes

Treatment (In order as per Nice guidelines)


CBT
Medication : first line SSRI ( Fluoxetine, Paroxetine)
Consider clomipramine or imipramine after 12 weeks of unsuccessful treatment
Step 3 Consider Alternative treatments
Step 4. Referral to specialist Mental health institution.

956. A 65yo man on dexamethasone underwent surgery. During and after the surgery, his blood glucose was
around 17-19 mmol/l. What will you give the pt?
a. Insulin
b. Oral hypoglycemic
c. Remove dexamethasone
d. IV Saline
e. IX dextrose

957. A 61yo man who had stroke 2y ago is on aspirin. He has RA but suffers from pain and cant tolerate it.
He is taking senna for constipation. What is the best med to relieve his pain?
a. DMARDs
b. Ibuprofen
c. Co-codamol
d. Paracetamol

ANS: B
Reason: DMARDS have no role in acute RA pain control. Paracetamol and weak opiates are rarely effective , (
another contraindication is constipation). NSAIDS are good for symptom relief , recommended are Ibuprofen,
naproxen,diclofenac.
Steroids are preferred for acute relief and flares of RA

958. A young child was brought by his mother to the OPD complaining that he raised the vol of the TV and
didnt respond to her when she called him. Exam: tympanic membrane was dull greyish and no shadow of
handle of malleus. What is the most probable dx?
a. Chronic OM
b. Acute OM
c. Secretory OM
d. Otitis externa
e. Cholesteatoma

ANS: C
Reason: Otitis media with effusion (OME) is defined as inflammation of the middle ear, accompanied by the
accumulation of fluid in the middle-ear cleft, without the symptoms and signs of acute inflammation,It often
results in conductive hearing loss and is the most common cause of hearing loss and elective surgery in
childhood. In ears with documented fluid, the average hearing loss is 20 decibels (dB),but may be as high as 50
dB

S/S: Earache (not always present).Hearing loss and/or behavioural problems.Hearing loss, which may be
significant (20-30 dB), particularly if it occurs in both ears and has persisted for over one month.It can occur
without hearing loss.

Treatment: Medication not recommended , OBSERVE,


NICE recommends that children who most benefit from surgery are those with persistent bilateral OME lasting
three or more months with a hearing level in the better ear of 25-30 dB HL or worse averaged at 0.5, 1, 2 and
4 kHz

959. A 48yo woman always socially withdrawn has stopped going out of the house. She is afraid to socialize
because she fears that people will criticize her. What is the most probable dx?
a. Agoraphobia
b. PTSD
c. Social anxiety
d. OCD
e. GAD

ANS: C
Reason : Social Anxiety AKA Social Phobia, persistent fear and anxiety about one or more social or
performance situations.Social anxiety is a fear of being around people and having to interact with them.
Sufferers fear being watched and criticised. Normal activities such as working, shopping, or speaking on the
telephone are marked by persistent feelings of anxiety and self-consciousness. They feel dread as a situation
approaches and afterwards they analyse or ruminate on how they could have done better.They often
experience chronic insecurity about their relationships with others, excessive sensitivity to criticism, and
profound fears of being judged negatively, mocked, or rejected by others

Treatment: CBT ,
If patient Declines NICE recommends Consider SSRIs (SErtraline, Escitalopram), Pts who decline both consider
short-term psychodynamic psychotherapy.

960. Post gastric ulcer got perforated leading to bleeding involving the gastro-duodenal artery. Where
would fluid accumulate in the cavity?
a. Left paracolic gutter
b. Pelvic cavity
c. First part of duodenum
d. Under the diaphragm
e. Retroperitoneal
961. A 4yo boy presents with recurrent episodes of self limiting spontaneous bleeding. Coag test: PT normal,
bleeding time normal, APTT prolonged, Factor VIII decreased. His father and uncle suffer from a similar illness.
What is the most likely dx?
a. Hemophilia A
b. Hemophilia B
c. Von willebrands disease
d. ITP
e. TTP
ANS : A
Reason It is a bleeding disorder which presents with the deficiency of factor VIII, there are three forms,
Severe which usually presents in infancy ( time of circumcision or as intracranial hemorrhage )factor activity
<1%
Moderate which presents with bleeding following venepuncture (age Before 2 years) Factor Activity 1-5 %.
Mild. Only bleed after major trauma or surgery.Age >2 years, Factor Activity >5%

INvestigations : PT,BT,fibrinogen and Von willebrand Normal


APTT) - usually prolonged, Factor VIII:C - is reduced

Treatment:
Children with severe haemophilia (ONce weekly prophylaxis with FActor VIII)
Recombinant factor VIII preferred. Fresh frozen plasma and cryoprecipitate should only be used in an
emergency when the concentrates are not available
Desmopressin boosts Factor VIII activity.

962. A 53yo lady presents with hot flash and night sweats. Her LMP was last year. She had MI recently.
What is the most appropriate management for her?
a. Raloxifene
b. Estrogen
c. COCP
d. Evening primrose
e. Clonidine

Ans : Correct Answer is Clonidine


.Clonidine has cardiac uses like antihypertensive and diagnosis of pheochromocytoma (by reducing nor
epinephrine)
And non cardiac uses like post menopausal vasomotor symptoms ,opioid withdrawal,diabetic
diarrhoea,smoking cessation, and analgesia
HRT contraindicated in pts with recent MIs or ischemic episodes.

Contraindications:

pregnancy
undiagnosed abnormal vaginal bleeding
active thromboembolic disorder or acute-phase myocardial infarction
suspected or active breast or endometrial cancer
active liver disease with abnormal liver function tests
porphyria cutanea tarda
As transdermal oestrogen is associated with fewer risks than oral HRT, a transdermal route may be preferable
for many women. This route is also advantageous for women with diabetes, hypertension and other
cardiovascular risk factors, and also especially with advancing age

963. A 73yo man who was a smoker has quit smoking for the past 3yrs. He now presents with hoarseness of
voice and cough since past 3wks. XR: mass is visible in the mediastinum. What is the best inv to confirm the
dx?
a. Bronchoscopy
b. Thoracoscopy
c. US
d. CT thorax
e. LN biopsy

Ans: E
Reason : the patient seems to be affecting from Bronchogenic Carcinoma, The Key diagnosis as rccomended by
NICE
Xray
CT
Bronchoscopy with Biopsy
Biopsy of a convenient metastatic site should be performed if this is easier than biopsy of a primary site .
(Biopsy remains the best investigation to confirm the diagnosis)
NICE strongly recommends a new imaging test,18F-deoxyglucose positron emission tomography (FDG-PET), to
help stage tumours

Treatment
Small cell tumours are usually treated with 4-6 cycles of multi-drug platinum-based chemotherapy with the
possibility of added radiotherapy in limited stage disease.
Most Rapidly Growing+Worst Prognosis

Staging of the disease and the patients overall fitness and exercise ability will determine the type of
treatment chosen
Surgery , Radiotherapy, Radical Radiotherapy , RADIO+ Chemo

964. A 52yo man known DM presents to ED with sudden onset of pain in the left loin and hematuria. Inv:
8mm stone in left lower ureter. Nifedipine with steroids was prescribed as initial tx with supportive therapy.
He returned complaining of worsening pain, vomiting with passing of 2 stones. Renal function tests indicate
impending ARF. How will you manage this pt?
a. Continue same tx
b. Start alpha blocker
c. ESWL
d. Percutaneous nephrolithotomy
e. Percutaneous nephrostomy
f. Open surgery

Ans: E
Reason : Emergency Treatment with percutaneous nephrostomy and or ureteric stent insertion is necessary if
either pain or obstruction is persistent.
95% of those 2-4 mm in diameter pass spontaneously but passage may take as long as 40 days
(ESWL) - shock waves are directed over the stone to break it apart. The stone particles will then pass
spontaneously.(Acute urinary tract infection or urosepsis Contraindication for ESWL)
Percutaneous nephrolithotomy (PCNL) - used for large stones (>2 cm), staghorn calculi and also cystine stone
Open surgery reserved for multiple stone/complicated cases

965. A lady who is alcohol dependent wants to quit but wants someone to encourage her. What would you
do?
a. Medication
b. Refer to social services
c. Refer to psychology
d. CBT

ANS: B
She should be Advised to join a Support Group to Help her Quit.

966. A young girl presented to OBGYN assessment unit with lower abdominal pain and per vaginal bleeding
after a hx of hysterosalpingograph as a part of her infertility tx. Observation: BP=90/50mmHg, pulse=120bpm,
exam revealed rigid abdomen. What is the most appropriate next inv?
a. CT
b. XR erect and supine
c. US abdomen
d. Coag profile
e. CXR

ANS: C
Reason : USG abdomen will confirm the presence of blood in the abdominal cavity, infections can occur after
Hysterosalpingography along with vaginal bleeding, fever and foul smelling discharge.
TEST RISKS
allergic reaction to contrast dye
endometrial (uterine lining) or fallopian tube infection
injury to the uterus, such as perforation

967. A 21yo woman who is on COCP had to take azithromycin. What should be advised for her
contraception?
a. Using 7d condoms after antibiotics and avoid pill free break
b. Using 14d condoms after antibiotics and avoid pill free break
c. Using 7d condoms after antibiotics
d. No extra precaution
e. Using 14d condoms after antibiotics

ANS: D
REASON : NON enzyme inducing antibiotics no extra precaution necessary.
Type of medication Advice given

Non-enzyme-inducing Women should be advised that no additional contraception is required.


antibacterial.

Short course of enzyme- Women are advised to continue taking the COCP and use additional
inducing antibacterials precautions. Monophasic 21-day pills should be taken either as an extended
rifampicin or rifabutin. regimen (continue packets without a break until 3-4 days of BTB occurs, then
have 4-day pill-free interval) or a tricycling regimen (three packets without a
break then a 4-day pill-free interval). Additional contraception should be
continued for 28 days after stopping the rifampicin/rifabutin.

Long-term course of Should be advised to use an alternative, non-hormonal method where possible
enzyme-inducing (very potent enzyme inducers).
antibacterials rifampicin or
rifabutin.

Other enzyme-inducing Short course: advice is as per that above for rifampicin/rifabutin. Long course:
drugs, including women should be encouraged to use alternative methods of contraception. If,
anticonvulsants, St John's having considered alternatives, they still choose the COCP, the patient should
wort, etc. be advised of the increased risk of pregnancy. Should use a preparation
containing at least 50 micrograms of oestrogen. Tricycling or extended regimens
as above should be used. If BTB occurs on 50 micrograms, the dose should be
increased to a maximum of 70 micrograms.

Lamotrigine. Women should be advised not to take lamotrigine with the COCP and should
seek another form of contraception (unless also taking a non-enzyme-inducing
anticonvulsant such as sodium valproate).

Antiretroviral therapies. Those women on ritonavir-boosted protease inhibitors should be advised to use
alternative methods of contraception.

Ulipristal acetate. Women should use additional contraceptive precautions for 14 days after taking
ulipristal acetate as ellaOne for emergency contraception (16 days for Qlaira).
Those taking ulipristal in a higher dose as Esmya for fibroids should not be
advised to use alternative contraception.

968. A 60yo woman presented with radial fx and had a colles fx and supracondylar fx in the past. What inv
is req to detect her possibility of having the same prb later?
a. Dexa scan
b. MRI
c. Nuclear bone scan
d. CT
e. Bone biopsy

ANS: A
Reason: Increasing Age is associated with osteoporosis and increasing incidence of fractures, DEXA scan
measures the density of the bone and chances of bone fracture.
969. A 43yo woman presents with low mood, loss of libido, sleep disturbance, tiredness, palpitation, chest
discomfort, irritability and recurrent worries. What is the most likely dx?
a. Seasonal Affective Disorder
b. Mod depression
c. Dysthymia
d. GAD
e. Bipolar disorder

And: moderate depression


Typical features of depression , season affective disorder mostly affects during specific time of the year mostly
in winters ,patient tend to be low ,loss of interest and increased somnolence.
dysthymia is a serious state of chronic depression, which persists for at least two years

970. Which of the following is true for tamoxifen?


a. Increased incidence of endometrial carcinoma
b. Increased risk of breast ca
c. Increased risk of osteoporosis
d. Increased risk of ovarian ca

Ans: A
Tamoxifen is associated with increased risk of endometrial cancer .

971. A 45yo male complains of tremors in hands. Exam: tremors are absent at rest but present when arms
are held outstretched and persist on movement. What is the most probable dx?
a. Parkinsonism
b. Benign essential tremor
c. Cerebellar disease
d. Liver failure
e. Stroke

Ans: B
Reason . the patient is suffering from postural tremors , no tremors at rest ,present on persisted posture and
movement . Causes : Benign Essential Tremor

Rest tremor: abolished on voluntary movement. Cause: parkinsonism.


Intention tremor: irregular, large-amplitude, worse at the end of purposeful acts, eg finger-pointing or using a
remote control. Cause: cerebellar damage (eg MS, stroke). Postural tremor: absent at rest, present on
maintained posture (arms out-stretched) and may persist (but is not worse) on movement. Causes: Benign
essential tremor (autosomal dominant; improves with alcohol), thyrotoxicosis, anxiety, -agonists.

972. Pregnant lady had her antenatal screening for HIV and Hep B. what more antenatal inf should she be
screened for?
a. Rubella and syphilis
b. Toxoplasma and rubella
c. Syphilis toxoplasma
d. Hep C & E
e. Hep A & C

ANs : A
Reason: Routine ANtenatal INfectious Screen .. Rubella, Syphilis, HIV, HEP B
Hep C offered if Patient thinks she may be infected.

973. A young man has been found in the park, drunk and brought to the ED by ambulance. He recently lost
his job and got divorced. He thinks nurses are plotting against him. What is the most likely dx?
a. Schizoid personality
b. Borderline personality
c. Schizophrenia
d. Psychotic depression
e. Paranoid personality

Ans: C
Reason : delusional, history of unstable job and family life.

974. An elderly man who used to work in the shipyard industry presented with cough and SOB few weeks to
months. He was given salbutamol nebulization and antibiotics and admitted to the ward. He died 3d later. CT:
patchy infiltrates, pleural thickening and pleural effusion. Why is this a coroners case?
a. Pt got wrong dx or management
b. Pt died soon after admission
c. Death could be due to occupational illness

Ans: C
Reason: history suggests this could be a case of asbestosis.

975. A 26yo lady came with abdominal pain, vaginal discharge and low grade fever. What is the most likely
dx?
a. HELLP syndrome
b. Acute PID
c. Ectopic pregnancy
d. Appendicitis

Ans: B
Reason : typical cilinical picture of PID.

S/S lower abdominal pain, deep dyspareunia , abnormal vaginal or cervical discharge often purulent .lower
abdominal tenderness, fever.

Diagnosis : testing for gonorrhea and chlamydia in lower genital tract.


Elevated ESR ,C reactive protein

D/D .ectopic pregnancy , appendicitis


Management : broad spectrum antibiotics to cover gonorrhea and chlamydia.
Cefoxitin/ceftriaxone followed by doxycline showed good outcome .
Oral ofloxacin 400 mg bd + mteronodalzole 400 mg bd 14 days
976. A new screening test has been devised to detect early stages of prostate ca. However, the test tends to
dx a lot of ppl with no cancer, although they do have cancer as dx by other standard tests. What is this flaw
called?
a. False +ve
b. True +ve
c. False ve
d. True ve
e. Poor specificity
ans: C
Reason : diagnosing disease positive patients as no disease present known as false negative .

977. A 26yo political refugee has sought asylum in the UK and complains of poor conc. He keeps getting
thoughts of his family whom he saw killed in a political coup. He is unable to sleep and feels hopeless about his
survival. Because of this he is afraid to go out. What is the most likely dx?
a. Acute stress disorder
b. PTSD
c. Social phobia
d. OCD
e. GAD

Ans: B
Reason : post traumatic stress disorder is delayed or prolonged response to stressful situation. E.g
Sexual abuse, War, Road traffic accident, human disaster .

978. A 2yo boy presented with gradual swelling of feet and poor feeding. He has gained weight and has
dark urine. What is the single most appropriate inv?
a. Serum albumin
b. 24h urinary protein
c. Serum calcium
d. BUE
e. Serum glucose

Ans: the clinical features correlate with loss of protein , it could be due to any underlying disease presenting
itself as nephrotic syndrome .

979. A 26yo lady presents with high fever, lower abdominal pain and purulent vaginal discharge. She looks
very unwell. What is the most appropriate management?
a. Tetracycline 250mg QD
b. Doxycycline 100mg BD and metronidazole 400mg BD
c. IV Ceftriaxone 2g with doxycycline 100mg
d. IV ceftriaxone 2g with doxycycline 500mg
e. Ofloxacin 400mg BD and metronidazole 400mg BD

Ans: C
Reason : patient seems to be suffering from acute PID and the recommended management is broad spectrum
antibiotics therapy to cover chlamydia and gonorrhea.
First line in inpatient patients severe infection is Iv Ceftriaxone 2g + doxycline 100 mg bd.
980. A 39wk pregnant woman came to labor suite 3d after an obstructed labour presents with pain and
swelling of one leg. Exam: leg has blue mottling and is cold. What is the dx?
a. DVT
b. Post phlebitis syndrome
c. Embolus
d. Varicose vein
e. Herpes gladiatorum

Ans:C
Reason : clinical features of limb ischemia suggestive of thromboembolic phenomena ,

S/S The affected part becomes pale, pulseless, painful, paralysed, paraesthetic and 'perishing with cold' ('the 6
Ps').[1]
The onset of fixed mottling of the skin implies irreversible changes

INV: Hand held Doppler , Blood Tests , identify Source ( ECG,Echo,Aortic USG, Popliteal femoral artery USG)

Treatment :- emergency , urgent heparinization

If occlusion embolic : embolectomy with Fogarty catheter , bypass graft ( post op heparinization required)

If occlusion thrombotic : I) intra arterial thrombolysis II) Angioplasty III) bypass surgery

After thrombolysis or amputation if ischemia progressed treat underlying cause .

981. An 8yo boy has his tonsils and adenoids removed. On the 7th post-op day, he comes back to the ED
with hemoptysis and fever. What is the most appropriate management?
a. Admit for IV antibiotics
b. Prescribe oral antibiotics and discharge
c. Packing
d. Surgery
e. Reassurance
Ans A
Hemorrhage between 1 to 2 weeks post op is called secondary hemorrhage and is due to infection , depending
on the severity of the condition the patient should be admitted and managed on iv antibiotics .

982. A 50yo female had swelling in her ankles. She is a known alcoholic. Now she presented with
breathlessness and palpitations. What is the most likely cause of her condition?
a. VT
b. SVT
c. A-flutter
d. A-fib
e. V-ectopics
Ans D
Holiday Heart Syndrome : Holiday heart syndrome is an irregular heartbeat pattern presented in individuals
who are otherwise healthy. Coined in 1978.the term is defined as arrhythmias sometimes following excessive
alcohol consumption; usually temporary"
Holiday heart syndrome can be the result of stress, dehydration, and drinking alcohol. It is sometimes
associated with binge drinking common during the holiday season
Most Common Arryhtmias : Afib , Aflutter sometimes SVts.
Increased alcohol promotes alcoholic cardiomyopathy, heart failure and arrhythmias.
S/S Alcoholic Cardiomyopathy
shortness of breath
swelling of the legs
rapid and irregular heartbeat
rapid and irregular pulse
fatigue, weakness, dizziness, fainting
an enlarged liver
cough that produces a frothy, pink mucus

983. A young boy has acute scrotal pain for a few hours. Exam: one testis is very painful to touch. He had
this kind of pain before but it was mild and resolved itself within 30mins. What would you do next?
a. Urgent exploration
b. US
c. Antibiotics
d. IV fluids
e. Doppler US
Ans: A
Reason Testicular Torsion , usually affects unilaterally.typically sudden, severe pain in one testis.abdominal
pain , often comes during sports or physical injury, history of previous, brief episodes of similar pain
(presumably a torsion that corrected itself ) nausea vomiting,

Diagnosis:
most important investigation is ultrasound integrated with colour Doppler. A very significant finding is the
detection of presence/absence of intratesticular blood flow for the early identification of testicular torsion.
An acute scrotum in a child requires surgical exploration for a definitive diagnosis

Management : It may be possible to reduce torsion manually , But the Prefered option is always exploration.

984. An 8wk pregnant woman presents with persistent vomiting and weight loss. Exam: HR=110bpm.
Dehydration was corrected with NS infusion and K+. The condition didnt improve so IM cyclizine was given.
She is still vomiting. What is the next appropriate management?
a. IV fluids
b. IV antiemetics
c. IV steroids
d. Terminate pregnancy
e. Thiamine

Ans: C
Hyperemesis Gravidarum
intractable vomiting associated with weight loss of more than 5% of pre-pregnancy weight, dehydration,
electrolyte imbalances, ketosis, and the need for admission to hospital. Usually Occurs before 12 weeks of
gestation.

Treatment:
Fluid Resuscitation with potassium replacement should be done
Vitamins should be routinely given , thiamine , pyridoxine
Nutritional support
thromboprophylaxis
Antiemetics to control vomiting
Corticosteroids to control intractable cases

985. A 28yo lady presents with dyspareunia and dysmenorrhea. She is very obese. She now wants
reversible contraceptive method. Which of the following will be most suitable for her?
a. Minera
b. COCP
c. POP
d. Copper T
e. Barrier method

Ans: A
Mirena preferred in Obese,diabetes,epilepsy,migraine and in women with contraindication to
oestrogens.Reduced menstrual loss and Dysmenorrhea , rapid return of fertility with removal.
Other Users: Menorrhagia and HRT (prevention of endometrial hyperplasia during oestrogen therapy)

986. A young lady who is 28wks pregnant presents with vaginal bleeding. She has lost about 200 ml of
blood. Exam: uterus is tender. Resuscitation has been done. What is the most imp inv to establish the dx?
a. US
b. CT
c. D-dimer
d. Clotting profile
e. None

Ans: A
USG abdomen can easily visualize underlying cause of bleeding. To see fetal status and uterus.

987. A 14yo girl presents with primary amenorrhea and a short stature. What is the most likely dx?
a. Downs syndrome
b. Klinefeltners syndrome
c. Turners syndrome
d. Fragile X syndrome
e. Normal finding

ANs: C
most likely it is Turner, which is associated with impaired pubertal growth spurt and ovarian dysgenesis.
D/D Constitutional Growth failure
988. A 32yo woman wants reversible form of contraception. She has one child delivered by emergency C-
section. She also suffers from migraine and heavy periods. What is the most suitable form of contraception for
this lady?
a. COCP
b. Mini pill
c. IUCD
d. Barrier method
e. Abstinence

Ans: C
Reason : Mirena preferred in Obese,diabetes,epilepsy,migraine and in women with contraindication to
oestrogens.Reduced menstrual loss and Dysmenorrhea , rapid return of fertility with removal.
Other Users: Menorrhagia and HRT (prevention of endometrial hyperplasia during oestrogen therapy)

989. A 45yo known hypertensive man presents with hematuria, proteinuria and edema. What is the
definitive dx test for him?
a. Urine protein
b. Renal biopsy
c. Renal function test
d. Urine microscopy
e. Serum protein

Ans: B
Pt seems to be suffering from nephritic syndrome , the definitive diagnosis can be made on renal biopsy.

990. A 47yo man presents with proteinuria, BP=160/95 mmHg, small kidneys that have smooth renal pelvis.
What is the most probable dx?
a. GN
b. Chronic pyelonephritis
c. Unilateral renal artery stenosis
d. Multiple myeloma
e. ARF

ANS : A
Reason : GN is a common cause of CKD, presenting with proteinuria, hypertension and small kidneys.

991. You are the HO in the hospital and the lab report of a pt shows glucose=4mmol/l, K+=5.2mmol/l,
Na+129mmol/l. what is the most appropriate management?
a. NS 0.9%
b. NS 0.45%
c. NS 0.9% and insulin
d. Insulin
e. Dextrose

ANS: A
Reason: The immediate concern is correction of hyponatremia.
992. A 27yo man presents with abdominal pain. He says his urine is dark. Exam: BP=160/105mmHg. What is
the most appropriate inv?
a. US
b. Renal biopsy
c. CT
d. Urine protein
e. Urine microscopy

Ans:A
USG will confirm
Abdominal pain, hematuria and hypertension are classic feature of autosomal dominant polycystic kidney
disease. The disease process usually begins before the age of 30 yrs and renal failure are evident at about 60
yrs of age

993. A 12m child with AIDS is due for his MMR vaccination. What is the single most appropriate action?
a. Defer immunization for 2wks
b. Dont give vaccine
c. Give half dose of vaccine
d. Give paracetamol with future doses of the same vaccine
e. Proceed with standard immunization schedule

ANS: B
Reason: Measles, mumps and rubella (MMR) vaccine is a freeze-dried preparation containing live attenuated
measles, mumps and rubella viruses
HIV-positive individuals. Severely immunocompromised patients should not be given the vaccine but it is
indicated for patients with mild-to-moderate immunosuppression. The degree of immunosuppression is
estimated using the patient's age and CD4 count

994. A young man presents with sudden, severe pain and swelling in the scrotum. Exam: one testis seems
higher than the other. What is the most probable dx?
a. Varicocele
b. Hematocele
c. Testicular tumor
d. Epidiymo-orchitis
e. Testicular torsion

ANS: E
Reason:
History: Testicular Torsion , usually affects unilaterally.typically sudden, severe pain in one testis.abdominal
pain , often comes during sports or physical injury, history of previous, brief episodes of similar pain
(presumably a torsion that corrected itself ) nausea vomiting
EXAM:usually reddening of the scrotal skin.There is a swollen, tender testis retracted upwards.Lifting the testis
up over the symphysis increases pain, whereas in epididymitis this .usually relieves pain.testes on both sides
are characteristically in the 'bell-clapper position' with a horizontal long axis.
995. A 24yo male involved in RTA with XR: fx neck of humerus. What is the single most associated nerve
injury?
a. Axillary nerve
b. Radial nerve
c. Median nerve
d. Ulnar nerve

ANS:A
Most common nerve injury associated with Humerus neck fracture is Axillary Nerve injury.

996. A 64yo man complains of increasing SOB and cough for the past 18m. He coughs up a Tbsp of
mucopurulent sputum with occasional specks of blood. What is the most likely underlying cause?
a. Acute bronchitis
b. Bronchiectasis
c. Chronic bronchitis
d. Lung cancer
e. Pneumonia

ANS: B

It is usually accompanied by a chronic cough, mucopurulent sputum production and recurrent infections
presentation:daily expectoration of large volumes of purulent sputum.
symptoms including dyspnoea, chest pain and haemoptysis

suspect bronchiectasis ,presents with Persistent productive cough, especially if any one of the following:
Young age at presentation.History of symptoms over many years.Absence of smoking history.Daily
expectoration of large volumes of very purulent sputum.Haemoptysis.Sputum colonisation with P. aeruginosa

Diagnosis: CXR normal or show ring or tubular opacities, tramlines and fluid levels.
HRCT gold standard for diagnosis
Sputum Exam

Management: Physiotherapy
Antibiotic therapy in Acute Exacerbation. Amoxicillin, Clarithromycin, Ciprofloxacin in P.aeruginosa.
Long term Antibiotic : pts having three or more exacerbations per year.
NOT RECOMMENDED Corticosteroids - inhaled or oral - unless there is co-existent asthma.
Mucolytics.Leukotriene receptor antagonists.

Surgery:Lung resection surgery may be considered in patients with localised disease in whom symptoms are
not controlled by medical treatment

997. A 55yo man who is hypertensive suddenly lost his vision. The retina is pale and fovea appears as a
bright cherry red spot. What is the single most appropriate tx?
a. Pan retinal photocoagulation
b. Corticosteroids
c. Scleral buckling
d. Surgical extraction of lens
e. Pressure over eyeball

ANSS: E
Reason: Presentaion in Central Retinal Artery occlusion is painless loss of vision unilaterally over a few
seconds.Can be a HX of Amaurosis Fugax.
Exam:afferent pupillary defect, a pale retina with attenuation of the vessels. segmentation of the blood
column in the arteries ('cattle-trucking') and the centre of the macula (supplied by the intact underlying
choroid) stands out as a cherry-red spot

Management: Presentation within 90-100 min Occular massage can be tried.


Paracentesis and acetazolamide to reduce intraocular pressure
Sublingual isosorbide dinitrate.
Oral pentoxyphylline

998. A 32yo man with schizophrenia and a hx of violence and distressing auditory hallucinations was
admitted to the ward with aggressive behavior and has already smashed his room. He is refusing any oral
meds. What is the single most appropriate injection?
a. Flupenthixol
b. Fluphenazine
c. Haloperidol
d. Paraldehyde
e. Risperidone

ANS: C
Haloperidol indicated acute psychosis and violent settings,hyperactivity aggression,hyperactive delirium,
Haloperidol can be used to treat acute psychosis and has proven efficacy for agitation.benzodiazepines can
decrease agitation and have efficacy similar to Haloperidol but cause more sedation , Benzodiazepines other
than lorazepam and midazolam should not be administered IM because of erratic absorption.
Ziprasidone, 20 mg IM, is well tolerated and has been shown to be effective in decreasing acute agitation
symptoms in patients with psychotic disorders. Olanzapine is as effective as haloperidol in decreasing agitation
in patients with schizophrenia, with lower rates of EPS.Both IM ziprasidone and olanzapine have a relatively
rapid onset of action (within 30 minutes), which makes them reasonable choices in the acute setting.
Olanzapine has a long half-life (21 to 50 hours); therefore, patients comorbid medical conditions, such as
cardiac abnormalities or hypotension, must be considered.If parenteral medication is required, IM olanzapine
or IM ziprasidone is recommended. IM haloperidol with a benzodiazepine also can be considered.

999. A 65yo man complains of hematuria, frequency, hesistancy and nocturia. He reports that on certain
occasions he finds it difficult to control the urge to pass urine. Urine microscopy confirms the presence of
blood but no other features. What is the most porbable dx?
a. BPH
b. Bladder ca
c. Prostatic ca
d. Pyelonephritis
e. Prostatitis

ANS: C
Reason: Prostatic CA, presents with symptoms of LUTS initially and in locally invasive disease there can be
hematuria,dysuria,incontinence.

1000. A 60yo man presents with mass in the groin. Exam: mass lies below the midpoint of the inguinal
ligament and is pulsatile. What is the most probable dx?
a. Direct inguinal hernia
b. Saphenavarix
c. Femoral hernia
d. Irreducible hernia
e. Femoral aneurysm

ANS: E
Femoral aneurysms:
These are the second most common peripheral aneurysm.Patients present with local pressure symptoms,
thrombosis, or distal embolisation.A pulsatile mass can be felt in the groin

1001. An 82yo man has woken up with incoherent speech and difficulty in finding the right words. Exam:
otherwise normal, good comprehension. Which anatomical site is most likely to be affected?
a. Brocas area
b. Wernickes area
c. Midbrain
d. Parietal cortex
e. Pons

ANS: A
Reason: Brocas area is associated with motor part of speech, incoherent speech, though the patient is able to
understand speech.
Wernicke area is associated with sensory part of speech ,it affects understanding and as well as speech
production

1002. A 25yo woman has a recent cough, hoarseness and swelling in the neck. There are several non-tender
swellings on both sides of her neck. She has lost 13kgs. She takes recreational drugs. What is the most
probable dx?
a. Thyrotoxicosis
b. Hyperthyroidism
c. Vocal cord nodules
d. Carcinoma bronchus
e. TB

ANS: TB
History and clinical picture suggests pulomonary TB spread locally and distantly into lymph nodes.( non tender
matted rubbery lymph nodes).Wt loss cough. Immunocompromised , HIV, drug abusers are at increased risk of
developing

active TB.
Treatment Active Respiratory TB: 2month initial phase Isoniazid+Rifampicin+erhambutol+pyrizinamide
4month continuition with Isoniazid+rifamipicin
Meningeal TB treated for 12 months : 2months Isoniazid+Rifampicin+erhambutol+pyrizinamide
10 months Isoniazid+rifamipicin
Glucocorticoid with gradual withdrawal.

LTBI: 3month treatment with rifampicin + isoniazid

1003. A 30yo woman presents with acute headache. She complains of seeing halos especially at night. What is
the single most likely defect?
a. Paracentral scotoma
b. Monocular field loss
c. Tunnel vision
d. Central scotoma
e. Cortical blindness
This is glaucoma
paracentral scotoma first, f/b a SEIDEL'S scotoma, f/b an arcuate and a double arcuate scotoma and finally a
tunnel vision leading to blindness.

1004. A 35yo man presents with a headache that worsens on bending his head forward. What is the most
likely dx?
a. Chronic sinusitis
b. SAH
c. Migraine
d. Cluster headache
e. Tension headache

Ans : A
Reason : None of the other type of headaches have any effect on bending.

1005. A 20yo man presents with painful swallowing. Exam: trismus and unilateral enlargement of his tonsils.
The peritonsillar region is red, inflamed and swollen. What is the most appropriate tx?
a. Oral antibiotics
b. IV antibiotics and analgesics
c. I&D with antibiotics
d. Analgesics with antipyretics
e. Tonsillectomy

Ans:C/E
( Both Answers considered Equally Acceptable , But tonsillectomy should offer better treatment)

Reason: The Patient Seems to be suffering from peritonsillar abscess

IV fluids and IV antibiotics along with analgesics should be prescribed.Medical treatment alone is no longer
considered sufficient,Needle aspiration, incision and drainage and quinsy tonsillectomy are all considered
acceptable for the surgical management of acute peritonsillar abscess.Some surgeons advocate acute
(immediate) tonsillectomy as a treatment for peritonsillar abscess.
1006. A 40yo manual worker presents with a swelling in the groin. Exam: mass is found to be just above and
lateral to the pubic tubercle. It is reducible. On applying pressure on the internal ring there is no cough
impulse seen. What is the most probable dx?
a. Direct inguinal hernia
b. Indirect inguinal hernia
c. Femoral hernia
d. Strangulated hernia
e. Femoral aneurysm

Ans: B
Ring occlusion done to confirm it

1007. A 34yo male presents with headache and vomiting. Exam: temp=38.5C, neck stiffness, discharge from
left ear and right sided hyperreflexia with an extensor plantar response. What is the most likely dx?
a. Cerebral tumor
b. Meningitis
c. Cerebellar tumor
d. Cerebral abscess
e. Normal pressure hydrocephalus

Ans: D
Reason:
unilateral (Right Sided)hyperreflexia and extensor plantar response indicates upper motor type palsy, due to
space occupying lesion( tumour/abscess) on the left side. Headache & vomiting indicates raised intracranial
pressure but raised temperature and neck stiffness favours abscess/ meningitis. in meningitis the
aforementioned unilateral focal signs (hyperreflexia) usually absent. left sided ear discharge indicates origin of
abscess from left middle ear. so roughly all symptoms indicates Left sided brain abscess

S/S Fever, headache, and neurological problems, while classic, only occur in 20% of people.
The symptoms of brain abscess are caused by a combination of increased intracranial pressure due to a space-
occupying lesion (headache, vomiting, confusion, coma), infection (fever, fatigue etc.) and focal neurologic
brain tissue damage (hemiparesis, aphasia etc.). The most frequent presenting symptoms are headache,
drowsiness, confusion,seizures, hemiparesis or speech difficulties together with fever with a rapidly
progressive course. The symptoms and findings depend largely on the specific location of the abscess in the
brain
Diagnosis: Ct brain , shows space occupying lesion after a few days there is ring enhancement .

Management: IV antibiotics , sensitivity should be done . hyperbaric oxygen therapy .

1008. A 26yo male presents with speech difficulties. Exam: nystagmus. Which anatomical site is most likely to
be affected?
a. Midbrain
b. Pons
c. Cerebellum
d. Cerebrum
e. Vestibule cochlear nerve

ANS: C
Scanning speech , nystagmus, ataxia, dysdiadochokinesia, tremors,past pointing.loss of balance.

1009. A 75yo man presents with Bells palsy. His PMH is significant for late onset asthma and heart failure. He
also reports to have consulted his GP for generalized rash prv. CXR: multiple soft shadows and CBC:
eosinophilia. What is the single most likely positive antibody?
a. P ANCA
b. C ANCA
c. Anti Ro
d. Anti DS DNA
e. Anti centromere

Ans: A
Reason: Churg Straus Syndrome
A rare diffuse vasculitic disease affecting coronary, pulmonary, cerebral, abdominal visceral and skin
circulations. The vasculitis affects small- and medium-sized arteries and veins and is associated with asthma.

Presentation: Allergic rhinitis, Pulmonary Symp(ASthmna, granulomatous infiltrates). Cardiac S/S (cardiac
failure, myocarditis). Skin Manfestations ( Purpura , nodules, livedo reticularis). Glomerulonephritis,
Peripheral neuropathy,myositis.Bowel Ischemia,bleeding,perforation.

Investigations:
Antineutrophil cytoplasmic antibodies (ANCA): 70% of patients are perinuclear staining (p-ANCA) positive (anti
myeloperoxidase antibodies).
eosinophilia and anaemia on the FBC; elevated ESR and CRP; elevated serum creatinine; increased serum IgE
levels, hypergammaglobulinemia; proteinuria, microscopic haematuria and red blood cell casts in the
urine.Chest X-ray: pulmonary opacities, transient pulmonary infiltrates, pleural effusions.

Treatment: High dose steroids usually enough. Cyclophosphamide and azathioprine in severe cases.
IV immune globulins +Plasma exchange

Overall prognosis good

1010. A 50yo man complains of visual prbs and dull pain in the left eye. Fundoscopy reveals papilloedema. He
was dx with MS 2yrs ago. There is no consensual light reflex of the right eye. What is the single most likely
defect?
a. Paracentral scotoma
b. Mono-ocular field loss
c. Homonymous upper quadrantanopia
d. Central scotoma
e. Homonymous lower quadrantanopia

ANS: B
Reason : As the Pt wa diagnosed with M.S, M.S mostly affects vision unilaterally, with optic
neuritis,papilloedema ,painful eye , decreased vision, blindness or hemianopia.thus loss of consensual light
reflex in the opposite eye.

1011. A 54yo pt wakes up with right sided weakness. His current medication is bendroflumethiazide for HTN.
Pulse=92bpm, BP=160/90mmHg. CT shows left cerebral infarct. What is the most appropriate tx?
a. Alteplase
b. Aspirin
c. Clopidogrel
d. Dipyridamole
e. Simvastatin

Ans: A
Reason:Reason : If a patient with ischemic stroke presents within 4.5 hours of ischemic episode thrombolysis
can be tried. Do a CT 24 hrs Post thrombolysis.after that patients are prescribed Aspirin 300 mg for 2 weeks.(
ischemic stroke is a gradual process and episodes might take place during sleep)
CI:
major infarct or hemorrhage
mild deficit
Recent Birth,Surgery,Trauma
Past CNS bleed
Seizures at presentation
platelets <100
BP >220/130

1012. A 33yo man presented to the GP with hx of headaches and photophobia. The GP examines him and
finds a rash and is now ringing you at the hospital for advice. What would you advise the GP?
a. Send pt home
b. Start IV benzylpenicillin
c. Conduct LP
d. Start IV ceftriaxone

ANS: B
Reason: Headache,photophobia and skin rash points towards developing Meningococcal meningitis due to
N.Meningitidis, the patient should be immediately started on Iv Benzylpenicillin.

1013. An 89yo pt has lung cancer. His Na+=122mmol/l. What is the tx for this?
a. Demeclocycline
b. Vasopressin
c. Restrict fluids
d. Reassure

ANS C

Common paraneoplastic syndrome due to small cell lung cancer is SIADH,


Treatment: Always treat the underlying cause

Hyponatremia is generally defined as a serum sodium level of less than 135 mEq/L and is considered severe
when the serum sodium level is below 125 mEq/L
Midl hyponatremia present with suble unrecognized symptoms (131 mmol or above ) change of gait,posture .
Neurological S/S develop below 115 mmol/L , seizures ,coma.
The development of S/S also depends upon how fast or severe the sodium change is , sometimes in chronic
settings pts are even able to handle severe diminished sodium levels due to adaptation .

In asymptomatic Chronic setting : Fluid Restriction is firstline, If it is not sufficient move towards drugs,
DEMECLOCYCLINE (potent inhibitor of vasopressin)

In acute not so severe


Hypovolemic : 0.9%/saline replacement .
Normovolemic: Fluid restrict (500-1000 ml/day) if inadequate consider NaCl tablets or 3% saline .
Furosemide,demeclocycline,vaptans

In Acute (Symptomatic Patients) emergency : 3% NS should be given, the target should be to increase serum
NA <4-6 mmol/L over 1-2 hours before more gradual correction. 0.5-1 meq Na/hr and 10-12meq/24 hrs.

1014. A 25yo woman who is 11wks pregnant had central abdominal pain for 36h. The pain is now colicky.
There is no vaginal bleeding. She has vomited once and has had an episode of loose motion. She looks ill,
temp=37.8C and there is rebound tenderness in the RIF. What is the most probable dx?
a. Salpingitis
b. PID
c. Appendicitis
d. Ovarian torsion
e. Uterine fibroid

Ans: C
increased temperature, vomiting nausea, anorexia,pt usually constipated but may have diarrhea,central
abdominal pain which moves after few hours to RIF,and rebound tenderness in RIF point towards appendicitis.

Alvardo Scoring
Symptoms: migratory RIF pain (scores 1), nausea or vomiting (1), anorexia (1).
Signs: tenderness in the RIF (2), rebound tenderness in the RIF (1), elevated temperature (1).
Laboratory findings: leukocytosis (2), shift to the left of neutrophils (1).
From a total possible score of 10, one study recommended further investigation with CT scan for a
score of 4-6, and consideration of appendicectomy for scores of 7 or above

1015. A 42yo man presents with stroke. He is not able to walk straight and his speech is slurred. What is the
initial appropriate inv?
a. CT brain
b. PET brain
c. MRI brain
d. Carotid angiography
e. Monitor for 24h

ANS: C
Reason : Imaging for Cerebellar stroke
When neuroimaging is indicated, diffusion-weighted magnetic resonance imaging (MRI) with magnetic
resonance angiography is currently considered the optimal study. For hemorrhagic strokes, computed
tomography (CT) and MRI are both excellent studies. However, for ischemic strokes, MRI is clearly superior
with an 83% sensitivity compared to 26% for CT. Physicians should therefore not rely on CT scanning to rule
out cerebellar infarction

1016. A 24yo woman has severe depression 3m after the birth of her first child. She is breastfeeding but is
otherwise unable to look after the baby and is convinced that her family is likely to kill her. She has no interest
in anything and keeps crying. What is the most appropriate tx?
a. Fluoxetine
b. Citalopram
c. CBT
d. ECT
e. Haloperidol

Ans: D
The Patient is suffering from Postpartum psychosis ,Postpartum psychosis is a severe mental illness which
develops acutely in the early postnatal period. It is a psychiatric emergency,
Symptoms may be depressive in nature (withdrawal, confusion, loss of competence, distraction, catatonia) or
manic (elation, lability, agitation, rambling). There may be delusions (paranoia, jealousy, persecution,
grandiosity). There may be hallucinations which may be auditory, visual, olfactory or tactile. There may be odd
beliefs about the baby.Health professionals must take into account the needs of the family and new baby, as
well as the risks of medication whilst breast-feeding

Management is Primarily Pharmacological , Using same guidance as for other psychosis. Antipsychotics and
Mood stabilising drugs. ECT is also considered in some cases
As this patient is breast feeding ECT should be considrerd as there is risk of drug secretion in breast milk.

1017. A 20yo woman with no prv hx of ear complains, presents with 1d hx of severe pain in the right ear which
is extremely tender to examine. What is the single most likely dx?
a. Chondromalasia
b. Furuncle
c. Myringitis
d. OE
e. OM

Ans: D
Reason : Acute Otitis Externa
his produces a similar temperature and lymphadenopathy. Swelling is more diffuse and pain is variable with
possible pruritus. Moving the ear or jaw is painful. The canal, external ear, or both, are red, swollen, or
eczematous, with shedding of the scaly skin. There may be little,but thick, discharge in the acute stage but it
can become bloody if chronic. Hearing is often impaired
Management:
clean ear canal, keep it dry,
Acetic acid 2% ear drops can be very effective against both bacterial and fungal infection
Antibiotic drops may be used if infection is present. These are often given in combination with steroids

1018. A couple has just finished their detox regime and wants a drug with a pharmacological action to serve as
a deterrent when they take alcohol. What drug is the appropriate choice?
a. Disulfiram
b. Acamprosate
c. Vitamin supplement
d. Naloxone
e. Naltrexone

Ans: A
Disulfiram is used alongside other treatments and counselling for alcoholism. It is only suitable for people who
have been through detoxification ('detox') and have stopped drinking alcohol. It acts as a deterrent to drinking
further alcohol
Drinking even small amount of alcohol produces a very unpleasant reaction, disulfiram achieves this by
blocking the enzyme which breaks down acetaldehyde (intermediary product of alcohol metabolism)

Acamprosate is used for maintaining abstinence.

1019. A 68yo woman presents to the ED with confusion. Temp=39.3C and productive cough. Sputum is rusty
colored after 2 days. CXR shows right lower lobe consolidation. What is the most likely organism?
a. Streptococcus pneumonia
b. Staphylococcus aureus
c. Coxiella burnetti
d. Mycoplasma pneumonia

Ans: A
Reason:most common pathogen causing pneumonia in elderly, Rusty Coloured Sputum, lobar cosolidation on
cxr.
treatment : Amoxixillin 5-8 days
benzylpenicillin, cephalosporin

1020. A 70yo man with prostatic cancer has had severe acute back pain waking him up at night for 6wks. What
is the most appropriate inv?
a. MRI spine
b. Radionuclide bone scan
c. DEXA scan
d. Serum ALP concentration
e. Serum calcium concentration
Ans: B
Reason: A Radionuclide bone scan can detect tumor metastasis in bones,diagnosis of fractures that may not
be visible on traditional x-rays.a nuclear bone scan is a functional test: it measures an aspect of bone
metabolism or bone remodelling , which is higher in a tumor abiding bone tissue.
DEXA scan measures bone density which has significance in detecting osteoporosis.

1021. An asymptomatic 56yo man who has never consumed alcohol came for a routine checkup. Exam:
increased skin pigmentation, spider angioma, cardiomegaly, S3 gallop, liver firm with 8cm span, no ascites. He
is in the risk of which condition?
a. Cerebellar degeneration
b. Wernieckes encephalopathy
c. Renal failure
d. Hepatoma
e. Hepatic vein thrombosis

Ans: D
The patient seems to be suffering from Hereditary Haemochromatosis, there is a defect in HFE gene which
affects iron metabolism by affecting hepcidin and results in increased absorption of iron from intestine and
increased deposition in the body tissues. HH presents itself as Skin bronzing, pancreatic insufficiency,
hepatomegaly and later cirrhosis (These patients are also at a higher risk of developing hepatocellular
carcinoma) cardiomyopathy, neurological or psychiatric symptoms.

1022. A 39yo male presents with visual symptoms. Ophthalmoscopy shows papilloedema. Which anatomical
site is most likely to be affected?
a. Optic nerve
b. Optic disc
c. Optic radiation
d. Occulomotor nerve
e. Optic chiasma

Ans:B
Optic disc swelling can be caused by a number of conditions; papilloedema relates more specifically to optic
disc swelling secondary to raised intracranial pressure.

Disc swelling is distinct from disc atrophy which refers to a loss of nerve fibres at the optic nerve head and
which results in a pale disc. Atrophy may be primary (where it occurs without prior disc swelling) or secondary
(where it is preceded by disc swelling)

1023. A 75yo man has been attending the clinic for lower urinary tract symptoms. His mood is very low and he
says he feels unhappy, anxious and unable to sleep. He has been dx with moderate depression. What tx would
be most effective for this pt?
a. Amitriptyline
b. Citalopram
c. CBT
d. Dosulepin
e. Diazepam

ANS; C
amitriptyline and Dosulepin both cause urinary retention, Citalopram causes insomnia in more than 15%
patients, patient is already suffering from both problems.
MILD DEPRESSION:initial recommended treatment for mild depression is CBT.
MODERATE-SEVERE: Offer antidepressant medication combined with high-intensity psychological treatment
(CBT or interpersonal therapy (IPT)). For an individual with a chronic health problem and moderate
depression, this should be high-intensity psychological treatment alone in the first instance

1024. A 48yo pt after surgical removal of mandibular ca presents with perioral paresthesia and severe pain
which is not relieved by oral morphine. What is the next step in treating this pt?

a.Oral amitriptyline
b.Oral oxycodone
c.PCA
d.IV morphine
e.Fentanyl patch
f.Gabapentine

And: A
This is a neuropathic pain,According to NICE , offer a choice of amitriptyline,duloxetine,gabapentin or
pregabalin as initial treatment for neuropathic pain.often switching to another drug if one fails.

First line antidepressants Amitriptyline


2nd line antiepileptics Gabapentin
3rd line Duloxetine
4th line Pregabalin.

1025. A 34yo man was slapped over his right ear in a fight. There is blood coming from his external auditory
canal and he has pain, deafness and ringing in his ears. What is the most appropriate initial inv?
a. CT
b. MRI
c. Otoscopy
d. Skull XR
e. Facial XR

Ans: C

Reason : traumatic perforation of the tympanic membrane can causes pain,bleeding,hearing loss,tinnitus and
vertigo. Diagnosis is based on otoscopy ,treatment often is unnecessary.

Traumatic causes of preparation include


Insertion of objects
Concussion caused by an explosion or open handed slap across the ear
Head trauma with or without Mozilla fracture
Sudden negative pressure
Barotrauma

Diagnosis: otoscopy , audiometry


Management: often no treatment required ,for dirty injuries prescribe antibiotics amoxicillin 500 mg tid 8
days.most perforation close spontaneously ,those persisting more than 2 months require surgery .

1026. A 45yo man has developed an annular rash with a scaly edge on his thigh. The rash has been spreading
over the last 3wks. He has some general aches and pains. What is the single most useful investigation?
a. ANA
b. Biopsy lesion
c. Lyme antibodies
d. Skin scrap for mycology
e. Skin swab for bacteria

Annular spreading rash points towards erythema migrans, which is characterisitic of Lyme disease. Also
accociated with general aches and myalgia.
Lyme disease caused by bacteria borrelia burgdorferi, spirochaete.
Humans may have it if bitten by tics infected by bacteria.
Presententation depends on stage of disease.
Stage 1, early/localized: characteristic is erythema migrans. (spreading annular rash over 3-36 days, central
clearing, target like appearance, bulls eye lesion) associated flu like symptoms.
Stage 2, disseminated: occurs days to months later. Flu like symptoms. Neurological involvement. Unilateral,
bilateral facial nerve palsy, meningitis, encephalitis, peripheral mononeuritis. Lymphocytic meningo rediculitis.
Cardio vascular problems: myo/pericarditis, heart block. Lymphocytoma. Cutaneous lesion.
Stage 3/late: arthritis in knee. Acrodermatitis chronic. Late neurological disorders.
Post lyme syndrome: similar to chronic fatigue syndrome.
Investigations: clinical diagnosis. Serology: antibodies. PCR.
Rx: Doxycycline 100mg/day. Or amoxicillin.

1027. A 80yo man with prostatic cancer has confusion, thirst, abdominal pain and an abnormal ECG. What is
the most appropriate inv?
a. MRI spine
b. Radionuclide bone scan
c. DEXA scan
d. Serum ALP concentration
e. Serum calcium concentration

man presenting with symptoms of hypercalcemia. Indicated by prostatic cancer with mets.
Normal ca levels: 2.25-2.5mmol/L
Hypercalcemia: less than 2.8: ployuria, polydipsia, dyspepsia, depression,
At levels less than 3.5: all previous and muscle weakness, constipation, anorexia, fatigue.
At levels >3.5: all prev and abd pain, vomiting, dehydration, arrhythmia, pancreatitis, coma. Nephrocalcinosis.
Causes: malignancy and prim hyper PTH, sarcoisosis, TB, Endocrine conditions, thiazide, vit D, Familial FHH,
prolonged immobilization.
Investigations: Corrected Ca levels.
Rx: rehydrate with Normal Saline, furosemide. IV Bisphosphonates after hydration
.
1028. A 27yo lady after C-section developed epigastric pain after 8h. What is the appropriate inv?
a. ABG
b. Coag profile
c. Liver enzyme
d. Liver biopsy
symptom is pointing towards liver abnormality. So C is most likely. Could be because of HELLP syndrome
characterized by hemolysis, elevated liver enzymes, low platelets. Usually occurs in pregnant females of pre
eclampsia and eclampsia. May present in last half of pregnancy or shortly after delivery. Initial symptoms are
non specific. Like malaise, fatigues, epigastric pain, nausea
Investigation: CBC with peripheral smear, raised serum LDH, Bilirubin, liver enzymes.
Rx: deliver as soon as possible
If post partum then give steroids and plasma exchange.
Complications: DIC, pulmonary edema, renal failure, liver hemorrhage and failure. Retinal detachment

1029. A 35yo woman presents with visual problems. CT brain reveals pituitary tumor. What is the single most
likely defect?
a. Homonymous hemianopia
b. Homonymous upper quadrantopia
c. Bitemporal hemianopia
d. Cortical blindness
e. Homonymous lower quadrantopia

dx: pituitary adenoma. Which presses on optic chiasm leading to bitemporal hemianopia. Optic chiasm
contains nasal fibres from both eyes.
If optic nerve is damaged it will lead to complete blindness in eye of same side.
If lesion is at optic chiasm, then bitemporal hemianopia
If the lesion is at optic tract, then contralateral homonymous hemianopia
If the Lesion is at temporal fibres of optic radiation, then contralateral upper quadrant hemianopia. If parietal
fibres are involved, then lower quadrant hemianopia.
Lesions in primary visual cortex then contralateral homonymous hemianopia with macular sparing.
Cortical blindness is blindness with intact pupillary reflexes.

1030. A 45yo heroin addict was involved in a car crash and is now paraplegic. During the 1st week of hospital
stay he cried everyday because he couldnt remember the accident. What is the most likely dx?
a. PTSD
b. Severe depression
c. Organic brain damage
symptoms are pointing towards brain damage.
It is a brain injury resulting from a medical cause and not a psychiatric cause. For eg trauma, hemorrhage,
concussion, hypoxia, hypercapnia, stroke, heart infections, Alzhiemers, degenerative disorders, metabolic
causes, kidney liver disease, drug and alcohol.
Inv and Rx depend on underlying disorder.
Symptoms: agitation, confusion, dementia, delirium.
1031. A pt with T1DM has a fundus showing micro-aneurysm and hard exudate. What is the single most likely
dx?
a. Macular degeneration
b. Hypertensive retinopathy
c. MS
d. Diabetic background
e. Proliferative DM retinopathy
different stages of diabetic retinopathy:
Background retinopathy: microaneurysm, hemorrhagic blots, hard exudates
Pre proliferative: cotton wool spots, hemorrhage, venous bleeding
Proliferative: neo vascularisation
Maculopathy: visual acuity decreased, new vessels on optic disc, retinal detachment
1032. A 62yo man has multiple liver mets due to adenocarcinoma with an unknown primary. He is deeply
jaundiced and has ascites with edema upto the buttocks. He is now drowsy and his family are worried that he
is not drinking enough. His meds include: haloperidol 1.5mg, lactulose 10ml. Bloods taken 3d ago: electrolytes
normal, urea=6.5mmol/l, creatinine=89mmol/l, calcium=2.04mmol/l, albumin=17g/L, total
bilirubin=189mmol/l. What is the single most appropriate management of his fluid intake?
a. Albumin infusion
b. Crystalloids IV
c. Crystalloids SC
d. Fluids via NGT
e. Fluids PO
scenario of decompensated liver disease as indicated by ascites, jaundice and drowsiness. Fluid restriction
should be done because of edema and ascites. His albumin levels are low as normal value of albumin is 35-
50g/L. ascites is to be managed with fluid restriction, low salt, diuretics and daily weighing.
1033. A 2yo with atrophy of the buttocks, distended abdomen with frequent offensive smelly stool. Choose
the single most likely inv?
a. Upper GI endoscopy
b. Endomyseal/alpha glidin antibody
c. Sweat test
d. Colonscopy
e. Stool culture
Dx: celiac disease:
Immune mediated inflammatory systemic disorder provoked by gluten and prolamines in genetically
susceptible people. Gluten in wheat rye and barley.
Associated with HLA DQ2 and DQ8.
May present at any age. Babies and yung present after weening. Symptoms: malabsorption, weight loss,
failure to thrive, vomiting, anorexia, abd distension.
Older child presents with anemia, abd pain, malabsorption, mouth ulcers.
Dermatitis herpetiformis classic manifestation of skin Involvement.
INV: Anti TTG, Anti endomysial, anti gliadin antibodies.
IgA anti TTG preferred.
Antibodies used to monitor disease.
Confirmation by duodenal biopsy.
Rx: gluten restriction
1034. A 78yo woman is brought to the hospital complaining of back pain and is referred to the surgeon. She
has been saying that her mother is due to visit her today and that somebody must have broken her lower back
as she is in agony. Labs: creatinine=295 mmol/l, calcium=3.03mmol/l. Which inv is most likely to lead to a dx?
a. US KUB
b. XR Spine
c. IVU
d. Bence-Jones Protein
e. Mental state exam
points in favour of myeloma: Age, low back pain, increased creatinine and hypercalcemia. XRAY spine would
not lead to diagnosis and MSE would have been done if the labs were normal.
Myeloma is due to abnormal proliferation of a single clone of plasma cells leading to secretion og Ig
immunoglobulin ot Ig fragment.
Symptoms: bone lesions, anemia, neutropenia, thrombocytopenia, recurrent infection, renal impairment.
Dx made on:
serum or urine electrophoresis.
Plasma cells increased on marrow biopsy
End organ damage or bone lesion
Complications: hypercalcemia
Spinal cord compression
Hyperviscosity
Acute renal injury
Rx: supportive and chemotherapy. Allogenic tx.
1035. A 40yo woman presents with dysphagia. Exam: febrile with erythema and middle neck swelling. What is
the best management strategy?
a. IV antibiotics and drainage
b. Antipyretics
c. XR neck
d. Endoscopic diverticulectomy
e. I&D
presentation is that of a neck abscess. IV abx and InD forms the basis of Rx for neck abscess.
1036. A young lady presents with gradually worsening headaches, visual disturbance, and lack of energy. MRI
shows 15mm tumor in the pituitary fossa. What is the tx of choice?
a. Radiotherapy
b. Octreotide
c. Reassurance and f/u after 6m
d. Surgery
e. Chemotherapy
pituitary tumor: almost always benign and incurable. May be associated with MEN 1 syndrome. Types of
tumors include
adenoma
prolactinoma
GH secreting
ACTH secreting
TSH, FSH, LH secreting
Symptoms depend on the hormone tht is being produced.
Local effects of tumor includes:: retro orbital headache, worse on waking up.
Obstruction of CSF resulting in hydroceph
Ocular nerve palsy causing squint
Disorder of thirst, appetite, temperature regulation if extended to hypothalamus.
INV: MRI
Rx: depends on type of tumor. Surgery in most cases
Bromocriptine in Prolactin secreting tumors.
Somatostatin analogues in GH secreting tumors
Small non functioning adenomas in asymptomatic pts dont require any Rx.

1037. A man with dementia has an ulcerative lesion on his forehead. He wants it removed so it can help
improve his memory. Wife says he is not fit to give consent. What will you do?
a. Get letter signed from the GP
b. Get letter signed from the wife
c. Get letter signed from the pt
d. Refer to psychiatrist to assess the mental capacity to give consent
According to OHCM, consent in incapacitated requires a formal assessment to be documented in medical
notes. No one is able to give consent on behalf of any adult even if he is incapacitated. So he should be
referred to a psychiatrist.
1038. A pt with flame shaped hemorrhage on long term tx with nifedipine. What is the single most likely dx?
a. Macular degeneration
b. HTN retinopathy
c. MS
d. DM background
e. Proliferative DM retinopathy
f. SLE
pt is on Ca channel blocker indicating he is hypertensive.
Fundoscopic findings in hypertensive retinopathy:
Grade 1: tortuous arteries with thick shiny walls, silver copper wiring
Grade 2: AV nipping
Grade 3: flame hemorrhages, dot and blot, hard soft exudates
Grade 4: papilledema
Rx. Control BP
1039. A pt whose pain is not relieved by oral codeine. What is the best management?
a. Oral oxycodone
b. Co-codamol
c. PCA
d. IV morphine
e. Oral morphine
pain management steps:
1. Non opiods: paracetamol, NSAID,
2. if not controlled with above, use weak opioids ie codeine and tramadol
3. Still not controlled then strong opioids like morphine, dimorphine, oxycodone, fentanyl.
Points in favour of morphine:
First line for severe pain in palliative care
Beneficial effect such as euphoria and detachment
4. IV morphine, pethidine and fentanyl patch if all of the above fail
1040. A 6wk baby with vomiting, irritability and palpable mass in the abdomen on feeding. Choose the single
most likely inv?
a. Upper GI endoscopy
b. Barium meal
c. US
d. CT abdomen
e. Barium enema
scenario of infantile hypertrophic pyloric stenosis. Presents at 3-8 weeks with vomits esp after feed, large
volume and projectile. Differentiating point btw other causes of vomiting is that vomitus doesnt contain bile,
no diarrhea but constipation. Alert, anxious. Hungry
o/e: left to right LUQ peristalsis during a feed. Olive sized pyloric mass present in RUQ
labs: water and NACL deficit, hypochloremic, hypo kalemic metabolic alkalosis.
Dx. Clinical. US may be done
Rx ramsdeths pyloromyotomy ot endoscopic surgery
1041. A 79yo man who is being treated with GnRH antagonist for proven adenocarcinoma of the prostate
attends a follow up session. What is the most appropriate inv?
a. Serum AFP
b. Serum PSA
c. Serum acid phosphates conc
d. Serum ALP isoenzyme conc
e. Trans rectal US
urologists rely on rising PSA results to signal that a radical intervention (usually either chemotherapy or
radiotherapy) is necessary. This is particularly appropriate for older patients with comorbidities, on the basis
that they are likely to die of some other cause before a slow-growing prostate tumour has an effect on their
lifespan. Such 'active monitoring' is also appropriate for any patient who wishes to avoid the side-effects of
interventional management.
Most prostate cancers are adenocarcinomas arising in the peripheral zone of the prostate gland
Risk factors: Age
Black-african
Family hx
Factors such as food consumption, pattern of sexual behaviour, alcohol consumption, exposure to
ultraviolet radiation, chronic inflammation and occupational exposure have all been considered as possible
risk factors
* Local disease:
o Raised PSA on screening.
o Weak stream, hesitancy, sensation of incomplete emptying, urinary frequency, urgency, urge incontinence.
o Urinary tract infection.
* Locally invasive disease:
o Haematuria, dysuria, incontinence.
o Haematospermia.
o Perineal and suprapubic pain. o Obstruction of ureters, causing loin pain, anuria, symptoms of acute kidney
injury or chronic kidney disease.
o Impotence.
o Rectal symptoms - eg, tenesmus.
* Metastatic disease:
o Bone pain or sciatica.
o Paraplegia secondary to spinal cord compression.
o Lymph node enlargement.
o Loin pain or anuria due to ureteric obstruction by lymph nodes.
o Lethargy (anaemia, uraemia).
o Weight loss, cachexia
* Abdominal palpation may demonstrate a palpable bladder due to outflow obstruction.
* DRE may reveal a hard, irregular prostate gland. Indications of possible prostate cancer are:
o Asymmetry of the gland.
o A nodule within one lobe.
o Induration of part or all of the prostate.
o Lack of mobility - adhesion to surrounding tissue.
o Palpable seminal vesicles.
Differential diagnosis * All other causes of haematuria (eg, urinary tract infection) and urinary tract
obstruction. * Benign prostatic hyperplasia. * Prostatitis. * Bladder tumours.
INV:
PSA
Transrectal needle biopsy
Urinalysis to exclude renal and bladder pathology. Urine sent for microscopy, culture and sensitivities.
Renal function tests to help exclude renal disease.
MRI should be considered for men with a negative TRUS core biopsy to determine whether another biopsy is
needed
MRI for staging
Bone scan for mets
The National Institute for Health and Care Excellence (NICE) referral guidelines for suspected cancer state:[11]
* Men presenting with symptoms suggesting prostate cancer should have a DRE and PSA test after
counselling. Symptoms will be related to the lower urinary tract and may be inflammatory or obstructive.
Prostate cancer is also a possibility in male patients with any
of the following unexplained symptoms: erectile dysfunction, haematuria, lower back pain, bone pain or
weight loss, especially in the elderly.
* Urinary infection should be excluded before PSA testing, especially in men presenting with lower tract
symptoms. The PSA test should be postponed for at least one month after treatment of a proven urinary
infection.
* If a hard, irregular prostate typical of a prostate carcinoma is felt on DRE, then the patient should be referred
urgently. The PSA should be measured and the result should accompany the referral.
* Patients do not need urgent referral if the prostate is simply enlarged and the PSA is in the age-specific
reference range.
* In a man with or without LUTS and in whom the prostate is normal on DRE but the age-specific PSA is raised
or rising, an urgent referral should be made. Symptomatic patients with high PSA levels should be referred
urgently.
* If there is doubt about whether to refer an asymptomatic man with a borderline level of PSA, the PSA test
should be repeated after an interval of one to three months. If the second test indicates that the PSA level is
rising, the patient should be referred urgently.
Rx: low risk localized tumor: active surveillance/surgery (personal preference)
Intermediate to high risk:
Men with intermediate and high-risk localised prostate cancer should be offered a combination of radical
radiotherapy and androgen deprivation therapy, rather than radical radiotherapy or androgen deprivation
therapy alone.
Men with intermediate and high-risk localised prostate cancer should be offered 6 months of androgen
deprivation therapy before, during or after radical external beam radiotherapy.
Continuing androgen deprivation therapy for up to 3 years should be considered for men with high-risk
localised prostate cancer.
High-dose rate brachytherapy in combination with external beam radiotherapy should be considered for men
with intermediate and high-risk localised prostate cancer. Brachytherapy alone should not be offered to men
with high-risk localised prostate cancer.
Locally advanced: pelvic radiotherapy
Metastasis: bilateral orchidectomy
Urinary tract obstruction, acute kidney injury, chronic kidney disease.
Sexual dysfunction: erectile dysfunction, loss of libido.
Metastatic spread: bone pain, pathological fractures, spinal cord compression.

1042. A middle aged woman has some weakness of hand after an injury. Which vertebra will be the lowest to
be included on cervical XR to dx the injury?
a. C7/T1
b. C8/T1
c. C5/C6
d. C6/C7
weakness of hand shows brachial plexus routes involvement ie C5-T1, so in Cervical Xray, lowest vertebra
included should be C7/T1 which will include all the nerve roots of brachial plexus.
1043. A 50yo man with a known hx of stroke. He cant remember anything about his life. What is the single
most likely defect?
a. Homonymous hemianopia
b. Homonymous upper quadrantanopia
c. Bitemporal hemianopia
d. Binasal hemianopia
e. Homonymous lower quadrantanopia

Memory storing site is in the temporal lobe. Lesion of the temporal lobe leads to homonymous upper
quadrantanopia.

1044. An 18yo girl has been dx with anorexia nervosa and has mild depressive symptoms. She has cut down
her food intake for the last 18m and exercises 2h everyday. Her BMI=15.5, BP=90/60mmHg. What would be
the single most appropriate management?
a. Refer to eating disorder clinic
b. Refer to psychodynamic therapy
c. Refer to acute medical team
d. Prescribe antidepressant
Answer should be A, according to OHCM. As the pt has moderate anorexia. Hence should be referred to EDU
rather than a medical unit as there are no severe symptoms at present.
mild anorexia BMI >17.5: focus on building a trusting relationship and encourage use of self help books and
food diary. If no response within 8 wks, then consider referral to sec care
moderate anorexia (BMI 15-17.5) routine referral to mental health team or adolescent unit or eating disorder
unit
severe anorexia (BMI <15, rapid wt loss, evidence of system failure): urgent referral to eating disorder unit,
medical unit
Admission to hospital should be considered if:
* There is risk of suicide or severe self-harm (acute psychiatric ward).
* Severe deterioration (may require admission to an acute medical ward).
* There is very low body weight (BMI less than 15 kg/m2) or rapid weight loss (admission to an eating
disorders unit may be most appropriate).
* Medical complications, such as pronounced oedema, severe electrolyte disturbance, bradycardia,
hypoglycaemia, or severe intercurrent infection (may require admission to an acute medical ward).
* Home environment is impeding recovery.

Anorexia:
The defining clinical features are:
* Refusal to maintain a normal body weight for age and height.
* Weight below 85% of predicted. This means in adults a body mass index (BMI) below 17.5 kg/m2.
* Having a dread of gaining weight.
* Disturbance in the way weight or shape is experienced, resulting in over-evaluation of size.
Other features include:.
* Other physical: includes fatigue, hypothermia, hypotension, peripheral oedema, gaunt face, lanugo hair,
scanty pubic hair, acrocyanosis (hands or feet are red or purple), and bradycardia. Delay in secondary sexual
development if pre-puberty.
* Symptoms such as fatigue, fainting, dizziness, constipation and intolerance of cold.
* Denial of the problem.
* Lack of desire for intervention, or resistance to it.
* Social withdrawal; few interests.
* Enhanced weight loss by over-exercise, diuretics, laxatives and self-induced vomiting.
Diagnosis:
SCOFF questionnaire (below) is a useful screening tool. Two or more positive answers should prompt a more
detailed history:
* Do you ever make yourself Sick because you feel uncomfortably full?
* Do you worry you have lost Control over how much you eat?
* Have you recently lost more than One stone in a three-month period?
* Do you believe yourself to be Fat when others say you are too thin?
* Would you say that Food dominates your life?
* Hypokalaemia: common and may cause fatal arrhythmias. * Hypotension. * Other cardiac problems
including arrhythmias, mitral valve prolapse, peripheral oedema, sudden death. * Anaemia and
thrombocytopenia. * Hypoglycaemia. * Osteoporosis: restoring the patient's weight is the best treatment.
Bone loss may never recover completely even once weight is restored. * Constipation.
* Lack of growth in teenagers, and lack of development of secondary sexual characteristics. * Infertility.
* Infections. * Renal calculi * Acute kidney injury or chronic kidney disease. * Alcoholism in some patients. *
Anxiety and mood disorders.
* Social difficulties.

1045. A 36yo woman has an injury to the right external laryngeal nerve during a thyroid surgery. What
symptom would be expected in this pt?
a. Stridor
b. Hoarseness
c. Aphonia
d. Dysphonia
e. Aphasia
stridor occurs in upper airway obstruction.
Hoarsness and aphonia in recurrent laryngeal nerve palsy
Dysphonia occurs in external laryngeal nerve pasly
Aphasia is a feature of stroke and any cortical lesion.
1046. A 75yo woman has weakness of the left side of her face. She has had a painful ear for 48h. There are
pustules in the left ear canal and on the eardrum. What is the single most likely dx?
a. Chronic serous OM
b. Herpes zoster infection
c. Impacted earwax
d. Perforation of eardrum
e. Presbycusis
given symptoms particularly pustules point towards herpes infection. when the varicella zoster virus
(chickenpox) becomes reactivated in the geniculate ganglion of the VIIth cranial nerve (facial nerve), it is called
ramsay hunt syndrome.
ramsay Hunt syndrome:
presenting features:
* Vertigo and ipsilateral hearing loss.
* Tinnitus.
* Facial weakness or face drop.
* The patient also complains of rash or blisters which may be on the skin of the ear canal, auricle or both, and
may become infected secondarily, causing cellulitis
Signs
* There is a rash or herpetic blisters in the distribution of the nervus intermedius.
* The distribution of the rash varies, as does the area innervated by the nervus intermedius. It may include the
following:
o The anterior two thirds of the tongue.
o The soft palate.
o The external auditory canal.
o The pinna.
* An ipsilateral face drop or weakness may be obvious or it may be elicited on testing.
* There may be hyperacusis on that side due to paralysis of the stapedius and tensor tympani.
* The patient may have associated ipsilateral hearing loss and balance problems.
The unilateral facial weakness is very similar to Bell's palsy but the rash is the characteristic diagnostic feature
to differentiate the two.
There is not always a rash, especially in younger patients. In children aged 5 to 15, acute facial palsy, like a
Bell's palsy and without a rash, may be produced by the varicella-zoster virus.[6]
Trigeminal neuralgia is paroxysmal and tends to be precipitated by a stimulus such as a cold wind or washing
the face.
Other conditions in the differential diagnosis include postherpetic neuralgia, persistent idiopathic facial pain
and temporomandibular disorders.[7]
You may also consider otitis (external, media),[8] referred pain (eg dental abscess) and carcinoma of the
nasopharynx
Diagnosis: usually clinical. Occasionally audiometry and NCV for facial nerve damage.

1047. An 8wk baby boy is noted to be jaundiced. He is breast-feeting well and has gained 300g since birth. His
stools are yellow and his urine is pale straw colored. What is the most likely dx?
a. Galactosemia
b. Biliary atresia
c. G6PD deficiency
d. Breast milk jaundice
e. Congenital viral infection
question is quite confusing as only some features point toward galactosemia. Firstly it usually presents in
neonatal period whereas this baby is 8 wks. However its variants may present later in life. Secondly they feed
poorly, but this is not the case here. Points in favour of galactosemia are jaundice and poor wt gain.
Biliary atresia is acause of obstructive jaundice hence will cause dark urine and pale stools, whereas here it is
yellow stool and pale straw colored urine which points towards galactosemia.
Breast milk jaundice is not associated with poor wt gain and the child would be healthy
Galactosemia:
* There is often feeding difficulty, with vomiting and failure to gain weight, with poor growth in the first few
weeks of life.
* Lethargy and hypotonia occur.
* Jaundice and hepatomegaly develop.
* There are often associated coagulation defects.
* Haemolytic anemia
* Metabolic acidosis
* Sepsis (often with Escherichia coli) can be fatal.
* Cataracts may be apparent even in the early days of life.
* Ascites may even be apparent in early life.
* The fontanelle is full.
* Developmental delay may affect speech, language and general learning.
* Adults may have short stature, ataxia and/or tremor.
* Hypergonadotrophic hypogonadism is common and in women, premature ovarian failure. Those who
conceive often have variant disease.
Galactosaemia should be considered when a term infant gets an E. coli sepsis and when a neonate develops
cataracts. It should also be considered with neonatal jaundice and haemorrhage.
INV
Beutler's test involves a fluorescent spot test for GALT activity. It is now widely used for the diagnosis of
galactosaemia but will give false positives with glucose-6-phosphate dehydrogenase deficiency.
positive test with Fehling's or Benedict's reagent
Rx
Remove milk from diet
Antibiotics, intravenous fluids and vitamin K are often required.
1048. A lady developed breast abscess after delivery. What is the most likely organism?
a. Staph aureus
b. Staph albus
c. GBS
d. Strep pyogenes
e. Strep faecalis
Dx: puerperal mastitis leading to abscess
The usual infecting organism is Staphylococcus aureus, although it may also be Staphylococcus albus and
streptococci. Meticillin-resistant Staphylococcus aureus (MRSA) infection is increasing, and may be more
common in women who have had a caesarean section.
Dx: clinical
Rx
Incision and drainage of abscess with cavity packed open with gauze is recommended if the overlying skin is
thin or necrotic.
Parenteral antibiotics should be administered at the same time, with added coverage for anaerobic bacteria.
Fluid from the abscess should be cultured, and results used to determine ongoing antibiotic treatment.
Needle aspiration of the abscess, repeated every other day until the pus no longer accumulates, has been
suggested as an alternative to open drainage.
In some cases breast-feeding may have to cease until the abscess is successfully treated, but can usually
resume later.
1049. A 32yo man suffering from MS presents with blurring of vision. Ophthalmoscopy shows pallor of the
optic disc. Which anatomical site is most likely to be affected?
a. Optic nerve
b. Optic disc
c. Optic radiation
d. Trigeminal
e. Oculomotor nerve
Optic neuritis is an acute, sometimes painful, reduction or loss of vision in one eye, and is a relatively common
presenting symptom of MS. Optic neuritis (ON) is inflammation of the optic nerve. Classically there is a triad of
clinical features - reduced vision (of varying severity), eye pain (particularly on movement) and impaired
colour vision.
Double vision
Facial weakness
Deafness
Depression
Taste and smell alteration
Loss of sensation in legstr. Myelitis
Urgency and frequency in passing urine
Impotence
Loss of thermoregulation
INV
Electrophysiology: can detect demyelination in apparently unaffected pathways with characteristic delays.
Visual evoked potential studies should be the first choice.
MRI scan: 95% of patients have periventricular lesions and over 90% show discrete white matter
abnormalities. Areas of focal demyelination can also be seen as plaques in the optic nerve, brainstem and
spinal cord.
Cerebrospinal fluid: rise in total protein with increase in immunoglobulin concentration with presence of
oligoclonal cases.
Rx:
Decrease stress
Steroid
Interferon
Monoclonal antibody, alemtuzumab
AZT for relapsing and remitting MS
1050. A 23yo man presents with severe pain in the right flank radiating to his groin. He is rolling about on the
floor. An IVU confirms a stone in the ureter which is 8mm in size. Which tx modality will be most effective?
a. Fluids and alpha blockers
b. ESWL
c. CCB
d. Dormier basket
e. PCNL
Rx options for stone include:
Extracorporeal shock wave lithotripsy (ESWL) - shock waves are directed over the stone to break it apart. The
stone particles will then pass spontaneously.
Percutaneous nephrolithotomy (PCNL) - used for large stones (>2 cm), staghorn calculi and also cystine
stones. Stones are removed at the time of the procedure using a nephroscope.
1051. A 37yo woman believes that her neighbours have been using her shower while she is away from home.
Her 42yo partner is convinced about this and calls the police. What term best describes this situ?
a. Capgras syndrome
b. Cotard syndrome
c. Delusion of persecution
d. Folie a deux
e. Munchausen syndrome
Folie a duex is defined as a delusion or mental condition shared by two people in close association.
Munchausen syndrome is when a person feigns illness to gain attention and sympathy.
Capgras syndrome is the irrational belief that a familiar person or place has been replaced with an exact
duplicate.
Cotards syndrome is the delusion where the patient believes that he/she is literally or figuratively dead.
Delusion of persecution is a delusion where a person falsely believes that they are being persecuted by
someone who intends to do them harm.
Thus since both partners believe in the neighbors using their shower, the answer is folie a duex.
1052. A 45yo woman has dull pain in her right ear which has been present for several weeks. There is no
discharge. Chewing is uncomfortable and her husband has noticed that she grinds her teeth during sleep. The
eardrum appears normal. What is the single most likely dx?
a. Dental caries
b. Mumps
c. OM
d. Temporomandibular joint pain
e. Trigeminal neuralgia
Dx. Temporomandibular joint pain
Earache, facial pain, and joint clicking/popping related to malocclusion, teeth-grinding (bruxism)
or joint derangement.
Stress making this a biopsychosocial disorder which may become a chronic pain syndrome
Signs:
Joint tenderness exacerbated by lateral movement of the open jaw, or trigger
points in the pterygoids.
Imaging: MRI.
Associations: Depression; Ehlers Danlos
Rx: NSAIDs (PO or topical, eg Diclofenac); Stabilizing orthodontic occlusal prostheses; cognitive therapy;
physiotherapy; biofeedback; Reconstructive Surgery; acupuncture.
1053. A 42yo lady had corrective surgery for cyanotic congenital heart disease at the age of 3y, after a
palliative operation during infancy. There is a parasternal impulse and an early diastolic murmur. What is the
most probable dx?
a. Aortic regurgitation
b. Ischemic mitral regurgitation
c. Aortic stenosis
d. Pulmonary stenosis
e. Pulmonary regurgitation
Dx is pulmonary regurgitation.
42 yo lady has parasternal impulse and early diastolic murmur. Received corrective surgery for cyanotic
congenital heart disease (pulmonary hypertension, Eisenmenger syndrome)
Early diastolic murmur only in aortic regurgitation or pulmonary regurgitation.
Causes of pulmonary hypertension also cause pulmonary regurgitation.
{Graham steell murmur if mitral stenosis+ pulmonary hypertension}
1054. A 45yo lady presents with hx of double vision and facial numbness. Which anatomical site is most likely
to be affected?
a. Cerebral cortex
b. Trigeminal nerve
c. Oculomotor nerve
d. Brain stem
e. Basal ganglia
Dx is brain stem.
Vertibobasilar circulation. Supplies the cerebellum, brainstem, occipital lobes. Occlusion causes signs relating
to any or all 3: hemianopia; cortical blindness; diplopia; vertigo; nystagmus; ataxia; dysarthria; dysphasia;
hemi- or quadriplegia; unilateral or bilateral sensory symptoms; hiccups or coma.
1055. A 30yo woman has experienced restlessness, muscle tension and sleep disturbance on most days over
the last 6m. She worries excessively about a number of everyday events and activities and is unable to control
these feelings which are impairing her ability to hold down her job. What is the most likely dx?
a. Panic disorder
b. GAD
c. Pheochromocytoma
d. Acute stress disorder
e. Social phobia
Dx is GAD.
GAD is anxiety and +3 somatic symptoms present over a course of 6 months.
Panic disorder is the experience of intense anxiety along with 4 symptoms of autonomic hyperactivity lasting
less than 30 mins.
Acute stress disorder is the experience of symptoms by a person under a maximum period of 1 month
following exposure to a traumatic event.
Social phobia is the fear of a situation where something potentially embarrassing might happen.
Causes of GAD are genetic predisposition, stress and events involving stress.
Treatment of GAD is through symptom control, exercise, meditation, behavioural therapy, hypnosis and
various drugs such as benzodiazepines, ssris, azapirones, beta blockers and antihistamines.
Prognosis gets better by age 50 years.
1056. Which of the following is not a degenerative corneal disease?
a. Band keratopathy
b. Marginal dystropathy
c. Fatty/lipid degeneration
d. Moorens ulcer
e. Keratoconus
Dx is Moorens ulcer.
Band keratopathy is characterized by the appearance of a band across the central cornea, formed by the
precipitation of calcium salts on the corneal surface (directly under the epithelium). This form of corneal
degeneration can result from a variety of causes, either systemic or local, with visual acuity decreasing in
proportion to the density of the deposition.
Pellucid marginal degeneration is a degenerative corneal condition, often confused with keratoconus.
Keratoconus is a degenerative disorder of the eye in which structural changes within the cornea cause it to
thin and change to a more conical shape than the more normal gradual curve.
Moorens ulcer is a chronic, painful peripheral corneal ulcer of unknown cause that easily leads to loss of
vision. Severe pain, red, tearing and photophobic.
Fatty/lipid degeneration is degenerative.
In moorens ulcer severe pain is common and eye(s) may be very red, photophobic, and tearing. It is more
common in southern and central Africa, China, and India. Treatments tried: steroidal and nonsteroidal anti-infl
ammatory drops, cytotoxics (topical and systemic), conjunctivectomy, and cornea debridement (superficial
keratectomy). None is known to be superior

1057. A 30yo man presents to hosp complaining that his urine has been very dark recently, resembling coffee
at worst. He has been under the weather 2wks back and had taken a few days off work with a sore throat and
coryzal symptoms. Urine dipstick in hosp returns highly positive for blood and protein. He is admitted for
supportive management and is scheduled for a renal biopsy, which shows mesangial proliferation with a
positive immune-flurescence pattern. What is the most probable dx?
a. Membranous glomerulonephropathy
b. SLE
c. Wegeners granulomatosis
d. Post strep GN
e. IgA nephropathy
Dx is Post strep GN
History of sore throat, mesangial proliferation and immune flouresence pattern point to post strep GN.
Presentation is usually nephritic syndrome.
Renal biopsy isnt performed unless atypical presentation.
IF shows IgG and C3 deposits
Serology shows inc ASOT and inc C3
Supportive treatment with more than 95% function recovered
1058. A 65yo lady presents with a 6h hx of facial droop and weakness in the left side of her body. What single
agent will she be prescribed for her whole life?
a. Clopidogrel
b. Altepase
c. Aspirin
d. Labetalol
Rx is clopidogrel.
65 yo lady with facial droop and weakness on left side is suggestive of a stroke.

1059. A 10yo boy is brought to the hosp with a rash over his buttocks a/w abdominal pain and vomiting. In the
ED, he is accompanied by his mother and stepfather. His mother had left him for the weekend with the
stepfather and was called to come back from holiday as he started to have some hematuria with the rash.
Social services had been notified on arrive to hospital. What is the most probably dx?
a. NAI
b. ITP
c. HSP
d. ALL
e. HUS

HSP is a Small vessel vasculitis with purpura (non blanching purple papules)-buttocks and extensor surfaces.
Young. Glomerulonephritis, arthritis, abd pain (+_ intussusception) may mimic an acute abdomen. Rx is
supportive.

1060. A man with hx of fall had confusion and laceration mark on the head. Which is the most appropriate
vessel affected?
a. Basilar artery
b. Middle meningeal artery
c. Vertebral artery
d. Diploic vein
The history of fall and laceration mark suggests an extradural hemorrhage. Therefore, artery affected would
be middle meningeal.
Test : CT scan to confirm hemorrhage.
Stabilize and transfer urgently (with skilled medical and nursing escorts) to a neurosurgical unit for clot
evacuation ligation of the bleeding vessel. Care of the airway in an unconscious patient and measures to ICP
often require intubation and ventilation (+ mannitol IVI
Prognosis Excellent if diagnosis and operation early. Poor if coma, pupil abnormalities, or decerebrate rigidity
are present pre-op
1061. A 72yo lady is drowsy and her relatives want to take her home. She has been prescribed diazepam
2.5mg. What is the best delivery route?
a. Oral
b. IV
c. IM
d. Per rectal
e. SC
Delivery is Per rectal.
Diazepam is not absorbed properly via the oral or IM route.
The IV route presents difficulties for non-medical carers.
The SC route is contraindicated in old people.
1062. A nonsmoker who has worked in coal mines for 20yrs presents with gradually increasing SOB, limited
exercise tolerance and a dry cough. His CXR shows round fibrotic tissue demonstrating a mixed restrictive and
obstructive ventilator defect with irreversible airflow limitation and reduced gas transfer. What is the single
most appropriate dx?
a. Churg-strauss syndrome
b. Cryptogenic organizing
c. Extrinsic allergic alveolitis
d. Good pastures syndrome
e. Progressive massive fibrosis
f. Molluscum
Key: E
Page 192, Industrial Dust Disease; Chest Medicine; OHCM 9TH Edition.
Coal Workers Pneumonia (CWP): Underground Coal Mines. Over 15-20 years. Fibrosis- round opacities (1-10
mm) esp in upper zone.
Clinical features: asymptomatic (co existing chronic bronchitis common)
PMF: Due to progression of CWP, which causes progressive dyspnoea, brosis, and, eventually, cor pulmonale.
CXR: upper-zone brotic masses (110cm).
Management: Avoid exposure to coal dust; claim compensation.
[In Extrinsic AA: farmer- mushroom worker, bird fancier, malt worker, sugar or bagassosis worker
4-6h post exposure: Fever, rigors, myalgia, dry cough, dyspnoea, crackles
In IPF (CFA): Dry cough, exertional dyspnea, dec weight, arthralgia, cyanosis, clubbing]
1063. A pt was complaining of pain within 6h after his appendectomy for gangrenous appendix. What med is
the best option for his pain relief?
a. IV morphine
b. Diclofenac per rectal
c. PCA
d. Tramadol
Rx is Tramadol
Who Pain Ladder
Rung 1 Non-opioid Paracetamol; NSAIDS
Rung 2 Weak opioid Codeine; dihydrocodeine; tramadol
Rung 3 Strong opioid Morphine; diamorphine; hydromorphone; oxycodone; fentanyl; buprenorphine (
adjuvant analgesics)
Tramadol as pain is of moderate severity.
Morphine is for severe pain.
Diclofenac is for mild pain.
PCA is not a drug.
1064. A 62yo farmer presents with a persistent firm irregular lesion on upper part of pinna which grew over
the last few months. What is the most appropriate dx?
a. Basal cell
b. Squamous cell
c. Keratocanthoma
Dx is basal cell
Basal cell is most common.
Squamous and keratocanthoma are less common and similar in presentation.
GOLJAN PATHOLOGY it says that such lesions above the upper lip is basal cell. And below that is SCC- if on
face.
1065. A 24yo schizophrenic has been under antipsychotic tx for the last 1 yr and now complains of Erectile
Dysfunction. Which drug is most likely to have caused this?
a. Fluoxetine
b. Citalopram
c. Clozapine
d. Haloperidol
e. Risperidone
The answer here should be E Risperidone. C,D and E are the antipsychotics from the options. OHCS states that
atypical antipsychotics cause erectile dysfunction so haloperidol goes out of the race. Since there are great
chances of agranulocytosis by using clozapine and the first choice is risperidone for schizo so the patient
would have been advised Risperidone which has caused ED.

1066. What is the most likely dx based on this ECG?


a. Normal
b. VT
c. Sinus Tachycardia
d. WPW syndrome
e. A-fib
Key: C
No other apparent abnormality
1067. A 45yo woman has recently been dx with MS and has been started on oral steroids. She is brought to
the hosp after having ingested 100 paracetamol tablets 4h ago. She is refusing all med tx. What is the next
best step?
a. Observe
b. Refer to psychiatrist to assess pts ability to refuse tx
c. Gastric lavage
d. Activated charcoal
e. Refer to social worker
Key says B.
Not a very clear answer though something like this given on:
Page 403, Psychiatry; OHCS 9TH Edition.
1068. A 44yo obese pt with findings: FBS=6 mmol/l, OGTT=10 mmol/l. What is the most likely dx?
a. Impaired glucose tolerance
b. Diabetes insipidus
c. T1DM
d. T2DM
e. MODY
Dx is IGT
IGT: fasting <7 and OGTT >7.8 but <11.1
WHO criteria (for DM):
* Symptoms (polydipsia, polyuria, weight loss, lethargy, blurring, genital thrush)+ fasting >7 and random >11.1
* Raised venous fasting >7 and random >11.1- on 2 separate occasions or OGTT> 11.1
* HB 1Ac> 48mmol/L (6.5%)
1069. A child distressed with fever, stridor and unable to swallow saliva. His RR=40bpm. What is the initial step
that needs to be taken?
a. Examine throat
b. Secure airway
c. Keep him laid flat
d. IV penicillin
Key: B
Page 158, URTI; Paeds; OHCS 9TH Edition.
DD for Stridor is Epiglotitis, bacterial tracheitis and Viral Croup. Based on symptomology its epiglotitis [croup
has barking cough and hoarseness, Bac Tracheitis has mucopurulent exudates not cleared by coughing]
Cause of risk of obstruction in both tracheitis and epiglotittis, we first call the anesthetist to secure the airway.
[after first laryngoscopy that shows cherry red swollen epiglottis ] Then 3rd generation Cefotaxime. For
tracheitis, give additional flucloxacillin.
1070. A pt presents with hemoptysis 7d after tonsillectomy and adenoidectomy. What is the next step of
management?
a. Explore again
b. Pack it
c. Oral antibiotics and discharge
d. Admit and IV antibiotics
e. Ice cream and cold fluid
Key: D
Page 565, Tonsillectomy; ENT; OHCS 9TH Edition.
Primary: within 24 hr needs return to theatre. Secondary: from after 24 hrs to 5-10 days post surgery is due to
infections. Needs admission and IV antibiotics [along with HO gargles and vasoconstrictors]
When severe hemorrhage- Admit, O2, IVI, crossmatch, antibiotics (co-amoxiclav)
1071. A 55yo man presents with swelling at the angle of the mandible which is progressively increasing in size
and its mobile for 6m. What is the most probable dx?
a. Benign parotid
b. Mandible tumor
c. Tonsillar carcinoma
Key: A
Page 578, Salivary Gland Tumours; ENT; OHCS 9TH Edition.
80% - benign pleomorphic adenoma, parotid gland and in superficial lobe.
Middle aged, man, parotid gland, slow growth and mobile: all favoring a benign parotid swelling like
Pleomorphic Adenoma.
Rx: Removal by superficial parotidectomy or Enucleation.
1072. A 61yo man, known smoker, comes to the hospital with complaints of painless hematuria, urgency and
dysuria. He has been worried about his loss of weight and reduced general activity. Which inv would be
diagnostic of his condition?
a. Urine microscopy
b. IVU
c. CT
d. Cystoscopy
e. US abdomen
f. KUB
g. Cystoscopy with biopsy
h. Mid stream urine for culture
i. Transrectal US
Key: G
Page 648, Bladder Tumors; Surgery; OHCM 9TH Edition.
Transitional Cell Carcinoma: painless hematuria, frequency urgency and dysuria ; UTI; UTO plus Smoking is
considered one of the important causative factors to TCC. Cystoscopy with Biopsy is confirmatory
1073. An 8wk pregnant lady is brought to the ED due to severe vomiting. She was administered IV fluids and
oral anti-emetics. She still cant tolerate anything orally. What is the next best tx?
a. IV feeding
b. IV antiemetics
c. Termination of pregnancy
d. PPI
e. IV steroid
Tx is IV anti emetics.
Woman may be progressing towards Hyperemesis Gravidarum.
IV feeding, IV steroids and PPIs are options if IV antiemetics fail to work
Termination is only the last option.

1074. A 48 yo man presents with bone pain. Labs: ALP=high, phosphate=normal. What is the most likely dx?
a. Osteoporosis
b. Osteomalacia
c. Pagets disease
d. Fx
e. Myeloma
Key: C
Page 699, Clinical Chemistry; OHCM 9TH Edition.
Also known as osteitis Deformans. There is Inc bone turnover ass with osteoclastic and osteoblastic activity
causing brittle bones and bone pain. Mostly pts over 40 and Ca and PO4 are normal. ALP is raised
1075. A 54yo lady presents with sudden severe pain in the left half of her skull. She also complains of pain
around her jaw. What is the most likely next step?
a. CT
b. MRI
c. Fundoscopy
d. ESR
e. Temporal artery biopsy
Key: D
Page 558, Vasculitis; Rheumatology; OHCM 9TH Edition.
Giant cell arteritis (GCA)= cranial or temporal arteritis. Common in the elderlyconsider Takayasus if under
55yrs.
It is associated with PMR in 50%
Symptoms: Headache, temporal artery and scalp tenderness (eg when combing hair), jaw claudication,
amaurosis fugax, or sudden blindness, typically in one eye. Extracranial symptoms may include dyspnoea,
morning stiffness,and unequal or weak pulses.
If you suspect GCA, do ESR and start prednisolone 60mg/d PO immediately then go for temporal artery biopsy
which is definitive.

1076. A 7yo school boy has been dx with meningococcal meningitis. What is the advice for schoolmates and
staff?
a. Rifampicin for the whole class and family
b. Rifampicin for the whole school and family
c. Meningococcal vaccine for the family
d. Benzylpenicillin
e. IV cefotaxime
Key is A
Prophylaxis: household contacts in droplet range or Those who have kissed the patient's mouth. Give
rifampicin (600 mg 12 hrly PO for 2 days).
1077. A pt came with dyskaryosis to the OPD. She is a heavy smoker and alcoholic. Cervical smear shows
abnormal cells. What is the best advice for her?
a. Colposcopy
b. Biopsy
c. Endocervical sample
d. Repeat after 4m
e. None
f. Cone biopsy
If there is class 3 mild moderate or class 4 severe dyskaryosis on smear, the next step is ro refer the lady for
colposcopy, and if needed punch biopsy.
Class 1
Normal pap smear: repeat in 3 years
Class 2
Inflammatory pap smear: Take swab and treat infection. Repeat in 6 months. Colposcopy after 3 abnormal
smears
Mild atypia: repeat in 4 months. Colposcopy after 2 abnormal smears
Class 3
Mild dyskaryosis: HPV test +/- colposcopy
Moderate dyskaryosis: colposcopy
Class 4
Severe dyskaryosis: colposcopy
Class 5
Suspected invasion and abnormal glandular cells: urgent colposcopy
1078. Pt with pain and swelling in left leg and thigh up to the level of inguinal ligament. Where is the level of
occlusion?
a. Femoro-popliteal artery
b. Left common iliac artery
c. Aortoiliac artery
d. Femoral artery
e. Profound femoral artery
The location of the pain in patients with peripheral arterial occlusive disease (PAOD) is determined by the
anatomic location of the arterial lesions. PAOD is most common in the distal superficial femoral artery (located
just above the knee joint), a location that corresponds to claudication in the calf muscle area (the muscle
group just distal to the arterial disease). When atherosclerosis is distributed throughout the aortoiliac area,
thigh and buttock muscle claudication predominates.
In this scenario, there is pain in left leg and thigh, so femoral artery is more likely to be occluded in this
patient. Risk factors:Smoking, Diabetes mellitus. Hypertension. Hyperlipidaemia: high total cholesterol and low
high-density lipoprotein (HDL) cholesterol are independent risk factors. Physical inactivity. Obesity.
The most common symptom is muscle pain in the lower limbs on exercise (intermittent claudication):
* Walking impairment - eg, fatigue, aching, cramping or pain in the buttock, thigh, calf or foot, particularly
when symptoms are quickly relieved at rest. Pain comes on more rapidly when walking uphill than on the flat.
Claudication can occur in both legs but is often worse in one leg.
* Similar pain may occur in the buttocks and thighs, associated with absent femoral pulses and male
impotence (Leriche's syndrome; caused by aorto-iliac obstruction).
* The differential diagnosis of pain in the lower limb when walking includes sciatica and spinal stenosis, deep
vein thrombosis, entrapment syndromes and muscle/tendon injury.
* The main method to confirm the diagnosis is Doppler ultrasonography (duplex scanning). The ratio of
systolic blood pressure at the ankle and in the arm - ankle-brachial pressure index (ABPI) - provides a
measure of blood flow at the level of the ankle (as a general guide, normal = 1, claudication 0.6-0.9, rest
pain 0.3-0.6, impending gangrene 0.3 or less). The ABPI is a strong marker of cardiovascular disease and is
predictive of cardiovascular events and mortality
* Treatment includes reduction and modification of risk factors, medical management with anti platelets and
peripheral vasodilators. Surgical procedures such as endovascular revascularization and bypass surgery.
1079. A 65yo man presents with dyspnea 3d after an MI. On auscultation he has a pansystolic murmur at the
apex radiating to the axilla. What is the most likely dx?
a. Ruptured papillary muscle
b. Ventricular aneurysm
c. Pericarditis
d. Pericardial effusion
e. VSD
Complications post MI
Cardiac arrest, unstable angina, bradycardia or heart block, tachyarrhythmias, right ventricular failure,
pericarditis, DVT and PE, systemic embolism, cardiac tamponade, mitral regurgitation, VSD, late malignant
vent arrhythmia, dresslers syndrome, left ventricular aneurysm.
In this scenario, patient has presented to us post MI with complains of dysnea (pulmonary edema).
Auscultation reveals a pansystolic murmur at apex radiating to axilla characteristic of MR murmur. MR post MI
occurs as a result of papillary muscle dysfunction (mild MR) or papillary muscle or chordal rupture or ischemia
(severe MR).
Vsd will also present with pansystolic murmur but at lower left sternal edge.
Pericarditis and pericardial effusion with muffled heart sounds and pericardial pain relieved by sitting forward.
Ventricular aneurysm occurs late after 4-6 weeks. Presents with LVF, angina, recurrent VT or systemic
embolization. Persistent ST segment elevation. Treatment is excise and coagulate.

1080. A 64yo man with multiple myeloma has been vomiting since the past 2days. Labs: Ca2+=3.2mmol/l,
K+=5mmol/l, Na+=149mmol/l and PCV=55%. What is the most appropriate next step?
a. IV insulin
b. IV calcium gluconate
c. IV fluids
d. IV bisphosphonates
e. Oral bisphosphonates
Multiple myeloma, vomiting point toward diagnosis of hypercalcemia indicated by raised serum calcium and
dehydration indicated by raised sodium, potassium and increased pcv. There has been inadequate water
replacement as a result of vomiting.
Treatment is to give IV fluids initially
Iv bisphosphonates may be used later
Iv furosemide may also be used after rehydration. Acute hypercalcaemia[11]
Treatment should be initiated in hospital on the advice of a specialist and should include:
* Increasing the circulating volume with 0.9% saline, helping to increase the urinary output of calcium.
* A loop diuretic such as furosemide. This is occasionally used where there is fluid overload but it does not
reduce serum calcium .
* After rehydration, bisphosphonates (which act by reducing bone turnover) should be administered
intravenously. Pamidronate and zoledronic acid are commonly used. Salmon calcitonin may also be given. It
has fewer side-effects than bisphosphonates but is less effective in reducing hypercalcaemia.
* Glucocorticoids are useful for hypercalcaemia due to vitamin D toxicity, sarcoidosis and lymphoma.
* Gallium was identified as a useful drug when it was found that patients with malignancy having gallium scans
did not develop hypercalcaemia. It may be given intravenously to patients with malignant hypercalcaemia who
do not respond to bisphosphonates.[14]
* Cinacalcet hydrochloride is a calcimimetic (= mimicking the action of calcium) agent that effectively reduces
parathyroid levels in patients with secondary hyperparathyroidism.[12]
* A new experimental approach to malignancy-associated hypercalcaemia involves the blockade of receptor
activator of nuclear factor kappa-B ligand, usually abbreviated as RANKL. RANKL is a key element in the
differentiation, function and survival of osteoclasts, which plays an essential role in removing calcium ions
from the bone in response to PTH stimulation.[15] Denosumab, a human monoclonal antibody that acts in this
manner, is licensed for the prevention of osteoporotic fractures but is also useful for patients with persistent
or relapsed hypercalcaemia of malignancy.[16]
* Haemodialysis or peritoneal dialysis may be relevant in patients with severe hypercalcaemia secondary to
renal failure.
Non-PTH-mediated hypercalcaemia
Treatment depends on the underlying condition. PTH-mediated hypercalcaemia[8]
* Asymptomatic patients may be treated conservatively with regular monitoring of bone density, renal
function and serum and urinary calcium levels.
* For symptomatic patients, dietary calcium should be reduced - eg, minimise the intake of dairy products and
leafy vegetables. This approach has been questioned for asymptomatic patients, in whom 1000-1200 mg
calcium daily has been recommended.[17]
* Bed-bound patients should be mobilised if possible. Symptomatic patients will respond well to having the
affected part of the parathyroid gland removed.
* There is no consensus on the operative treatment of asymptomatic patients. In general, it tends to be
reserved for patients who have impaired renal function, hypercalciuria, low bone mineral density or severe
hypercalcaemia

1081. A 30yo man from Australia returned from a business trip to Indonesia 6d ago presenting with complaints
of fever, joint and muscle ache and headache, in particular behind the eye for the past 2 days. What is the
most probable dx?
a. Malaria
b. Chicken pox
c. TB
d. Lymes disease
e. Dengue
a. Dengue
Fever, joint and muscle ache and headache behind eye, hx of travel to Indonesia all point towards diagnosis of
Dengue fever.
Symptoms
* Haemorrhagic fever syndromes begin with abrupt onset of fever and myalgia.
* Fever is associated with frontal or retro-orbital headache accompanied by onset of a generalised rash.
* Symptoms regress for a day or two but may recur, although fever is rarely as high as at the onset.
* Dengue fever cases experience severe bony and myalgic pain in legs, joints and lower back which may last
for weeks (hence, breakbone fever).
* Nausea, vomiting, cutaneous hyperaesthesia, taste disturbance and anorexia are common.
* Abdominal pain may occur and, if severe, suggests possible DHF.
1082. A lady came for OB GYN assessment unit with hx of 8wk pregnancy and bleeding per vagina for last 2
days. On bimanual exam, uterus =8wks in size. On speculum exam, cervical os is closed. How do you confirm
the viability of the fetus?
a. Transvaginal US
b. Serum BHCG
c. Urinary BHCG
d. Abdominal US
e. Per speculum exam
Transabdominal ultrasound will provide a panoramic view of the abdomen and pelvis and is noninvasive,
whereas transvaginal ultrasound provides a more limited pelvic view and requires insertion of a probe into the
vagina.
Transabdominal ultrasound cannot reliably diagnose pregnancies that are less than 6 weeks gestation.
Transvaginal ultrasound, by contrast, can detect pregnancies earlier, at approximately 4 to 5 weeks
gestation. Prompt diagnosis made possible by transvaginal ultrasound can, therefore, result in earlier
treatment.
Scenario is that of threatened miscarriage.
Serum and urine b hcg maybe raised a few days until after death of fetus, per speculum exam is not done for
miscarriage.
1083. A 24yo lady has been low after the death of her husband and had stopped contacting her family. She
was started on SSRI tx and starts feeling better after a few months. On discontinuing the meds she starts
feeling that she has developed cancer just like her husband. What is the most appropriate next step?
a. Continue SSRI
b. Add TCA
c. Neuropsychiatric analysis
d. CBT
e. Antipsychotics
Delusion of hypochondriasis is the diagnosis. Lady requires a neuropsychiatric analysis. All the other options
are wrong.
Hypochondriasis
This is a disorder where people fear that minor symptoms may be due to a serious disease. For example, that
a minor headache may be caused by a brain tumour, or a mild rash is the start of skin cancer. Even normal
bodily sensations such as 'tummy rumbling' may be thought of as a symptom of serious illness. People with
this disorder have many such fears and spend a lot of time thinking about their symptoms.
This disorder is similar to somatisation disorder. The difference is that people with hypochondriasis may
accept the symptoms are minor but believe or fear they are caused by some serious disease. Reassurance by a
doctor does not usually help, as people with hypochondriasis fear that the doctor has just not found the
serious disease.
1084. A 24yo male who is sexually active with other males with hx of discharge per urethra. Dx of chlamydia
has been made. What is the possible complication if left untreated?
a. Orchitis
b. Balanitis
c. Epididymo-orchitis
d. Acute abdomen
Chlamydia genital tract infection in men can lead to epididymo-orchitis and infertility if left untreated.
Other causes are e.coli, mumps, gonorrhea and tb
CHLAMYDIA is common in males 35 years and above. Doxycycline 100 mg bd for 10 days is treatment.
1085. A person doesnt go outside the home because he thinks that people will look at him and talk about
him. He finds it difficult to associate with his peers in a restaurant or under social settings. What is the most
likely dx?
a. Agoraphobia
b. GAD
c. Panic disorder
d. Adjustment disorder
e. Social phobia
Agoraphobia is fear of open spaces.
Panic disorder or panic attack consists of episodes in which patient feels as if they are going to die. Last for 10
mins.
Phobic disorders: anxiety in special situations.
Social phobia: phobia of situations in which a person may be closely minutely observed. Such as in this
scenario.
* Social anxiety is a fear of being around people and having to interact with them. Sufferers fear being
watched and criticised. Normal activities such as working, shopping, or speaking on the telephone are marked
by persistent feelings of anxiety and self-consciousness. They feel dread as a situation approaches and
afterwards they analyse or ruminate on how they could have done better. Hence, it may be seen as a
fundamentally normal response but exaggerated to the point of being pathological.
* Physical symptoms include trembling, blushing, sweating and palpitations.
* They often experience chronic insecurity about their relationships with others, excessive sensitivity to
criticism, and profound fears of being judged negatively, mocked, or rejected by others.
* There are two forms of the condition:
* Generalised social anxiety which affects most, if not all areas of life. This is the more common type and
affects around 70% of sufferers.
* Performance social anxiety, where these feelings only occur in a few specific situations such as public
speaking, eating in public or dealing with figures of authority.
* Treatment is CBT
Generalized anxiety disorder:
* Excessive anxiety and worry (apprehensive expectation), occurring more days than not for at least six
months, about a wide range of events or activities (such as work or school performance).
* The person finds it difficult to control the worry.
Adjustment disorder is a group of symptoms, such as stress, feeling sad or hopeless, and physical symptoms
that can occur after you go through a stressful life event. The symptoms occur because you are having a hard
time coping. Your reaction is stronger than expected for the type of event that occurred.

1086. A 63yo man presented with sudden onset of severe dyspnea, orthopnea, raised JVP and bilateral basal
crackles 3d after an episode of MI. A dx of acute congestive cardiac failure was made and IV furosemide was
started for this pt. What electrolyte abnormality is expected?
a. High Na+, Low K+
b. Low Na+, High K+
c. Low Na+, Low K+
d. High Na+, High K+
e. Low Na+, Normal K+
Furosemide is a loop diuretic which inhibits transport of na/k/2cl in the thick ascending limb of loop of henle.
Diuresis usually occurs 4 hr after a dose. Effects is a massive nacl excretion. Calcium secrestion also increased.
Used in treatment of edema in heart failure, ascites and pulmonary edema. Also used in severe
hypercalcemia. Hypokalemic metabolic alkalosis, hypovolemia, ototoxicity and allergic reactions are side
effects.
1087. A 70yo hypertensive white british man on thiazide diuretics needs a 2nd drug to control his BP. Which
one of the following is the best choice for him?
a. Amlodipine (CCB)
b. Enapril (ACEi)
c. Propranolol (BB)
d. Increase dose of diuretic
e. Prazocin (Alpha blocker)
Key is b.
Hypertension management
If age is greater than 55, or black patient of any age: ca channel blocker or thiazide (C/D)
If age is less than 55: ace inhibitor(A)
B blocker to be considerd in young, pregnancy or if increased sympathetic drive
Combination therapy
Ace inhibitor plus ca channel blocker or thiazide(A+C/D)
3 drugs needed: ace inhibitor plus ca channel blocker plus diuretic (A+C+D)
4th drug needed. Add higher dose diuretic or b blocker or alpha blocker(A+C+D+B)

1088. A 74yo lady who has had a stroke in the past has an indwelling catheter for 10m. She presents with
bluish-purple discoloration of the catheter bag. What is the most likely explanation for this?
a. Normal change
b. Catheter degradation
c. Acidic urine
d. Alkaline urine
e. Bacterial colonization of the urinary tract
Explained in Samson notes. Renal section.

1089. A 62yo man has slow palpitations and the following ECG. What is the most likely dx?
a. Sinus bradycardia
b. 1st degree heart block
c. Mobitz type 1 block
d. Mobitz type 2 block
e. Complete heart block
Progressive prolongation of PR interval followed by missed beat is indicative of MOBITZ type 1 block.
Bradycardia is to be treated with atropine. If not controlled, then temporary pacing may be required.
1090. A 29yo woman presents with lid lag, lid retraction and diplopia. What is the most appropriate next step?
a. TFT
b. Tensilon test
c. Fundoscopy
d. Autoantibodies
e. EMG
Scenario suggestive of Graves disease, hyperthyroidism. Graves' disease:[9]
* This is the most common cause of hyperthyroidism and has an autoimmune basis. It is mediated by B and T
lymphocytes, characterised also by the presence of thyroid-
stimulating immunoglobulins (TSIs). These are directed at four different thyroid antigens:
* Thyroglobulin.
* Thyroid peroxidase (or antimicrosomal antibodies).
* Sodium-iodide symporter.
* TSH receptor.
* The condition is characterised by a small to moderate diffuse, firm goitre with 50% of these showing
ophthalmopathy.
* There may be a personal or family history of autoimmune disease.
* <5% have pretibial myxoedema called thyroid dermopathy (as can occur anywhere, particularly following
trauma). This is usually associated with moderate to severe ophthalmopathy. 10-20% have clubbing (thyroid
acropathy). Thyroid dermopathy usually appears as non-pitting plaques with pink/purple colour. There are
also nodular and generalised forms.
* There may also be lymphoid hyperplasia including splenomegaly and an enlarged thymus. * Associated with
other autoimmune conditions - eg, pernicious anaemia, type 1diabetes mellitus.
Investigations * Thyroid function tests (TFTs): serum TSH can exclude primary thyrotoxicosis. Confirm the
diagnosis with free T4 levels. If TSH is suppressed but free T4 levels are normal, then if not previously supplied,
free T3 level is needed (T3 toxicosis occurs in 5% of patients).
* Autoantibodies - these are most commonly seen in Graves' disease:
* Antimicrosomal antibodies - against thyroid peroxidase.
* Antithyroglobulin antibodies.
* TSH-receptor antibodies which are commonly present in Graves' disease but are not routinely measured.
* TSI if elevated helps to establish a diagnosis of Graves' disease.
1091. A 41yo man presents with longstanding foul smelling ear discharge and progressive hearing loss.
Otoscopy showed perforation of the pars flaccida and a mass in the upper part of the middle ear. What is the
most likely dx?
a. ASOM
b. CSOM
c. Acquired cholesteatoma
d. Congenital cholesteatoma
e. Barotrauma
Long standing foul smelling discharge and progressive hearing loss indicates CSOM, but otoscopic findings of
perforation and mass point towards acquired choleastoma which can also be a complication of asom.
Secondary acquired cholesteatoma This arises as a result of insult to the tympanic membrane, such as
perforation secondary to acute otitis media or trauma, or due to surgical manipulation of the drum. Squamous
epithelium may be inadvertently implanted by the insult so triggering the process of cellular growth resulting
in cholesteatoma formation.
* Frequent or unremitting painless otorrhoea which may be foul-smelling.
* Recurrent otitis, poorly responsive to antibiotic treatment.
* Progressive, unilateral conductive hearing loss.
* Tympanic membrane perforation (~90% of cases) or retracted tympanum.
* The only finding may be a pus-filled canal with granulation tissue
CT imaging is the investigation of choice if there is a need for assessment of the extent of the lesion and to
assess subtle bony defects.
Treatment
Surgery:
Open technique: tympanomastoidectomy
Closed technique : tympanoplasty
1092. A 9yo child presented with a rash on his skin which didnt respond to antibacterial ointment. What med
should be added next?
a. Corticosteroid
b. Antifungal
c. Emollient
d. Permethrin
e. Coal tar
1093. A young boy has a hx of epistaxis. CBC=normal, except APTT=47s. What is the most likely dx?
a. Hemophilia
b. ITP
c. Sickle cell
d. HUS
e. Thalassaemia
Epistaxis and prolonged APTT point toward intrinsic pathway defect , so hemophilia is the likely key.
Hemophilia A IS A FACTOR 8 DEFiciency inherited in a x linked recessive pattern. Presentation early in life or
after trauma/surgery. Bleeding into joints leading to crippling arthropathies and muscle causing hematomas
with nerve compression and compartment syndrome. Diagnosed by inc APTT and dec factor 8 assay.
Treatment with desmopressin which inc factor 8 levels and recombinant factor 8 in major or life threatening
bleeds.
Hemophilia B is a similar condition arising due to deficiency of factor 9. Also inherited in x linked recessive
pattern. Behaves clinically like hemophilia A.

1094. A 29yo young man presents with complaints of recurrent attacks of diarrhea. He says his stools contain
blood and mucus. Sometimes he has low grade fever. What is the most appropriate inv for his condition?
a. Stool culture
b. Plain abdominal XR
c. Per rectal exam
d. Barium enema
SCENARIO suggestive of Ulcerative Colitis:
Per rectal exam will not provide us with any evidence. Plain abdominal x ray will be useful in an acute
exacerbation of disease. Barium enema is the key but is not an investigation of choice but can be used in acute
setting to diagnose.. Here stool culture seems appropriate also so we can rule out an infectious cause to the
condition.
The diagnosis should be made on the basis of clinical suspicion supported by appropriate macroscopic findings
on sigmoidoscopy or colonoscopy, typical histological findings on biopsy and negative stool examinations for
infectious agents.
* Initial investigations include:
* FBC, renal function and electrolytes, LFTs, ESR and CRP.
* Low magnesium and serum albumin levels are sometimes found in ulcerative colitis. * Stool culture,
including ova, cysts and parasites and also Clostridium difficile toxin.
* Serological markers have been developed to differentiate ulcerative colitis from Crohn's disease. p-ANCA is
more commonly associated with ulcerative colitis, whilst ASCA is more commonly associated with Crohn's
disease.
* Abdominal imaging: this is essential in the initial assessment of patients with suspected ulcerative colitis, to
exclude toxic dilatation and perforation. It may also help to assess disease extent or identify proximal
constipation. In milder forms, ultrasound, CT, MRI and radionuclide scanning may all be contributory.
* Sigmoidoscopy and rectal biopsy: for all patients presenting with diarrhoea, rigid sigmoidoscopy should be
performed unless there are immediate plans to perform flexible sigmoidoscopy.
* Colonoscopy:
* This is usually preferable to flexible sigmoidoscopy, because the extent of disease can be assessed but, in
moderate-to-severe disease there is a higher risk of bowel perforation and flexible sigmoidoscopy is safer
1095. A 26yo young man presents with chx of passing loose stools for the past 2m. He says his stools contain
blood and mucus and are a/w abdominal pain. He undergoes a colonscopy after which he was started on tx.
What is the most appropriate tx for his condition?
a. Mesalazine
b. Corticosteroids
c. Infliximab
d. Cyclosporine
The scenario is that of Ulcerative colitis.
MANAGEMENT
Aminosalicylates: Mesalazine - 5-aminosalicylic acid (5-ASA) - is now the treatment of choice for induction and
maintenance of remission of mild-to-moderate ulcerative colitis.
Corticosteroids are used to induce remission in relapses of ulcerative colitis. They have no role in maintenance
therapy.
Thiopurines: Azathioprine and its active metabolite 6-mercaptopurine may be used when:
* Patients are intolerant to corticosteroids.
* Patients need two or more corticosteroid courses in a calendar year.
* Disease relapses when the dose of prednisolone is less than 15 mg a day.
* Disease relapses within six weeks of stopping steroids.
Ciclosporin: This is an effective salvage therapy for patients with severe refractory colitis and it has a rapid
onset of action.
Infliximab is recommended as an option for the treatment of acute exacerbations of severely active ulcerative
colitis only in patients in whom ciclosporin is contra-indicated or clinically inappropriate Up to 30% of patients
will ultimately require colectomy for ulcerative colitis.[3]
* Colectomy is an option for patients who do not respond to, or are intolerant of, medical treatment, or in
those with complications such as colorectal neoplasia.[15] * Because ulcerative colitis is confined to the
colorectum, colectomy is curative.[15]
* The usual procedure is a restorative proctocolectomy with ileal pouch-anal anastomosis.[15]
1096. A 52 yo male with poorly controlled DM has now presented to his GP with pain in the ear. Exam: skin
around the ear is black in color and there was foul smelling discharge from the ear. Pt also had conductive
hearing loss. What is the most probable dx?
a. Carbuncle
b. Folliculitis
c. Malignant OE
d. Cholesteatoma
e. Furuncle
Poorly controlled diabetes in this man points towards the diagnosis of Malignant Otitis Externa. It occurs in the
age group 45 to 75. It is aggressive life threatening infection of the external year that can lead to temporal and
mastoid destruction and base of skull osteomyelitis. Also facial
nerve palsy. Diabetes and immunosuppression are risk factors. Usual cause is pseudomonas, klebsiella and
proteus may be involved as well. There maybe hearing loss in otitis externa. Treatment is by surgical
debridement, systemic and topical antibiotics.
1097. A 55yo male has been admitted for elective herniorrhaphy. Which among the following can be the
reason to delay his surgery?
a. Controlled asthma
b. Controlled A-fib
c. DVT 2yrs ago
d. DBP 90 mmHg
e. MI 2m ago
Controlled asthma and controlled afib and dvt 2 years back are not contraindications. MI 2 months ago seems
appropriate but new guidelines suggest even a period of two months after MI is enough. Otherwise atleast a
period of 3 months is needed.
1098. A 21yo female in her first pregnancy at 38wks was brought to the ED with generalized tonic clonic
seizure. IV MgSO4 was given but fits was not controlled. She is having fits again. What is the single most imp
immediate management of this pt?
a. IV MgSO4
b. IV diazepam
c. Immediate C-section
d. IV phenytoin
e. MgSO4 bolus
f. IV lorezepam
Treatment of Eclampsia
Treat first seizure with 4g MGSO4 in 100 ml NS IVI over 5 min and mantainenece IVI of 1g per hr for 24 hr.
If recurrent, give 2g ivi magnesium sulphate over 5 mins.
Use diazepam once if fits continue.
According to this regimen, the correct answer
1099. A 24yo lady with BMI=30 complains of facial hair growth and hx of amenorrhea. FSH=10.9,
prolactin=400IU, estradiol=177.8mmol/l, progesterone=normal, LH=33.2. What is the most probable dx?
a. PCOS
b. Pregnancy
c. Cushings disease
d. CAH
e. POF
Young lady, obese, with facial hair and history of amenorrhea. Looks like PCOS as cause seems to be ovarian
indicated by a raised LH and FSH, and normal prolactin.
It encompasses a syndrome of polycystic ovaries, in association with systemic symptoms causing reproductive,
metabolic and psychological disturbances. These most commonly present with infertility, amenorrhoea, acne
or hirsutism.
The patient often presents in the peripubertal period through to her mid 20's.
Symptoms:
Oligomenorrhoea (defined as <9 periods per year)
Infertility or subfertility, Acne, Hirsutism, Alopecia, Obesity or difficulty losing weight, Psychological
symptoms[5] - mood swings, depression, anxiety, poor self-esteem Sleep apnoea
Differential diagnosis * Thyroid disorder (particularly hypothyroidism),Hyperprolactinaemia,Cushing's
syndrome,Late-onset congenital adrenal hyperplasia (very rare),Androgen-secreting ovarian or adrenal
tumours,Ovarian hyperthecosis (signs of virilism and biochemical androgen excess are much more prominent
in the latter three)
Investigations
* This may show LH elevated, LH:FSH ratio increased (>2), with FSH normalFree testosterone levels may be
raised, but if total testosterone is >5 nmol/L, exclude androgen-secreting tumours and congenital adrenal
hyperplasia.
* Laparoscopy or ultrasound shows characteristic ovaries (the average volume is three times that of normal
ovaries .Other blood tests, where indicated from the clinical picture, to exclude other potential causes - eg,
TFT (thyroid dysfunction), 17-hydroxyprogesterone levels (congenital adrenal hyperplasia ), prolactin
(hyperprolactinaemia), DHEA-S and free androgen index (androgen-secreting tumours), and 24-hour urinary
cortisol (Cushing's syndrome).
* Fasting glucose
* Fasting lipid levels should be checked.
Diagnosis Two of the three following criteria are diagnostic of the condition (Rotterdam criteria):[6]
* Polycystic ovaries (either 12 or more peripheral follicles or increased ovarian volume (greater than 10 cm3)
* Oligo-ovulation or anovulation
* Clinical and/or biochemical signs of hyperandrogenism
MANAGEMENT
They should be advised on weight control and exercise.] It has been shown to improve ovulation, pregnancy
rate
Pharmacological treatment
There is no treatment which reverses the hormonal disturbances of PCOS and treats all clinical features, so
medical management is targeted at individual symptoms, and only in association with lifestyle changes.
For women not planning pregnancy
* Co-cyprindrol: is licensed for treating hirsutism and acne, although not specifically in PCOS. Combined oral
contraceptive pill (COCP): is also used to control menstrual irregularity.
* Metformin: has been increasingly used off-licence for PCOS; however, a Cochrane review showed it to be
less effective than the COCP for menstrual irregularity, hirsutism and acne,[13] and the National Institute for
Health and Care Excellence (NICE) Evidence Summary suggests its side-effects and cost outweigh its benefits
and any, as yet unproven, long-term health benefits.[14]
* Eflornithine: may be used for hirsutism, as can cosmetic treatments (electrolysis, laser, waxing, bleaching).
* Orlistat: can help with weight loss in women with PCOS and may improve insulin sensitivity[15]
For women wishing to conceive and presenting with infertility
2013 NICE guidelines advise (after weight loss where indicated, and a full fertility work-up) women should be
treated with clomifene, metformin or a combination of the two.[10] (A Cochrane review in 2012, however,
found no benefit in live birth rates from the use of metformin or other insulin-sensitising drugs, although it did
improve numbers of pregnancies.[16])
Complications
* Oligomenorrhoea or amenorrhoea are known to predispose to endometrial hyperplasia and endometrial
cancer in untreated cases. It has been suggested that women with PCOS may have a higher cardiovascular risk
than weight-matched controls.
* Women presenting with PCOS, particularly if they are obese (BMI greater than 30), have a strong family
history of type 2 diabetes or are over the age of 40 years, are
atincreased risk of type 2 diabetes and should be offered screening. The risk may be as high as 10-20%.
* Women diagnosed with PCOS (or their partners) should be asked about snoring and daytime
fatigue/somnolence and informed of the possible risk of sleep apnoea. They should be offered investigation
and treatment when necessary.[20]
* Complications in pregnancy: there is a higher risk of gestational diabetes in women with PCOS, which may be
more than double..
1100. A 17yo girl with a lump in her breast was seen in the clinic. Exam: the lump was free and mobile and not
attached to the skin. Her mother wants further tests done. What should be the next step?
a. CT
b. US breast
c. Punch biopsy
d. Reassure and send home
e. Stereotactic biopsy
Diagnosis is that of a Fibroadenoma breast. It is a benign tumour common in young women. Diagnosis is made
and confirmed on ultrasound Fibroadenoma This is a benign tumour that is common in young women, mostly
aged under 40 years. It is composed of stromal and epithelial elements and probably represents increased
sensitivity to oestrogens:
* Complex and multiple fibroadenomas are associated with a two-fold increase in the risk of breast cancer.
* They represent a hyperplasia or proliferation of a single terminal duct unit.
* Most stop growing at about 2 or 3 cm, but they can enlarge rather further.
* About 10% disappear each year.
* They tend to regress after the menopause.
* They occur in about half of women who receive ciclosporin after renal transplant.
* They are the most common tumour of the breast in those under 30 years old, but overall they are second to
breast cancer.
* Juvenile fibroadenomas can occur in teenage girls.
Both mammography and ultrasound may be used to examine the lump:
* Ultrasound tends to be preferred in younger women with dense breasts, as mammograms are more difficult
to interpret in this group. Routine mammography, as a population screening tool, is not performed below the
age of 50 years.
* Imaging studies may fail to give a firm diagnosis and biopsy or excision may be required for peace of mind of
both the patient and doctor.
* If there is confidence in the diagnosis then inactivity may lead to spontaneous regression, but the patient
must be advised to check the lump regularly and to return if it starts to enlarge.
* Assessment often includes examination, imaging studies and fine-needle aspiration.

1100. A 17yo girl with a lump in her breast was seen in the clinic. Exam: the lump was free and mobile

and not attached to the skin. Her mother wants further tests done. What should be the next

step?

a. CT

b. US breast

c. Punch biopsy

d. Reassure and send home

e. Stereotactic biopsy

key: B

cause: young girl , lump mobile and free >>> fibroadenoma

Triple assesment for breast lump

clinical examination

imaging :mamography if older than 35 years or u/s for younger patients

FNAC

THIS is a case of fibroadenoma and examination was done so the next step is imaging (pt is 17 so u/s)

Fibroadenoma

This is a benign tumour that is common in young women, mostly aged under 40 years.

They are the most common tumour of the breast in those under 30 years old, but overall they are second to
breast cancer.

Assessment often includes examination, imaging studies and fine-needle aspiration.

Ultrasound >>> in younger women with dense breasts, as mammograms are more difficult to interpret in this
group.

Routine mammography, as a population screening tool, is not performed below the age of 50 years.

- The initial examination depends upon the age of the patient. Ultrasound for <35 years old; Mammography
and then Ultrasound for >35 years old.
- Ultrasound is best for dense breast tissue, whereas Mammography is best for less dense breast tissue; eg,
after menopause.

- CT is not done for breast lesions.

- Stereotactic Biopsy is the investigation of choice only when there are no palpable masses.

- You cannot reassure and send home as it might be a Fibroadenoma since the patient is young.

1101. A lady comes with a missing IUCD thread. Her LMP was 2wks ago. What is the single most

appropriate next step in management?

a. Abdominal US

b. Prescribe contraceptives

c. CT

d. Serum BHCG

e. Vaginal exam

key: A

cause: in case of lost thread we advise the pt with extra contraception like condom then we start managing
the case by : pregnancy test but in this case her LMP was two weeks ago so no need for than , so the next step
is to locate the IUCD using imaging studies ( us first then xray)

lost thread : OCS pg 298

the IUCD may have been expelled, so advise extra contaception and exclude pregnancy

seek coil on u/s ; if missing arrange x-ray to exclude extra-uterine coils ( surgical retrieval is advised)

- In case of a lost thread, a number of measures should be taken. They are:

- If threads are not visible, or if they are but the stem of the device is palpable, the woman should be advised
to use condoms or abstain from intercourse until the site of the device (if present) can be determined.

- Perform a speculum examination to ensure the device is not in the posterior fornix.

- Determine whether the woman is already pregnant.

- With consent, explore the lower part of the endocervical canal with narrow artery forceps: threads which
have been drawn a little way up are usually found by this method.

- An experienced operator may, after appropriate analgesia (eg, mefenamic acid 500 mg) explore the uterine
cavity with a retriever hook.

- Hormonal emergency contraception may be indicated.

- Ultrasound should be arranged to locate the device.


- If ultrasound does not locate the device and there is no definite history of expulsion then abdominal X-ray
should be performed to look for an extrauterine device.

- Expulsion should not otherwise be assumed.

- Hysteroscopy can be helpful if ultrasound is equivocal.

- Surgical retrieval of an extrauterine device is advised.

- The question asks for the most appropriate next step in the management of this case. That step would be to
exclude pregnancy via an abdominal ultrasound. IUCDs increase the risk of ectopic pregnancy, for which again,
the next step would be an abdominal ultrasound.

1102. A 32yo woman presents with hx of lower abdominal pain and vaginal discharge. She had her

menses 4wk ago. She has a temp of 38.6C. What is the most suitable dx?

a. Acute appendicitis >>> (nausia,vomiting,RIF pain)

b. Acute PID

c. Endometriosis>>> dysparaunia

d. Ectopic pregnancy>>> missed period,abnormal vaginal bleeding

e. UTI>>>urinary symptoms

key: B

cause: clinical signs and symptoms fit ( fever above 38 + bilatral lower abdominal pain + vaginal discharge)

PID

Diagnosis of acute PID made only on clinical signs

signs & symptoms

Bilateral lower abdominal pain.

Deep dyspareunia.

Abnormal vaginal bleeding (postcoital, intermenstrual or menorrhagia).

Vaginal or cervical discharge that is purulent.

Fever above 38C

Investigations:

Pregnancy test (pregnant women with PID should be admitted; ectopic pregnancy may be confused
with PID).

Cervical swabs for chlamydia and gonorrhoea

Endocervical swabs for C. trachomatis and N. gonorrhoeae


Treatment:

Mild or moderate disease can be managed in primary care or outpatients, whereas clinically severe
disease requires hospital admission for intravenous (IV) antibiotics.

The current outpatient treatment recommendation is ceftriaxone 500 mg as a single (IM) dose,
followed by doxycycline 100 mg orally twice daily and metronidazole 400 mg twice daily for 14 days

Initial treatment is with doxycycline, single-dose IV ceftriaxone and IV metronidazole, then change
to oral doxycycline and metronidazole to complete 14 days of treatment.

- Symptoms of PID

- Bilateral Lower/Pelvic abdominal pain that ranges from mild to severe.

- Pain during sex

- Abnormal vaginal discharge

- Abnormal vaginal bleeding (1 in 4 cases)

- Risk Factors

- A recent change of sexual partner. The risk goes up with the number of partners.

- A previous episode of PID or sexually transmitted disease.

- A recent abortion.

- A recent operation or procedure on the womb (uterus).

- A contraceptive coil inserted recently.

- Tests to be done

- Endocervical Swab is the investigation of choice. High vaginal swab can also be taken.

- If the above doesnt show anything, an U/S can be done to look for inflamed fallopian tubes.

- Complications

- Infertility

- Ectopic Pregnancy

- Miscarriage and Still Births

- Persistent pain (including pain during sex)

- Reiters Syndrome

- Abscess formation

- Treatment

- Antibiotics
- The partner must also be treated

1103. A 40yo female was on COCP which she stopped 6m ago. But she has not had her periods since

then. Labs: FSH=22, LH=24, prolactin=700, estradiol=80. What is the most appropriate dx?

a. Hypothalamic amenorrhea

b. Post pill amenorrhea

c. Prolactinoma

d. Pregnancy

e. Premature ovarian failure

key: E

reason: this is a case of secondary amenorrhea with elevated gonadotropins indicating ovaian failure and the
patient is 40 years so it's premature ovarian failure

causes of secondary amenorrhea:

with no androgen excess:

1-pregnancy(the most common cause of secondary amenorrhea),lactation,menopause.

2-premature ovarian failure:

this is a poorly understood condition that may represent an autoimmune phenomenon. It can also follow
radiotherapy or chemotherapy. With all these causes, menstruation and fertility can sometimes resume
spontaneously. Ovarian failure will cause elevation of gonadotrophins and so hot flushes are likely. Premature
menopause is defined as occurring before the age of 40.

3-depot and implant contraception

4-cevical stenosis and intrauterine adhessins >>> Asherman's syndrome

5-wt loss >>> especially if rapid

6-pitutary disease and hyperprolactinaemia

7-sheehan syndrome >>> the pituitary may be damaged by tumours, trauma, cranial irradiation, sarcoidosis or
tuberculosis

8- post pill amenorrhea

with androgen excess :

PCOS

Premature Ovarian Failure Secondary Amenorrhea (E)


- Causes

- Hyperprolactinemia

- PCOS

- Premature Ovarian Failure

- Post-pill Amenorrhea

- Ashermans Syndrome

- Investigations

- Pregnancy test (if appropriate)

- Follicle-stimulating hormone (FSH) and luteinising hormone (LH)

- PRL

- Total testosterone and sex hormone-binding globulin

- TSH

- A pelvic ultrasound may be useful in patients with suspected PCOS

- Treatment

Treatment is related to cause. Premature Ovarian Failure is irreversible but hormone replacement is necessary
for controlling the symptoms of Estrogen deficiency and protection against Osteoporosis. Pregnancy can be
achieved by oocyte donation or in vitro fertilization techniques.

- What to look for in the question

- Secondary amenorrhea before or at the age of 40 (The age is 40 in this question)

- A raised Gonadotropin level

1104. A 25yo woman presents with a single lump in the breast and axilla. The lump is mobile and hard

in consistency. The US, mammogram and FNA turn out to be normal. What is the most

appropriate inv to confirm the dx?

a. FNAC

b. MRI

c. Punch biopsy

d. Genetic testing and counselling

e. Core biopsy

key: E
reason : in palpable mass we do triple assessment : clinical examination,imaging,FNAC >>>if not conclusive we
do core biopsy then excional biopsy if still non conclusive

Breast lump triple assessment

examination

imaging: mamogaraphy,u/s

biopsy

* non palpable lesion:

1-core biopsy(image -guided)

2- open biopsy(needle localisation >>> radio opaque needles are used to guide the boipsy0

*palpable lesion:

1-FNAC

2-core biopsy

3-excision biopsy (entire lesion is removed)

4-incision biopsy (part of the lesion)

All patients with breast lumps must undergo triple assessments.

1st Assessment: Clinical examination of the breast including axillary lymph nodes

2nd Assessment: Imaging

- U/S for <35 years old

Mammography and THEN U/S for >35 years old patient.

3rd Assessment: Cytology

- If it is a cyst, perform FNAC

-Clear Fluid? Reassure the patient

- Blood-stained aspirate? Send to lab for Cytology

- Clear fluid but residual mass? Perform core biopsy

In this particular question:

The patient is a young woman with a single lump in the breast and the axilla. The lump is hard and mobile
which points towards Fibroadenoma but a swollen lymph node in the axilla points towards a carcinoma. To
find out which one it is, we need a core biopsy since an ultrasound, a Mammogram and an FNAC turned out to
be normal. The usual typical order of investigations in such cases is:

- Ultrasound/ Mammogram depending upon the age.

- Mammogram
- FNAC

- Core Biopsy

1105. A 37yo lady stopped taking COCP 18m ago and she had amenorrhea for 12m duration. Labs:

FSH=8, LH=7, prolactin=400, estradiol=500. What is the cause?

a. Hypothalamic amenorrhea

b. PCOS

c. Prolactinoma

d. Post pill amenorrhea

e. POF

key:D

reason :this is a case of secondary amenorrhea following COCP use,,,

decreased gonadotropins >>> so it's not POF,,

and there are no symptoms of increased androgens>>> so it's not PCOS ,,

NO increased prolactin >>>so not prolactinoma,,,

the history of amenorrhea after COCP use + decreased gonadotropins fit with >>> post pill amenorrhea

Post-pill amenorrhoea

this is when stopping oral contraceptives does not lead to a resumption of a normal menstrual cycle. It usually
settles spontaneously in around three months but, if not, it requires investigation. The condition is probably
not a true entity but the cause of amenorrhoea started whilst taking the contraceptives that induced an
artificial cycle until they were stopped.

Post pill amenorrhea is described as the loss of menstrual periods for at least 6 months after stopping birth
control pills. The incidence of post-pill amenorrhea ranges from 0.2% to 3%.

- Cause of Post-pill Amenorrhea

Post-pill amenorrhea is believed to be due to suppression of the pituitary gland by the birth control pills.

- Investigations

- The diagnosis of post-pill amenorrhea is usually made when there is loss of periods after a prolonged history
of taking birth control pills. - Ultrasonography will reveal ovaries with no signs of developing follicles and
ovulation even after having stopped the pills for 6 months.

- Blood tests showing a low level of FSH, LH and estrogen is usually sufficient to confirm the diagnosis.

- Treatment
- The first line of treatment in case of post-pill amenorrhea is waiting for a spontaneous remission of the
amenorrhea and a spontaneous occurrence of periods.

- The time limit is usually six months. But if the woman is anxious to get her periods, active treatment may be
started after waiting for only three months.

- The standard treatment of post-pill amenorrhea is by stimulating the pituitary to produce FSH and LH. This is
done by the drug clomiphene citrate.

1106. A lady with a firm smooth breast lump in outer quadrant had a FNAC done. Results showed

borderline benign changes. She also has a fam hx of breast cancer. What is the your next?

a. Mammography

b. US

c. Core biopsy

d. Genetic testing and counselling

e. Punch biopsy

key : D

reason: assessment of the case is complete (borderline benign change),,,next step is genetic testing and
counselling.

breast lump >>>discussed earlier

In this question a lady underwent an FNAC and the results have showed benign breast changes. She also has a
risk for developing breast cancer because of her family history. All options but one are investigations, which
we dont need at this point because we got all we could get from the investigations. What needs to be done
now is to assess the risk of breast cancer in this patient. Also, the patient needs to be counselled about the
disease. Therefore, we should for genetic testing and counselling.

1107. A pt presents with mild dyskaryosis. 1y ago smear was normal. What is the most appropriate

next step?

a. Cauterization

b. Repeat smear

c. Swab and culture

d. Cone biopsy

e. Colposcopy

key: E

Cervical screening
Cervical cancer is the third or fourth most common female malignancy worldwide

The screening process is done using LBC (liquid based cytology) or older method (PAP)

Interpreting smear results:

Cells are analysed to look for abnormalities in the appearance of the nucleus and other aspects of cell
morphology (dyskaryosis)

Negative (normal) >>> treat incidental findings eg,infection & recall as appropriate

inadequate : insufficient or unsuitable material sampled >>>>Repeat sample immediately after


treating any infection,,Repeat sample as soon as convenient if technically inadequate,,,if persistent
(three inadequate samples), advise assessment by colposcopy

Borderline changes and mild dyskaryosis>>>> HPV testing ,,if positive >> colopscopy,,

if negative >>> normal recall

if unreliable >>> repeat HPV in six month and if +ve >> colopscopy

moderate and severe dyskaryosis >>>> refer to colopscopy

Cervical Cancer When to refer for Colposcopy (E)

- Any smear showing mild, moderate or severe Dyskaryosis

- Any suggestion of malignancy

- 3 consecutive inflammatory smears

- 2 consecutive atypical smears

- 3 consecutive borderline smears

- 3 consecutive inadequate smears

- Post-coital bleeding

1108. An African lady presents with heavy but regular periods. Her uterine size correlates to 14wks

pregnancy. What is the most appropriate dx?

a. Blood dyscrasia

b. Hematoma

c. Fibroids

d. Adenomyosis

e. Incomplete abortion

key: C
reason: fibroids are three times more common in African americans than white american+ enlarged uterus +
heavy menses

Fibroids

-age :commonly 30-50 ys

- more common in : obese, early menarche , African Americans ,

-protective factors: exercise, increased parity , may be smoking

types>> submucosal,,intramural,,subserosal

*presentation:

.. excessive or prolonged bleeding,,pelvic pain

..enlarged uterus>>pressure symptoms>>heaviness,constipation,urinary symptoms

..submucosal>>infertility,miscarriage,intermenstrual bleeding

..during pregnancy>>red degeneration

Investigations:

pelvic u/s

TVUS >> more accurate

saline infusion u/s is superior to TVUS and hysteroscopy in detection of submucosal fibroids

MRI>>if myomectomy is considered and u/s is not conclusive

Endometrial sampling>>>in abnormal uterine bleeding

combination LDH, LDH isoenzyme 3 and gadolinium-enhanced MRI is highly accurate in diagnosing
leiomyosarcoma pre-operatively, if sarcoma is suspected clinically

Treatment :

only required if symptomatic

Medical:

NSAIDs (ibuprofen)

Antifibrinolytic (tranexamic acid)

Combined hormonal contraception ( CHC)

Merina,,danazol,,GnRH agonists,,Mifepristone,,ulipristal acetate,,aromatase inhibitor>>letrozole

Surgical:

indications>>failure of medical treatment,,infertility,,pressure symptoms,,excessively enlarged


uterine size
*myomectomy>> for those who want to maintain fertility

abdominal myomectomy : safe alternative to hysterectomy + risk of blood loss

laparoscopic myomectomy: less pain, shorter hospital stay and reduced recovery time,subserous
fibroid

hysteroscopic myomectomy : established surgical procedure for women with submucosal fibroids
and excessive uterine bleeding, infertility or repeated miscarriages.

laparoscopic laser myomectomy >> not recommended by NICE

* TOTAL hysterectomy

laparotomy

laparoscopic assisted hysterectomy >> urinary tract injury+severe bleeding

* uterine artery embolization UAE

*MRI-guided transcutaneous focused ultrasound

- Notice that the question mentions an African female with regular but heavy periods. This is because fibroids
are mostly common in Afro-Caribbean females.

- Fibroids are responsive to estrogen and therefore increase in size, which in turn increases the size of the
uterus. E.g., in this patient, her uterine size correlates to 14 weeks pregnancy.

- Other symptoms

- Pelvic pain (Compression on to adjacent structures)

- Infertility/Recurrent Miscarriages

- Pelvic Mass

- The investigation of choice is an U/S

- Management

- Mirena Coil is the first choice if the fibroids are not big enough to restrict its insertion.

- If < 3 cm

- Trial of pharmacologic treatment first (Tranexamic Acid) first

- If it fails and uterus is not bigger than 10-week pregnancy, do endometrial ablation

- If the above fails, perform a hysterectomy

- If > 3 cm and wishes to retain uterus and/or wants to avoid surgery

- Go for Uterine Artery Embolization

- If > 3 cm and wishes to retain uterus, go for a hysteroscopic myomectomy or a myomectomy


1109. A 29yo at 38wks GA presents with a 2h hx of constant abdominal pain. She then passes 100 ml of

blood per vagina. What is the next appropriate inv?

a. USS

b. CTG

c. Clotting screen

d. Hgb

e. Kleihauer Betke test

key: A

reason : in case of antenatal bleeding we do u/s to investigate the cause of bleeding

Antenatal bleeding

never perform PV >> might increase bleeding

dangerous causes: Abruption,placenta praevia,vasa praevia

other caused:circumvallate placenta,placental sinuses cervical polyps,erosions and


carcinoma,cervicitis,vaginitis,vulvar varicosities

placental abruption>>>> part of placenta becomes detached ,, the outcome depends on the
amount of detachment and blood loss,,bleeding might be concealed

associations>>> preeclampsia,smoking,cocaine,IUGR,PROM,abdominal trauma,multiple


pregnancy,polyhydramnios,increased maternal age,non vertex presentation,assisted reproductive
techniques

placenta praevia>>>placenta is in the lower uterine symptoms,,bleeding is always revealed

Management:

in severe bleeding>>> admission,,IV lines , O2 mask,, blood transfusion if in shock,,catheterize,,if


severe bleeding >> CS

in mild cases>>> IVI,,Hb , cross match,check vitals >> establish dx by u/s >>if placenta praevia >>>cs

Bleeding During Third Trimester APH (A)

- Bleeding during the third trimester is either painful or painless bleeding.

- Painful bleeding points to Placental Abruption while painless bleeding points to Placenta Previa

- In this case, the patient is experiencing painful vaginal bleeding in the third trimester, so this appears to be
the case of Placental Abruption

- The investigation of choice in this case is an Ultrasound

- Risk Factors for Abruption

- Hypertension
- Smoking

- Multiple Pregnancy

- Cocaine/Amphetamine Use

- Increased Maternal Age

- Trauma to the abdomen

- Polyhydramnios

- Investigation

- Diagnosis is clinical but U/S is done to exclude Placenta Praevia and to check the well-being of the baby.

- Management

- Always admit the patient to hospital for assessment and management. Phone 999/112/911 if there are any
major concerns regarding maternal or fetal well-being.

- The mainstays of management are resuscitation and accurate diagnosis of the underlying cause.

- Severe bleeding or fetal distress: urgent delivery of the baby, irrespective of gestational age.

- Admit to hospital, even if bleeding is only a very small amount. There may be a large amount of concealed
bleeding with only a small amount of revealed vaginal bleeding.

- No vaginal examination should be attempted, at least until a placenta praevia is excluded by ultrasound. It
may initiate torrential bleeding from a placenta praevia.

- Resuscitation can be inadequate because of underestimation of blood loss and misleading maternal
response. A young woman may maintain a normal blood pressure until sudden and catastrophic
decompensation occurs.

- Take blood for FBC and clotting studies. Crossmatch, as heavy loss may require transfusion.

- Gentle palpation of the abdomen to determine the gestational age of the fetus, presentation and position.

- Fetal monitoring.

- Arrange urgent ultrasound.

- With every episode of bleeding, a rhesus-negative woman should have a Kleihauer test and be given
prophylactic anti-D immunoglobulin

- Complications

- Premature Labour

- DIC

- Renal Tubular Necrosis

- PPH

- Placenta Accreta
- Points to look for diagnosis of Abruption

- Shock is out of proportion from visible blood loss

- Constant pain

- Tender tense uterus

- Fetal heart sounds absent/distressed

- Coagulation problems like DIC

1110. A 26yo woman had amenorrhea for 10wks and is pregnant. She experiences hyperemesis. Now

she presents with vaginal bleed. Exam: uterus=16wks, closed os. What is the most probable dx?

a. Thyrotoxicosis

b. Hyperemesis gravidarum

c. Twins

d. Wrong dates

e. Molar pregnancy

key : E

reason: increased uterine size for date + hyperemesis + vaginal bleeding

Molar pregnancy

complete mole >>> MRI-guided transcutaneous focused ultrasound,,the most common

partial mile >>>the trophoblast cells have three sets of chromosomes (triploid)

risk factors>>> mother over 45ys,,previous molar pregnancy,multiple pregnancy,menarche over 12


ys ,asian women , oral contraceptive pills

presentation >>>vaginal bleeding in the first trimester , hyperemesis, abnormal uterine


enlargement, hyperthyroidism, anaemia, respiratory distress and pre-eclampsia are now rare as a
result of routine use of ultrasound in early pregnancy

Investigations

Urine and blood levels of hCG >>> for follow up after evacuation

histology>>>> Definitive diagnosis is made by histological examination of the products of


conception.

u/s>>>>Ultrasound in the first trimester may not be reliable. The typical 'snowstorm' appearance
occurs mainly in the second trimester

treatment :
Suction curettage is the method of choice of evacuation for complete molar pregnancies.

Suction curettage is the method of choice of evacuation for partial molar pregnancies except when
the size of the fetal parts deters the use of suction curettage and then medical evacuation can be
used.

follow up :

A urinary pregnancy test should be performed three weeks after medical management of failed
pregnancy if products of conception are not sent for histological examination.

complications: choriocarcinoma >>> follows a molar pregnancy and should always be considered
when a patient has continued vaginal bleeding after the end of a pregnancy. It has the ability to
spread locally, as well as metastasise.

Molar Pregnancy

- Tumor consists of chorionic villi which have swollen and degenerated

- It makes a lot of HCG and therefore gives rise to exaggerated pregnancy symptoms, e.g. Hyperemesis
Gravidarum

- Signs

- Exaggerated pregnancy symptoms

- A larger for dates uterus

- Hyperthyroidism

- Most present with early pregnancy failure

- Investigation

- Snowstorm appearance on U/S

- Management

- Suction and curettage is the method of choice for evacuation

- Give anti-D prophylaxis

- Monitor HCG levels every two weeks until they are normal

- Monitor monthly for six months after they return to normal

- Pregnancy should be avoided for a year while HCG levels are being monitored

- Measure HCG 6-8 weeks after any future pregnancy regardless of the outcome

1111. A pregnant woman of G2, GA 11wks presents with heavy vomiting, headache and reduced urine

output. Urine analysis shows ketonuria. Choose the next best step?
a. US

b. Oral fluid replacement

c. Serum BHCG

d. Parental anti-emetics

e. IV fluids

key: E

reason : the pt has reduced urine output,vomiting ,headache ,ketonuria >>which indicates hypovolemia >>> IV
fluids

Vomiting in pregnancy

Presentation

Symptoms usually start between 4 and 7 weeks of gestation and resolve by 16 weeks in about
90% of women. Check for signs of dehydration

Management

Most cases are mild and do not require treatment

Diet >>> Advise the patient to rest; eat small, frequent meals that are high in carbohydrate and low
in fat

Anti-emetic drug treatment>>> This should only be given when symptoms are persistent, severe
and preventing daily activities( cyclizine, metoclopramide, prochlorperazine, promethazine,
chlorpromazine, domperidone and ondansetron )

Proton pump inhibitors and histamine H2-receptor antagonists may be used in women who also
have dyspepsia

Hyperemesis gravidarum:

intractable vomiting associated with weight loss of more than 5% of pre-pregnancy weight,
dehydration, electrolyte imbalances( hyponatremia,hypokalemia), ketosis, and the need for
admission to hospital.

management:

dietary advice

Fluid and electrolyte replacement >>> intravenous fluid and electrolyte replacement

Nutritional support (enteral or parenteral)

Vitamin supplements >>> thiamine

Thromboprophylaxis >>> LMWH

Anti-emetic drugs
Corticosteroids: may be used for intractable (failure to respond to conventional treatment) cases of
severe hyperemesis gravidarum in secondary care.

In cases of hyperemesis gravidarum: renal function and electrolytes, LFTs, midstream urine and
ultrasound (exclude multiple or molar pregnancy).

Treating Dehydration due to Heavy Vomiting (E)

- This patient has heavy vomiting which causes dehydration.

- The immediate next step here would be fluid correction through IV route.

1112. A pt had inflammatory changes on cervical smear. There is no vaginal discharge, no pelvic pain

and no fever. What is the next step?

a. Repeat smear in 6m

b. Take swab

c. Treat with antibiotics

d. Colposcopy

e. Cone biopsy

key : b

reason : if inaccurate repeat the test ( full topic discussed earlier )

Inflammatory Changes on Cervical Smear (A)

- OHCS on encountering inflammatory changes: Take swab. Treat infection. Repeat in 6 months. Colposcopy
after 3 abnormal.

1113. A 37yo infertile lady with 5 cm subserosal and 3 cm submucosal fibroid is trying to get pregnant.

Which is the most suitable option?

a. Clomiphene therapy

b. IVF

c. Myomectomy

d. Hysterectomy

e. IU insemination

key : c

reason : submucosal fibroids cause infertility >> myomectomy ( full topic discussed earlier)

- Refer to 1108. It is explained in detail over there.


1114. A young tall man and his wife are trying for babies and present at the infertility clinic. On inv the

man has primary infertility and azoospermia. What other inv should be done?

a. Testosterone

b. LSH

c. FSH

d. Estradiol

e. Karyotyping

key : e

reason : in primary azoospermia + tall man >>> karyotyping >>> klinefelter's syndrome

Klinefelter's syndrome

karyotype XXY is associated with hypogonadism and disorders of spermatogenesis.

The classic clinical description

Infertility and small firm testes are present in about 99% of individuals ,decreased facial
and pubic hair; loss of libido; impotence.

Tall and slender, with long legs, narrow shoulders, and wide hips.

Gynaecomastia or history of gynaecomastia during puberty; decreased libido; history of


undescended testes.

Learning disability; delayed speech development; behavioural problems; psychosocial disturbances.

tiredness, reduced muscle power and stamina, and truncal obesity

Investigations

XXY males may be diagnosed before birth, through amniocentesis or chorionic villus sampling.

Later, serum testosterone is low or low normal. FSH and LH are elevated (FSH >LH).

Diagnosis is confirmed by chromosomal analysis. The most common indications for karyotyping are
hypogonadism and infertility.

Management

testosterone replacement

fertility treatment : ICSI

Surgical treatment : for gynaecomastia

- Young tall man with primary infertility and Azoospermia points in the direction of Klinefelters Syndrome.
- The definitive diagnosis is chromosomal analysis also known as karyotyping.

1115. A woman who is on regular COCP presented to you for advice on what to do as she has to now

start to take a course of 7d antibiotics. What would you advice?

a. Continue regular COC

b. Continue COCP and backup contraception using condoms for 2d

c. Continue COCP and backup contraception using condoms for 7d

d. Continue COCP and backup contraception using condoms for 2wks

key : D

reason : There are many commonly used medications which can affect the efficacy of the pill

INTERACTIONS :

*Non-enzyme-inducing antibacterial >>> Women should be advised that no additional


contraception is required.

*Enzyme-inducing antibacterials rifampicin or rifabutin , and anticonvulsants, St John's wort, (Short


course two months or less ) >>> Women are advised to continue taking the COCP ,use additional
precautions AND should be continued for 28 days after stopping the rifampicin/rifabutin.

*Enzyme-inducing antibacterials rifampicin or rifabutin,and anticonvulsants, St John's wort (Long-


term course) >>> Should be advised to use an alternative, non-hormonal method

*Lamotrigine >>> Women should be advised not to take lamotrigine with the COCP and should
seek another form of contraception (unless also taking a non-enzyme-inducing anticonvulsant such
as sodium valproate)

*Antiretroviral therapies>>> women on ritonavir-boosted protease inhibitors should be advised to


use alternative methods of contraception.

*Ulipristal acetate>>>Women should use additional contraceptive precautions for 14 days after
taking ulipristal acetate

- Antibiotics like Rifampicin and Griseofulvin increase the breakdown of estrogen and therefore can cause
unwanted pregnancy.

- In such cases, backup contraception using condoms for two weeks should be advised.

1116. A lady presents with hot flashes and other symptoms of menopause. What is the tx option?

a. Raloxifene

b. HRT

c. Bisphosphonate
d. COCP

e. Topical estrogen

key :B

reason : HRT is the gold standard treatment for hot flushes.

MENOPAUSE

The menopause is a natural phenomenon occurs in all women when the number of ovarian follicles are
depleted.

oestrogen & progesterone fall, and LH &FSH increase in response. Menstruation becomes erratic and
eventually stops

PRESENTATION

Menstrual irregularity >>> which may last for up to four years,The cycle may lengthen or shorten,A
slight increase in the amount of menstrual blood loss,10% of women have an abrupt cessation of
periods.

Hot flushes and sweats

Urinary and vaginal symptoms >>> dyspareunia, vaginal discomfort and dryness, recurrent lower
urinary tract infection and urinary incontinence

Sleep disturbance , Loss of libido AND Mood changes

Management

Healthy lifestyle >>> Stopping smoking, losing weight and limiting alcohol

Hormone replacement therapy (HRT) >>> the most effective treatment to completely relieve the
symptoms particularly Vasomotor symptoms (hot flushes/night sweats) ,Mood swings ,Vaginal
and bladder symptoms caused by the menopause. It also prevents and reverses bone loss.

Alternatives to HRT >>> Herbal or complementary treatments

Other drugs >>> GABA , SSRI

1116. Menopause (B)

- Permanent cessation of menstruation for a minimum of 12 months in the absence of other causes of
Amenorrhea.

- Usually occurs between 45-55 years; average age is 52.

- Symptoms

- Hot flushes

- Depression

- Anxiety
- Irritability

- Mood swings

- Vaginal dryness

- Atrophic Vaginitis leading to dyspareunia

- Treatment

- Hormone-Replacement Therapy is the gold standard.

- Estrogen-only HRT is suitable for candidates who have had a hysterectomy

- Combined HRT is suitable for patients with a uterus

- Vaginal dryness and atrophic vaginitis should be treated with topical estrogen

- Long-term Complications

- Osteoporosis

- Cardiovascular disease

- Urogenital Atrophy Atrophic Vaginitis

- HRT Contraindications

- Estrogen-dependent cancer

- Past pulmonary embolus

- Undiagnosed PV bleeding

- Increased LFTs

- Pregnancy

- Breastfeeding

- Phlebitis

1117. A 28yo woman at 34wks GA for her first pregnancy attends antenatal clinic. Her blood results:

Hgb=10.6, MCV=95, MCHC=350. What do you do for her?

a. Folate

b. Dextran

c. Ferrous sulphate

d. None

e. IV FeSO4

f. Explain this physiologic


hemodynamic anemia

g. Blood transfusion

key : F

reason : normal MCV,MCHC + in the second and third trimester anemia is considered when Hb is less than
10.5

Anemia in pregnancy

definition >>> Hb level <11.0 g/dL at booking. In the second and third trimesters the diagnostic level for
anaemia is an Hb level of <10.5 g/dL. Postpartum the diagnostic level is 10.0 g/dL.

Iron-deficiency anaemia accounts for 85% of all cases of anaemia that are identified and is characterised by
MCV, and it's caused by nutritional deficiency or low iron stores

Presentation >>> often asymptomatic,Fatigue,Dyspnoea m pallor

Investigations

Hb

MCV 76 >>> the probable cause is iron deficiency

Normal MCV (76-96 fl) with low Hb is typical of pregnancy

Serum ferritin 10-50 g/L needs monitoring and <10 g/L requires treatment.

Management:

Women with known haemoglobinopathy >>> check serum ferritin >>> if low give oral supplements

Women with unknown haemoglobinopathy status with a normocytic or microcytic anaemia>>> oral
supplements and haemoglobinopathy screening

Non-anaemic women at increased risk of iron deficiency >>> check serum ferritin >>> if low give
oral supplements

Women with established iron deficiency anaemia >>> give 100-200 mg elemental iron daily,
continue for at least three months

* Referral to secondary care in case of >>> significant symptoms ,severe anaemia (Hb<70 g/dL) ,late
gestation (>34 weeks) or failure to respond to a trial of oral iron.

- Hb < 11.5 g/dl

- 85% of the times the cause is Iron deficiency

- Other causes are less common

- Investigations

- Hb
- MCV: If less than 76, it is most probably due to Iron deficiency. If it is normal then it is the typical dilutional
anemia of pregnancy.

- Serum Ferritin: 10-50 ug/L needs monitoring; less than 10 ug/L requires treatment

- Management

- Routine Iron Replacement is not recommended in the UK.

- Treat with oral Ferrous Sulphate if Hb is less than 11.5 g/dl

1118. A 34yo woman who never had fits or high BP developed fits 6h after delivery of a term healthy

child. What is the most likely dx?

a. Eclampsia

b. Preeclampsia

c. Epilepsy

d. Pulmonary embolism

e. Pregnancy induced HTN

key : C

reason: postpartum fits>> occurs in 45% of cases of eclampsia

Eclampsia

Eclampsia is defined as the occurrence of one or more convulsions superimposed on pre-eclampsia. 44% of
seizures occur postnatally, the remainder being antepartum (38%) or intrapartum (18%).

Management of eclampsia:

Resuscitation >> The patient should be placed in the left lateral position and the airway secured +
O2

Treatment and prophylaxis of seizures >> Magnesium sulfate is the anticonvulsant drug of choice,
intubation may become necessary in women with repeated seizures

Treatment of hypertension >> (blood pressure >160/110 mmHg or mean arterial pressure >125
mmHg) ,,,Intravenous hydralazine or labetalol are the two most commonly used drugs ( SE >> fetal
distress,so continuous fetal heart rate monitoring)

Fluid therapy

Delivery >>>The definitive treatment of eclampsia is delivery

Complications :

haemolysis, HELLP syndrome (3%), disseminated intravascular coagulation (3%), renal failure (4%)
and adult respiratory distress syndrome (3%).
Cerebrovascular haemorrhage is a complicating factor in 1-2%.

- Symptoms of pre-eclampsia+Seizure = Eclampsia

- NOTE: If a woman has a fit within a few days after delivery, it is always eclampsia until proven otherwise.

- Management

- ABC

- MgSO4 IV bolus 4g, then 1g IV infusion for 24 hours, and if seizure recurs give IV bolus.

- Monitor BP, pulse, RR and Oxygen saturation every 15 minutes.

- If BP> 160/110, give antihypertensive (Hydralazine, Labetalol, Nifedipine)

- NOTE: If the patient has been given MgSO4 and experiences another fit, repeat MgSO4.

1119. A 30yo lady who already has one child through a prv C-section demands a reversible

contraception. She presently experiences heavy and painful periods. What is the most

appropriate contraceptive you will recommend for her?

a. COCP

b. POP

c. Implanon

d. Danazol

e. Mirena

f. IUCD

key : E

reason: Women with idiopathic menorrhagia, Consider the LNG-IUS(Merina) as the first-line option

Contraception in special groups

Menorrhagia >>> Women with idiopathic menorrhagia>>> Consider the LNG-IUS(Merina) as the
first-line option, COCP as the second-line option, and POP and progestogen-only injectables as
third-line options.

Fibroid >>> With distortion of the uterine cavity: cu-IUCDs and the LNG-IUS should NOT be used

obese >>> COCP shouldn NOT be used

Migraine :

-WITH AURA >>> don't use COCP ---- use POP, progestogen-only implants and injectables, and the LNG-
IUS BUT they are not recommended to continue if the woman developed migraine with aura while using them
-WITHOUT AURA (<35 years of age) >>> NOT usually recommended >>> continuation of COCP, the
combined contraceptive patch, and combined contraceptive vaginal ring

-WITHOUT AURA ( 35 years of age) >>> don't use COCP

DM >>> Methods that should not usually be used: progestogen-only injectables. COCP, the
combined contraceptive patch and the combined contraceptive vaginal ring

Hypertension >>> Methods that are not usually recommended: COCP, the combined
contraceptive patch and combined contraceptive vaginal ring.

Irregular menses >>> COCP

Smoking >>> Methods that should not be used: COCP, combined contraceptive patch and
combined contraceptive vaginal ring.

Multiple risk factors for cardiovascular disease>>> Methods that should not be used: COCP,
combined contraceptive patch and combined contraceptive vaginal ring AND POP are not usually
recommended

Venous thromboembolism or risk of thromboembolism>>> Methods that should not be used:


COCP, the combined contraceptive patch, and the combined contraceptive vaginal ring.

Sickle cell disease >>> intrauterine devices are not recommended, as they may be associated with
uterine bleeding and infection , Depot contraceptive (Depo-Provera) is safe and has been found to
improve the blood picture and reduce pain crises.

Women taking anticoagulants for VTE >>> Methods that should not be used: COCP, combined
contraceptive patch and combined contraceptive vaginal ring AND POP are not usually
recommended

Sexually transmitted infection or pelvic inflammatory disease >>> insertion of a cu-IUCD or the
LNG-IUS is not recommended.

Contraception for those with learning disabilities >>> use of injectable contraceptives and IUCDs is
high

Contraception for those also taking enzyme enhancers >>>COCP - all women should be advised to
switch to a contraceptive method unaffected by enzyme inducers (eg progestogen-only
injectable, copper IUCD (Cu-IUCDs) or LNG-IUS).or to cover with another method eg. condoms

POP>>> Advise alternative contraceptive methods

Progestogen-only implants >>> May continue with progestogen-only implants


with additional contraceptive protection, such as condoms,
when taking liver enzyme-inducers and for four weeks after they are stopped.

drugs which induce liver enzymes include:

Antifungals: griseofulvin.

Antibiotics: rifampicin and rifabutin.


Anti-epileptics: carbamazepine, eslicarbazepine, phenytoin, phenobarbital, primidone, oxcarbazepine,
topiramate.

Central nervous system stimulant: modafinil.

Antiretroviral drugs: nelfinavir, nevirapine, ritonavir.

St John's wort.

- Mirena, also known as Intrauterine System, is a Levonorgestril-containing coil which is inserted into the
uterus.

- Local effect: reversible endometrial atrophy, makes implantation less likely and periods lighter (20%
reversible amenorrhea)

- Less risk of ectopic pregnancy

- Risk of STDs is reduced

- May benefit women with endometriosis, adenomyosis, fibroids or endometrial hyperplasia

- NOTE: Avoid if breast cancer.

1120. A 32yo woman comes with intermenstrual bleeding. Her last cervical smear was 1y ago and was

negative. What test would you recommend for her initially?

a. Colposcopy

b. Cervical smear

c. Endocervical swab

d. Transvaginal US

e. Pelvic CT

key : b

reason : to exclude cervical carcinoma

- In this patient there is no indication for colposcopy. Indications for colposcopy include:

- Smear showing mild, moderate or severe Dyskaryosis

- Any suggestion of malignancy

- 3 consecutive inflammatory smears

- Glandular abnormal cells

- 2 consecutive atypical smears

- 3 consecutive borderline or inadequate smears


- Post-coital bleeding

- Endocervical Swab is not advisable at this point due to absence of any signs of infection

- Initially, a cervical smear should be taken as the previous one was negative one year ago (25-50 years, take
cervical smear every 3 years)

1121. A 20yo woman has had abdominal pain in the LIF for 6wks duration. Over the past 48h, she has

severe abdominal pain and has a fever of 39.1C. Pelvic US shows a complex cystic 7 cm mass in

the LIF. What is the most likely dx?

a. Endometriosis

b. Dermoid cyst

c. Ovarian ca

d. Tubo-ovarian abscess

e. Ectopic pregnancy

key : D

reason : fever 39.1 +cystic pelvic mass + localized abdominal pain

Tubo-ovarian abscess

A tubo-ovarian abscess is one type of pelvic abscess which is found in women of reproductive age, and may be
a complication of pelvic inflammatory disease. In this case it is an inflammatory mass which involves the ovary
and Fallopian tube

Presentation

Systemic features of toxicity: fever, malaise, anorexia, nausea, vomiting, pyrexia.

Local effects: eg, pain, deep tenderness , diarrhoea, tenesmus, mucous discharge per rectum,
urinary frequency, dysuria, vaginal bleeding or discharge.

Investigations

FBC >>> raised white cell count often but not invariably

Ultrasound

CT/MRI scanning may be more effective at identifying the origin of the abscess

Management:

Hospital admission

Drainage of the abscess along with antibiotic treatment

Antibiotics used alone are occasionally effective for very early, small abscesses
Procedures used for drainage of the pelvic abscess >>> Ultrasound-guided aspiration and drainage-
---CT-guided aspiration and drainage----Endoscopic ultrasound-guided drainage ----Laparotomy or
laparoscopy with drainage of abscess

Definitive surgery may be required after initial drainage for some causes of pelvic abscess, such as
salpingo-oophorectomy for tubo-ovarian abscess.

-General symptomatology of pelvic infections

- Systemic features of toxicity: fever, malaise, anorexia, nausea, vomiting, pyrexia.

- Local effects: eg, pain, deep tenderness in one or both lower quadrants, diarrhoea, tenesmus, mucous
discharge per rectum, urinary frequency, dysuria, vaginal bleeding or discharge.

- Rectal or vaginal examination: may reveal tenderness of the pelvic peritoneum and bulging of the anterior
rectal wall.

- Partial obstruction of the small intestine: this may sometimes occur.

-Investigations

- FBC showing increased WBC count

- U/S

- CT/MRI scanning may help in tracing the origin of the abscess

- Management

- Arrange urgent admission to hospital.

- Management is usually by drainage of the abscess along with antibiotic treatment. Antibiotics used alone are
occasionally effective for very early, small abscesses

- Antibiotic choice is guided by the likely cause and local resistance patterns and guidelines, but usually needs
to be broad-spectrum until the pathogens are determined.

- Points in favour of Tubo-Ovarian Abscess

- Fever

- Pain in the LIF

- Severe abdominal pain

- U/S showing a cystic mass in the LIF

1122. A woman is 16wk pregnant and she is worried about abnormal chromosomal anomaly in her

child. What is the definitive inv at this stage?

a. Amniocentesis
b. CVS

c. Parents karyotyping

d. Coombs test

e. Preimplantation genetic dx

key : A

Prenatal diagnosis

Amniocentesis : This is normally carried out from 15 weeks of gestation. A needle is inserted into
the amniotic cavity and amniotic fluid is sampled, allowing culture and assessment of fetal cells.the
most invasive prenatal diagnostic procedure.

Chorionic villus sampling : carried out at 11-13 weeks. Risk of miscarriage may be slightly higher
than for amniocentesis. sampling of the developing placenta and the same type of analysis of fetal
cells to detect chromosomal, genetically inherited and endocrine or metabolic conditions.

Fetoscopy : This allows visualisation of the fetus, using endoscopic techniques. at 18-20 weeks of
gestation

Cordocentesis/percutaneous umbilical blood sampling : at 18 weeks.ultrasound guidance to


obtain fetal blood cells from the umbilical cord.It enables karyotyping/chromosome analysis,
Intrauterine blood transfusions, Fetal viral infection confirmation.

Fetal radiology : in suspected skeletal dysplasia, ultrasound (2D and 3D) and MRI are now the
investigations of choice

Ultrasound-guided percutaneous skin and organ biopsy : This can also be carried out to allow skin,
muscle, liver and other fetal organ analysis

Maternal blood tests : eg. Maternal serum alpha-fetoprotein levels can be measured to aid the
diagnosis of neural tube defects between 15-22 weeks

- Definitive tests

- Pre-implantation Genetic Diagnosis (earliest possible diagnosis; available to couples at risk of having a child
with specific genetic or chromosomal disorder such as CF, Thalassemia and Huntingtons etc)

- Chorionic Villous Sampling (done between 10 and 13 weeks; enables early detection and provides the
mother with a choice to either continue or terminate pregnancy)

- Amniocentesis (done between 14-18 weeks; carries a very low risk of miscarriage)

- The patient is 16 weeks pregnant, so the best definitive testing that can be carried out at this stage would be
Amniocentesis.

1123. A 28yo lady with a fam hx of CF comes for genetic counselling and wants the earliest possible dx

test for CF for the baby she is planning. She is not in favor of termination. What would you
recommend for her?

a. CVS

b. Amniocentesis

c. Pre-implantation genetic dx

d. Chromosomal karyotyping

e. Maternal serum test

f. Reassure

key : C

reason :pre-implantation genetic dx is the earliest diagnostic test

Pre-implantation prenatal diagnosis

This is a technique that allows the analysis of oocytes or embryos conceived through in vitro fertilisation (IVF).
This information then informs the choice of optimal embryos to be transferred back to the mother. This is an
accepted technique for avoiding the birth of affected children from parents with a known genetic abnormality

- There is a positive family history of CF, therefore the earliest possible diagnosis can be made via Pre-
implantation genetic testing.

- See above for further details

1124. A 39yo woman in her 36th week GA with acute abdominal pain is rushed for immediate delivery.

Her report: BP=110/60mmHg, Hgb=low, bilirubin=22, AST=35, Plt=60, APTT=60, PT=30,

Fibrinogen=0.6. What is the cause?

a. Pregnancy induced hypertension

b. DIC

c. HELLP syndrome

d. Acute fatty live

e. Obstetric cholestasis

key : B

reason : low fibrinogen,increased bilirubin ,normal AST , prolonged PT,PTT

DIC

The diagnosis of DIC should include both clinical and laboratory information:

(PT) elevated.

(aPTT) elevated.
Platelet counts in DIC are typically low

Fibrinogen level low.

Confirmatory tests :

The D-dimer test gives strong evidence of DIC.

Fibrin degradation products (FDPs) are helpful but can occur in other conditions such as deep
vein thrombosis (DVT)

In acute DIC, PT and aPTT are prolonged, and the platelet count and fibrinogen decrease. D-dimer,
FDP, and fibrin monomer levels are elevated

Management :

The cornerstone of the the management of DIC is treatment of the underlying condition. Thus, infection will
need antibiotics, and obstetric complications may need intervention

plasma and /or platelet transfusion

Conditions that may be complicated by DIC include:

* Infections, especially septicaemia, Escherichia coliO157, typhoid fever, Rocky Mountain spotted fever and
parasites. The rash of meningococcal septicaemia is classical.

* Malignancy, especially leukaemias.

* Major trauma including crush syndrome and, occasionally, burns. * Some connective tissue disorders
including antiphospholipid syndrome. * Complications of pregnancy including the placental problem of
placental abruption, amniotic fluid embolism, severe hypertension of pregnancy with fulminating pre-
eclampsia and HELLP syndrome. A retained dead fetus tends to produce a thrombotic rather than a
haemorrhagic state.

* Incompatible blood transfusion.

* Heat stroke.

* Dissecting aortic aneurysm.

* Some snake bites

If DIC is suspected then clotting screen tests are followed by confirmation:

- Prothrombin time (PT) elevated.

- Activated partial thromboplastin time (aPTT) elevated.

- Platelet counts in DIC are typically low, especially in acute sepsis-associated DIC, but may be increased in
malignancy-associated chronic DIC.

- Fibrinogen level low. If two results are positive,DIAGNOSIS is possible; if three are positive, it is likely; if all
four are positive, it is extremely likely.

- Confirmatory tests look for evidence of the simultaneous formation of thrombin and plasmin.
- The D-dimer test gives strong evidence of DIC.

- Fibrin degradation products (FDPs) are helpful but can occur in other conditions such as deep vein
thrombosis (DVT) and, in severe disease, they may be negative.

- In acute DIC, PT and aPTT are prolonged, and the platelet count and fibrinogen decrease. D-dimer, FDP, and
fibrin monomer levels are elevated.

1125. A 36wk pregnant woman presents with sudden onset of uterine pain and bleeding, uterus is

tender, no prv LSCS. What is the most appropriate cause?

a. Preeclampsia

b. DIC

c. Placental abruption

d. Placental previa

e. Ectopic pregnancy

f. Missed abortion

g. Ectropion

key : C

reason : a case of antepartum haemorrhage in the third trimester >>> pain + bleeding >>> abruptin ( pain is
common with abruption) ( full topic discussed earlier)

- Bleeding during the third trimester is either painful or painless bleeding.

- Painful bleeding points to Placental Abruption while painless bleeding points to Placenta Previa

- In this case, the patient is experiencing painful vaginal bleeding in the third trimester, so this appears to be
the case of Placental Abruption

- The investigation of choice in this case is an Ultrasound

- Risk Factors for Abruption

- Hypertension

- Smoking

- Multiple Pregnancy

- Cocaine/Amphetamine Use

- Increased Maternal Age

- Trauma to the abdomen

- Polyhydramnios
- Investigation

- Diagnosis is clinical but U/S is done to exclude Placenta Praevia and to check the well-being of the baby.

- Management

- Always admit the patient to hospital for assessment and management. Phone 999/112/911 if there are any
major concerns regarding maternal or fetal well-being.

- The mainstays of management are resuscitation and accurate diagnosis of the underlying cause.

- Severe bleeding or fetal distress: urgent delivery of the baby, irrespective of gestational age.

- Admit to hospital, even if bleeding is only a very small amount. There may be a large amount of concealed
bleeding with only a small amount of revealed vaginal bleeding.

- No vaginal examination should be attempted, at least until a placenta praevia is excluded by ultrasound. It
may initiate torrential bleeding from a placenta praevia.

- Resuscitation can be inadequate because of underestimation of blood loss and misleading maternal
response. A young woman may maintain a normal blood pressure until sudden and catastrophic
decompensation occurs.

- Take blood for FBC and clotting studies. Crossmatch, as heavy loss may require transfusion.

- Gentle palpation of the abdomen to determine the gestational age of the fetus, presentation and position.

- Fetal monitoring.

- Arrange urgent ultrasound.

- With every episode of bleeding, a rhesus-negative woman should have a Kleihauer test and be given
prophylactic anti-D immunoglobulin

- Complications

- Premature Labour

- DIC

- Renal Tubular Necrosis

- PPH

- Placenta Accreta

- Points to look for diagnosis of Abruption

- Shock is out of proportion from visible blood loss

- Constant pain

- Tender tense uterus

- Fetal heart sounds absent/distressed

- Coagulation problems like DIC


1126. A 28wk pregnant woman presents with uterine bleeding after sexual intercourse. What is the

most appropriate cause?

a. Preeclampsia

b. DIC

c. Placental abruption

d. Placental previa

e. Ectopic pregnancy

f. Missed abortion

g. Ectropion

key : G

reason :postcoital bleeding >>> mostly ectropion

Causes of postcoital bleeding

Infection,Trauma.

Cervical ectropion - especially in those women taking the combined oral contraceptive pill (COCP).

Cervical or endometrial polyps.

Vaginal cancer , Cervical cancer - usually apparent on speculum examination.

Ectropion

It is most commonly seen in teenagers, during pregnancy and in women on combined hormonal
contraception.

It is generally an asymptomatic condition but patients occasionally present with bleeding or


excessive discharge.

Once a normal cervical smear has been confirmed, it is actively managed only if there are
symptoms. Over time, vaginal acidity promotes metaplasia to squamous epithelium when the
symptoms will disappear.

After stopping any oestrogen-containing contraceptive, treatment options are controversial but
include diathermy, cryotherapy, surgery with laser treatment and microwave therapy.

- The cervix enlarges under the influence of oestrogen and as a result the endocervical canal is everted. It is
seen on examination as a red ring around the os and is so common as to be regarded as normal. - It is most
commonly seen in teenagers, during pregnancy and in women on combined hormonal contraception.

- This seems to be the most appropriate among the given options because the patient is 28 weeks pregnant
and had sex at this point.
- There are no signs of pre-eclampsia, DIC, Placental abruption and Placenta Praevia. Ectopic pregnancy and
missed abortion do not present at this stage.

1127. A 6wk pregnant woman presents with abdominal pain. She has prv hx of PID. What is the most

likely dx?

a. Preeclampsia

b. DIC

c. Placental abruption

d. Placental previa

e. Ectopic pregnancy

f. Missed abortion

g. Ectropion

key : E

reason : hx of PID + early pregnancy + abdominal pain

Ectopic pregnancy

The majority of ectopic pregnancies occur in the ampullary or isthmic portions of the Fallopian tubes

Risk factors

IUCD

PID

in tubes that have been divided in a sterilisation operation and where they have been
reconstructed to reverse on

Presentation :

30% of ectopic pregnancies present before a period has been missed.

Abdominal pain.

Pelvic pain.

Amenorrhoea or missed period.

Vaginal bleeding (with or without clots)

Investigations:

The most accurate method to detect a tubal pregnancy is transvaginal ultrasound.

hCG levels are performed in women with pregnancy of unknown location who are clinically stable
Management :

Medical management: systemic methotrexate is offered first-line to those women who are able to
return for follow-up and who have the following:

No significant pain,Unruptured ectopic pregnancy with an adnexal mass <35 mm and no visible
heartbeat, No intrauterine pregnancy seen on ultrasound scan ,Serum hCG <1500 IU/L.

Surgical management: offered to those women who can not return for follow-up after
methotrexate or to those who have any of the following:

Significant pain.

Adnexal mass 35 mm.

Fetal heartbeat visible on scan.

Serum hCG level 5000 IU/L.

A laparoscopic approach is preferable. A salpingectomy should be performed, unless the woman


has other risk factors for infertility, in which case a salpingotomy should be undertaken.

- Symptoms may develop at any time between 4 and 10 weeks of pregnancy. * - Pain on one side of the lower
tummy (abdomen). It may develop sharply, or may slowly get worse over several days. It can become severe. *
Vaginal bleeding often occurs, but not always. It is often different to the bleeding of a period. For example, the
bleeding may be heavier or lighter than a normal period. The blood may look darker. However, you may think
the bleeding is a late period.

* Shoulder tip pain due to irritation of the diaphragm.

- Risk factors

- Women over the age of 35

- Previous ectopic pregnancy

- Scarring, damage or other structural abnormality of the fallopian tube

- PID (This patient in question has a history of PID)

- IUCDs

- Investigations

- Urine pregnancy test

- Transvaginal U/S

- Beta-HCG

- Diagnostic Laproscopy

- Treatment options

- Ruptured ectopic pregnancy


Emergency surgery is needed if a Fallopian tube ruptures with heavy bleeding. The main aim is to stop the
bleeding. The ruptured Fallopian tube and remnant of the early pregnancy are then removed. The operation is
often life-saving.

- Early ectopic pregnancy - before rupture

Ectopic pregnancy is most often diagnosed before rupture. Your doctor will discuss the treatment options with
you and, in many cases, you are able to decide which treatment is best for you. These may include the
following:

* Surgery. Removal of the tube (either the whole tube or part of it) and the ectopic pregnancy is most
commonly performed by keyhole surgery (a laparoscopic operation). Removal of the Fallopian tube containing
the ectopic pregnancy (salpingectomy) is usually performed if the other tube is healthy. Removal of only a
section of the tube with the ectopic pregnancy in it (salpingotomy) is usually performed if the other tube is
unhealthy; for example, scarred from a previous infection. However, many women with an ectopic pregnancy
do not need to have an operation.

* Medical treatment. Medical treatment of ectopic pregnancies is now more common and avoids the need for
surgery. A medicine called methotrexate is often given, usually as an injection. It works by killing the cells of
the pregnancy growing in the Fallopian tube. It is normally only advised if the pregnancy is very early. The
advantage is that you do not need an operation. The disadvantage is that you will need close observation for
several weeks with repeated blood tests andSCANS to check it has worked. You will need to have a blood test
for hCG every 2-3 days until your levels are low. Scans are usually repeated weekly. Methotrexate can cause
side-effects which include nausea and vomiting in some women. It can be common for some abdominal pains
to develop 3-7 days after having methotrexate.

1128. A 33wk pregnant woman presents with vaginal bleeding, low Hgb, low plt, increased bilirubin,

AST normal, APTT & PT increased. What is the most likely dx?

a. Preeclampsia

b. DIC

c. Placental abruption

d. Placental previa

e. Ectopic pregnancy

f. Missed abortion

g. Ectropion

key : B

reason : low Hb,plt,,, high PT,PTT ,bilirubin,,,normal AST (full topic discussed earlier)

Conditions that may be complicated by DIC include:

* Infections, especially septicaemia, Escherichia coliO157, typhoid fever, Rocky Mountain spotted fever and
parasites. The rash of meningococcal septicaemia is classical.
* Malignancy, especially leukaemias.

* Major trauma including crush syndrome and, occasionally, burns. * Some connective tissue disorders
including antiphospholipid syndrome.[2] * Complications of pregnancy including the placental problem of
placental abruption, amniotic fluid embolism, severe hypertension of pregnancy with fulminating pre-
eclampsia and HELLP syndrome. A retained dead fetus tends to produce a thrombotic rather than a
haemorrhagic state.

* Incompatible blood transfusion.

* Heat stroke.

* Dissecting aortic aneurysm.

* Some snake bites

If DIC is suspected then clotting screen tests are followed by confirmation:

- Prothrombin time (PT) elevated.

- Activated partial thromboplastin time (aPTT) elevated.

- Platelet counts in DIC are typically low, especially in acute sepsis-associated DIC, but may be increased in
malignancy-associated chronic DIC.[3]

- Fibrinogen level low. If two results are positive,DIAGNOSIS is possible; if three are positive, it is likely; if all
four are positive, it is extremely likely.

- Confirmatory tests look for evidence of the simultaneous formation of thrombin and plasmin.

- The D-dimer test gives strong evidence of DIC.

- Fibrin degradation products (FDPs) are helpful but can occur in other conditions such as deep vein
thrombosis (DVT) and, in severe disease, they may be negative.

- In acute DIC, PT and aPTT are prolonged, and the platelet count and fibrinogen decrease. D-dimer, FDP, and
fibrin monomer levels are elevated.

1129. A 25yo lady at her 28th week GA came for check up. Her BP=160/95mmHg, protein in urine=6g/d.

What is the most likely dx?

a. Essential HTN

b. Gestational HTN

c. Chronic HTN

d. Preeclampsia

key : D

reason : HTN + protienuria +/- edema >>> preeclampsia

Pre-eclampsia
Pre-eclampsia is pregnancy-induced hypertension in association with proteinuria (>0.3 g in 24 hours) with or
without oedema.

Severe pre-eclampsia is defined as diastolic blood pressure of at least 110 mm Hg, or systolic blood pressure of
at least 160 mm Hg, and/or symptoms, and/or biochemical and/or haematological impairment

Presentation:

New hypertension ,New and/or significant proteinuria.

Other clinical features of severe pre-eclampsia include:

Severe headache - usually frontal.

Platelet count falling to below 100 x 109/

Abnormal liver enzymes (ALT or AST rising to above 70 IU/L).

HELLP syndrome: H (haemolysis) EL (elevated liver enzymes) LP (low platelets).

Investigations :

Urinalysis

Frequent monitoring of FBC, LFTs, renal function, electrolytes and serum urate

Clotting studies if there is severe pre-eclampsia or thrombocytopenia

24-hour urine collections for protein quantification and creatinine clearance.

Assessment of fetus -ultrasound

Management:

Control Blood pressure

Prevention of seizures >>> Magnesium sulfate

Fluid balance

Delivery

- Pre-eclampsia: BP > 140/90 and 300 mg proteinuria in 24-hour urine collection

- Mild to moderate: BP <160/110 with significant proteinuria and no maternal complications

- Severe: BP >160/110 with significant proteinuria or if maternal complications occur

- Risk factors

- Previous history of pre-eclampsia

- Maternal age > 40 years

- Family History
- DM, HTN, Renal Disease

- This is a case of severe pre-eclampsia as the BP of the patient in question suggests. This should be managed
along the following lines.

- Anti-hypertensives to bring BP down to less than 160/110

- IV Hydralazine is the first choice

- Labetolol

- MgSO4 to prevent eclampsia

- CTG and U/S to monitor the baby

NOTE: If less than 34 weeks gestation, give steroids to help production of surfactant.

- Complications

- Eclampsia

- HELLP Syndrome

- DIC

- Renal Failure

- Placental Abruption

1130. A 32yo woman has a hx of spontaneous abortions at 6wks, 12wks, and 20wks. She is now keen

to conceive again. Which of the following would you prescribe for the next pregnancy?

a. MgSO4

b. Aspirin

c. Warfarin

d. Mefenamic acid

e. Heparin

key : B

reason : antiphospholipid syndrome is the most important treatable cause of recurrent miscarriage >>>> Rx
>>> Asprin

recurrent miscarriage

def. >>> the loss of three or more consecutive pregnancies

Aetiology :
Antiphospholipid syndrome >> investigation : anticardiolipin antibodies , lupus anticoagulant >>>
Rx : Aspirin

structural >>> uterine anomalies , fibroids , cervical incompetence ( late miscarriage) >>>
investigation : pelvic u/s >>> Rx in cervical incompetence : cerclage (complication : uterine rupture)

endocrine >>> PCOS , uncontrolled diabetes

immune

thrombophilia

genetic abnormality

infection >>> bacterial vaginosis in the first trimester is a risk of second trimester miscarriage and
preterm delivery

- Recurrent miscarriage is defined as the loss of three or more consecutive pregnancies. - Miscarriage, the
most common complication of pregnancy, is the spontaneous loss of a pregnancy before the fetus has reached
viability. The term therefore includes all pregnancy losses from the time of conception until 24 weeks of
gestation in the UK. - Antiphospholipid syndrome (APS):

- This is the most important treatable cause of recurrent miscarriage.

- Investigations

- Antiphospholipid antibodies:

The presence of these is associated with early miscarriages and maternal morbidity and is referred to as
primary APS. There is requirement for two tests at least six weeks apart showing either lupus anticoagulant or
anticardiolipin antibodies at significant levels.

- Women with recurrent first-trimester miscarriage and all women with one or more second-trimester
miscarriages should be screened for antiphospholipid antibodies before pregnancy.

- Women with second-trimester miscarriage should be screened for inherited thrombophilias including factor
V Leiden, factor II (prothrombin) gene mutation and protein S.

- All women with recurrent first-trimester miscarriage and all women with one or more second-trimester
miscarriages should have pelvic ultrasound to assess uterine anatomy.

- If uterine anomalies are detected then further investigations, such as hysteroscopy and/or laparoscopy, may
be required.

- Management

- General advice

- Reassurance should be given about the high probability of a successful outcome. In a large trial that included
women with 4.2 consecutive miscarriages and an average age of 32.7 years, the placebo group was shown to
have a live birth rate of 65%.

Pharmacological treatment
- In primary APS patients, heparin combined with low-dose aspirin improves live birth rate to 70%.[5] There
ARE only limited data supporting the use of heparin in women without APS

- There is some evidence suggesting that use of metformin during pregnancy is associated with a reduction in
the miscarriage rate in women with polycystic ovarian syndrome

- However, the RCOG DOES NOT recommend its use in pregnancy at present until further randomised
prospective study results are available to provide adequate evidence of safety and efficacy of its use.

- A Cochrane review found evidence of benefit for progestogen therapy in women with a history of recurrent
miscarriage. There was no statistically significant difference in rates of adverse effects.

- However, there is currently a large randomised, double-blind, placebo-controlled multicentre trial underway
- the Progesterone in recurrent miscarriage (PROMISE) study - which aims to provide a definitive answer
regarding progesterone use in women with recurrent miscarriages.

Surgical

- Cervical cerclage is used where cervical incompetence is suspected. However, it is overdiagnosed as a cause
of second-trimester miscarriage. The cerclage procedure also carries a risk of stimulating uterine contractions.

- Cerclage benefit increases as the cervix shortens to less than 25 mm. It has also been shown to be beneficial
in those women with a shortened cervical length of less than 25 mm.

1131. A 6yo child presents with hx of recurrent jaundice. Between the episodes he is totally fine.

Mother gives hx of jaundice being brought about by ongoing infections. What is the most likely

dx?

a. Hereditary spherocytosis

b. G6PD deficiency

c. Thalassemia

d. Sickle cell disease

e. Congenital storage disorder

key : B

reason : hemolytic attack triggered by infection + normal in between attacks >> G6PD deficiency

haemolytic anemia

DDx

Genetic

Red cell membrane abnormalities: hereditary spherocytosis, elliptocytosis.

Haemoglobin abnormalities: sickle cell anaemia, thalassaemia.


Enzyme defects: glucose-6-phosphate dehydrogenase (G6PD), pyruvate kinase deficiency.

Acquired

Immune:

- Isoimmune: haemolytic disease of newborn, blood transfusion reaction.

Autoimmune:

-Warm antibody : SLE

-Cold antibody type :

-Drug-related :

Non-immune: trauma ,infection , hypersplenism, membrane disorders, paroxysmal nocturnal


haemoglobinuria, liver disease.

G6PD

PRESENTATION :

Most are asymptomatic,, History of drug or infection induced haemolysis,,Gallstones are common, history of
neonatal jaundice ,During a crisis jaundice occurs,

investigations :

G6PD enzyme activity - is the definitive test

FBC : anemia ,macrocytosis , reticulocytosis during the attack

Blood film in acute attack : Heinz bodies

Haemolysis - reduced levels of haptoglobin and elevated levels of bilirubin; haemoglobinuria.

Ultrasound examination of the abdomen may reveal splenomegaly and gallstones.

Management of acute haemolysis :

blood transfusion may be needed,Dialysis may be required in acute kidney injury.

Infants - more susceptible to neonatal jaundice, especially if premature, and exchange transfusion may be
required.

Management of chronic haemolysis :

Splenectomy may help ,Supplementation with folic acid

Avoid : precipitating drugs, broad beans and naphthalene - found in mothballs.

- X-linked disease with about 300 variants reported

- Precipitating factors

- Certain drugs like Primaquine, Methylthioninium, Nitrofurantoin, Sulfonamides including co-Trimoxazole,


Dapsone etc.
- Certain foods like broad beans

- Severe infection

- DKA

- Acute Kidney Injury

- Presentation

- Pallor of anemia

- Jaundice during crisis

- Back or abdominal pain

- Splenomegaly may occur

- Investigations

* FBC - anaemia.

* Macrocytosis - due to reduced folic acid which is required for erythropoiesis.

* Reticulocyte count - raised; gives indication of the bone marrow activity (bone marrow sampling thus not
needed).

* Blood film - acute haemolysis from G6PD deficiency can produce Heinz bodies, which are denatured
haemoglobin and bite cells (cells with Heinz bodies that pass through the spleen have part of the membrane
removed).

* Haemolysis - reduced levels of haptoglobin and elevated levels of bilirubin; haemoglobinuria.

* Direct antiglobulin test - to look for other causes of haemolysis; should be negative in G6PD deficiency.

* Renal function - to ensure no renal failure as a precipitant.

* LFTs - to exclude other causes of raised bilirubin.

* G6PD enzyme activity - is the definitive test (as opposed to the amount of G6PD protein).

* Performing assays for G6PD during haemolysis and reticulocytosis may affect levels and not reflect baseline
values.

* Ultrasound examination of the abdomen may reveal splenomegaly and gallstones.

- Management

Avoidance of substances that may precipitate hemolysis is essential. Usually no further management is
required, although if hemolysis is marked there may be benefit from folate supplementation.

Management of acute haemolysis

* Seek specialised advice.

* Blood transfusions may be needed.


* Dialysis may be required in acute kidney injury.

* Infants - more susceptible to neonatal jaundice, especially if premature, and exchange transfusion may be
required.

Management of chronic haemolysis or stable disease

* Splenectomy may help.

* Supplementation with folic acid.

* Avoidance of precipitating drugs, and broad beans (usually favism occurs in the Mediterranean variety of the
disease).

* Avoid naphthalene - found in mothballs.

1132. A 42yo woman who smokes 20 cigarettes/d presents with complaints of heavy bleeding and

prolonged menstrual period. What is the most appropriate tx for her?

a. Tranexamic acid

b. COCP

c. Mefenamic acid

d. IUCD

e. Norethisterone

key : D

reason : smoker >>> so no use of COCP ,,, heavy bleeding >>> IUCD ( FULL TOPIC DICUSSED EARLIER)

The key says IUCD (D) but it is not the first choice in heavy/prolonged menstrual periods. Mirena Coil is. The
second line is Tranexamic Acid. Third line is COCPs. Fourth line is Endometrial ablation or Hysterectomy (If
there is no desire to conceive). IUCD, according to patient.info actually causes heavy or painful periods. The
logical choice in this case would be Tranexamic Acid since Mirena is not mentioned in the given options.

1133. A 17yo senior schoolgirl with complain of prolonged irregular menstrual period and heavy blood

losses. What is the most appropriate tx for her?

a. Mefenamic acid

b. COCP

c. POP

d. IUCD

e. Mirena
KEY : B

reason : irregular menses + heavy bleeding >>> COCP can treat both (Full topic discussed earlier)

- COCPs are widely used for irregular menstrual periods.

- They also carry the advantage of causing a decrease in bleeding and menstrual pain (can be used for
dysmenorrhea and menorrhagia)

1134. A 32yo presents with heavy blood loss, US: uterine thickness>14mm. What is the most

appropriate tx for her?

a. Mefenamic acid

b. COCP

c. POP

d. IUCD

e. IU system (mirena)

key :E

reason : heavy bleeding >>> IUS is the first line (full topic discussed earlier)

- Mirena, also known as Intrauterine System, is a Levonorgestrel-containing coil which is inserted into the
uterus.

- Local effect: reversible endometrial atrophy, makes implantation less likely and periods lighter (20%
reversible amenorrhea)

- Less risk of ectopic pregnancy

- Risk of STDs is reduced

- May benefit women with endometriosis, adenomyosis, fibroids or endometrial hyperplasia

- NOTE: Avoid if breast cancer.

1135. A 37yo woman presents with heavy bleeding. Inv show subserosal fibroid=4 cm and intramural

fibroid=6cm. Which is the most appropriate tx?

a. UAE

b. Abdominal hysterectomy

c. Hysteroscopic Myomectomy

d. Vaginal Hysterectomy
e. Abdominal myomectomy

key : e

reason : abdominal myomectomy is the best alternative for hysterectomy in patients who want to keep their
fertility ,, pt is 37 ys and this procedure can treat both intramural and subserosal fibroids (full topic discussed
earlier)

- Fibroids are responsive to estrogen and therefore increase in size, which in turn increases the size of the
uterus.

- Other symptoms

- Pelvic pain (Compression on to adjacent structures)

- Infertility/Recurrent Miscarriages

- Pelvic Mass

- The investigation of choice is an U/S

- Management

- Mirena Coil is the first choice if the fibroids are not big enough to restrict its insertion.

- If < 3 cm

- Trial of pharmacologic treatment first (Tranexamic Acid) first

- If it fails and uterus is not bigger than 10-week pregnancy, do endometrial ablation

- If the above fails, perform a hysterectomy

- If > 3 cm and wishes to retain uterus and/or wants to avoid surgery

- Go for Uterine Artery Embolization

- If > 3 cm and wishes to retain uterus, go for a hysteroscopic myomectomy or a myomectomy

NOTE: This patient has a subserosal fibroid so an abdominal approach should be adopted.

1136. A woman with sickle cell disease complains of heavy menstrual blood loss. What is the most

appropriate tx?

a. COCP

b. Mirena

c. Depot provera

d. Copper IUS

e. Transdermal patch

key : c
reason : sickle cell disease :

intrauterine devices are not recommended, as they may be associated with uterine bleeding and infection.

Combined hormonal methods are not recommended , because of the risk of thromboembolism in sickle cell
patients

Depot contraceptive (Depo-Provera) is safe and has been found to improve the blood picture and reduce
pain crises.[15]

The choice of contraceptive method needs to be considered carefully. The coil (intrauterine contraceptive
device) may cause particularly heavy painful periods. The use of injectable contraceptives (such as Depo-
Provera) has been reported to provide some protection against sickling episodes.

1137. A 70yo woman is admitted with diarrhea, vomiting and dehydration. Exam: yellow visual halos in

her eyes, ECG=bradycardia. She has a hx of chronic A-fib. Which drug causes the above

mentioned side effects?

a. Nifedipine

b. Ramipril

c. Atenolol

d. Lithium

e. Digoxin

key : e

reason : symptoms are classic for digoxin toxicity

Features suggestive of toxicity include nausea, vomiting, diarrhoea, dyspnoea, confusion, dizziness, headache,
blurred vision and diplopia ( yellow halos),bradycardia, skin rash, renal dysfunction and hypokalaemia

Plasma concentrations is helpful when initiating therapy, checking compliance or detecting toxicity. Levels
above 2 nmol/L suggest toxicity.

- Adverse Effects of Digoxin

- Diarrhea

- Nausea

- Vomitting

- Dizziness

- Headache

- Maculopapular rash
- Cardiac dysrhythmia

- Arrhythmia in children

- Visual disturbances (Blurred or yellow vision)

- Heart Block

- Asystole

1138. A 33yo lady who is a drug addict wants to quit. She says she is ready to stop the drug abuse. She

is supported by her friends and family. What drug tx would you give her?

a. Benzodiazepines

b. Diazipoxide

c. Lithium

d. Methadone

e. Disulfiram

key : D

reason :methadone is used to treat opioid withdrawal symptoms

Methadone or buprenorphine can be used in opioid dependance treatment

NICE recommends that, if both drugs are equally suitable, methadone should be prescribed as first choice

- Methadone is used as a pain reliever and as part of drug addiction detoxification and maintenance programs.

- For detoxification during withdrawals Methadone is the first choice.

- Methadone is also used for maintenance.

- Benzodiazepines, Diazepoxide and Disulfiram are specifically used in alcohol withdrawal.

- Lithium is used in Mania and Bipolar Affective Disorder.

1139. A 50yo lady has been suffering from chronic RA and is on methotrexate and naproxen. Her CBC

shoes microcytic anemia. What is the most likely cause?

a. Anemia of chronic disease

b. GI hemorrhage

c. Menorrhagia

key : B
reason: because of use of NSAIDs >>> naproxen

anemia of chronic illness is normocytic normochromic

microcytic anemia >>> iron deficiency due to chronic blood loss >>> GI hrg

- There is a history of chronic NSAID use, this leads to weakening of defense mechanism of the mucosa of the
stomach.

- This leads to exposure of the mucosa to gastric acid and causes ulcers.

- Bleeding from these ulcers can lead to anemia.

1140. A 15yo male noticed swelling on the left knee following a fall while playing. The swelling has not

subsided in spite of rest and analgesia. Exam: full knee movement with slight tenderness. He has

painless palpable mass in left inguinal region. What is the most probable dx?

a. Osteosarcoma

b. Ewings sarcoma

c. Chondrosarcoma

d. Lymphangiosarcoma

e. Osteodosteoma

key : A

reason : age ( highest in 15-19 years) + site ( around knee) + LN metastasis

Osteosarcoma

The most common primary bone malignancy in children. The incidence is highest in 15-19 ys

The male:female ratio is 1.4:1.

The most common sites are around the knee (75%), or proximal humerus.

Often presents as a relatively painless tumour.

Rapidly metastasises to the lung >>associated with a poorer prognosis

X-ray shows combination of bone destruction and formation. Soft tissue calcification produces a
'sunburst' appearance.

Disease-free survival has increased to 55-75% with surgery and effective


chemotherapy ,Chemotherapy alone is not as effective.

Presentation

pain, swelling and localised tenderness,Rapid growth and erythema ,pathological fractures
Investigations : Plain X-ray ,MRI and CT scan ,Biopsy

Treatment : surgery + chemotherapy

Osteosarcoma

This is the most common type of primary bone cancer but even this is rare. It only affects around 150 people a
year in the UK. It arises from bone-forming cells. Most cases occur in young people between the ages of 10 to
25. It can, however, occur at any age. It typically develops in the growing ends of the bone in young people,
most commonly in bones next to the knee and the upper arms. However, any bone can be affected.

Ewing's sarcoma

The cells of this cancer look different to the more common osteosarcoma. It only affects around 100 people a
year in the UK. Most cases occur in young people between the ages of 10 to 20, but it can occur at any age. It
most commonly affects the hips (pelvis) and long bones in the leg.

- The age and symptoms fit both the above conditions.

1141. A 45yo female looking pale has bluish discoloration of hands whenever she goes out in the cold.

She has also noticed some reddish spots on her body. She has symmetrical peripheral

arthropathy for the last yr. What is the most probable dx?

a. RA

b. Osteosarcoma

c. Limited systemic sclerosis

d. Diffuse systemic sclerosis

e. Chondrosarcoma

key : C

reason : classic picture of limited SSc

it's a disease of connective tissue disease like as RA, SLE. so arthritis here and symmetrical arthritis. two
variety of Systemic sclerosis. LCSS- Involvement distal to the knee and elbows with CREST syndrome, DCSS-
involvement proximal to knee and elbows with renal involvement (scleroderma renal crisis ). presence of
telangiectasia and Raynaud pheno indicates LCSS.in RA, arthritis present but not raynauds pheno. Raynaud's
also present in SLE.

scleroderma

Common presenting symptoms are Raynaud's phenomenon, skin hardening in hands or face, and oesophageal
symptoms.

types of SSC
Limited scleroderma >>> Generally a milder disease, with less skin involvement, slow onset and slow
progression.

70% of SSc cases.

Affects only the face, forearms and lower legs up to the knee.

The older term for limited scleroderma is CREST syndrome (= Calcinosis, Raynaud's disease, (O)Esophageal
dysmotility, Sclerodactyly, Telangiectasia).

Diffuse scleroderma >>> Usually a more rapid onset, with skin thickening and Raynaud's phenomenon
occurring together or within a short interval

30% of SSc cases ,, Involves also the upper arms, thighs or trunk

- Formerly known as the CREST Syndrome

- C: Calcinosis

- R: Raynauds Phenomenon

- E: Esophageal and Gut dysmotility

- S: Sclerodactyly

- T: Telangiectasias (Reddish Spots)

1142. A 60yo female has pain and stiffness in her right hip joint. Pain is not severe in the morning but

increases as the day progresses. She has noticed some nodules in her hands. Inv: Hgb=low. What

is the most probable dx?

a. RA

b. Osteoarthritis

c. Gout

d. Pseudogout

e. Multiple myeloma

key : B

reason : A diagnosis of OA can be made clinically without investigations if a person:

Is aged 45 years or over; and

Has activity-related joint pain; and

Has either no morning joint-related stiffness or morning stiffness that lasts no longer than 30
minutes.
osteoarthritis

Symptoms

Joint pain that is exacerbated by exercise and relieved by rest. Rest and night pain can occur in advanced
disease. Knee pain due to OA is usually bilateral and felt in and around the knee. Hip pain due to OA is felt in
the groin and anterior or lateral thigh. Hip OA pain can also be referred to the knee and, in males, to the
testicle on the affected side.

Joint stiffness in the morning or after rest.

Signs

Pain and Reduced range of joint movement.

Joint swelling/synovitis (warmth, effusion, synovial thickening).

Bony swelling and deformity due to osteophytes

- Commonest joint condition

- Usually monoarthritis

- Usually affects females 50 years and above

- Commonly affects the weight-bearing joints e.g, Hip and knee joints. (Notice the joint involved)

- Pain on movement and worsening towards the end of the day

- Bouchards nodes and Heberdens nodes are usually present.

- Investigations

- X-ray

- Treatment

-Advise exercise and activity, and physiotherapy

- Paracetamol

- Topical NSAIDs

- Intra-articular steroids

- Low-dose of tricyclic antidepressants for pain at night

- Weight reduction

- Joint replacement in end-stage Osteoarthritis

1143. A 30yo female has chronic diarrhea, mouth ulcers and skin tags. She complains of visual prbs,

low back pain and morning stiffness. Inv: ESR and CRP=raised, Hgb=10 mg/dl. What is the most
probable dx?

a. SLE

b. Reactive Arthritis

c. Gout

d. Pseudogout

e. Seronegative arthritis

key : B

reason : the symptoms fits ( GIT or urinary tract symptoms >> diarrhea + law back pain + visual problems >>
uveitis + lab >> anemia and inreased ESR & CRP )

Reactive arthritis or Reiter's syndrome >>> is a form of seronegative spondyloarthritis clinically associated
with inflammatory back pain, GIT symptoms . The presence of large joint oligoarthritis, urogenital tract
infection and uveitis characterises Reiter's syndrome as a clinical subtype of reactive arthritis.

Presentation

develops 2-4 weeks after a genitourinary or gastrointestinal infection.

The onset is most often acute, with malaise, fatigue, and fever.

An asymmetrical, predominantly lower extremity, Low back pain often occurs.

The complete Reiter's triad of urethritis, conjunctivitis, and arthritis may occur.

Skin ,nails and mucous membranes (mouth ulcers) may all be affected.

Eyes: uveitis, episcleritis, keratitis, and corneal ulcerations.

Gastrointestinal: abdominal pain and diarrhoea

Investigations :

**Once arthritis is observed, microbial tests and blood or synovial fluid cultures are negative, and only serum
antibodies are detected.

ESR and CRP are usually very high.

FBC: normocytic normochromic anaemia

HLA-B27 is positive

Management

In the acute phase, rest affected joints, aspirate synovial effusions.

Physiotherapy.

Non-steroidal anti-inflammatory drugs (NSAIDs).

Corticosteroids
Antibiotics to treat an identified causative organism

1144. A 28yo woman has been on tx for RA for 3yrs. She has gradual loss of vision in both eyes. Her

IOP is normal. Red reflex is absent in both eyes. What is the single most likely dx?

a. Cataract

b. DM retinopathy

c. Hypermetropia

d. Macular degeneration

e. HTN retinopathy

key : A

reason : absent red reflex + tx for RA for 3yrs+ gradual loss of vision in both eyes.

the treatment plan of rheumatoid arthritis includes : corticosteroids which induce cataract formation with
long term use

- Cataracts are cloudy (opaque) areas that develop in the lens of an eye and affects vision

- Vision becomes gradually worse over the years.

- Most affected people develop a cataract for no apparent reason. Factors that may increase the chance of
developing cataracts include:

- Having a poor diet.

- Smoking.

- Being exposed to a lot of ultraviolet light.

- Diabetes.

- Steroid medicines.

- Having a family history of cataracts

- There are no medicines, eye drops or lasers that can treat cataracts. The only way of treating cataracts is
with an operation. This is a very common operation.

1145. An elderly man with recently dx HF has been treated with diuretics. He now develops severe

joint pain in his left ankle with swelling and redness. What is single most likely inv?

a. XR of bone

b. Plasma RF
c. Joint fluid uric acid crystals

d. ESR

key : c

reason : both thiazides and loop diuretics can precipitate or worsen pre-existing gout. If a diuretic is
unavoidable, consider prophylaxis with allopurinol.

Gout

Primary gout occurs mainly in men aged 30-60 years presenting with acute attacks.

secondary gout is due to chronic diuretic therapy. It occurs in older subjects, both men and women, and is
often associated with osteoarthritis.

Risk factors :

Male sex ,Meat ,Seafood ,Diuretics ,Obesity ,Hypertension ,Coronary heart disease ,Diabetes mellitus ,Chronic
renal failure, High triglycerides

Presentation:

50% of all attacks and 70% of first attacks affect the first metatarsophalangeal joint.

Other sites often affected are: Knee, Midtarsal joints ,Wrists, Ankles ,Small hand joints ,Elbows

Management :

colchicine and/or NSAIDs as the first-line option for acute gout.

- There is a clear history of diuretic use which causes Hyperuricemia in almost 40% of the patients.

- Hyperuricemia can lead to development of gout (Notice the redness, severe pain, swelling and involvement
of a single joint).

- Gout is usually precipitated by Trauma, Tumor Lysis Syndrome, Surgery, Infection, Diuretics, Polycythemia,
Leukemia, Cytotoxic Drugs and Alcohol Abuse

- Investigation

- Joint aspiration for microscopy, C/S, which shows negatively birefringent crystals

- Treatment

- NSAIDs

- If contraindicated, give Colchicine

- If there is renal failure, then both NSAIDs and Colchicine are problematic, so use steroids.

1146. A 60yo lady with a hx of HTN and suffering from RA since the last 10y now presents with hot,
swollen and tender knee joint. What inv would you do for her?

a. XR

b. C&S of joint aspirate

c. US

d. MRI

e. CT

key : B

reason : septic arthritis is the most important diagnosis to exclude as, if left untreated, the sequelae include
permanent joint damage, impairment of function and even death

DDx of hot swollen tender joint

Infection:

Septic arthritis >> acute onset , monoarthritis, mostly knee in adults and hip in children , plus
constitutional symptoms ( fever , malaise)

Neisseria gonorrhoeae

Lyme disease >> erythema migrans

Rheumatoid arthritis >>> insidious onset ,polyarthritis

Crystal arthropathies - gout and pseudogout >>> acute onset , monoarthritis mostly, 70% of attacks first occur
in the big toe in gout

Reactive arthritis (now considered synonymous with Reiter's syndrome). >>> polyarthritis , Hx of
gastrointestinal or genitourinary infection

Trauma >>> Hx of trauma

- History of Rubor, Erythema and swelling in a single joint is Septic Arthritis until proven otherwise.

- Therefore the investigation of choice would be C/S of joint aspirate.

- Rest of the investigations do not help in making the diagnosis or exclusion of septic arthritis.

1147. A 34yo man after an RTA was brought to the ED. He has BP=50/0mmHg and chest wall with

asymmetrical movement, RR=34bpm. What would be the initial action?

a. IV fluid infusion

b. Intubation and ventilation

c. CT chest

d. Transfer to ITU
key : B

reason : in trauma patient we follow : Airway ,Breathing ,Circulation ,Disability , Exposure>> and that patient
has asymmetrical chest movement and increased RR>>> we have to secure the air way and ventilate the pt

Initial assessment

the 'ABCDE' principles :

Airway maintenance with cervical spine protection

Breathing and ventilation

Circulation with haemorrhage control

Disability: neurological status

Exposure/environmental control

1148. A 7yo presented with chronic cough and is also found to be jaundiced on exam. What is the

most likely dx?

a. Congenital diaphragmatic hernia

b. Congenital cystic adenomatoid malformation

c. Bronchiolitis

d. RDS

e. Alpha 1 antitrypsin deficiency

key : E

reason : clinical picture fits >>> The organs most commonly involved are the lungs and the liver.

Alpha 1 antitrypsin deficiency

A1AT deficiency is an inherited condition.

In A1AT deficiency, the protein is still produced but the A1AT molecule configuration is changed. As
a result, it cannot pass out of the liver into the bloodstream and so cannot pass to the lungs and
the rest of the body

Some people with A1AT deficiency develop liver disease. This results from the congestion of A1AT
in the liver cells, leading to cell destruction.

If there is a deficiency of A1AT then elastase can break down elastin unchecked; in the lungs this
can lead to the destruction of alveolar walls and emphysematous change >>> COPD

Neonates with A1AT deficiency may present with neonatal jaundice and hepatitis; older children
may develop hepatitis, cirrhosis and liver failure due to A1AT deficiency.
Investigations

Serum levels of alpha-1 antitrypsin

Phenotyping

CXR and lung function testing ,CT scanning of the chest.

LFTs and possibly liver biopsy.

- It is an autosomal recessive inherited disorder

- It commonly affects lungs (Emphysema) and liver (Cirrhosis and Hepatocellular Carcinoma)

- Investigation

- Serum alpha 1 antitrypsin levels

- Management

- Supportive treatment for Emphysema and Liver disease may be sufficient for some.

- Among the given options no condition can explain the involvement of lungs and liver at the same time.

1149. A 65yo man had a bowel resection 5d ago. He is anuric and breathless. His BP=150/110 mmHg.

He has crackles at both lung bases and sacral edema. Bloods: K+=6.8mmol/l, urea=58 mmol/l,

creatinine=600 umol/l. What is the single most appropriate immediate management?

a. Bolus of 20U insulin

b. Calcium resonium enema

c. Dextrose-saline infusion

d. 5% dextrose infusion

e. 10U insulin, 50ml of 50% dextrose infusion

key : e

reason : this is a case of acute kidney injury following major surgery ( increased urea and creatinine +
hyperkalemia ( normal range 3.5-5 ) >>> to treat hyperkalemia >>> Shift potassium into cells using 10U insulin,
50ml of 50% dextrose infusion

Hyperkalaemia

normal k+ 3.5-5 mmol/L

Mild Hyperkalaemia - 5.5-5.9 mmol/L

Moderate Hyperkalaemia - 6.0-6.4 mmol/L


Severe Hyperkalaemia - >6.5 mmol/L

Causes:

Renal causes: eg. Acute kidney injury (AKI) ,Chronic kidney disease (CKD)

Increased circulation of potassium:

Exogenous - eg, potassium supplementation.

Endogenous - eg, tumour lysis syndrome, rhabdomyolysis, trauma, burns.

A shift from the intracellular to the extracellular space:

Acidosis - eg, diabetic ketoacidosis (DKA).

Medications - eg, digoxin toxicity, suxamethonium, beta-blockade, theophylline.

ECG changes in hyperkalemia : Peaked T waves ,prolonged PR interval ,Wide QRS,bradycardia.

Management :

Stop further potassium accumulation: Stop any potassium supplements stop digoxin and beta-
blockers,Decrease potassium in the diet

Protect cardiac membrane: Give 10 ml 10% calcium gluconate

Shift potassium into cells: Insulin-glucose IV >> usually 10 units of Actrapid are added to 50 ml of
glucose 50% and infused over 30 minutes.

Remove potassium from the body : Calcium polystyrene sulfonate resin (Calcium Resonium) with
regular lactulose will remove potassium via the gastrointestinal tract.

** in resistant hyperalaemia Haemodialysis may be required but is invasive.

1150. A 25yo woman presents with a painful shallow ulcer on the vulva. What inv has to be done?

a. HSV antibodies

b. Syphilis serology

c. Swab for haemophilus ducreyi

d. Urine culture

e. Blood culture

key : c

reason : shallow painful ulcer >> chancroid

chancroid:
Chancroid is a sexually transmitted disease (STD) caused by haemophilus ducreyi characterized by painful
shallow with soft ragged margins necrotizing genital ulcers that may be accompanied by inguinal
lymphadenopathy.

- The symptoms point towards the causative organism being Haemophilus Ducreyi as it causes painful shallow
ulcers.

- Mnemonic: YOU CRY WITH DUCREYI

- The other conditions cannot explain the symptoms above as they do not cause painful shallow ulcers. HSV
cause vesicles while Syphilis causes painless ulcers.

- Urine and Blood Culture are not required for the same reason as stated above.

1151. A child was admitted with fever, generalized skin lesion, some of them are weeping lesions and

some of them are crusted. What is the most probable dx?

a. Varicella

b. Impetigo

c. Drug reaction

d. Contact dermatitis

e. Scabies

Impetigo (B)

-Infection due to Staph. Aureus.

- Usually on the face with honey-coloured fluid in an erythematous base.

- Common in children.

- Children should be kept off school or nursery until there is no more blistering or crusting or until 48 hours
after antibiotic treatment has been started.

-Treatment

- Flucloxacillin

1152. A pt comes with 6m hx of painless bilateral swelling of the face which has been progressively

increasing in size. On routine CXR, he is found to have perihilar lymphadenopathy. What is the

most probable dx?

a. Chronic sialadenitis

b. Thyroid adenoma
c. Carcinoma of salivary gland

d. Adenoid cystic carcinoma

e. Mikuliczs disease

Mikuliczs Disease (E) - Bilateral parotid and lacrimal gland enlargement was characterized by the term
Mikulicz's disease if the enlargement appeared apart from other diseases.

- In 80% of cases, the parotid gland is affected. Lacrimal glands are also affected. - The gland affected has a
diffuse swelling. The swelling can be asymptomatic, but mild pain can also be associated. - A biopsy is needed
to distinguish benign lymphoepithelial lesions from sialadenosis (sialosis).

- Treatment

- Treatment usually consists of observation unless the patient has concern, there is pain, drainage, or other
symptoms related to the lesion. Surgical removal of the affected gland would be recommended in those cases.

1153. A woman has widespread metastasis from a carcinoma. She presented with severe back pain.

Where do you expect the cancer to be?

a. Lungs

b. Cervix

c. Ovary

d. Uterus

e. Breast

E - Breast.

- Occasionally, breast cancer presents as metastatic diseasethat has spread beyond the original organ. The
symptoms caused by metastatic breast cancer will depend on the location of metastasis. Common sites of
metastasis include bone, liver, lung and brain.

- Bone is the commonest site of metastasis in Breast Cancer

1154. A 10yo child has got progressive bilateral hearing loss. He has started to increase the TV volume.

All other examination is normal. What is the most likely dx?

a. Wax

b. Foreign body

c. Bilateral OM with effusion

d. SNHL
e. Meningitis due to meningococcus

Progressive Bilateral Hearing Loss in a Child (C)

- Otitis media is an inflammation in the middle ear (the area behind the eardrum) that is usually associated
with the buildup of fluid. The fluid may or may not be infected.

- Symptoms, severity, frequency, and length of the condition vary. At one extreme is a single short period of
thin, clear, non-infected fluid without any pain or fever but with a slight decrease in hearing ability. At the
other extreme are repeated bouts with infection, thick "glue-like" fluid and possible complications such as
permanent hearing loss.

- Fluctuating conductive hearing loss nearly always occurs with all types of otitis media. In fact it is the most
common cause of hearing loss in young children.

Management General advice

* Give written information about OME to the parents.

* Advise parents or carers not to expose the child to tobacco smoke.

NICE recommends hearing aids for children with bilateral OME and hearing loss where surgery is not
acceptable or is contra-indicated. Each case needs to be considered on its own merits: the need to assist
hearing during a period of active observation, for example, has to be weighed against evidence that the use of
aids in children can increase anxiety.

1155. A child had a patchy rash following tx for sore throat & cervical LN enlargement. Which is the

most appropriate antibiotic?

a. Ampicillin

b. Erythromycin

c. Cefuroxime

d. Metronidazole

e. Tetracycline

EBV infection (A)

NOTE: The question here is not asking for appropriate treatment. It is actually trying to ask about the drug that
has caused a rash in a child with the given symptoms.

- A child with a sore throat and Cervical Lymphadenopathy has an active EBV infection until proven otherwise.
- Ampicillin and amoxicillin are contraindicated during acute EpsteinBarr virus infection since the vast
majority of patients treated with them develop a diffuse non-allergic rash.

1156. A child with a hx of asthma is brought to ED with a cut on knee and sprained on her left wrist.
Which is the best analgesic for her?

a. Paracetamol

b. NSAIDs

c. Co Codamol

d. Ibuprofen

A - Paracetamol

Paracetamol. paracetamol should only be given in this patient to relieve pain. NSAIDS should not be given as
the child has h/o asthma as nsaids may increase the risk of acute bronchospasm and co codamol has codeine
which can cause respiratory depression.

1157. A 15m baby girl presented to the ED with difficulty in breathing. Exam: she has intercostal

recessions and a wheeze. Temp=normal. What is the most likely dx?

a. URTI

b. Pneumonia

c. Bronchiolitis

d. RDS

e. Alpha 1 antitrypsin deficiency

key : c

reason : age : (younger than 2 years (most common between 2 and 6 month) + wheezy chest + intercostal
recession + normal temp )

Bronchiolitis is an acute infectious disease of the lower respiratory tract that occurs primarily in the very
young, most commonly infants between 2 and 6 months old.

It is a clinical diagnosis based upon: Breathing difficulties , Cough , Decreased feeding ,Irritability ,Apnoeas in
the very young ,Wheeze or crepitations on auscultation

the causative organism :

Respiratory syncytial virus (RSV) : the most common cause

Human metapneumovirus (hMPV) - the second most common cause

Investigations:

Pulse oximetry.

Nasopharyngeal aspirate for: RSV rapid testing

Viral cultures for RSV, influenza A and B, parainfluenza and adenovirus


Management:

Most infants with acute bronchiolitis will have mild, self-limiting illness and can be managed at home.
Supportive measures , with attention to fluid input, nutrition and temperature control.

1158. An 8yo boy develops a seizure affecting his right arm, seizure lasts for several mins. He doesnt

remember anything what happened. On his CT: lesion in left hemisphere. What is the most

probable dx?

a. Epilepsy

b. Space occupying lesion

c. Dementia

d. Huntingtons chorea

e. Intracranial HTN

key : b

reason : CT lesion >>> Space occupying lesion

brain tumors in children

the most common subtype >>> astrocytoma followed by

the second most common >>> embryonal tumours ( primitive neuroectodermal tumours and
medulloblastoma)

Presentation:

increased intracranial pressure : Headache, nausea and vomiting, abnormalities of gait and
coordination, and papilloedema.

frontal lobe tumours are associated with personality change and occipital lobe tumours are
associated with visual deficits

Brainstem tumours: Abnormal gait and coordination, cranial nerve palsies, pyramidal signs,
headache and squint.

Central brain tumours: Headache, abnormal eye movements, squint, and nausea and vomiting

Supratentorial tumours: Unspecified symptoms and signs of raised intracranial pressure, seizures
and papilloedema.

Posterior fossa tumours: Nausea and vomiting, headache, abnormal gait and coordination, and
papilloedema

investigations :

MRI , CT : MRI is better , it provides better images and there is no radiation involved.
excision biopsy

1159. A 28yo female presented with complains of difficulties in swallowing liquids only. She also

suffers from recurrent chest infection in the past few months. What is the most probable dx?

a. Foreign body

b. Plummer vinson syndrome

c. Achalasia cardia

d. Peptic stricture

e. Esophageal carcinoma

key : c

reason : liquid dysphagia + recurrent chest infection ( due to regurgitation)

Achalasia is a motility disorder of the lower oesophageal or cardiac sphincter. The smooth muscle layer of the
oesophagus has impaired peristalsis and failure of the sphincter to relax causes a functional stenosis

presentation :

The most common presenting feature is dysphagia. This affects solids more than soft food or
liquids.

Regurgitation , Chest pain , Heartburn ,

Nocturnal cough and even inhalation of refluxed contents >> recurrent chest infection

Investigations :

Manometry is the gold standard for diagnosis of achalasia.

CXR : The classical picture of a CXR in achalasia shows a vastly dilated oesophagus behind the heart

Barium swallow : the bird beak appearance

Endoscopy : can detect approximately a third of achalasia

lowe oesophogeal PH monitoring >> to exclude GERD

MANAGEMENT :

the Heller myotomy is the best treatment for those who are fit

Pneumatic dilatation is the preferred option for older unfit patients

Calcium-channel blockers and nitrates may be used for those who are unable to tolerate other
forms of treatment

Endoscopic injection of botulinum toxin >>> recurrence

1160. Mother having 2 children with CF. What is the risk of getting another baby?
a. 1:2

b. 1:8

c. 1:4

d. 1:16

e. 1:1

key : c

reason : cystic fibrisis is autosomal recessive gene so :

Cc Cc

CC Cc Cc cc

normal carrier diseased

1161. A 14yo boy has been dx with nephrotic syndrome. 5d later he presents with flank pain,

hematuria and fluctuating urea levels. A dx of renal vein thrombosis is made. What is the most

likely cause for renal vein thrombosis?

a. Protein C deficiency

b. Vasculitis

c. Loss of antithrombin III

d. High estrogen levels

e. Stasis

key : c

Complications of nephrotic syndrome include:

Decreased resistance to infections, due to urinary immunoglobulin loss.

Increased risk of arterial and venous thrombosis, due to loss of antithrombin III and plasminogen in
the urine, combined with an increase in hepatic synthesis of clotting factors. Adults with
membranous nephropathy are at particular risk

Acute kidney injury

Chronic kidney disease may occur as a result of an underlying cause - eg, amyloidosis or diabetes.

Increased risk of osteitis fibrosa cystica and osteomalacia due to loss of vitamin D-binding protein

1162. A 36yo woman presented with massive bleeding from multiple sites. Lab: fibrin degradation

products: +++, plt=30, bleeding time=prolonged, PT=prolonged, APTT=prolonged. What is the


most likely dx?

a. Hemophilia

b. DIC

c. ITP

d. Factor V leiden

e. Warfarin

key : B

cause : lab criteria ( decreased fibrinogen and platelets +increased PTT,PT )

full topic discussed earlier

1163. A study was done amongst 2 hosp for the equal number of cancer pts. It was noted that hosp A

had the higher rate of mortality than hosp B for treated cancer pts. What is the study done here

classified as?

a. Retrospective

b. Observational

c. Cohort

d. Case study

key : c

Longitudinal or cohort studies:

A group of people is followed over many years to ascertain how variables such as smoking habits,
exercise, occupation and geography may affect outcome.

Prospective studies are more highly rated than retrospective ones, although the former obviously
take many years to perform. Retrospective studies are more likely to produce bias.

1164. A 17yo girl comes to see her GP after having unprotected sex 2d ago. She asks if her GP can

explain to her how this prescribed procedure would work by helping her not to get pregnant.

a. It helps to prevent implantation

b. It helps in preventing or delaying ovulation

c. It causes an early miscarriage

d. It releases progesterone and stops ovulation

e. It causes local enzymatic reaction

key:A
reason : CU IUD prevents implantation

emergency contraception:

Progestogen-only - levonorgestrel >>> early in the cycle inhibits ovulation. later in the cycle, it is
unclear how it has its effect. use within 72 hours of UPSI

ulipristal acetate >>> inhibits ovulation. effective up to 120 hours after UPSI. Pregnancy or
suspected pregnancy should be excluded before use

IUCD >>> inhibitory effect on both fertilisation and implantation ( direct toxicity effects of the
copper on both ovum and sperm). up to five days after UPSI

1165. A 2d babys mother is worried about the babys hearing. Mother has a hx of conductive hearing

loss. What is the most appropriate test?

a. Brain stem evoked response

b. CT

c. Fork test

d. MRI

e. Reassure

KEY :A

Neonatal hearing screening tests

Automated otoacoustic emissions (AOAE) test >>> measures the integrity of the inner ear

Automated auditory brainstem responses (AABR) test >>> measures not only the integrity of the
inner ear, but also the auditory pathway.

1166. A healthy 8yo boy had antibiotic tx for meningitis. Initially he wasnt resuscitated. What will be

the outcome if he receives full tx?

a. He will recover fully to his prv health

b. He will have hearing impairment

c. He will have brain abscess

d. He will have encephalitis

key : a

1167. A pt presented with jaundice, fever and upper abdominal pain within 24h after removal of

gallstone by ERCP. The cholangiography was done and it was patent. What is the possible cause
of his complaints?

a. Biliary infection

b. Acute pancreatitis

c. Perforation

key : b

cause : post ERCP pancreatitis

ERCP Complications

Pancreatitis - 2-9% of patients will develop pancreatitis , perioperative indomethacin or


diclofenac reduce the incidence of pancreatitis.

Infection may occur - although rates are low.

Bleeding may occur - although severe haemorrhage is rare.

Perforation of the duodenum with development of an acute abdomen.

Failure of gallstone retrieval - may need to revert to open or more invasive procedures.

Prolonged pancreatic stenting is associated with stent occlusion, pancreatic duct obstruction and
pseudocyst formation.

1168. A mother presents with her 14m child. He holds furniture and other things to help him stand

and walk. He can say mama and papa. He makes eye contact and smiles. He can transfer

objects from one hand to another. He responds to his name. what do you interpret from his

development?

a. Delayed gross motor development

b. Delayed fine motor development

c. Dela

yed verbal development

d. Normal development

e. Delayed social development

KEY : D
1169. A young child, 3yo, has presented with vomiting for 3d. Exam: mild-mod dehydration. What is

his ABG profile likely to show?

a. pH low, PCO2 low

b. pH low, PCO2 high

c. pH high, PCO2 low

d. pH high, PCO2 high

e. pH normal, PCO2 normal

key : d

reason : vomiting causes metabolic alkalosis ( high ph ), and it is compansated by increased PCO2

1170. A 68yo woman has been admitted with poor appetite, weight loss, poor concentration and self

neglect for 3wks. She has not been eating or drinking adequately and has rarely left her bed. She

is expressive suicidal ideas and is convinced that people are out to kill her. She has been on

antidepressant therapy for the past 3m with no improvement. What is the most appropriate tx?

a. Antidepressants

b. CBT

c. Interpersonal therapy

d. ECT

e. Antipsychotics

KEY : D

REASON : suicidal thought is an indication of ECT specially after treatment failure

Indications of ECT

Severe depressive illness or refractory depression.

Catatonia.

A prolonged or severe episode of mania.

It should only be used if other treatment options have failed or the condition is potentially life-
threatening (eg, personal distress, social impairment or high suicide risk).

ECT is not useful in schizophrenia


1171. A 78yo retired teacher was admitted for a hernioplasty procedure. After the operation he

became agitated, aggressive and confused. What is the most appropriate management?

a. Diazepam

b. Chlordiazepoxide

c. Vit B

d. Clozapine

e. Thiamine

key : b

reason :delirium tremens >>> first line >>> chlordiazepoxide

second line >>> diazepam

Delirium tremens:

Delirium tremens usually begins 24-72 hours after alcohol consumption has been reduced or stopped,,there
are signs of altered mental status eg, Hallucinations ,Confusion, Delusions ,Severe agitation , Seizures can also
occur.

1172. A 25yo girl saw a tragic RTA in which a young boy was killed. The night of the event she couldnt

sleep and the day after she suddenly lost her vision. She was prv fine and there was no hx of

medical or psychological prbs. What is the dx?

a. Conversion

b. Somatization

c. PTSD

d. Dissociation

e. GAD

key : a

reason : sudden lost her vision after she saw the accident >>> physical symptom after psychological trauma
>>> conversion

somatization >>> This is a chronic condition in which there are numerous physical complaints. These
complaints can last for years and result in substantial impairment.

1173. A 25yo man has been suffering from breathlessness and wheeze for 3m. He has been taking

salbutamol 2puffs as required. In the last 2 wks his symptoms have worsened and he has to take

salbutamol more frequently during the day time. He also complains of excessive dyspnea at
night. What drugs or regimen would you like to add?

a. Prednisolone

b. Fluticasone + salbutamol inhaled

c. Beclomethasone inhaled

d. Montelukast PO

e. Salmetrol PO

key : c

reason :

asthma management in adults :

Step 1: mild, intermittent asthma >>> inhaled short-acting beta2 agonist

Step 2: introduction of regular preventer therapy >>> Inhaled steroids ( beclomethasone )are the
most effective preventer drugs

indications :

A recent exacerbation ,Nocturnal asthma , Daytime symptoms or use of an inhaled short-acting beta2
agonist more than three times per week.

Step 3: add-on therapy >>> inhaled long-acting beta2 agonists (LABAs) such as salmeterol or
formoterol.

Step 4: poor control on moderate dose of inhaled steroid plus add-on therapy >>> Trial an
additional fourth drug over six weeks (eg, leukotriene receptor antagonist, sustained-release
theophylline or beta2 agonist tablet) and increase the inhaled steroid to high-dose ranges.

Step 5: continuous or frequent use of oral steroids >>> he use of daily steroid tablet in the lowest
dose providing adequate control is suggested

1174. A 64yo man who was exposed to asbestos for 40yrs presents with weight loss and chest pain.

The dx of mesothelioma has been made. He develops SOB and XR=pleural effusion. What is the

most appropriate management?

a. Thoracocenthesis

b. Chest drain

c. Radiation therapy

d. Pneumonectomy

e. Chemotherapy

key : e

chemotherapy in mesothelioma:
promising results have been achieved with pemetrexed and raltitrexed in combination with
cisplatin and other combinations, including cisplatin and gemcitabine.

Single-agent therapy with vinorelbine may provide useful palliation with low toxicity

(NICE) has recommended pemetrexed as a possible treatment for malignant pleural mesothelioma
in people:

With advanced disease.

Whose cancer is not suitable for surgical resection.

Who have (WHO) performance status of 0 (able to carry out all normal activity without
restriction) or 1 (restricted in strenuous activity but able to move around and carry out light work).

1175. A 72yo presents with polyuria and polydipsia. The fasting blood sugar is 8 and 10mmol/l.

BP=130/80 mmHg and the level of cholesterol=5.7mmol. There is microalbuminuria. What is

the single most appropriate next management?

a. ACEi and sulfonylurea

b. Statin and biguanide

c. Statin and glitazone

d. Insulin and ACEi

e. Statin and ACEi

key : e

reason : ACEi for microalbuminuria + statin for hyperlipidemia + Medication to control


hyperglycaemia may be required at the time of diagnosis of type 2 diabetes or soon after.

Initial treatment for newly diagnosed diabetes :

Advice on diet and exercise

Prevention of coronary heart disease: blood pressure control , cholesterol-lowering drugs , low
dose aspirin and stop smoking

All patients with microalbuminuria or proteinuria should start (ACE) inhibitor, if there are no
contra-indications

Medication to control hyperglycaemia may be required at the time of diagnosis of type 2 diabetes
or soon after.

insulin therapy should be started immediately in those who are ill at presentation or who have a
high level of ketones in their urine. Insulin should also be considered, regardless of age, if one or
more of the following are present:

Rapid onset of symptoms.


Substantial loss of weight.

Weakness.

A first-degree relative who has type 1 diabetes

1175. A 72yo presents with polyuria and polydipsia. The fasting blood sugar is 8 and 10mmol/l.
BP=130/80 mmHg and the level of cholesterol=5.7mmol/l. There is microalbuminuria. What is
the single most appropriate next management?
a. ACEi and sulfonylurea
b. Statin and biguanide
c. Statin and glitazone
d. Insulin and ACEi
e. Statin and ACEi

e. Statin and ACEi


ACEI should be considered in diabetic patients , especially in those with renal complications. A statin is
considered in all diabetic patients above the age of 40 years ( BNF ) . An oral hypoglycemic drug does not seem
to be needed since the sugar levels aren't that high and may respond to lifestyle and dietary modifications
alone.

when microalbuminuria is present , the target BP is 125/75. therefore , ACE-i should be given. Cholesterol is
also raised , so statins
Diabetic pt, target BP <=130/80.
Diabetic pt plus microalbuminaria , target BP <=125/75.
1st line in diabetes RX is lifestyle modification than biguanides (if not responding to 1st)

1176. A 49yo woman presents to the OPD. Her oral glucose test after 2h of glucose intake vs plasma
level in 2 different tests are 6mmol/l and 10mmol/l. This situation can be categoraized as
a. Impaired glucose tolerance
b. Impaired fasting glucose
c. T1DM
d. T2DM
e. Metabolic syndrome

a. Impaired glucose tolerance


A fasting glucose greater than or equal to 6.1 but less than 7.0 mmol/l implies impaired fasting glucose (IFG)

Impaired glucose tolerance (IGT) is defined as fasting plasma glucose less than 7.0 mmol/l and OGTT 2-hour
value greater than or equal to 7.8 mmol/l but less than 11.1 mmol/l

Diabetes UK suggests:
'People with IFG should then be offered an oral glucose tolerance test to rule out a diagnosis of
diabetes. A result below 11.1 mmol/l but above 7.8 mmol/l indicates that the person doesn't have
diabetes but does have IGT.'
1177. A white Englishman with a past hx of MI is a known HTN and DM. He is currently on aspirin, statin and
metformin. What would you add to the tx?
a. ACEi
b. Diuretic
c. Insulin
d. Beta blocker
e. CCB

a. ACEi
ACEI should even be added in patients who are non hypertensive as it reduces morbidity and mortality post
MI.
ACE inhibitors are also used to treat diabetic nephropathy and have a role in secondary prevention of
ischaemic heart disease.

1178. A 57yo man who had MI a few months ago has been having a low mood. A dx of moderate
depression has been established. Which medication is the best tx for him?
a. SSRI
b. TCA
c. MAOi
d. Benzodiazepam
e. Mood stabilizer

a. SSRI
Selective serotonin reuptake inhibitors (SSRIs) are considered first-line treatment for the majority of patients
with moderate depression.
citalopram (re: QT interval) and fluoxetine are currently the preferred SSRIs
sertraline is useful post myocardial infarction as there is more evidence for its safe use in this situation
than other antidepressants
SSRIs should be used with caution in children and adolescents. Fluoxetine is the drug of choice when an
antidepressant is indicated

1179. A 12yo presents with chest pain. Exam: tachycardia, hypotension, dilated neck veins and the trachea is
not centrally placed. What is the next appropriate management?
a. Portable XR
b. Needle thoracocentesis
c. Chest drainage
d. ABG
e. CTPA

b. Needle thoracocentesis
clincher -trachea deviated, chest pain ..it is tension pneumo for sure !!
tension pneumo !! don't wait to fr CXR to confirm as delay in treatment may cause cardiorespiratory arrest
Pneumothoraces are never awaited for an imaging unless they are chronic enough that the lung underlying is
adapted to one. Considering vitals of the patient it is a primary pneumothorax with tension type dynamics.
treatment includes two stages. first convert it into an open one to decompress the tension via a wide bore iv
line in 2nd ICS along the midclavicular line immediately. Secondly convert it into a closed one by putting a
chest drain attached to an underwater seal.
1180. A 7yo child is being inv for TB. His parents dont agree for taking a BAL. what other sample will
show growth of the organism?
a. Blood test
b. Throat swab
c. Gastric washing
d. Mantoux test
e. CSF

c. Gastric washing
For children we do early morning gastric washing because they swallow their sputum
throat swabs and sputum cultures are less preferred way to obtain a sample from children.. Most children
swallow their sputum so gastric washings samples are taken in children.
If spontaneous sputum samples are not possible then consider bronchoscopy and lavage or, in children, gastric
washings.
1st- sputum for AFB
2nd-BAL

1181. A 51yo man had a MI a few days ago. He developed breathlessness. Echo was done and showed a
pansystolic murmur. What can be the cause of this symptom?
a. Ruptured papillary muscle
b. Acute pericarditis
c. Dresslers syndrome
d. Malignant VT
e. Ventricular aneurysm

a. Ruptured papillary muscle


Acute mitral regurgitation
More common with infero-posterior infarction and may be due to ischaemia or rupture of the papillary
muscle. An early-to-mid systolic murmur is typically heard. Patients are treated with vasodilator therapy but
often require emergency surgical repair.

1182. A 61yo man was found with K+=7.5 and ECG with prolong QRS complex. What is the best
possible tx option?
a. Dialysis
b. IV calcium gluconate
c. IV insulin and dextrose
d. Salbutamol nebulizer
e. Loop diuretics

b. IV calcium gluconate
calcium gluconate stabilize the membrane and prevent arrythmia. Give it first
In this question the potassium levels have increased and wide QRS means calcium levels have decreased.
Increases Potassium levels cause Ventricular Fibrillation.
D.O.C is Calcium Gluconate.
Wide QRS means Calcium levels decreased and can cause Laryngospasm and Tetany.
D.O.C is calcium Gluconate.

Stabilisation of the cardiac membrane


intravenous calcium gluconate
Short-term shift in potassium from extracellular to intracellular fluid compartment
combined insulin/dextrose infusion
nebulised salbutamol

Removal of potassium from the body


calcium resonium (orally or enema)
loop diuretics
dialysis

1183. A 38yo man presents with acute infection of skin in the leg. Dx of sellutitis has been made. What
meds should be prescribed?
a. Penicillin + Flucloxacillin
b. Metronidazole + erythromycin
c. Vancomycin + metronidazole
d. Ceftriaxone + terbinafine
e. Ceftriaxone + flucloxacillin

a. Penicillin + Flucloxacillin
Penicillin ( streptococcal)+ flucloxacillin ( anti staphylococcal )
Cellulitis is most commonly caused by staph and strep hence fluclox and penicillin respectively. But sometimes
with IV Drug users MRSA ( vanco and cefazolin)and in burns or immunocompromised or neutropenic it might
be pseudomonas( ceftazidime, cefepime, or piperacillin tazobactam, carbapenems or aminoglycosides )

1184. A 72yo man presents to the ED with chest pain. The following ECG was taken. What is the most likely
dx?

a. Anterior MI
b. Inferior MI
c. Lateral MI
d. Posterior MI
e. NSTEMI

e. NSTEMI

1185. A 36yo woman has recently spent a lot of money on buying clothes. She goes out almost every
night with her friends. She believes that she knows better than her friends, so she should choose
the restaurant for eating out with her friends. She gave hx of having low mood at 12y. What is
the dx?
a. Mania
b. Depression
c. Bipolar affective disorder
d. Borderline personality disorder
e. Dysthymia

c. Bipolar affective disorder


a mental condition marked by alternating periods of elation and depression.

1186. A homeless lady presents with cough and fever. She complains of night sweats and weight loss.
CXR has been done and shows opacity. What is the next appropriate management?
a. AFB
b. Mantoux test
c. IFN gamma testing
d. Bronchoscopy
e. CT

a. AFB
Mantoux test can only tell you "the exposure which may be the past", can't confirm active disease
INVESTIGATIONS:
1- CXR
2- Sputum for AFB/culture
3- BAL

1187. A 32yo woman presents with malaise fatigue and fever. She complains about weight loss. Exam:
malar rash with sparing of nasolabial fold can be seen. What is the most appropriate inv?
a. Anti ds DNA
b. Anti histone
c. Anti centromere
d. Anti Jo
e. Anti Scl70

a. Anti ds DNA

SLE . Butterfly Rash, middle-aged woman.


The American College of Rheumatology Classification system for SLE suggests that a person may be
classified as having lupus if four or more of the following 11 criteria are present (which do not have to
occur at the same time but can be cumulative over a number of years):
Malar rash.
Discoid lupus.
Photosensitivity.
Oral or nasopharyngeal ulcers.
Non-erosive arthritis involving two or more peripheral joints.
Pleuritis or pericarditis.
Renal involvement with persistent proteinuria or cellular casts.
Seizures or psychosis.
Haematological disorder: haemolytic anaemia or leukopenia or lymphopenia or thrombocytopenia.
Immunological disorder: anti-DNA antibody or anti-Sm or antiphospholipid antibodies.
A positive antinuclear antibody.
1188. A 75yo man presents with back pain. Inv: plasma cells are found. What is the most probable dx?
a. Multiple myeloma
b. AS
c. Disc prolapse
d. Leukemia
e. Myelofibrosis

a. Multiple myeloma

Myeloma: features
Multiple myeloma is a neoplasm of the bone marrow plasma cells. The peak incidence is patients aged 60-70
years.

Clinical features
bone disease: bone pain, osteoporosis + pathological fractures (typically vertebral), osteolytic lesions
lethargy
infection
hypercalcaemia
renal failure
other features: amyloidosis e.g. Macroglossia, carpal tunnel syndrome; neuropathy; hyperviscosity

Diagnosis is based on:


monoclonal proteins (usually IgG or IgA) in the serum and urine (Bence Jones proteins)
increased plasma cells in the bone marrow
bone lesions on the skeletal survey

Hypercalcaemia in myeloma

1189. A 45yo woman presents with complaints of abdominal pain and blood in stool. She brings the stool
sample from home but has never been able to produce a sample at the hospital. Her urine and blood tests are
normal. Exam: multiple scars on the abdomen consistent with laparoscopies
and appendectomy. She insists on getting further inv although no abnormalities are found.
What is the most likely dx?
a. Malingering
b. Somatization
c. Hypochondriasis
d. Conversion disorder
e. Munchausen syndrome

Somatization disorder
multiple physical SYMPTOMS present for at least 2 years
patient refuses to accept reassurance or negative test results

Hypochondriac disorder
persistent belief in the presence of an underlying serious DISEASE, e.g. cancer
patient again refuses to accept reassurance or negative test results

Conversion disorder
typically involves loss of motor or sensory function
the patient doesn't consciously feign the symptoms (factitious disorder) or seek material gain
(malingering)
patients may be indifferent to their apparent disorder - la belle indifference - although this has not
been backed up by some studies

Dissociative disorder
dissociation is a process of 'separating off' certain memories from normal consciousness
in contrast to conversion disorder involves psychiatric symptoms e.g. Amnesia, fugue, stupor
dissociative identity disorder (DID) is the new term for multiple personality disorder as is the most
severe form of dissociative disorder

Munchausen's syndrome
also known as factitious disorder
the intentional production of physical or psychological symptoms

Malingering
fraudulent simulation or exaggeration of symptoms with the intention of financial or other gain

1190. A 36yo woman contacts the police to notify them she was responsible for a recent disastrous flood with
loss of lives. What kind of delusions is she suffering from?
a. Persecutory
b. Poverty
c. Guilt
d. Nihilistic
e. Reference

c. Guilt
an emotion that occurs when a person feels that they have violated a moral standard.
Persecutory: This is one of the most common types of delusions, centering around a person's fixed, false belief
that others aim to obstruct, harm, or kill him/her.

Poverty: The person strongly believes that he is financially incapacitated

Nihilistic: the delusion that things (or everything, including the self) do not exist; a sense that everything is
unreal

Reference: A neutral event is believed to have a special and personal meaning. For example, a person with
schizophrenia might believe a billboard or a celebrity is sending a message meant specifically for them.

1191. A 27yo man presents with symptoms characterized by alternating mood swings a/w flight of
ideas, elation, over activity and disinhibition, or low mood with lack of energy and social
withdrawal. What is the most probable dx?
a. Bipolar affective disorder
b. Dysthymia
c. Mania
d. Hypomania
e. Cyclothymia

a. Bipolar affective disorder


Basic definition

1192. Healthy parents have 2 children, a child with CF and a healthy child. They want to have another child.
What are the chances of that child being a carrier?
a. 1:4
b. 1:2
c. 2:3
d. 1:8
e. 1:16

b. 1:2
cystic fibrosis is autosomal recessive, thus when you cross match the genetics according to Mendelian
inheritance, if they had 4 children 1 would be normal, two would be carriers and 1 would have cystic fibrosis
(since the parents have a child with cystic fibrosis both parents are carriers). This however does not translate
to immediate application but only tells us the probability of having one of the 3 scenarios (normal, carrier, and
affected). So chance of having a normal child is 1:4, carrier is 2:4 (divided translates to 1;2) and affected by
disease 1:4)

1193. A 64yo man believes a female newscaster is communicating directly with him when she turns a
page. What kind of delusions is he suffering from?
a. Persecutory
b. Control
c. Grandeur
d. Nihilistic
e. Reference

e. Reference
A neutral event is believed to have a special and personal meaning. For example, a person with schizophrenia
might believe a billboard or a celebrity is sending a message meant specifically for them.

Delusion of control: This is a false belief that another person, group of people, or external force controls one's
general thoughts, feelings, impulses, or behavior.

Grandeur: fixed, false belief that one possesses superior qualities such as genius, fame, omnipotence, or
wealth.

1194. A 7yo girl with allergy became acutely unwell while visiting a friends house and has been
brought immediately to the ED. She is fully conscious but has got stridor, wheeze and
erythematous rash. She is receiving oxygen. What is the single immediate management?
a. Check airway patency and prepare intubation
b. Give 0.25ml in 1000U epinephrine IM
c. Give 10 mg chlorpheniramine IM
d. Give 50 ml hydrocortisone IM
e. Obtain secure IV access

b. Give 0.25ml in 1000U epinephrine IM


Adrenaline is by far the most important drug in anaphylaxis and should be given as soon as possible. The
recommended doses for adrenaline, hydrocortisone and chlorphenamine are as follows:

Adrenaline Hydrocortisone Chlorphenamine

< 6 months 150 micrograms (0.15ml 1 in 1,000) 25 mg 250 micrograms/kg

6 months - 6 years 150 micrograms (0.15ml 1 in 1,000) 50 mg 2.5 mg

6-12 years 300 micrograms (0.3ml 1 in 1,000) 100 mg 5 mg

Adult and child > 12 years 500 micrograms (0.5ml 1 in 1,000) 200 mg 10 mg

Adrenaline can be repeated every 5 minutes if necessary. The best site for IM injection is the anterolateral
aspect of the middle third of the thigh.

1195. A terminally ill pt with metastatic carcinoma presents with dysphagia and difficulty in
swallowing. What is the best possible tx?
a. Nystatin suspension
b. Amphotericin B IV
c. PO fluconazole
d. Cotrimazole
e. Analgesic

c. PO fluconazole
Oesophageal candidiasis is the most common cause of oesophagitis in patients with HIV. It is generally seen in
patients with a CD4 count of less than 100. Typical symptoms include dysphagia and odynophagia. Fluconazole
and itraconazole are first-line treatments

1196. A couple attends their GP because of marital problems. The wife states that her husband is
having affairs although she has no proof of this. The husband states that she even had him
followed by a private detective and this is putting considerable strain on their marriage. What is
the most likely dx?
a. Fregoli syndrome
b. Cotard syndrome
c. Mood disorder
d. Ekbom syndrome
e. Othello syndrome

e. Othello syndrome
Delusional jealousy (Othello's syndrome) - eg believing a partner is being unfaithful.

The Fregoli delusion, or the delusion of doubles, is a rare disorder in which a person holds a delusional belief
that different people are in fact a single person who changes appearance or is in disguise.

The Cotard delusion (also Cotard's syndrome and walking corpse syndrome) is a rare mental illness, in which
the afflicted person holds the delusion that he or she is dead, either figuratively or literally

Ekbom syndrome, also called delusional parasitosis, is a psychiatric disorder characterized by the patient's
conviction that he or she is infested with parasites.

1197. A 65yo lady who is on thiazide suffers from falls in the morning. What is the cause for her
symptoms?
a. Orthostatic hypotension
b. TIA
c. Epilepsy

a. Orthostatic hypotension
Common adverse effects
dehydration
postural hypotension
hyponatraemia, hypokalaemia, hypercalcaemia
gout
impaired glucose tolerance
impotence

1198. A boy was admitted with partial thickness burn, what is your next step?
a. Escharectomy
b. Dressing
c. Burst blisters
d. Local antibiotics
e. Refer to burn unit

All complex injuries should be referred - particularly


Age under 5 years or over 60 years.
Site of injury: face, hands, perineum, any flexure (including neck or axilla) and circumferential dermal
burns or a full-thickness burn of the limb, torso or neck.
Inhalation injury.
Mechanism of injury:
o Chemical burns affecting over 5% total body surface area burned (over 1% for hydrofluoric acid
burns).
o Exposure to ionising radiation.
o High-pressure steam injury.
o High-tension electrical injury.
Suspected non-accidental injury in a child.
Large affected area:
o Age under 16 years: over 5% total body surface area burned.
o Age 16 years or older: over 10% total body surface area burned.
Co-existing conditions - eg, serious medical conditions, pregnancy or associated fractures, head injury
or crush injuries.

1199. A 28yo man presents with a 2h hx of rapid palpitations. He feels a little light headed but is
otherwise well. Exam: pulse=170bpm and regular, BP=100/68mmHg. He has had 2 similar
episodes in the past. What is the most likely rhythm disturbance?
a. SVT
b. VF
c. VT
d. V-ectopics
e. A-fib

a. SVT
Acute management
vagal manoeuvres: e.g. Valsalva manoeuvre
intravenous adenosine 6mg 12mg 12mg: contraindicated in asthmatics - verapamil is a preferable
option
electrical cardioversion

Prevention of episodes
beta-blockers
radio-frequency ablation

1200. A child has hypothyroidism. What feature is a/w it?


a. Microglossia
b. Prolonged neonatal jaundice
c. Undescended testis
d. Anal tag
e. Left soft palate
b. Prolonged neonatal jaundice

CONGENITAL HYPOTHYROIDISM

Symptoms
Feeding difficulties
Somnolence
Lethargy
Low frequency of crying
Constipation

Signs
Large fontanelles
Myxoedema - with coarse features and a large head and oedema of the genitalia and extremities
Nasal obstruction
Macroglossia
Low temperature (often <35C) with cold and mottled skin on the extremities
Jaundice - prolongation of the physiological jaundice
Umbilical hernia
Hypotonia
Hoarse voice
Cardiomegaly
Bradycardia
Pericardial effusion - usually asymptomatic
Failure of fusion of distal femoral epiphyses
The growing child will have short stature, hypertelorism, depressed bridge of nose, narrow palpebral
fissures and swollen eyelids
Refractory anaemia

1201. A 2wk girl presents with E-coli which is confirmed by urine culture. What is the most appropriate next
inv?
a. US
b. IVU
c. CT kidney
d. BUE
e. MCUG

a) US

ultrasonography - even one case of UTI in children or males needs to be investigated, in case of females
recurrences require investigation.

Child <3 months - refer to paediatrician

3 months to 3 years - urgent microscopy and culture


>3 years - Urine sample, dipstick test and culture

First line investigation, best initial image showing renal size, hydronephrosis, perinephric collection, bladder
residual volume, prostate.

1202. A lady from Asia presented with lump in her neck. FNAC has been done and revealed lesions with
caseous material in the center surrounded by fibrosis. What is the most probable dx?
a. Thyroid carcinoma
b. TB lymphadenitis
c. Lymphoma
d. Inf Mono
e. Mesothelioma

b. TB lymphadenitis

Submandibular triangle of neck

Palpable nodes initially tender, firm, discrete, later turn matted, suppurative with discharging sinuses.

Caseous material confirms diagnosis

1203. A 32yo woman has undergone a biopsy for a breast lump. The report says: a well circumscribed
lump with clear margins and separated from the surrounding fatty tissue. What is the most
appropriate interpretation of this report?
a. Fibroadenosis
b. Ca Breast
c. Mammary abscess
d. Fibroadenoma
e. Fat necrosis

d. Fibroadenoma

Mobile (the breast mouse), regular margins smooth lump, multiple lumps

One third regress, one third stay the same. one third get bigger

conservative treatment, observe. If in doubt FNAC with/without excision

1204. A young boy presented with peri-oral blisters. Some of which are weeping and others are
crusted. What is the single most appropriate dx?
a. Impetigo
b. Varicella zoster
c. Shingles
d. Scabies
e. Herpes simplex

a. Impetigo

Non-bullous - tiny pustules change into honey coloured crusted plaques <2cm
Mostly on exposed area like face extremities especially where bites, abrasions, lacerations, scratches, burns or
trauma have occurred.

Spreads rapidly

Some itching may be present

Bullous - thin roof, so rupture spontaneously


Occur on face, trunk, extremities, buttocks or perineal region
1205. A 39yo man comes with umbilicated papules on his face. His CD4 count is measured to be 35. What is
the single most appropriate option?
a. Mycobacterium avium
intercellular
b. CMV
c. Streptokinase
d. Toxoplasmosis
e. Pneumocystis jerovici
f. Moluscum contagiosum

f. Moluscum contagiosum

Viral skin infection (Opportunistic) in immunocompromised individuals

Firm, smooth, Umbilicated papules

Skin coloured, white, translucent or slightly yellow

Occurs in clusters, in children, found on trunk or extremities

In adults, found on lower abdomen, inner thighs or genital region

1206. A 45yo man is admitted to ED with excruciating pain in the right leg. Exam: limb is pale and
dorsalis pedis and posterior tibial pulses are absent. Pulse=88 bpm, irregular and he has a
pansystolic murmur at apex. What is the most probable dx?
a. Thromboangiitis Obliterans
b. Sciatica
c. DVT
d. Atherosclerosis
e. Embolus

e. Embolus

Acute limb ischemia is caused


6 Ps - pale, pulseless, painful, paralysed, paraesthetic, perishing with cold

Ischaemia, occurs from abrupt interruption of blood flow due to emboli. A cardiac origin is the source of
emboli in most of the cases associated with atrial fibrillation, as in this case.
1207. An 18yo man has a smooth, tender swelling extending from the ear to the angle of the jaw of sudden
onset. Temp=38.5C. What is the single most likely dx?

a. Dental caries
b. Mumps
c. OE
d. OM
e. Temporomandibular joint pain

b. Mumps

Fever, myalgia , malaise painful parotid swelling which becomes bilateral in 70%

Spread - droplets/saliva

Incubation period - 14-21 days

Complications - rare but may lead to orchitis arthritis, meningitis

1208. A 6wk baby has a blue mark near coccyx since birth. His mother is worried. What would you do?
a. Reassure
b. Coag profile
c. Karyotyping
d. Skeletal survey
e. CT

a. Reassure

The mark is a Mongolian spot which is benign, flat, congenital birthmark with wavy borders, irregular shape,
blue to blue/black in colour

Usually present on buttocks or back

Disappears by 3-5 years of age, almost always by puberty

It resembles a bruise and might cause concern, however, the determining factor is presence since birth

1209. A man presents with inoperable carcinoma and back pain. His pain has been well controlled with
morphine but he develops vomiting. Morphine was stopped and he was started on
metoclopramide and fentanyl patches. He then develops neck stiffness and fever. What is the
cause of these symptoms?

a. Metoclopramide
b. Fentanyl
c. Morphine
d. Meningitis
e. Metastasis

e. Metastasis
Leptomeningeal meningitis is caused due to metastasis

Associated with spinal signs like neck pain, back pain, neck stiffness, weakness of limbs

1210. A 51yo man has become increasingly fatigued for the past 10m. PE: no abnormal findings. Labs:
Hgb=9.2, Hct=27.9%, MCV=132fl, plt=242, WBC=7.59. Which of the following morphologic
findings is most likely to be present on examination of his peripheral blood smear?
a. Hypersegmented neutrophils
b. Nucleated RBC
c. Blasts
d. Hypochromic, microcytic RBC
e. Schistocytes

a. Hypersegmented neutrophils
Megaloblastic anaemia is caused by folate deficiency and vitamin B12 deficiency causing decreased Hg and
increased MCV. They have similar blood film and bone marrow biopsy appearances.

Blood film - hypersegmented neutrophils and macrocytes

1211. A 9yo girl with weekly abdominal pain and occasional headaches but not a/w vomiting or
diarrhea. She maintains a good appetite. Lab: normal. CBC, BUE, etc are normal. Exam: no
abnormality as found and the abdomen was soft and non-tender. What would you do for her
next?
a. US abdomen
b. CT thorax
c. LFT
d. Reassure
e. Analgesics

d. Reassure

Childhood periodic syndromes - cyclical vomiting, abdominal migraine

Age 5-10 years

Bilateral pain with abdominal cramps and vomiting, The child usually falls asleep within an hour of pain onset.
Sleeping relieves pain.

No medication required. Reassurance as disappears with age.

1212. A 54yo male pt DM with BMI=33 who has been treated using dietary control up till now presents to his
GP with a fasting blood sugar of 14mmol/l and creatinine=90mmol/l. Urine shows
glycosuria. No other abnormalities are found. What is the best next step in management?
a. Biguanide
b. Sulfonylurea
c. Insulin
d. Sugar free diet
e. ACEi
a. Biguanide

Biguanide (Metformin) are first line treatment for type 2 Diabetes Mellitus. They work by increasing insulin
sensitivity. Additionally, they help with weight loss. They do not cause hypoglycaemia.

NICE guidelines

1. Metformin
2. Metformin + Sulfonylureas
3. Add Thiazolidinedione or Insulin
4. Insulin + Metformin + Sulfonylureas
5. Increase Insulin dose as needed

1213. What are the side effects of thiazide diuretics?


a. Hypocalcemia
b. Hyponatremia
c. Hypernatremia
d. Hyperkalemia

b. Hyponatremia

Thiazide diuretics - Bendroflumethiazide, hydrochlorothiazide

Low dose to treat hypertension

Also used in combination with loop diuretics to treat heart failure (Metolazone)

In case of hypertension, if patient is intolerant to calcium channel blockers, thiazide diuretics are prescribed.

Side effects include hyponatremia, hypokalemia, hypomagnesemia, hyperuricaemia, metabolic alkalosis,


hypotension, hypovolaemia

1214. A 46yo man who is a heavy drinker is brought to the ED in a drowsy state. He is responding
vaguely to questions. Exam: nystagmus and hyperreflexia. MCV=103fl. What is the most likely
cause for his cognitive impairment?
a. B1 deficiency
b. B12 deficiency
c. Folate deficiency
d. B6 deficiency
e. Alcohol withdrawal

a. B1 deficiency

Thiamine deficiency -

Wernickes Encephalopathy - ophthalmoplegia (Nystagmus, lateral rectus palsy)

Ataxia - wide based gait


Confusion

Caused by alcoholism, malnutrition, eating disorders, prolonged vomiting

Korsakoff Psychosis - decreased ability to acquire new memories, hallucinations

1215. A 23yo female presented with a swelling of her neck that moved upwards
on protrusion of tongue. What is the next appropriate inv?
a. FNAC
b. Punch biopsy
c. Core biopsy
d. MRI neck
e. Radioactive thyroid scan

e. Radioactive thyroid scan

Thyroglossal cyst- A fluctuant swelling in the midline of the neck that moves upwards on protruding the
tongue
Usually non- tender and mobile
May be tender if infected, and is associated with dysphagia, dysphonia, draining sinus, fever or increasing neck
mass
May lead to airway obstruction in some cases
Should be palpated on physical examination.
Thyroid Function Tests
If not palpable, USG and CT scan are first choices.
Radio active Thyroid scanning may be used to demonstrate any functioning ectopic thyroid accompanying the
cyst.

1216. A 34yo man from Asia presented with 5m hx of productive cough, night sweats and weight loss. His CXR
reveals some shadowing in the left upper zone. What is the single most discriminating
inv?
a. AFB for sputum
b. CXR
c. CT
d. TFT
e. US abdomen

a. AFB for sputum

In case of pulmonary TB, chest x-ray is done first and if suggestive of TB, sputum samples are taken. Three
sputum samples are required, with at least one of them an early morning sample, preferably before starting
treatment or within the first week.

Send samples for microscopy with ZN stain and culture for acid fast bacilli.

If sputum can not be obtained, bronchoscopy and lavage should be used. Incase of children, gastric washings
can be used.
1217. A prv healthy 23yo presented a week hx of bloody diarrhea and abdominal pain with cramps and fever.
Exam: tenderness in lower abdomen. What is the most appropriate dx?
a. Celiac disease
b. Colorectal polyps
c. UC
d. Laxative abuse
e. Gastroenteritis

e. Gastroenteritis

It is a combination of nausea, vomiting, diarrhoea and abdominal pain, usually of infectious origin.

In the UK, the two most common causes of gastroenteritis in adults are the norovirus and food poisoning
(most often caused by salmonella or campylobacter bacteria). Norovirus is a common cause of winter
vomiting. Bloody diarrhoea is usually due to bacterial infection.

Its important to assess the degree of dehydration -

Mild - lassitude, anorexia, nausea, postural, hypotension

Moderate - apathy, dizziness, muscle cramps, dry tongue, sunken eyes, reduced skin elasticity, oliguria, SBP
>90

Severe - profound apathy, confusion leading to coma, shock, SBP <90, Oliguria or , Anuria

1218. A 10yo boy presents with irritability, sudden onset of pain and discharge from the right ear.
Which antibiotic would be the 1st line of tx?
a. Amoxicillin
b. Ciprofloxacin
c. Flucloxacillin
d. Ceftazidime

e. Benzylpenicillin

a. Amoxicillin

Acute Otitis Media is inflammation of middle ear presenting with pain, malaise, irritability, fever, vomiting.
Red and bulging ear drums may be present. Also, hearing loss may be present.

Usually resolves spontaneously without specific treatment. Analgesia may be required

In cases with symptoms longer than 2-3 days, children <2 years with bilateral disease or bulging drum, any
child with Otorrhoea, antibiotics are required. In such cases, a five day course of Amoxicillin is prescribed. In
case the patient is allergic to penicillin, erythromycin is given.

1219. A 26yo man strongly believes that every elderly man he meets is his father. Although they look different,
he is sure it is father wearing different disguises. What kind of delusions is this man
suffering from?
a. Delusion of persecution
b. Erotomania
c. Delusion of grandeur
d. Delusion of doubles
e. Delusion of reference

d. Delusion of doubles

Alson known as Fregoli Delusion these are extremely rare delusions in which a person believes different
people are all just a single person that is capable of morphing his or her appearance as a disguise. It has been
speculated that these delusions are usually associated with brain lesions or damage.

1220. A 26yo passed a 4mm stone in his urine. On US a 3mm stone is found in the renal pelvis. What is the
single most appropriate management?
a. ESWL
b. None
c. Open Surgery
d. Conservative
d. Conservative

Renal stones are usually associated with sudden severe pain in the loin, radiating to the groin, often caused by
a moving stone. However, most symptomatic renal stones are small, ie <5mm in diameter and pass
spontaneously. In some people, stones up to 1cm pass spontaneously. Any stones >1cm usually require
intervention. A stone that has not passed within 1-2 months is unlikely to do spontaneously.

1221. A 35yo man has had acute pain and swelling below the mandible on the left side for 2h. The
swelling occurred after eating a large meal. What is the single most likely dx?
a. Laryngocele
b. Ranula
c. Neck abscess
d. Parotid calculus
e. Submandibular calculus

e. Submandibular calculus

A stone formed in one of the salivary glands can block their opening causing saliva to back up into the gland
leading to pain and swelling. This pain is heightened just after starting a meal. Eventually, infection of the
gland may occur, even forming an abscess in some cases. This is known as Sialolithiasis and most commonly
affects the submandibular gland.

1222. A 45yo man has had impaired vision and pain on eye movement in his left eye over the last 5 days. He
also notes loss of color vision in the same eye. In the left eye, the visual acuity is up to
counting fingers. When the pupil is stimulated with light, it dilates. His fundus is normal. What is
the single most appropriate clinical dx?
a. Acute dacryocystitis
b. Acute iritis
c. Papillitis
d. Retrobulbar neuritis
e. Scleritis
d. Retrobulbar neuritis

Optic neuritis or retrobulbar neuritis is inflammation of the optic nerve. It consists of the classic triad -

1. Reduced vision
2. Eye pain, particularly on movement
3. Impaired colour vision

Common causes are multiple sclerosis, giant cell arteritis.

Typical signs - decreased pupillary light reaction in affected eye, or Marcus Gunn Pupil

1223. A 56yo pt has been dx with MS. She presents with a positive Rombergs test. She also has
weakness and loss of sensations in all her 4 limbs. Which site is most likely to be affected?
a. Cerebral cortex
b. Cerebellum
c. Cervical spinal cord
d. Thoracic spinal cord
e. Brain stem

c. Cervical spinal cord

Rombergs Test is used to identify instability of either peripheral or central cause -

The patient stands up straight with feet together (or at a distance for them to be steady) with arms
outstretched. Then ask them to shut their eyes.
If they are unable to maintain their balance with their eyes closed, the test is positive (usually fall
to the side of the lesion so stay close by to prevent them falling).
A positive test suggests a problem with proprioception or vestibular function. Romberg's test can
also be positive in neuromuscular disorders and may not be reliable in very elderly people.

Multiple sclerosis causes demyelination of neurons in brain and spinal cord leading to neurological symptoms
and loss of sensation, difficulty in movement, coordination and balance. In this case, all limbs are affected, so
the level of lesion will be higher, ie. cervical spain.

1224. A 58yo man suddenly becomes shocked several days after suffering an acute ant MI. His CXR shows a
large globular-shaped heart and clear lung fields. What is the single most likely
explanation for the abnormal inv?
a. Acute pericarditis
b. Cardiac tamponade
c. Atrial thrombus
d. Left ventricular aneurysm
e. Dressler syndrome

b. Cardiac tamponade

It is caused by accumulation of blood or any other fluid or gas in the pericardial, leading to reduced ventricular
filling and haemodynamic compromise.
It is an emergency

Often associated with pericarditis, can occur after acute myocardial infarction, sometimes as Dresslers
syndrome.

Dyspnoea, tachycardia and tachypnoea, cold, clammy extremities.

Distended neck veins, muffled heart sounds

Becks Triad - Jv distension, hypotension, diminished heart sounds

Pulsus paradoxus

1225. A 56yo alcoholic man who has increased the amount of alcohol he is using wants to attend his
daughters wedding that is in 2wks. He is now coming to you for help. How would you help him?
a. Acamprosate
b. Refer to clinical psychologist
c. Refer to GP
d. Desipramine
e. Refer to community mental health support group

a. Acamprosate

It is used after successful withdrawal of alcohol to prevent relapse. Used in combination with individual
psychological intervention

Possible neuroprotective role in detoxification

It does not interact with alcohol

Instead, it works by reducing cravings

1226. An 80yo woman fell over at her nursing home. XR shows fx of radius with <10degree of dorsal
angulation. What is the single most appropriate tx?
a. Below elbow full plaster of paris
b. Below elbow splint plaster of paris
c. Closed reduction of fx
d. Elasticated support bandage
e. Open reduction and internal fixation

Colles Fracture (C)


* The classical definition is a fracture through the distal metaphysis of the radius, approximately 4 cm proximal
to the articular surface. The term is now more loosely used for any fracture of the distal radius, with or
without involvement of the ulna, with dorsal (backward) displacement of the fracture fragments.
* It is common in older people who fall and have osteoporosis. Osteoporosis should be considered in anyone
with a Colles' fracture (see 'Osteoporosis and wrist fractures', below).
* It can also occur in younger people with normal bones.
- Investigation of choice is X-rays of the wrists.
- Management
- Uncomplicated: Closed reduction
- Complicated: Open reduction and internal fixation (ORIF)

1227. A 16yo girl who is normally fit and well attends her GP complaining of heavy and painful periods.
She is requesting tx for these complaints. She denies being sexually active. Select the most
appropriate management for her menorrhagia?
a. Antifibrinolytics (tranexamic acid)
b. COCP
c. Endometrial ablation
d. IUS progestrogens (mirena)
e. NSAIDS (mefenamic acid)

Heavy and Painful periods (A)


- First-line treatment for menorrhagia is Mirena coil, but the patient in question does not need contraception
and she is having painful periods. The second line is Mefenamic acid or Tranexamic Acid. Third line is COCPs.
Fourth line is Endometrial ablation or Hysterectomy (If there is no desire to conceive). IUCD, according to
patient.info actually causes heavy or painful periods.
- Therefore the best choice in this case is Mefenamic acid (NSAIDs)

1228. A 67yo lady with an ulcer on the anal margin. Which is the single most appropriate LN involved?
a. External iliac LN
b. Pre-aortic LN
c. Aortic LN
d. Inguinal LN
e. Iliac LN

Lymph Node Involvement (D)


- Superficial inguinal lymph nodes drains from Penis, Scrotum, Perineum, buttock, vulva and abdominal wall
below the umbilicus which then drains into the deep inguinal lymph nodes.
- There inguinal nodes (D) is the correct choice.

1229. A branch of the dominant coronary artery that supplies the inferior portion of the septum. What is the
single most appropriate option?
a. Septal branches
b. Obtuse marginal branches
c. Circumflex artery
d. Left main stem, post descending artery
e. Diagonal branch

Blood supply of the heart (D)


The Posterior Descending Artery is an artery running in the posterior interventricular sulcus to the apex of the
heart where it meets with the anterior interventricular artery. It supplies the posterior (inferior, see below)
1/3rd of the interventricular septum.
The anatomical position of the artery is not really posterior, but inferior. The terminology posterior is based on
the viewing of the heart from Valentine position, not by the hearts actual position.
1230. A 55yo female presented with anemia and dysphagia. There is a feeling of something stuck in
the throat. The esophagus cant be negotiated beyond the crico-pharynx. What is the most
probable dx?
a. Foreign body
b. Plummer vinson syndrome
c. Pharyngeal carcinoma
d. Barrets esophagus
e. Esophageal carcinoma

Plummer-Vinson Syndrome (B)


This is a condition where iron deficiency is associated with a post-cricoid oesophageal web.
The syndrome most often affects middle-aged women (this may be related to a propensity for iron deficiency),
although it can occasionally present in a child.[2] The web (containing mucosa and submucosa) occurs at the
anterior post-cricoid area of the upper oesophagus.
The presentation is usually with painless, intermittent dysphagia. It tends to be with solid foods but, if
untreated, may progress to soft foods and even liquids
Examination: The features that appear on examination are those associated with iron deficiency. There may
be pallor and even tachycardia if anaemia is marked. There may be koilonychia (spoon-shaped nails), angular
cheilitis and glossitis.
- Investigation
* FBC will show a microcytic, hypochromic anaemia. Ferritin is low. * Barium swallow may show the web. This
may need to be enhanced with videofluoroscopy.
* Biopsy may be required if malignancy is suspected clinically.
* Treatment
- Iron Replacement
- Endoscopic Dialatation

1231. A pt is on cancer tx with dexamethasone. According to her biochemical results her K+=normal and her
Na+=low. What is the dx?
a. Addisons
b. Dexamethasone side effect
c. Dilutional hyponatremia

Dexamethasone Dilutional Hyponatremia (C)


Exogenous glucocorticoids exert a tonic suppression of ADH secretion in the body. This causes retention of
water which leads to a decrease in the serum osmolality. Therefore the correct option is C Dilutional
Hyponatremia.

1232. A diabetic has been prescribed a long acting hypoglycemic in the morning and short acting in the
evening. He takes a regular lunch, but has been having hypoglycemic attacks at around 4pm each day. What is
the most appropriate intervention?
a. Recommend a heavier lunch
b. Review morning drug
c. Review evening drug
d. Review both drug
e. Reassure
Diabetes Mellitus (B)
- Recommending a heavier lunch carries the risk of increasing the blood sugar. Reviewing the evening drug has
no use since the patient does not experience any hypoglycemic attacks after the evening drug until after he
takes the morning drug. This automatically rules out the review of both drugs. This patient cannot be
reassured as he is having hypoglycemic attacks for which an intervention is necessary.
- Since, the patient is on a long-acting hypoglycemic agent, and experiencing hypoglycemic attacks at around 4
pm, this means that the drug being used in the morning must be reviewed for appropriate intervention and
replacement.

1233. A male pt presented with blood and mucus in stool. He has also noticed weight loss but has no
hx of altered bowel habits. What is the dx?
a. Carcinoma of cecum
b. Carcinoma of descending colon
c. Carcinoma of sigmoid colon
d. Carcinoma of rectum

Carcinoma of the Cecum (A)


- The symptoms of Cecum Cancer can be difficult to detect. The most common symptoms of Colorectal Cancer
include a feeling of fullness or pressure in the rectum, rectal bleeding and a frequent urge to defecate
(tenesmus). Inflammations in the cecum, unlike the rectum or the sigmoid colon or the descending colon, will
not make the patient feel the urge to defecate or cause bowel habit changes/irregularities, because the stool
passing through the rectum is slushy and can easily bypass masses in this part of the intestine.

1234. A 22yo man keeps having persistent and intrusive thoughts that he is a dirty thief. No matter
what he tries these thoughts keep coming to him. Any attempt to avoid these thoughts leads to
serious anxiety. What is the most likely dx?
a. Schizophrenia
b. OCD
c. PTSD
d. Mania
e. Psychotic depression

OCD (B)
Symptoms typically include recurring thoughts and repetitive actions in response to the recurring thoughts. A
common example is recurring thoughts about germs and dirt, with a need to wash your hands repeatedly to
"clean off the germs".
- Treatment
The usual treatment for OCD is:
- Cognitive behavioural therapy (CBT); or
- Medication, usually with an SSRI antidepressant medicine; or
- A combination of CBT plus an SSRI antidepressant medicine.

1235. A 45yo female comes to the ED while having a generalized tonic clonic seizure and she has
having difficulty breathing and is cyanosed. What is the tx option for her?
a. Secure airways
b. IV diazepam
c. IV phenytoin
d. Oxygen mask

Management of Status Epilepticus (A)


- Step-wise approach
- Open and maintain the airway, lay in recovery position. Remove false teeth if poorly fitting, insert oral nasal
airway, intubate if necessary
- Oxygen, 100% + suction (as required)
- IV access and take blood for investigations
- Slow IV bolus phase: to stop seizures, e.g, Lorazepam, Diazepam
- Correct hypotension with fluids
- IV infusion phase: If seizures continue start phenytoin. Alternate: Diazepam infusion
- General anesthesia phase

1236. A 30yo man is becoming concerned about the safety of his family. He has been checking the
locks of the door every hour during the night. He becomes very anxious if his wife tries to stop
him. What is the most likely dx?
a. Paranoid delusion
b. PTSD
c. Social phobia
d. OCD
e. GAD

OCD (D)
Described in 1234

1237. A 6wk baby has been dx as HIV+ve. Which immunization plan will you opt for him?
a. Dont give any vaccine
b. Give all vaccines except live attenuated vaccines
c. Give only BCG vaccine
d. Give all vaccines except BCG vaccine

Vaccination in Children with Immunodeficiency states (D)


- An asymptomatic child with HIV infection can be given any vaccine except BCG.
- BCG vaccine is contraindicated in symptomatic HIV-infected child as well along with a few live attenuated
vaccines.
- In this scenario the child has just been diagnosed with an HIV infection which most probably means that he is
asymptomatic. So the answer would be D, Give all vaccines except BCG.

1238. A 36yo man has been dx with DI. What electrolyte picture is expected to be seen?
a. High serum Na, low serum osmolarity, high urine osmolarity
b. Low serum Na, low serum osmolarity, high urine osmolarity
c. Low serum Na, high serum osmolarity, high urine osmolarity
d. High serum Na, high serum osmolarity, low urine osmolarity
e. Normal Na, normal serum osmolarity, normal urine osmolarity
Diabetes Insipidus (D)
Diabetes insipidus is a condition in which your ability to control the balance of water within your body does
not work properly. The kidneys are not able to regulate the amount of water that passes out in the urine. This
means that the patient passes large amounts of dilute urine polyuria.
Because of passing more urine, and therefore losing more fluid from the body, to try to compensate for this,
the patient becomes thirstier and wants to drink more polydipsia.
Patients with this condition become dehydrated easily. The levels of sodium and potassium salts in the blood
can also become unbalanced and too high.
- Points to look for:
- Dilute Urine -> Low urine osmolality
- Dehydration -> High serum sodium and hence high serum osmolality.

1239. The artery that supplies the ant right ventricular wall. What is the single most appropriate
option?
a. Acute marginal branch
b. Left ant descending artery
c. Coronary sinus
d. Circumflex artery
e. Right coronary artery

Blood supply of the heart (A)


- Acute Marginal Branch of the Right coronary artery supplies the anterior right ventricular wall.

1240. A 55yo male presents to the ED after an RTA with breathlessness, engorged neck veins and a
dull percussion note on the right side of his chest. Exam: pulse=140 bpm, BP=80/50 mmHg. What
is the most likely dx?
a. Hemothorax
b. Hemopneumothorax
c. Tension pneumothorax
d. Simple pneumothorax

Hemopneumothorax (B)
Hemothorax is rarely a solitary finding in blunt trauma.
Dull percussion note, in particular, takes the diagnosis away from any kind of pneumothorax.
Low BP in this scenario indicates shock secondary to heavy blood loss
The reason Hemopneumothorax is the correct option is the presence of engorged neck veins. Looking at the
stem of the question now, we can see that the patient has signs of both Hemothorax and Pneumothorax.
Therefore the correct choice is B.

1241. A 32yo woman presents with complaints of having low back pain. She is taking analgesics for it. All inv
are normal. What will you advise her?
a. Bed rest
b. Physiotherapy
c. Advice to be more active
d. Admit
Back Pain Management (C)
- For nonspecific back pain focus on education and self-management.
- Advise patients to continue with normal activities and be active.
- Manage pain with analgesics.
- Offer physiotherapy, acupuncture or an exercise programme.
- Address psychosocial issues which may predispose to developing chronic pain and disability.
- Last option is surgery.

1242. A 32yo woman suffers an episode of severe occipital headache with vomiting and LOC. She is
brought to the hosp where she is found to be conscious and completely alert. Exam: normal
pulse and BP with no abnormal neurological sign. What is the next step in her management?
a. Admission for observation
b. CT brain
c. MRI head
d. Reassurance and discharge home
e. XR skull

Severe headache with LOC (B)


- The next step in this case should be CT-Brain to look for or rule out any organic cause.
- Reassurance and discharging the patient is a risky option.
- X-ray Skull does not have any utility in this condition.
- MRI and Admission should follow CT-Brain if required.

1243. A 30yo woman is taking tx for asthma. She has a HR=130bpm and peak expiratory flow rate=400.
What is the most appropriate management?
a. Atenolol
b. Digoxin
c. Review drugs

Asthma (C)
The patient has a heart rate that is more than a 110, which means that she is having a severe attack of asthma.
Her expiratory flow rate is also decreased. She is already on medication which apparently is not adequate to
alleviate her symptoms. Digoxin and Atenolol have no role in the treatment of Asthma. The correct option is to
review the drugs she is taking.
- Treatment of severe or life-threatening asthma
- Salbutamol 5m nebulized with oxygen and give prednisolone 30 mg PO
- If PEF remains less than 75%, repeat Salbutamol
- Monitor O2 saturation, HR and RR.

1244. A pt presents with a mask face. He also has gait prbs. Which class of drug is causing this?
a. Anti-depressant
b. Antipsychotic
c. Anti-HTN

Anti-Psychotics Side-effects (B)


- Mask face and gait problems suggest Parkinsonism which is caused by deficiency of Dopamine and anti-
psychotic medications are known to deplete Dopamine and cause Parkinsonism.
- The other medications mentioned do not cause the symptoms such as in this patient.
1245. A 16yo boy came home from boarding school with a cough. His CXR showed bilateral
consolidations. What is the most likely organism which would have caused his symptoms?
a. Legionella pneumophila
b. Mycoplasma pneumonia
c. Mycobacterium TB
d. Pneumocystis jiroveci
e. Pseudomonas aeruginosa

Atypical Pneumonia (A)


- History of living in boarding
- Bilateral consolidations
- Dry Cough
- Crepitations on auscultation
- Fever, anorexia and lethargy
The above points point clearly towards Legionella (A)
- Mycoplasma does not cause bilateral consolidation
- Staphylococcus causes bilateral basal cavitation and presents with a productive cough.
- Streptococcus shows lobar consolidation and presents with a productive cough.

1246. After an MI, a man presents with pansystolic murmur which is radiating to the axilla. What is the
dx?
a. Tricuspid regurgitation
b. Mitral regurgitation
c. Aortic stenosis
d. Mitral stenosis

Mitral Regurgitation (B)


- A Pansystolic Murmur at the apex that radiates to the axilla is characteristic of Mitral Regurgitation.
- The murmur of tricuspid regurge is also Pansystolic (soft) but does not radiate to the axilla.
- Aortic stenosis presents with an ejection systolic murmur, while Mitral Stenosis presents with a mid-diastolic
murmur at the apex.
- Therefore the correct option is B, Mitral Regurgitation.

1247. A 34yo laborer developed severe pain in his lower back after lifting a sack of sand. He also
complains of shooting pain down his leg. The GP has prescribed him complete bed rest, with
painkillers and also scheduled an MRI for him. What is the most likely dx?
a. Peripheral vascular disease
b. Intervertebral disc prolapse
c. Hairline fx of the spine
d. Sprain of the back muscles
e. Muscle injury

Sudden Severe Back Pain after Lifting Heavy Load (B)


- Sudden onset severe lower back pain accompanied by shooting pain down the leg is characteristic of
Intervertebral Disc Prolapse.
- There are no signs of other systems involved so this makes the diagnosis of peripheral vascular disease very
unlikely.
- Muscle injuries, sprain of back muscles, and hairline fracture of the spine cannot explain the shooting pain
down the leg.

1248. A young man returns to his hostel and gets headache and lethargy. Now presents with fever. There are
crepitations on the auscultation of lung. What is the most likely organism which would have caused his
symptoms?
a. Legionella pneumonia
b. Mycoplasma
c. Staphylococcus
d. Streptococcus

Atypical Pneumonia (A)


- History of living in boarding
- Bilateral consolidations
- Dry Cough
- Crepitations on auscultation
- Fever, anorexia and lethargy
The above points point clearly towards Legionella (A)
- Mycoplasma does not cause bilateral consolidation
- Staphylococcus causes bilateral basal cavitation and presents with a productive cough.
- Streptococcus shows lobar consolidation and presents with a productive cough.

1249. A pt is about to undergo surgery. Her Hgb=8.9g/dl and MCV=70. What is the best option for her?
a. Inv and postpone the surgery
b. Transfuse and proceed with surgery
c. Transfuse and defer surgery
d. Continue with surgery

Pre-Operative Considerations (A)


- Patients must have a full blood count even if fully fit
- If the patient is about to undergo an emergency operation, proceed even if Hb is low.
- If Hb is less than 8 then stabilize before proceeding with the operation
- For elective operations, only proceed if Hb is greater than 10.
- If Hb is less than 10 then defer the operation and investigate (as in this case)
- If Hb is less than 8 then blood transfusion must be done
- Since this patient has an Hb of 8.9 (less than 10) we have to postpone the surgery and investigate.
- Blood transfusion would be required if Hb had been less than 8.
- We cannot proceed with surgery since Hb is less than 10

1250. A 24yo male presents with discomfort in the groin area and scrotal swelling. Exam: scrotal skin is
normal. What would be the next best step?
a. Urgent US
b. Urgent surgery
c. OPD referral
d. Antibiotics

Groin Discomfort and Scrotal Swelling (C)


-Discomfort in the groin and scrotal swelling dont indicate an emergency. This rules out urgent ultrasound
and urgent surgery.
- Since the diagnosis is unclear at this point and there are no signs of infection, antibiotics cannot be
prescribed.
- The only logical approach would be an OPD referral (C).

1251. A 22yo girl unhappy about her weight with BMI=22. She likes to have her dinner in an expensive
restaurant. She does excessive shopping. K+=3.3. What is the dx?
a. Anorexia nervosa
b. Bipolar
c. OCD
d. Bulimia
Answer: D. Bulimia. Patient likes to have dinner in expensive restaurant, normal BMI ( as patient after eating
go for self induced vomiting) and low potassium point towards the diagnosis. Bulimia nervosa is a type of
eating disorder characterised by episodes of binge eating followed by intentional vomiting
EXCLUSION:
A. Anorexia Nervosa:
People with anorexia nervosa maintain a low body weight as a result of a preoccupation with weight,
construed as either a fear of fatness or a pursuit of thinness. In spite of this, they believe they are fat and are
terrified of becoming what is, in reality, a normal weight or shape. A diagnosis of anorexia nervosa is based on
low body weight, weight loss measures (particularly extreme dieting), psychological features (usually including
distorted body image), along with physical and endocrine sequelae.
B. Bipolar Disorder:
Bipolar disorder is a chronic episodic illness associated with behavioural disturbances. It used to be called
manic depression. It is characterised by episodes of mania (or hypomania) and depression. Either one can
occur first and one may be more dominant than the other but all cases of mania eventually develop
depression.
Bulimia nervosa is an eating disorder characterised by repeated episodes of uncontrolled overeating (binges)
followed by compensatory weight loss behaviours.

Features include:

Excessive preoccupation with body weight and shape


Undue emphasis on weight in self-evaluation
Feeling of lack of control over eating
Compensatory weight control mechanisms which can be:
o Self-induced vomiting
o Fasting
o Intensive exercise
o Abuse of medication such as laxatives, diuretics, thyroxine or amfetamines

Presentation
The history often dates back to adolescence. The core features include:
o Regular binge eating. Loss of control of eating during binges.
o Attempts to counteract the binges - eg, vomiting, using laxatives, diuretics, dietary
restriction and excessive exercise.
o Preoccupation with weight, body shape, and body image.
o Preoccupation with food and diet. This is often rigid or ritualistic, and deviations from a
planned eating programme cause distress. The affected person therefore starts to
avoid eating with others and becomes isolated.
o Mood disturbance and anxiety are common, as are low self-esteem, and self-harm.
o Severe comorbid conditions may be present - eg, depression and substance abuse.
o Periods may be irregular.

Investigations
These are usually normal apart from serum potassium, which is often low.
Renal function and electrolytes should be checked in view of frequent self-induced vomiting.

Management:

referral for specialist care is appropriate in all cases

cognitive behaviour therapy (CBT) is currently considered first-line treatment


interpersonal psychotherapy is also used but takes much longer than CBT
pharmacological treatments have a limited role - a trial of high-dose fluoxetine is currently licensed for
bulimia but long-term data is lacking

1252. A 59yo pt has been dx with HTN. His BP has been >160/90 mmHg on 3 separate occasions. His
biochemical profile is as follows: Na+=145 mmol/l, K+=6.2mmol/l, creatinine=112 umol/l, urea=5.7mmol/l.
What is the most appropriate anti-HTN drug for him?
a. Amlodipine
b. Bendroflumethiazide
c. Ramipril
d. Lorsartan
e. Propranolol
Answer: B.Bendroflumethiazide. the patient has hyperkalemia so as in order to correct that thiazide should be
given as thiazide cause hypokalemia. so it B bendroflumethiazide although recent guidelines now recommend
other thiazide like diuretics such as Indapamide and chlortalidone but it's an old question so
bendroflumethiazide is the answer.

Thiazide Diuretics
Thiazide diuretics work by inhibiting sodium absorption at the beginning of the distal convoluted tubule (DCT).
Potassium is lost as a result of more sodium reaching the collecting ducts. Thiazide diuretics have a role in the
treatment of mild heart failure although loop diuretics are better for reducing overload. The main use of
bendroflumethiazide was in the management of hypertension but recent NICE guidelines now recommend
other thiazide-like diuretics such as indapamide and chlortalidone.
Common adverse effects
dehydration
postural hypotension
hyponatraemia, hypokalaemia, hypercalcaemia
gout
impaired glucose tolerance
impotence

Rare adverse effects


thrombocytopenia
agranulocytosis
photosensitivity rash
pancreatitis

1253. A 22yo girl had a fight with her boyfriend and then took 22 tabs of paracetamol. She was
commenced on N-acetyl cysteine and she was medically fit to go home the following day. Which
of the following does she require?
a. OPD referral to relationship counselor
b. OPD referral to psychiatrist
c. Inpatient referral to psychiatrist
d. Inpatient referral to psychologist
Answer: C. inpatient referral to psychiatrist. In this case the patient is suicidal so we can not discharge the
patient and patient should be admitted in psychiatry department.

1254. A 74yo man presents with sudden onset of with right sided weakness and slurred speech. He also has
loss of sensation over the right side of the body and visual field defects. CT shows
ischemic stroke. What is the most appropriate management?
a. Alteplase
b. Streptokinase
c. Nimodipine
d. Aspirin
e. Labetalol
Answer: D. Aspirin. The CT scan shows ischemic stroke and time when patient presented is not given if it
would have less than 4.5hrs we would have given alteplase.

STROKE MANAGEMENT
The Royal College of Physicians (RCP) published guidelines on the diagnosis and management of patients
following a stroke in 2004. NICE also issued stroke guidelines in 2008, although they modified their guidance
with respect to antiplatelet therapy in 2010.

Selected points relating to the management of acute stroke include:


blood glucose, hydration, oxygen saturation and temperature should be maintained within normal limits
blood pressure should not be lowered in the acute phase unless there are complications e.g.
Hypertensive encephalopathy*
aspirin 300mg orally or rectally should be given as soon as possible if a haemorrhagic stroke has been
excluded
with regards to atrial fibrillation, the RCP state: 'anticoagulants should not be started until brain
imaging has excluded haemorrhage, and usually not until 14 days have passed from the onset of an
ischaemic stroke'
if the cholesterol is > 3.5 mmol/l patients should be commenced on a statin. Many physicians will delay
treatment until after at least 48 hours due to the risk of haemorrhagic transformation
Thrombolysis
Thrombolysis should only be given if:
it is administered within 4.5 hours of onset of stroke symptoms (unless as part of a clinical trial)
haemorrhage has been definitively excluded (i.e. Imaging has been performed)
Alteplase is currently recommended by NICE.
Secondary prevention

NICE also published a technology appraisal in 2010 on the use of clopidogrel and dipyridamole

Recommendations from NICE include:


clopidogrel is now recommended by NICE ahead of combination use of aspirin plus modified release
(MR) dipyridamole in people who have had an ischaemic stroke
aspirin plus MR dipyridamole is now recommended after an ischaemic stroke only if clopidogrel is
contraindicated or not tolerated, but treatment is no longer limited to 2 years' duration
MR dipyridamole alone is recommended after an ischaemic stroke only if aspirin or clopidogrel are
contraindicated or not tolerated, again with no limit on duration of treatment

With regards to carotid artery endarterectomy:


recommend if patient has suffered stroke or TIA in the carotid territory and are not severely disabled
should only be considered if carotid stenosis > 70% according ECST** criteria or > 50% according to
NASCET*** criteria

1255. The artery that runs along the left AV groove. What is the single most appropriate option?
a. Left internal mammary artery
b. Left anterior descending artery
c. Circumflex artery
d. Left main stem (LMS) post descending artery
e. Diagonal branch
Answer: C. Circumflex Artery.
Left Anterior Descending artery descends into interventricular groove.

1256. A 26yo man presents with painless hematuria. He has no other complaints and on examination no other
abnormality is found. What is the most appropriate initial inv to get to a dx?
a. Cystoscopy
b. Midstream urine for culture
c. Abdominal US
d. MRI spine
e. Coag screening
Answer:C.Abdominal US. As the patient is young so we can't go for cystoscopy. The initial investigation in this
patient should be abdominal US to exclude any pathology in the genitourinary tract. PKD can also be
suspected here so US should be done to exclude that aswell first.

1257. A pt, 50yo smoker and heavy drinker, presents with complaints of racing heart. A 24h EKG
comes out normal. What is your next step in management?
a. Echo
b. Reassure
c. Stress test
Answer: B.Reassure. Everything is normal in this patient so reassure the patient. If there would have been
chest pain or past h/o chest pain then stress test could be a viable option.

1258. A 36yo woman came with uterine bleeding. Vaginal US reveals uterine thickness=12mm. what is
the most probable dx?
a. Cervical ca
b. Endometrial ca
c. Ovarian ca
d. Breast ca
e. Vaginal ca
Answer: B.Endometrial CA. The increased uterine thickness points towards the diagnosis.
Endometrial cancer is classically seen in postmenopausal women but around 25% of cases occur before the
menopause. It usually carries a good prognosis due to early detection

The risk factors for endometrial cancer are as follows*:


obesity
nulliparity
early menarche
late menopause
unopposed oestrogen. The addition of a progesterone to oestrogen reduces this risk (e.g. In HRT). The
BNF states that the additional risk is eliminated if a progestogen is given continuously
diabetes mellitus
tamoxifen
polycystic ovarian syndrome

Features
postmenopausal bleeding is the classic symptom
pre-menopausal women may have a change intermenstrual bleeding
pain and discharge are unusual features

Investigation
first-line investigation is trans-vaginal ultrasound - a normal endometrial thickness (< 4 mm) has a high
negative predictive value
hysteroscopy with endometrial biopsy
Management
localised disease is treated with total abdominal hysterectomy with bilateral salpingo-oophorectomy.
Patients with high-risk disease may have postoperative radiotherapy
progestogen therapy is sometimes used in frail elderly women not consider suitable for surgery

1259. A 30yo woman has PID which was treated with metronidazole and cephalosporin. It is getting worse.
What is the next best inv?
a. Endocervical swab
b. US
c. Laparotomy
d. High vaginal swab
Answer: B. US. US is done to rule out tubo ovarian abscess.

Pelvic inflammatory disease (PID) is a term used to describe infection and inflammation of the female pelvic
organs including the uterus, fallopian tubes, ovaries and the surrounding peritoneum. It is usually the result of
ascending infection from the endocervix

Causative organisms
Chlamydia trachomatis - the most common cause
Neisseria gonorrhoeae
Mycoplasma genitalium
Mycoplasma hominis

Features
lower abdominal pain
fever
deep dyspareunia
dysuria and menstrual irregularities may occur
vaginal or cervical discharge
cervical excitation

Investigation
screen for Chlamydia and Gonorrhoea

Management
due to the difficulty in making an accurate diagnosis, and the potential complications of untreated PID,
consensus guidelines recommend having a low threshold for treatment
oral ofloxacin + oral metronidazole or intramuscular ceftriaxone + oral doxycycline + oral
metronidazole
RCOG guidelines suggest that in mild cases of PID intrauterine contraceptive devices may be left in. The
more recent BASHH guidelines suggest that the evidence is limited but that ' Removal of the IUD should
be considered and may be associated with better short term clinical outcomes'

Complications
infertility - the risk may be as high as 10-20% after a single episode
chronic pelvic pain
ectopic pregnancy
perihepatitis
tubo ovarian abscess
reiter's syndrome

1260. A pregnant woman had hit her chest 3wks ago. Now she is 24wks pregnant and presents with left upper
quadrant mass with dimpling. What is the most probable dx?
a. Breast ca
b. Carcinoma
c. Fibroadenoma
d. Fibroadenosis
e. Fatty necrosis of breast
Answer:E. Fatty necrosis of breast. H/o of trauma to the breast and lump with dimpling point towards the
diagnosis.

Fat necrosis

It tends to be large, fatty breasts in obese women that have this problem:

It usually follows trauma.


The lump is usually painless and the skin around it may look red, bruised or dimpled.
Biopsy may be required, but if the diagnosis is confirmed, no further management is indicated.

1261. A pregnant pt with Rh ve who hasnt been prv sensitized delivers her first baby without any
prbs. What would be the latest time to administer anti-sensitization?
a. 6h PP
b. 24h PP
c. 48h PP
d. 72h PP
e. 5d PP
Answer: D.72h pp.
Anti-D immunoglobulin should be given as soon as possible (but always within 72 hours) in the following
situations:
delivery of a Rh +ve infant, whether live or stillborn
any termination of pregnancy
miscarriage if gestation is > 12 weeks
ectopic pregnancy
external cephalic version
antepartum haemorrhage
amniocentesis, chorionic villus sampling, fetal blood sampling

1262. A 30yo primigravida who is 30wks GA presents to the L&D with absent fetal movements. She also
complains of severe headache, heartburn and seeing floaters before her eyes for the last
few days. Exam: BP=170/110 mmHg, urine protein=++++, rock hard uterus, no visible signs of
fetal movements. Choose the single most likely dx?
a. Abruption of placenta 2nd pre-eclampsia
b. Antepartum hemorrhage
c. Placenta previa
d. Primary PPH
e. IUFD
f. Abruption of placenta due to trauma
Answer: A. Abruption of placenta secondary to preeclampsia. Maternal Hypertension is the most important
cause of placental abruption.
Rigid abdomen/ hard rock uterus here indicates peritoneal irritation due to bleeding (concealed haemorrhage)
exclusion:
IUFD: We need a lot more to conclude IUFD and cannot be based on just rock hard uterus and no visible signs
of fetal movements (such as auscultation, cardiotocography, real time ultrasonography etc)
ABRUPTION OF PLACENTA
Abruption is the premature separation of a normally placed placenta before delivery of the fetus, with blood
collecting between the placenta and the uterus. It is one of the two most important causes of antepartum
haemorrhage (the other being placenta praevia), accounting for 30% of all cases of antepartum haemorrhage.

There are two main forms:

Concealed (20% of cases) - where haemorrhage is confined within the uterine cavity and is the
more severe form. The amount of blood lost is easily underestimated.
Revealed (80%) - where blood drains through the cervix, usually with incomplete placental
detachment and fewer associated problems.

Risk factors

There are recognised factors that increase the risk - these include:

Previous abruption carries the highest risk of abruption in current pregnancy.


Multiple pregnancy: twice as common with a twin pregnancy than with a singleton.
Trauma:
o Road traffic accident.
o Domestic violence.
o Iatrogenic - eg, external cephalic version.
Threatened miscarriage earlier in current pregnancy.
Pre-eclampsia and maternal hypertension (most imp risk factor accounting for approx: 44% of
cases)
Multiparity.
Previous caesarean section.
Non-vertex presentations.
Smoking.
Cocaine or amphetamine use during pregnancy.
Thrombophilia.
Intrauterine infections.
Polyhydramnios.

The clinical features of placental abruption depend on the size and site of the bleeding.
The grades of haemorrhage described are:

mild - in this case there is only a small area of placental separation and the blood loss is usually less
than 200 ml. There may be abdominal discomfort and the uterus may be tender
moderate - up to a 1/3 of the placenta separates. There is more severe bleeding (200-600 ml). The
patient complains of abdominal pain. On examination the patient may have tachycardia but does not
have signs of hypovolemia. The uterus is tender. Fetal heart sounds are present
severe - in this condition more than half of the placenta separates. The abdominal pain is more severe.
On examination the uterus is tender and rigid (hard) - it may be impossible to feel the fetus. Fetal heart
sounds are reduced or absent. The patient may be in a state of hypovolaemic shock

Diagnosis
Abruption is a clinical diagnosis with no available sensitive or reliable diagnostic tests.
Management:
Guidance from the Royal College of Obstetricians and Gynaecologists for moderate or severe placental
abruption is to follow ABCD of resuscitation:

Assess Airway and Breathing: high-flow oxygen.


Evaluate Circulation:
Access fetus and Decide on Delivery

1263. A 38yo woman, 10d post partum, presents to her GP with a hx of passing blood clots per vagina since
yesterday. Exam: BP=90/40 mmHg, pulse=110 bpm, temp=38C, uterus tender on palpation and fundus is 2 cm
above umbilicus, blood clots +++. Choose the single most likely dx?
a. Abruption of placenta 2nd preeclampsia
b. Concealed hemorrhage
c. Primary PPH
d. Secondary PPH
e. Retained placenta
f. Scabies
Answer:D. Secondary PPH. The 10day post partum, signs of shock and blood clots all point towards the
diagnosis of secondary pph.

Post-partum haemorrhage

Post-partum haemorrhage (PPH) is defined as blood loss of > 500mls and may be primary or secondary

Primary PPH
occurs within 24 hours
affects around 5-7% of deliveries
most common cause of PPH is uterine atony (90% of cases). Other causes include genital trauma and
clotting factors

Risk factors for primary PPH include*:


previous PPH
prolonged labour
pre-eclampsia
increased maternal age
polyhydramnios
emergency Caesarean section
placenta praevia
macrosomia
ritodrine (a beta-2 adrenergic receptor agonist used for tocolysis)

Management
ABC
IV syntocinon (oxytocin) 10 units or IV ergometrine 500 micrograms
IM carboprost
other options include: B-Lynch suture, ligation of the uterine arteries or internal iliac arteries
if severe, uncontrolled haemorrhage then a hysterectomy is sometimes performed as a life-saving
procedure

Secondary PPH
occurs between 24 hours - 12 weeks**
due to retained placental tissue or endometritis

*the effect of parity on the risk of PPH is complicated. It was previously thought multiparity was a risk factor
but more modern studies suggest nulliparity is actually a risk factor

1264. A 22yo lady who is in her last trimester of pregnancy comes with hx of exposure to a child dx with
chicken pox 1d ago. She was investigated and was +ve for varicella antibody. What is the single most
appropriate management?
a. Give varicella I/g
b. Quarantine
c. Give varicella vaccination
d. Oral acyclovir
e. Reassure,
Answer: E. Reassure. Lady is +ve for varicella antibody so no need to give varicella zoster Igs, just reassure the
patient.

Chickenpox exposure in pregnancy


Chickenpox is caused by primary infection with varicella zoster virus. Shingles is reactivation of dormant virus
in dorsal root ganglion. In pregnancy there is a risk to both the mother and also the fetus, a syndrome now
termed fetal varicella syndrome

Risks to the mother


5 times greater risk of pneumonitis
Fetal varicella syndrome (FVS)
risk of FVS following maternal varicella exposure is around 1% if occurs before 20 weeks gestation
studies have shown a very small number of cases occurring between 20-28 weeks gestation and none
following 28 weeks
features of FVS include skin scarring, eye defects (microphthalmia), limb hypoplasia, microcephaly and
learning disabilities

Other risks to the fetus


shingles in infancy: 1-2% risk if maternal exposure in the second or third trimester
severe neonatal varicella: if mother develops rash between 5 days before and 2 days after birth there
is a risk of neonatal varicella, which may be fatal to the newborn child in around 20% of cases

Management of chickenpox exposure


if there is any doubt about the mother previously having chickenpox maternal blood should be
urgently checked for varicella antibodies
if the pregnant women is not immune to varicella she should be given varicella zoster immunoglobulin
(VZIG) as soon as possible. RCOG and Greenbook guidelines suggest VZIG is effective up to 10 days post
exposure
consensus guidelines suggest oral aciclovir should be given if pregnant women with chickenpox present
within 24 hours of onset of the rash

1265. A 22yo woman who is 20wk pregnant came with pain and bleeding per vagina. Exam: os is not open.
What is the single most likely dx?
a. Threatened abortion
b. Missed abortion
c. APH
d. Miscarriage
e. Inevitable abortion
Answer. A. Threatened Abortion. Pain and bleeding per vagina and os closed all point towards the diagnosis.
MISCARRIAGE
Miscarriage is defined as the loss of a pregnancy before 24 weeks of gestation.
Classification of miscarriage is as follows:
Threatened miscarriage: mild symptoms of bleeding. Usually little or no pain. The cervical os is
closed.
Inevitable miscarriage: usually presents with heavy bleeding with clots and pain. The cervical os is
open. The pregnancy will not continue and will proceed to incomplete or complete miscarriage.
Incomplete miscarriage: this occurs when the products of conception are partially expelled. Many
incomplete miscarriages can be unrecognised missed miscarriages.
Missed miscarriage: the fetus is dead but retained. The uterus is small for dates. A pregnancy test
can remain positive for several days. It presents with a history of threatened miscarriage and
persistent, dirty brown discharge. Early pregnancy symptoms may have decreased or gone.
Habitual or recurrent miscarriage: three or more consecutive miscarriages.
1266. A 32yo lady G1, 28wks GA came to her ANC with a concern about pain relief during labour. She has no
medical illnesses and her pregnancy so far has been uncomplicated. She wishes to feel her baby being born
but at the same time she wants something to work throughout her labour. What method of pain relief best
matches this ladys request?
a. C-section
b. Pudendal block
c. Entonox
d. TENS
e. Pethidine

C. Entonox.

Pain Relief In Labor:


1. Transcutaneous electrical nerve stimulation (TENS)
Randomised controlled trials provide no compelling evidence for TENS having any analgesic effect during
labour. so it is not recommended by NICE.

2. Acupuncture and hypnosis may be beneficial for the management of pain during labour;
3. Water/birthing pool: Immersion in water during labour is claimed to increase maternal relaxation and
reduce analgesic requirements. It is supported by the Royal College of Obstetricians and Gynaecologists
(RCOG) for healthy women with uncomplicated pregnancies.

4. Nitrous oxide and oxygen (Entonox)


This is a 50:50 mixture inhaled during painful contractions during the first and second stages of labour. It is
often used as a supplement to pethidine.

The main advantages are that it is under the patient's control, it takes effect within seconds and
wears off quickly with no side-effects.
Inhaled analgesia appears to be effective in reducing pain intensity and in giving pain relief in
labour

5. Intramuscular opiate: Parenteral opioids provide some relief from pain in labour but are associated with
adverse effects - eg, maternal nausea, vomiting and drowsiness.

6. Epidural analgesia: Epidural analgesia is a central nerve block technique achieved by injection of a local
anaesthetic close to the nerves that transmit pain. It is widely used as a form of pain relief in labour.
Advantages: It is the most effective way of relieving pain in labour - providing complete relief in 95% of
cases. It also has the benefit of avoiding need for greater analgesia/general anaesthetic if forceps, vacuum
extraction or caesarean section are required. It is not associated with increase in symptoms related to perineal
trauma and pelvic floor muscle weakness.

7. Ambulatory epidural: This is a low-dose epidural that relieves pain, but allows women to walk about during
labour.

8. Local analgesia: This is used for women who have not had an epidural but require forceps or vacuum
extraction delivery. It is also used for repair of episiotomy or perineal tear.
Pudendal nerve block: using lidocaine behind each ischial spine of the pelvis via the vagina.

1267. A primipara at full term in labor has passed show and the cervix is 3cm dilated. What is the single most
appropriate management for her labor?
a. Repeat vaginal examination in 4h
b. CTG
c. IV syntocin drip
d. Repeat vaginal examination in 2h
e. Induction of labour

It is the first stage of labor since the cervix is only 3cm dilated. First stage of labour ends when the cervix is
10cm dilated.

The first stage of labour


Latent phase (not necessarily continuous):
there are painful contractions, the cervix initially effaces (becomes shorter and
softer) then dilates to 4cm.
Established phase:
contractions with dilatation from 4 cm. A satisfactory rate of dilatation from 4 cm is 0.5cm/h.

The 1st stage generally takes 818h in a primip, and 512h in a multip.

During the first stage check maternal BP, and T 4-hourly, pulse hourly;
assess the contractions every 30min, their strength and their frequency (ideally 34 per 10min, lasting up to 1
min).
Offer vaginal examination e.g every 4h to assess the degree of cervical dilatation, the position and the station
of the head.
Auscultate fetal heart rate (if not continuously monitored), by Pinard or Doppler every 15min, listening for
1min after a contraction.

1268. A 36yo pregnant woman comes for evaluation with her husband. Her husband has been
complaining of morning sickness, easy fatigability and even intermittent abdominal pain. What
is the husband suffering from?
a. Ganser syndrome
b. Couvade syndrome
c. Pseudo-psychosis
d. Stockholm syndrome
e. Paris syndrome

Ganser syndrome is a type of factitious disorder, a mental illness in which a person deliberately and
consciously acts as if he or she has a physical or mental illness when he or she is not really sick.

Couvade syndrome, also called sympathetic pregnancy, is a proposed condition in which a partner
experiences some of the same symptoms and behavior of an expectant mother. These most often include
minor weight gain, altered hormone levels, morning nausea, and disturbed sleep patterns.
Stockholm syndrome, or capture-bonding, is a psychological phenomenon in which hostages express empathy
and sympathy and have positive feelings toward their captors, sometimes to the point of defending and
identifying with the captors.

Pseudo Psychosis: As the name itself says, pseudo (psychosis), which means false, is not a form of psychosis,
but instead, pseudo psychosis is when someone is convinced that they are suffering from psychosis when they
are not. This doesnt mean that the person is pretending or faking the symptoms of psychosis such as
hallucinations, hearing voices or other forms of being completely detached from reality.

Paris Syndrome: is a transient psychological disorder exhibited by some individuals visiting or vacationing in
Paris or elsewhere in Western Europe. It is characterized by a number of psychiatric symptoms such as acute
delusional states, hallucinations, feelings of persecution (perceptions of being a victim of prejudice,
aggression, or hostility from others), derealization, depersonalization, anxiety, and also psychosomatic
manifestations such as dizziness, tachycardia, sweating, and others

1269. A woman comes to the ED complaining of pain in the right side of the abdomen, she has 7wks
amenorrhea. Her pregnancy test is +ve and US scan shows an empty uterus. What is the next step?
a. Laparoscopy
b. HCG measurements
c. US
d. Laparotomy
e. Culdo-centhesis

Answer is B.
This is a case of ectopic pregnancy.
Always think of an ectopic in a sexually active woman with abdominal pain; bleeding; fainting; or diarrhoea
and vomiting. There is generally ~8 weeks amenorrhoea but an ectopic may present before a period is
missed. An early sign is often dark blood loss (prune juice, as the decidua is lost from the uterus) or fresh.
Diagnosis: Early diagnosis is vital. Dipstix testing for HCG (human chorionic gonadotrophin)
is sensitive to values of 25IU/L. do ultrasound. If HCG >6000IU/L and an intrauterine gestational sac is not
seen, ectopic pregnancy is very likely, as is the case if HCG 10001500IU/L and no sac is seen on transvaginal
ultrasound.

1270. A 23yo woman who has had several recent partners has experienced post-coital bleeding on gentle
contact. What is the single most likely cause of her vaginal discharge?
a. Cervical ca
b. Cervical ectropion
c. CIN
d. Chlamydial cervicitis
e. Gonococcal cervicitis

Answer is D

Causes of postcoital bleeding


Infection.
Cervical ectropion - especially in those women taking the combined oral contraceptive pill (COCP).
Cervical or endometrial polyps.
Vaginal cancer.
Cervical cancer - usually apparent on speculum examination.
Trauma.

In this case the history of several recent partners points towards a sexually transmitted disease so it is
chlamydial cervicitis as chlamydia is transmitted sexually.

Chlamydial cervicitis:

Risk factors
Age <25 (the highest prevalence in women occurs between ages 16-19 years and in men between
ages 20-24 years).
Two or more sexual partners in the preceding year.
A recent change in sexual partner.
Non-barrier contraception.
Infection with another STI.
Poor socio-economic status.
Genetic predisposition

Symptoms: Vaginal discharge.

Dysuria (always consider chlamydia as a cause of sterile pyuria).


Vague lower abdominal pain.
Fever.
Intermenstrual or postcoital bleeding.
Dyspareunia.

Signs:

A friable, inflamed cervix, sometimes with a follicular or 'cobblestone' appearance, with contact
bleeding.
Mucopurulent endocervical discharge.
Abdominal tenderness.
Pelvic adnexal tenderness on bimanual palpation.
Cervical excitation.

Treatment: Doxycycline for 7 days or azithromycin single dose.

1271. A 68yo woman presents with post-coital bleeding following her first episode of sexual
intercourse in 10yrs. What is the single most likely cause that has led to post-coital bleeding?
a. Endometrial ca
b. Atrophic vaginitis
c. Endometrial polyp
d. Cervical ca
e. Cervical ectropion
Answer is Atrophic vaginitis.
In a case of post menopausal bleeding always first rule out endometrial Carcinoma. Here since most likely
cause is asked it is atrophic vaginitis.

Aetiology
Vaginal atrophy. The most common cause of PMB.
Use of HRT.
Endometrial hyperplasia; simple, complex, and atypical.
Endometrial cancer. The probability of a woman presenting with PMB having endometrial cancer is
10%. However, 90% of women with endometrial cancer present with PMB.[2]
Endometrial polyps or cervical polyps.
Cervical cancer; remember to check if the cervical smear is up-to-date.
Uterine sarcoma (rare).
Ovarian cancer, especially oestrogen-secreting (theca cell) ovarian tumours.
Vaginal cancer (very uncommon).
Vulval cancer may bleed, but the lesion should be obvious.
Non-gynaecological causes including trauma or a bleeding disorder.

1272. A 28yo woman 8wks GA had PID treated prvly and now comes with vaginal bleeding, rigid
abdomen, BP=80/50 mmHg, pulse=140 bpm. What is the most probable dx?
a. Threatened abortion
b. Miscarriage
c. Missed abortion
d. Tubal pregnancy
e. Inevitable abortion

MISCARRIAGE
Miscarriage is defined as the loss of a pregnancy before 24 weeks of gestation.
Classification of miscarriage is as follows:
Threatened miscarriage: mild symptoms of bleeding. Usually little or no pain. The cervical os is
closed.
Inevitable miscarriage: usually presents with heavy bleeding with clots and pain. The cervical os is
open. The pregnancy will not continue and will proceed to incomplete or complete miscarriage.
Incomplete miscarriage: this occurs when the products of conception are partially expelled. Many
incomplete miscarriages can be unrecognised missed miscarriages.
Missed miscarriage: the fetus is dead but retained. The uterus is small for dates. A pregnancy test
can remain positive for several days. It presents with a history of threatened miscarriage and
persistent, dirty brown discharge. Early pregnancy symptoms may have decreased or gone.
Habitual or recurrent miscarriage: three or more consecutive miscarriages.

1273. A 34yo primigravida who is 16wk GA comes for routine antenatal check up. Her
BP=160/100mmHg. She has a hx of repeated childhood UTI. What is the most likely cause of her
high BP?
a. Essential HTN
b. Chronic pyelonephritis
c. Acute pyelonephritis
d. Pre-eclampsia
e. Chronic UTI

Recurrent episodes of UTI point towards chronic pyelonephritis.


Pre eclampsia has HTN with proteinuria/oedema after 20 weeks of gestation.
Risk Factors:
any structural renal tract anomalies, obstruction or calculi
Children with vesicoureteral reflux
Intrarenal reflux in neonates
Genetic predisposition
Any factors predisposing to recurrent urinary infection - eg, neurogenic bladder

Presentation:
Fever
Malaise
Loin pain
Nausea
Vomiting
Dysuria
Hypertension
Failure to thrive

1274. A 24yo woman has had lower abdominal pain for 12h. She is otherwise well. She is at 10wks GA in a
planned pregnancy. What is the single most appropriate test to inv the cause of acute abdomen in this lady?
a. Abdominal US
b. Anti-phospholipid screen
c. CBC
d. Transvaginal US
e. Laparoscopy
The 2 main differentials of lower abdominal pain in the first part of pregnancy are ectopic pregnancy and
miscarriage. Both can have vaginal bleeding to but in miscarriage vaginal bleeding is more pronounced and
mostly occurs before abdominal pain so we should first go for Transvaginal US to rule out ectopic pregnancy
first as The most accurate method to detect a tubal pregnancy is transvaginal ultrasound.

1275. A pt is at term and in labor, the membranes have ruptured, the liquor contains meconium but the CTG is
normal. The cervix is 3cm dilated. What is the single most appropriate action?
a. BP monitoring
b. CTG
c. C-section
d. Fetal scalp blood sample
e. Internal rotation
d. Fetal scalp blood sample

NICE suggests continuous CTG monitoring, if its in the options it would have been the best choice. FBS
recommended only when CTG is abnormal.
one statement: Since ctg is normal, we need to do fetal scalp blood sampling to rule in/out fetal distress.
These recommendations are from the National Institute for Health and Care Excellence (NICE), 2014
Intrapartum
If significant meconium staining is noted in labour, there should be continuous electronic foetal
monitoring.
This is defined as dark green or black amniotic fluid that is thick or tenacious, or any amniotic fluid that
contains lumps of meconium.
Transfer mother to obstetric-led care, if it is safe to do so and delivery is not imminent.
If there are signs of fetal distress, a fetal blood sample should be obtained. If pH is <7.21, there should
be emergency delivery.
Ensure that the advanced resuscitation unit and appropriately trained staff are available.
There should be no suction prior to delivery.

1276. A pt is at term and labor. The head has been delivered and you suspect shoulder dystocia. What is the
single most appropriate action?
a. C-section
b. Episiotomy
c. External rotation
d. Fetal scalp blood sample
e. Instrumental delivery

b. Episiotomy
Head is already delivered none of the other options except episiotomy seems appropriate

Royal College of Obstetricians and Gynaecologists (RCOG) guidelines for shoulder dystocia.[1]
Get help. In addition to a senior obstetrician, an anaesthetist and paediatrician should be called.
Stop the mother pushing. This may make impaction of the shoulders worse.
McRoberts' manoeuvre - the patient hyperflexes her hips so they are against her abdomen. Mothers in
labour may not have enough energy to do this by themselves and may need the assistance of others in
the room - which is usually the case. Posterolateral pressure is applied suprapubically with traction on
the fetal head. This is the most effective procedure and should be performed first (success rates are up
to 90%).
If this fails, an episiotomy may be needed to facilitate the obstetrician trying second-line manoeuvres -
but the need for a caesarean section should be considered.
Rubin's manoeuvre - press on the posterior fetal shoulder, thereby creating more space to allow the
anterior shoulder to be delivered.
Woods' screw manoeuvre - turning the anterior shoulder to the posterior position.
If these fail then delivery of the posterior shoulder may help.
However, at all times the need for a caesarean section should be considered and should not be
delayed.
NB: fundal pressure should NOT be applied.

1277. A 29yo female at 28wks GA presents to you with complains of hard stools and constipation for last 2wks.
CTG shows fetal tachycardia. What is the single most appropriate tx?
a. Oral laxatives
b. Fiber diet
c. Phosphate enema
d. Lactulose
e. Reassure
b. Fiber diet
fiber diet free of any side-effects and safe in pregnancy

1278. A 16yo girl presents with heavy bleeding. What is the most appropriate initial inv?
a. Endometrial sampling
b. Transvaginal US
c. Hysteroscopy
d. Pelvic US
e. Exam under anesthesia

d. Pelvic US
1st initial investigation is the less invasive one always.
to confirm /exclude a pregnancy /mass

1279. A woman who is 7wks pregnant presents with excessive and severe vomiting and put on IV
fluids and anti-emetic (ondansteron). She is complaining of severe headache and cant take oral
fluids. What is the most appropriate management?
a. Termination of pregnancy
b. TPN
c. Feeds via NGT
d. P6 acupressure
e. IV hydrocortisone

e. IV hydrocortisone
antiemetics .. if they do not work .. steroids
tpn very rarely used
B and C for long term is not viable. D has little benefit in terms of allopathy. A is too extreme at this early stage
and it is only done when all methods have failed and there is risk to the mother. E will alleviate the symptoms
and improve likelihood of a viable pregnancy.
HYPEREMESIS GRAVIDARUM
Vomiting that begins after 12 weeks of gestation is unlikely to be caused by hyperemesis gravidarum

Management[3]
Advice, including dietary advice, and support
Fluid and electrolyte replacement
Nutritional support (enteral or parenteral) may be required.
Vitamin supplements
Thromboprophylaxis
Anti-emetic drugs
Drug treatment options include cyclizine, metoclopramide, prochlorperazine, promethazine,
chlorpromazine, domperidone and ondansetron (selective 5-hydroxytryptamine receptor antagonist),
or combinations of these agents.
There is no evidence that any one anti-emetic is better than another.
Corticosteroids: may be used for intractable (failure to respond to conventional treatment) cases of
severe hyperemesis gravidarum in secondary care.
1280. A young lady with primary amenorrhea has normal LH, FSH, estradiol and prolactin. Choose the single
most likely dx?
a. PCOS
b. POF
c. Absent uterus
d. Absent ovaries
e. Turners syndrome

c. Absent uterus
in others hormones will be deranged.
PCOS raised LH:FSH ratio, Prolactin may be normal or mildly elevated. Testosterone may be normal or mildly
elevated.
POF raised FSH, LH levels
Turners syndrome LH and FSH may be elevated
Mayer-Rokitansky-Kster-Hauser (MRKH) syndrome is a disorder that occurs in females and mainly affects the
reproductive system. This condition causes the vagina and uterus to be underdeveloped or absent.

1281. An obese lady presents with primary amenorrhea. She has high LH, normal FSH and slightly high
prolactin levels. Choose the single most likely dx?
a. PCOS
b. POF
c. Hypothyroidism
d. Pregnancy
e. Primary obesity

a. PCOS

Features
subfertility and infertility
menstrual disturbances: oligomenorrhea and amenorrhoea
hirsutism, acne (due to hyperandrogenism)
obesity
acanthosis nigricans (due to insulin resistance)

Investigations
pelvic ultrasound: multiple cysts on the ovaries
FSH, LH, prolactin, TSH, and testosterone are useful investigations: raised LH:FSH ratio is a 'classical'
feature but is no longer thought to be useful in diagnosis. Prolactin may be normal or mildly elevated.
Testosterone may be normal or mildly elevated - however, if markedly raised consider other causes
check for impaired glucose tolerance

1282. A 38yo lady presents with amenorrhea has very high LH and FSH levels, normal prolactin and low
estradiol. Choose the single most likely dx?
a. PCOS
b. POF
c. Hypothyroidism
d. Pregnancy
e. Menopause

b. POF

Premature ovarian failure is defined as the onset of menopausal symptoms and elevated gonadotrophin levels
before the age of 40 years. It occurs in around 1 in 100 women.

Causes
idiopathic - the most common cause
chemotherapy
autoimmune
radiation

Features are similar to those of the normal climacteric but the actual presenting problem may differ
climacteric symptoms: hot flushes, night sweats
infertility
secondary amenorrhoea
raised FSH, LH levels

1283. A 77yo publican was admitted for an appendectomy. Post-op he becomes confused, agitated and starts
to pick at things. He is then given an IV drug which settles this confusion. Which of the following drugs was
given for his confusion?
a. Diazepam
b. Chlordiazepoxide
c. Thiamine
d. Vit B

b. Chlordiazepoxide
For alcohol withdrawal, chlordiazepoxide is 1st line. Diazepam is 2nd line.
Benzodiazepines are the recommended drugs for detoxification. They have a slower onset of action and
therefore are less likely to lead to abuse. A reducing dose of chlordiazepoxide over 5-7 days is commonly used.
Diazepam is an alternative.

Symptoms typically present about eight hours after a significant fall in blood alcohol levels. They peak
on day 2 and, by day 4 or 5, the symptoms have usually improved significantly.
Minor withdrawal symptoms (can appear 6-12 hours after alcohol has stopped)
o Insomnia and fatigue.
o Tremor.
o Mild anxiety/feeling nervous.
o Mild restlessness/agitation.
o Nausea and vomiting.
o Headache.
o Excessive sweating.
o Palpitations.
o Anorexia.
o Depression.
o Craving for alcohol.
Alcoholic hallucinosis (can appear 12-24 hours after alcohol has stopped)
o Includes visual, auditory or tactile hallucinations.
Withdrawal seizures (can appear 24-48 hours after alcohol has stopped)
o These are generalised tonic-clonic seizures.
Alcohol withdrawal delirium or 'delirium tremens' (can appear 48-72 hours after alcohol has stopped).

1284. A 65yo lady presents with dyspareunia. What will you give her for her condition?
a. HRT
b. COCP
c. Estrogen gel
d. Testosterone gel

c. Estrogen gel
In older women vaginal dryness due to hormonal deficiency mainly oestrogen
Atrophic vaginitis
Current treatment guidelines for vaginal atrophy recommend the use of minimally absorbed local vaginal
oestrogens, along with non-hormonal lubricants or moisturisers, coupled with maintenance of sexual activity.

1285. A 35yo lady with subserosal fibroid=4cm and submural fibroid=6cm is planning for a child.
Which way will you remove the fibroids?
a. Laproscopy
b. Vaginal myomectomy
c. Abdominal myomectomy
d. Drugs
e. Reassure

c. Abdominal myomectomy
Subserous...laparoscopic
Intramural...abdominal
Submucous...hysteroscopy
Mixed ...abdominal

1286. A 32yo presents with heavy blood loss, US: uterine thickness>14mm. What is the best possible
management for her?
a. COCP
b. UAE
c. Hysteroscopy myomectomy
d. Abdominal myomectomy
e. Endometrial ablation

b. UAE
UAE can shrink the endometrial bulk and resolve menorrhagia. So preferred here.
for Menorrhagia in young patient , childbearing age, UAE is preferred choice.
If fertility is not needed, thickness is < 3mm then endometrial ablation.
If fertility is required and >3mm then UAE
If fertility is not needed and >3mm then hysterectomy...
Since here the pt is 32yo so we assume we need to preserve her fertility and size is 14 mm so UAE is the
answer.
Uterine artery embolism UAE would be a very good option here as it would help reduce the endometrial
thickness, I assume she has endometrial hyperplasia here, myomectomy is treatment of choice for uterine
fibroids in women who are subfertile, COCP contain estrogen which would further worsen the hyperplasia,

Uterine fibroids
Features
may be asymptomatic
menorrhagia
lower abdominal pain: cramping pains, often during menstruation
bloating
urinary symptoms, e.g. frequency, may occur with larger fibroids
subfertility

Diagnosis
transvaginal ultrasound

Management
medical: symptomatic management e.g. with combined oral contraceptive pill. GnRH agonists may
reduce the size of the fibroid but are typically useful for short-term treatment
surgery is sometimes needed: myomectomy, hysterscopic endometrial ablation, hysterectomy
uterine artery embolization

1287. A pt comes with sudden loss of vision. Exam: high BP. Fundoscopy: retina appears swollen.
Which blood vessel occlusion is involved?
a. Branch RVO
b. Branch RAO
c. CRAO
d. CRVO

d. CRVO
In CRAO you'd see a pale white retina,and is usually secondary to a thromboembolus ie in people with
condition predisposing to a hypercoagulative state,and yes HTN is one such state...But a swollen retina
indicates blood stasis,ie congestion in the retina due to outflow obstruction due to blockage of the main blood
outflow channel from the retina ie CRV
In CRAO retina would be pale and you'd see a cherry red macula
In Non-ischaemic - mild defect. There are widespread dot-blot and flame haemorrhages throughout the
fundus and some disc oedema. In Ischaemic - severe visual impairment, the fundus looks similar to the non-
ischaemic picture but disc oedema is more severe.
CRVO: tomato splash, swollen huge optic disc, congested fundus
CRAO: white retina, pale fundus, cherry red spot

1288. A 2yo girl has had a temp=39C, poor appetite, abdominal pain and urinary frequency for 3d.
What is the single most appropriate inv?
a. Catheter specimen of urine for culture
b. Clean catch urine specimen for culture
c. CBC
d. KUB US
e. Supra-pubic aspirate of urine for culture

b. Clean catch urine specimen for culture


protocol...clean catch of urine ,if positive for nitrates do US if negative do msu

Urine collection method


clean catch is preferable
if not possible then urine collection pads should be used
cotton wool balls, gauze and sanitary towels are not suitable
invasive methods such as suprapubic aspiration should only be used if non-invasive methods are not
possible

Management
infants less than 3 months old should be referred immediately to a paediatrician
children aged more than 3 months old with an upper UTI should be considered for admission to
hospital. If not admitted oral antibiotics such as cephalosporin or co-amoxiclav should be given for 7-10
days
children aged more than 3 months old with a lower UTI should be treated with oral antibiotics for 3
days according to local guidelines, usually trimethoprim, nitrofurantoin, cephalosporin or amoxicillin.
Parents should be asked to bring the children back if they remain unwell after 24-48 hours
antibiotic prophylaxis is not given after the first UTI but should be considered with recurrent UTIs

1289. A child with T1DM who is not compliant with meds and eats a lot. He thinks that he is short in his class.
He is not happy. What would you do next?
a. Refer to psychologist
b. Refer to pediatrician
c. Refer to GP
d. Refer to social services
e. Change type of insulin

a. Refer to psychologist
patient is non-compliant... he is insecure and unhappy...so A

1290. An 8yo boy with a BMI=28 was admitted to a surgical ward following a MVC. He was found to have
glycosuria. When he recovered from his injury the glycosuria resolved. What is the single
most appropriate follow-up inv?
a. Fasting blood glucose conc
b. Glycosylated hemoglobin - HbA1c
c. OGTT
d. Random blood glucose conc
e. Serum cortisol conc

a. Fasting blood glucose conc


to diagnose Diabetes, primordial mode of investigation is fasting blood glucose on different occasions.
FBS to differentiate btw Stress-Induced Hyperglycaemia and DM.
1291. At birth, a baby boy at 38wks GA weighs 1.8kgs. He has hepato-splenomegaly and a rash. Blood test
show raised level of bilirubin and liver enzymes. What is the most likely dx?
a. Galactosemia
b. Biliary atresia
c. G6PD deficiency
d. Rh incompatibility
e. Congenital viral infection

e. Congenital viral infection


congenital rubella....blueberry muffin rash with extra medullary hematopoiesis leading to
hepatosplenomegaly, deranged liver enzymes with low birth weight

Congenital infections
The major congenital infections encountered in examinations are rubella, toxoplasmosis and
cytomegalovirus

Cytomegalovirus is the most common congenital infection in the UK. Maternal infection is usually
asymptomatic

Rubella Toxoplasmosis Cytomegalovirus

Characteristic Sensorineural deafness Cerebral calcification Growth retardation


features Congenital cataracts Chorioretinitis Purpuric skin lesions
Congenital heart disease (e.g. patent Hydrocephalus
ductus arteriosus)
Glaucoma

Other features Growth retardation Anaemia Sensorineural


Hepatosplenomegaly Hepatosplenomegaly deafness
Purpuric skin lesions Cerebral palsy Encephalitis/seizures
'Salt and pepper' chorioretinitis Pneumonitis
Microphthalmia Hepatosplenomegaly
Cerebral palsy Anaemia
Jaundice
Cerebral palsy

1292. A 12yo boy with T1DM has poor long-term control. He is unconscious, hyperventilating and
dehydrated. His blood glucose is 28mmol/l. What is the single most imp initial tx?
a. Albumin IV
b. Bicarbonate IV
c. Insulin IV
d. Insulin SC
e. Saline 0.9% IV

e. Saline 0.9% IV
Management
fluid replacement: most patients with DKA are deplete around 5-8 litres. Isotonic saline is used initially
insulin: an intravenous infusion should be started at 0.1 unit/kg/hour. Once blood glucose is < 15
mmol/l an infusion of 5% dextrose should be started
correction of hypokalaemia

1293. A 30yo woman on OCP presents with dilated tortuous veins crossing her abdomen to join the tributaries
to SVC. What is the single most likely cause?
a. Intra-abdominal malignancy
b. Ovarian cyst
c. Fibroids
d. Ascites
e. DVT

a. Intra-abdominal malignancy

1294. An 84yo woman with drusen and yellow spots in the center of retina. What is the single most likely dx?
a. Macular degeneration
b. HTN retinopathy
c. MS
d. DM background
e. Proliferative DM retinopathy

a. Macular degeneration
dry (geographic atrophy) macular degeneration: characterised by drusen - yellow round spots in
Bruch's membrane
wet (exudative, neovascular) macular degeneration: characterised by choroidal neovascularisation.
Leakage of serous fluid and blood can subsequently result in a rapid loss of vision. Carries worst
prognosis
Features
reduced visual acuity: 'blurred', 'distorted' vision, central vision is affected first
central scotomas
fundoscopy: drusen, pigmentary changes

1295. A pt presents with headache, blurring of vision and acuity loss. On fundoscopy, dots and blots
were noted with huge red swollen optic disc. What is the most probable dx?
a. CRAO
b. Branch RAO
c. CRVO
d. Optic atrophy
c. CRVO

Branch RVO: unilateral, painless blurred vision, metamorphopsia (image distortion) a field defect.
Fundoscopy will reveal vascular dilatation and tortuosity of the affected vessels, with associated
haemorrhages in that area only (look for an arc of haemorrhages, like a trail left behind a cartoon image of a
shooting star).

Retinal vein occlusion is one of the most common causes of sudden painless unilateral loss of vision. Loss of
vision is usually secondary to macular oedema.

Central retinal vein occlusion


incidence increases with age, more common than arterial occlusion
causes: glaucoma, polycythaemia, hypertension
severe retinal haemorrhages are usually seen on fundoscopy

1296. A 64yo DM has come for a routine eye check up. Fundoscopy: new vessels all over the retina. What is
the most appropriate management?
a. Strict sugar control
b. Regular eye check ups
c. Non urgent referral to specialist
d. Laser photocoagulation
e. Insulin

d. Laser photocoagulation
This has been the mainstay of treatment for a period of 25 years: the aim is to induce regression of new blood
vessels and reduce central macular thickening.

Traditional classification New classification

Background retinopathy Mild NPDR


microaneurysms (dots) 1 or more microaneurysm
blot haemorrhages (<=3)
hard exudates Moderate NPDR
microaneurysms
Pre-proliferative retinopathy blot haemorrhages
cotton wool spots (soft exudates; hard exudates
ischaemic nerve fibres) cotton wool spots, venous beading/looping and
> 3 blot haemorrhages intraretinal microvascular abnormalities (IRMA) less
venous beading/looping severe than in severe NPDR
deep/dark cluster haemorrhages
more common in Type I DM, treat Severe NPDR
with laser photocoagulation blot haemorrhages and microaneurysms in 4 quadrants
venous beading in at least 2 quadrants
IRMA in at least 1 quadrant

1297. A 25yo primigravida of 8wk GA presents with severe lower abdominal pain, vaginal bleeding and
passage of clots. The internal os is open. What is the most likely dx?
a. Appendicitis
b. Placental abruption
c. Ectopic pregnancy
d. Abortion

d. Abortion
Threatened miscarriage: mild symptoms of bleeding. Usually little or no pain. The cervical os is closed.
Inevitable miscarriage: usually presents with heavy bleeding with clots and pain. The cervical os is
open. The pregnancy will not continue and will proceed to incomplete or complete miscarriage.
Incomplete miscarriage: this occurs when the products of conception are partially expelled. Many
incomplete miscarriages can be unrecognised missed miscarriages.
Missed miscarriage: the fetus is dead but retained. The uterus is small for dates. A pregnancy test can
remain positive for several days. It presents with a history of threatened miscarriage and persistent,
dirty brown discharge. Early pregnancy symptoms may have decreased or gone.
1298. A man developed intense pain after using the end of a pencil to scratch his inner ear. He took out the
pencil from his ear and realized the end of the pencil with the rubber part is still stuck in his ear. What is the
most appropriate management?
a. Remove with a hook
b. Instill olive oil
c. Remove GA
d. Remove with magnet instrument
e. Do syringing

a. Remove with a hook


Insects should be killed prior to removal, using 2% lidocaine.
Remove batteries or magnets as soon as possible to prevent corrosion or burns. Do not crush a battery
during removal.
Adhesives (eg, Super Glue) may be removed manually within 1-2 days once desquamation has
occurred.

Methods for removal


Forceps or hook: grasp the object with forceps, or place a hook behind the object and pull it out.
Irrigation is often effective. Irrigation with water is contra-indicated for soft objects, organic matter or
seeds (which may swell and increase the level of pain and difficulty to remove if exposed to water).
Suction with a small catheter held in contact with the object may be effective.

1299. A 16yo boy presents with acute pain in the right ear and little bleeding from the same ear. He had been
in a boxing match and had sustained a blow to the ear. There is little amount of blood
in the auditory canal and a small perforation of the eardrum. What is the most appropriate
management?
a. Admission for parental
antibiotics
b. Nasal decongestant
c. Oral amoxicillin
d. OPD review
e. Packing of ear
f. Surgical intervention
g. Syringing ENT

c. Oral amoxicillin
A torn (perforated) eardrum will usually heal by itself within 6-8 weeks. It is a skin-like structure and, like skin
that is cut, it will usually heal. In some cases, a doctor may prescribe antibiotic medicines if there is an
infection or risk of infection developing in the middle ear whilst the eardrum is healing.
Management
no treatment is needed in the majority of cases as the tympanic membrane will usually heal after 6-8
weeks. It is advisable to avoid getting water in the ear during this time
it is common practice to prescribe antibiotics to perforations which occur following an episode of acute
otitis media. NICE support this approach in the 2008 Respiratory tract infection guidelines
myringoplasty may be performed if the tympanic membrane does not heal by itself
1300. A 45yo man has noticed difficulty hearing on the telephone. He is concerned because his father has
been moderately hard of hearing since middle age. BC=normal. An audiogram shows
moderate hearing loss in both ears across all frequencies. What is the single most likely dx?
a. Acoustic neuroma
b. Menieres disease
c. Noise induced deafness
d. Otosclerosis
e. Presbyacusis

d. Otosclerosis
Presbycusis is after age of 60 due to degenerative changes..... It's otosclerosis cox it has strong family history in
about 50 to 60 percent of cases....secondly it's ...bilateral ....negative rinne's test.
autosomal dominant
Age - it usually presents between teen years and middle age (typically between the ages of 15 and 35).
However, the average age for having surgery is rising
Positive family history - there is approximately 1/4 risk if one parent is affected; 1/2 risk if both parents are
affected.
Patients may have low-volume speech (enhanced bone conduction leads to the perception of their own
speech as 'loud').
Schwartze's sign - reddish-blue discolouration over promontory and oval window niche, due to vascular
hyperaemia of immature abnormal bone.
Tuning fork tests (Rinne's and Weber's tests) reveal conductive pattern deafness in the majority of
cases.
Audiometry is the primary investigation of choice. Bone and air conduction must be tested and typically reveal
a purely conductive

PRESBYCUSIS
Problems are often first noted in noisy environments; there is usually a slow, insidious onset of
symptoms with gradual progression.
The ability to understand speech is often the earliest symptom as high-frequency hearing loss
predominates. It may be the patient's friends/relatives who note the problem, rather than the patient.
When assessing elderly patients with depression or cognitive impairment, consider hearing loss as a
cause of the symptoms. Tinnitus may be a feature of presbyacusis when the hearing impairment
becomes marked.

ACOUSTIC NEUROMA
Unilateral hearing loss or tinnitus
Impaired facial sensation
Balance problems
1301. The biological parents of a child with CF come to you to know about the chances of their future
children with the same disease. What would you say to them?
a. There is a 1:4 chance that your future child will have this disease
b. All their unaffected children will be carriers of CF
c. Nothing can be predicted
d. It can 100% dx antenatally
Cystic Fibrosis is an autosomal recessive disorder.
When two people who carry the cystic fibrosis gene have a child, there is a:
1 in 4 chance that the child will have cystic fibrosis (by inheriting the cystic fibrosis gene from both parents).

2 in 4 chance that the child will not have cystic fibrosis but will be a carrier (by inheriting a cystic fibrosis gene
from one parent but the normal gene from the other parent).

1 in 4 chance that the child will not have cystic fibrosis and will not be a carrier (by inheriting the normal
gene from both parents).

1302. A 14yo boy presents with recurrent abdominal pain, malaise and weight loss over 6m. Exam:
vague mass is felt in RIF. Colonoscopy shows transmural inflammation and granulomata. What is
the most appropriate management?
a. Sulfasalazine
b. Paracetamol
c. Metronidazole
d. Ibuprofen
Colonoscopy indicate colonic involvement in this case of Crohns ( clinchers here are transmural inflammation
and granulomata ). Clinical experience suggests a modest benefit of metronidazole (10 or 20 mg/kg/day) or
the combination of metronidazole and ciprofloxacin for primary or adjunctive therapy of colonic Crohn's
disease, but not for isolated small intestinal disease.it is also to warn that Metronidazole in particular can be
associated with permanent peripheral neuropathy when used for prolonged periods, especially at higher
doses.
Prednisolone is the drug of first choice for Crohns disease.
1303. A 62yo prv shipyard worker complains of breathlessness and chest pain for 6m. He has now developed a
large pleural effusion. Which is the single best diagnostic inv?
a. ABG
b. Bronchoscopy
c. CXR
d. Pleural biopsy
e. Transfer factor

ans. Pleural biopsy


Dx Malignant mesothelioma
Malignant mesothelioma should be considered in patients with history of asbestos exposure and presenting
with pleural effusion+chest pain .
Mesothelioma is an uncommon type of cancer that occurs in the tissues covering the lungs or the abdomen.
Past exposure to asbestos is a risk factor for mesothelioma. Initial symptoms are variable but can include
shortness of breath, chest pain or abdominal swelling, cough, hoarse voice.
1304. A 67yo man presents with a hx of increasing confusion and drowsiness. He had a fall 2wk ago. CT head
reveals a chronic subdural hematoma. What is the best management for this pt?
a. Craniotomy
b. Burr hole drainage
c. Conservative management
d. Excision and biopsy
B. Burr hole craniotomy.
1305. A 45yo male with epigastric discomfort has been given triple therapy. He has now returned after 4wks
of epigastric discomfort. What inv would you do for him?
a. ECG
b. H.pylori breath test
c. Endoscopy and biopsy
d. US
As the pt is below 55 yrs old, Urea breath test should be done following a 4wks triple therapy.. If H.pylori has
not been eradicated, we give triple regime for one more month, and if it's normal (h pylori eradicated) then
proceed with endoscopy. Had his age been more than 55yrs, we would have directly gone for endoscopy.
OHCM 243
1306. A 13yo boy with umbilical pain for the last 12h presents with anorexia, nausea and has not passed a
bowel motion 24h. What is your dx?
a. Acute appendicitis
b. IBD
c. IBS
d. Meckels diverticulum
e. Muscle strain
f. Ovarian cysts
g. PID
h. Psoas hematoma
i. Pyelonephritis
j. Uretric calculus
k.
its MD because it also presents with symptoms like that of Appendicitis except rebound tenderness.
Moreover, had it been AA, there would have been hx of rebound tenderness and perhaps pain in RIF.
MD presents in 10-43% of symptomatic patients. The frequency of complications of Meckel's diverticulum
varies widely in the literature. Studies varyingly report intestinal obstruction or haemorrhage as the most
common complication in adults.
The presenting symptoms are usually abdominal pain, vomiting and constipation. Various mechanisms
produce the obstruction, including a fibrotic band attaching the diverticula to the abdominal wall causing a
volvulus of the small bowel and intussusception in which the diverticulum is the lead point. An intussusception
may present with redcurrant jelly stools or a palpable lump in the lower abdomen.Meckel's diverticulum
should always be considered in the differential diagnosis of patients presenting with rectal bleeding or
intestinal obstruction.( patient.co)
MECKELS' Presentation
Asymptomatic
Meckel's diverticulum is a common incidental finding at laparotomy. The vast majority of those with Meckel's
diverticulum are asymptomatic. Complications are most likely to occur when the diverticulum contains
heterotypic tissue. This is most often gastric, but may also be pancreatic, jejunal or colonic mucosa. The
lifetime risk of developing a complication that requires surgery is thought to be 4-6%.
Haemorrhage
This accounts for 25-50% of all complications. It is more common in children younger than 2 years (in which
age group it is the most common complication) and in males. The patient usually reports bright red blood in
the stools. The amount may vary from minimal recurrent episodes to a large shock-producing haemorrhage.
Meckel's diverticulum should always be excluded in a child presenting with massive painless rectal bleeding.
The blood may be bright red if the bleeding is brisk, or darker if it is milder and transit time is slow. Melaena-
like tarry stool may also be seen if gastric tissue present in the diverticulum ulcerates, or if it produces acid
which causes damage to the adjacent ileal mucosa.
APPENDICITIS is most common between the ages of 10 and 20 years but can occur at any age.
Classic symptoms often do not appear in young children, in pregnant women and in the elderly and the
diagnosis is particularly easy to miss in these age groups. The classical presentation consists of:
Pain:
Early periumbilical pain moves, after hours or sometimes days, to the right iliac fossa (RIF) as the peritoneum
becomes involved. Pain which wakes the patient or keeps a child awake is significant.
Movement and coughing aggravate the pain. The patient may lie still with shallow breathing. Deep breathing
and coughing hurt.
Nausea, vomiting, anorexia. The patient is usually constipated or simply does not want to have the bowels
open, but may have diarrhoea. Rapidly progressive cases may have recurrent vomiting without fever and
diarrhoea. This may be marked in post-ileal appendix (which is rare).

1307. A 46yo man with tachycardia has the following ECG. What is the most likely dx?
a. SVT
b. VT
c. Mobitz I heart block
d. Atrial fibrillation
e. WPW syndrome

1308. A 24yo male is admitted with acute seere asthma. Tx is initiated with 100% oxygen, nebulized
salbutamol and ipratropium bromide nebulizers and IV hydrocortisone. Despite initial tx there is
no improvement. Which is the next step in management?
a. IV aminophylline
b. IV magnesium sulphate
c. IV salbutamol
d. IM adrenaline
e. IV adrenaline
British Thoracic Society guidelines:
magnesium sulphate recommended as next step for patients who are not responding (e.g. 1.2 - 2g IV over 20
mins)
little evidence to support use of IV aminophylline (although still mentioned in management plans)
if no response consider IV salbutamol
1309. A 49yo man first presented with increasing difficulty in swallowing. Several months later he developed
weakness in his right foot. Now he can no longer feed himself, he chokes on food and has become confined to
a wheelchair. What is the most likely dx?
a. Cerebral tumor
b. Myasthenia gravis
c. Lambert-Eaton syndrome
d. Motor neuron disease
e. Cerebro-vascular disease
In MND, motor nerves become damaged and eventually stop working. Therefore, the muscles that the
damaged nerves supply gradually lose their strength. There are various subtypes of MND. In each type,
symptoms tend to start in different ways. However, as the disease progresses, the symptoms of each type of
MND tend to overlap. This means that symptoms in the later stages of each type of MND become similar. The
main types of MND are:
Amyotrophic lateral sclerosis (ALS). This is the classical MND and the most common type. About 8 in 10
people with MND have this type. Symptoms tend to start in the hands and feet. The muscles tend to become
stiff as well as weak at first.
Progressive bulbar palsy (PBP). About 2 in 10 people with MND have this type. The muscles first affected are
those used for talking, chewing and swallowing (the bulbar muscles).
Progressive muscular atrophy (PMA). This is an uncommon form of MND. The small muscles of the hands and
feet are usually first affected but the muscles are not stiff.
Primary lateral sclerosis (PLS). This is a rare type of MND. It mainly causes weakness in the leg muscles. Some
people with this type may also develop clumsiness in the hands or develop speech problems.
1310. A 10yo boy with lower abdominal pain for the last 10d presents with a hx of passing 6-8 loose stools.
Temp=38.8C. He is tender in the right lower quadrant and has an anal fistula. Choose the single most likely
cause of abdominal pain.
a. IBD
b. IBS
c. Pyelonephritis
d. Uretric calculus
e. Gastroenteritis
Anal fistula is the distinctive feature here only seen in Crohns (IBD)
Symptoms of Crohns Disease
Diarrhoea
Weight loss
Ulcers.
Generally feeling unwell,
Anaemia
Mouth ulcers
Anal fissures
Tenesmus
Symptoms of IBS include :
Pain and discomfort
Bloating
Changes in stools:
:Some people have bouts of diarrhoea, and some have bouts of constipation.
Some people have bouts of diarrhoea that alternate with bouts of constipation.
Other symptoms are:
o Feeling sick (nausea).
o Headache.
o Belching.
o Poor appetite.
o Tiredness.
o Backache.
o Muscle pains.
o Feeling quickly full after eating.
o Heartburn.
o Bladder symptoms (an associated irritable bladder).
1311. A 28yo woman with hx of drug addiction wants to start a family and have a baby. She would like to stop
taking heroin and asked for something to help her stay away from it. What drug tx would you give her?
a. Naloxone
b. Acamprosate
c. Methadone
d. Chlordiazepoxide
e. Naltrexone
C
Treatment is different if a pt comes with opioid intoxication (NALOXONE should be given ), and if someone
comes and wants to leave opioids i.e, detoxification (Methadone or buprenorphene would be the choice then
) Chlordiazepoxide and Acamprosate are used for alcoholism. Vit B Complex and Chlordiazepoxide for alcohol
withdrawal and acamprosate for dependence. Oxazepam is the drug of choice for alcohol detoxification in pts
with liver disease as its not metabolized by liver.
Treatment of opioid intoxication
If a patient has collapsed and is thought to be acutely intoxicated, call 999/112/911 and
refer urgently to hospital.
Naloxone (a pure opioid antagonist used for reversing opioid intoxication) has a rapid onset
of action and can be given IM, IV or subcutaneously.
Therapy is otherwise mostly supportive, eg maintain airway, ventilation if necessary and IV
fluids.
Tx of detoxification
Methadone, and buprenorphine are equally effective in detoxification regimens. The place of
lofexidine in detoxification programmes requires further research.[4]
Opioid detoxification should be offered as part of a package including preparation and post-
detoxification support to prevent relapse.
Psychosocial interventions (eg talking therapies, cognitive behavioural therapy, family therapy) and
keyworking should be delivered alongside pharmacological interventions.[2]
If detoxification is unsuccessful, patients should have access back into maintenance and other
treatment.

1312. A pt with vesicles in the maxillary division of trigeminal nerve. Which area of mucus membrane will be
involved?
a. Palate
b. Cheek
c. Cornea
d. Conjunctiva

The ophthalmic nerve (V1) carries sensory information from the scalp and forehead, the upper eyelid, the
conjunctiva and cornea of the eye, the nose (including the tip of the nose, except alae nasi), the nasal mucosa,
the frontal sinuses and parts of the meninges (the dura and blood vessels). The maxillary nerve (V2) carries
sensory information from the lower eyelid and cheek, the nares and upper lip, the upper teeth and gums, the
nasal mucosa, the palate and roof of the pharynx, the maxillary, ethmoid and sphenoid sinuses and parts of
the meninges. The mandibular nerve (V3) carries sensory information from the lower lip, the lower teeth and
gums, the chin and jaw (except the angle of the jaw, which is supplied by C2-C3), parts of the external ear and
parts of the meninges. The mandibular nerve carries touch-position and pain-temperature sensations from the
mouth. Although it does not carry taste sensation (the chorda tympani is responsible for taste), one of its
branchesthe lingual nervecarries sensation from the tongue.
for mucous membrane its palate for Maxillary... if they had not mentioned mucous then mid face externally is
supplied by Maxillary. Mandibular is division that gives buccal branch and supplies the buccal mucosa with
sensory fibers...
Ans should be palate as question is being asked about the mucous membrane.
1313. A 52yo man presents with visual hallucinations and features of cognitive impairment. What is
the most likely dx?
a. Frontotemporal dementia
b. Lewy body dementia
c. Delirium tremens
d. Alzheimers disease
e. Huntingtons disease
Dementia is a syndrome caused by a number of brain disorders which cause memory loss, decline in some
other aspect of cognition, and difficulties with activities of daily living.
The symptoms fall into three groups:
Cognitive impairment: causing difficulties with memory, language, attention, thinking, orientation,
calculation, and problem-solving.
Psychiatric or behavioural disturbances: changes in personality, emotional control, and social
behaviour; depression, agitation, hallucinations, and delusions.
Difficulties with activities of daily living, such as driving, shopping, eating, and dressing.
There are subtle differences in the presentation of different types of dementia. Alzheimer's disease tends to
have an insidious onset, whereas vascular dementia typically has a series of stepwise increases in symptom
severity. DLB may present with fluctuating levels of consciousness, hallucinations, sleep disorders, falls and
Parkinsonian features. In Parkinson's disease dementia, the Parkinsonian features predate the dementia by a
significant amount of time. In frontotemporal dementia, behavioural changes (such as disinhibition or apathy)
and language disturbances are often presenting features. It may be important to determine the type of
dementia - in DLB, for example, making this diagnosis will have important implications for treatment (use of
neuroleptics is avoided, as motor and mental impairment is worsened and mortality may be increased).
Ans should be Lewy Body dementia bcoz of presence of hallucinations . Rest of the types are explained above
and DT is a psychotic condition typical of withdrawal in chronic alcoholics, involving tremors, hallucinations,
anxiety, and disorientation.
1314. A 40yo woman who has recently returned from working in the middle east complains of thirst, episode
of loin pain, urinary frequency, dysuria and has passed a urinary stone. All inv are normal. She plans to return
to the Middle East in a months time. What is the single best advice
to prevent recurrent stone formation?
a. Drink less milk
b. High fibre diet
c. Increase fluid intake
d. Low calcium diet
e. Low protein diet
The cause in this scenario seems to be dehydration as the pt lives in middle east. She should increase her fluid
intake in order to maintain adequate urinary flow by preventing urinary stasis.
Several risk factors are recognised to increase the potential of a susceptible individual to develop stones.
These include:
Anatomical anomalies in the kidneys and/or urinary tract - eg, horseshoe kidney, ureteral stricture.
Family history of renal stones.
Hypertension.
Gout.
Hyperparathyroidism.
Immobilisation.
Relative dehydration.
Metabolic disorders which increase excretion of solutes - eg, chronic metabolic acidosis, hypercalciuria,
hyperuricosuria.
Deficiency of citrate in the urine.
Cystinuria (an autosomal-recessive aminoaciduria).
Drugs - eg, diuretics such as triamterene and calcium/vitamin D supplements.
More common occurrence in hot climates.
Increased risk of stones in higher socio-economic groups.
Contamination - as demonstrated by a spate of melamine-contaminated infant milk formula

1315. A 32yo man presents with 3d of scrotal pain. Exam: thickening o the left testis and it is hot to touch.
What is the most appropriate management?
a. Analgesia
b. Reassurance
c. Antibiotics
d. Referral to surgeon
Epididymo-orchitis
Causes :
In men under 35 years old, infection is most often due to a sexually transmitted pathogen - eg,
Chlamydia trachomatis and Neisseria gonorrhoeae.
In men over 35 years old, infection is most often due to a non-sexually transmitted Gram-negative
enteric organism causing urinary tract infections - eg, Escherichia coli,Pseudomonas spp. Specific risk
factors include recent instrumentation or catheterisation.
Signs n symptoms :
It usually presents with unilateral scrotal pain and swelling of relatively acute onset.
Acute epididymitis is usually unilateral but is bilateral in 5-10% of the patients.
Tenderness to palpation on the affected side.
Palpable swelling of the epididymis, starting with the tail at the lower pole of the testis and spreading
towards the head at the upper pole of the testis with or without involvement of the testicle.
There may also be urethral discharge, secondary hydrocele, erythema and/or oedema of the scrotum
on the affected side and pyrexia.
Clincher here for making the diagnosis is the unilateral tenderness and the hot temperature.
Differential would be testicular torsion , but that normally occurs in men who are younger than 20 years
though it can occur at any age.
A painful swollen testicle in an adolescent boy or a young man should be managed as torsion until proven
otherwise.
Torsion is more likely if the onset of pain is acute (typically around four hours at presentation) and the
pain is severe.
Empirical therapy should be given to all patients with epididymo-orchitis before culture/NAAT results are
available. The antibiotic regimen chosen should be determined in the light of the immediate tests (urethral or
FPU smear, urinalysis) as well as age, sexual history including insertive anal intercourse, any recent
instrumentation or catheterisation and any known urinary tract abnormalities. So, answer should be
ANTIBIOTICS .
1316. A 34yo woman presents with truncal obesity, easy bruising, hyperglycemia, high BP and depression.
Which of the following invs will be most helpful in localizing the cause for Cushings syndrome?
a. Serum cortisol
b. 24h urinary cortisol
c. Low dose dexamethasone suppression test
d. High dose dexamethasone suppression test
e. Overnight dexamethasone suppression test

Ans : High dose Dexamethasone suppression test

Investigations to confirm the presence of Cushing's syndrome :


The recommended diagnostic tests for the presence of Cushing's syndrome are 24-hour urinary free cortisol, 1
mg overnight dexamethasone suppression test and late-night salivary cortisol

Once we know the diagnosis , following tests help us locate the position of abnormality

Investigations to identify the cause of Cushing's syndrome :


Plasma ACTH
Secretion is pulsatile and shows a diurnal variation, with the plasma concentrations highest at 8 am
and lowest at midnight. The secretion of ACTH is increased by stress.
An undetectable plasma ACTH with an elevated serum cortisol level is diagnostic of ACTH-independent
Cushing's syndrome, which is due to a primary cortisol-producing adrenal adenoma or carcinoma, or
exogenous glucocorticoid use. This should then be followed by an abdominal CT or MRI scan if
exogenous glucocorticoids are excluded as the cause.
An elevated ACTH level is consistent with ACTH-dependent Cushing's syndrome.
If the plasma ACTH level is detectable then the following tests are indicated:
o High-dose dexamethasone suppression test:
The 8 mg overnight dexamethasone suppression test and the 48-hour high-dose
dexamethasone test may be useful when baseline ACTH levels are equivocal. They
also help in determining whether a patient has pituitary or ectopic ACTH production.
Greater than 90% reduction in basal urinary free cortisol levels supports the diagnosis
of a pituitary adenoma; ectopic ACTH causes lesser degrees of suppression.
o Inferior petrosal sinus sampling (IPS):
Performed with CRH stimulation to aid in determining the source of excess ACTH.
A baseline and stimulated ratio of IPS to peripheral ACTH of less than 1.8 is suggestive of
ectopic ACTH, while a ratio of IPS to peripheral ACTH of greater than 2 is consistent with
a pituitary adenoma.
o MRI of the pituitary.
o Chest and abdominal CT scans: for patients with suspected adrenal tumours or ectopic ACTH.
o Plasma CRH: ectopic CRH production is a very rare cause of Cushing's syndrome.
1317. A 32yo man develops hematuria 2wks after a sore throat. What is the dx?
a. Post infection nephritis
b. IgA nephropathy
c. Membranous nephritis
d. Glomerulonephritis
Given key is d, but that is a broad term. One should always go for a specific answer. We are now left with
Post infection Nephritis and igA nephropathy as they both follow a throat infection. In, igA nephropathy,
symptoms appear 1-3 days after the infection, whereas in Post infection, symptoms appear 1-12 weeks after
the sore throat. So, our answer here should be A.
1318. An elder man who has anorexia, prostate symptoms and HTN. There are small kidneys on US. What is
the dx?
a. Hypertensive renal disease
b. Prostate ca
c. BPH
Only Hypertensive nephropathy leads to shrunken kidneys. Prostate symptoms are not explained by
Hypertension nephropathy unless there is an outflow obstruction and the other two options can be the
cause. Should go with answer A. Needs to be discussed though. NOTE : Kidney size is increased in the case of
diabetic nephropathy.
1319. A 55yo woman with breast ca which has spread to lung, liver and bone now presents with increasing
constipation, weakness, thirst and anorexia for the past 3d. Her only medication is haloperidol for hiccoughs.
Today she is disorientated and has left sided weakness. What is the most likely dx?
a. Brain mets
b. Hypercalcemia
c. Liver failure
Pt has all the features of Hyepercalcemia except the left sided weakness. But, Brain mets can only cause
localized weakness . So., the ans would be A.
1320. A 22yo man presents with a red, hot, swollen, metatarsal phalangeal joint, sarcoilitis and
onycholysis. What is the single most likely cause of his condition?
a. Gout
b. RA
c. Reiters syndrome
d. Psoriatic arthropathy
Sacroilitis and nail changes are the clnchers here for PA.
Psoriatic arthropathy correlates poorly with cutaneous psoriasis and often precedes the development of skin
lesions. Around 10-20% percent of patients with skin lesions develop an arthropathy with males and females
being equally affected

Types*

rheumatoid-like polyarthritis: (30-40%, most common type)


asymmetrical oligoarthritis: typically affects hands and feet (20-30%)
sacroilitis
DIP joint disease (10%)
arthritis mutilans (severe deformity fingers/hand, 'telescoping fingers')

Management

treat as rheumatoid arthritis


but better prognossis
1321. Which of the following conditions requires operative management?
a. Cellulitis
b. Dyshidrosis
c. Erysipelas
d. Fourniers gangrene
e. Lymphangitis
Fourniers gangrene is a medical emergency that is difficult to recognise in the early stages
It can be classified according to the causative organism:

type 1 is caused by mixed anaerobes and aerobes (often occurs post-surgery in diabetics)
type 2 is caused by Streptococcus pyogenes

Features

acute onset
painful, erythematous lesion develops
extremely tender over infected tissue

Management

urgent surgical referral debridement


intravenous antibiotics
1322. A 55yo pt presents with collapse and complains of abdominal pain that radiates to the back. An
expansile abdominal mass if felt on examination and the pt is in shock. What is the single most likely dx?
a. Ruptured aortic aneurysm
b. Renal colic
c. Trauma
d. Endocarditis
e. Atheroma
Expansile mass and radiating towards the back are the clinchers here. Ans is RAA. Has already been discussed
in detail in mcq 76.
1323. A house-bound 78yo man with severe COPD has had a gradual deterioration over recent months and is
now breathless at rest. He is on maximal inhaled medical therapy. Result: pH=7.36, PaCO2=5.9kPa,
PaO2=6.9kPa. What is the single most appropriate additional tx?
a. Aminophylline PO
b. ACEi PO
c. Antibiotic PO
d. Oxygen
e. Steroid PO
The 2010 NICE guidelines on COPD clearly define which patients should be assessed for and offered long-term
oxygen therapy (LTOT). Patients who receive LTOT should breathe supplementary oxygen for at least 15 hours
a day. Oxygen concentrators are used to provide a fixed supply for LTOT.

Assess patients if any of the following:

very severe airflow obstruction (FEV1 < 30% predicted). Assessment should be 'considered' for patients
with severe airflow obstruction (FEV1 30-49% predicted)
cyanosis
polycythaemia
peripheral oedema
raised jugular venous pressure
oxygen saturations less than or equal to 92% on room air

Assessment is done by measuring arterial blood gases on 2 occasions at least 3 weeks apart in patients with
stable COPD on optimal management.
Offer LTOT to patients with a pO2 of < 7.3 kPa or to those with a pO2 of 7.3 - 8 kPa and one of the following:

secondary polycythaemia
nocturnal hypoxaemia
peripheral oedema
pulmonary hypertension
1324. A 79yo man has a swelling of the right groin which was clinically dx to be indirect inguinal hernia. What
is the single feature of the hearnia sac that would confirm the dx?
a. Comes through femoral ring
b. Doesnt pass through the deep inguinal ring
c. Lies below and lateral to the pubic tubercle
d. Only passes through the superficial inguinal ring
e. Passes through the deep inguinal ring
Ans E
There are two types of inguinal hernia, direct and indirect, which are defined by their relationship to the
inferior epigastric vessels. Direct inguinal hernias occur medial to the inferior epigastric vessels when
abdominal contents herniate through a weak spot in the fascia of the posterior wall of the inguinal canal,
which is formed by the transversalis fascia. Indirect inguinal hernias occur when abdominal contents protrude
through the deep inguinal ring, lateral to the inferior epigastric vessels; this may be caused by failure of
embryonic closure of the processus vaginalis..
1325. A 56yo woman with hx of breast cancer 10y ago has undergone radical mastectomy and axillary LN
removal, now complains of swollen upper limb 3wks after an insect bite. The bite site is better but gross
edema is still present. What is the cause?
a. Lymphedema
b. Breast Ca
c. Allergy
d. Filariasis
Upper limb lymphatics are drained by the Axillary lymph nodes. So, their removal results in lymphedema.
1326. A homeless person is found wandering on the street. He had ataxic gait, nystagmus and
opthalmoplegia. He looked unkempt and his clothes had a sweaty odour. He had a dry mucous membrane
with a BP=118/70mmHg and PR=90bpm. Blood sugar level=8. Alcohol breath test= -ve. What would the most
imp initial inv?
a. IV insulin
b. Vit B complex
c. Bolus IV 0.9%NS
d. IV dextrose
e. Antibiotics
Ans B
Vit B complex deficiency leads to symptoms :
Extreme tiredness or fatigue.
A lack of energy or lethargy.
Being out of breath.
Feeling faint.
Headache.
Ringing in the ears (tinnitus)
Lack of appetite.
Peripheral neuropathy
confusion, anxiety, paranoia
1327. A 34yo man has supra-orbital pain and tenderness and developed tenderness over the maxilla. He also
has mild fever. What is the single likely cause for these symptoms?
a. Acute sinusitis
b. GCA
c. Trigeminal neuralgia
d. Maxillary carcinoma
Ans is A
Sinusitis describes an inflammation of the mucous membranes of the paranasal sinuses. The sinuses are
usually sterile - the most common infectious agents seen in acute sinusitis are Streptococcus pneumoniae,
Haemophilus influenzae and rhinoviruses.

Predisposing factors include:

nasal obstruction e.g. Septal deviation or nasal polyps


recent local infection e.g. Rhinitis or dental extraction
swimming/diving
smoking

Features

facial pain: typically frontal pressure pain which is worse on bending forward
nasal discharge: usually thick and purulent
nasal obstruction: e.g. 'mouth breathing'
post-nasal drip: may produce chronic cough

Management of acute sinusitis

analgesia
intranasal decongestants
oral antibiotics are not normally required but may be given for severe presentations. Amoxicillin is
currently first-line
1328. A 51yo woman presents with painful tongue and complains of tiredness. She is pale and has angular
stomatitis and a smooth red tongue. There is no koilonychea. Choose the single cell type you will find on the
blood film.
a. Numerous blast cells
b. Oval macrocytes
c. Spherocytes
d. Microcytic hypochromic
e. Mexican hat cells
f. Erythrocytes
These are the features of vit. b12 and folic acid deficiency. Angular stomatitis is the clincher here. Ans would
be B
1329. A 24yo woman presents with tingling and twitching of her fingers followed by throbbing
unilateral headache. What is the most likely dx?
a. Tension headache
b. Migraine
c. Cluster headache
d. TIA
e. SAH
Migraine is a common type of primary headache. It is characterised typically by:

a severe, unilateral, throbbing headache


associated with nausea, photophobia and phonophobia
attacks may last up to 72 hours
patients characteristically go to a darkened, quiet room during an attack
'classic' migraine attacks are precipitated by an aura. These occur in around one-third of migraine
patients
typical aura are visual, progressive, last 5-60 minutes and are characterised by transient hemianopic
disturbance or a spreading scintillating scotoma
formal diagnostic criteria are produced by the International Headache Society (see below)

Epidemiology

3 times more common in women


prevalence in men is around 6%, in women 18%

Common triggers for a migraine attack

tiredness, stress
alcohol
combined oral contraceptive pill
lack of food or dehydration
cheese, chocolate, red wines, citrus fruits
menstruation
bright lights

Migraine diagnostic criteria

A At least 5 attacks fulfilling criteria B-D

B Headache attacks lasting 4-72 hours* (untreated or unsuccessfully treated)

C Headache has at least two of the following characteristics:

1. unilateral location*
2. pulsating quality (i.e., varying with the heartbeat)
3. moderate or severe pain intensity
4. aggravation by or causing avoidance of routine physical activity (e.g., walking or climbing stairs)

D During headache at least one of the following:


1. nausea and/or vomiting*
2. photophobia and phonophobia

E Not attributed to another disorder (history and examination do not suggest a secondary headache
disorder or, if they do, it is ruled out by appropriate investigations or headache attacks do not occur for
the first time in close temporal relation to the other disorder)
1330. A young child dx with chicken pox. Usually goes to day care. What is the most appropriate
advice?
a. Child should be admitted to hospital straight away
b. Isolate the child from parents and siblings at home
c. Advice that he can go back to nursery when the rash is crusted over
Ans is C
Chickenpox is caused by primary infection with varicella zoster virus. Shingles is reactivation of dormant virus
in dorsal root ganglion

Chickenpox is highly infectious

spread via the respiratory route


can be caught from someone with shingles
infectivity = 4 days before rash, until 5 days after the rash first appeared*
incubation period = 10-21 days

Clinical features (tend to be more severe in older children/adults)

fever initially
itchy, rash starting on head/trunk before spreading. Initially macular then papular then vesicular
systemic upset is usually mild

Management is supportive

keep cool, trim nails


calamine lotion
school exclusion: current HPA advice is 5 days from start of skin eruption. They also state
'Traditionally children have been excluded until all lesions are crusted. However, transmission has
never been reported beyond the fifth day of the rash.'
immunocompromised patients and newborns with peripartum exposure should receive varicella zoster
immunoglobulin (VZIG). If chickenpox develops then IV aciclovir should be considered

A common complication is secondary bacterial infection of the lesions. Rare complications include

pneumonia
encephalitis (cerebellar involvement may be seen)
disseminated haemorrhagic chickenpox
arthritis, nephritis and pancreatitis may very rarely be seen
1331. A 7yo boy is brought by his mother. There are multiple perioral and forehead vesicles. Some vesicles are
crusted and some are not. The face is hot. What is the most likely dx?
a. Varicella zoster
b. Herpes zoster
c. Fungal infection
d. Impetigo
e. Psoriasis
Pain and Blisters are in the area of nerve supply. Herpes Zoster seems to be the answer.
Shingles is an acute, unilateral, painful blistering rash caused by reactivation of the Varicella Zoster Virus (VZV).

The 'shingles vaccine'

In 2013 the NHS introduced a vaccine to boost the immunity of elderly people against herpes zoster. Some
important points about the vaccine:

will be offered to patients at the age of 70 years (a catch-up programme will also be launched initially)
is live-attenuated and given sub-cutaneously

As it is a live-attenuated vaccine the main contraindications are immunosuppression.

Side-effects

injection site reactions


less than 1 in 10,000 individuals will develop chickenpox

Management of shingles

Oral aciclovir is first-line. One of the main benefits of treatment is a reduction in the incidence of post-herpetic
neuralgia.
1332. A 5yo boy is rescued from a burning building and is presented to the ED. He has 5% partial
thickness burns over the arms and legs and had soot in the mouth and nose. His breathing has
become noisy. What is the single most immediate management?
a. Nebulized adrenaline
b. Nebulized salmetarol and oxygen
c. Needle cricothyrodotomy
d. Oropharyngeal airway
e. Intubation of airway
Intubation of airway should be done immediately in order to prevent blockage of airways secondary to
laryngeal edema.
1333. A new born bay is borught with pansystolic murmur at sternal border but the baby is not
cyanosed. What is the dx?
a. VSD
b. ASD
c. TOF
d. PDA
Pansystolic murmers are seen in the following :
Holosystolic (pansystolic)

mitral/tricuspid regurgitation (high-pitched and 'blowing' in character)


VSD ('harsh' in character)
VSD:
A ventricular septal defect (VSD) is a defect in the ventricular septum, the wall dividing the left and right

ventricles of the heart.

The ventricular septum consists of an inferior muscular and superior membranous portion and is extensively

innervated with conducting cardiomyocytes.

During ventricular contraction, or systole, some of the blood from the left ventricle leaks into the right
ventricle, passes through the lungs and reenters the left ventricle via the pulmonary veins and left atrium. This
has two net effects. First, the circuitous refluxing of blood causes volume overload on the left ventricle.
Second, because the left ventricle normally has a much higher systolic pressure (~120 mmHg) than the right
ventricle (~20 mmHg), the leakage of blood into the right ventricle therefore elevates right ventricular
pressure and volume, causing pulmonary hypertension with its associated symptoms.
In serious cases, the pulmonary arterial pressure can reach levels that equal the systemic pressure. This
reverses the left to right shunt, so that blood then flows from the right ventricle into the left ventricle,
resulting in cyanosis, as blood is by-passing the lungs for oxygenation.[7]
This effect is more noticeable in patients with larger defects, who may present with breathlessness, poor
feeding and failure to thrive in infancy. Patients with smaller defects may be asymptomatic. Four different
septal defects exist, with perimembranous most common, outlet, atrioventricular, and muscular less
commonly
Inv :
Auscultation
Echo
1334. A woman complaining of diarrhea, abdominal pain and fatigue. All the tests are found to be
normal. What is the cause?
a. Somatization
b. Conversion
c. Hypochondriasis
Somatisation disorder

multiple physical SYMPTOMS present for at least 2 years


patient refuses to accept reassurance or negative test results
1335. A 26yo man has returned from NY to the UK and noticed weight loss, night sweats, temp=37.5C
and cervical lymphadenopathy. He also has splenomegaly. What is the dx?
a. TB
b. Lymphoma
c. Bronchial carcinoma
d. Bronchitis
Night sweats and cervical lymohadenopathy is a clincher here.
Lymphoma
Hodgkin's lymphoma is a malignant proliferation of lymphocytes characterised by the presence of the Reed-
Sternberg cell. It has a bimodal age distributions being most common in the third and seventh decades.

1336. A mother got infected with Hep B during pregnancy. Her child is born and she is worried about
the risk of infection to the baby with Hep B. What would you give to the baby?
a. Hep B Ig only
b. Hep B full vaccine and Ig
c. Hep B vaccine only once
d. Nothing until immune status is checked
e. Hep B vaccine once and Ig
Ans is B
Hepatitis B and pregnancy
Basics

all pregnant women are offered screening for hepatitis B


babies born to mothers who are chronically infected with hepatitis B or to mothers who've had acute
hepatitis B during pregnancy should receive a complete course of vaccination + hepatitis B
immunoglobulin
studies are currently evaluating the role of oral antiviral treatment (e.g. Lamivudine) in the latter part
of pregnancy
there is little evidence to suggest caesarean section reduces vertical transmission rates
hepatitis B cannot be transmitted via breastfeeding (in contrast to HIV)
1337. A man suffers from Herpes Zoster affecting his face. Which of the following mucos membrane is
to be affected?
a. Cheek
b. Cornea
c. Conjunctiva
d. Oropharynx
e. Palate
This one is controversial. Tried to find out on Facebook. Most of the people agreed on conjunctiva as cornea is
not a mucous membrane.
1338. A 34yo man sustains a fx to shaft of femur after falling from the roof of his house. Exam: distal
pulses are not palpable. Which vessel is damaged?
a. Femoral artery
b. Circumflex femoral artery
c. Profundafemoris artery
d. Popliteal artery
e. Obturator artery
f. Dorsalispedis artery
Femoral artery is the main artery and seems to be damaged as distal pulses are absent.

Shaft of femur fractures


These are caused by a high-energy injury, such as road traffic accidents, unless pathological fracture

in a patient with osteoporosis or metastatic disease.

There are often associated injuries to the hip, pelvis, knee and other parts of the body.

Diagnosis
Deformity, shortening, external rotation and abduction at the hip on the affected side.
Management
Initial management:

Assess vital functions and any associated chest, head, abdominal or spinal injuries. Resuscitate

and treat life-threatening injuries as necessary.

Splint fractures (Thomas' splint or equivalent traction splint).

X-rays of the femur.

Blood tests, including blood for cross-matching.

Obtain intravenous access and start fluid replacement.

Peripheral sensation and pulses should be closely monitored.

Analgesia: adequate intravenous analgesia. Femoral nerve block is usually effective.

Further management

Intramedullary nailing is used for treating fractures of the femoral shaft. [10]

Early immobilisation and treatment reduce the risk of complications. NICE recommends

physiotherapy assessment and, unless medically or surgically contra-indicated, mobilisation on the

day after surgery. Patients should be offered mobilisation at least once a day and regular

physiotherapy review.[1]

Complications
Closed fractures may be associated with a large volume of blood loss before becoming obvious with

swelling of the thigh.

Later complications include fat embolism, deep vein thrombosis, pulmonary embolism, infection,

shortening, angulation and nonunion.

1339. A 9yo child doesnt play with his peers and has collected 200 cars. He doesnt respond to any criticism.
What is the dx?
a. Autism
b. Personality disorder
c. Schizophrenia
d. Rett syndrome
e. Social anxiety
Epidemiology
75% of children are male
usually develops before 3 years of age

All 3 of the following features must be present for a diagnosis to be made

global impairment of language and communication


impairment of social relationships
ritualistic and compulsive phenomena

Other features

most children have a decreased IQ - the 'idiot savant' is rare

Associated conditions

Fragile X
Rett's syndrome
1340. A 63 yo man with vague but persistent pain. On endoscopy: columnar epithelium was found to be
pouched into muscularis. What is the dx?
a. Adenocarcinoma
b. Adenoma
c. Peptic ulcer
d. H. pylori infection
Adenocarcinoma arises from glandular cells present in the lower third of the esophagus, often where they
have already transformed to intestinal cell type (a condition known as Barrett's esophagus).
Adenocarcinoma is now the most common type of oesophageal cancer and is more likely to develop in
patients with a history of gastro-oesophageal reflux disease (GORD) or Barrett's.

The majority of tumours are in the middle third of the oesophagus.

Risk factors

smoking
alcohol
GORD
Barrett's oesophagus
achalasia
Plummer-Vinson syndrome
rare: coeliac disease, scleroderma
1341. A 24yo man after a head injury presents with difficulty dressing himself, difficulty in writing and inability
to differentiate the fingers of his hand. Which part of the brain is most likely to be affected?
a. Frontal lobe
b. Parietal lobe
c. Temporal lobe
d. Occipital lobe
e. Brainstem
The parietal lobe is one of the four major divisions of the cerebral cortex. This lobe receives and processes
sensory information from the body.
Functions of major lobes are :
1. Frontal lobeconscious thought; damage can result in mood changes, social differences, etc. The

frontal lobes are the most uniquely human of all the brain structures.

2. Parietal lobeplays important roles in integrating sensory information from various senses, and in

the manipulation of objects; portions of the parietal lobe are involved with visuospatial processing

3. Occipital lobesense of sight; lesions can produce hallucinations

4. Temporal lobesenses of smell and sound, as well as processing of complex stimuli like faces and

scenes.

5. Limbic lobeemotion, memory

6. Insular cortexpain, some other senses.

1342. A 16yo boy in boarding school feels unwell. He developed cough and rash .His CXR showed bilateral
consolidations. What is the cause of his symptoms?
a. Staph aureus
b. Legionella
c. Mycoplasma
d. Streptococcus
Homeless shelters ,Young army recruits living in barracks ,young students in dormitories(in boarding schools) -
------>mycoplasma,
In legionella questions u ll mostly find some water related hints or outdoor activity near water or air
conditioning system mention and most of the times GI symptoms along with respiratory symptoms
Mycoplasma pneumoniae is a cause of atypical pneumonia which often affects younger patients. It is
associated with a number of characteristic complications such as erythema multiforme and cold autoimmune
haemolytic anaemia. Epidemics ofMycoplasma pneumoniae classically occur every 4 years. It is important to
recognise atypical pneumonias as they may not respond to penicillins or cephalosporins due to it lacking a
peptidoglycan cell wall.

Features

the disease typically has a prolonged and gradual onset


flu-like symptoms classically precede a dry cough
bilateral consolidation on x-ray
complications may occur as below

Complications

cold agglutins (IgM) may cause an haemolytic anaemia, thrombocytopenia


erythema multiforme, erythema nodosum
meningoencephalitis, Guillain-Barre syndrome
bullous myringitis: painful vesicles on the tympanic membrane
pericarditis/myocarditis
gastrointestinal: hepatitis, pancreatitis
renal: acute glomerulonephritis

Investigations

diagnosis is generally by Mycoplasma serology


positive cold agglutination test
1343. A 10yo boy is brought to the ED 10h after injury to the foot. It was punctured with a metal spike that
passed through his shoe. What is the next best step?
a. Ig
b. Ig and vaccine
c. Vaccine only
d. Clean the wound
e. Antibiotics
Initial step is always to clean the wound in order to get rid of source.
1344. A 56yo male presents with persistent watery diarrhea. What is the most likely dx?
a. Treponema pallidum
b. Nesseria meningitides
c. Cryptosporidium
d. Staph aureus
e. Pseudomonas aeruginosa
It should be c.
C coz
T pallidum...syphilis...not diarrhoea
N menigitidis....menigitis
S aureus...also not organisms for diarrhea
P aeruginosa....again not causing diarrhoea

1345. A 2yo girl has frequency, urgency and burning micturition. She has some supra pubic tenderness.
Which one of the following is the most appropriate initial inv?
a. Supra pubic aspiration of urine for C&S
b. Clean catch of urine for C&S
c. USG
d. IVU
e. MCUG
Ans is clean catch
Urinary tract infections (UTI) are more common in boys until 3 months of age (due to more congenital
abnormalities) after which the incidence is substantially higher in girls. At least 8% of girls and 2% of boys will
have a UTI in childhood

Presentation in childhood depends on age:

infants: poor feeding, vomiting, irritability


younger children: abdominal pain, fever, dysuria
older children: dysuria, frequency, haematuria
features which may suggest an upper UTI include: temperature > 38C, loin pain/tenderness

NICE guidelines for checking urine sample in a child


if there are any symptoms or signs suggestive or a UTI
with unexplained fever of 38C or higher (test urine after 24 hours at the latest)
with an alternative site of infection but who remain unwell (consider urine test after 24 hours at the
latest)

Urine collection method

clean catch is preferable


if not possible then urine collection pads should be used
cotton wool balls, gauze and sanitary towels are not suitable
invasive methods such as suprapubic aspiration should only be used if non-invasive methods are not
possible

Management

infants less than 3 months old should be referred immediately to a paediatrician


children aged more than 3 months old with an upper UTI should be considered for admission to
hospital. If not admitted oral antibiotics such as cephalosporin or co-amoxiclav should be given for 7-10
days
children aged more than 3 months old with a lower UTI should be treated with oral antibiotics for 3
days according to local guidelines, usually trimethoprim, nitrofurantoin, cephalosporin or amoxicillin.
Parents should be asked to bring the children back if they remain unwell after 24-48 hours
antibiotic prophylaxis is not given after the first UTI but should be considered with recurrent UTIs
1346. An 89yo man presents with carcinoma of posterior oropharynx. Which is the single most appropriate LN
involved?
a. Pre-aortic LN
b. Aortic LN
c. Submental LN
d. Submandibular LN
e. Deep cervical LN
Oropharyngeal Lumphatics >>> Retropharyngeal Ln >> Deep cervical LN
1347. A young boy presented to the OPD 12wks after renal transplantation with fever and pain in lower
abdomen. Renal functions were deranged. Renal biopsy showed immune cell infiltrate and tubular damage.
What is the most probable dx?
a. Pyelonephritis
b. Chronic graft rejection
c. Acute rejection
d. Drug toxicity
e. Graft vs host disease
Hyperacute rejection : Within minutes of transplant
Acute: After one week upto months
Chronic : After years due to fibrosis
1348. A 56yo lady presents with a pathological fx of T11 vertebra. There is found to be an underlying
metastatic lesion. What is her most common primary ca?
a. Lung
b. Breast
c. Uterine
d. Brain
Breast CA is most notorious for bony mets.
1349. A 6m infant has breast milk jaundice. He is otherwise feeding well and is not dehydrated. What would
his LFTs look like?
a. Total bilirubin:40, conjugated bilirubin<5%
b. Total bilirubin:300, conjugated bilirubin 85%
c. Total bilirubin:500, conjugated bilirubin>85%
d. Total bilirubin:400, conjugated bilirubin<85%
Breast milk jaundice will not have such high levels of total bilirubin. Plus the majority would be

unconjugated hence A

Breast milk jaundice: the baby is well and the jaundice usually resolves by six weeks but occasionally

continues for up to months.

1350. A 29yo man took a tour of Japan and also travelled to other parts of Asia, developed fever,
petecia and rash on his body. He didnt take malaria prophylaxis prior to travel. What is the most
likely dx?
a. Malaria
b. HSP
c. HIV
d. Dengue fever
e. ITP
Petechae due to low platelets and travel Hx to asia are cinchers here. Dengue fever is a viral infection which
can progress to viral haemorrhagic fever (also yellow fever, Lassa fever, Ebola)

Basics

transmitted by the Aedes aegyti mosquito


incubation period of 7 days
a form of disseminated intravascular coagulation (DIC) known as dengue haemorrhagic fever (DHF)
may develop. Around 20-30% of these patients go on to develop dengue shock syndrome (DSS)

Features

causes headache (often retro-orbital)


fever
myalgia
pleuritic pain
facial flushing (dengue)
maculopapular rash

Treatment is entirely symptomatic e.g. fluid resuscitation, blood transfusion etc


1351. A 4yo boy ingested his grandmothers medicine and has developed dilated pupil. What is the
cause?
a. Amitryptiline
b. Paracetamol
c. Iron
d. Digoxin
Amtriptyline has side effects common to anticholinergics which include mydriasis.
1352. A 46yo male presents with confusion and drowsiness. What is the most likely dx?
a. Cryptococcus neoformans
b. Toxoplasma gondii
c. HSV
d. CMV
e. Candida albicans
Headache, confusion, drowsiness can be caused by Cryptococcus neoformans as well as HSV. But, neoformans
is relatively rare and usually seen in immunocompromised, so we should go with HSV
1353. A child has developed rash after the tx of penicillin. What will be the cause of rash?
a. Drug reaction
b. Kawasaki
c. Inf Mono
Drug reaction
Mild to moderate allergic reactions to penicillin are common, and symptoms may include any of the
following:
Hives (raised, extremely itchy spots that come and go over a period of hours)
Tissue swelling under the skin, typically around the face (also known as angioedema)
Throat tightness.
Wheezing.
Coughing.
1354. A child comes with recurrent joint pain, multiple bruises, swollen ankle and unable to move his legs.
What is the inv of choice?
a. ESR
b. RF
c. Clotting factors
Clotting factor deficiency presents with hemarthromas.
1355. A 66yo man has renal colic. He has also presented with acute onset pain in his knee in the past.
What is the single most likely cause for renal failure?
a. SLE associated GN
b. Hypercalcemia
c. HTN
d. Hyperuricemia
e. Hyperoxaluria
Gouty arthritis with renal stone
Whereas if hypercalcemia would have been a cause then renal stone .excessive thirst secondary to
dehydration with generalized bony pain .(SLE is excluded as it occurs in females 8 times more than males &
affects symmetrically small joints) So, ans would be hyperuricemia
1356. A boy with a hx of recurrent swollen tender joints on both knees and elbows and not able to participate
in sports. What is the inv of choice to dx?
a. RF/ASO titre
b. Clotting factor
c. ESR
Clotting factor deficiency presents with hemarthromas.
1357. A 26yo man is referred for gastroscopy because of a hx of several months of dyspepsia. He has
routine bloods checked and is found to have a serum calcium level=3.2mmol/l with a venous
bicarbonate level of 33mmol/l. Renal and LFT are both mornal. CXR is normal. What is the most
likely cause of his hypercalcemia?
a. Melanoma
b. Metastatic malignancy
c. Milk alkali syndrome
d. Primary hyperparathyroidism
e. Sarcoidosis
Its milk or calcium alkali syndrome as there is hypercalcemia and metabolic alkalosis.

Interpreting Laboratory Values in Hypercalcaemia[7]

Condition Serum Serum Alkaline Urine Urine PTH


Phosphate Phosphatase Calcium Phosphate

Hyperparathyroidism Low Normal-high High (in High High


67% of
patients)

Vitamin D excess Normal- Low High High Low


high

Malignancy Often low High (except in Variable High Variable


haematological
malignancy, when
normal)

Granulomatous Normal- Normal-high High Normal Low


disease high

Calcium alkali Normal- Normal Normal Normal Low


syndrome high
Familial hypocalciuric Normal or Normal Low (<200 Normal High
hypercalcemia low mg/day)

1358. A 3yo boy presents with difficulty in walking and skin lesions. What is the most likely causative
agent?
a. Strep pyogenes
b. Rubella virus
c. Parvovirus
d. Papovirus
e. Paramyxovirus
Fifth disease (also called erythema infectiosum) is caused by parvovirus B19. A human virus, parvovirus B19 is

not the same parvovirus that veterinarians may be concerned about in pets, especially dogs, and it cannot be

passed from humans to animals or vice versa.

Fifth disease starts with a low-grade fever, headache, rash, and cold-like symptoms, such as a runny or stuffy

nose. These symptoms pass, then a few days later the rash appears. The bright red rash most commonly

appears in the face, particularly the cheeks. This is a defining symptom of the infection in children (hence the

name "slapped cheek disease"). Occasionally the rash will extend over the bridge of the nose or around the

mouth. In addition to red cheeks, children often develop a red, lacy rash on the rest of the body, with the

upper arms, torso, and legs being the most common locations. The rash typically lasts a couple of days and

may itch; some cases have been known to last for several weeks. Patients are usually no longer infectious once

the rash has appeared.[2][3]

Teenagers and adults may present with a self-limited arthritis. It manifests in painful swelling of the joints that

feels similar to arthritis. Older children and adults with fifth disease may have difficulty in walking and in

bending joints such as wrists, knees, ankles, fingers, and shoulders.

1359. A pt after his house fire came with hematemesis with erosion/ulcer of esophagus and on examination
there is 55% burn and on endoscopy there is a stomach/gastric erosion and soot in the mouth. What is the tx?
a. PO PPI
b. IV PPI
c. PPI and antibiotic
d. H. pylori test
e. Tracheal intubation

Tracheal intubation should be done to prevent airway blockage secondary to laryngeal edema.
1360. A 40yo man complains of severe colicky loin pain that radiates to his scrotum. He is noted to
have microscopic hematuria. No masses are palpated. What is the single most likely cause?
a. Acute cystitis
b. Bladder ca
c. Renal vein thrombosis
d. Acute pyelonephritis
e. Ureteric calculus
Its E
Presentation :

Many stones are asymptomatic and discovered during investigations for other conditions.

The classical features of renal colic are sudden severe pain. It is usually caused by stones in the

kidney, renal pelvis or ureter, causing dilatation, stretching and spasm of the ureter. In most cases no

cause is found:

Pain starts in the loin about the level of the costovertebral angle (but sometimes lower) and

moves to the groin, with tenderness of the loin or renal angle, sometimes with haematuria.

If the stone is high and distends the renal capsule then pain will be in the flank but as it moves

down pain will move anteriorly and down towards the groin.

A stone that is moving is often more painful than a stone that is static.

The pain radiates down to the testis, scrotum, labia or anterior thigh.

Whereas the pain of biliary or intestinal colic is intermittent, the pain of renal colic is more

constant but there are often periods of relief or just a dull ache before it returns. The pain may

change as the stone moves. The patient is often able to point to the place of maximal pain and this

has a good correlation with the current site of the stone.

Investigations
Basic analysis should include:
Stick testing of urine for red cells (suggestive of urolithiasis), white cells and nitrites (both

suggestive of infection) and pH (pH above 7 suggests urea-splitting organisms such as Proteusspp.

whilst a pH below 5 suggests uric acid stones).

Midstream specimen of urine for microscopy (pyuria suggests infection), culture and

sensitivities.

Blood for FBC, CRP, renal function, electrolytes, calcium, phosphate and urate, creatinine.

Prothrombin time and international normalised ratio if intervention is planned.

Non-enhanced CT scanning is now the imaging modality of choice and has replaced intravenous

pyelogram (IVP).[12] Ultrasound scanning may be helpful to differentiate radio-opaque from radiolucent

stones and in detecting evidence of obstruction.

Plain X-rays of the kidney, ureter and bladder (KUB) are useful in watching the passage of radio-

opaque stones (around 75% of stones are of calcium and so will be radio-opaque).

The European Association of Urology's guidelines on urolithiasis recommend stone analysis for:

All first-time stone formers.

All patients with recurrent stones who are on pharmacological preventing therapy.

Patients who have had early recurrence after complete stone clearance.

Late recurrence after a long stone-free period (stone composition may change).

Encourage the patient to try to catch the stone for analysis. This may mean urinating through a tea

strainer, a filter paper such as a coffee filter or a gauze.

Management[2]
Initial management can either be done as an inpatient or on an urgent outpatient basis, usually depending on

how easily the pain can be controlled.

Indications for hospital admission


Fever.

Solitary kidney.

Known non-functioning kidney.

Inadequate pain relief or persistent pain.

Inability to take adequate fluids due to nausea and vomiting.

Anuria.

Pregnancy.

Poor social support.

Inability to arrange urgent outpatient department follow-up.

People over the age of 60 years should be admitted if there are concerns on clinical condition or

diagnostic certainty (a leaking aortic aneurysm may present with identical symptoms).

1361. A 55yo man is having slow growing ascites. When we tap the peritoneal fluid the protein is <25 and it is
clear and yellow. What could be the origin for ascites?
a. Budd-Chiari
b. Gastrinoma
c. Hepatoma
d. TB
e. Pancreatitis
Budd Chiari = occlusion of the hepatic vein by, for eg hepatoma, causes transudative ascitis. Such occlusion
increases the hydrostatic pressure within the vessel, giving rise to the fluid shift from the intra vascular
compartment to the interstitium with consequent ASCITIS
If SAAG is more than 11g/L(exudative protein less than 25g/L) , fluid is termed as Transudate.
Causes of high SAAG ("transudate") are:

Cirrhosis - 81% (alcoholic in 65%, viral in 10%, cryptogenic in 6%)


Heart failure - 3%
Nephrotic syndrome
Hepatic venous occlusion: Budd-Chiari syndrome or veno-occlusive disease
Constrictive pericarditis
Kwashiorkor (childhood protein-energy malnutrition)

If SAAG is less than 11g/L(exudative protein more than 25g/L) , fluid is termed as exudate
Causes of low SAAG ("exudate") are:

Cancer (metastasis and primary peritoneal carcinomatosis) - 10%


Infection: Tuberculosis - 2% or Spontaneous bacterial peritonitis
Pancreatitis - 1%
Serositis
Hereditary angioedema
Other Rare causes:

Meigs syndrome
Vasculitis
Hypothyroidism
Renal dialysis
Peritoneum mesothelioma
Abdominal tuberculosis
1362. A 7yo boy presents with his mother to GP surgery. His mother describes he had presented this since
3wks ago. He had not experienced any trauma. No other symptoms a/w the condition. Exam: non tender
swollen ankles bilaterally. There is no rash or lesion. He is otherwise well. Which single test would be the best
as an initial assessment?
a. Plasma electrolytes
b. Albumin
c. Total serum protein
d. Anti-streptolysin
24 Hr urinary protein gold standard, if not given then opt for serum albumin..
Its nephrotic Albumin For nephrotic syndrome ..minimal change disease
1363. In lyme disease, which complication is most likely to lead to collapse?
a. Dilated CM
b. AV block
c. Mild encephalitis
d. Meningitis
e. Myocarditis
Ans is b
Lyme disease is caused by the spirochaete Borrelia burgdorferi and is spread by ticks

Features

early: erythema chronicum migrans + systemic features (fever, arthralgia)


CVS: heart block, myocarditis
neuro: cranial nerve palsies, meningitis

Investigation

serology: antibodies to Borrelia burgdorferi

Management

doxycycline if early disease. Amoxicillin is an alternative if doxycycline is contraindicated (e.g.


pregnancy)
ceftriaxone if disseminated disease
Jarisch-Herxheimer reaction is sometimes seen after initiating therapy: fever, rash, tachycardia after
first dose of antibiotic (more commonly seen in syphilis, another spirochaetal disease)
1364. A 30yo pt came to the OPD with complaint of breathlessness and dry cough. He has lost 5kgs in 2m. He
is an IV drug abuser. Inv: CXR=bilateral interstitial shadowing. What is the single most likely causative
organism?
a. Klebsiella
b. TB
c. Chlamydia pneumonia
d. PCP
e. Chlamydia psitacci
IV drug abuser is a clincher pointing towards HIV. Most common infections in HIV pts are caused by PCP.
HIV: Pneumocystis jiroveci pneumonia
Whilst the organism Pneumocystis carinii is now referred to as Pneumocystis jiroveci, the term Pneumocystis
carinii pneumonia (PCP) is still in common use

Pneumocystis jiroveci is an unicellular eukaryote, generally classified as a fungus but some authorities
consider it a protozoa
PCP is the most common opportunistic infection in AIDS
all patients with a CD4 count < 200/mm should receive PCP prophylaxis

Features

dyspnoea
dry cough
fever
very few chest signs

Pneumothorax is a common complication of PCP.

Extrapulmonary manifestations are rare (1-2% of cases), may cause

hepatosplenomegaly
lymphadenopathy
choroid lesions

Investigation

CXR: typically shows bilateral interstitial pulmonary infiltrates but can present with other x-ray findings
e.g. lobar consolidation. May be normal
exercise-induced desaturation
sputum often fails to show PCP, bronchoalveolar lavage (BAL) often needed to demonstrate PCP (silver
stain shows characteristic cysts)

Management

co-trimoxazole
IV pentamidine in severe cases
steroids if hypoxic (if pO2 < 9.3kPa then steroids reduce risk of respiratory failure by 50% and death by
a third)
1365. A 27yo female who had a RTA 7m back now complaints of attacks of sudden onset rotational
vertigo which comes on with sharp movements of the head and neck. Which of the following
would be most helpful?
a. Caloric testing
b. Hallpikes maneuver
c. Gutenbergers test
d. Menieres test
e. Otoscopy
DixHallpike test. The DixHallpike test or NylenBarany test is a diagnostic maneuver used to identify
benign paroxysmal positional vertigo (BPPV).
Benign paroxysmal positional vertigo (BPPV) is one of the most common causes of vertigo encountered. It is
characterised by the sudden onset of dizziness and vertigo triggered by changes in head position. The average
age of onset is 55 years and it is less common in younger patients.

Features

vertigo triggered by change in head position (e.g. rolling over in bed or gazing upwards)
may be associated with nausea
each episode typically lasts 10-20 seconds
positive Dix-Hallpike manoeuvre

BPPV has a good prognosis and usually resolves spontaneously after a few weeks to months. Symptomatic
relief may be gained by:

Epley manoeuvre (successful in around 80% of cases)


teaching the patient exercises they can do themselves at home, for example Brandt-Daroff exercises

Medication is often prescribed (e.g. Betahistine) but it tends to be of limited value.


1366. A man rescued from a building on fire presented with unconsciousness without any evidence of burns or
external injury or soot. What would you do next?
a. 100% oxyen inhalation
b. 24% oxygen by mask
c. Hyperbaric oxygen in a hyperbaric chamber
d. Intubation
e. Refer to specialist unit

I am confused about this one as we should always be following ABC first, but rule of thumb for securing an
airway is GCS less than 8 or a rapid falling GCS. And there is no soot or burn, obstruction due to laryngeal
edema is less likely. Unconcioussness does not mean a low GCS too low that intubation is indicated. As it
seems to be the case of CO poisoning, First give him isobaric Oxygen via a facemask and calculate his GCS ( by
giving painful stimuli etc ) and then try securing his airway via a LMA or Guedel's airway or an ETT if expertise
is available
1367. A pt has had 1 ep of depression and 2 eps of mania over the last year and now presents with depression.
He is on anti-depressants. What additional pharmacological tx would now act as a prophylaxis for his
condition?
a. Antidepressants
b. Antipsychotics
c. Mood stabilizers
d. No additions req
Ans is C
Long-term treatment in secondary care to prevent relapse or recurrence
After each acute episode of mania or bipolar depression, a discussion should be had with the patient and/or

carer about the nature and course of the disorder, treatment options, the risk of relapse after stopping

treatment and the risks and benefits of pharmacological and psychological therapy. Risks may be particularly

relevant in women of child-bearing age. Factors to take into account include:

The severity and frequency of episodes.

Previous response to therapy.

Symptoms between episodes.

Relapse triggers, warning signs of relapse and coping strategies.

Potential length of treatment and review arrangements.

Provide clear written information about bipolar disorder, including NICE's information for the public and

ensure there is enough time to discuss options and concerns.:[1]

Options available include:

Pharmacological.

Lithium should be considered first-line, with the addition of valproate if ineffective.

Valproate or olanzapine should be considered for patients intolerant of lithium or who are not

prepared to undergo regular monitoring.

If symptoms still continue then the patient should be referred to a mental health specialist.

Medications that might be used in this situation are lamotrigine (especially in bipolar II disorder) or

carbamazepine.

Lithium will require monitoring of levels and monitoring of renal function and thyroid function.

Patients need to be advised of adequate rehydration and the dangers of suddenly stopping

treatment.
Long-term therapy usually continues for two years but may be needed for as long as five years.

If medication is stopped, patients should be made aware of early warning symptoms of

recurrence. Medication should be tailed off gradually (unless acute toxicity develops). Mood should

be monitored for two years after treatment is stopped.

Cognitive behavioural therapy, interpersonal therapy or behavioural couples therapy may be

appropriate.

NICE provides a link to an evidence-based manual to a psychological intervention that has been

developed specifically for bipolar disorder .

Psychosocial education is beneficial. Various methods are available, including teaching coping

strategies and managing communication difficulties. Psychosocial interventions are particularly important

for paediatric and adolescent patients, to provide families with an understanding of symptoms, course,

and treatment.

1368. A man presented with a purplish swelling at the anal area. It is acutely painful and he complains of
constipation for the last 2m. What is the most appropriate management?
a. I&D
b. I&D + antibiotics
c. Reassure
d. Analgesia
e. Sclerotherapy
Ans is B
Piles (haemorrhoids) and perianal haematoma

Anal pain can sometimes be caused by piles or a perianal haematoma (burst blood vessel under the skin at the
edge of the anus).

Piles can become painful when they become "strangulated" and bulge outside the anus, developing a blood
clot. They are usually treated with painkillers, ointments and sometimes ice packs, although surgery is
occasionally needed.

A perianal haematoma is usually relieved by a simple procedure to remove the clot using a local anaesthetic,
sometimes done by your GP and then antibiotics are given
1369. A pt came to the ED after he had banged his car quite a few times on reversing. He was complaining of
seeing double while he tried to look back during the process of reversing the car, he also complains of double
vision on looking at an outward gaze. Which nerve is involved?
a. Abducent nerve
b. Trochlear nerve
c. Oculomotor nerve
d. Optic nerve
e. Trigeminal nerve

External Ocular Paralysis

Muscle Direction of pull Result of paralysis Cranial nerve

Medial rectus Medially Lateral III

Superior rectus Upwards Downwards III

Lateral rectus Laterally Medial VI

Inferior rectus Downwards Upwards III

Superior oblique Down and out Up and in IV

Inferior oblique Up and out Down and in III

Ans is abducens nerve


1370.. A pt had a stroke Now, there is left sided weakness and right side facial numbness. CT shows ischemic
stroke. Which one would you prescribe?
a. Alteplase
b. Aspirin
c. Clopidogrel
d. Heparin
e. Warfarin
Ans is A
The Royal College of Physicians (RCP) published guidelines on the diagnosis and management of patients
following a stroke in 2004. NICE also issued stroke guidelines in 2008, although they modified their guidance
with respect to antiplatelet therapy in 2010.

Selected points relating to the management of acute stroke include:

blood glucose, hydration, oxygen saturation and temperature should be maintained within normal
limits
blood pressure should not be lowered in the acute phase unless there are complications e.g.
Hypertensive encephalopathy*
aspirin 300mg orally or rectally should be given as soon as possible if a haemorrhagic stroke has been
excluded
with regards to atrial fibrillation, the RCP state: 'anticoagulants should not be started until brain
imaging has excluded haemorrhage, and usually not until 14 days have passed from the onset of an
ischaemic stroke'
if the cholesterol is > 3.5 mmol/l patients should be commenced on a statin. Many physicians will delay
treatment until after at least 48 hours due to the risk of haemorrhagic transformation

Thrombolysis

Thrombolysis should only be given if:

it is administered within 4.5 hours of onset of stroke symptoms (unless as part of a clinical trial)
haemorrhage has been definitively excluded (i.e. Imaging has been performed)

Alteplase is currently recommended by NICE.


1371. A young boy presents with fever and cough. His father was dx with TB a week ago. The parents dont
want him to have a BAL under anesthesia. Which other samples can be taken for dx?
a. Urine
b. Blood
c. CSF
d. Gastric washing
e. Sweat
Ans D
Coz goblet cells secrete mucins n they r found in airways n stomach too.
1372. A 50yo man came to the hosp a few months after he had a MI. Exam: everything normal, S1 and S2 were
heard on auscultation, but there is a new pan-sytolic murmur. What is the most appropriate inv of choice?
a. ECG
b. 24h ECG
c. Echo
d. CXR
e. CT
Holosystolic (pansystolic)

mitral/tricuspid regurgitation (high-pitched and 'blowing' in character)


VSD ('harsh' in character)
Inv of choice would be ECHO
1373. A 73yo stroke pt has been on aspirin for 2yrs. He now presents with epigastric pain and is asking for a tx.
What is the most appropriate management?
a. Laparotomy
b. NSAIDs
c. Omeprazole
Morphine
d. Tramadol
PPIs should be introduced in cases of discomfort by the prophylactic use of NSAIDS.
1374. A 2yo girl is brought to the ED by her mother. The child is screaming that there is something in
her ear and she appears agitated. Exam: a plastic bead is seen inside the ear. What is the best
method of removal?
a. Forceps
b. Hook
c. Under general anesthesia
d. Syringing
e. Magnet
Ans is C as the kid is irritable.

Presentation
Most older children and adults will know that there is something in their ear but sometimes a

foreign body may get into the external ear canal without the patient realising.

The patient may present with pain, deafness or discharge. Live insects may cause a buzzing in the

ear.

The appearance will vary according to the object and length of time it has been in the ear:

An inanimate object that has been in the ear a very short time presents with no abnormal

finding other than the object itself.

Pain or bleeding may occur with objects that abrade the ear canal, from rupture of the

tympanic membrane, or from the patient's attempts to remove the object.

With delayed presentation, erythema and swelling of the canal and a foul-smelling discharge

may be present.

Management
A great deal of care is required in order not to push the object deeper into the ear canal and not to damage

the ear canal. There is a high failure rate in removal of foreign bodies from the ear.

Insects should be killed prior to removal, using 2% lidocaine.


Remove batteries or magnets as soon as possible to prevent corrosion or burns. Do not crush a

battery during removal.

Adhesives (eg, Super Glue) may be removed manually within 1-2 days once desquamation has

occurred. Referral to an ear, nose and throat specialist is required if an adhesive is in contact with the

tympanic membrane.

Methods for removal


Forceps or hook: grasp the object with forceps, or place a hook behind the object and pull it out.

Irrigation is often effective. Irrigation with water is contra-indicated for soft objects, organic matter

or seeds (which may swell and increase the level of pain and difficulty to remove if exposed to water).

Suction with a small catheter held in contact with the object may be effective.

1375. During antenatal visits, the following tests are routinely offered to all pregnant mothers apart
from HIV and Hep B?
a. Rubella and syphilis
b. Syphilis and toxoplasmosis
c. Hep C & thalassemia
d. CMV and rubella
e. Sickle cell anemia and Hep
Gestation Purpose of visit

8 - 12 weeks (ideally < 10 Booking visit


weeks)

general information e.g. diet, alcohol, smoking, folic acid, vitamin D,


antenatal classes
BP, urine dipstick, check BMI
Booking bloods/urine

FBC, blood group, rhesus status, red cell alloantibodies,


haemoglobinopathies
hepatitis B, syphilis, rubella
HIV test is offered to all women
urine culture to detect asymptomatic bacteriuria

10 - 13+6 weeks Early scan to confirm dates, exclude multiple pregnancy

11 - 13+6 weeks Down's syndrome screening including nuchal scan


16 weeks Information on the anomaly and the blood results. If Hb < 11 g/dl consider iron
Routine care: BP and urine dipstick

18 - 20+6 weeks Anomaly scan

25 weeks (only if primip) Routine care: BP, urine dipstick, symphysis-fundal height (SFH)

28 weeks Routine care: BP, urine dipstick, SFH


Second screen for anaemia and atypical red cell alloantibodies. If Hb < 10.5 g/dl
consider iron
First dose of anti-D prophylaxis to rhesus negative women

31 weeks (only if primip) Routine care as above

34 weeks Routine care as above


Second dose of anti-D prophylaxis to rhesus negative women*
Information on labour and birth plan

36 weeks Routine care as above


Check presentation - offer external cephalic version if indicated
Information on breast feeding, vitamin K, 'baby-blues'

38 weeks Routine care as above

40 weeks (only if primip) Routine care as above


Discussion about options for prolonged pregnancy

41 weeks Routine care as above


Discuss labour plans and possibility of induction
1376. A 32yo male complains of tremors everytime he tends to use his muscles and when he is pointing at
objects. No complaints at rest. His father complained of similar problems. What is
the most probable dx?
a. Parkinsonism
b. Lithium toxicity
c. Thyrotoxicosis
d. Benign essential tremor
Essential tremor (previously called benign essential tremor) is an autosomal dominant condition which usually
affects both upper limbs

Features

postural tremor: worse if arms outstretched


improved by alcohol and rest
most common cause of titubation (head tremor)

Management
propranolol is first-line
primidone is sometimes used
1377. A 40yo woman with breast cancer has back pain which keep her awake at night. She blames it on a gym
session she had 2wks ago. She now has difficulty in climbing stairs. There is tenderness over the right thoracic
spine. She has diminished fine touch and temp sensation in her right foot.
What is the single most appropriate inv?
a. Bone density scan
b. CT head
c. MRI spine
d. Nuclear bone scan
e. XR thoracolumbar spine
This was a case of spine metastasis (vertebra) and stress during exercising in gym caused collapsing fracture of
vertebra which lead to spinal cord compression. X-ray will only show vertebral fracture and not the
compression. As there is neuro features so to deliniate the spinal cord compression we have to do MRI.
1378. A pregnant lady at her 39wk GA present with eclampsia. Soon after her arrival in the labour suit, IV
MgSO4 and IV hydralazine has been prescribed. The pt then develops another fit in the hosp and maintenance
dose of MgSO4 has been started. What is your next step in management?
a. Mg SO4 bolus
b. Delivery of baby
c. MgSO4 loading dose
d. Diazepam
Definitive tx of Eclampsia is delivery. Pt had already been given mgso4 twice so we should now go for delivery.

Management of eclampsia
Resuscitation:

The patient should be placed in the left lateral position and the airway secured.

Oxygen should be administered.

Treatment and prophylaxis of seizures:

Magnesium sulfate is the anticonvulsant drug of choice.

Intubation may become necessary in women with repeated seizures in order to protect the

airway and ensure adequate oxygenation.

Treatment of hypertension:

Reduction of severe hypertension (blood pressure >160/110 mm Hg or mean arterial pressure

>125 mm Hg) is essential to reduce the risk of cerebrovascular accident. Treatment may also reduce

the risk of further seizures.


Intravenous hydralazine or labetalol are the two most commonly used drugs. Both may

precipitate fetal distress and therefore continuous fetal heart rate monitoring is necessary.

Fluid therapy:

Close monitoring of fluid intake and urine output is mandatory.

Pre-loading the circulation with 400-500 ml colloid prior to regional anaesthesia or

vasodilatation with hydralazine may reduce the risk of hypotension and fetal distress.

Delivery:

The definitive treatment of eclampsia is delivery. Attempts to prolong pregnancy in order to

improve fetal maturity are unlikely to be of value.

However, it is unsafe to deliver the baby of an unstable mother even if there is fetal distress.

Once seizures are controlled, severe hypertension treated and hypoxia corrected, delivery can be

expedited.

Vaginal delivery should be considered but Caesarean section is likely to be required in

primigravidae, well before term and with an unfavourable cervix.

After delivery, high-dependency care should be continued for a minimum of 24 hours.

All patients need careful follow-up and a formal postnatal review to establish if there is chronic hypertension,

proteinuria or liver damage

1379. A man suffering from Influenza A since 5d ago. CXR: pneumonia. What organism is responsible for

pneumonia in this pt?

a. Hemophilius influenze
b. Klebsiella
c. Staphylococcus aureus
d. Streptococcus pneumonia
e. Pseudomonas
Ans would be staphylococcus aureus
1380. A pt admitted due to repeated attacks of pancreatitis presents with dementia and loss of proprioception
in the legs. What is the most appropriate tx?
a. Thiamine
b. Pyridoxine
c. Cobolamin
d. Lipase
e. Antibiotics
Ans is C
Pancreatic enzymes in the duodenum cleave off the R-binders(which come from salivary glads) from Vit-B12,
which then combines with IF and this complex is absorbed in terminal ileum. Pancreactic enzyme deficiency
therefore leads to impaired absorption of Vit B12 and sub-acute combined degeneration of spinal cord
.
Alcoholic + pancreatitis more in favor of B12
Alcoholic only think of thiamine
1381. A man after MI presented with sudden breathlessness and dyspnea. Exam: scattered pansystolic
murmur all over the precordium. What is the next inv that will lead to dx?
a. ECG
b. Echo
c. CT
d. Blood culture
e. CXR
We would go for echo !!!!
1382. During a laparoscopic procedure, a trochar is inserted halfway between the umbilicus and the ant
superior iliac spine. What are the structures most likely to be pierced?
a. Rectus sheath
b. Linea alba
c. External oblique aponeurosis
d. Internal oblique and transverse abdominal
e. Both C and D
There is a confusion regarding the answer of this question. But, I think the answer should be E considering the
insertions of ext oblique.
1383. A pt, a small child presented with URTI and later developed fever, earache and tympanic membrane is
dull. What is the likely dx?
a. OM
b. OE
c. Glue ear
d. Perforation of the tympanic membrane
e. Referred ear ache
Ans is A
The answer is A. Acute OM without perforation presents as such. The pus collected in the middle ear causes
the TM to lose its pearly white colour and appear dull or sometimes yellow. And it is usually associated with
pain. Glue ear presents with all the above mentioned points except the fever. Here fever developed
afterwards whereas glue ear usually follows a viral urti.
The differentiating point here is the presentation ..
AOM .. Otalgia ..
OME .. Hearing impairment noticed by parents is the mode of presentation in 80%.
1384. A 72yo male who is a regular smoker has come to the ED with complaints of loss of weight and loss of
appetite. He also complains of odynophagia. Exam: actively bleeding ulcer on right tonsil. What is the most
appropriate dx?
a. Tonsillar ca
b. Vincents angina
c. Irritant ingestion
d. Paracoccidiodmycosis
e. Herpes simplex infection
More than 70% of tonsillar cancers are SCC. Most of the others are lymphomas. Metastases to the palatine

tonsils are rare but there have been reports of secondaries from breast, lung, renal, pancreatic and colorectal

malignancies.

Patients with tonsillar carcinomas may present with a neck mass, usually in the jugulodigastric

region. Even if the neck mass is not evident on casual inspection, careful palpation may reveal cervical

lymphadenopathy.

Sore throat, ear pain, foreign body or mass sensation, and bleeding may occur.

Trismus is an ominous sign because it probably indicates involvement of the parapharyngeal space.

Such tumours may be large enough to involve or encase the carotid sheath. In addition, the tumour may

extend to the skull or mediastinum.

If the tumour has involved the tongue base, contralateral nodes may be involved.

Primary tonsillar tumours may grow entirely beneath the surface. The clinician may therefore see

nothing suspicious or may see only a slight increase in the size of the tonsil or the firmness of the area.

Alternatively, an exophytic fungating mass with central ulceration and heaped-up edges may be

present. It may be deep red to white.

Weight loss and fatigue are not uncommon.

Treatment may lead to pain, xerostomia, infections, poor wound healing, dysphagia, fistula

formation, trismus, potential disfigurement and fatigue.

1385. A pt with regular episodes of SNHL, vertigo and tinnitus lasting >30min. Neurological
exam=normal. What is the likely dx?
a. Menieres disease
b. Acoustic neuroma
c. Otosclerosis
d. Benign positional vertigo
e. Labrynthitis
Meniere's disease is a disorder of the inner ear of unknown cause. It is characterised by excessive pressure
and progressive dilation of the endolymphatic system. It is more common in middle-aged adults but may be
seen at any age. Meniere's disease has a similar prevalence in both men and women.

Features

recurrent episodes of vertigo, tinnitus and hearing loss (sensorineural). Vertigo is usually the
prominent symptom
a sensation of aural fullness or pressure is now recognised as being common
other features include nystagmus and a positive Romberg test
episodes last minutes to hours
typically symptoms are unilateral but bilateral symptoms may develop after a number of years

Natural history

symptoms resolve in the majority of patients after 5-10 years


the majority of patients will be left with a degree of hearing loss
psychological distress is common

Management

ENT assessment is required to confirm the diagnosis


patients should inform the DVLA. The current advice is to cease driving until satisfactory control of
symptoms is achieved
acute attacks: buccal or intramuscular prochlorperazine. Admission is sometimes required
prevention: betahistine may be of benefit
1386. A pt with celiac disease from birth, now as an adult presented with some abdominal symptoms. The
biopsy shows infiltration of the gastric epithelium by lymphocytes. What is the most likely dx?
a. Lymphoma
b. Diverticular disease
c. Lynch syndrome
d. Gastric TB
e. Peritoneal tumor
Lympho because that's the main complication of Celiac disease and in biopsy v can see lymphocytic
infiltrates. Pts with celiac diasease are at a riskt of developing MALT lymphoma (enteropathy-
associated T-cell lymphoma of small intestine)
1387. A 55yo man presented with hot, raised, tender area of skin on his right leg. He is febrile with rigors. He
has been started on flucloxacillin. What other meds will you add?
a. Ciprofloxacin
b. Gentamicin
c. Metronidazole
d. Benzylpenicillin
e. Ceftriaxone
Cellulitis is a term used to describe an inflammation of the skin and subcutaneous tissues, typically due to
infection by Streptococcus pyogenes or Staphylcoccus aureus.

Features

commonly occurs on the shins


erythema, pain, swelling
there may be some associated systemic upset such as fever

Management

The BNF recommends flucloxacillin as first-line treatment for mild/moderate cellulitis. Clarithromycin or
clindamycin is recommend in patients allergic to penicillin.

Many local protocols now suggest the use of oral clindamycin in patients who have failed to respond to
flucloxacillin.

Severe cellulitis should be treated with intravenous benzylpenicillin + flucloxacillin.


1388. A 65yo man has incurable bronchial cancer. He is unable to cough up his secretions. This is leading to a
distressing cough. Which of the following drugs is most likely to help him?
a. Scopolamine
b. Xanomeline
c. Aceclidine
d. Pilocarpine
e. Cevimiline
Ans A
Scopolamine : Antisecretory
Xanomeline : Trial in Alzheimers and Pscizophrenia
Aceclidine : tx of narrow angle glaucoma
Cevimiline : Dry mouth
1389. A pt presented after eating a seafood dish at a local restaurant. He complains of difficulty in breathing.
His speech is slurred and his BP=85/55mmHg. What would be the most appropriate next step?
a. IV adrenaline
b. IM adrenaline
c. SC adrenaline
d. PO chlorpheniramine
e. IV chlorpheniramine
Anaphylaxis may be defined as a severe, life-threatening, generalised or systemic hypersensitivity reaction.

Anaphylaxis is one of the few times when you would not have time to look up the dose of a medication. The
Resuscitation Council guidelines on anaphylaxis have recently been updated. Adrenaline is by far the most
important drug in anaphylaxis and should be given as soon as possible. The recommended doses for
adrenaline, hydrocortisone and chlorphenamine are as follows:

Adrenaline Hydrocortisone Chlorphenamine

< 6 months 150 micrograms (0.15ml 1 in 1,000) 25 mg 250 micrograms/kg

6 months - 6 years 150 micrograms (0.15ml 1 in 1,000) 50 mg 2.5 mg


6-12 years 300 micrograms (0.3ml 1 in 1,000) 100 mg 5 mg

Adult and child > 12 years 500 micrograms (0.5ml 1 in 1,000) 200 mg 10 mg

Adrenaline can be repeated every 5 minutes if necessary. The best site for IM injection is the anterolateral
aspect of the middle third of the thigh.

Common identified causes of anaphylaxis

food (e.g. Nuts) - the most common cause in children


drugs
venom (e.g. Wasp sting)
1390. A 7yo boy presents with proptosis and periorbital edema. What is the immediate action that needs to
be taken?
a. IV morphine and immediate ophthalmoscopy
b. IV morphine
c. Observation only
Seems to be an incomplete recall. Considering the age, and unilateral problem, diagnosis may be unilateral
orbital cellulitis. Ans seems to be A
Presentation :
Sudden onset of unilateral swelling of conjunctiva and lids.
Proptosis (bulging of the eye).
Pain with movement of the eye, restriction of eye movements.
Blurred vision, reduced visual acuity, diplopia.
Pupil reactions may be abnormal - relative afferent pupillary defect (RAPD); see the separate article on
Examination of the Eye.
Fever, severe malaise.
Management :
Hospital admission under the joint care of the ophthalmologists and the ENT surgeons is mandatory.
Intravenous antibiotics are used (eg, cefotaxime and flucloxacillin) in addition to metronidazole in
patients over 10 years old with chronic sinonasal disease.[3]
Clindamycin plus a quinolone such as ciprofloxacin are used where there is penicillin sensitivity.
Vancomycin is also an alternative.
Optic nerve function is monitored every four hours (pupillary reactions, visual acuity, colour vision and
light brightness appreciation).
Treatment may be modified according to microbiology results and lasts for 7-10 days.
Surgery is indicated where there is CT evidence of an orbital collection, where there is no response to
antibiotic treatment, where visual acuity decreases and where there is an atypical picture which may
warrant a diagnostic biopsy. Surgery often concurrently warrants drainage of infected sinuses
1391. A schizophrenic man complains that he can hear voices talking about him and telling him to end
his life by cutting his throat. He only hears them when he wakes up from sleep and not at other
times. What type of hallucinations is he having?
a. Somatic
b. Kinesthetic
c. Hypnogogic
d. Hypnopompic
e. Lilliputian
Hypnagogic - occur on falling asleep and are harmless.
Hypnopompic - occur on waking up and are harmless.
Auditory - of one or more talking voices; seen commonly in schizophrenia.
Charles Bonnet's syndrome - visual hallucinations that blind persons experience

1392. A 28yo woman complains of hearing strange voices in her bedroom as she is falling asleep in the night.
She says there is no one in the room except her. On evaluation she has no other problems. What is she
suffering from?
a. Delusion of persecution
b. Cotard syndrome
c. Hypnogogic hallucinations
d. Lilliputian hallucinations
e. Schizophrenia
Types of Hallucinations :
Hypnagogic - occur on falling asleep and are harmless.
Hypnopompic - occur on waking up and are harmless.
Auditory - of one or more talking voices; seen commonly in schizophrenia.
Charles Bonnet's syndrome - visual hallucinations that blind persons experience

1393. A 32yo man on psychiatric meds presents with coarse tremors and diarrhea. What is the most likely
altnernate to the drug causing the prb?
a. Lithium
b. Diazepam
c. Haloperidol
d. Valproate
e. Citalopram
Seems that Lithium has caused the symptoms. Its adverse effects are :
G I upset.
Coarse tremors
Hypo/hyper thyroidism
Diabetes insipidus
Ebstein anomaly(in foetus)
Lithium is used for bipolar disorder, alternatively we can give valproate (2nd line)
1394. A man is brought to the ED after he was stabbed in the best. Chest is clear bilaterally with muffled heart
sounds. BP=60/0mmHg, pulse=120bpm, JVP is raised. What is the most probable dx?
a. Pulmonary embolism
b. Cardiac tamponade
c. Pericardial effusion
d. Hemothorax
e. Pneumothorax
Cardiac tamponade
Features
dyspnoea
raised JVP, with an absent Y descent - this is due to the limited right ventricular filling
tachycardia
hypotension
muffled heart sounds
Kussmaul's sign (much debate about this)
ECG: electrical alternans
Pulsus Paradoxus

1395. A 64yo alcoholic who has been dx with liver cirrhosis presents with a massive ascites. What is
the mechanism of fluid accumulation in a pt with liver disease?
a. Cirrhosis
b. Portal HTN
c. Hypoalbuminemia
d. Liver failure
e. Hepatic encephalopathy
This question asks about the cause of a broader term - fluid accumulation - which is mainly mediated by
hypoalbuminemia. Liver cirrhosis is the final stage of liver disease and hypoalbuminemia can be noticed prior
to that stage.
1396. A 38yo man presented to ED with severe pain in upper abdomen. He has already taken course of
triple therapy and now had elective endoscopy 2d ago. He is in shock. What is the most
probable dx?
a. Ca esophagus
b. Barrets esophagus
c. Mediastinitis
d. Ca stomach
Always suspect mediastinitis in a pt with shock and fever after a hx of
- Recent cardiothoracic surgery or instrumentation.
- Upper GI endoscopy.
- Bronchoscopy.
- Recent dental or oropharyngeal infection.
- Upper respiratory tract infection
- Ingestion of a foreign body (particularly button batteries by young children, which may cause oesophageal
rupture).
other signs may include edema of the neck and face , and crunching sound when auscultation of the heart
Surgical referral is urgent
Boerhaave's syndrome
The term Boerhaave's syndrome is reserved for the 10% of esophageal perforations which occur due to
vomiting
This can occur due to with Alcoholics who drink a lot and then vomit forcefully , this may cause esophageal
perforation thereafter mediastinitis followed by death .....
MalloryWeiss , also known as , gastro-esophageal laceration syndrome refers to bleeding from tears ... rather
than perforation .....
1397. A 68yo man who is a known case of liver cirrhosis has developed ascites. What is the mechanism for the
development of ascites?
a. Portal HTN
b. Hypoalbuminemia
c. Congestive heart failure
d. Liver failure
Ans can be A or B. Trigger factor for ascities is splancnic vasodilation due to PHT. Hypoalbuminemia plays a
vital role too. According to ohcm pg 260, Portal HTN is mentioned as a cause of ascities. Mechanism of "fluid
accumulation" (BROADER TERM ) is hypoalbuminaemia and ascites is portal HTN
1398. A man feels mild discomfort in the anal region and purulent discharge in underpants. What is the most
likely dx?
a. Feacal incontinence
b. Anal abscess
c. Fistula in ano
d. Anal tags
e. Rectal Ca
Fistula in ano is commonly seen in otherwise fit, young males. Associations: crohn's, diabetes, obesity Causes :
perianal sepsis, crohns. TB, Rectal CA. Immunocompromise
Inv : Endoanal USS
In anal abscess- there would be severe pain & fever. Abscess is confined in the cavity and is very painful. Rectal
ca would have been presented with bleeding or feeling of imcomplete defecation. . Some fistulla are painless
specially if old.
1399. A 38yo female presents with difficulty in looking upward and on examination she was found to have lid
lag as well. She also complains of her heart racing at times. Which test will help in dx?
a. Tensilon test
b. 24h ECG
c. TFT
d. Schimmer test
e. Young Helmholtz ophthalmoscopy
Hyperthyroidism
Causes
Graves' disease
toxic nodular goitre
subacute (de Quervain's) thyroiditis
post-partum thyroiditis
acute phase of Hashimoto's thyroiditis (later results in hypothyroidism)
toxic adenoma (Plummer's disease)
amiodarone therapy
Investigation
TFTS (TSH down, T4 and T3 up)
thyroid autoantibodies
other investigations are not routinely done but includes isotope scanning

1400. A young anxious mother of a 10m boy comes to you and requests a test for CF. What is the most
appropriate inv?
a. Sweat test
b. Heel prick test
c. Breath test
d. CXR
CF is an autosomal recessive disease caused by mutations in the CF transmembrane conductance regulator
(CFTR) gene, on chromosome 7. The only risk factor is a family history of the condition.
Signs
These may include:

Finger clubbing.
Cough with purulent sputum.
Crackles.
Wheezes (mainly in the upper lobes).
Forced expiratory volume in one second (FEV1) showing obstruction.

Babies diagnosed with CF will usually have no signs or symptoms.

Investigations
Sweat testing confirms the diagnosis and is 98% sensitive. Chloride concentration >60
mmol/L with sodium concentration lower than that of chloride on two separate occasions.
Molecular genetic testing for CFTR gene.
Sinus X-ray or CT scan - opacification of the sinuses is present in almost all patients with CF.
CXR or CT of thorax.
Lung function testing - spirometry is unreliable before 6 years.
Sputum microbiology - common pathogens include Haemophilus influenzae,Staphylococcus
aureus, Pseudomonas aeruginosa, Burkholderia cepacia, Escherichia coli and Klebsiella
pneumoniae.
Various blood tests including FBC, U&Es, fasting glucose, LFTs and vitamin A, D and E levels
are usually performed.

Management of cystic fibrosis involves a multidisciplinary approach

Key points

regular (at least twice daily) chest physiotherapy and postural drainage. Parents are usually taught to
do this. Deep breathing exercises are also useful
high calorie diet, including high fat intake*
vitamin supplementation
pancreatic enzyme supplements taken with meals
heart and lung transplant

1400. A young anxious mother of a 10m boy comes to you and requests a test for CF. What is the most
appropriate inv?
a. Sweat test
b. Heel prick test
c. Breath test
d. CXR

Answer= A- sweat test (Sweat testing confirms the diagnosis of cystic fibrosis and is 98% sensitive)
exclusion of other options:
Heel prick test= this test is usually done on the 5th to 6th day of life. A blood spot is taken from babys heel it
is done for screening diseases like sickle cell disease, cystic fibrosis, congenital hypothyroidism,
phenylketonuria, homocystinuria etc.In cystic fibrosis the heel prick test detects a chemical called
immunoreactive trypsinogen.
CXR=not diagnostic in CF

Cystic Fibrosis:
CF is an autosomal recessive disease caused by mutations in the CF transmembrane conductance regulator
(CFTR) gene, on chromosome 7.
CFTR is an ATP-responsive chloride channel that also affects other cellular activities, such as sodium transport
across the respiratory epithelium, composition of cell surface glycoprotein and antibacterial defences.

Clinical features:
neonates= failure to thrive,meconium ileus,rectal prolapse
children and adults= respiratory: recurrent chest infections ( recurrent lower respiratory tract infection (LRTI)
with chronic sputum production is the most common presentation)
GIT: pancreatic insufficiency, gallstones, cirrhosis etc
others= male infertility,vasculitis,nasal polyps,arthritis,osteoporosis,hypertrophic pulmonary osteoarthropathy
Signs= finger clubbing,cyanosis,bilateral coarse crackles

Investigations=
DIAGNOSIS by SWEAT TEST= . Chloride concentration >60 mmol/L with sodium concentration lower than that
of chloride on two separate occasions.
Molecular genetic testing for CFTR gene
Sputum microbiology - common pathogens include Haemophilus influenzae,Staphylococcus aureus,
Pseudomonas aeruginosa, Burkholderia cepacia, Escherichia coli and Klebsiella pneumoniae.
Treatment=
Patient care is most effective when provided in specialist centres by multidisciplinary teams. symptomatic
treatment for instance, respiratory= chest physiotherapy
GIT= pancreatic enzyme replacement, fat sol vitamin supplementation etc

1401. A 22yo Greek man presents with rapid anemia and jaundice following tx of malaria. He is noted to have
Heinx bodies. Choose the single most likely cause from the given options?
a. G6PD deficiency
b. Anemia of chronic disease
c. Pernicious anemia
d. IDA
e. Vit B12 deficiency

Answer= Glucose 6 phosphate dehydrogenase deficiency (G6PD). Acute haemolysis from G6PD deficiency can
produce HEINZ BODIES which are denatured haemoglobin and bite cells. it is precipitated by drugs such as
primaquine (antimalarial),sulfonamides,aspirin.

exclusion of other causes=


anemia of chronic disease: commonest anemia in hospital patients.common causes include Chronic
infection,Inflammation - including connective tissue disorders,Neoplasia.
pernicious Anemia=type of megaloblastic anemia due to impaired absorption of vitamin b12 deficiency.it is
caused by autoimmune atrophic gastritis leading to acchlorhydria and lack of intrinsic factor secretion.
Iron def Anemia=microcytic hypochromic anemia caused by iron deficiency d/t blood loss, poor
diet,malabsorption etc. blood film shows anisocytosis and poikilocytosis
vit b12 def= macrocytic anemia caused by vit b12 deficiency d/t dietary def,pernicious anemia,ileal
resection,gastrectomy.Blood film will show hypersegmented polymorphs

G6PD is x linked chief rbc enzyme defect disorder. it mainly affects males. it is percipitated by drugs like
primaquine,sulfonamides and aspirin.Usually asymptomatic.In attacks,there is rapid anemia and jaundice.
tests: fbc= anemia
Blood film= heinz bodies
G6PD enzyme activity - is the definitive test (as opposed to the amount of G6PD protein).

Treatment= Avoidance of the substances that may precipitate hemolysis is essential. If severe then transfuse.
if severe hemolysis, folate supplementation may be beneficial.

1402. A 65yo has terminal cancer and his pain is relieved by a fentanyl patch but he now complains of
shooting pain in his arm. Which of the following will add to his pain relief?
a. Gabapentin
b. Radiotherapy
c. Amitriptyline
d. Morphine

answer=A. Gabapentin.
it is the neuropathic pain and the first line treatment for neuropathic pain according to recent nice guidelines
is amitriptyline, duloxetine, gabapentin,pregabalin.if the first line drug treatment doesn't work try one of the
other 3 drugs

Gabapentin is 1st line for neuropathic pain. Amitriptyline is 1st line for diabetic neuropathy but it is off license
so now the first line for diabetic neuropathy is duloxetine. .

Neuropathic pain is defined as pain arising as a direct consequence of a lesion or disease affecting the
somatosensory system. it is often difficult to treat and responds poorly to standard analgesia.
The discomfort is usually of a chronic nature and may be described by the patient as a burning sensation, a
sharp, stabbing or shooting pain, or 'like an electric shock'.
examples include= diabetic neuropathy,trigeminal neuralgia,post herpetic neuralgia, prolapsed intervertebral
discs. There are various causes of it including infections like guillain barr syndrome, hiv etc, drugs like isoniazid,
vincristine, cisplatin,nitrofurantoine etc, malignancy.
treatment= pharmacological

All neuropathic pain (except trigeminal neuralgia): Offer a choice of amitriptyline, duloxetine, gabapentin or
pregabalin as initial treatment for neuropathic pain (except trigeminal neuralgia). If the initial treatment is not
effective or is not tolerated, offer one of the remaining three drugs, and consider switching again if the second
and third drugs tried are also not effective or not tolerated.

1403. A 45yo male alcoholic presents after a large hematemesis. He has some spider naevi on his chest,
BP=100/76mmHg, pulse=110bpm. He has a swollen abdomen with shifting dullness.
a. Gastric ca
b. Mallory-weiss tear+
c. Esophageal ca
d. Esophageal varices
e. Esophagitis
f. Peptic ulceration
Answer= esophageal varices. The most common causes of upper GI bleeding are peptic ulcer and oesophago-
gastric varices.Factors that increase the risk of variceal bleeding are the decompensation of liver disease
(ascites,bleeding esophageal varices,oedema etc),alcohol intake,aspirin,nsaids etc spider naevi is also one of
the signs of chronic liver disease.

exclusion of other causes:

peptic ulceration= although the most common causes of upper GI bleeding are peptic ulcer and oesophago-
gastric varices but Helicobacter pylori infection is associated with about 95% of duodenal ulcers and 80% of
gastric ulcers.other causes may include nsaids,pepsin,smoking,alcohol etc.symptoms commonly include
epigastric pain,nausea,dyspepsia etc

mallory weiss tear= Mallory-Weiss syndrome (MWS) is characterised by upper gastrointestinal bleeding (UGIB)
from mucosal lacerations in the upper gastrointestinal tract, usually at the gastroesophageal junction or
gastric cardia. Mallory Weiss tears account for 4-8% of cases of UGIB.
Haematemesis due to a Mallory Weiss tear usually occurs after a prolonged or forceful bout of retching,
vomiting, coughing, straining or even hiccupping.

Acute treatment of variceal haemorrhage


ABC
correct clotting: FFP, vitamin K
vasoactive agents: terlipressin is currently the only licensed vasoactive agent and is supported by NICE
guidelines. Octreotide may also be used.
prophylactic antibiotics reduce mortality in patients with liver cirrhosis
endoscopy: endoscopic variceal band ligation is superior to endoscopic sclerotherapy. NICE
recommend band ligation
Sengstaken-Blakemore tube if uncontrolled haemorrhage
Transjugular Intrahepatic Portosystemic Shunt (TIPSS) if above measures fail

Prophylaxis of variceal haemorrhage


propranolol: reduced rebleeding and mortality compared to placebo
endoscopic variceal band ligation (EVL) is superior to endoscopic sclerotherapy. It should be performed
at two-weekly intervals until all varices have been eradicated. Proton pump inhibitor cover is given to
prevent EVL-induced ulceration

1404. A 23yo woman presents with a 1cm small smooth, firm, mobile mass in her left breast. She is very
anxious. What is the most appropriate inv?
a. Mammography
b. US breast
c. FNAC
d. Mammography and US
Answer= US breast. In triple assessment for breast lump, ultrasound is done for <35yrs old and ultrasound and
mammography are done for >35ys old
Diagnosis=
Fibroadenoma:
it usually presents <30yrs.It is due to benign overgrowth of the collagenous mesenchyme of one breast
lobule..it is firm,smooth,mobile and painless lump. may be multiple.
regress, stay the same and get bigger.
investigations=
Ultrasound tends to be preferred in younger women with dense breasts, as mammograms are more difficult
to interpret in this group. Routine mammography, as a population screening tool, is not performed below the
age of 50 years. treatment= reassurance and observation. surgical excision if large.

triple assessment of breast lump:


1. clinical examination
2.radiology=us for <35yrs and us and mammography for >35yrs old
3. histology/cytology (fnac or core biopsy: u/s guided core biopsy is best for new lumps)
fnac is for cystic lesions and core biopsy for solid lumps.

1405. A pt was admitted with abdominal pain, diarrhea, pigmented palmar creases and buccal mucosa.
What is the most probable dx?
a. Addisons disease
b. Cushing syndrome
c. Pheochromocytoma
d. Hyperthyroidism
e. Hypoparathyroidism
answer= addisons disease. Unexplained abdominal pain and vomiting plus the pigmented palmar creases and
buccal mucosa are its main features pointing towards the diagnosis.

exclusion of other causes:


cushing syndrome= Cushing's syndrome is caused by prolonged exposure to elevated levels of either
endogenous or exogenous glucocorticoids (mainly cortisol)
main features include: truncal obesity, weight gain,buffalo hump,moon facies,facial plethora, gonadal
dysfunction (hirsutism,irregular menses,erectile dysfunction) etc
pheochromocytoma=catecholamine producing tumors causing hypertension, episodic headaches.
hyperthyroidism= increased levels of thyroid hormones. features of that.
hypoparathyroidism= features of hypocalcemia like cramps, paraesthesias, carpopedal spasm etc

addisons disease
it is the primary adrenocortical insufficiency. the destruction of adrenal cortex leads to the glucocorticoid
(cortisol) and mineralocorticoid (aldosterone) deficiency.
80% are due to the autoimmunity in uk. other causes may include TB (commonest cause worldwide),adrenal
metastasis,lymphoma, opportunistic infections in HIV, adrenal hemorrhage (waterhouse friderichsen
syndrome)

features=unexplained abdominal pain or vomiting, diarrhea/constipation, pigmented palmar creases and


buccal mucosa ( acth increases; crossreacts with melanin receptors),postural hypotension, vitiligo.
Tests:
NA decreased and potassium increased, calcium increased,low glucose, cortisol reduced. plasma renin and
aldosterone levels to assess mineralocorticoid status

short ACTH stimulation test= may be done to confirm the diagnosis


21 hydroxylase adrenal autoantibodies positive in autoimmune disease in >80%
treatment=
patient education
Hormone replacement: both mineralocorticoid and glucocorticoid should be replaced
glucocorticoid replacement= hydrocortisone is mainstay
mineralocorticoid replacement= fludrocortisone is used

1406. A 36yo pt came with diarrhea, bleeding, weight loss and fistula. What is the single most likely dx?
a. Colorectal ca
b. Celiac disease
c. CD
d. UC
e. IBS
Answer= C- crohn's disease. Diarrhea with weight loss and abdominal pain are the common symptom esp in
young patients these symptoms raise the suspicion of crohn's disease. extraintestinal manifestations include
anal fistulas, fissures and perianal abscess etc

exclusion of other causes:


colorectal ca=commonly occurs in people aged 65 or more.
celiac disease= peaks in infancy and 50-60yrs, diarrhea with weight loss or anemia, steatorrhea, bloating,
failure to thrive in children
Ulcerative colitis= chronic diarrhea with or without blood and mucus,crampy abdominal discomfort.
Extraintestinal features does not include fistula and fissures
IBS= no organic cause can be found in it. Only diagnose it if abdominal pain is either relieved by defecation or
associated with altered stool form and there are more than or equal to 2 of : urgency, incomplete evacuation,
abd bloating or distension, mucosal PR, worsening of symptoms after food. There is no weight loss in it

CROHNS DISEASE

Crohn's disease is a form of inflammatory bowel disease. It commonly affects the terminal ileum and colon but
may be seen anywhere from the mouth to anus.

Pathology
cause is unknown but there is a strong genetic susceptibility
inflammation occurs in all layers, down to the serosa. This is why patients with Crohn's are prone to
strictures, fistulas and adhesions

Crohn's disease typically presents in late adolescence or early adulthood. Features include:
presentation may be nonspecific symptoms such as weight loss and lethargy
diarrhoea: the most prominent symptom in adults. Crohn's colitis may cause bloody diarrhoea
abdominal pain: the most prominent symptom in children
perianal disease: e.g. Skin tags or ulcers
extra-intestinal features are more common in patients with colitis or perianal disease
'extraintestinal' features of inflammatory bowel diseases

Common to both Crohn's disease (CD)


and Ulcerative colitis (UC) Notes

Related to disease Arthritis: pauciarticular, asymmetric Arthritis is the most common extra-
activity intestinal feature in both CD and UC
Erythema nodosum
Episcleritis is more common in CD
Episcleritis

Osteoporosis

Unrelated to Arthritis: polyarticular, symmetric Primary sclerosing cholangitis is much


disease activity more common in UC
Uveitis
Uveitis is more common in UC
Pyoderma gangrenosum

Clubbing

Primary sclerosing cholangitis

1407. A 45yo man has undergone detox and now wants a drug to stop him from craving alcohol. What
med would be that drug of choice?
a. Disulfiram
b. Acamprosate
c. Thiamine
d. Naloxone
e. Diazepam
Answer= B. Acamprosate. After successful withdrawal, acamprosate or oral naltrexone can be considered in
combination with an individual psychological intervention in abstinence or for prevention of relapse.
Acamprosate may help intense anxiety, craving and insomnia.

exclusion of other options=


Disulfiram can be used to treat chronic alcohol dependence. it causes extremely unpleasant effects to any
alcohol ingestion e.g flushing, throbbing headache and palpitations.
Diazepam= it is used in the treatment of acute withdrawal of alcohol.

1408. A 68yo man awoke to find that he is unable to close his left eye and dribbling saliva from the left angle
of his mouth. What is the single most appropriate option?
a. Facial nerve
b. Glossopharyngeal nerve
c. Hypoglossal nerve
d. Optic nerve
e. Vagus nerve
answer= A. Facial nerve. features of 7th nerve palsy include drooling of saliva, failure to close the eye,
unilateral sagging of the mouth, absence of nasolabial fold, loss of taste in anterior 2/3rds of the tongue,
hyperacusis (hypersensitivity to sounds) (loss of innervation to stapedius)
exclusion of other nerves=
glossopharyngeal nerve
Contains sensory, motor (stylopharyngeus only) and parasympathetic fibres (salivary glands). it supplies tonsil,
palate and posterior third of tongue. so in its lesion there will be loss of taste on posterior third of tongue and
deviation of palate to one side.
Hypoglossal nerve=it supplies motor fibres to the tongue and most of the infrahyoid muscles. An LMN lesion
produces wasting of the ipsilateral side of the tongue, with fasciculation; and on attempted protrusion the
tongue deviates towards the affected side, but the tongue deviates away from the side of a central lesion.

optic nerve=.Visual field defects, pupillary abnormalities,


Optic neuritis (pain on moving the eye, loss of central vision, afferent pupillary defect, papilloedema),

The VIIth cranial (facial) nerve is largely motor in function (some sensory fibres from external acoustic meatus,
fibres controlling salivation and taste fibres from the anterior tongue in the chorda tympani branch). It also
supplies the stapedius (so a complete nerve lesion will alter auditory acuity on the affected side).

lower motor neuron facial nerve palsy - forehead affected so patient cant wrinkle the forehead
*upper motor neuron lesion 'spares' upper face so patient can wrinkle the forehead

1409. A 19yo female dx with trichomonas vaginalis. LMP was 10d ago. What is the best antibiotic tx?
a. Erythromycin
b. Vancomycin
c. Metronidazole
d. Penicillin
e. Clarithromycin
f. Doxycycline
g. Fluconazole
h. Clotrimazole

Answer= C. Metronidazole. treatment for trichomonas vaginalis is Metronidazole 2g po stat or 400mg/12h po


for 5 days (e.g if pregnant). it should be given to the partner too.
Trichomonas vaginalis is a highly motile, flagellated protozoan parasite

Features=
vaginal discharge: offensive and thin fishy smelly discharge, yellow/green, frothy
vulvovaginitis,strawberry cervix, pH > 4.5
in men is usually asymptomatic but may cause urethritis
investigations= microscopy of a wet mount shows motile trophozoites
1410. A 35yo man has been given a dx of allergic rhinitis and asthma. Exam: peripheral neuropathy with
tingling and numbness in a glove and stocking distribution. Skin lesions are present in the
form of tender subcutaneous nodules. The pt is responding well to corticosteroids. What is the
single most appropriate dx?
a. AS
b. Churg-strauss syndrome
c. Cryptogenic organizing
d. Extrinsic allergic alveolitis
e. Tropical pulmonary eosinophilia

Answer= B. churg- strauss syndrome. allergic rhinitis with asthma points towards the diagnosis of churg
strauss syndrome.
mnemonic : BEAN SAP

BE: Blood Eosinophilia


A : Asthma
N : Neuropathy (mononeuritis multiplex) - usually common peroneal nerve
S : Sinus abnormality
A : Allergies
P : Perivascular eosinophils / vasculitis

Churg Strauss Syndrome

A triad of adult-onset asthma, eosinophilia, and vasculitis


( vasospasm MI DVT), affecting lungs, nerves, heart, and skin.
A septic-shock picture/systemic inflammatory response syndrome may occur (with glomerulo nephritis/
renal failure, esp. if ANCA +ve).
Presentation:
The physical findings are specific to organ system involvement. There are three phases:
Allergic rhinitis and asthma.
Eosinophilic infiltrative disease, such as eosinophilic pneumonia or gastroenteritis.
Systemic medium and small vessel vasculitis with granulomatous inflammation.

Investigations: Antineutrophil cytoplasmic antibodies (ANCA): 70% of patients are perinuclear staining (p-
ANCA) positive (anti myeloperoxidase antibodies).Other likely findings include eosinophilia and anaemia on
the FBC; elevated ESR and CRP; elevated serum creatinine; increased serum IgE levels

Treatment= Steroids; biological agents if refractory disease,


eg rituximab.
1411. A 28yo woman comes with sudden onset vomiting and pain per abdomen. Exam: mobile
swelling in the right iliac fossa. What is the most probable dx?
a. Ectopic pregnancy
b. Tubo-ovarian abscess
c. Acute appendicitis
d. Ovarian torsion
e. Diverticulitis
Answer= D Ovarian Torsion. Mobile swelling in rif sudden pain vomiting in female

exclusion of other options:


ectopic pregnancy= history of amenorrhea and pv bleeding and it won't be palpable
tubo- ovarian abscess= history of fever present
acute appendicitis= mass wont be mobile in it rather will be fixed
diverticulitis= pain on the left side

OVARIAN TORSION
Ovarian torsion (adnexal torsion) is an infrequent but significant cause of acute lower abdominal pain in
women. This condition is usually associated with reduced venous return from the ovary as a result of stromal
edema, internal hemorrhage, hyperstimulation, or a mass. The ovary and fallopian tube are typically involved.

ETIOLOGY
Pregnancy is associated with, and may be responsible for, torsion in approximately 20% of adnexal torsion
cases
Ovarian tumors, both benign and malignant, are implicated in 50-60% of cases of torsion.
Dermoid tumors are most common.
patients with a history of pelvic surgery (principally tubal ligation) are at increased risk for torsion.

Features;
Classically, patients present with the sudden onset (commonly during exercise or other agitating movement)
of severe, unilateral lower abdominal pain that worsens intermittently over many hours.
The pain usually is localized over the involved side, often radiating to the back, pelvis, or thigh
Nausea and vomiting
Fever may occur as a late finding as the ovary becomes necrotic.

Investigations:
Diagnostic ultrasonography should be the first examination performed; typically, the affected ovary is
enlarged, with multiple immature or small follicles along its periphery.
Ultrasonography with color Doppler analysis is the method of choice for the evaluation of adnexal torsion

Treatment:
Outpatient care has no role in the treatment of ovarian torsion. Patients with either a suspected or confirmed
diagnosis of ovarian torsion should be admitted and either operated on or observed by a gynecologist.
Laparoscopy can be used for both confirmation of the diagnosis and treatment.
(source medscape)

1412. A 68yo man on tx for an irregular heart beat comes to the ED. He has palpitations for the last 3h. Exam:
pulse=regular, 154bpm. Carotid sinus massage settled his pulse down to 80bpm. What is the most likely
rhythm disturbance?
a. SVT
b. V-fib
c. VT
d. V-ectopics
e. A-fib
Answer= A.SVT Palpitations and dizziness, which are the most common symptoms reported in svt.
During an attack, tachycardia may be the only finding if the patient is otherwise healthy and there is no cardiac
dysfunction. During an episode of SVT the pulse rate is 140-250 beats per minute (bpm).In haemodynamically
stable regular narrow QRS-complex tachycardia, vagal manoeuvres - eg, Valsalva, carotid massage, facial
immersion in cold water.

1413. A 43yo man with a hx of hospital admissions talk about various topics, moving from one loosely
connected topic to another. What is the most likely dx?
a. Psychosis
b. Mania
c. Schizophrenia
d. Pressured speech
e. Verbal diarrhea
Answer= B. Mania. rapid flit from one subject to another ('flight of ideas') and pressured speech (speech is
rapid and copious) both are features of mania..

The presence of psychotic symptoms differentiates mania from hypomania

Psychotic symptoms
delusions of grandeur
auditory hallucinations

The following symptoms are common to both hypomania and mania

Mood
predominately elevated
irritable

Speech and thought


pressured
flight of ideas
poor attention

Behaviour
insomnia
loss of inhibitions: sexual promiscuity, overspending, risk-taking
increased appetite

1414. An 18yo girl presents with rahs on her trunk, abdominal pain, arthritis, proteinuria and
hematuria. What is the most probable dx?
a. TTP
b. ITP
c. HSP
d. HUS
e. Measles
Answer= C. HSP. Henoch-Schnlein purpura (HSP) is an IgA-mediated, autoimmune hypersensitivity vasculitis
of childhood. The main clinical features are skin purpura, arthritis, abdominal pain, gastrointestinal bleeding,
orchitis and nephritis. In classic cases palpable purpuric rash (with localized oedema) over buttocks and other
sites are extensor surfaces of arms and legs and trunk. Features of IgA nephropathy may occur e.g.
haematuria, renal failure.

exclusion of other causes:


HUS=Haemolytic uraemic syndrome is generally seen in young children and produces a triad of:acute renal
failure, microangiopathic haemolytic anaemia and
thrombocytopenia. Typical (or infection-induced) HUS is most commonly associated with Escherichia coli with
somatic (O) antigen 157 and flagella (H) antigen 7 - hence the designation O157:H7.The classical presenting
feature is profuse diarrhoea that turns bloody 1 to 3 days later.

Henoch-Schnlein purpura (HSP) is an IgA-mediated, autoimmune hypersensitivity vasculitis of childhood.


The main clinical features are skin purpura, arthritis, abdominal pain, gastrointestinal bleeding, orchitis and
nephritis. In classic cases palpable purpuric rash (with localized oedema) over buttocks and other sites are
extensor surfaces of arms and legs and trunk. Features of IgA nephropathy may occur e.g. haematuria, renal
failure.
Treatment:
analgesia for arthralgia
treatment of nephropathy is generally supportive. There is inconsistent evidence for the use of steroids and
immunosuppressants
Prognosis:
usually excellent, HSP is a self-limiting condition, especially in children without renal involvement
around 1/3rd of patients have a relapse
HSP = 10 P's
PALPABLE
PURPURA
PLATELETS OK -- DDx from ITP
PRURITUS -- URTICARIA
PAIN -- ABDOMEN and LEGS
POSITIVE +guaiac -- FECES
PRESSURE -- increased blood pressure
PROTEINURIA
PREDNISONE = Tx

1415. A pt is on loop diuretics. What effect do loop diuretics produce?


a. Low Na+, low K+
b. Low Na+, normal K+
c. Normal Na+, normal K+
d. High Na+, low K+
e. High Na+, high K+
Answer= A. low na+, Low K+

Loop Diuretics

loop diuretics act by inhibiting the Na-K-Cl cotransporter (NKCC) in the thick ascending limb of the loop of
Henle, reducing the absorption of NaCl.
There are two variants of NKCC; loop diuretics act on NKCC2, which is more prevalent in the kidneys.

Indications= heart failure: both acute (usually intravenously) and chronic (usually orally), resistant
hypertension, particularly in patients with renal impairment.

Adverse effects= hypotension, hyponatremia, hypokalemia,hypocalcemia,hypochloremic alkalosis,


hyperglycemia (less common with thiazides), renal impairment( from dehydration+direct toxic effect) and gout

1416. A 6yo girl is being investigated for renal failure. She is found to have a congenital abnormality of the
insertion of the ureters into the urinary bladder. What is the single most likely cause for renal failure in this pt?
a. SLE
b. PKD
c. Wilms tumor
d. Acute tubular necrosis
e. Reflux nephropathy
Answer= Reflux Nephropathy

1417. A 76yo man is in the CCU 2d after an acute MI. He tells you that he had an episode of rapid pounding in
the chest lasting for about 2mins. He remains conscious throughout. What is the most likely rhythm?
a. SVT
b. VF
c. VT
d. V-ectopics
e. A-fib
Answer= C. Ventricular tachycardia. Palpitations (pounding chest) or arrhythmias 48hrs post MI is almost
always Ventricular tachycardia until proven otherwise.

Ventricular tachycardia (VT) is broad-complex tachycardia originating from a ventricular ectopic focus. It has
the potential to precipitate ventricular fibrillation and hence requires urgent treatment.
There are two main types of VT:
monomorphic VT: most commonly caused by myocardial infarction
polymorphic VT: A subtype of polymorphic VT is torsades de pointes which is precipitated by
prolongation of the QT interval.
Management:
If the patient has adverse signs (systolic BP < 90 mmHg, chest pain, heart failure or rate > 150 beats/min) then
immediate cardioversion is indicated.
In the absence of such signs antiarrhythmics may be used.
If these fail, then electrical cardioversion may be needed with synchronised DC shocks.
Drug therapy:
amiodarone: ideally administered through a central line
lidocaine: use with caution in severe left ventricular impairment
procainamide
Verapamil should NOT be used in VT
If drug therapy fails
electrophysiological study (EPS)
implantable cardioverter-defibrillator (ICD) - this is particularly indicated in patients with significantly
impaired LV function

1418. A 49yo man comes with hx of cough and SOB. His CD4 count is measured as 350. CXR shows lobar
consolidation. What is the single most appropriate option?
a. Mycobacterium avium intracellulare
b. CMV
c. Streptococcus
d. Toxoplasmosis
e. Pneumocystis jiroveci
Answer= C Streptococcus. The patient presents with h/o cough. SOB and cxr shows lobar consolidation. these
point towards the diagnosis of pneumonia and streptococcus is the most common cause of community
acquired pneumonia.

exclusion of other options:


pneumocystis Jiroveci (PCP): PCP is the most common opportunistic infection in AIDS which usually occur
when CD4 count <200/mm. all patients with a CD4 count < 200/mm should receive PCP prophylaxis

Community acquired pneumonia (CAP) may be caused by the following infectious agents:
Streptococcus pneumoniae (accounts for around 80% of cases)
Haemophilus influenzae
Staphylococcus aureus: commonly after the 'flu
atypical pneumonias (e.g. Due to Mycoplasma pneumoniae)
viruses

Klebsiella pneumoniae is classically in alcoholics


Streptococcus pneumoniae (pneumococcus) is the most common cause of community-acquired pneumonia

Clinical features
Symptoms: Fever, rigors, malaise, anorexia, dyspnoea, cough, purulent sputum, haemoptysis, and pleuritic
pain.
Signs: Pyrexia, cyanosis, confusion (elderly usually), tachypnoea, tachycardia, hypotension, signs of
consolidation (diminished expansion, dull percussion note, tactile vocal fremitus/vocal resonance, bronchial
breathing), and a pleural rub.

Tests:
CXR: lobar or multilobar infiltrates, cavitation or pleural effusion.
Assess oxygenation: oxygen saturation (ABGs if SaO2 <92% or severe pneumonia) and BP.
Blood tests: FBC, U&E, LFT, CRP, blood cultures. Sputum for microscopy and culture.
Pleural fluid may be aspirated for culture.
Consider bronchoscopy and bronchoalveolar lavage if patient is immunocompromised or on
ITU.
Management:
CURB-65 criteria of severe pneumonia
Confusion (abbreviated mental test score <= 8/10)
Urea > 7 mmol/L
Respiratory rate >= 30 / min
BP: systolic <= 90 or diastolic <= 60 mmHg
age >= 65 years

Patients with 3 or more (out of 5) of the above criteria are regarded as having a severe pneumonia
The British Thoracic Society published guidelines in 2009:
low or moderate severity CAP: oral amoxicillin. A macrolide should be added for patients admitted to
hospital
high severity CAP: intravenous co-amoxiclav + clarithromycin OR cefuroxime + clarithromycin OR
cefotaxime + clarithromycin
the current BNF has slightly different recommendations for high severity CAP: intravenous
benzylpenicillin + clarithromycin OR benzylpenicillin + doxycycline. For 'life-threatening' infections the
BNF recommends the same as the BTS guidelines for high-severity CAP

1419. A 32yo woman with prv hx of PID now presents with severe abdominal pain. Her LMP was 8wks ago.
What is the most probable dx?
a. Ectopic pregnancy
b. Ovarian torsion
c. Hematometrium
d. Chronic PID
e. Cholecystitis
Answer= A. Ectopic Pregnancy. typical history of 8 weeks amenorrhea with lower abdominal pain. PID is one of
its risk factors.
Ectopic pregnancy:
An ectopic pregnancy is one that occurs anywhere outside the uterus. By far the most common place for
ectopic pregnancy is the Fallopian tubes.

Risk Factors: Fertility treatments and intrauterine contraceptive devices (IUCDs) are the most important
associated risk factors. Pelvic inflammatory disease may cause complete tubal occlusion or delay the transport
of the embryo so that implantation occurs in the tube. Adhesions from infection and inflammation from
endometriosis may play a part.

presentation:
Symptoms and signs of ectopic pregnancy can resemble those of other more common conditions, including
urinary tract infections and gastrointestinal conditions.The most common symptoms are= Abdominal
pain.Pelvic pain.Amenorrhoea or missed period (typically 6 to 8 weeks) Vaginal bleeding (with or without
clots).

Investigations: urine pregnancy test should be performed in all the women of childbearing age presenting
with lower abd pain. The most accurate method to detect a tubal pregnancy is transvaginal ultrasound.This
can identify the location of the pregnancy and also whether there is a fetal pole and heart beat.

Human chorionic gonadotrophin (hCG) levels are performed in women with pregnancy of unknown location
who are clinically stable. hCG levels are taken 48 hours apart.

Treatment: Anti D prophylaxis to all rhesus negative women. Surgical: if presents with shock then Urgent
Laparotomy. otherwise laparoscopy is
preferable. A salpingectomy should be performed, unless
the woman has other risk factors for infertility, in which case a salpingotomy should be undertaken.

1420. A 25yo who is 38wks pregnant presents to the labour ward with a hx of fewer fetal movements than
usual during the evening. She also says that abdominal contractions are coming very few minutes and she is
having a blood stained show per vagina for the last few minutes. Exam: cervix is fully effaced, 9cm dilated,
cephalic presentation and station is +1. Choose the single most likely dx?
a. APH
b. Concealed hemorrhage
c. Labour
d. IUFD
e. IUGR
Answer= C.Labour. Normal labour occurs after 37 weeks gestation. This is the first stage of labour as there are
contractions and cervix is fully effaced.
NORMAL LABOUR
Normal labour occurs after 37wks gestation and results in spontaneous
vaginal delivery of the baby within 24h of the onset of regular spontaneous contractions.
It is often heralded by a show, ie a plug of cervical mucus and a little
blood as the membranes strip from the os (membranes may then rupture).

The first stage of labour


Latent phase (not necessarily continuous):
there are painful contractions, the cervix initially effaces (becomes shorter and
softer) then dilates to 4cm.
Established phase:
contractions with dilatation from 4 cm. A satisfactory rate of dilatation from 4 cm is 0.5cm/h.

The 1st stage generally takes 818h in a primip, and 512h in a multip.

During the first stage check maternal BP, and T 4-hourly, pulse hourly;
assess the contractions every 30min, their strength and their frequency (ideally 34 per 10min, lasting up to 1
min).
Offer vaginal examination e.g every 4h to assess the degree of cervical dilatation, the position and the station
of the head.
Auscultate fetal heart rate (if not continuously monitored), by Pinard or Doppler every 15min, listening for
1min after a contraction.

The second stage:


Passive stage is complete cervical dilatation (but no desire to push).
In active stage, the baby can be seen, there is full dilatation with expulsive contractions and maternal effort
(using abdominal muscles and the Valsalva manoeuvre until the baby is born)

The third stage is delivery of the placenta.


As the uterus contracts to a <24-week size after the baby is born, the placenta separates from the uterus
through the spongy layer of the decidua basalis. It then buckles and a small
amount of retroplacental haemorrhage aids its removal.

1421. A 30yo woman has a painless lump in the outer aspect of her left breast. She has had a prv breast lump.
Her grandmother had breast cancer at 70yrs. She has a 1 cm smooth, firm, discrete, mobile lump in the other
quadrant region of the left breast. What is the single most likely dx?
a. Breast abscess
b. Breast carcinoma
c. Breast cyst
d. Fibroadenoma
e. Sebaceous cyst

Answer= Fibroadenoma= age is 30yrs, painless,mobile,smooth and firm lump.

Fibroadenoma:
it usually presents <30yrs.It is due to benign overgrowth of the collagenous mesenchyme of one breast
lobule..it is firm,smooth,mobile and painless lump. may be multiple.
regress, stay the same and get bigger.
investigations=
Ultrasound tends to be preferred in younger women with dense breasts, as mammograms are more difficult
to interpret in this group. Routine mammography, as a population screening tool, is not performed below the
age of 50 years. treatment= reassurance and observation. surgical excision if large.

1422. A 38yo woman is in the ED following an OD of her meds. She doesnt need med tx for the OD. She says
she wishes to be discharged. What is the single most appropriate management?
a. Community psychiatric nurse visit
b. Psychiatric OPD review the next day
c. Prescribe anti-depressants
d. Admission under the mental health act
e. Discharge and allow to go home
Answer: D. Admission under the mental health act. Patient has overdosed, she is suicidal and can do the same
act again so she should not be discharged and she should be admitted for her mental health assessment.

1423. A 63yo male presents after having had a seizure. Exam: alert and oriented. Exam: inattention on
the left side and hyperreflexia of the arm. What is the most probable dx?
a. Cerebral tumor
b. Pituitary adenoma
c. Cerebellar abscess
d. Huntingtons chorea
e. Parkinsonism
Answer: A. Cerebral tumor. H/o seizure and inattention on left side and hyperreflexia of the arm all point
towards the diagnosis.

Primary brain tumours represent about 2% of all tumours diagnosed in the UK.

The presentation will depend on location and rate of growth but includes features of a space-occupying lesion
and raised intracranial pressure (ICP):
Headache, which is typically worse in the mornings.
Nausea and vomiting.
Seizures.
Progressive focal neurological deficits - eg, diplopia associated with a cranial nerve defect, visual
field defect, neurological deficits affecting the upper and/or lower limb.
Cognitive or behavioural symptoms.
Symptoms relating to location of mass - eg, frontal lobe lesions associated with personality
changes, disinhibition and parietal lobe lesions might be associated with dysarthria.
Papilloedema (absence of papilloedema does not exclude a brain tumour).

Diagnosis:
largely rests on brain imaging - eg, CT scan and/or MRI scan (both with or without contrast). MRI is more
sensitive.

Management:
Surgery= Tumours should be resected whenever possible but some may be inaccessible.
Surgery should also be considered to reduce mass effect and treat hydrocephalus in order to
provide symptomatic relief.
If surgery is not an option then radiotherapy should be considered.
Radiotherapy= prolongs survival.
Chemotherapy= Its role is not as marked as in other tumors. Chemo-radiotherapy is used post-op for gliomas
or metastases.
Complications:
Acute haemorrhage into a tumour.
Blockage of cerebrospinal fluid outflow, causing hydrocephalus.
Sudden increases in ICP may lead to life-threatening brain herniation through the foramen magnum or
transtentorial foramina.
Complications of radiotherapy.
1424. A 70yo lady on Raloxifene for osteoporosis has recently to the UK from Australia. She now presents with
severe chest pain, SOB and suddenly collapsed in the ED. What is the single most appropriate dx?

a. MI

b. Aortic dissection

c. Pulmonary embolism
d. Costochondritis
e. Pneumothorax
Answer: C. Pulmonary Embolism. patient is on Raloxifene which is selective estrogen receptor modulator and
it increases the risk of thromboembolic events.

PULMONARY EMBOLISM
Always suspect pulmonary embolism (PE) in sudden collapse 12wks after surgery.

Mechanism= Venous thrombi, usually from DVT, pass into the pulmonary circulation
and block blood flow to lungs. The source is often occult.

Risk factors
Malignancy.
Surgeryespecially pelvic and lower limb (much lower if prophylaxis used).
Immobility.
Combined oral contraceptive pill (there is also a slight risk attached to HRT).
Previous thromboembolism and inherited thrombophilia.

Signs and symptoms


Acute dyspnoea, pleuritic chest pain, haemoptysis, and syncope.
Hypotension, tachycardia, gallop rhythm, JVP, loud P2, right ventricular heave,
pleural rub, tachypnoea, cyanosis, AF.

Investigation: (2012 NICE guidelines)

All patients with symptoms or signs suggestive of a PE should have a history taken, examination performed
and a chest x-ray to exclude other pathology.

If a PE is still suspected a two-level PE Wells score should be performed:

Clinical feature Points

Clinical signs and symptoms of DVT (minimum of leg swelling and pain with palpation of the deep 3
veins)
An alternative diagnosis is less likely than PE 3

Heart rate > 100 beats per minute 1.5

Immobilisation for more than 3 days or surgery in the previous 4 weeks 1.5

Previous DVT/PE 1.5

Haemoptysis 1

Malignancy (on treatment, treated in the last 6 months, or palliative) 1

Clinical probability simplified scores


PE likely - more than 4 points
PE unlikely - 4 points or less

If a PE is 'likely' (more than 4 points) = immediate computed tomography pulmonary angiogram (CTPA).
If there is a delay in getting the CTPA = give low-molecular weight heparin until the scan is performed.
If a PE is 'unlikely' (4 points or less) = D-dimer test.
If this is positive= immediate computed tomography pulmonary angiogram (CTPA).
If there is a delay in getting the CTPA= give low-molecular weight heparin until the scan is performed.
If the patient has an allergy to contrast media or renal impairment a V/Q scan should be used instead of a
CTPA.
Management: (NICE guidelines)
Low molecular weight heparin (LMWH) or fondaparinux should be given initially after a PE is diagnosed
(except in massive PE)
a vitamin K antagonist (i.e. warfarin) should be given within 24 hours of the diagnosis.
Low molecular weight heparin (LMWH)= upto 5 days or until INR >2.
warfarin should be continued for at least 3 months.
NICE advise extending warfarin beyond 3 months for patients with unprovoked PE. This essentially means that
if there was no obvious cause or provoking factor (surgery, trauma, significant immobility) it may imply the
patient has a tendency to thrombosis and should be given treatment longer than the norm of 3 months
for patients with active cancer NICE recommend using LMWH for 6 months.
Thrombolysis is now recommended as the first-line treatment for massive PE where there is circulatory failure
(e.g. hypotension).

1425. A 35yo woman complains of hoarseness of voice 3h after partial thyroidectomy. She had no hx of
phonation probs before the surgery. What is the single most appropriate inv?
a. Laryngoscopy
b. Bronchoscopy
c. CT neck
d. CXR
e. Barium swallow
Answer= A.Laryngoscopy. h/o thyroidectomy which may lead to recurrent laryngeal nerve injury resulting in
hoarseness of voice or breathlessness. so laryngoscopy is done to look for vocal cords.

THYROIDECTOMY
Possible complications following thyroid surgery include:

Minor complications such as collections of serous fluid (they resolve spontaneously if small and asymptomatic
but may require single or repeated aspiration if large) and poor scar
formation. Bleeding, which may cause tracheal compression.

Recurrent laryngeal nerve injury: Innervates all of the intrinsic muscles of the larynx, except the cricothyroid
muscle. Patients with unilateral vocal fold paralysis
present with postoperative hoarseness. Bilateral vocal fold
paralysis may occur following a total thyroidectomy and usually presents immediately after
extubation. Hypoparathyroidism
Thyrotoxic storm Superior
laryngeal nerve injury:. Trauma to the nerve results in an inability to lengthen a vocal fold and thus to create a
higher-pitched sound.The external branch is probably the most commonly injured nerve in thyroid surgery.
Most patients do not notice any change but the problem may be career-ending for a professional
singer. Infection.
Hypothyroidism

1426. A 40yo pt came to OPD with complaint of fever, pleuritic chest pain, productive cough and painful
vesicles around the lips. Exam: temp=38C. He has a hx of splenectomy last yr. What is the single most likely
causative organism?

a. Pneumococcal pneumonia
b. Staphylococcus
c. Klebsiella
d. Streptococcus
e. Chlamydia psitacci

Answer= A Pneumococcal pneumonia. the patient has a history of splenectomy and pts with splenectomy are
more prone to develop pneumococcus pneumonia that is why they are given vaccines prophylactically.

Community acquired pneumonia (CAP) may be caused by the following infectious agents:
Streptococcus pneumoniae (accounts for around 80% of cases)
Haemophilus influenzae
Staphylococcus aureus: commonly after the 'flu
atypical pneumonias (e.g. Due to Mycoplasma pneumoniae)
viruses

Klebsiella pneumoniae is classically in alcoholics


Streptococcus pneumoniae (pneumococcus) is the most common cause of community-acquired pneumonia

Clinical features
Symptoms: Fever, rigors, malaise, anorexia, dyspnoea, cough, purulent sputum, haemoptysis, and pleuritic
pain.
Signs: Pyrexia, cyanosis, confusion (elderly usually), tachypnoea, tachycardia, hypotension, signs of
consolidation (diminished expansion, dull percussion note, tactile vocal fremitus/vocal resonance, bronchial
breathing), and a pleural rub.

Tests:
CXR: lobar or multilobar infiltrates, cavitation or pleural effusion.
Assess oxygenation: oxygen saturation (ABGs if SaO2 <92% or severe pneumonia) and BP.
Blood tests: FBC, U&E, LFT, CRP, blood cultures. Sputum for microscopy and culture.
Pleural fluid may be aspirated for culture.
Consider bronchoscopy and bronchoalveolar lavage if patient is immunocompromised or on
ITU.
Management:
CURB-65 criteria of severe pneumonia
Confusion (abbreviated mental test score <= 8/10)
Urea > 7 mmol/L
Respiratory rate >= 30 / min
BP: systolic <= 90 or diastolic <= 60 mmHg
age >= 65 years

Patients with 3 or more (out of 5) of the above criteria are regarded as having a severe pneumonia
The British Thoracic Society published guidelines in 2009:
low or moderate severity CAP: oral amoxicillin. A macrolide should be added for patients admitted to
hospital
high severity CAP: intravenous co-amoxiclav + clarithromycin OR cefuroxime + clarithromycin OR
cefotaxime + clarithromycin
the current BNF has slightly different recommendations for high severity CAP: intravenous
benzylpenicillin + clarithromycin OR benzylpenicillin + doxycycline. For 'life-threatening' infections the
BNF recommends the same as the BTS guidelines for high-severity CAP

1427. A 37yo male pt who recently returned back to UK from UAE attends the OPD with complaint of dry
cough, breathlessness and anorexia. According to him he had flu like symptoms a week ago. He is slightly
confused. Inv: lymphopenia & decreased Na+. CXR: bi-basal consolidation. What is the single most likely
causative organism?
a. Legionella
b. Chlamydia pneumoniae
c. PCP
d. Viral pneumonia
e. Chlamydia psittaci
Answer= A. legionella. travel history,flue like symptoms with dry cough and dyspnea, confusion. chest x ray
showing bi-basal consolidation. blood tests showing lymphopenia and hyponatremia are all features of
pneumonia caused by Legionella pneumophila.

LEGIONELLA PNEUMOPHILA
it colonizes water tanks kept at <60C (hotel air conditioning and hot water systems) causing outbreaks of
legionnaires disease.

flu like symptoms (fever,malaise,myalgia) precede dry cough and dyspnea.


Extra pulmonary features include anorexia, hepatitis,renal failure,confusion and coma.

CXR shows bi- basal consolidation. blood tests show lymphopenia and hyponatremia and deranged LFTs.
urinalysis may show hematuria.

Diagnosis= Legionella urine antigen/culture

Treatment= fluoroquinolone for 2-3 weeks or clarithromycin.

1428. A 20yo student came to the OPD with complains of headache, malaise, dry cough, joint pain and
vomiting. Exam: temp=39C. CXR: patchy consolidation. What is the single most likely causative organism?
a. Pneumococcal pneumonia
b. Mycoplasma
c. Klebsiella
d. Streptococcus
e. PCP
Answer= B. Mycoplasma. flu like symptoms followed by dry cough. chest xray shows patchy consolidation
often of one lower lobe or reticulo nodular shadowing.

MYCOPLASMA PNEUMONIA
occurs in epidemics about every 4yrs. it presents insidiously with flu like symptoms (fever, malaise,arthralgia)
followed by a dry cough.
CXR= reticulo nodular shadowing and patchy consolidation often of one lower lobe.
Diagnosis= PCR sputum or serology
Treatment= clarithromycin (500mg/12h) or doxycycline or a fluoroquinolone

Complications= skin rash (erythema multiforme), stevens johnson syndrome, meningoencephalitis or myelitis,
guillain- barre syndrome

1429. A 45yo man presented to his GP with vague symptoms of headache, proximal muscle weakness and
nocturia. Test results show him to be severely HTN (230/130mmHg) and hypokalemic. What is the most
probable dx?
a. Addisons disease
b. Conns disease
c. Familial hyperaldosteronism
d. Cushings disease
e. Cushings syndrome
Answer: B. Conns Disease. proximal muscle weakness, nocturia, HTN and hypokalemia are the features of
conns disease which is due to excessive levels of aldosterone.
PRIMARY HYPERALDOSTERONISM

Primary hyperaldosteronism is excess production of aldosterone, independent of


the reninangiotensin system, causing sodium and water retention, and renin release.

Consider if the following features: hypertension, hypokalaemia or alkalosis in


someone not on diuretics. Sodium tends to be mildly raised or normal.
Symptoms:
signs of hypokalemia, weakness (even quadriparesis), cramps, paresthesia, polyuria, polydipsia. raised blood
pressure.

Causes: ~ are due to a solitary aldosterone-producing adenoma (Conns syndrome).


~ are due to bilateral adrenocortical hyperplasia.
Rare causes: adrenal carcinoma or glucocorticoid-remediable aldosteronism (GRA).

Investigations:
high serum aldosterone
low serum renin
high-resolution CT abdomen
adrenal vein sampling

Management
adrenal adenoma: surgery (laparoscopic adrenalectomy)
bilateral adrenocortical hyperplasia: aldosterone antagonist e.g. spironolactone
1430. A man says his insides are rotting and nobody has buried him. Which term best describes his condition?
a. Delusion of nihilism
b. Delusion of guilt
c. Delusion of persecution
d. Incongruent affect
e. Clang association
Answer: A. Delusion of Nihilism. these are the delusions in which people deny their existance and believe that
they are rotting or they demand burial.

Cotards syndrome (nihilistic delusions)


We deny our existence, or believe we are rotting, or we demand burial, thinking we are a corpse.
Cause: Depression, alcohol, syphilis, parietal lobe lesion, or just being born.

DELUSIONS
A delusion is a false belief which is firmly sustained and based on incorrect inference about reality. This belief
is held despite evidence to the contrary and is not accounted for by the person's culture or religion.

Types of Delusions:
Monothematic - delusions are only relating to one particular topic.
Polythematic - range of delusional topics (seen in schizophrenia).

They can also be classified as:


Primary - occur in the mind, fully formed with no preceding reasons; strongly suggestive of schizophrenia.
Secondary - eg depressed person feeling worthless.
Examples:
Delusional jealousy (Othello's syndrome) - eg believing a partner is being unfaithful.

Capgras' delusion - belief that a close relative has been replaced by someone else who looks the
same.
Unilateral neglect - belief that one limb or side does not exist.
Thought insertion - belief that someone is putting thoughts into the brain.
Grandiose delusion - belief of exaggerated self-worth.

1431. A man with chronic cough presents with copious purulent sputum. What is the single most dx?

a. Bronchitis
b. Bronchiectasis
c. COPD
d. Pneumonia
e. Emphysema
Answer= b. Bronchiectasis. Copious purulent sputum with persistent cough are classical features of
bronchiectasis.

exclusion of other causes:


COPD= it includes chronic bronchitis and emphysema. symptoms include chronic cough but it is not
copious,dyspnea.usually there is h/o smoking.
Bronchiectasis is described as permanent dilatation of the airways secondary to chronic infection or
inflammation.
Most common organisms isolated from patients with bronchiectasis:
Haemophilus influenzae (most common), Pseudomonas aeruginosa, Klebsiella spp, Streptococcus pneumoniae
Causes: post-infective:( tuberculosis, measles, pertussis, pneumonia) cystic fibrosis, bronchial obstruction (e.g.
lung cancer/foreign body), immune deficiency: (selective IgA, hypogammaglobulinemia), allergic
bronchopulmonary aspergillosis (ABPA), ciliary dyskinetic syndromes: (Kartagener's syndrome, Young's
syndrome), yellow nail syndrome
features= persistent cough, purulent copious sputum,intermittent hemoptysis. finger clubbing, coarse
inspiratory crepitations, wheeze ( all found in other diseases)
Tests: sputum culture. chest xray shows cystic shadows, thickened bronchial walls (tramline and ring
shadows). other investigations according to the causes.

Management:
postural drainage,
chest physiotherapy
antibiotics usually for acute exacerbations
surgery in selected cases (e.g localized disease,to control severe hemoptysis)

1432. A 32yo man working in a shipyard comes with SOB. Exam: dullness on left side of the chest, pain
in left side of chest, pleuritic rub and crackles been heard on the same side. What is the single
most likely dx?
a. Pericarditis
b. Pleurisy
c. Pleural effusion
d. CCF
e. TB
Answer= C. Pleural effusion. shipyard worker therefore exposure to asbestos which leads to mesothelioma
causing pleural effusion. dyspnea, pleuritic chest pain, decreased chest expansion, stony dull percussion note,
diminished breath sounds on affected side are its features.

Pleural Effusion is the fluid in the pleural space.


effusions can be divided by their protein concentration into transudates (<30g/l) and exudates (>30g/l)
Causes=
Transudates: commonly heart failure, cirrhosis, hypoalbuminemia, peritoneal dialysis, constrictive pericarditis,
hypothyroidism, nephrotic syndrome etc
Exudative: pneumonia, malignancy (bronchogenic ca, mesothelioma, lymphoma etc), TB, rheumatoid
arthritis,pulmonary infarction, pancreatitis, SLE etc

Features= asymptomatic or dyspnea,pleuritic chest pain, decreased chest expansion, stony dull percussion and
diminished breath sounds on the affected side. features of other diseases may be present.
Tests= Chest X-ray: this is the first investigation if a pleural effusion is suspected clinically. small effusions
blunt the costophrenic angles. large are seen as water dense shadows with concave upper borders. Bilateral
effusions with an enlarged heart shadow are commonly caused by congestive cardiac failure.

Ultrasound= done to identify the presence of fluid and in guiding therapeutic or diagnostic aspiration.
Diagnostic aspiration and pleural biopsy are also done.
Management:
Small effusions that are not causing respiratory embarrassment may be managed by observation.
symptomatic effusions= drainage. fluid is best removed slowly (0.5 -1.5L/hr)
Pleurodesis with tetracycline, bleomycin or talc may be helpful for recurrent effusions.
surgery= persistent collections and increasing pleural thickness (on U/S) require surgery

1433. A 67yo female presents with balance prbs. Exam: nystagmus on left lateral gaze, a loss of the let corneal
reflex and reduced hearing in the left ear. What is the most likely dx?
a. Menieres disease
b. Acoustic neuroma
c. Cerebral abscess
d. Pituitary tumor
e. Gentamicin
Answer: B. Acoustic Neuroma. Balance problems, nystagmus, loss of corneal reflex and reduced unilateral
hearing points towards the diagnosis.

Acoustic neuromas (more correctly called vestibular schwannomas) account for approximately five percent of
intracranial tumours and 90 percent of cerebellopontine angle.

These are typically indolent, histologically benign subarachnoid tumours


that cause problems by local pressure, and then behave as space-occupying
lesions

Features can be predicted by the affected cranial nerves


cranial nerve VIII: hearing loss, vertigo, tinnitus
Progressive ipsilateral tinnitus sensorineural deafness (cochlear nerve compression).
cranial nerve V: absent corneal reflex
cranial nerve VII: facial palsy

Tests:
MRI of the cerebellopontine angle is the investigation of choice

Treatment:
Surgery: (difficult, and often not needed, eg if elderly)
Hearing and the facial nerve can be preserved
(eg continuous intraoperative monitoring of evoked electromyograms).
Gamma knife surgery is one good way to neuroma volume.

1434. A 22yo man reports a 2d hx of hoarseness of voice. He denies any weight loss but he has been smoking
for 4yrs. What is the single most appropriate inv?
a. None
b. Laryngoscopy
c. Bronchoscopy
d. BAL
e. CXR
Answer: A. None. Young patient, only presents with a 2 day h/o hoarseness of voice and denies weight loss so
CA is less likely and there is no need to investigate.
HOARSENESS OF VOICE
Hoarseness entails difficulty producing sound with change in voice pitch or quality (breathy,
scratchy, husky).
Investigate hoarseness (esp in smokers) lasting >3wks, as it is the chief (and
often the only) presentation of laryngeal carcinoma.

Ask about:
Gastrooesophageal reflux (GORD), dysphagia, smoking, stress, singing & shouting. Voice overuse is a
common cause (prevalence >50% in eg in teachers).

Tests:
Laryngoscopy (to assess cord mobility, inspect the mucosa, exclude local causes);
slowmotion videolaryngostroboscopy/acoustic analysis
(causes have characteristic vibration patterns).

1435. A 34yo IVDA presents with a 4m hx of productive cough. He has lost 10kgs. What is the single
most appropriate inv?
a. Sputum for AFB
b. Laryngoscopy
c. Bronchoscopy
d. CT neck
e. CXR
Answer= A. sputum for AFB. chronic productive cough and h/o weight loss points towards the diagnosis of
tuberculosis so sputum for AFB should be done.

Tuberculosis (TB) is an infection caused by Mycobacterium tuberculosis that most commonly affects the lungs.

primary tuberculosis=
A nonimmune host who is exposed to M. tuberculosis may develop primary infection of the lungs. A small lung
lesion known as a Ghon focus develops. The Ghon focus is composed of tubercle-laden macrophages. The
combination of a Ghon focus and hilar lymph nodes is known as a Ghon complex.
In immunocompetent people the initially lesion usually heals by fibrosis. Those who are immunocompromised
may develop disseminated disease (miliary tuberculosis).
secondary tuberculosis=
If the host becomes immunocompromised the initial infection may become reactivated. Reactivation generally
occurs in the apex of the lungs and may spread locally or to more distant sites. Possible causes of immuno
compromise include: immunosuppressive drugs like steroids, Hiv, malignancy.
The lungs remain the most common site for secondary tuberculosis. Extrapulmonary infection may occur in
the following areas: central nervous system (tuberculous meningitis - the most serious complication),
vertebral bodies (Pott's disease), cervical lymph nodes (scrofuloderma), renal, gastrointestinal tract.
risk factors= close contacts of TB patient, alcoholics and other drug abusers, hiv positive and other
immunocompromised patients.etc

presentation=
general symptoms: fever, malaise, night sweats, fatigue, weight loss etc
pulmonary Symptoms= include chronic, productive cough with purulent bloodstained sputum. May result in
lobar collapse, bronchiectasis, pleural effusion and pneumonia.

1436. A 25yo pt came to the OPD with complaint of fever, malaise, breathlessness, cough and anorexia. His gf
has got similar symptoms. He had hx of sore throat and ear discharge a month ago. What
is the single most likely causative organism?
a. Legionella
b. Mycoplasma
c. Chlamydia pneumoniae
d. PCP
e. Chlamydia psittaci
Answer: C.chlamydia pneumoniae. person to person contact, h/o sore throat and ear discharge are diagnostic
points.

Chlamydophila pneumoniae is the commonest chlamydial infection. person to person spread occurs. features
include pharyngitis, hoarseness, otitis followed by pneumonia.

Diagnosis= chlamydophila complement fixation test,pcr invasive samples

treatment= doxycycline or clarithromycin.

1437. A 72yo male presents with acute confusion. He has been in the hosp for 2wks having been
treated for a DVT. The nurses have noticed that he became increasingly drowsy. Exam: small
scalp laceration, a GCS of 8 and bilateral up-going plantar response.
a. Infection toxicity
b. Delirium tremens
c. Extradural hematoma
d. Subdural hematoma
e. Electrolyte imbalance
Answer= D. subdural Hematoma. elderly patient, having been treated for DVT so on anticoagulants,increasing
drowsiness, scalp lacerations (head trauma) all points towards the diagnosis.

exclude other causes=


extradural hematoma= almost always there his h/o of head injury, both extradural and sub dural have lucid
interval. But extra dural has shorter time duration presenting in hours to days while subdural may take weeks
to months to present.

SUBDURAL HEMATOMA
Consider in all whose conscious level fluctuates, and elderly and also in those having an evolving stroke,
especially if on anticoagulants.

Bleeding is from bridging veins between cortex and venous sinuses (vulnerable to deceleration
injury), resulting in accumulating haematoma between dura and arachnoid.

This gradually raises ICP, shifting midline structures away from the side of the clot
and, if untreated, eventual tentorial herniation and coning.
Most subdurals are from trauma but the trauma is often forgotten as it was so minor or so long ago (up to 9
months).
It can also occur without trauma.

The elderly are most susceptible, as brain atrophy makes bridging veins vulnerable. Other risk factors: falls
(epileptics, alcoholics); anticoagulation.

Symptoms:
Fluctuating level of consciousness (seen in 35%) insidious physical or intellectual slowing, sleepiness,
headache, personality change, and unsteadiness.
Signs:
Raised ICP, seizures

Imaging:
CT/MRI shows clot midline shift
Look for crescent-shaped collection of blood over 1 hemisphere. The sickle shape
differentiates subdural blood from extradural haemorrhage.

Treatment:
1st line= Irrigation/evacuation, eg via burr twist drill and burr hole craniostomy.
2nd-line= craniotomy.

1438. A 50yo DM pt came to the OPD with complaint of of fever, muscle ache, dry cough and anorexia. Inv:
CXR=upper lobe cavitation. What is the single most likely causative organism?
a. Legionella
b. Mycoplasma
c. Staphylococcus
d. Klebsiella
e. Streptococcus
Answer= D. Klebsiella. elderly, diabetic,upper lobe cavitation all points towards the diagnosis.

klebsiella pneumonia occurs in elderly, diabetics and alcoholics. flu like symptoms.It causes a cavitating
pneumonia ,particularly of upper lobes,which is often drug resistant.

treatment= cefotaxime or imipenem

1439. A 20yo man complains that all his movements are being watched. Sometimes he feels as though his
actions are being controlled by his radio. At other times he is aware of voices describing what he is doing.
What is the most probable dx?
a. Mania
b. Drug induced psychosis
c. Delusion of control
d. Schizophrenia
e. Korsakoff psychosis

Answer= D. Schizophrenia. Auditory hallucinations, especially the echoing of thoughts, or a third person
'commentary' on one's actions.Thought insertion, removal or interruption - delusions about external control of
thought.
Thought broadcasting - the delusion that others can hear one's thoughts are the features of schizophrenia
pointing towards the diagnosis.

Schizophrenia is the most common form of psychosis. It is a lifelong, condition, which can take on either a
chronic form or a form with relapsing and remitting episodes of acute illness.

Multiple factors are involved in schizophrenia - eg, genetic, environmental and social.
risk factors include= family history, intrauterine and perinatal complications,Intrauterine infection, particularly
viral, Abnormal early cognitive/neuromuscular development etc
features=
The hallmark symptoms of a psychotic illness are: Delusions,Hallucinations (auditory hallucination like two or
more voices discussing the patient in the third person,thought echo,voices commenting on the patient's
behaviour), Thought disorder (thought insertion,thought withdrawal,thought broadcasting), Lack of insight.

These first Rank or positive symptoms of schizophrenia are absent in other psychotic disorders.

NICE published guidelines on the management of schizophrenia in 2009. Key points:

oral atypical antipsychotics are first-line


cognitive behavioural therapy should be offered to all patients
close attention should be paid to cardiovascular risk-factor modification due to the high rates of
cardiovascular disease in schizophrenic patients (linked to antipsychotic medication and high smoking
rates)

1440. A 35yo is agitated and euphoric. He claims to be helping the prime minister with economic
policy, although this is not true when checked. What is the most likely dx?
a. Mania
b. Schizophrenia
c. Hypomania
d. Drug induced personality disorder
e. Delusion of grandeur
Answer= E. Delusions of grandeur. it's characterized by exaggerated belief of knowledge, power, self worth,
relation to a famous person.

DELUSIONS
A delusion is a false belief which is firmly sustained and based on incorrect inference about reality. This belief
is held despite evidence to the contrary and is not accounted for by the person's culture or religion.

Types of Delusions:
Monothematic - delusions are only relating to one particular topic.
Polythematic - range of delusional topics (seen in schizophrenia).

They can also be classified as:


Primary - occur in the mind, fully formed with no preceding reasons; strongly suggestive of schizophrenia.
Secondary - eg depressed person feeling worthless.
Examples:
Delusional jealousy (Othello's syndrome) - eg believing a partner is being unfaithful.
Capgras' delusion - belief that a close relative has been replaced by someone else who looks the
same.
Unilateral neglect - belief that one limb or side does not exist.
Thought insertion - belief that someone is putting thoughts into the brain.
Grandiose delusion - belief of exaggerated self-worth.

1441. A 20yo student who recently visited Asia came to the OPD with complains of low grade fever,
night sweats, anorexia and productive cough. Inv: CXR=cavitatory lesions in upper lobes. What is
the single most likely causative organism?
a. Mycoplasma
b. Klebsiella
c. TB
d. PCP
e. Viral pneumonia
Answer= C. Tuberculosis. travel history, low grade fever with night sweats, anorexia and productive cough,
cavitation on chest x ray all are features of tuberculosis.

investigations for pulmonary TB:


Chest xray= cavitation, consolidation, fibrosis, calcification.

Microbiological samples: firm diagnosis rests on isolating the infecting organism, and subsequent sensitivity
testing can be used to guide antibiotic therapy.

for respiratory Tb = Send at least three spontaneous sputum samples for culture and microscopy (including
one early morning sample). Samples are analysed by Staining with Ziehl-Neelsen (ZN) stain and rapid direct
microscopy for acid/alcohol-fast bacilli.
Histologic = hallmark is caseating granulomas
Mantoux test for contact tracing.

Drug treatment for respiratory TB=


The standard therapy for treating active tuberculosis is:

Initial phase - first 2 months (RIPE)


Rifampicin, Isoniazid, Pyrazinamide, Ethambutol
(the 2006 NICE guidelines now recommend giving a 'fourth drug' such as ethambutol routinely - previously this
was only added if drug-resistant tuberculosis was suspected)
Continuation phase - next 4 months = Rifampicin,Isoniazid
The treatment for latent tuberculosis is isoniazid alone for 6 months

Patients with meningeal tuberculosis are treated for a prolonged period (at least 12 months) with the addition
of steroids.

Directly observed therapy with a three times a week dosing regimen may be
indicated in certain groups, including: homeless people with active tuberculosis, patients who are likely to
have poor concordance, all prisoners with active or latent tuberculosis.
1442. A 35yo man with T1DM is dehydrated with BP=90/50mmHg. What is the single most
appropriate initial inv?
a. ABG
b. CBC
c. HbA1c
d. LFT
e. BUE
Answer= A. ABGS. type one diabetic, dehydrated, low BP are the features of Diabetic ketoacidosis for which
ABGs are done to confirm it and they will show Acidosis (low ph) and low bicarbonate.

Diabetic Ketoacidosis(DKA) is a medical emergency that is characterized by Hyperglycemia, Acidaemia (ph<


7.3) and ketonaemia.

American Diabetes Association diagnostic criteria are as follows:


blood glucose >13.8 mmol/l
pH < 7.30
serum bicarbonate <18 mmol/l
anion gap > 10
ketonaemia

The most common precipitating factors of diabetic ketoacidosis (DKA) are infection, missed insulin doses and
myocardial infarction.
the most common symptoms include polydipsia, polyuria, nausea, vomiting, dehydration,gradual
drowsiness,coma or deep breathing (kussmaul hyperventilation.
Arterial blood gases - metabolic acidosis with low pH and low HCO3; pCO2 should be normal but can be
depressed by respiratory compensation; low pO2 may indicate primary respiratory problem as a precipitant.

Management: fluid replacement: most patients with DKA are deplete around 5-8 litres.
Isotonic saline is used initially
insulin: an intravenous infusion should be started at 0.1 unit/kg/hour. Once blood glucose is < 15 mmol/l an
infusion of 5% dextrose should be started
correction of hypokalaemia
1443. A 45yo woman presents with pruritis. Exam: skin pigmentation. Inv: raised ALP and presence of anti-
mitochondrial antibodies. What is the single most likely dx?
a. Psoriasis
b. Scabies
c. Atopic eczema
d. Dermatitis herpetiformis
e. Hyperthyroidism
f. Primary biliary cirrhosis
Answer= F. primary biliary cirrhosis. Middle aged women presenting with pruritis, skin pigmentation and the
ALP is raised along with the presence of anti microbial antibodies all these features point towards the
diagnosis.
Primary biliary cirrhosis is a chronic liver disorder typically seen in middle-aged females (female:male ratio of
9:1).
The aetiology is not fully understood although it is thought to be an autoimmune condition. Interlobular bile
ducts become damaged by a chronic inflammatory process causing progressive cholestasis which may
eventually progress to cirrhosis. The classic presentation is itching in a middle-aged woman
Antimitochondrial antibodies (AMA) are the hallmark of PBC.

Risk if:
+ve family history, many UTIS; smoking; past pregnancy; other autoimmune
diseases; use of nail polish/hair dye.

Features:
Lethargy, sleepiness, and pruritus may precede jaundice by years, Jaundice;
skin pigmentation; xanthelasma; xanthomata; hepatosplenomegaly

Associations:
Sjogren's syndrome (seen in up to 80% of patients)
rheumatoid arthritis
systemic sclerosis
thyroid disease
Tests:
Blood: Alk phos,gamma GT, and mildly AST & ALT; late disease: bilirubin, albumin, prothrombin
time
anti-mitochondrial antibodies (AMA) M2 subtype are present in 98% of patients and are highly specific
smooth muscle antibodies in 30% of patients
raised serum IgM

Management:
pruritus: cholestyramine
fat-soluble vitamin supplementation
ursodeoxycholic acid
liver transplantation e.g. if bilirubin > 100 (PBC is a major indication) - recurrence in graft can occur but is not
usually a problem

1444. A 60yo man complains of tiredness, lethargy and itching that is severe after a hot bath. He also has
nocturia, polyuria and nausea and vomiting. Exam: pallor, pigmentation and generalized edema. What is the
single most likely dx?
a. Hyperthyroidism b. Lichen planus
c. Lymphoma
d. Eczema
e. Liver failure
f. CRF
Answer: F. CRF (Chronic Renal Failure), tiredness, lethargy, itching, nocturia, polyuria,pigmentation,
generalized edema all point towards the diagnosis.
CHRONIC RENAL FAILURE
Impaired renal function for >3 months based on abnormal structure or function, or GFR <60mL/min/1.73m2
for >3 months with or without evidence of kidney damage.
Symptoms usually only occur once stage 4 is reached (GFR <30).

End-stage renal failure (ESRF) is defined as GFR <15 mL/min/1.73m2


or need for renal replacement therapy (RRTdialysis or transplant)

Common causes of chronic kidney disease


diabetic nephropathy
chronic glomerulonephritis
chronic pyelonephritis
hypertension
adult polycystic kidney disease
pyelonephritis and reflux nephropathy

Symptoms:It may be discovered by chance following a routine blood or urine test.


Specific symptoms usually develop only in severe CKD, and include anorexia, nausea, vomiting,
fatigue, weakness, pruritus, lethargy, peripheral oedema, dyspnoea, insomnia, muscle cramps,
pulmonary oedema, nocturia, polyuria and headache.
Sexual dysfunction is common.
Hiccups, pericarditis, coma and seizures are only seen in very severe CKD.

CKD may be classified according to GFR:

CKD
stage GFR range

1 Greater than 90 ml/min, with some sign of kidney damage on other tests (if all the kidney tests*
are normal, there is no CKD)

2 60-90 ml/min with some sign of kidney damage (if kidney tests* are normal, there is no CKD)

3a 45-59 ml/min, a moderate reduction in kidney function

3b 30-44 ml/min, a moderate reduction in kidney function

4 15-29 ml/min, a severe reduction in kidney function

5 Less than 15 ml/min, established kidney failure - dialysis or a kidney transplant may be needed
Tests:
Blood: Hb (normochromic, normocytic anaemia), ESR, U&E, glucose (DM),
Ca2+, PO4, alk phos (renal osteodystrophy).
PTH if CKD stage 3 or more.
Urine: Dipstick, MC&S, albumin: creatinine ratio or protein : creatinine ratio (p286).
Imaging: USS to check size, anatomy and corticomedullary differentiation.
In CKD kidneys are usually small (<9cm) but can be enlarged in infiltrative disorders (amyloid,
myeloma), APKD and DM.
Histology: Consider renal biopsy if rapidly progressive disease or unclear cause and normal sized kidneys.
Management:
Refer early to a nephrologist if the patient meets any of the following criteria:
Stage 4 and 5 CKD
Moderate proteinuria (ACR >70mg/mmol, see p286) unless due to DM and already appropriately treated
Proteinuria with haematuria
Rapidly falling eGFR (>5mL/min/1.73m2 in 1yr, or >10mL/min/1.73m2 within 5yrs)
BP poorly controlled despite 4 antihypertensive drugs at therapeutic doses
Known or suspected rare or genetic causes of CKD
Suspected renal artery stenosis
Management of patients can be split into four main approaches:
investigations, limiting progression/complications, symptom control and preperation for renal replacement
therapy.

1445. A 30yo man complains of vague pain in the loin with BP=140/90mmHg. He is found to have proteinuria
and hematuria. What is the inv to confirm the dx?
a. Abdominal US
b. ANCA
c. ANA
d. Urine microscopy and culture
e. Stool culture
Answer= A. Abdominal US. Pain in the loin with hematuria and proteinuria and with high blood pressure points
towards the Polycystic Kidney Disease. so investigation of choice in this case would be Abdominal US to
confirm the diagnosis.

ADULT POLYCYSTIC KIDNEY DISEASE


It is an autosomal dominant condition. 85% of patients have mutations in PKD1 (chromosome 16)
and Remainder have mutation in PKD2 (chromosome 4)

Features=
Loin pain is the most common symptom.
Hypertension is a common presenting feature
Bilateral kidney enlargement - abdominal examination may reveal enlarged and palpable kidneys.
Gross haematuria following trauma (haemorrhage into a cyst) is a classic presenting feature of ADPKD. It
occurs in 30-50%.
Cyst infection, renal calculi, progressive renal
failure.
Extrarenal: liver cysts, intracranial aneurysm, SAH (subarachnoid haemorrhage), mitral valve prolapse, ovarian
cysts and diverticular disease

Investigations=
routine blood tests: fbc, urea,cr and electrolytes, GFR etc
urine analysis
Imaging is used to establish the diagnosis and to monitor disease progression
USS screening offers good sensitivity and specificity depending
on age.

Management=
Monitor U&E.
BP should be treated aggressively, with target levels of <130/80 mmHg (ACE-i are best choices).
Treat infections, dialysis or transplantation for End stage renal failure, genetic
counselling.
Pain may be helped by laparoscopic cyst removal or nephrectomy.
Water intake, Na+ intake, and avoiding caffeine may also help.

1446. A 54yo man comes with sudden onset of palpitations and breathlessness. His HR=164bpm. What is the
single most appropriate tx in the acute phase?
a. Adenosine
b. Metaprolol
c. Verapamil
d. Amiodarone
Answer= A. Adenosine. sudden onset palpitations and breathlessness is SVT. For those not hemodynamically
compromised after Vagal Maneuvers first appropriate treatment in acute phase is Adenosine.

SUPRAVENTRICULAR TACHYCARDIA
SVT is usually paroxysmal and episodes may occur regularly or very infrequently (sometimes years apart).
Episodes may only last for a few minutes or may last for up to several months.
Episodes are characterised by the sudden onset of a narrow complex tachycardia, typically an atrioventricular
nodal reentry tachycardia (AVNRT). Other causes include atrioventricular re-entry tachycardias (AVRT) and
junctional tachycardias.

Risk Factors= SVT may occur at any age but often occurs in younger people in the absence of heart disease.
Risk factors include: Previous Myocardial infarction, mitral valve prolapse, rheumatic heart diseases,
pericarditis, pneumonia, chronic lung disease, chronic alcohol intoxication, digoxin toxicity
Presentation: Palpitations and dizziness are the most common symptoms reported.
Other symptoms including fatigue, light-headedness, chest discomfort, dyspnoea, polyuria and syncope.
Investigations:
ECG= P wave absent or inverted after QRS.
Management=
Vagal manoeuvres (breath-holding, valsalva manoeuvre, carotid massage) are
1st line treatments if haemodynamically stable. IV adenosine is the drug of choice.
If adenosine fails, use verapamil 5 mg IV over 2min, or over 3min if elderly.
If haemodynamically unstable= DC (direct current) cardioversion.

1447. A 29yo woman has developed and itchy scaly rash particularly over her wrist with fine white streaks
overlying the lesion. Her nails have ridges and her buccal mucosa is lined with a lacy white pattern. What is the
single most likely dx?
a. Psoriasis
b. Scabies
c. Urticaria
d. Dermatitis herpetiformis
e. Hyperthyroidism
f. Lichen planus
Answer= F. Lichen Planus. itchy scaly rash particularly over wrist, white lace pattern all point towards the
diagnosis.
Exclusion of other options:
Psoriasis= well defined scaly red/ pink plaques with silvery scale on extensor aspects of the elbows, knees,
scalp and sacrum.
Scabies= widespread pruritus, linear burrows on the side of fingers, interdigital webs and flexor aspects of the
wrist
Dermatitis herpetiformis= itchy, vesicular skin lesions on the extensor surfaces (e.g. elbows, knees, buttocks).
it is associated with coeliac disease and is caused by deposition of IgA in the dermis.

Lichen Planus
Lichen planus is a skin disorder of unknown aetiology, most probably being immune mediated.

Features= lesions e.g on flexor aspects of wrist,forearms, ankles and legs


purple, pruritic, poly angular, planar (flat topped) papules, seen at any age + white lacy markings ( known as
Wickhams striae).
lesions elsewhere: scalp (scarring alopecia), nails ( longitudinal ridges), tongue, mouth (lacy white areas on
inner cheeks) and genital lesions.
Lesions often arise at the site of trauma.
usually persist for 6-18 months.

Lichenoid drug eruptions - causes: gold, quinine, thiazides

Treatment=
Topical Steroids ( with or without topical antifungals) are 1st line (esp in oral disease e.g fluticasone spray).
extensive lichen planus may require oral steroids or immunosuppression

1448. The artery that runs in the ant inter-ventricular groove. What is the single most appropriate option?
a. Acute marginal branch
b. Left ant descending artery
c. Coronary sinus
d. Circumflex artery
e. Right coronary artery
Answer= B. Left Anterior Descending Artery.

1449. Which virus is transmitted by the fecal-oral route?


a. Hep C
b. Coxsackie virus
c. Dengue
d. None of the above
Answer= B. Coxsackie virus. Spread is usually from the faeco-oral route or oral-oral route, with an incubation
period of 2-6 days.

The Coxsackieviruses are RNA viruses of the Picornaviridae family, Enterovirus genus which includes
echoviruses and polioviruses. Infections are often asymptomatic. They are divided into groups A (Usually
affects skin and mucous membranes ) and B (Usually affects the heart, lungs, pancreas and liver).
Coxsackieviruses of both types are a leading cause of aseptic meningitis. They may also cause nonspecific
febrile and upper respiratory tract illness.

Usually diagnosis is clinical but some laboratory tests are available.

The virus can be isolated from throat, vesicle or rectal swabs (placed in viral transport medium) or from faecal
culture
IgM with enzyme-linked immunosorbent assay (ELISA) can aid diagnosis.

There is no known treatment for Coxsackievirus infections, so management is supportive

1450. A 40yo woman presented with generalized itching and soreness for few months. She gave a hx of heavy
menstrual periods. Exam: pallor. What is the single most likely causative factor?
a. IDA
b. Lichen planus
c. Dermatitis herpetiformis
d. Eczema
e. Uremia
Answer= A. IDA (iron deficiency anemia). features of anemia like tiredness and pallor due to the menorrhagia
points towards the diagnosis

Iron Deficiency Anemia


it is microcytic hypochromic anemia. most common cause of anemia. (seen in up to 14% of menstruating
women).
Causes:
Blood loss, eg menorrhagia or GI bleeding
Poor diet may cause IDA in babies or children (but rarely in adults), those on special
diets, or wherever there is poverty.
Malabsorption (eg coeliac disease) is a cause of refractory IDA.
In the Tropics, hookworm (GI blood loss) is the most common cause.

Presentation:
koilonychia, atrophic glossitis, angular cheilosis and rarely, post-cricoid webs (Plummer-Vinson syndrome).
Tests:
Microcytic, hypochromic anaemia with anisocytosis and poikilocytosis
MCV, MCH and MCHC. Confi rmed by ferritin (also serum iron with
total iron-binding capacityTIBC, but these are less reliable).

Management:
Treat the cause.
Oral iron, eg ferrous sulfate 200mg/8h PO.( SE: nausea,
abdominal discomfort, diarrhoea or constipation, black stools).
Continue until Hb is normal and for at least 3 months, to replenish stores.

1451. A 7yo child presents with lesions on the trunk. Exam: some lesions are weeping and others are crusted
with a red base. What is the causative organism?
a. Herpes simplex
b. Varicella zoster
c. Rubella virus
d. Herpes zoster
Answer= B. Varicella zoster. lesions on the trunk that are weeping and crusting are all the features of
chickenpox

CHICKEN POX
Chickenpox is caused by primary infection with varicella zoster virus. Shingles is reactivation of dormant virus
in dorsal root ganglion.

Chickenpox is highly infectious


spread via the respiratory route
can be caught from someone with shingles
infectivity = 4 days before rash, until 5 days after the rash first appeared
incubation period = 10-21 days

Clinical features (tend to be more severe in older children/adults)


fever initially
itchy, rash starting on head/trunk before spreading. Initially macular then papular then vesicular
systemic upset is usually mild
Management is supportive:
keep cool, trim nails, calamine lotion
school exclusion: current HPA advice is 5 days from start of skin eruption. They also state 'Traditionally
children have been excluded until all lesions are crusted. However, transmission has never been reported
beyond the fifth day of the rash.' ( oxford says until lesions scabbed)
immunocompromised patients and newborns with peripartum exposure should receive varicella zoster
immunoglobulin (VZIG). If chickenpox develops then IV aciclovir should be considered
Complications: A common complication is secondary bacterial infection of the lesions. Others: purpura
fulminans/DIC, pneumonitis, and ataxia, are commoner in pregnancy and adults than in children.
Shingles Treatment= acute zoster, eg with aciclovir 800mg 5 times/d PO for 7d if eGFR >25;
if immunocompromised: 10 mg/kg/8h slowly IVI for 10d;
alternative: famciclovir or valaciclovir
If conjunctiva affected, use 3% acyclovir ointment Beware iritis; test acuity often.
1452. An 87yo woman with a hx of HTN has acute breathlessness. She has a RR=32bpm, widespread lung
crackles, pulse=120bpm, BP=160/90mmHg and elevated venous pressure. Her peripheral O2 sat=85%. What is
the single most appropriate initial management?
a. IV antibiotics
b. IV furosemide
c. Nitrate infusion
d. Neb. Salbutamol
e. 100% oxygen
Answer= E. 100% oxygen. the pt is hypertensive with acute breathlessness and raised JVP pointing towards the
diagnosis of Acute Heart Failure.

Management of acute heart failure

Sit the patient upright



Oxygen
100% if no pre-existing lung disease

IV access and monitor ECG
Treat any arrhythmias, eg AF

Investigations whilst continuing treatment

Diamorphine 1.255mg IV slowly
Caution in liver failure and COPD

Furosemide 4080 mg IV slowly
Larger doses required in renal failure

GTN spray 2 puffs SL or 2 0.3mg tablets SL
Dont give if systolic BP <90 mmHg

Necessary investigations, examination, and history

If systolic BP 100 mmHg, start a nitrate infusion,
eg isosorbide dinitrate 210mg/h IVI; keep systolic BP 90 mmHg

If the patient is worsening:
Further dose of furosemide 4080mg
Consider CPAPimproves ventilation by recruiting more alveoli, driving fluid out
of alveolar spaces and into vasculature (get help before initiating!)
Increase nitrate infusion if able to do so without dropping systolic BP <100

If systolic BP <100 mmHg, treat as cardiogenic shock and refer to ICU

1453. A 25yo man presented with painless cervical lymphadenopathy with lethargy, night sweats and itching.
What is the single most likely causative factor?
a. Lymphoma
b. Polycythemia
c. IDA
d. Uremia
e. Drug induced
Answer= A. Lymphoma. painless cervical lymphadenopathy with lethargy, itching and night sweats are the
features of hodgkin's lymphoma

Hodgkins lymphoma: Lymphomas are disorders caused by malignant proliferations of lymphocytes. These
accumulate in the lymph nodes causing lymphadenopathy.In Hodgkins lymphoma,characteristic cells with
mirror-image nuclei are found, called ReedSternberg cells

Cause: 2 peaks of incidence: young adults and elderly. EBV has a role in causing it.

Symptoms Often presents with enlarged, painless, non-tender, rubbery superficial lymph nodes, typically
cervical (6070%), also axillary or inguinal nodes. Nodes may become matted. 25% have constitutional
symptoms.
Mediastinal lymph node involvement can cause features due to mass effect, eg bronchial or SVC obstruction
or direct extension, eg causing pleural effusions.

Signs Lymph node enlargement. Also, cachexia, anaemia, spleno- or hepatomegaly.

Types:

Tests Tissue diagnosis: Lymph node excision biopsy if possible. Inc ESR or dec Hb show worse prognosis.
Staging is by Ann-Arbor system.
Treatment: Depends on chemotherapy and radiotherapy.
1454.A 25yo male presents with fever and pain in the right lower thigh of 1m duration. Exam: lower third of
his thigh is red, hot and tender. The XR showed new bone formation. What is the most probable dx?
a. Osteosarcomoa
b. Ewings sarcoma
c. Tuberculus arthritis
d. Exotosis
e. Fibrosarcoma
Answer= B. Ewings Sarcoma.

Ewings Sarcoma
it is the malignant round cell tumor of long bones (typically diaphysis) and limb girdles, usually presents in
adolescents.
Radiographs show bony destruction, concentric layers of new bone formation (onion ring sign) and a soft
tissue mass. MRI is helpful.
typically those with ewing sarcoma have a t 11:22 chromosomal translocation.

Treatment=
Chemotherapy, surgery and radiotherapy are required. The key adverse prognostic factor is metastases at
diagnosis

1455. A 76yo man presents with sore throat, local irritation by hot food, dysphagia and a sensation of
a lump in his throat. He has a 20y hx of smoking. What is the single most likely dx?
a. Nasopharyngeal ca
b. Pharyngeal ca
c. Sinus squamous cell ca
d. Squamous cell laryngeal ca
e. Hypopharyngeal ca
Answer= B. Pharyngeal carcinoma. elderly, sore throat, dysphagia and sensation of lump in the throat are the
features of pharyngeal carcinoma.

Exclusion of other options:


Nasopharyngeal ca= most likely to cause a lump in the neck but may also cause nasal obstruction, deafness
and postnasal discharge.
Hypopharyngeal ca= problems with swallowing and ear pain are common symptoms and hoarseness is not
uncommon.

Pharyngeal carcinoma
Cancer of the pharynx is less common than other head and neck cancers. It occurs in three locations:
oropharynx, nasopharynx and hypopharynx.
Oropharyngeal cancer = typically affects people aged between 40 and 70 years

common symptoms: are a persistent sore throat, a lump in the mouth or throat, pain in the ear.

Investigation: Biopsy is the only way to establish the diagnosis. A fine-needle aspiration (FNA) or biopsy may
be an alternative for a neck mass; lesions that are harder to reach may require endoscopy.
Treatment:
surgery (using open or minimally invasive approaches for tumour resection and reconstruction), radiotherapy,
chemotherapy, or a combination of these methods.
1456. A 42yo female who is obese comes with severe upper abdominal pain and right shoulder tip pain with a
temp=37.8C. She has 5 children. What is the most probable dx?
a. ERCP
b. LFT
c. Serum amylase
d. MRCP
e. US abdomen
Answer= E. US Abdomen. Female, fat, fertile and forty with upper abdominal pain points towards the
diagnosis of gall stones for which investigation of choice is US abdomen.

Gallstones:
Pigment stones: (<10%)
Small, friable, and irregular. Causes: haemolysis.
Cholesterol stones: Large, often
solitary. Causes: age, obesity (Admirands triangle: risk of stone if lecithin, bile
salts, cholesterol).
Mixed stones: Faceted (calcium salts, pigment, and cholesterol).
Gallstone prevalence: 8% of those over 40yrs. 90% remain asymptomatic.
Risk factors for stones becoming symptomatic: smoking; parity.

Acute cholecystitis follows stone or sludge impaction in the neck of the gallbladder, which may cause
continuous epigastric or RUQ pain (referred to the right shoulder, vomiting, fever, local peritonism, or a GB
mass. If the stone moves to the common bile duct (CBD), obstructive jaundice and cholangitis may occur

Tests: WCC,
Ultrasound Is the best way to demonstrate stones, being 90-95% sensitive.
it shows a thick-walled, shrunken GB (also seen in chronic disease)

Treatment:
NBM, pain relief, IVI, and eg cefuroxime 1.5g/8h IV.
Laparoscopic cholecystectomy is the treatment of choice for all patients fit for GA.
Open surgery is required if there is GB perforation.
Cholecystostomy is also the preferred treatment for acalculous cholecystitis

1457. A 37yo laborer comes with hx of redness of left eye with foreign body sensation in the same eye. What
is the single most appropriate option?
a. Ciliary body
b. Sclera
c. Conjunctivitis
d. Cornea
e. Iris
Answer= D. Cornea. Injury is caused by a foreign body and foreign bodies most like injure cornea leading to
corneal ulcer if not removed.

Corneal injuries may be physical, chemical, environmental (eg, ultraviolet (UV) damage) or infective.
Corneal abrasions are common. There is usually a history of minor trauma from a scratch, grit or contact lens
problem.
If there is no history of injury, consider infection - eg, herpes simplex infection, chemical injury and corneal
foreign bodies.
Symptoms of superficial corneal abrasion or corneal FB : Redness, pain, watering (usually). FB sensation
(usually), Blurred vision, Photophobia,Pain on eye movement, Patients are fairly reliable at locating the FB.

Investigations: are not required if you can be sure that the injury is superficial.Plain X-rays of of the orbit/face
can be used to exclude known radiopaque FBs

Treatment= Removal of Foreign body. Use a topical anesthetic

Management of corneal abrasion= refer if large abrasion otherwise: Analgesia= Paracetamol or ibuprofen are
first line. topical antibiotics for 7 days= Chloramphenicol for 1st line.

1458. An 11yo boy came to the hospital with pain after falling off his bicycle. XR= fx at distal radius with
forward angulations. What is the single most probable dx?
a. Dinner fork deformity
b. Cubitus valgus
c. Gun stock deformity
d. Garden spade deformity
e. Genu valgus
Answer= D. Garden Spade Deformity. the fracture of distal radius with forward angulations is smiths fracture
the characteristic appearance of which is Garden Spade Deformity.
Exclusion of other options:
Dinner Fork deformity= it is seen in Colles Fracture.Colles type (common in osteoporotic postmenopausal
women who fall on an outstretched hand). There is dorsal angulation and displacement producing a dinner-
fork wrist deformity.

SMITHs FRACTURE
Sometimes called a 'reverse Colles fracture',
In this injury, the distal radius fractures with the fragment angled and displaced forward/palmarly.
It is usually caused by landing with the wrist in flexion - a backward fall on the palm of an outstretched hand.
The characteristic appearance is called a 'garden spade deformity'.
The X-ray of a Smith's fracture is very similar to a Colles' fracture except with the displacement anteriorly
instead of posteriorly.
Fixation is needed in these fracture more commonly than in Colles fractures, as the fracture fragment tends
to migrate palmarly.

1459. A middle aged man with a lump in front of his neck which moves up while hes swallowing. US shows a
mass replacing the left lobe of thyroid. And spread to the sternocleidomastoid and adjacent muscles. What is
the most probable dx?

a. Thyroid ca
b. Pharyngeal pouch
c. Bronchus ca
d. Thyroid cyst
e. Larynx ca
Answer= A.Thyroid Carcinoma. lump in the neck, us showing mass replacing left lobe of thyroid and metastasis
points towards the diagnosis

Thyroid Cancers
1 Papillary: (60%). Often in younger patients. Spread: lymph nodes & lung (jugulodigastric
node metastasis is the so-called lateral aberrant thyroid). : total thyroidectomy
to remove non-obvious tumour node excision radioiodine to ablate
residual cells. Give thyroxine to suppress TSH. Prognosis: better if young.

2 Follicular: (25%). Occur in middle-age & spreads early via blood (bone, lungs).
Well-differentiated. : total thyroidectomy + T4 suppression + radioiodine ablation.

3 Medullary: (5%). Sporadic (80%) or part of MEN syndrome (p215). May produce
calcitonin which can be used as a tumour marker. They do not concentrate iodine.
Perform a phaeochromocytoma screen pre-op. : thyroidectomy + node
clearance. External beam radio therapy should be considered to prevent regional
recurrence.
4 Lymphoma: (5%). May present with stridor or dysphagia. Do full staging
pre-treatment (chemoradiotherapy). Assess histology for mucosa-associated
lymphoid tissue (MALT) origin (associated with a good prognosis).

5 Anaplastic: Rare. :3:1. Elderly, poor response to any treatment. In the absence
of unresectable disease, excision + radiotherapy may be tried.

1460. A 28yo male complains of severe pain while trying to grasp any object. It started since he
participated in skiing and had a fall and caught his thumb in the matting. Exam: rupture of the
ulnar collateral ligament of MCP joint of the thumb. What is the single most probable deformity?
a. Dinner fork deformity
b. Game keeper thumb
c. Mallet finger
d. Gun stock deformity
e. Garden spade deformity
Answer= B. GameKeepers Thumb. Rupture of ulnar collateral ligament of metacarpophalangeal joint (MCP)
during skiing points towards the diagnosis.

Gamekeepers thumb
This is so-called because of the laxity of the ulnar collateral ligament of the metacarpophalangeal joint of the
thumb during the forced thumb abduction that occurs when wringing a pheasants neck.
The same injury is described in dry ski slope participants who fall and catch their
thumb in the matting (skier's thumb).
Diagnosis can be difficult as the thumb is so painful to examine.
Treatment:
complete tears = surgical. Radiographic
evaluation will detect a bony avulsion fragment.

Partial tears (clinically stable), or those associated with undisplaced avulsion


fractures of the proximal phalanx= simple
short-arm thumb spica casting.
1461. A 25yo male had an injury to the knee while playing football. XR=condylar fx of tibia. What is the
single most probable deformity?
a. Dinner fork deformity
b. Gibbus
c. Cubitus valgus
d. Garden spade deformity
e. Genu valgus
Answer: E. Genu Valgus. (knock knee deformity seen in knee injuries) The typical gait pattern is circumduction,
requiring that the individual swing each leg outward while walking in order to take a step without striking the
planted limb with the moving limb.

Condylar Fracture of Tibia


The most common mechanism resulting in a tibial plateau fracture (condylar) is a valgus force with axial
loading. Of these fractures, 80% are motor vehiclerelated injuries, and the remainder are sports-related
injuries.
Trauma can be direct or can be related to a fall from a height, an industrial accident, or a sports injury.
Tibial plateau fractures may be either low-energy or high-energy. Low-energy fractures occur in osteoporotic
bone and typically are depressed fractures. High-energy fractures occur in low-energy patients often as a
result of motor vehiclerelated trauma, and the most common pattern of fracture in this group is a splitting
fracture.
Approximately 50% of the knees with closed tibial plateau fractures have injuries of the menisci and cruciate
ligaments that usually require surgical repair.
Because of the valgus stress at the moment of impact, the medial collateral ligament is at greater risk than the
lateral collateral ligament;
however, disruption of the lateral collateral ligament is of grave concern because of possible injuries to the
peroneal nerve and the popliteal vessels.

Management:
The following are absolute indications for surgery:
Open plateau fractures
Fractures with an associated compartment syndrome
Fractures associated with a vascular injury

1462. A 50yo man presents with itching after hot shower with dizziness, chest pain after exercise. Exam:
splenomegaly. What is the single most likely causative factor?
a. ALL
b. Lymphoma
c. Polycythemia
d. Scabies
e. Eczema
Answer= C. Polycythaemia. Itch after a hot bath, dizziness and splenomegaly are characteristics of
polycythaemia rubra vera

Polycythaemia rubra vera (PRV) is a myeloproliferative disorder caused by clonal proliferation of a marrow
stem cell leading to an increase in red cell volume, often accompanied by overproduction of neutrophils and
platelets.
A mutation in JAK2 is present in >90%.

Features:
hyperviscosity, headaches, dizziness, tinnitus, visual disturbance.
Itch after a hot bath, and erythromelalgia, a burning sensation in fingers and toes, are
characteristic.
Facial plethora and splenomegaly
Hypertension in a third of patients.

Investigations:
FBC: RCC, Hb, HCT, PCV, often also WBC and platelets B12 Marrow
shows hypercellularity with erythroid hyperplasia , serum erythropoietin
Raised red cell mass on
51Cr studies and splenomegaly, in the setting of a normal PaO2, is diagnostic

Treatment:
Aim to keep HCT <0.45 to risk of thrombosis. In younger patients at low risk, this is done by venesection.
If higher risk (age >60yrs, previous thrombosis)= hydroxycarbamide (=hydroxyurea) is used.
-interferon is preferred in women
of childbearing age. Low-dose aspirin 75 mg daily PO is also given.

1463. A man presented with carcinoma of the bladder and has been working in factories. He wants to know
what dye has caused it. What is the single most likely cause?
a. Aniline
b. Asbestos
c. Latex
d. Silica
Answer= A. Aniline. Occupational exposure to aromatic amines (aniline) is the second most important risk
factor for bladder cancer first being smoking.

BLADDER CANCER:
In developed countries 90% of bladder cancers are transitional cell carcinomas.
Squamous cell carcinomas make up most of the remainder.

Grade 1differentiated;
Grade 2intermediate;
Grade 3poorly differentiated.
80% are confined to bladder mucosa, and only ~20% penetrate muscle.

Presentation:
Painless haematuria; recurrent UTIs; voiding irritability.
Risk Factors:
Smoking; aromatic amines (rubber industry); chronic cystitis; schistosomiasis
(risk of squamous cell carcinoma); pelvic irradiation.

Tests:
Cystoscopy with biopsy is diagnostic
Treatment:
Tis/Ta/T1: (80% of all patients) Diathermy via transurethral cystoscopy/transurethral
resection of bladder tumour (TURBT).
Consider intravesical chemotherapeutic agents for multiple small tumours or high-grade tumours

T23: Radical cystectomy is the gold standard.


Post-op chemotherapy (eg M-VAC: methotrexate, vinblastine, adriamycin, and
cisplatin) is toxic but effective.

T4: Usually palliative chemo/radiotherapy

Tumour spread:
Local to pelvic structures;
lymphatic to iliac and para-aortic nodes;
haematogenous to liver and lungs.

TNM Staging:
Tis Carcinoma in situ Not felt at EUA
Ta Tumour confined to epithelium Not felt at EUA
T1 Tumour in lamina propria Not felt at EUA
T2 Superficial muscle involved Rubbery thickening at EUA
T3 Deep muscle involved EUA: mobile mass
T4 Invasion beyond bladder EUA: fixed mass

(EUA = examination under anaesthetic)

1464. A 62yo man presents with left sided hearing loss and tinnitus. He also complains of vomiting and
headache. Exam: papilledema and SNHL in the left ear. What is the single most likely dx?
a. Meningioma
b. Nasopharyngeal ca
c. Acoustic neuroma
d. Pharyngeal ca
e. Menieres disease
Answer: C. Acoustic Neuroma. Tumor itself compresses 8th cranial nerve palsy causing features like hearing
loss, tinnitus which leads to the raised ICP causing headache, vomiting, papilloedema all points towards the
diagnosis.

Acoustic neuromas (more correctly called vestibular schwannomas) account for approximately five percent of
intracranial tumours and 90 percent of cerebellopontine angle.

These are typically indolent, histologically benign subarachnoid tumours


that cause problems by local pressure, and then behave as space-occupying
lesions

Features can be predicted by the affected cranial nerves


cranial nerve VIII: hearing loss, vertigo, tinnitus
Progressive ipsilateral tinnitus sensorineural deafness (cochlear nerve compression).
cranial nerve V: absent corneal reflex
cranial nerve VII: facial palsy
Tests:
MRI of the cerebellopontine angle is the investigation of choice

Treatment:
Surgery: (difficult, and often not needed, eg if elderly)
Hearing and the facial nerve can be preserved
(eg continuous intraoperative monitoring of evoked electromyograms).
Gamma knife surgery is one good way to neuroma volume.

1465. A HIV +ve 55yo man presents with painless lymphadenopathy, fever, night sweats and weight loss. What
is the most probable dx?
a. Hodgkins lymphoma
b. NHL
c. ALL
d. AML
e. CML
Answer= B. NHL ( non Hodgkins Lymphoma) . Features like painless lymphadenopathy, fever, night sweats
and weight loss in HIV patient points towards the diagnosis of NHL.

This includes all lymphomas without ReedSternberg cells a diverse group.


Most are derived from B-cell lines; diffuse large B-cell lymphoma (DLBCL) is commonest.

Causes:
Immunodeficiencydrugs; HIV, HTLV-1, H. pylori ; toxins; congenital

presentation:
Nodal disease (75% at presentation): superficial lymphadenopathy.
Extranodal disease (25%)Skin: T-cell lymphomas
Systemic symptomsfever, night sweats, weight loss (less common than in
Hodgkins lymphoma, and indicates disseminated disease).
Pancytopenia from marrow involvementanaemia, infection, bleeding (platelets).

Tests:
Blood: FBC, U&E, LFT, LDH worse prognosis, reflecting cell turnover.
Marrow and node biopsy for classification.
CT/MRI of chest, abdomen, pelvis for staging.
Send cytology of any effusion;
LP for CSF cytology if CNS signs.

Treatment:
Low grade: If symptomless, none may be needed. Radiotherapy may be curative in localized disease.
Chlorambucil= diffuse disease.
Remissions =interferon or rituximab
High grade: (eg large B-cell lymphoma, DLBCL)=
R-CHOP regimen: Rituximab Cyclophosphamide, Hydroxydaunorubicin,
vincristine and Prednisolone.

1466. A 22yo man says that he can hear the voice of his deceased uncle telling him that he is being spied on.
The pt is distressed by this becoming low in mood and anxious and has not left the house for 2wks. He is
starting to drink increasing quantities of alcohol. He is noticed to have thought-block and passivity
phenomena. What is the single most suitable med to treat his
symptom?
a. Diazepam
b. Disulfiram
c. Fluoxetine
d. Lithium
e. Olanzapine
Answer= E. Olanzapine. Auditory hallucinations, thought block and passivity points towards the diagnosis of
Schizophrenia for which atypical antipsychotics are 1st line.

Schizophrenia

NICE published guidelines on the management of schizophrenia in 2009.


Key points:
oral atypical antipsychotics are first-line
Examples of atypical antipsychotics
clozapine, olanzapine,risperidone,quetiapine,amisulpride
adverse effects:
weight gain
clozapine is associated with agranulocytosis
cognitive behavioural therapy should be offered to all patients
close attention should be paid to cardiovascular risk-factor modification due to the high rates of
cardiovascular disease in schizophrenic patients (linked to antipsychotic medication and high smoking
rates)

1467. A middle age Asian presents with episodes of fever with rigors and chills for last 1yr. Blood film: ring
form of plasmodium with schuffner's dots in RBCs. What is the drug to eradicate this infection?
a. Doxycycline
b. Mefloquine
c. Proguanil
d. Quinine
e. Artesunate
Answer= Mefloquine.Schffner's dots refers to a hematological finding that is associated with malaria,
exclusively found in Plasmodium ovale and Plasmodium vivax.
Plasmodium vivax induces morphologic alterations in infected host erythrocytes that are visible by light
microscopy as multiple brick-red dots. These morphologic changes, referred to as Schffner's dots, are
important in the identification of this species of malarial parasite
Non Falciparum Malaria

The most common cause of non-falciparum malaria is Plasmodium vivax, with Plasmodium ovale and
Plasmodium malariae accounting for the other cases.

Features
general features of malaria: fever, headache, splenomegaly
Plasmodium vivax/ovale: cyclical fever every 48 hours. Plasmodium malariae: cyclical fever every 72
hours
Plasmodium malariae: is associated with nephrotic syndrome

Ovale and vivax malaria have a hypnozoite stage and may therefore relapse following treatment.

Treatment
non-falciparum malarias are almost always chloroquine sensitive
patients with ovale or vivax malaria should be given primaquine following acute treatment with
chloroquine to destroy liver hypnozoites and prevent relapse

1468. A 50yo man presents with flight of ideas which are rambling and disinhibited. He is distractible,
confused and overactive. What is the most likely dx?
a. Dementia
b. Mania
c. Schizophrenia
d. Psychosis
e. Acute confusional state
Answer= B. Mania. Flight of ideas, distraction, confusion and overactive nature points towards the diagnosis of
mania.

Signs of mania
Mood:
Irritability (80%), euphoria (71%), lability (69%).
Cognition:
Grandiosity (78%); flight of ideas/racing thoughts (71%); distractibility/ poor concentration (71%); confusion
(25%), many conflicting lines of thought urgently racing in contrary directions; lack of insight.
Behaviour:
Rapid speech (98%), hyperactivity (87%), sleep (81%), hypersexuality (57%),
extravagance (55%).
Psychotic symptoms: Delusions (48%), hallucinations (15%).
Less severe states are termed hypomania.
If depression alternates with mania, the term bipolar affective disorder is used .

Cyclical mood swings without the more florid features (as above) are termed cyclothymia.
Causes=
Physical: Infections, hyperthyroidism; SLE; thrombotic thrombocytopenic purpura; stroke; water
dysregulation/Na+
Drugs: Amphetamines, cocaine, antidepressants (esp. venlafaxine), captopril, steroids, procyclidine,L-dopa,
baclofen.
Bipolar disorder: (Age at onset: <25.)

Tests= CT of the head, EEG, and screen for drugs/toxins.

Treating acute mania=

acute moderate/severe mania: olanzapine 10mg PO, adjust to


520mg/day (SE: weight; glucose), or valproate semisodium, eg 250mg/8h PO

Prophylaxis=
Those who have bipolar affective disorder after successful treatment of the manic or depressive episode
should have a mood stabilizer (lithium if good compliance) for longer-term control.

1469. A pt presents with a lid lag, bulging eyes, ophthalmoplegia and thyroid bruit. What inv will you do?
a. TFT
b. Eye sight
c. Tensilon test
d. US
e. FNAC
Answer= TFTS. lid lag, bulging eyes, ophthalmoplegia, thyroid bruit all points towards the diagnosis of
thyrotoxicosis (Graves disease) for which TFTS should be done.

Graves' disease is the most common cause of thyrotoxicosis. It is typically seen in women aged 30-50 years.

Features
typical features of thyrotoxicosis
specific signs limited to Grave's (see below)

Features seen in Graves' but not in other causes of thyrotoxicosis


eye signs (30% of patients): exophthalmos, ophthalmoplegia
pretibial myxoedema
thyroid acropachy

Autoantibodies
anti-TSH receptor stimulating antibodies (90%)
anti-thyroid peroxidase antibodies (50%)

Treatment
1 Drugs: B-blockers (eg propranolol 40mg/6h) for rapid control of symptoms.
Anti-thyroid medication: 2 strategies (equally effective)
A) Titration, eg carbimazole 2040mg/24h PO for 4wks, reduce according to TFTs every 12 months.
B) Block-replace: Give carbimazole + thyroxine simultaneously (less risk of iatrogenic hypothyroidism).
In Graves, maintain on either regimen for 1218 months then withdraw.
Carbimazole SE: agranulocytosis
2 Radioiodine: Most become hypothyroid post-treatment.
CI: pregnancy, lactation.
3 Thyroidectomy: Carries a risk of damage to recurrent laryngeal nerve (hoarse voice) and
hypoparathyroidism. Patients may become hypothyroid.

1470. A 30yo lady complaining of right ear deafness with decreased corneal reflex and past pointing. Acoustic
analysis shows SNHL. What is the next most appropriate inv to do?
a. CT brain
b. CT acoustic canal
c. MRI brain
d. MRI acoustic canal
e. PET brain
Answer= C. MRI brain. unilateral ear deafness, SNHL (sensorineural hearing loss) and decreased corneal reflex
all points towards the diagnosis of Acoustic Neuroma for which MRI is the investigation of choice.

Acoustic neuromas (more correctly called vestibular schwannomas) account for approximately five percent of
intracranial tumours and 90 percent of cerebellopontine angle.

These are typically indolent, histologically benign subarachnoid tumours


that cause problems by local pressure, and then behave as space-occupying
lesions

Features can be predicted by the affected cranial nerves


cranial nerve VIII: hearing loss, vertigo, tinnitus
Progressive ipsilateral tinnitus sensorineural deafness (cochlear nerve compression).
cranial nerve V: absent corneal reflex
cranial nerve VII: facial palsy

Tests:
MRI of the cerebellopontine angle is the investigation of choice

Treatment:
Surgery: (difficult, and often not needed, eg if elderly)
Hearing and the facial nerve can be preserved
(eg continuous intraoperative monitoring of evoked electromyograms).
Gamma knife surgery is one good way to neuroma volume.

1471. A 29yo woman who returned from Egypt 2wks ago now presents with difficulty in breathing, chest pain,
cough and purulent sputum with an episode of blood staining. She is on COCPs.
What is the most likely dx?
a. Pulmonary embolism
b. Pneumonia
c. Lung abscess
d. Pneumothorax
e. Pulmonary edema
Answer= Pneumonia. difficulty in breathing, chest pain, cough with purulent sputum all points towards the
diagnosis of pneumonia. cocps is a distracter here.
exclusion of other options:
Pulmonary Embolism= though cocps increase the risk of PE but no purulent sputum is seen in it and it appears
earlier.

Community acquired pneumonia (CAP) may be caused by the following infectious agents:
Streptococcus pneumoniae (accounts for around 80% of cases)
Haemophilus influenzae
Staphylococcus aureus: commonly after the 'flu
atypical pneumonias (e.g. Due to Mycoplasma pneumoniae)
viruses

Klebsiella pneumoniae is classically in alcoholics

Streptococcus pneumoniae (pneumococcus) is the most common cause of community-acquired pneumonia

Clinical features
Symptoms: Fever, rigors, malaise, anorexia, dyspnoea, cough, purulent sputum, haemoptysis, and pleuritic
pain.
Signs: Pyrexia, cyanosis, confusion (elderly usually), tachypnoea, tachycardia, hypotension, signs of
consolidation (diminished expansion, dull percussion note, tactile vocal fremitus/vocal resonance, bronchial
breathing), and a pleural rub.

Tests:
CXR: lobar or multilobar infiltrates, cavitation or pleural effusion.
Assess oxygenation: oxygen saturation (ABGs if SaO2 <92% or severe pneumonia) and BP.
Blood tests: FBC, U&E, LFT, CRP, blood cultures. Sputum for microscopy and culture.
Pleural fluid may be aspirated for culture.
Consider bronchoscopy and bronchoalveolar lavage if patient is immunocompromised or on
ITU.
Management:
CURB-65 criteria of severe pneumonia
Confusion (abbreviated mental test score <= 8/10)
Urea > 7 mmol/L
Respiratory rate >= 30/min
BP: systolic <= 90 or diastolic <= 60 mmHg
age >= 65 years
Patients with 3 or more (out of 5) of the above criteria are regarded as having a severe pneumonia
The British Thoracic Society published guidelines in 2009:
low or moderate severity CAP: oral amoxicillin. A macrolide should be added for patients admitted to
hospital
high severity CAP: intravenous co-amoxiclav + clarithromycin OR cefuroxime + clarithromycin OR
cefotaxime + clarithromycin
the current BNF has slightly different recommendations for high severity CAP: intravenous
benzylpenicillin + clarithromycin OR benzylpenicillin + doxycycline. For 'life-threatening' infections the
BNF recommends the same as the BTS guidelines for high-severity CAP
1472. A 60yo pt recovering from a surgery for toxic goiter is found to be hypotensive, cyanosed in the recovery
room. Exam: neck is tense. There is oozing of blood from the drain. What is the most probable dx?
a. Thyroid storm
b. Reactionary hemorrhage
c. Secondary hemorrhage
d. Primary hemorrhage
e. Tracheomalacia
Answer= B. Reactionary Hemorrhage. It a post operative complications that occurs within 1st 48hrs of surgery
usually in the recovery room but usually hemorrhage followed by thyroid surgery which can cause tracheal
compression occurs within 1st 24hrs immediately after surgery.

POSTOPERATIVE HEMORRHAGE
Primary Hemorrhage= it occurs at the time of injury or during the surgery.
Replace blood loss. If severe, return to theatre for adequate haemostasis. Treat shock
vigorously
Reactive haemorrhage: it occurs within 1st 24 to 48 hrs of the surgery usually in the recovery room. It is due to
the slippage of ligature or dislodgment of a clot
Replace blood and re-explore wound.
Secondary haemorrhage= (caused by infection) occurs 12 weeks (7 to 14 days) post-op.

1473. A 40yo woman has had varicose vein surgery, planned as a day pt. After the op, she is distressed by
repeated retching and vomiting. Her pain is currently well controlled. What is the best management strategy?
a. Tramadol
b. Co-codamol
c. IM morphine
d. IV ondansetron
e. PO ondansetron
Answer= D. IV Ondansetron

Since the pain is well settled so options A,B and C are irrelevant here as they are opiods for pain relief. D & E
are the two valid options. But in this case IV route is better because she has repeated vomitting.

1474. A pt with renal failure has serum K+=7.5, raised creatinine and broad complex tachycardia. What is the
most appropriate management?
a. Calcium gluconate
b. Sodium bicarbonate
c. Dialysis
d. Furosemide
e. Sotalol
Answer= A. calcium gluconate. serum potassium is raised in this patient and it can be life threatening so we
need to manage that for that we give 10 ml of 10% of calcium gluconate in order to stabilize cardiac
membrane.

HYPERKALAEMIA MANAGEMENT
Untreated hyperkalaemia may cause life-threatening arrhythmias. Precipitating factors should be addressed
(e.g. acute renal failure) and aggravating drugs stopped (e.g. ACE inhibitors). Management may be categorised
by the aims of treatment

Stabilisation of the cardiac membrane


intravenous calcium gluconate (10 ml of 10% calcium gluconate)

Short-term shift in potassium from extracellular to intracellular fluid compartment


combined insulin/dextrose infusion ( usually 10 units of insulin (actrapid) are added to 50 ml of glucose
50% and infused over 30 minutes)
nebulised salbutamol

Removal of potassium from the body


calcium resonium (orally or enema)
loop diuretics
dialysis

1475. An 18yo lady in her 30th week of pregnancy is brought to the hosp in an altered sensorium. She is taking
slow, shallow breaths and her breath has a fruity smell. An ABG: pH=7.20, urine ketones: +ve. What is the
most probable dx?
a. HONK
b. DKA
c. HELLP syndrome
d. PIH
e. GDM
Answer= B. DKA. Slow shallow breaths and fruity smell in breath, ph< 7.30, ketonuria all points towards the
diagnosis of DKA ( diabetic keto acidosis).
Diabetic Ketoacidosis(DKA) is a medical emergency that is characterized by Hyperglycemia, Acidaemia (ph<
7.3) and ketonaemia.

American Diabetes Association diagnostic criteria are as follows:


blood glucose >13.8 mmol/l
pH < 7.30
serum bicarbonate <18 mmol/l
anion gap > 10
ketonaemia

The most common precipitating factors of diabetic ketoacidosis (DKA) are infection, missed insulin doses and
myocardial infarction.
the most common symptoms include polydipsia, polyuria, nausea, vomiting, dehydration,gradual
drowsiness,coma or deep breathing (kussmaul hyperventilation.
Arterial blood gases - metabolic acidosis with low pH and low HCO3; pCO2 should be normal but can be
depressed by respiratory compensation; low pO2 may indicate primary respiratory problem as a precipitant.

Management: fluid replacement: most patients with DKA are deplete around 5-8 litres.
Isotonic saline is used initially
Insulin: an intravenous infusion should be started at 0.1 unit/kg/hour. Once blood glucose is < 15 mmol/l an
infusion of 5% dextrose should be started
correction of hypokalaemia

1476. A 26yo man presented with abdominal distension and pain. His stools have been mucoid and
sometimes blood stained. What is the most appropriate inv?
a. Stool C&S
b. Gastroscopy
c. IgG tissue transglutaminase
d. Barium meal
e. Jejunal biopsy
Answer= A. Stool C&S. this questions is an incomplete question. Mucoid and blood stained stools with
abdominal pain points towards the diagnosis of dysentery either by shigella or campylobacter for which Stool
C&S should be done. If we are suspecting Inflammatory bowel disease then investigation should be barium
Enema and no such option is given.

Exclusion of other options:


Barium meal= it is done for upper GI
Jejunal Biopsy= It will not tell about the cause of the condition.

BACILLARY DYSENTERY
Shigella causes abdominal pain and bloody diarrhoea sudden fever, headache, and
occasionally neck stiffness. CSF is sterile.
Incubation: 17d.
Spread: Faecaloral.
Diagnosis: Stool culture.
Treatment: Fluids PO. Avoid antidiarrhoeal drugs. Drugs: ciprofloxacin 500mg/12h PO for 35d.

shigellosis is often resistant to several antimicrobials: sensitivity testing is important


for all enteric fevers. There may be associated spondyloarthritis
CAMPYLOBACTER INFECTION
Incubation period= 2-5 days
There is a prodromal illness of fever, headache and myalgia lasting up to 24 hours. The fever may be as high as
40C and, whether high or low, may persist for a week.
There are abdominal pains and cramps and profuse diarrhoea with up to 10 stools a day. The stool is watery
and often bloody.
Diagnosis: Stool culture.
Treatment: Fluids PO. Avoid antidiarrhoeal drugs. Drugs: ciprofloxacin 500mg/12h PO for 35d.

1477. An 83yo elderly woman presented in the ED with cough, fever and sneezing. Tx was given but she
became confused and again presented with above said symptoms. What is the cause of her condition?

a. Aspiration due to confusion


b. Alveolar damage due to drugs
c. Drug toxicity
d. Pneumothorax
Answer= A. Aspiration due to confusion.

1478. A 37yo man presents with some raised lesions on the shin. He came with cough and also complains of
arthralgia. Exam: bilateral hilar lymphadenopathy and erythema nodosum is
present. What is the single most likely cause?
a. CD
b. UC
c. Sarcoidosis
d. Streptococcal infection
e. TB
Answer= C. Sarcoidosis. Bilateral hilar lymphadenopathy (BHL), erythema nodosum, fever and polyarthralgia.
these are the typical features of Lofgrens Syndrome which is acute form of the disease sarcoidosis.

Sarcoidosis is a multisystem disorder of unknown aetiology characterised by non-caseating granulomas. It is


more common in young adults and in people of African descent

Features:
acute= erythema nodosum, bilateral hilar lymphadenopathy, swinging fever, polyarthralgia
insidious= dyspnoea, non-productive cough, malaise, weight loss
skin= lupus pernio
hypercalcaemia: macrophages inside the granulomas cause an increased conversion of vitamin D to its active
form (1,25-dihydroxycholecalciferol)

Syndromes associated with sarcoidosis


Lofgren's syndrome= is an acute form of the disease characterised by bilateral hilar lymphadenopathy (BHL),
erythema nodosum, fever and polyarthralgia. It usually carries an excellent prognosis
In Mikulicz syndrome= there is enlargement of the parotid and lacrimal glands due to sarcoidosis, tuberculosis
or lymphoma
Heerfordt's syndrome (uveoparotid fever)= there is parotid enlargement, fever and uveitis secondary to
sarcoidosis
1479. A young lady with cervical ectropion bleeds on touch. What is the most appropriate next inv?
a. Transvaginal US
b. Cervical smear
c. Punch biopsy
d. Serum estradiol
e. Colposcopy
Answer= B. Cervical Smear.

CERVICAL ECTROPION
On the ectocervix there is a transformation zone where the stratified squamous epithelium meets the
columnar epithelium of the cervical canal.
Elevated oestrogen levels (ovulatory phase, pregnancy, combined oral contraceptive pill use) result in larger
area of columnar epithelium being present on the ectocervix.
It is seen on examination as a red ring around the os and is so common as to be regarded as normal.
This may result in the following features=
vaginal discharge, post-coital bleeding (usually asymptomatic)
Once a normal cervical smear has been confirmed, it is actively managed only if there are symptoms

Treatment:
Ablative treatment (for example 'cold coagulation') is only used for troublesome symptoms. otherwise no
treatment is required.

1480. A 28yo man with recent onset of dyspepsia after eating spicy food and alcohol consumption. H. pylori
fecal antigen was negative. He returns after 1m with similar symptoms despite being given omeprazole 40mg.
What is the single best initial inv?
a. Hydrogen breath test
b. Gastroscopy
c. Barium meal
d. None
Answer= B. Gastroscopy. It is a stepwise approach . Patient has developed similar symptoms after treatment
with PPI when h.pylori fecal antigen test s negative so the next step is to do Endoscopy (gastroscopy).
Alarm symptoms: Anaemia (iron deficiency); loss of weight; anorexia;
recent onset/progressive symptoms; melaena/haematemesis; swallowing difficulty.

>55yrs or ALARM signs




yes NO
Do upper GI endoscopy Stop drugs causing dyspepsia, eg NSAIDS
Lifestyle changes (p244)
Over-the-counter antacids, eg
magnesium trisilicate 10mL/8h PO
Review after 4wks
(no improvement)
If neg,PPIs or H2 blockers for 4wks test for H.pylori
(eg omeprazole 20 mg/24h PO or
ranitidine 150mg/12 PO). if positive treat to eradicate H.pylori
review after 4 wks if improvement
( no improvement) no action required
improvement no improvement urea breath test
(h.pylori eradicated?)
no further Long-term,low dose yes
action req treatment consider upper GI endoscopy
consider upper GI
endoscopy
1481. A 35yo woman who usually has 4 days mid-cycle bleeding, had her period 10d ago. She has now
presented with spots of blood. Her smear was normal 6m ago. Exam: cervical ectropion which doesnt bleed
on touch. What would you do?
a. Cervical smear
b. Endocervical swab
c. US guided biopsy
d. Laparotomy
e. Transvaginal US
f. Punch biopsy
g. Serum estradiol
h. Colposcopy
Answer= A Cervical Smear. examination shows cervical ectropion that doesn't bleed on touch and most of the
ectropion reverse to normal so we repeat cervical smear to see that whether it persists or progresses or has
regressed.

CERVICAL ECTROPION
On the ectocervix there is a transformation zone where the stratified squamous epithelium meets the
columnar epithelium of the cervical canal.
Elevated oestrogen levels (ovulatory phase, pregnancy, combined oral contraceptive pill use) result in larger
area of columnar epithelium being present on the ectocervix.
It is seen on examination as a red ring around the os and is so common as to be regarded as normal.
This may result in the following features=
vaginal discharge, post-coital bleeding (usually asymptomatic)
Once a normal cervical smear has been confirmed, it is actively managed only if there are symptoms

Treatment:
Ablative treatment (for example 'cold coagulation') is only used for troublesome symptoms. otherwise no
treatment is required.

1482. A 7yo boy presents with epistaxis of 2h duration. The bleeding has been controlled. Inv: Plts=210,
PT=13, APTT=42, bleeding time=normal. Which of the following is the most likely dx?
a. Hemophilia
b. Von willebrand disease
c. ITP
d. Vit K deficiency
e. Liver disease
f. Anatomical defect
Answer= F.Anatomical defect. All parameters are normal so answer in this case would be Anatomical defect.
von willebrand disease= prolonged bleeding time Haemphilia: prolonged APTT

1483. A pregnant woman returns from Sudan, now presenting with intermittent fever, rigor and
seizures. What is the dx?
a. TB
b. Malaria
c. Meningitis
d. Lyme disease
Answer= B. Malaria. Travel history with intermittent fever, rigor and seizures all point towards the diagnosis.
Falciparum malaria:
90% present within 1 month of the mosquito event, with prodromal headache, malaise, myalgia
anorexia before the 1st fever paroxysm ( faints).
There may be no pattern to fever spikes (esp . initially);

Signs: Anaemia, jaundice, and hepatosplenomegaly. No rash or lymphadenopathy.


Complications:
Anaemia is common, Thrombocytopenia.
5 grim signs:
1 Consciousness/coma 2 Convulsions 3 Coexisting chronic illness 4 Acidosis
5 Renal failure (eg from acute tubular necrosis).

Pregnancy: Use chemoprophylaxis when pregnant in endemic areas.

Diagnosis: serial thin and thick films

Management:
Current guidelines suggest all patients with falciparum malaria should be admitted to hospital initially - even
semi-immune patients may worsen quickly.

uncomplicated falciparum malaria treatment:


Current UK guidelines suggest as possible alternative regimens for adults:
Oral quinine sulfate 600 mg/8 hours for 5-7 days plus doxycycline 200 mg daily (or clindamycin 450
mg/8 hours for pregnant women) for 7 days.
Atovaquone-proguanil: 4 standard tablets daily for 3 days.
Artemether with lumefantrine: if weight >35 kg, 4 tablets stat and then a further 4 tablets at 8, 24,
36, 48 and 60 hours.

the WHO revised their treatment guidelines in 2010. These recommend that
artemisinin-based combination therapies should be used first-line in preference to quinine.
complicated or severe Falciparum malaria treatment:
IV quinine dihydrochloride is the first-line antimalarial drug.
Oral quinine sulfate 600 mg tds should be substituted once the patient is well enough to complete a 5- to 7-
day course in total.
Artesunate regimen - for named adult patient use only, on expert advice
A second drug should always accompany these regimes.

1484. A pt is unresponsive and cyanosed. What is the most definitive 1st step in management?
a. Chest compressions
b. Check airway
c. Call 999
d. Mouth to mouth
e. Recovery position
Answer= B. check airway
1485. A man was bitten by a drug addict and comes to the hosp with a wound. What inv should be
undertaken?
a. Hep C
b. Lyme disease
c. Hep B
d. Syphilis
e. Hep A
Answer= C.Hep B. Hepatitis B spreads via blood products, IV drug abusers, direct contact.
Hepatitis B virus (HBV, a DNA virus.)
Spread: Blood products, IV drug abusers (IVDU),sexual, direct contact.

Risk groups: IV drug users and their sexual partners/carers; health workers; haemophiliacs; job exposure to
blood ; haemodialysis (and chronic renal failure);
close family members of a carrier or case; staff or residents of
institutions/prisons; babies of HBSAg +ve mothers

Incubation= 1- 6 months

Signs: Resemble hepatitis A but arthralgia and urticaria are commoner.

Tests: HBSAg (surface antigen) is present 16 months after exposure.


HBeAg (e antigen) is present for 13 months after acute illness and implies high infectivity.
HBSAg persisting for >6 months defi nes carrier status and occurs in 510% of infections;
Antibodies to HBCAg (anti-HBc) imply past infection;
Antibodies to HBSAg (anti-HBs) alone imply vaccination.
HBV PCR allows monitoring of response to therapy.
Vaccination:
Passive immunization (specific anti-HBV immunoglobulin) may be given
to non-immune contacts after high-risk exposure.

Treatment:
Avoid alcohol. Immunize sexual contacts.
Refer all with chronic liver inflammation (eg ALT 30iu/L) for antivirals, eg pegylated (PEG) interferon alfa-2a,
lamivudine, entecavir, adefovir.
The aim is to clear HBSAg and prevent cirrhosis and HCC (risk is if HBSAg and HBeAg +ve).

Other complications: fulminant hepatic failure,


cholangiocarcinoma, cryoglobulinemia.

1486. An 18yo woman says that she cant walk around as she is very big for that room. What is the most likely
hallucination?
a. Extracampine visual hallucinations
b. Lilliputian visual hallucinations
c. Alice in wonderland syndrome
d. Hypnagogic hallucinations
Answer= B. lilliputian visual hallucinations.These are hallucinations seen in Alice in wonderland syndrome.
there is altered perception in size and shape of body parts or objects an impaired sense of passing time.

Alice in Wonderland Syndrome (also known as Todd's syndrome, or lilliputian hallucinations) is a disorienting
neurological condition that affects human perception. People experience micropsia, macropsia, pelopsia,
teleopsia, or size distortion of other sensory modalities. It is often associated with migraines, brain tumors,
and the use of psychoactive drugs. It can also be the initial symptom of the EpsteinBarr virus.
1487. A middle aged lady presented with fever, altered sensorium, bleeding gums and jaundice.
Labs:deranged renal function tests, normal PT/APTT, fragmented RBCs and low plts. Whats the most
likely dx?
a. Cholesterol emboli
b. HUS
c. TTP
d. Hepatorenal syndrome
e. Sepsis
Answer= C. TTP. altered sensorium, jaundice, low plts all point towards the diagnosis.
exclusion of other causes:
Haemolytic uraemic syndrome (HUS) Though it has same features as TTP but it is usually common in
children. HUS is characterized by microangiopathic haemolytic anaemia (MAHA): intravascular haemolysis +
red cell fragmentation. Causes: 90% are from E. coli strain O157
This typically affects young children in outbreaks (more common than sporadically) after eating undercooked
contaminated meat. Signs: Abdominal pain, bloody diarrhoea,
and AKI.
Tests: Haematuria/proteinuria. Blood film: fragmented RBC (schistocytes, platelets, Hb. Clotting tests are
normal.
treatment: Seek expert advice.
Dialysis for AKI may be needed. Plasma exchange is used in severe persistent disease.

TTP (THROMBOTIC THROMBOCYTOPENIC PURPURA)


There is an overlap between TTP and HUS, and many physicians consider them a spectrum of disease.
All patients have MAHA (severe, often with jaundice) and low platelets.
Other features can include AKI, fluctuating CNS signs (eg seizures, hemiparesis, consciousness, vision) and
fever.
The classic description included the full pentad of features, but with the advent of plasma exchange this is
rarely seen.
Mortality is higher than childhood HUS and can be >90% if untreated, though reduced to ~20% with plasma
exchange.
Pathophysiology: There is a genetic or acquired deficiency of
a protease (ADAMTS13) that normally cleaves multimers of von Willebrand factor
(VWF).
Causes: Idiopathic (40%), autoimmunity (eg SLE), cancer, pregnancy, drug associated (eg quinine), bloody
diarrhoea prodrome (as childhood HUS)

Tests: As HUS
treatment:
Urgent plasma exchange may be life-saving.
Steroids are the mainstay for non-responders.
.
1488. A child came to the ED with severe asthma and not responding to salbutamol nebulizer and vomiting
many times. What is the most appropriate management?
a. Salmeterol
b. Montelukast
c. Prednisolone
d. Budesonide inhaler
e. Oxygen
f. IV salbutamol
Answer= F. IV Salbutamol. it is a severe attack where optimum treatment is failed.
1489. A 73yo woman with skeletal and brain mets from breast ca has worsening low back pain and blurring of
vision. She has weakness of her legs, minimal knee and absent ankle tendon reflexes, a palpable bladder, a
power of 2/5 at the hip, 3/5 at the knee and ankle, and tenderness over the 2nd lumbar vertebra. There is
reduced sensation in the perineum. She has been started on dexamethasone 16mg daily.What is the single
most likely cause of her weakness?
a. Paraneoplastic neuropathy
b. Progression of brain tumor
c. PID at L2/L3
d. Spinal cord compression
e. Steroid induced myopathy
Answer= D. spinal cord compression.
Spinal cord compression (as probably at L2/L3 region associated cauda equina syndrome). Here blurring of
vision can be explained by raised intracranial pressure causing papilloedema (due to brain metastasis) which
excludes blurring of vision from "Paraneoplastic neuropathy". In progressive brain tumour you will get upper
motor neurone lesions which is not evident here. rather lower motor neurone type features and bladder
involvement and reduced perineal sensation justifies spinal cord compression associated with cauda equina
syndrome.
1490. A 78yo woman presents with unilateral headache and pain on chewing. ESR=70mm/hr. She is on oral
steroids. What is the appropriate additional therapy?
a. Bisphosphonates
b. HRT
c. ACEi
d. IFN
e. IV steroids
Answer= A. Bisphosphonates. unilateral headache and pain on chewing with raised ESR all points towards the
diagnosis of GIANT CELL ARTERITIS the management of which is high dose steroids. so osteoporosis
prophylaxis is given in these patients, thus bisphosphonates are given.

Giant cell arteritis (GCA) = cranial or temporal arteritis

It is common in the elderlyconsider Takayasus if under 55yrs.


It is associated with PMR (polymyalgia Rheumatica) in 50%

Symptoms:
Headache, temporal artery and scalp tenderness (eg when combing hair), jaw claudication, amaurosis fugax,
or sudden blindness, typically in one eye.
Extracranial symptoms may include dyspnoea, morning stiffness, and unequal or weak pulses.
If you suspect GCA, do ESR and start prednisolone 60mg/d PO immediately. The risk is irreversible bilateral
visual loss, which can occur suddenly if not treated.

Tests: ESR & CRP , platelets, alk phos, Hb.


Get a temporal artery biopsy within 7 days of starting steroids.

Prognosis: Typically a 2-year course, then complete remission.


Reduce prednisolone once symptoms have resolved and ESR; dose if symptoms recur.
The main cause of death and morbidity in GCA is long-term steroid treatment so consider
risks and benefits!
Give gastric and bone protection (PPI & bisphosphonate).

1491. A 48yo woman is admitted to the ED with a productive cough and mod fever. She often has central
chest pain and she regurgitates undigested food most of the time but doesnt suffer
from acid reflux. These symptoms have been present for the last 3.5m which affects her daily
food intake. CXR: air-fluid level behind a normal sized heart. What is the single most likely dx?
a. Pharyngeal pouch
b. Hiatus hernia
c. Bulbar palsy
d. Achalasia
e. TB
Answer= D.Achalasia. Regurgitation of undigested foods, respiratory infection, absence of acid reflux, air fluid
level on chest x ray all point towards the diagnosis.

exclusion of other options:


Hiatal hernia=Hiatus hernia is a risk factor for GORD,in GORD patient will have regurgitation of digested food
particles and acid reflux.
ACHALASIA CARDIA
Failure of oesophageal peristalsis and of relaxation of lower oesophageal sphincter (LOS) due to degenerative
loss of ganglia from Auerbach's plexus i.e. LOS contracted, oesophagus above dilated. Achalasia typically
presents in middle-age and is equally common in men and women.

Clinical features
dysphagia of BOTH liquids and solids
typically variation in severity of symptoms
heartburn
regurgitation of food - may lead to cough, aspiration pneumonia etc
malignant change in small number of patients

Investigations
manometry: excessive LOS tone which doesn't relax on swallowing - considered most important
diagnostic test
barium swallow shows grossly expanded oesophagus, fluid level, 'bird's beak' appearance
CXR: wide mediastinum, fluid level

Treatment
intra-sphincteric injection of botulinum toxin
Heller cardiomyotomy
balloon dilation
drug therapy has a role but is limited by side-effects

1492. A retired ship worker has pleural effusion and pleural thickening on right side with bilateral lung
shadowing. What would you do to improve his symptoms?
a. Aspiration
b. Chest drain
c. Chemotherapy
d. Diuretic
Answer= C, Chemotherapy. ship worker developing pleural effusion and pleural thickening
points towards the diagnosis of Mesothelioma for which Chemotherapy is definite treatment.

Malignant mesothelioma is a tumour of mesothelial cells that usually occurs in


the pleura, and rarely in the peritoneum or other organs. It is associated with occupational
exposure to asbestos,

Clinical features: Chest pain, dyspnoea, weight loss, finger clubbing, recurrent pleural
effusions.
Signs of metastases: lymphadenopathy, hepatomegaly, bone pain/tenderness,
abdominal pain/obstruction (peritoneal malignant mesothelioma).

Tests:
CXR/CT: pleural thickening/effusion. Bloody pleural fluid.

Treatment:
Patients are usually offered palliative chemotherapy (Pemetrexed + cisplatin) and there is also a limited role
for surgery and radiotherapy.

1493. An 88yo woman is a known smoker. She had an attack of MI 2y back and is known to have peripheral
vascular disease. She presents with an irreducible herniation over the incision region of a surgery which she
underwent in her childhood. What is the most appropriate tx?
a. Truss
b. Elective herniorrhaphy
c. Urgent herniorrhaphy
d. Elective herniotomy
e. Reassure
Answer= B. Elective herniorrhaphy.
Incisional hernia is a risk of any abdominal surgery and is estimated to occur in 15% of abdominal operations.

They are caused essentially by failure of the wound to heal but are probably the result of multiple
patient and technical factors.
Advances in technique and materials have not removed this problem.

Management
They require urgent repair with reinforcing mesh used in large hernias. This is required particularly
where the patient is obese.
Recurrence occurs in up to 50% of large hernias.

1494. A 72yo woman who is taking loop diuretics for left ventricular failure. She now is suffering from
palpitations and muscle weakness. What is the electrolyte imbalance found?
a. Na+=130mmol/L, K+=2.5mmol/L
b. Na+=130mmol/L, K+=5.5mmol/L
c. Na+=140mmol/L, K+=4.5mmol/L
d. Na+=150mmol/L, K+=3.5mmol/L
e. None
Answer= A. Na+=130mmol/L, K+=2.5mmol/L

Loop Diuretics
loop diuretics act by inhibiting the Na-K-Cl cotransporter (NKCC) in the thick ascending limb of the loop of
Henle, reducing the absorption of NaCl.
There are two variants of NKCC; loop diuretics act on NKCC2, which is more prevalent in the kidneys.
Indications= heart failure: both acute (usually intravenously) and chronic (usually orally), resistant
hypertension, particularly in patients with renal impairment.
Adverse effects= hypotension, hyponatremia, hypokalemia,hypocalcemia,hypochloremic alkalosis,
hyperglycemia (less common with thiazides), renal impairment( from dehydration+direct toxic effect) and gout

1495. A young woman who is a marathon runner comes with secondary amenorrhea. Inv: normal LH, FSH and
estradiol, prolactin=600. What is the most likely dx?
a. Hypothalamic amenorrhea
b. Pregnancy
c. PCOS
d. Prolactinoma
e. Anorexia
Answer= A. Hypothalamic Amenorrhea. patient is a marathon runner and LH,FSH may be normal if weight loss
or excessive exercise are the cause. prolactins levels can also be increased by stress.
Hypothalamic amenorrhea : excessive exercise can impair the pulsatile release of GnRH from the
hypothalamus. Although in most of such cases LH and FSH are low , a normal level can be present depending
on the timing of the test with regards to the menstrual cycle.
In general , prolactin levels can be interpreted in the following way :
<500mU/L : normal.
500-1000 : drugs , stress ( including physical stress).
1000-5000 : drugs , microprolactinoma , disconnection hyperprolactinemia.
> 5000 : macroprolactinoma.
During pregnancy and lactation :750-8000 mU/L.

Hypothalamicpituitaryovarian causes are common (34% of cases) as control of the menstrual cycle is easily
upset, eg by stress (emotions, exams), exercise, weight loss. Up to 44% of competitive athletes have
amenorrhoea.
Tests HCG (eg urinary) to exclude pregnancy. FSH/LH (low if hypothalamic pituitary cause but may be
normal if weight loss or excessive exercise the cause. Raised eg FSH>20 IU/L if premature menopause
Prolactin (by stress, hypothyroidism, prolactinomas and drugs, eg phenothiazines, domperidone,
metoclopramide).

Treatment is related to cause.


Hypothalamicpituitary axis malfunction:
If mild (eg stress, moderate weight loss) =there is sufficient activity to stimulate enough ovarian oestrogen to
produce an endometrium (which will be shed after a progesterone challenge,
eg medroxyprogesterone acetate 10mg/24h for 10 days), but the timing is disordered so cycles are not
initiated.

If the disorder is more severe the axis shuts down (eg in severe weight loss)= Reassurance and advice on diet
or stress management, or psychiatric help if appropriate.
she should be advised to use contraception as ovulation may occur at any time.

1496. A 4yo child comes with a sprain in his foot. Hx reveals that the child has had recurrent
admissions to the hosp due to severe asthma. What is the most appropriate analgesic?
a. Diclofenac sodium
b. Ibuprofen.
c.Paracetamol
d. Codeine
Answer= C.Paracetamol. paracetamol should only be given in this patient to relieve pain. NSAIDS should not
be given as the child has h/o asthma as nsaids may increase the risk of acute bronchospasm and codeine can
cause respiratory depression.

1497. A 34yo pregnant woman, 38wk GA is in labor. She had a long 1st stage and troublesome 2nd stage, has
delivered a baby. After her placenta was delivered she had a convulsion. What is the most probable
management?
a. MgSO4 IV
b. Diazepam IV
c. IV fluid
d. Hydralazine IV
e. Anti-epileptic
Answer= MGSO4 IV. whenever a woman develops a fit few days after delivery, it is always eclampsia until
proven otherwise.
Treatment of hypertension:
If BP >160/110 mmHg or mean arterial pressure >125 mmHg, use labetalol
20mg IV increasing after 10min intervals to 40 mg then 80 mg until 200mg
total is given. Aim for BP 150/80100 mmHg.
Alternative is hydralazine.

Give prophylactic H2 blockers until normal postnatal care starts.


Restrict fluids to 80 mL/h. Hourly urine output. Renal failure is rare. Maintain
fluid restriction until postpartum diuresis. Fluid restriction is inappropriate
if there is haemorrhage.

Treatment of seizures (eclampsia):


Treat a first seizure with 4g magnesium sulfate in 100 mL 0.9% saline IVI
over 5min + maintenance IVI of 1g/h for 24h. Beware respiration.

If recurrent seizure give 2g IV magnesium sulfate over 5 min.

Check tendon reflexes and respiratory rate every 15min.

Stop magnesium sulfate IV if respiratory rate <14/min or tendon reflex


loss, or urine output <20 mL/h. Have IV calcium gluconate ready in case of
MgSO4 toxicity: 1g (10mL) over 10 min if respiratory depression.

Use diazepam once if fits continue (eg 510 mg slowly IV). If seizures continue,
ventilate and consider other causes (consider CT scan).

1498. A 23yo woman presents with offensive vaginal discharge. Vaginal pH=4.5. What is the most likely
organism?
a. Gardenella
b. Trichomonas
c. Candida
d. Mycoplasma
Answer= A. Gardnerella. offensive vaginal discharge with a vaginal ph of 4.5 points towards the organism
involved.

Bacterial vaginosis (BV) describes an overgrowth of predominantly anaerobic organisms such as Gardnerella
vaginalis. This leads to a consequent fall in lactic acid producing aerobic lactobacilli resulting in a raised vaginal
pH.

Whilst BV is not a sexually transmitted infection it is seen almost exclusively in sexually active women.

Features
vaginal discharge: 'fishy', offensive
asymptomatic in 50%

Amsel's criteria for diagnosis of BV - 3 of the following 4 points should be present


thin, white homogenous discharge
clue cells on microscopy: stippled vaginal epithelial cells
vaginal pH > 4.5
positive whiff test (addition of potassium hydroxide results in fishy odour)

Management
oral metronidazole for 5-7 days
70-80% initial cure rate
relapse rate > 50% within 3 months
the BNF suggests topical metronidazole or topical clindamycin as alternatives

1499. A 62yo man has had anorectal pain aggravated by defecation for 3d. Rectal exam: purple, tender lump
at the anal verge. Flexible sigmoidoscopy: normal rectal mucosa and hard feces. What is the best management
strategy?
a. Anal hematoma
b. Anal fissure
c. Rectal ca
d. Diverticulitis
e. Angiodysplasia
Answer= A. Anal Hematoma. anorectal pain aggravated by defecation and tender lump at the anal verge point
towards the diagnosis.

Perianal haematoma (AKA thrombosed external pile)


Strictly, it is actually a clotted venous saccule.
It appears as a 24mm dark blueberry under the skin at the anal margin. It may be evacuated under LA or left
to resolve spontaneously.

Haemorrhoidal tissue is part of the normal anatomy which contributes to anal continence. These mucosal
vascular cushions are found in the left lateral, right posterior and right anterior portions of the anal canal (3
o'clock, 7'o'clock and 11 o'clock respectively). Haemorrhoids are said to exist when they become enlarged,
congested and symptomatic

Clinical features
painless rectal bleeding is the most common symptom
pruritus
pain: usually not significant unless piles are thrombosed
soiling may occur with third or fourth degree piles

Types of haemorrhoids

External
originate below the dentate line
prone to thrombosis, may be painful

Internal
originate above the dentate line
do not generally cause pain

Grading of internal haemorrhoids


Grade I Do not prolapse out of the anal canal

Grade II Prolapse on defecation but reduce spontaneously

Grade III Can be manually reduced

Grade IV Cannot be reduced

Management
soften stools: increase dietary fibre and fluid intake
topical local anaesthetics and steroids may be used to help symptoms
outpatient treatments: rubber band ligation is superior to injection sclerotherapy
surgery is reserved for large symptomatic haemorrhoids which do not respond to outpatient
treatments
newer treatments: Doppler guided haemorrhoidal artery ligation, stapled haemorrhoidopexy

Acutely thrombosed external haemorrhoids


typically present with significant pain
examination reveals a purplish, oedematous, tender subcutaneous perianal mass
if patient presents within 72 hours then referral should be considered for excision. Otherwise patients
can usually be managed with stool softeners, ice packs and analgesia. Symptoms usually settle within
10 da

1500. A 43yo presents with severe vertigo on moving sidewards whilst sleeping. What test would you do to
confirm the dx?
a. Hallpikes manoeuvre
b. Rombergs test
c. Trendelenburg test
d. Heel-shin test
Answer= A. Hallpikes manoeuvre
. 1501. A 23yo man is having difficulty in speaking following a stab wound to the right of his neck. On being
asked to protrude his tongue, the tip deviated to the right. Which anatomical site is most likely to be affected?
a. Facial nerve
b. Hypoglossal nerve
c. Vagus nerve
d. Trigeminal nerve
e. Glossopharyngeal nerve
Answer: B
The hypoglossal nerve is the twelfth cranial nerve XII, and innervates muscles of the tongue.The nerve is
involved in controlling tongue movements required for speech, food manipulation (i.e. formation of bolus),
and swallowing.Damage or lesions affecting functions of the nerve may be supranuclear (as in cortical or
rostrally to the hypoglossal nucleus in the brainstem), nuclear or infranuclear (as in below or distal to the
nucleus), as well as unilateral or bilateral and will give symptoms which differ accordingly.[2] Supranuclear
injury of the brainstem affecting the hypoglossal nerve may give rise to crossed symptoms due to a majority of
the supranuclear innervation to the hypoglossal nucleus being crossed. Symptoms often show deviation of the
tongue towards the paralyzed side when it is stuck out. This is because of the weaker genioglossal muscle.

1502. A girl presents with signs of hyperventilation. What is the most likely ABG derangement?
a. pH increased, PCO2 increased
b. pH decreased, PCO2 increased
c. pH increased, PCO2 decreased
d. pH decreased, PCO2 decreased

Answer: I think C but given in key is A


Hyperventilation occurs when the rate and quantity of alveolar ventilation of carbon dioxide exceeds the
body's production of carbon dioxide. When alveolar ventilation is excessive, more carbon dioxide will be
removed from the blood stream than the body can produce. This causes the concentration of carbon dioxide
in the blood stream to fall and produces a state known as hypocapnia. The body normally attempts to
compensate for this metabolically. If excess ventilation cannot be compensated metabolically, it will lead to a
rise in blood pH. This rise in blood pH is known as respiratory alkalosis.
Involuntary hyperventilation can occur in response to both physical and emotional stimuli. These include
reduced air pressure at high altitudes,[6] raised progesterone levels in pregnancy, head injury, stroke,
respiratory disorders such as asthma and pneumonia, cardiovascular problems such as pulmonary embolisms,
anemia, adverse reactions to certain drugs, physical or emotional stress, fear, pain, and anxiety.
Hyperventilation can also be mechanically produced in people on respirators.[7]

1503. A pt presents with skin pigmentation, diarrhea, vomiting, abdominal pain and postural
hypotension. What electrolyte abnormality is likely to occur?
a. Na+=130, K+=6.5
b. Na+=130, K+=2.5
c. Na+=13, K+=6.0
d. Na+=140, K+=8
e. Na+=130, K+=1.5

Answer: A
Addisons disease (also Addison disease, chronic adrenal insufficiency, hypocortisolism, and hypoadrenalism)
is a rare, chronic endocrine system disorder in which the adrenal glands do not produce sufficient steroid
hormones (glucocorticoids and mineralocorticoids). Persistent nonspecific symptoms which should provoke
consideration of a diagnosis of adrenal insufficiency include:

Fatigue and weakness (common feature).


Anorexia.
Nausea.
Vomiting.
Weight loss.
Abdominal pain.
Diarrhoea.
Constipation.
Cravings for salt and salty foods such as soy sauce or liquorice (primary insufficiency).
Muscle cramps and joint pains.
Syncope or dizziness (due to hypotension).
Confusion.
Personality change.
Irritability.
Loss of pubic or axillary hair in women, delayed puberty in children.
Signs:
Hyperpigmentation - look at buccal mucosa, lips, palmar creases, new scars and in areas subject to pressure
such as elbows, knuckles and knees. (Not present in secondary adrenal insufficiency.)
Hypotension.
Postural hypotension.

Laboratory abnormalities in adrenal insufficiency


Sodium - reduced in 90% of newly diagnosed cases of primary adrenal insufficiency.
Potassium - raised in 50% of newly diagnosed cases of primary adrenal insufficiency.
Calcium - raised in 10-20 % of newly diagnosed cases of primary adrenal insufficiency.
FBC - there may be anaemia, mild eosinophilia and lymphocytosis.
Glucose - often low in children.
LFTs - may be raised liver transaminases.
Cortisol - usually reduced:
Levels are highest between 8 am and 9 am when blood test should be taken.
Specialist advice should be sought in interpreting results for people on shift work (diurnal variation may be
altered), people taking oestrogen (can increase cortisol-binding globulin production by the liver), pregnant
women and people on long-term steroids.
Different assays are used so refer to local reference ranges. Generally levels of <100 nmol/L should prompt
urgent investigation or admission, and levels of 100-150 nmol/L require further investigation.
Salivary cortisol has been used for diagnosis, but not yet fully validated.
ACTH (also known as corticotropin) - when measured together with cortisol allows differentiation of primary
vs secondary insufficiency:
Levels are raised in primary insufficiency.
Levels are low or low normal in secondary insufficiency.
Plasma renin and aldosterone levels - will give an indication of mineralocorticoid activity. (Renin is often high
and aldosterone low in Addison's disease. Usually unaffected in secondary insufficiency.)

Management:
Both glucocorticoid and mineralocorticoid replacement are required.

Glucocorticoid replacement - hydrocortisone is the mainstay of treatment; usually 15-30 mg in three divided
doses with the highest dose in the morning (thus stimulating the normal diurnal adrenal rhythm). Twice daily
regimens are also used, although opinion on their benefit varies. A modified-release once-daily preparation is
also now available and licensed for use, and is still being evaluated.
During minor illness or minor surgery, glucocorticoid doses may be increased up to three times their normal
dose to avoid adrenal crisis, and up to ten times for major illness or major surgery.
If there is coexistent thyroid deficiency then thyroid hormones should not be replaced before glucocorticoids,
as a crisis may be precipitated.
Mineralocorticoid replacement - this is usually required in primary adrenal insufficiency. Fludrocortisone is
used and the usual adult dose is 50-300 micrograms per day, depending on activity levels, weight and
metabolism.
Assessing adequacy of therapy involves monitoring symptoms and signs, measuring blood pressure and
looking for postural hypotension and normalising of serum electrolytes (Na and K).
Signs of over-replacement include raised blood pressure, thin skin, striae, easy bruising, glucose intolerance,
hyperglycaemia and electrolyte abnormalities.
Signs of under-replacement are the symptoms of Addison's disease persisting, ie fatigue, postural
hypotension, nausea, weight loss, and salt craving.

1504. A 10yo boy develops nasal bleeding. What is the best way to stop the bleeding from the nose?
a. Pressure over base of the nose
b. Ice packs
c. Pressure over the soft tissues
d. Nasal packing
e. Surgery

Answer: C
The classification of nosebleeds is as anterior or posterior, depending upon the source of bleeding. The blood
supply to the nose is derived from branches of the internal (anterior and posterior ethmoid arteries) and
external carotid arteries (sphenopalatine and branches of the internal maxillary arteries). Bleeding usually
occurs when the mucosa is eroded and vessels become exposed and subsequently break.
Epistaxis is usually benign, self-limiting and spontaneous. The majority are caused by simple trauma. Although
most incidents are not life-threatening, they can cause significant parental concern when they occur in
children.
Management of epistaxis:
Resuscitate the patient (if necessary) - remember the ABCD(E) of resuscitation.
Ask the patient to sit upright, leaning slightly forward, and to squeeze the bottom part of the nose (NOT the
bridge of the nose) for 10-20 minutes to try to stop the bleeding. The patient should breathe through the
mouth and spit out any blood/saliva into a bowl. An ice pack on the bridge of the nose may help.
Monitor the patient's pulse and blood pressure.
If bleeding has stopped after this time (as it does in most cases) proceed to inspect the nose, using a nasal
speculum; consider cautery.
If the history is of severe and prolonged bleeding, get expert help - and watch carefully for signs of
hypovolemia.

1505. A pt came to the hosp with a complaint of severe chest pain lasting for >1h. Following ECG test, pt
revealed to have ST depression. He was already on aspirin. What is the most specific tx for this pt?
a. GTN
b. Simvastatin
c. Clopidogrel
d. BB
e. LMWH

Answer: Key says B but i think A (should be E- most specific)


Non-ST-elevation ACS (NSTE-ACS): patients present with acute chest pain but without persistent ST-segment
elevation. The ECG shows persistent or transient ST-segment depression or T-wave inversion, flat T waves,
pseudo-normalisation of T waves, or no ECG changes at presentation.
NSTE-ACS is further divided into:
Unstable angina: normal troponin levels.
Non-ST-elevation MI (NSTEMI): a rise in troponin levels.
The presentation of unstable angina and NSTEMI may be indistinguishable, and also indistinguishable from
acute STEMI.
NSTE-ACS can present in a variety of ways, including:[2]
Prolonged (longer than 20 minutes) anginal pain at rest.
New-onset angina with limitation of daily activities.
Recent destabilisation of previously stable angina, with moderate or severe limitation of daily activities.
Post-myocardial infarction angina.

In unstable angina (and non-Q wave infarction), the ECG typically shows T-wave inversion or ST-segment
depression, but the ECG may be normal if some time has elapsed since the last episode of pain.

Cardiac enzymes:
Within the first 6 hours, the sensitivity of troponins is superior to CK-MB for the detection of myocardial
infarction.
Troponin I and T become detectable in serum 3-6 hours after infarction, peak at 12-24 hours, and remain
raised for up to 14 days.
Troponins are therefore usually tested 6 and 12 hours after the onset of pain.
In patients with unstable angina, minor troponin elevations may identify patients at risk for subsequent
cardiac events and death. Elevated troponin levels indicate an increased risk of mortality in both the short
term and long term. Patients with chest pain and elevated troponin levels should remain in hospital for further
assessment, including an inpatient coronary angiogram.

The National Institute for Health and Care Excellence (NICE) recommends the Global Registry of Acute Cardiac
Events (GRACE) risk score.

Immediate management of a suspected ACS


Arrange urgent hospital admission (phone 999/112/911).
Resuscitation as required.
Pain relief: GTN and/or an intravenous opioid (use an antiemetic with opioids).
Single loading dose of 300 mg aspirin unless the person is allergic.
A resting 12-lead ECG - but don't delay transfer to hospital.
Assess oxygen saturation, using pulse oximetry before hospital admission if possible. Give oxygen if oxygen
saturation (SpO2) is less than 94% with no risk of hypercapnic respiratory failure; aim for SpO2 of 94-98% (aim
for 88-92% for people with chronic obstructive pulmonary disease).

1506. A 69yo woman presents with a sudden onset of weakness of her right arm and leg. She is known to be
hypertensive. There has been no headache, LOC, visual, speech or sensory symptoms. Exam:
BP=180/90mmHg, pulse=100 and regular heart sounds, no carotid bruit. Higher mental function tests are
normal. No apraxia or neglect. Speech, swallowing and sensation are normal. There are no visual field defects.
There is a mild facial weakness sparing the forehead. The right arm and leg are flaccid and weak. Reflexes and
tone are normal. There is a right extensor plantar response. What is the most likely cause of this pts
symptoms?
a. Cardioembolic stroke
b. Lacunar stroke
c. Right internal carotid artery atheroembolic stroke
d. Right internal carotid artery dissection
e. Right vertebral artery atheroembolic stroke

Answer: B
Lacunar infarcts (25%):
Small infarcts around the basal ganglia, internal capsule, thalamus and pons.
May cause pure motor, pure sensory, mixed motor and sensory signs, or ataxia.
Intact cognition/consciousness.

1507. A 34yo man has an intermittent epigastric pain for 3wks. It is worse by food but helped by some tablets
he obtained from the pharmacy. He had a similar episode 3yrs ago and his doctor gave him a course of 3 types
of tablets at the time. What is the most appropriate next inv?
a. Abdomen US
b. Barium meal
c. Serum H.Pylori antibodies
d. C13 urea breath test
e. Upper GI endoscopy
Answer: D
Testing for H. pylori. Test using a carbon-13 urea breath test or a stool antigen test, or laboratory-based
serology where its performance has been locally validated. If re-testing is required, a carbon-13 urea breath
test is the chosen test. There is currently insufficient evidence to recommend the stool antigen test as a test of
eradication. Office-based serological testing is not currently recommended because of its inadequate
performance.

1508. A girl with sickle cell anemia has painful bleeding and vaso-occlusive crisis during her periods. What is
the best possible management for this pt?
a. COCP
b. Tranexamic acid
c. Copper IUS
d. UAE
e. Depot provera

Answer: E
NICE recommends DMPA as a management option for heavy menstrual bleeding in patients with sickle cell
anemia. It also improves symptoms of dysmenorrhoea and endometriosis.
No long-acting progestogen injection affects blood pressure.
Limited evidence suggests that the severity of the pain of sickle cell crises may be less in women on DMPA. It is
a safe option although there is a lack of evidence regarding the risks of venous thrombosis in women with
sickle cell disease.

1509. A 70yo pt comes with swelling in the parotid region for the last 10y. Exam: gland is soft and cystic.
Choose the most probable dx?
a. Pleomorphic adenoma
b. Carcinoma of the salivary glands
c. Mikuliczs disease
d. Adenoid cystic carcinoma
e. Parotid duct stones

Answer: D
Adenoid cystic carcinoma (ACC) is an uncommon and unusually indolent cancer arising within glands and
occurring mainly in the head and neck but also in the breast, trachea, lacrimal glands, skin and vulva.It can
present as a painless slow-growing mass in the face or mouth.

1510. A 74yo man has been admitted unconscious with no hx. He has a GCS=6 and a dilated left pupil which
becomes insensitive to light. What is the single most likely dx?
a. Extradural hematoma
b. Meningitis
c. Opioid OD
d. Pontine hemorrhage
e. SAH

Answer: E
Ruptured posterior communicating artery aneurysm causes unilateral 3rd nerve palsy and also SAH.(EDH also
producing 3rd nerve palsy due to uncal herniation and raised ICF and compressed brain but absent of relevant
history regarding EDH such as head trauma).
In elderly dura mater becomes more sticky and adherent to skull and it hardly allow any blood to take place
extradurally! So in elderly extradural hematoma is very unlikely.
Subarachnoid haemorrhage (SAH) is usually the result of bleeding from a berry aneurysm in the Circle of Willis.
These are called berry aneurysms because of their shape. They were once thought to be mostly congenital but
it is now thought that the aetiology may involve susceptibility of the elastic lamina, in some patients, to
stressors such as hypertension and atherosclerosis.
There may be warning symptoms in the three weeks prior to SAH that represent small leaks. These are called
sentinel bleeds or expansion of the aneurysm. These are usually headaches with the characteristics of SAH but
which resolve by themselves without further symptoms. They are estimated to occur in 10-15% of patients.

The most common symptoms are headache (48%), dizziness (10%), orbital pain (7%), diplopia (4%) and visual
loss (4%).
Signs may accompany these sentinel bleeds: sensory or motor disturbance (6%), seizures (4%), ptosis (3%),
bruits (3%) and dysphasia (2%).
If a sentinel bleed is suspected, patients should be admitted urgently for investigations (treat as if an SAH has
occurred).
Examination
Conscious level: on admission to hospital two thirds have a depressed level of consciousness, of whom half are
in coma. However, SAH patients can also walk into the surgery, complaining of sudden onset of headache.
Neck stiffness may occur due to meningeal irritation by blood in the CSF, but it is not invariable.
Ophthalmoscopy will show intraocular haemorrhages in around 15%, especially in those with a depressed
level of consciousness.
Isolated pupillary dilation with loss of light reflex may indicate brain herniation as a result of rising intracranial
pressure.
There may be focal neurological signs, suggestive of a stroke. Complete or partial palsy of the oculomotor
nerve is well recognised, especially with rupture of aneurysms of the internal carotid artery at the origin of the
posterior communicating artery.
Intraocular haemorrhage may occur in response to the raised pressure and is more common in more severe
SAH
Oculomotor nerve impairment may indicate bleeding from the posterior communicating artery.
Hypertension is a risk factor for the condition but a marked rise in blood pressure may also occur as a
sympathetic reflex following intracerebral haemorrhage. This sympathetic reflex can raise blood pressure to
life-threatening levels, and surges of adrenaline (epinephrine) may contribute to associated cardiac
arrhythmias which may both confuse the diagnosis and further threaten the patient.
In 3% of cases cardiac arrest follows SAH.
SAH in a person known to have seizures is suggestive of an arteriovenous malformation. New-onset seizures
are more indicative of ruptured berry aneurysm.

1511. A 27yo man presents to the ED with 2d hx of severe headache and pyrexia (38.9C). CT: petechial
hemorrhage in the temporal and inf frontal lobes. What is the most likely dx?
a. Brain abscess
b. Meningococcal meningitis
c. Cerebral malaria
d. Herpes simplex encephalitis
e. New variant CID

Answer: D
Herpes simplex encephalitis (HSE) is recognised worldwide as the most frequent infectious encephalitis.
In children older than 3 months and in adults: HSE is usually caused by herpes simplex virus type 1 (HSV-1) and
is localised to the temporal and frontal lobes.
In neonates: HSE is usually caused by herpes simplex virus type 2 (HSV-2) acquired at the time of delivery, and
brain involvement is generalised.
Other herpes viruses may cause encephalitis but much less frequently than HSV. However, cytomegalovirus
(CMV) encephalitis should be considered in those with immunodeficiency.

1512. A 44yo woman with memory loss, poor concentration and inability to recognize household
projects. She has right-handed involuntary writhing movement. There is strong fam hx of similar
complain. What is the single most likely dx?
a. Pics dementia
b. Wilsons disease
c. Huntingtons disease
d. HIV associated dementia
e. Fronto-temporal dementia

Answer: C
Huntington's disease is associated with cell loss within the basal ganglia and cortex. It is an autosomal-
dominant, progressive neurodegenerative disorder with a distinct phenotype, including chorea and dystonia,
incoordination, cognitive decline, and behavioural difficulties.[1] Huntington's disease was first described by
George Huntington in 1872. The disease is associated with increases in the length of a cysteine-adenosine-
guanine (CAG) triplet repeat present in a gene called 'huntingtin' located on chromosome 4p16.3.Huntington's
disease is the most common genetic cause of chorea.
Signs and Symptoms:
The mean age at onset of symptoms is 30-50 years.
In some cases symptoms start before the age of 20 years with behavioural disturbances and learning
difficulties at school (juvenile Huntington's disease)
Early signs may be personality change, self-neglect, apathy with clumsiness, fidgeting with fleeting facial
grimaces.
Huntington's disease then leads to progressive chorea, rigidity and dementia. It is frequently associated with
seizures.
Chorea is initially mild but may be severe and cause uncontrollable limb movements.
As the disease progresses, chorea is gradually replaced by dystonia and Parkinsonian features.
Dysarthria, dysphagia and abnormal eye movements are common. There may also be other movement
disorders - eg, tics and myoclonus.
Huntington's disease is associated with increasing depression, bradykinesia, cognitive impairment and
aggression as the disease progresses.Behavioural difficulties include apathy or lack of initiative, dysphoria,
irritability, agitation or anxiety, poor self-care, poor judgment and inflexibility.[1]Late features include
spasticity, clonus, supranuclear gaze palsy and extensor plantar responses. The rate of cognitive decline is very
variable.
Investigations:
MRI and CT scans in moderate-to-severe Huntington's disease show a loss of striatal volume and increased
size of the frontal horns of the lateral ventricles, but scans are usually unhelpful for diagnosis of early disorder.
If genetic testing is considered then extensive genetic counselling in a specialist unit is required in view of the
implications of an untreatable, familial, progressive, neurodegenerative disease.
Treatment:
Current drug therapy has no effect on the progression of disability.
Hyperkinesias and psychiatric symptoms may respond well to pharmacotherapy, but neuropsychological
deficits and dementia remain untreatable.[8]
Patients, their families and carers require a great deal of physical and emotional support.
Chorea:
Benzodiazepines, valproic acid, dopamine-depleting agents (eg, tetrabenazine) and neuroleptics may be
useful.

1513. A 54yo man has collapsed suddenly following a headache. He has hypertension and takes warfarin for
prosthetic heart valve. GCS=4 and dilated left pupil. What is the single most likely dx?
a. Ant circulation stroke
b. Post circulation stroke
c. Intracerebral hemorrhage
d. Intracerebellar hemorrhage
e. Pontine hemorrhage

Answer: C
Intracerebral hemorrhage occurs when a diseased blood vessel within the brain bursts, allowing blood to leak
inside the brain. (The name means within the cerebrum or brain). The sudden increase in pressure within the
brain can cause damage to the brain cells surrounding the blood. If the amount of blood increases rapidly, the
sudden buildup in pressure can lead to unconsciousness or death. Intracerebral hemorrhage usually occurs in
selected parts of the brain, including the basal ganglia, cerebellum, brainstem, or cortex.

What causes it?


The most common cause of intracerebral hemorrhage is high blood pressure (hypertension). Since high blood
pressure by itself often causes no symptoms, many people with intracranial hemorrhage are not aware that
they have high blood pressure, or that it needs to be treated. Less common causes of intracerebral
hemorrhage include trauma, infections, tumors, blood clotting deficiencies, and abnormalities in blood vessels
(such as arteriovenous malformations). View an interactive tutorial on arteriovenous malformations from the
Toronto Brain Vascular Malformation Study Group.

1514. A 5wk breast fed baby whose birth weight was 3.5kg and is now 4.5kg is thriving well but is deeply
jaundiced. What is the most likely dx?
a. Galactosemia
b. Breast milk jaundice
c. Thalassemia
d. Sickle cell disease
e. Congenital storage disorder

Answer: B (Breast milk jaundice: the baby is well and the jaundice usually resolves by six weeks)
Jaundice is clinically detectable in the newborn when the serum bilirubin levels are greater than 85 mol/L.
This occurs in approximately 60% of term infants and 80% of preterm infants. Hyperbilirubinemia is either
unconjugated (which is potentially toxic but may be physiological or pathological) or conjugated (not toxic but
always pathological). Without treatment, high levels of unconjugated bilirubin may lead to kernicterus.

Other causes of jaundice include:


Physiological jaundice:
This results from increased erythrocyte breakdown and immature liver function.
It presents at 2 or 3 days old, begins to disappear towards the end of the first week and has resolved by day
10.
The bilirubin level does not usually rise above 200 mol/L and the baby remains well.
However, the bilirubin level may go much higher if the baby is premature or if there is increased red cell
breakdown - eg, extensive bruising, cephalohematoma.
Early neonatal jaundice (onset less than 24 hours):
Haemolytic disease: eg, haemolytic disease of the newborn (rhesus), ABO incompatibility, glucose-6-
phosphate dehydrogenase deficiency, spherocytosis.
Infection: congenital (eg, toxoplasmosis, rubella, cytomegalovirus (CMV), herpes simplex, syphilis) or postnatal
infection.
Increased haemolysis due to haematoma.
Maternal autoimmune haemolytic anaemia: eg, systemic lupus erythematosus.
Crigler-Najjar syndrome or Dubin-Johnson syndrome.
Gilbert's syndrome.
Prolonged jaundice (jaundice lasting for longer than 14 days in term infants and 21 days in preterm infants):
Infection - eg, urinary tract infection.
Hypothyroidism, hypopituitarism.
Galactosaemia.
Breast milk jaundice: the baby is well and the jaundice usually resolves by six weeks but occasionally
continues for up to four months.
Gastrointestinal (GI): biliary atresia, choledochal cyst, neonatal hepatitis.
Conjugated hyperbilirubinemia:
Infection.
Parenteral nutrition.
Cystic fibrosis.
Metabolic: alpha-1-antitrypsin deficiency, galactosaemia, aminoacidurias, organoacidaemias.
GI: biliary atresia, choledochal cyst, neonatal hepatitis.
Endocrine: hypothyroidism, hypopituitarism.

1515. A 71yo man with no prv immediate hx is brought to the ED by his wife who says he has become
progressively more forgetful, tends to lose his temper and is emotionally labile. There is no hx of infectious
disease or trauma. Whats the single most likely dx?
a. Picks dementia
b. Fronto-temporal dementia
c. Huntingtons disease
d. Alzheimers disease
e. Vascular dementia
Answer: D
Alzheimer's disease is the most common cause of dementia, and involves a progressive degeneration of the
cerebral cortex. There is widespread cortical atrophy. Neurons affected develop surrounding amyloid plaques,
neurofibrillary tangles, and produce less acetylcholine. The cause is not yet known. Patients experience
irreversible global, progressive impairment of brain function, leading to reduced intellectual ability.
Diagnostic criteria
The National Institute for Health and Care Excellence (NICE) guidelines recommend the National Institute of
Neurological and Communicative Disorders and Stroke and the Alzheimer's Disease and Related Disorders
Association (NINCDS/ADRDA) diagnostic criteria be used for the assessment of Alzheimer's disease.
Alternatives are the ICD-10 or DSM-IV (now DSM-5 since the guidelines were written) criteria.[10][11]

The NINCDS/ADRDA criteria were proposed in 1984 and revised in 2011.[12] Core features for diagnosis of
Alzheimer's disease include:

Probable Alzheimer's disease


Dementia established by clinical examination and neuropsychological tests.
Deficits in two or more areas of cognition.
Insidious onset over months to years, and progressive worsening of memory and other cognitive functions.
No disturbance of consciousness.
Onset between ages of 40 and 90. (Criterion removed in latest revision.)
Absence of systemic disorders or other brain diseases that could account for the symptoms.
Possible Alzheimer's disease
Dementia with an atypical onset or course (ie sudden onset or insufficient documentation of progressive
decline); OR
Aetiologically mixed presentation (ie other criteria fit the diagnosis, but features of other brain disorders or
causes of dementia are present).
Mild cognitive impairment due to Alzheimer's disease
Newer diagnostic criteria have attempted to classify the symptomatic, pre-dementia stage of Alzheimer's
disease. The work group which revised the NINCDS/ADRDA criteria in 2011 refers to this stage as "mild
cognitive impairment" with clinical diagnostic criteria as follows:[13]

Concern regarding a change in cognition (from patient, informer or clinician)


Impairment of one or more cognitive domains
Preservation of independence in functional abilities
Not having the features of dementia

1516. A 38yo woman with hemophilia who received several blood transfusions a few years ago
presents with irritability and increasing memory deficit. She is unable to speak properly. He is on
anti-TB tx. What is the single most likely dx?
a. Creutzfeldt Jacob disease
b. Drug toxicity
c. Vascular dementia
d. HIV associated dementia
e. Space occupying lesion
Answer: D
Repeated BT ---> patient got HIV inf ( immunocompromised)---> developed TB --> HIV induced dementia
The use of plasma-derived factor VIII, before the availability of recombinant products, led to infection with
HIV, hepatitis B virus (HBV) and hepatitis C virus (HCV) in many haemophiliacs. One case of likely transmission
of variant Creutzfeldt-Jakob disease (vCJD) by UK factor VIII concentrates has been reported in an elderly
haemophilic patient in the UK. The recent report of a blood test that may be used to detect vCJD has raised
the possibility of a new way to identify infected individuals, perhaps even before the onset of clinical
symptoms.

1517. An 18yo girl has menorrhagia and dysmenorrhea and requires contraception. What drug will you
give her?
a. COCP
b. Mirena coil
c. Copper T
d. UAE
e. Depo provera
Answer: i think it should be B but in key it's A
Menorrhagia+dysmenorrhea------give COCP
Acc to nice guidelines the management steps of menorrhagia are:
When a first pharmaceutical treatment has proved ineffective then a second pharmaceutical treatment should
be considered rather than immediate referral to surgery. If there is iron deficiency it should be corrected with
oral iron.

First-line treatment
This is the levonorgestrel-releasing intrauterine system (IUS) - Mirena. This is long-term treatment and
should be left in situ for at least 12 months.

One recent study has shown that women with menorrhagia reported more improvement in bleeding and
quality of life with the levonorgestrel-releasing IUS than with other treatments available in primary care. In
addition, they were more likely to continue with this treatment.
However, the rate of discontinuation of Mirena treatment has been shown to be relatively high - 16% at 12
months and 28% by 2 years.
Second-line treatment
This includes tranexamic acid, mefenamic acid or the combined oral contraceptive pill (COCP):

Mefenamic acid works by inhibiting prostaglandin synthesis. It reduces menstrual loss by around 25% in three
quarters of women and is better tolerated than tranexamic acid.
Tranexamic acid is a plasminogen-activator inhibitor. It inhibits the dissolution of thrombosis that leads to
menstrual flow. It can reduce flow by up to 50%. It is most effective at reducing menstrual loss associated with
IUCDs, fibroids and bleeding diathesis. Other non-steroidal anti-inflammatory drugs (NSAIDs) may also be
used. Side-effects include nausea, vomiting and diarrhoea. If there is disturbance in colour vision then it
should be discontinued.
The COCP suppresses production of gonadotrophins and reduces menstrual blood loss by around 50%. It can
improve dysmenorrhoea, lighten periods, regulate the cycle, improve premenstrual symptoms, reduce the
risk of PID and protect the ovaries and endometrium against cancer.
Third-line treatment
This is with norethisterone.

The dose is 15 mg daily, from day 5 to 26 (or injected long-acting progestogens). This can result in a significant
reduction in menstrual blood loss, although women tend to find the treatment less acceptable than
intrauterine levonorgestrel. This regimen of progestogen may have a role in the short-term treatment of
menorrhagia.
However, there are very limited data regarding the use of progestogens and of oestrogens and progestogens
in combination in the treatment of irregular menstrual bleeding associated with anovulation. There is still no
consensus about which regimens are the most effective.[9]
In secondary care 3-4 months of a gonadotrophin-releasing hormone (GnRH) analogue may be offered before
hysterectomy or myomectomy, where the uterus is enlarged or distorted by fibroids. It is also a reasonable
choice of therapy if other methods are contra-indicated - but 'add-back' hormone therapy will be needed if
continued for >6 months.

In the acute situation, a bleeding episode may be so disabling for the woman that treatment with high-dose
norethisterone (30 mg daily) needs to be used. This is continued until bleeding is controlled, but is then tailed
off.

Surgical options
The choice of treatment will depend on both the uterine size and the patient's desire to retain her uterus.
1518. A pt of tuberculous abscess with the hx of prv abscess drainage presented with fever and
tenderness between L2/L3 vertebra. Which is the best inv for this pt?
a. XR
b. CT
c. US
d. MRI
e. Blood culture
Answer: D
Investigations for pott's disease:
MRI scanning may demonstrate the extent of spinal compression and can show changes at an early stage.
Bone elements visible within the swelling, or abscesses, are strongly suggestive of Pott's disease rather than
malignancy.
CT scans and nuclear bone scans can also be used but MRI is best to assess risk to the spinal cord.

1519. A 4yo child presents with repeated chest infections. He has yellow discoloration of sclera and the
mother gives a hx of diarrhea as well. What is the single inv most likely to lead to a dx?
a. Sweat chloride test
b. Anti-endomysial antiboides
c. LFT
d. Jejunal biopsy
e. TFT
Answer: A
Features of cystic fibrosis:
High sodium sweat
Primary secretion of sweat duct is normal but CFTR does not absorb chloride ions, which remain in the lumen
and prevent sodium absorption.

Pancreatic insufficiency
Production of pancreatic enzymes is normal but defects in ion transport produce relative dehydration of
pancreatic secretions, causing their stagnation in the pancreatic ducts.

Biliary disease
Defective ion transfer across the bile duct causes reduced movement of water in the lumen so that bile
becomes concentrated, causing plugging and local damage.

Gastrointestinal disease
Low-volume secretions of increased viscosity, changes in fluid movement across both the small and large
intestine and dehydrated biliary and pancreatic secretions cause intraluminal water deficiency.

Respiratory disease
Dehydration of the airway surfaces reduces mucociliary clearance and favours bacterial colonisation, local
bacterial defences are impaired by local salt concentrations and bacterial adherence is increased by changes in
cell surface glycoproteins.

Increased bacterial colonisation and reduced clearance produce inflammatory lung damage due to an
exuberant neutrophilic response involving mediators such as IL8 and neutrophil elastase.
1520. An 82yo woman has been admitted from a nursing home with dense hemiplegia and
homonymous hemianopia. She is dysphasic. What vessel is most likely to be involved?
a. Ant cerebral artery
b. Mid cerebral artery
c. Post cerebral artery
d. Internal carotid artery
e. Post inf cerebellar artery
Answer: B
Areas supplied by the middle cerebral artery include:

The bulk of the lateral surface of the hemisphere; except for the superior inch of the frontal and parietal lobe
(anterior cerebral artery), and the inferior part of the temporal lobe.
Superior division supplies latero inferior frontal lobe (location of Broca's area i.e. language expression)
Inferior division supplies lateral temporal lobe (location of Wernicke's area i.e. language comprehension)
Deep branches supply the basal ganglia as well as the internal capsule.
Occlusion of the middle cerebral artery results in Middle cerebral artery syndrome, potentially showing the
following defects:

Paralysis (-plegia) or weakness (-paresis) of the contralateral face and arm (faciobrachial)
Sensory loss of the contralateral face and arm.
Damage to the dominant hemisphere (usually the left hemisphere) results in aphasia i.e. Broca's or Wernicke's
Damage to the non-dominant hemisphere (usually the right hemisphere) results in contralateral neglect
syndrome
Large MCA infarcts often have dviation conjugue, a gaze preference towards the side of the lesion,
especially during the acute period. Contralateral homonymous hemianopsia is often present.

1521. A pt is dx with SIADH. Choose the appropriate biochemical change.


a. Plasma Na+ decrease and urine osmolarity increase
b. Plasma Na+ decrease and urine osmolarity decrease
c. Plasma Na+ increase and urine osmolarity decrease
d. Plasma Na+ increase and urine osmolarity increase
Answer: A
Syndrome of inappropriate ADH secretion (SIADH)[8][9]
Inappropriate ADH secretion from posterior pituitary or from ectopic source despite low serum osmolality.

Major diagnostic features


Hyponatraemia.
Plasma hypo-osmolality proportional to hyponatraemia.
Inappropriately elevated urine osmolality (>100 mOsmol/kg) commonly > plasma osmolarity.
Persistent urine [Na+] >40 mmol/L with normal salt intake.
Euvolemia.
Normal thyroid and adrenal function.
Extra features include an elevated ADH level and low blood uric acid level.

1522. A newborn that is electively intubated at birth and is due for surgery 48h after birth. The
condition was suspected on antenatal US on CXR. What is the most likely dx?
a. CF
b. Congenital diaphragmatic hernia
c. Congenital cystic adenomatoid malformation
d. RDS
e. Alpha 1 antitrypsin deficiency
Answer: B (Many infants are now diagnosed in utero by ultrasound scan.
CXR or ultrasound scan will confirm the diagnosis in a neonate who has not previously been diagnosed.)
Congenital diaphragmatic hernia is produced by the failure of the diaphragm to fuse properly during fetal
development, allowing the abdominal organs to migrate up into the chest cavity. This results in the two
primary problems underpinning congenital diaphragmatic hernias: pulmonary hypertension and pulmonary
hypoplasia. This is compounded by dysfunction of the surfactant. Associated diseases, notably cardiac
abnormalities, are frequent.
Congenital diaphragmatic hernia occurs in 1 in 2,500 births. It accounts for 8% of all major congenital defects.
Males are more commonly affected than females with a ratio of 3:2.
Right-sided lesions are rare (10-15%) compared with left-sided (85%) as the liver plugs the opening. Right
congenital diaphragmatic hernia carries a disproportionately high mortality and morbidity.[4][5]
Many cases are now diagnosed prenatally on routine ultrasound scans or scans following the discovery of
polyhydramnios in the mother.[7] This allows for detailed planning of the delivery and immediate aftercare of
the neonate. Previously undiagnosed cases still occur and these will usually present at or very soon after birth,
depending on the severity of the hernia. Signs include:

Cyanosis soon after birth.


Tachypnoea.
Tachycardia.
Asymmetry of the chest wall.
Absent breath sounds on one side of the chest, usually the left with the heart shifted to the right.
Bowel sounds audible over the chest wall.
The abdomen possibly feels 'less full' on palpation.

Management:
Children born without a prior diagnosis of congenital diaphragmatic hernia, present a paediatric emergency
and the initial management must be aimed at reducing the pressure in the chest and increasing oxygenation. If
bowel sounds are heard in the chest of a neonate who has respiratory distress, the child should be
resuscitated in a 'head up', rather than the more usual 'head down', position.
Endotracheal intubation and mechanical ventilation are required for all infants with severe disease who
present in the first hours of life.
Avoid bag-and-mask ventilation in the delivery room because the stomach and intestines become distended
with air and further impair lung function.
Passage of an orogastric tube will facilitate location of the stomach on X-ray, as well as permitting
decompression of the stomach.
Use of surfactant at an early stage may be beneficial.
Blood gases should be monitored and an indwelling arterial catheter is advantageous.
An indwelling venous catheter will enable administration of drugs (eg, inotropic agents and hypertonic
solutions).

Surgery consists of replacing the abdominal organs within the abdominal cavity and repairing the
diaphragmatic defect. It used to be performed early, in the first 24 hours of life. Some suggest that repair 24
hours after stabilisation is ideal but delays of up to 7 or 10 days are often well tolerated. Many surgeons now
prefer to operate when echocardiography has shown normal pulmonary arterial pressures maintained for at
least 24 to 48 hours. Therefore, delayed surgical repair is now usual, performed as an elective procedure, and
rarely as an out-of-hours procedure.

1523. A 63yo male undergoes abdominal surgery. On Monday morning, 3d post-op, repeat samples
confirm serum K+=7.1mmol/l. His ECG shows broad QRS complexes. Which one of the following
can be used as an effective tx for this pts hyperkalemia?
a. Calcium chloride IV
b. Calcium gluconate IV
c. Insulin subcutaneously
d. Furosemide IV
Answer: B
When arrhythmias occur, or when potassium levels exceed 6.5 mmol/l, emergency lowering of potassium
levels is needed. Several agents are used to transiently lower K+ levels. To treat myocardial excitability caused
by hyperkalemia, Calcium (calcium chloride or calcium gluconate) increases threshold potential through a
mechanism that is still unclear, thus restoring normal gradient between threshold potential and resting
membrane potential, which is elevated abnormally in hyperkalemia. Other agents used to shift K in the cells
are insulin or salbutamol. They control Hyperkalemia temporarily.

1524. A 25yo man attended in urological OPD has single testis. He was inv and other testis was located in the
abdomen. What is the best management plan for this pt?
a. Short trial of HCG
b. Orchidectomy
c. Orchidopexy
d. Reassurance
Answer: C
(should be B: Before 2 years of life, Orchidopexy may be done. But after that, tissue usually atrophy, if it
doesn't, there is a high probability of it developing into a tumour. Some Urologists use 12-18 months to do
Orchidopexy in Cryptorchidism, others have 9-12 months as reference age for pexy.)
An undescended testicle (testis) is more common in boys who are born prematurely. Although in the majority
of cases the testis descends by the age of 6 months, some boys will need an operation. This is called an
orchidopexy. This operation brings the testis down from the tummy (abdomen) into the testes' sac (scrotum).
There is an increased risk of infertility and also cancer if the testis remains in the abdomen.

1525. A 56yo male who presented with epilepsy like symptoms has been dx with an intracranial space
occupying lesion. He now complains of thirst and mild dehydration. His blood glucose is also increased. What
is the single most appropriate immediate tx?
a. Insulin
b. IV fluids
c. Stop dexamethasone
d. Stop sodium valproate and change to another anti-epileptic
Answer:B
Diabetes insipidus is a condition in which your ability to control the balance of water within your body is not
working properly. Your kidneys are not able to retain water and this causes you to pass large amounts of
urine. Because of this, you become more thirsty and want to drink more. There are two different types of
diabetes insipidus: cranial and nephrogenic.Treatment includes drinking plenty of fluids so that you do not
become dehydrated. Treatment with medicines may be also needed for both types of diabetes insipidus.
1526. A mother brings her newborn to the hosp concerned about a blue patch on the buttocks. The newborn
is of mixed race and was delivered normally. What is the most appropriate
management?
a. Reassurance
b. CBC
c. XR
d. Plt count
Answer: A
A Mongolian spot, also known as Mongolian blue spot, congenital dermal melanocytosis,[1] and dermal
melanocytosis is a benign, flat, congenital birthmark with wavy borders and irregular shape. It normally
disappears three to five years after birth and almost always by puberty.[6] The most common color is blue,
although they can be blue-gray, blue-black or even deep brown.

1527. The ECG of a 65yo shows absent P waves, narrow QRS complex, ventricular rate of 120bpm and irregular
R-R interval. What is the most probable dx?
a. A-fib
b. A-flutter
c. SVT
d. Mobitz type 1 2nd degree heart block
e. Sinus tachycardia
Answer: A
Atrial fibrillation (AF) is the most common sustained cardiac arrhythmia, characterised by irregularly irregular
ventricular pulse and loss of association between the cardiac apex beat and radial pulsation. Loss of active
ventricular filling is associated with:

Stagnation of blood in the atria leading to thrombus formation and a risk of embolism, increasing the risk of
stroke.
Reduction in cardiac output (especially during exercise) which may lead to heart failure.

1528. The ECG of an 80yo pt of IHD shows sawtooth like waves, QRS complex of 80ms, ventricular rate of
150bpm and regular R-R interval. What is the most probable dx?
a. A-fib
b. A-flutter
c. SVT
d. Mobitz type 1 2nd degree heart block
e. Sinus tachycardia

Answer: B
The common form of type I atrial flutter has saw-tooth flutter waves, best seen in leads II, III, and aVF, with
atrial rates of 240-340. A 12-lead ECG is gold standard for diagnosis.[4]
ATRIAL FLUTTER
In Atrial flutter, Pulse may be irregular or regular, but is usually rapid. Arteriovenous conduction is usually 2:1,
making the ventricular rate approximately 150 bpm. 1:1 atrioventricular (AV) conduction may lead to
haemodynamic collapse. Carotid massage may decrease the ventricular rate

1529. A man brings his wife into the ED after finding her unconscious at home. He says at breakfast
time she had complained of sudden severe headache. What is the most appropriate inv?
a. MRI
b. XR
c. CT brain
d. Carotid Doppler
Answer: C

The most characteristic presentation of subarachnoid hemorrhage is a sudden explosive headache. This may
last a few seconds or even a fraction of a second.
If SAH is suspected, CT scanning (without contrast) is the first line in investigation because of the
characteristically hyperdense appearance of blood in the basal cisterns.
Every patient in whom SAH is suspected should have a CT scan at the earliest opportunity. This should be done
immediately if the patient presents with sudden severe headache and as soon as possible in all other cases.
Treatment of SAH:
Rebleeding is the most imminent danger; a first aim is therefore occlusion of the aneurysm. Endovascular
obliteration by means of platinum spirals (coiling) is now the preferred mode of treatment, but some patients
require a direct neurosurgical approach (clipping).

1530. A 68yo lady with T2DM. Which drug should be prescribed?


a. Biguanides
b. Sulphonyl urea
c. Insulin
d. Lifestyle modifications
Answer: A
Pharmacologic Therapy
Early initiation of pharmacologic therapy is associated with improved glycemic control and reduced long-term
complications in type 2 diabetes. Drug classes used for the treatment of type 2 diabetes include the following:
Biguanides
Sulfonylureas
Meglitinide derivatives
Alpha-glucosidase inhibitors
Thiazolidinediones (TZDs)
Glucagonlike peptide1 (GLP-1) agonists
Dipeptidyl peptidase IV (DPP-4) inhibitors
Selective sodium-glucose transporter-2 (SGLT-2) inhibitors
Insulins
Amylinomimetics
Bile acid sequestrants
Dopamine agonists

Metformin is the only biguanide in clinical use.Metformin lowers basal and postprandial plasma glucose levels.
Its mechanisms of action differ from those of other classes of oral antidiabetic agents; metformin works by
decreasing hepatic gluconeogenesis production. It also decreases intestinal absorption of glucose and
improves insulin sensitivity by increasing peripheral glucose uptake and utilization. Unlike oral sulfonylureas,
metformin rarely causes hypoglycemia.

1531. In a laparoscopic mesh repair for hernia, when the trocar is inserted at midpoint between
umbilicus and ischial spine. What structure will be pierced?
a. Linea alba
b. Rectus muscle
c. Conjoint tendon
d. External and internal oblique muscles
e. Inguinal ligament

Answer: D
Structures in Anterior Abdominal Wall
In human anatomy, the layers of the abdominal wall are (from superficial to deep):
Skin
Subcutaneous tissue
Fascia
Camper's fascia - fatty superficial layer.
Scarpa's fascia - deep fibrous layer.
Muscle
External oblique muscle
Internal oblique muscle
Rectus abdominis
Transverse abdominal muscle
Pyramidalis muscle
Fascia transversalis
Peritoneum

1532. A 48yo man has intermittent left sided lower abdominal pain and feels generally unwell. He has lost his
appetite and has lost weight. Temp=38.3C and he has BP=190/100mmHg. What is the single inv most likely to
lead to dx?
a. Colonoscopy
b. Endomysial antibodies
c. Fasting serum glucose conc
d. TFT
e. US abdomen
Answer: E
Investigations recommended for left iliac fossa pain on patient.co. These should be tailored to the patient's
symptoms and the examination findings. In the GP setting there are a number of bedside tests that can be
done to aid diagnosis:

Dip urine for pus cells, leukocytes and/or nitrites if urinary tract infection (UTI) is suspected. Microscopic
haematuria is usually present in ureteric colic. It can also occur in abdominal aortic aneurysm.
Perform a pregnancy test if ectopic pregnancy or miscarriage is suspected.
If the pain is non-acute and can be managed in the GP setting, further investigations may be requested:
Blood tests may include FBC, renal function, LFTs.
Vaginal swab tests can help to exclude pelvic infection.
Ultrasound scanning can show ovarian or other mass.
Referral for further bowel investigations may be necessary - eg, referral under the two-week wait rule if bowel
carcinoma is suspected.

1533. A man with DM comes to the ED after he collapsed at home. His GCS=10. What should be the
next initial inv for this man?
a. Capillary blood sugar
b. MRI head
c. CT head
d. Serum electrolytes

Answer: A

Diabetic coma is a reversible form of coma found in people with diabetes mellitus. It is a medical emergency.
Three different types of diabetic coma are identified:
1. Severe low blood sugar in a diabetic person
2. Diabetic ketoacidosis advanced enough to result in unconsciousness from a combination of a severely
increased blood sugar level, dehydration and shock, and exhaustion
3. Hyperosmolar nonketotic coma in which an extremely high blood sugar level and dehydration alone are
sufficient to cause unconsciousness.
A quick look, and a glucose meter to determine the cause of unconsciousness in a patient with diabetes.
Laboratory confirmation can usually be obtained in half an hour or less. Other conditions that can cause
unconsciousness in a person with diabetes are stroke, uremic encephalopathy, alcohol, drug overdose, head
injury, or seizure.

1534. A 60yo DM pt presented with easy fatigability, weakness and numbness of hands and swollen feet.
Exam: pedal edema, sensory neuropathy and palpable liver and spleen. Urine: proteinuria. US abdomen:
enlarged kidney. Renal biopsy: amorphous homogenous substance that stained red with congo-red. What is
the dx?
a. DM retinopathy
b. Sarcoidosis
c. Wilms tumor
d. Amyloidosis
e. Glycogen storage disease

Answer: D (Aggregation of the congo-red dye and binding to amyloid fibrils tends to red-shift the absorption
spectrum, whereas binding to cellulose fibers has the opposite effect.)
Apple-green birefringence of Congo red stained preparates under polarized light is indicative for the presence
of amyloid fibrils.
Amyloidosis is a rare disease that results from accumulation of inappropriately folded proteins. These
misfolded proteins are called amyloids. When proteins that are normally soluble in water fold to become
amyloids, they become insoluble and deposit in organs or tissues, disrupting normal function. The type of
protein that is misfolded and the organ or tissue in which the misfolded proteins are deposited determines the
clinical manifestations of amyloidosis.
Amyloid deposition in the kidneys can cause nephrotic syndrome, which results from a reduction in the
kidney's ability to filter and hold on to proteins. In AA amyloidosis the kidneys are involved in 91-96% of
people,symptoms ranging from protein in the urine to nephrotic syndrome and rarely renal insufficiency.

1535. A 75yo man has urinary symptoms of hesitancy, frequency and nocturia. Rectal exam: large hard
prostate. What is the most appropriate inv?
a. CA 125
b. CA 153
c. CA 199
d. CEA
e. PSA

Answer: E
Prostate-specific antigen (PSA)

Cancer type: Prostate cancer


Tissue analyzed: Blood
PSA is produced exclusively by epithelial prostatic cells, both benign and malignant. It is also found in the
serum. Serum PSA is currently the best method of detecting localised prostatic cancer and monitoring
response to treatment but it lacks specificity, as it is also increased in most patients with benign prostatic
hyperplasia.

1536. A child suffering from CF developed pneumonia. Which organism is responsible for this
pneumonia?
a. H. influenza
b. Klebsiella
c. S. aureus
d. S. pneumonia
E. Pseudomonas

Answer: E
Chronic infections in cystic fibrosis,. Thick mucus in the lungs and sinuses provides an ideal breeding ground
for bacteria and fungi. People with cystic fibrosis may have frequent bouts of bronchitis or pneumonia
Pseudomonas aeruginosa: Typical pneumonia aspiration or inhalation green sputum, abscess
formation, Common cause of pneumonia in cystic fibrosis patients and those with severely compromised
respiratory defenses.

1537. An obese woman with hx of migraine presented with heavy bleeding during menstruation which is
painful and needs contraception too. What is the best possible management for this pt?
a. COCP
b. Mirena coil
c. Copper T
d. UAE
e. Depo provera

Answer: B

Management of menorrhagia:
First-line treatment
This is the levonorgestrel-releasing intrauterine system (IUS) - Mirena. This is long-term treatment and
should be left in situ for at least 12 months.[2]

Second-line treatment
This includes tranexamic acid, mefenamic acid or the combined oral contraceptive pill (COCP):

Mefenamic acid works by inhibiting prostaglandin synthesis. It reduces menstrual loss by around 25% in three
quarters of women and is better tolerated than tranexamic acid.
Tranexamic acid is a plasminogen-activator inhibitor. It inhibits the dissolution of thrombosis that leads to
menstrual flow. It can reduce flow by up to 50%.[8] It is most effective at reducing menstrual loss associated
with IUCDs, fibroids and bleeding diathesis. Other non-steroidal anti-inflammatory drugs (NSAIDs) may also be
used. Side-effects include nausea, vomiting and diarrhoea. If there is disturbance in colour vision then it
should be discontinued.
The COCP suppresses production of gonadotrophins and reduces menstrual blood loss by around 50%. It can
improve dysmenorrhoea, lighten periods, regulate the cycle, improve premenstrual symptoms, reduce the risk
of PID and protect the ovaries and endometrium against cancer.
Third-line treatment
This is with norethisterone.

The dose is 15 mg daily, from day 5 to 26 (or injected long-acting progestogens). This can result in a significant
reduction in menstrual blood loss, although women tend to find the treatment less acceptable than
intrauterine levonorgestrel. This regimen of progestogen may have a role in the short-term treatment of
menorrhagia.
However, there are very limited data regarding the use of progestogens and of oestrogens and progestogens
in combination in the treatment of irregular menstrual bleeding associated with anovulation. There is still no
consensus about which regimens are the most effective.

1538. A 2yo fell on outstretched hand on playground. He presents with pain on base of the thumb. XR=no fx.
What is the single most likely dx?
a. Colles fx
b. Head of radius
c. Mellet finger
d. Scaphoid fx
e. No fx

Answer: D
The scaphoid bone is one of the carpal bones in your hand around the area of your wrist. It is the most
common carpal bone to break (fracture). A scaphoid fracture is usually caused by a fall on to an outstretched
hand. Symptoms can include pain and swelling around the wrist. Diagnosis of a scaphoid fracture can
sometimes be difficult, as not all show up on X-rays. Treatment is usually with a cast worn on your arm up to
your elbow for 8 to 12 weeks. Sometimes surgery is advised. Correct diagnosis and prompt treatment of a
scaphoid fracture can help to reduce complications.

1539. A pt was admitted with increased frequency of passing urine, increased thirst, weakness and muscle
cramps. What is the most probable dx?
a. Conns syndrome
b. Cushings syndrome
c. Pheochromocytoma
d. Hyperthyroidism
e. Hypoparathyroidism
Answer: A

Classic presentation of hyperaldosteronism include:

Hypertension.
Hypokalaemia (usually <3.5 mmol/L, although 70% of patients may be normokalaemic).
Metabolic alkalosis.
Sodium may be normal or at the high end of normal.
Patients may also have polyuria and subsequent polydipsia due to reduced ability of the kidneys to
concentrate urine.
Weakness may be present from hypokalaemia.
Headaches and lethargy may also be present.

Differential diagnosis of hyperaldosteronism with low renin


Primary hyperaldosteronism (adenoma, BAH, GRA, carcinoma).
Congenital adrenal hyperplasia.
Differential diagnosis of hyperaldosteronism with high renin
Renal artery stenosis (RAS).
Coarctation of the aorta.
Fibromuscular dysplasia.
Renin-secreting tumours.
Congestive cardiac failure.
Nephrotic syndrome.
Gitelman's syndrome.
Bartter's syndrome.
Diuretic use.

1540. A 69yo male presented with sudden onset of dysphagia. He is neither able to swallow liquid nor
solid, he recently had a denture fitting. What is the most probable dx?
a. Foreign body
b. Plummer vinson syndrome
c. Achalasia cardia
d. Esophageal rupture
e. Esophageal ca
Answer: A
Causes of sudden onset of dysphagia could be:
Apthous ulcers
Carcinoma tongue
Oral cavity obstruction (fb/tumor)
Quinsy
Tonsillitis
Adenoiditis
Scleroderma
Dermatomyositis
Diphtheria
Esophageal spasm
Gastric carcinoma
Tetanus
Rabies

1541. A 62yo man with chronic schizophrenia presents with a mask like face and involuntary pill rolling
movement in both hands. He complains of chronic cough and forgetfulness. He is on long term antipsychotic
meds. What is the single most likely dx?
a. Shy drager syndrome
b. Parkinsonism
c. Huntingtons chorea
d. Tardive dyskinesia
e. Akathisia

Answer: B
Parkinsonism is a clinical syndrome characterized by tremor, bradykinesia, rigidity, and postural instability
which May later be associated with dementia or depression.
About 7% of people with parkinsonism have developed their symptoms following treatment with particular
medications. Side effect of medications, mainly neuroleptic antipsychotics.

1542. A 34yo female presented with vomiting preceded by an occipital headache of acute onset. Exam:
conscious and alert with photophobia but no neck stiffness. CT: normal. What is the most
appropriate further management?
a. CT brain with contrast
b. Repeat CT brain in 24h
c. CSF exam
d. Cerebral angio
e. MRI brain

Answer: C (to exclude increased intracranial pressure)

This test is done to measure pressures within the cerebrospinal fluid and to collect a sample of the fluid for
further testing. CSF analysis can be used to diagnose certain neurologic disorders, particularly infections (such
as meningitis) and brain or spinal cord damage.

1543. A lady with post ileocolectomy closure of stoma has a small 4 cm swelling around the stoma. What is the
most appropriate management of the swelling?
a. Local exploration of swelling
b. Exploratory laparotomy
c. Open laparotomy and re-closure
d. Abdominal binder
e. Truss
f. Laparotomy with mesh repair

Answer: D ( Most hernias can be managed without surgery. Support belts and appliances are the most
successful aids. These are available on prescription in the UK)
Parastomal hernia
A parastomal hernia involves an ostomy in the area where the stoma exits the abdominal cavity. The intestine
or bowel extends beyond the abdominal cavity or abdominal muscles; the area around the stoma appears as a
swelling or protuberance. Parastomal hernias are incisional hernias in the area of the abdominal musculature
that was incised to bring the intestine through the abdominal wall to form the stoma. They may completely
surround the stoma (called circumferential hernias) or may invade only part of the stoma.
Parastomal hernias can occur any time after the surgical procedure but usually happen within the first 2 years.
Recurrences are common if the hernia needs to be repaired surgically. Risk factors may be patient related or
technical. Patient-related risk factors include obesity, poor nutritional status at the time of surgery, presurgical
steroid therapy, wound sepsis, and chronic cough. Risk factors related to technical issues include size of the
surgical opening and whether surgery was done on an emergency or elective basis.
Parastomal hernias occur in four types. Initially, a parastomal hernia begins as an unsightly distention in the
area surrounding the stoma; the hernia enlarges, causing pain, discomfort, and pouching problems resulting in
peristomal skin complications that require frequent assessment. Conservative therapy is the usual initial
treatment. Adjustments to the pouching system typically are required so changes in the shape of the pouching
surface can be accommodated. Also, a hernia support binder or pouch support belt may be helpful. Avoid
convex pouching systems; if this isnt possible, use these systems with extreme caution. If the patient irrigates
the colostomy, an ostomy management specialist should advise the patient to discontinue irrigation until the
parastomal hernia resolves.
Surgery may be considered in extreme cases eg strangulation or obstruction or when the hernia affects the
seal between the skin and appliance causing leakage. Other reasons may be if the hernia is causing a lot of
pain or embarrassment to the ostomate,

1544. A 64 yo woman has been brought by her son for psychiatric evaluation. She says that she has stopped
living with her husband because she is convinced it is someone else posing to be him. What kind of delusions
is she suffering from?
a. Delusion of reference
b. Delusion of control
c. Delusion of guilt
d. Delusion of persecution
e. Delusion of doubles

Answer: B (most close option to capgras)

Capgras' delusion - belief that a close relative has been replaced by someone else who looks the same.
A delusion is a belief held with strong conviction despite superior evidence to the contrary.

Delusion of reference: The person falsely believes that insignificant remarks, events, or objects in one's
environment have personal meaning or significance.[

Delusion of control: This is a false belief that another person, group of people, or external force controls one's
general thoughts, feelings, impulses, or behavior.

Delusion of guilt or sin (or delusion of self-accusation): This is an ungrounded feeling of remorse or guilt of
delusional intensity

Delusion of persecution: Persecutory delusions are the most common type of delusions and involve the theme
of being followed, harassed, cheated, poisoned or drugged, conspired against, spied on, attacked, or
otherwise obstructed in the pursuit of goals.

Delusion of doubles: The Fregoli delusion, or the delusion of doubles, is a rare disorder in which a person holds
a delusional belief that different people are in fact a single person who changes appearance or is in disguise.

1545. A 19yo man with known hx of OM presents with headache, lethargy, sweating and shivering. What is the
single most likely dx?
a. Furuncle
b. Meningitis
c. Myringitis
d. Nasopharyngeal tumor
e. OM
Answer: B
A very rare and serious complication of OM is meningitis. This can occur if the infection spreads to the
protective outer layer of the brain and spinal chord (the meninges).
Symptoms of meningitis can include:
severe headache
being sick
a high temperature (fever)
stiff neck
sensitivity to light
rapid breathing

AOM is seen frequently in children but is less common in adults. Smoking is a risk factor. Om occurs more
commonly in winters.

Complications of AOM:

Infra-temporal infections can include:

Tympanic membrane perforation.


Mastoiditis.
Facial nerve palsy.
Acute labyrinthitis.
Petrositis.
Acute necrotic otitis.
Chronic otitis media.
Intracranial infections can include:

Meningitis.
Encephalitis.
Brain abscess.
Otitic hydrocephalus (hydrocephalus associated with AOM, usually accompanied by lateral sinus thrombosis
but the exact pathophysiology is unclear).
Subarachnoid abscess.
Subdural abscess.
Sigmoid sinus thrombosis.
Rarely, systemic complications can occur, including bacteraemia, septic arthritis and bacterial endocarditis.

1546. A 46yo woman has gained weight. She has sensitivity to cold. Her pulse = regular at 50 bpm and
heart=enlarged. What is the single most likely underlying mechanism for this condition
a. Autoimmune
b. Degenerative
c. Congenital
d. Infective
e. Nutritional
a. Autoimmune

1547. A 70yo man presents with a punched out ulcer between his toes. He is a heavy drinker and smoker.
Exam: ulcer is yellow and the foot turns red when dangling off the bed. What is the
single most likely dx?
a. Arterial ischemia ulcer
b. Malignancy
c. Neuropathic ulcer
d. Pressure ulcer
e. Venous stasis ulcer

Answer: A (These ulcers occur most commonly in areas of poor blood supply - eg, the tip of the toes or over
the tibia)
Arterial leg ulcers:

These are often more distal and on the dorsum of the foot or toes.Initially they have irregular edges, but this
may become more clearly defined. The ulcer base contains greyish, granulation tissue. Handling, such as
debriding these ulcers, produces little or no blood.Nocturnal pain is typical. It is worse when supine and is
relieved by dangling the legs out of bed.There are often features of chronic ischaemia, such as hairlessness,
pale skin, absent pulses, nail dystrophy and wasting of calf muscles.

Risks for arterial ulcers


Coronary heart disease.
History of stroke or transient ischaemic attack.
Diabetes mellitus.
Peripheral arterial disease including intermittent claudication.
Obesity and immobility.
Treatment:
These will usually require referral for assessment and care. It is really the management of peripheral vascular
disease. See separate article Peripheral Arterial Disease.

1548. A 65yo woman complains of a painful discharging ulcer above her ankle on the inner side of her left
lower leg. Exam: the base of the ulcer is red and covered by a yellow fibrous tissue. The border is irregular. The
skin is tight. What is the single most likely dx?
a. Arterial ischemia ulcer
b. Malignancy
c. Neuropathic ulcer
d. Pressure ulcer
e. Venous stasis ulcer

Answer: E (Venous ulceration is typically seen just above the medial malleolus.)
Exclusion of options
A; Arterial ulcer - look for reduced pulses in the foot, ankle and possibly femoral artery. These ulcers occur
most commonly in areas of poor blood supply - eg, the tip of the toes or over the tibia - and are typically
painful and deep. Other evidence of poor blood supply may include peripheral cyanosis and claudication.
An arterial ulcer tends to occur on lateral side of distal leg and leg is pulseless and cool.
B: Malignancy - malignant ulcers in this area are rare but the possibility should not be overlooked. Watch out
for an ulcer with a rolled everted edge. If ulceration occurs in the area of scar tissue, Marjolin's ulcer should be
considered. Chronic venous ulcers can develop into malignant ones, so any non-healing ulcer should be
referred for biopsy.

C: Neuropathic ulcer - this is painless, deep, often with overlying hyperkeratosis and occur at sites of loss of
nerve supply and recurrent trauma, ie the heel, metatarsal heads.

D: are localized injuries to the skin and/or underlying tissue that usually occur over a bony prominence as a
result of pressure, friction or rub.

Venous ulcers are caused by incompetent valves in the veins of the lower leg, especially in the perforators.
These incompetent valves cause blood to be squeezed out into the superficial veins, when the calf muscles are
contracted, instead of upwards towards the heart. Dilation of superficial veins occurs (varicosities) and the
subsequent raised venous pressure results in oedema, venous eczema and ulceration. Valves may also
become damaged following the venous hypertension that occurs in pregnant women and there may be
congenital absence of valves.

80% of all leg ulcers are venous ulcers and a large shallow relatively painless ulcer with an irregular granulating
base in the 'gaiter' region of the leg (between the knee and ankle) is likely to be venous in origin.

Investigations:

Measurement of ankle brachial pressure index (ABPI) using Doppler


Swabs for microbiology
Patch testing
Biopsy

Treatment:

Compression bandage
Debridement and cleaning
Dressing
Antibiotics
Pentoxifylline
Topical steroids
Aspirin

1549. A 55yo woman suffered from an acute MI 5d ago. While she was in the hosp the pt developed features
of pulmonary edema and heart failure. What is the most probable cause of her present condition?
a. VSD
b. Ruptured papillary muscle
c. Pericarditis
d. A-fib
e. Re-infarction

Answer: B
All of the options listed are complications of MI but the pulmonary edema and heart failure suggests rupture
Myocardial rupture is most common three to five days after myocardial infarction.
This may occur in the free walls of the ventricles, the septum between them, the papillary muscles, or less
commonly the atria.
Risk factors for myocardial rupture include completion of infarction (no revascularization performed), female
sex, advanced age, and a lack of a previous history of myocardial infarction.[2] In addition, the risk of rupture
is higher in individuals who are revascularized with a thrombolytic agent than with PCI.Rupture of the papillary
muscle may lead to acute mitral regurgitation and subsequent pulmonary edema and possibly even
cardiogenic shock.

Rupture of papillary muscle or chordae tendinae:


Causes severe mitral regurgitation within the first week after infarction and is a life-threatening complication.
It is most often seen with inferior infarctions.
One study found a median time for papillary muscle rupture in patients treated with fibrinolysis to be 13 hours
after AMI.[2]
Papillary muscle rupture following AMI usually requires mitral replacement.
Ventricular septal rupture and free wall rupture
Risk factors: older age, female gender, non-smoker, anterior infarction, worse Killip class on admission,
increasing heart rate on admission, first myocardial infarction and hypertension.[6]
Postinfarction VSD is relatively infrequent but life-threatening.[7] The incidence has dramatically decreased
with reperfusion therapy.
May develop as early as 24 hours after myocardial infarction but often presents 2-7 days afterwards. Mortality
rates are greater than 90%.
Ventricular septal rupture:[2]
Patients may initially have no clinically significant cardiopulmonary symptoms but rapid recurrence of angina,
hypotension, shock or pulmonary oedema develop.
Signs of ventricular septal rupture include a new harsh pansystolic murmur best heard at the left lower sternal
border, with worsening haemodynamic profile and biventricular failure.
Diagnosis is by transoesophageal echocardiography or by showing a step-up in oxygen saturation in the right
ventricle on pulmonary artery catheterisation.
Postinfarction ventricular septal defects require urgent surgical closure.[8]

1550. A 76yo woman presents with deep stroke 6h ago. What would the immediate tx be?
a. Aspirin 75mg
b. Aspirin 300mg
c. Streptokinase
d. IV heparin
e. Dipyridamole 200mg

Answer b. Aspirin 300mg


Aspirin (low-dose) is the most commonly used antiplatelet medicine when the stroke has just happened.
Exclusion of other options:
C: If an ischaemic stroke is diagnosed and it has been less than four and a half hours since symptoms started,
you may be given a medicine directly into a vein, called alteplase. (Here the time given is 6hr).
E: A combination of dipyridamole and aspirin (Acetylsalicylic acid/dipyridamole) is FDA-approved for the
secondary prevention of stroke, However, it is not licensed as monotherapy for stroke prophylaxis.
D: A review by the panel of the Stroke Council of the American Heart Association found no strong evidence for
effectiveness of anticoagulants in treating acute ischemic stroke.
There are two main types of stroke - ischaemic and haemorrhagic.
Stroke is the largest cause of disability in the UK and the third most common cause of death (after heart
disease and cancer). Most cases occur in people aged over 65. Each year about 1 in 100 people over the age of
75 will have a stroke.
Symptoms of stroke;
Weakness of an arm, leg, or both. This may range from total paralysis of one side of the body to mild
clumsiness of one hand.
Weakness and twisting of one side of the face. This may cause you to drool saliva.
Problems with balance, coordination, vision, speech, communication or swallowing.
Dizziness or unsteadiness.
Numbness in a part of the body.
Headache.
Confusion.
Loss of consciousness (occurs in severe cases).
Investigations:

A brain scan (CT scan or MRI scan). This can determine the type of stroke (ischaemic or haemorrhagic) and
may detect rarer conditions which may have caused the stroke or which may mimic a stroke.
Blood tests (to determine sugar and cholesterol levels)
CXR and Ecg( to rule out AF)
Immediate care
Ideally, you will be assessed quickly by a doctor. Commonly, a scan of the brain is organised as soon as
possible. The aim of the scan is to confirm the diagnosis and to tell whether the stroke is an ischaemic or
haemorrhagic stroke. This is very important to know, as the initial treatment of the two is very different.

If an ischaemic stroke is diagnosed and it has been less than four and a half hours since symptoms started, you
may be given a medicine directly into a vein, called alteplase. This is a clot-busting medicine which aims to
dissolve the blood clot. The medical word for this is thrombolysis. If the blood clot that caused the stroke can
be dissolved shortly after symptoms begin, it can improve the eventual outcome. This is because brain cells
that would have died are able to survive.

Further care

Antiplatelet medication. Platelets are tiny particles in the blood which help blood to clot. Antiplatelet
medication is usually advised if you have had an ischaemic stroke (due to a blood clot). Antiplatelet
medication reduces the stickiness of platelets. This helps to prevent blood clots forming inside arteries, which
helps to prevent a further stroke. Aspirin (low-dose) is the most commonly used antiplatelet medicine when
the stroke has just happened. Another antiplatelet medicine, clopidogrel, is usually given long-term after the
initial treatment.

If you have atrial fibrillation, you have an increased risk of a blood clot forming in a heart chamber and
travelling to the brain to cause a stroke. If you have atrial fibrillation (or certain other heart conditions), a
medicine called warfarin may be prescribed. Warfarin helps to prevent blood clots forming. Warfarin is an
anticoagulant.

If you have a bleeding (haemorrhagic) stroke and are taking an anticoagulant medicine such as warfarin,
treatment to reverse the effect of the anticoagulation is given.
If a subarachnoid haemorrhage is the cause of the stroke, an operation to fix the leaking blood vessel (artery)
is sometimes an option.

1551. A 19yo man accuses his friend of making his right arm swing out at a stranger. What is the best term to
describe his condition?
a. Control
b. Persecution
c. Guilt
d. Reference
e. Grandeur
Key: Control (A)
Reason: Grandeur signifies lofty ideas about oneself, Persecution signifies a mistrust in others and belief that
they want to harm you, Reference means believing people are talking about you, Guilt is the emotion of
feeling bad about something you did or didnt do.
Discussion: A delusion is a false belief which is firmly sustained and based on incorrect inference about reality.
This belief is held despite evidence to the contrary and is not accounted for by the person's culture or religion.
Karl Jaspers, a noted psychiatrist and philosopher, described the three main criteria required for a delusion.
* Certainty - the patient believes the delusion absolutely.
* Incorrigibility - the belief cannot be shaken.
* Impossibility - the delusion is without doubt untrue.
Types of delusions:
* Monothematic - delusions are only relating to one particular topic. * Polythematic - range of delusional
topics (seen in schizophrenia).

1552. A 26yo man with history of hereditary haemorrhagic telangiectasia is planning to start a family. What is
the mode of inheritance?
a. AD with incomplete penetrance
b. Autosomal co-dominant
c. AR with incomplete penetrance
d. AD
e. AR
Key: Autosomal Dominant (D)
Reason: This disease is also known as Osler-Weber-Rendu disease and is characterized by abnormal blood
vessels in skin, GIT, lung, liver, brain.
Discussion: Hereditary haemorrhagic telangiectasia (HHT) is also known as Osler-Weber-Rendu syndrome. The
condition is characterised by vascular dysplasia leading to telangiectasia. Epistaxis and gastrointestinal
bleeding are frequent complications of mucosal involvement. There are also often arteriovenous
malformations (AVMs), particularly of lungs, liver and brain. Most cases are due to mutations in the endoglin
(HHT1) or ACVRLK1 (HHT2) genes.
Presentation:
Age-related penetrance is seen in HHT. It does not present at birth but commonly presents with recurrent
epistaxis, usually in the teenage years. People with the condition develop mucocutaneous lesions, usually
involving the nasal mucosa, lips and tongue. These lesions are sharply demarcated red-purple macules,
papules or spider-like lesions comprising a mat of
tortuous vessels. These can also occur in the conjunctiva, upper respiratory tract, gastrointestinal (GI) tract,
bladder, vagina, bronchi, brain and liver. Cutaneous telangiectasias are often not evident until between 20 and
30 years of age.
DD:
CREST syndrome (= Calcinosis, Raynaud's disease, (o)Esophageal dysmotility, Sclerodactyly, Telangiectasia).
Von Willebrand's disease
Investigations: CT, MRI, Angiography.
Management:
* Acute bleeds may need transfusion.
* Surgical/laser ablation.
* Oestrogen therapy seems to be beneficial.
Complications:
* Hemorrhage
* Cirrhosis

1553. A 50 y/o man with a known history of stroke is unable to get out of his house because he cant find
where the door is. He refuses help from his wife and says he is not blind. What is the single most likely defect?
a. Paracentral scotoma
b. Tunnel vision
c. Total blindness
d. Central scotoma
e. Cortical blindness
Key: Cortical blindness (E)
Reason: The provided history of a CVA essentially rules out the rest of the options because the only thing that
applies here is a stroke in the Occipital cortex that causes complete cortical blindness which renders the man
helpless when it comes to finding where the door is in his house.
* Tunnel Vision is a symptom of Retinitis Pigmentosa and comprises of a loss of peripheral vision leading to a
tunnel-like field defect.
* A scotoma is a partial loss of vision or blind spot in an otherwise normal visual field.
Discussion: A stroke means that the blood supply to a part of the brain is suddenly cut off. The brain cells need
a constant supply of oxygen from the blood. There are two main types of stroke - ischaemic and
haemorrhagic.
Symptoms:
* Weakness of an arm, leg, or both. This may range from total paralysis of one side of the body to mild
clumsiness of one hand.
* Weakness and twisting of one side of the face. This may cause you to drool saliva.
* Problems with balance, co-ordination, vision, speech, communication or swallowing.
* Dizziness or unsteadiness.
* Numbness in a part of the body.
* Headache.
* Confusion.
* Loss of consciousness (occurs in severe cases).
Action (1st hour):
* Protect the airway.
* Pulse, BP, ECG (rule out AF for embolus).
* Blood glucose (rule out decreased blood sugar).
* Urgent CT scan.
* Thrombolysis of ischemic stroke and within 4.5 hours by alteplase, etc.
* NPO by passing NG tube.
* IV fluids, but take care not to over-hydrate keeping cerebral edema in mind.
* Once hemorrhagic stroke is excluded via CT scan, give aspirin 300mg atleast.
* Once the patient has been managed acutely, investigate further to find out the cause of the stroke while
organizing appropriate rehabilitation of the patient at the same time.

1554. An elderly lady presents with confusion. She is apyrexial but complains of dysuria for 2 days duration.
What is the definitive dx investigation?
a. Blood culture
b. Urine nitrites
c. CT head
d. ECG
e. IVU
Key: Urine Nitrites (B)
Reason: According to the OHCM pg. 288, if symptoms are present, dipstick the urine and treat empirically if
nitrites or leucocytes are positive while awaiting culture of an MSU.
ECG and CT Head are useless tests and IVU doesnt make sense because we arent looking for an obstructive
cause. You only go for imaging studies if it is recurrent and there is failure with standard treatment. Most
accurate test URINE CULTURE.
Discussion:
Urinary tract infection (UTI) - this implies the presence of characteristic symptoms and significant bacteriuria
from kidneys to bladder. Many laboratories regard 105 colony-forming units per millilitre (cfu/ml) as the
threshold for diagnosing significant bacteriuria.
Organisms:
Several micro-organisms are known to cause UTI, but the majority of infections will be produced by three
organisms:
* Escherichia coli
* Staphylococcus saprophyticus
* Proteus mirabilis
Infection with less common organisms is more likely to occur in patients who have underlying pathology
and/or frequent infections, are immunosuppressed, or who are catheterised. Organisms which may produce
infection under these circumstances include:
* Klebsiella spp.
* Proteus vulgaris
* Candida albicans
* Pseudomonas spp.
Signs: Fever, abdominal/ loin tenderness, foul smelling urine.
Symptoms: Frequency, dysuria, urgency, hematuria, suprapubic pain.
Tests:
* Dipstick urine nitrites or leukocytes positive mean UTI. Treat empirically.
* MSU sample for culture and sensitivity.
* CBC, Urea & Electrolytes, Blood culture if systemically unwell.
Imaging: usually done in males, children or persistent cases.
* USG and refer to urology for assessment.
* CT-KUB.
* Urodynamics.
Management:
* Drink plenty of fluids.
* Trimethoprim 200mg x 12 hourly PO OR Nitrofurantoin 50mg x 6 hourly PO for 3-6 days OR Amoxicillin
500mg x 8 hourly PO.
* Second-line: Co-Amoxiclav 7 day course PO.

1555. A 40yo woman on chemotherapy for metastatic breast carcinoma now presents with painful
swallowing. Exam: she has white plaques on top of friable mucosa in her mouth and more seen on
esophagoscopy. What is the most effective tx for this pt?
a. Antispasmodic
b. H2 blocker
c. Antibiotics
d. Antifungals
e. I&D
Key: Antifungals (D)
Reason: The immunocompromised state, location of infection, gross picture all point to Esophageal
Candidiasis which is treated with antifungals. The treatment of choice is Fluconazole 400mg STAT, then 200mg
per day PO.
Discussion:
* Candidal infection is the most common cause of invasive fungal infections in hospital patients.
* Dysphagia, retrosternal discomfort.
* Fluconazole 50-100mg PO x 24hourly for 7-14 days.
* If invasive, fluconazole 400mg/day. Side effects nausea, raised LFTs, thrombocytopenia.
* Severe systemic infection, amphotericin B.

1556. A 43yo woman has suffered with heavy periods for many years and has tried many medical tx without
success. She is constantly flooding and at times cant leave her house due to heavy bleeding. She has
completed her family of 5 children and her last blood test showed
Hgb=8.9g/dl. She feels that she cant cope with the bleeding anymore and her husband is asking for a tx that
can guarantee success. What is the most appropriate management to improve menorrhagia in this pt?
a. Endometrial ablation
b. Hysterectomy
c. Hysteroscopic/Laser resection of fibroids
d. Myomectomy
e. UAE
Key: Hysterectomy (B)
Reason: The woman is 43 years old, she has completed her family, her illness is of severe distress to her and
her family and her Hb is falling. Hysterectomy is the logical choice because it makes the most sense.
Endometrial ablation or laser resection have adverse effects, myomectomy isnt at all curative in this patient's
case and Uterine Artery Embolization leads to recurrence which would be an unwanted effect in this patient.
Hysterectomy is the t/m of choice.
Discussion:
* May be associated with RCC.
* Oestrogen dependent.
* Present with menorrhagia, fertility problems, pain, mass.
* Management: Mirena for menorrhagia. Hysterectomy in women who have completed their families. In
younger women, a reversible menopausal state may be induced with analogs of LHRH (goserelin). Side effects
include menopausal symptoms and bone loss. Fertility and fibroids used when drugs stopped, used pre-op to
reduce bulk and in those unfit for surgery.
* Myomectomy may be chosen in subfertility. Laparoscopic surgery with laser use is possible. Complications
severe haemorrhage requiring hysterectomy, post op adhesions, embolising fibroids can shrink them (not
widely available and very painful).

1557. A man on antipsychotic meds develops features of retinitis pigmentosa. Which drug is most likely to
cause these symptoms?
a. Thioridazine
b. Haloperidol
c. Chlorpromazine
d. Risperidone
Key: Thioridazine (A)
Reason: Simple factual statement. Thioridazine has been shown to cause symptoms similar to retinitis
pigmentosa including tunnel vision and night blindness.
Discussion:
* Mostly autosomal recessive. Also autosomal dominant or X-linked.
* Most common retinal hereditary disease.
Slit-lamp biomicroscopy is the key initial assessment. Further tests are to determine the functional integrity of
the retina and optic nerve:
* Visual acuity
* Visual field assessment
* Pupillary reflex response
* Colour defectiveness determination
* Refraction
Intraocular pressure will also need to be measured. To find out more about these tests, see the separate
article on Examination of the Eye. Imaging includes:
o Retinal photography
o Ultrasound of the eye
o Fluorescein angiography
o Optical computer tomography (OCT)
All of these can be performed in a general clinic. The most critical diagnostic test is the electroretinogram
(similar to the EEG of the brain or ECG of the heart).
Management:
o Vitamin A/beta-carotene
o Acetazolamide
o Lutein
o Bilberry
o Immunosuppressive agents (including steroids)

1558. Pt with low Hb, MCV=76, angular stomatitis, red tongue, and koilonychia. What is the most probable dx?
a. Folate def
b. B12 def
c. Iron def
d. Vit E def
e. Hemolytic anemia
Key: Iron Deficiency Anemia (C)
Reason: Folate and B12 are automatically ruled out with the low MCV. Vitamin E deficiency hasnt been shown
to cause anemia. This is probably a fore-runner of Plummer-Vinson syndrome which is related to Iron
Deficiency Anemia in this case. The treatment is oral Iron (Ferrous Sulphate 200mg PO x 8 hourly). Continue
until Hb normal and for atleast 3 months to replenish stores.

1559. A pt with sudden severe eye pain, red eye, visual blurring, acuity of only finger counting, nausea,
vomiting with a shallow ant chamber that is hazy on shining a torch. What is the dx?
a. CRVO
b. Acute closed angle glaucoma
c. Uveitis
d. Iritis
e. Open angle glaucoma
Key: Acute Closed Angle Glaucoma (B)
Reason: The painful presentation rules out CRVO and Open Angle Glaucoma. The characteristic nausea,
vomiting and shallow anterior chamber signifies an acute event, which in this case would be Acute Closed
Angle Glaucoma. Uveitis and Iritis will not have nausea or vomiting which signify markedly raised intraocular
pressure.
Treatment
* Pilocarpine 2 drops x 2 hourly
* Acetazolamide 500mg IV x STAT OR Mannitol IV
* Good analgesics
* Anti-emetics
* Admit to monitor IOP

1560. A patient who works in a pet shop has temp = 37.5C, dyspnoea, chest pain and cough. CXR: patchy
consolidation. What is the most suitable t/m?
a. Amoxicillin
b. Tetracycline
c. Erythromycin
d. Clarithromycin
e. Penicillin
Key: Amoxicillin (A)
Reason: This is a mild case of Community Acquired Pneumonia (CAP) and the patient has presented to you in a
GP setting. Now, while the history would point towards Chlamydia pneumonia or Chlamydia psittaci being the
commoner cause of this pneumonia, unless that is proved, you will treat this as Streptococcus pneumonia
acquired pneumonia which is the commonest form of pneumonia in this scenario. Treatment of choice will be
Oral Amoxicillin 500mg 1g x 8 hourly.

1561. A 50yo man complains of dysphagia after eating bread. Barium swallow reveals a lower
Oesophageal ring. What is the most appropriate t/m?
a. Reassurance
b. Antispasmodics
c. Dilatation of the LES
d. Endoscopic diverticulectomy
e. I&D
Key: Dilatation of the LES (C)
Reason: There is a ring at the lower oesophagus. It is likely a stricture and the treatment is endoscopic balloon
dilatation. Reassurance will not help the patient, antispasmodics are given in diffuse esophageal spasm and
would not be helpful here and diverticulectomy is mentioned for Zenkers diverticulum. Incision and drainage
is stupid.
Discussion:
* Esophageal rings are concentric, smooth, thin extension of normal oesophageal tissue, usually 3 to 5 mm
thick. They consist of mucosa, submucosa and muscle. They may be an incidental finding at barium studies or
endoscopy. Incidence is unknown as most are asymptomatic. There is no sex difference in the incidence of
rings except that multiple rings are usually found in young men. Rings are classified as A, B and C:
o A is uncommon and is a muscular ring several centimetres proximal to the squamocolumnar junction. It may
be an inconstant finding on barium swallow and there is some debate as to whether it really is an anatomical
entity.
o B ring or Schatzki's ring is really a web, as it involves only mucosa and submucosa. It tends to mark the
proximal part of a hiatus hernia and usually presents in a patient aged over 50 whose main complaint is
intermittent dysphagia to solid food, spanning months or years, and it is non-progressive.
o C ring is a rare X-ray finding of indentation caused by the diaphragmatic crura. It rarely causes symptoms.

1562. A 48yo nulliparous woman feels tired all the time. Her periods are regular but have always
lasted for at least 10d. Choose the single most appropriate initial investigation?
a. High vaginal swab
b. Serum Hgb conc
c. TFT
d. None
e. Abdominal US
Key: Serum Hb Conc. (B)
Reason: The single most appropriate investigation for an elderly woman presenting with heavy but regular
bleeding and history of fatigue will be Serum Haemoglobin to rule out anaemia. There should also be a low
threshold for doing TFTs in this patient as Hypothyroidism is a likely cause of menorrhagia and could also be
considered the answer. But the delay in result for TFTs would take it away from the most appropriate
investigation.
1563. A man got his hand caught in machinery at work. The fingers are swollen but the XR shows no fracture.
What is the most appropriate management?
a. Splint
b. Put in plaster
c. Broad arm sling for 1wk
d. Elevate in high sling for 2d
e. Neighbour strapping
Key: Elevate in high sling for 2 days (D)
Reason: The injury is not a severe one and shows swollen fingers indicating soft tissue injury with no fracture.
This essentially rules out all immobilizing treatment forms like splinting and neighbour strapping or POP
plastering. The best way to treat this would be with elevation for 2 days and painkillers for the pain.

1564. A 39yo woman presents with symptoms recurring annually characterized by depressed mood, being
socially withdrawn, hypersomnia, lack of enjoyment in life which last for several months. What is the most
likely dx?
a. Seasonal Affective Disorder
b. Moderate depression
c. Dysthymia
d. GAD
e. Bipolar disorder
Key: Seasonal Affective Disorder (A)
Reason & Discussion: The annual nature of her symptoms coupled with the duration would most likely make
this a case of seasonal affective disorder, which is a constellation of symptoms occurring mostly in the winter
months characterized by the aforementioned habits and chronic low mood. Depression has a time-frame that
this scenario doesnt meet, Dysthymia is depression with less severe but longer lasting symptoms, GAD is
unlikely as there arent any panic symptoms, Bipolar would require periods of mania. Treatment for SAD is CBT
and light therapy. The diagnosis of SAD is based on:
* Depression cycles on a regular basis during autumn/winter.
* Full remission of symptoms in spring/summer.
* Seasonal symptoms for at least two consecutive years.
* Atypical features, which may or may not be present.
During the psychological examination, it is also important to assess:
* Suicidal ideation.
* Abnormal mechanisms of coping - eg, social isolation, alcohol use.
Treatment options also include Bupropion, recently shown to have benefit in SAD and Agomelatine, a
melatonin agonist.

1565. A 75yo man presents with ARF. He has been troubled by recurrent epistaxis but over the last 3wks he
reports to have coughed up blood too. What is the single most likely positive antibody?
a. P ANCA
b. C ANCA
c. Anti Ro
d. Anti DS DNA
e. Anti centromere
Key: C ANCA (B)
Reason: This patient has Wegeners Granulomatosis which affects the upper and lower respiratory tract and
presents with renal insufficiency. It is C ANCA positive and is treated by steroids or cyclophosphamide.
Discussion: Also called granulomatosis with polyangiitis. Affects the upper respiratory tract, lungs and kidneys.
* Nasal obstruction, ulcers, epistaxis. Destruction of nasal septum causing a characteristic saddle nose
deformity.
* Rapidly progressive GN with crescent formation, hematuria and proteinuria can occur.
* Cough, hemoptysis, pleuritis.
Investigations:
* C-ANCA, Raised ESR and CRP.
* Urine R/E to check for proteinuria or hematuria. If present, renal biopsy to confirm.
* Chest X-ray Nodules and fluffy infiltrates of pulmonary haemorrhage.
* CT Scan Diffuse alveolar haemorrhage.
Treatment:
* Severe disease should be treated with steroids and cyclophosphamide or rituximab to induce remission.
* Methotrexate or azathioprine for maintenance.
* Patients with severe renal disease may benefit for plasma exchange.
* Co-Trimoxazole should be given as prophylaxis against P. jiroveci infection.

1566. A woman is admitted to the hospital for elective abdominal hysterectomy. 2 months ago she was dx
with DVT and pulmonary embolism and was started on warfarin. What is the most appropriate pre-op
measure you will take on this occasion?
a. Continue warfarin
b. Stop warfarin
c. Stop warfarin and start heparin
d. Increase warfarin dose
e. Add heparin
Key: Stop warfarin and start heparin (C)
Reason: Warfarin depletes Vit. K dependent clotting factors. It should be stopped for them to increase in
amount and heparin can provide short term cover in the meantime.
Discussion: A deep vein thrombosis (DVT) is a blood clot in a vein. Blood clots in veins most often occur in the
legs but can occur elsewhere in the body, including the arms. Warfarin works by depleting Vitamin K
dependent factors in the body and is used to maintain an INR according to the specific condition of the
patient. Now, heparin is a short term fix and is only available in the form of injectables. In the case of this
woman, we need anti-coagulant cover as well as making sure she doesnt bleed excessively during her surgery.
The only way to do that is to stop
Warfarin because of its delayed reversibility regarding its effect and to administer short acting heparin
injections in the gap created by halting oral Warfarin.

1567. This condition affects middle aged women more than men and is characterized by low mood, early
morning waking, loss of libido, tiredness and suicidal intention last for at least 2wks. What is the most
probable dx?
a. Bipolar affective disorder
b. Dysthymia
c. Major depressive disorder
d. Schizoaffective disorder
e. Recurrent brief depression
Key: Major depressive disorder (C)
Reason: This is C because it fulfils the five criteria of Major Depression which are (according to NICE
guidelines):
* Loss of interest or pleasure [HAS TO BE THERE]
Any four out of the following:
* Poor appetite or weight loss
* Early waking
* Psychomotor retardation
* Decreased libido
* Decreased ability to concentrate
* Ideas of worthlessness of guilt
* Suicidal thought or action
Severity is based on the extent of symptoms and their functional impact:
* Subthreshold depressive symptoms - <5 symptoms.
* Mild depression - few, if any, symptoms in excess of the 5 required to make the diagnosis, with symptoms
resulting only in minor functional impairment.
* Moderate depression - symptoms or functional impairment are between 'mild' and 'severe'.
* Severe depression - most symptoms present and the symptoms markedly interfere with normal function. It
can occur with or without psychotic symptoms.
Discussion: Antidepressants are not recommended for the initial treatment of mild depression, because the
risk: benefit ratio is poor. However, their use may be considered:
* If mild depression persists after other interventions, or is associated with psychosocial and medical
problems.
* In mild depression complicating the care of physical health problems.
* When a patient with a history of moderate or severe depression presents with mild depression.
* With subthreshold depressive symptoms present for at least two years or persisting after other
interventions.

1568. A 10yo boy has fallen from a tree and injured his right chest. He has pain and difficulty breathing. He is
tachypneic and tender with an area of paradoxical chest wall movement on the right side. What is the single
most likely dx?
a. Diaphragmatic rupture
b. Flail chest
c. Fractured ribs
d. Hemothorax
e. Tension pneumothorax
Key: Flail Chest (B)
Reason: H/O Fall, Pain and difficulty breathing, Tender with paradoxical chest wall movement ---> FLAIL CHEST.
Discussion: A case of flail chest should be managed according to the following principles:
* High flow O2.
* CPAP.
* Chest physiotherapy.
* Adequate analgesia.
* Early intubation and ventilation in patients going towards or currently in respiratory failure.
* Open fixation for patient who are unable to be weaned from ventilator or when thoracotomy is performed
for other injuries.
1569. A 37yo woman had an elective LSCS 1d ago. You are called to see her as she becomes SOB with left
sided chest pain and a cough. She has had 3 children, 2 born by LSCS. Exam: she has reduced air entry at left
lung base. Her observations include sat=92% on air, BP=105/84mmHg, pulse=120bpm, temp=37.2C. Choose
among the options which C-section complications has she developed?
a. Aspiration pneumonia
b. Aspiration pneumonitis
c. Spontaneous pneumothorax
d. Pulmonary embolism
e. DVT
Key: Pulmonary embolism (D)
Reason: Decreasing saturation, Tachycardia, Presentation 1 day after major surgery after Pregnancy and bring
post-partum. Reduced air entry cannot be positive in any other option and DVT would actually lead to
Pulmonary Embolism. Pulmonary Embolism is especially common in abdominal, pelvic surgeries ad hip/knee
replacements. Pregnancy and post-partum state is also a risk factor.
Discussion: Usually arises from a venous thrombosis in the pelvis or legs. Rare causes include right ventricular
thrombus, post MI, septic emboli, right sided endocarditis, fat/air/amniotic fluid embolism, neoplastic cells,
parasites.
Risk factors: Recent surgery especially abdominal, pelvic or hip/knee replacement. Thrombophilia
(antiphospholipid syndrome), leg fracture, prolonged bed rest, malignancy, pregnancy, post-partum state,
COCPs, HRT, previous pulmonary embolism.
Symptoms: Acute dyspnoea, pleuritic chest pain, hemoptysis, dizziness, syncope.
Signs: Pyrexia, cyanosis, tachypnoea, tachycardia, hypertension, raised JVP, pleural rub, pleural effusion. Look
for signs of a cause like DVT.
Tests: CBC, Urea & Electrolytes, Clotting profile, D- Dimers. ABGs may show decreased PaO2 and decreased
PaCO2.
Imaging: Chest xray usually normal, ECG may be normal or show right ventricular strain pattern (inverted T in
V1 V4). Check OHCM Pg. 183 for table showing diagnosis and scoring.
Management:
* Establish IV access.
* Start LMWH tinzaparin 175 units/kg/24h subcutaneous.
* Start colloid if systolic BP less than 90, manage BP accordingly, prevent hypovolemia.
* If BP is more than 90, start Warfarin. Loading regime 5-10mg PO.
* If obvious remedial cause, 6 weeks of warfarin is enough.
* If recurrent embolus or underlying malignancy, give Warfarin for more than 3-6 months.
* Investigate underlying cause.
* Stop heparin when INR is >2 and start warfarin.
* Thrombolysis for a massive pulmonary embolism, give alteplase 10mg IV over 1 minute, then 90mg IV over 2
hours. Max. 1.5mg/kg if less than 65 kg. Consider placement of a vena caval filter in patients who develop
emboli despite adequate anti-coagulation.

1570. A pt. presents with increasing retrosternal pain and dysphagia for both solids and liquids over 18m but
denies weight loss. Chest is clear. What is the most likely dx?
a. Achalasia
b. Pharyngeal carcinoma
c. Oesophageal spasm
d. Oesophageal stricture
Key: Oesophageal Spasm (C)
Reason: Pain and no h/o wt. loss rules out CA Pharynx, Achalasia has non-progressive dysphagia for both solids
and liquids. Solids first, progressing to liquids would indicate stricture benign or malignant. Spasm has
intermittent dysphagia plus chest pain.
1571. A 70yo man presents with a fluctuant swelling of the scrotum which feels like worms when he is
standing but regresses when he lies down. What is the most probable dx?
a. Varicocele
b. Hematocele
c. Testicular CA
d. Epididymal cyst
e. Saphena varix
Key: Varicocele (A)
Reason: BAG OF WORMS. Hello.
Hematocele would be a collection of blood in the tunica vaginalis with h/o trauma. CA would have history of
painless lump with weight loss and hormonal effects. Epididymal cyst usually develops in adulthood and
contains clear or milky fluid, lies above and behind the testes. Saphena Varix would lie below and lateral to the
scrotum. A varicocele is a collection of enlarged (dilated) veins (blood vessels) in the scrotum. It occurs next to
and above one testicle (testis) or both testes (testicles). Treatment is usually conservative with the best option
being to leave the varicocele alone if it isnt symptomatic. If it is, consider surgery or injection of a substance
to block the veins. Both methods work well usually.
1572. A 52yo woman has had a swelling in the neck, hoarseness and stridor-both inspiratory and expiratory
for 2 months. What is the most probable dx?
a. CA larynx
b. CA thyroid
c. Vocal cord nodules
d. CA bronchus
e. Thyrotoxicosis
Key: CA thyroid (B)
Reason: Vocal cord nodules will not have swelling, CA Larynx will not have swelling either. CA Bronchus would
present with hemoptysis and dyspnoea. Thyrotoxicosis would have other systemic signs. This is CA Thyroid
unless proven otherwise.
Discussion:
1. Papillary (60%): Younger patients, spread to lymph nodes and lung. Treatment total thyroidectomy to
remove non obvious tumour as well +/- node excision +/- radioiodine to ablate residual cells. Give thyroxine to
suppress TSH. Prognosis better if young and female.
2. Follicular (25%): Middle age, spreads early via blood to bone and lungs, well-differentiated. Treatment
total thyroidectomy + T4 suppression + radioiodine ablation.
3. Medullary (5%): Sporadic or part of MEN syndrome, may produce calcitonin which can be used as a tumour
marker, they do not concentrate iodine. Perform a pheochromocytoma screen post-op. Treatment is
thyroidectomy + node excision. External beam radiotherapy to prevent regional recurrence.
4. Lymphoma (5%): Female: male is 3:1. May present with stridor or dysphagia, do full staging pre-treatment.
Treatment chemoradiotherapy. Assess histologically for MALT origin.
5. Anaplastic (rare): Female to male ratio is 3:1. Elderly, poor response to treatment. In the absence of
unresectable disease, excise plus radiotherapy.

1573. A woman became acutely SOB in the recovery bay and is coughing after GA. Auscultation: reduced air
entry at the right lung base and diffuse wheeze. Observation: HR=88bpm, BP=112/76mmHg, temp=37.8C and
sat=91% in air. Choose among the options which C-section complication has she developed?
a. Aspiration pneumonitis
b. Spontaneous pneumothorax
c. Endometritis
d. Pulmonary embolism
e. Tension pneumothorax
Key: Aspiration pneumonitis (A)
Reason: Everything aside, look at the presentation of this illness. The woman presented acutely after GA and
with different vitals to the great mimicker in this period of illness which would be Pulmonary Embolism. Vitally
stable with mildly reduced SpO2 and coughs after GA. The wheeze also points to aspiration. Pneumothorax
wouldnt present this way and neither would a tension pneumothorax. Endometritis is just stupid here.
1574. A 23yo female presents with paraesthesia and loss of distal pulses in her arms. She is noted to be
hypertensive. She describes feeling unwell a month prior with fever and night sweats. What is the most
probable dx?
a. Kawasaki disease
b. Takayasu arteritis
c. Buergers disease
d. Embolism
e. Raynauds phenomenon
Key: Takayasu Arteritis (B)
Reason: Takayasu arteritis because it presents in age < 50 years and loss of distal pulses (pulseless disease). It
is a systemic vasculitis which often affects women 20-40 years. Systemic features are common like fever,
weight loss and night sweats. Hypertension is also likely due to renal artery stenosis. Treatment is
Prednisolone 1mg/kg/day.
Buergers would require a long smoking history. Kawasaki would present with mucocutaneous symptoms.
Raynauds would present with cyanosis of distal digits.
Discussion: Also called aortic-arch syndrome or pulseless disease. Rare outside of Japan, systemic vasculitis.
Affects the aorta and its major branches.
* Granulomatous inflammation, causes stenosis, thrombosis and aneurysms.
* Women aged 20-40 year.
* Symptoms depend on the artery involved with cerebral, eye and upper limb symptoms dizziness, visual
changes, weak arm pulses.
* Systemic features fever, weight loss, malaise, increased BP often due to renal artery stenosis.
* Complications aortic valve regurgitation, aortic aneurysm and dissection, ischemic stroke (increased BP
and thrombus) and IHD.
* Diagnosis increased ESR and CRP, MRI & PET allow earlier diagnosis than angiography.
* Treatment Initially prednisolone, methotrexate and cyclophosphamide in resistant cases. BP control.
Angioplasty, stenting, bypass for critical stenosis.

1575. A 35yo woman presents with mass in the groin. Exam: mass found just below and lateral to the pubic
tubercle. There is no cough impulse and it is irreducible. What is the most probable dx?
a. Direct inguinal hernia
b. Strangulated hernia
c. Femoral hernia
d. Saphena varix
e. Femoral aneurysm
Key: Femoral Hernia (C)
Reason: Femoral Hernias are common in women and they present below and lateral to the pubic tubercle.
They are likely to be irreducible and have no cough impulse. It isnt strangulated because strangulation
presents with systemic signs like vomiting, fever, pain and patient becomes toxic and requires surgery.
Inguinal hernias would be above and medial to the pubic tubercle. Saphena Varix is a dilatation of the
termination of the great saphenous vein into the sapheno-femoral junction but it has a positive cough impulse
and would also reduce spontaneously on lying down. The aneurysm would be pulsatile and have a bruit as
well.
1576. A 30yo woman has injured her left lower chest in a RTA. She has BP=80/50 mmHg,
Pulse= 120 bpm. Auscultation of chest= bowel sounds present. What is the single most likely dx?
a. Diaphragmatic rupture
b. Flail chest
c. Fractured ribs
d. Ruptured oesophagus
e. Tension pneumothorax
Key: Diaphragmatic Rupture (A)
Reason: She is vitally unstable and bowel sounds are auscultate-able in her chest. Clear answer of
diaphragmatic rupture secondary to RTA. A flail segment would have a paradoxically moving rib segment and
rib fractures would show up on CXR. Oesophageal rupture would have subcutaneous emphysema and Tension
pneumothorax would present with deviated trachea and absent breath sounds on one side.
1577. A lady presents with a swelling below the groin crease that can be reduced. There is no med hx of note.
What is the most probable dx?
a. Inguinal hernia
b. Strangulated hernia
c. Testicular tumor
d. Epididymal cyst
e. Femoral hernia
Key: Femoral Hernia (E)
Reason: Obvious. Only option that fits. None of the other swellings are below the groin crease, rather they are
located in the scrotum.
1578. A 32yo woman of 38wks GA attends the antenatal day unit with pain in the suprapubic area that
radiates to the upper thighs and perineum. It is worse on walking. Her urine dipstick showed a trace of protein
but no white cells, nitrates or blood. Whats the most likely dx?
a. Braxton Hicks contractions
b. Round ligament stretching
c. Symphysis pubis dysfunction
d. Labour
e. Complicated femoral hernia
Key: Symphysis Pubic Dysfunction (C)
Reason: It is a condition that causes excessive movement of the pubic Symphysis and pain or discomfort in the
pelvic region. The pain in the supra-pubic area radiating to the legs and perineum combined with worse
symptoms on walking would point to only one thing Symphysis Pubis Dysfunction. The Urine exam is a
detractor. Braxton-Hicks contractions or Labour would not present this way, neither in intensity nor radiation.
Round ligament stretching doesnt make sense here and femoral hernia is not associated with these
symptoms.

1579. A 45yo mechanic presents with a reducible swelling in the groin, impulse on coughing is present. He has
mild dragging pain in the abdomen, otherwise hes normal. What is the management strategy?
a. Truss
b. Elective herniorrhaphy
c. Urgent herniorrhaphy
d. Elective herniotomy
e. Reassure
Key: Elective Herniorrhaphy (B)
Reason: Herniotomy is done in children, Truss not suitable here, Urgent herniorrhaphy is done only in
strangulated cases. Reassurance isnt suitable here.

1580. A 25yo man present with a mass in the groin after heavy lifting. Exam: mass is found just above and
medial to the pubic tubercle. It is reducible. On applying pressure on the internal ring,
cough impulse is still present. What is the most likely dx?
a. Direct inguinal hernia
b. Indirect inguinal hernia
c. Femoral hernia
d. Strangulated hernia
e. Femoral aneurysm
Key: Direct inguinal hernia (A)
Reason: Above and medial to the pubic tubercle, reducible inguinoscrotal swelling, ring occlusion test ve
indicates a direct inguinal hernia rather than an indirect inguinal hernia. Discussion: Hernias always contain a
portion of peritoneal sac and may contain viscera, usually small bowel and omentum.
Presentation:
* Swelling in the groin that may appear with lifting and be accompanied by sudden pain.
* Indirect hernias are more prone to cause pain in the scrotum and cause a 'dragging sensation'.
* An impulse (increase in swelling) may be palpable on coughing.
* It may not be possible to see the hernia if it is reduced.
* If a lump is present, it may be reducible
There are two types of inguinal hernia:
* Indirect: a protrusion through the internal inguinal ring passes along the inguinal canal through the
abdominal wall, running laterally to the inferior epigastric vessels. This is the more common form accounting
for 80% of inguinal hernias, especially in children. It is associated with failure of the inguinal canal to close
properly after passage of the testis in utero or during the neonatal period.
* Direct: the hernia protrudes directly through a weakness in the posterior wall of the inguinal canal, running
medially to the inferior epigastric vessels. It is more common in the elderly and rare in children.
Differential diagnosis: * Femoral hernia: this is seen in various forms, at simplest as a small swelling in the top
of the inside of the thigh. Alternatively, it may be deflected to appear higher as an inguinal hernia. It is either
irreducible or reduces only slowly with pressure. * Hydrocele (when differentiating from an inguinoscrotal
hernia, note that it is possible to get above a hydrocele on examination).
* Spermatic cord hydrocele.
* Lymph node swelling. * Abscess.
* Saphena varix.
* Varicocele.
* Bleeding. * Undescended testis.
Management:
* Conventional surgery was based on Bassini's operation; this consisted of apposition of the transversus
abdominis and transversalis fascia and the lateral rectus sheath to the inguinal ligament. The Shouldice
technique uses two layers of running suture in a similar fashion.
* However, the Lichtenstein technique is widely used, where a piece of open-weave polypropylene mesh is
used to repair and reinforce the abdominal wall. This operation is easier to learn, gives earlier mobility and has
a very low recurrence rate. The standard repair now uses prostheses, usually polypropylene mesh. It is,
however, associated with a slightly increased risk of infection.
* Some of the traditional meshes are heavy and associated with postoperative stiffness and pain. This has led
to the development of lighter meshes. A systematic review has failed to find any differences in long-term and
short-term complications between the two.
* Laparoscopic repair is usually reserved for recurrences and bilateral hernias. There is less postoperative pain,
full recovery is better, and return to work is faster. However, the price is increased compared with the
conventional approach, and there appears to be a higher number of serious complications of visceral
(especially bladder) and vascular injuries.
* There are two approaches: either the transabdominal preperitoneal (TAPP) or the totally extraperitoneal
(TEP) procedure. In TAPP, the surgeon goes into the peritoneal cavity and places a mesh through a peritoneal
incision over possible hernia sites. TEP is different, as the peritoneal cavity is not entered and mesh is used to
seal the hernia from outside the peritoneum. The mesh, where used, becomes incorporated by fibrous tissue.
* Surgery can be performed on a day-case basis, and for seven days afterwards the patient should avoid
driving and lifting. The patient should be able to resume normal activities over the subsequent 2-3 weeks, but,
with a heavy job, it can take up to six weeks to return to work.
* A truss may be required where surgery is inadvisable or refused; however, it can be difficult for patients to
manage and cannot be recommended as a definitive form of treatment.
Complications:
These include:
* Recurrence: 1.0% - most happening within five years of operation. Recurrence rate increases:
o In children aged younger than 1 year
o In elderly patients
o After incarcerations
o In those with ongoing increased intra-abdominal pressure
o Where there is growth failure
o With prematurity
o Where there are chronic respiratory problems
o In girls with sliding hernias
* Infarcted testis or ovary with atrophy.
* Wound infection. * Bladder injury.
* Intestinal injury.
* A hydrocele from fluid accumulation in the distal sac usually resolves spontaneously but sometimes requires
aspiration.

1581. A 35yo woman presents with a swelling in the neck. The swelling has increased in size gradually over the
last two years and the patient feels she has difficulty with breathing. Exam: mass measures 8cm by 10 cm, soft
and not warm to touch. It moves with deglutition. Which is the most appropriate management of this mass?
a. Partial thyroidectomy
b. Oral thyroxine
c. Oral propylthiouracil
d. Excision biopsy
Key: Partial thyroidectomy (A)
Reason: The mass is large and obviously arising from the thyroid. The fact that it is causing difficulty breathing
would indicate a thyroidectomy. Thyroxine would be given in hypothyroidism, PTU is given as an antithyroid
drug and excision biopsy is undertaken if CA is suspected which isnt the case here as the swelling is not hot.
The mass is also soft which doesnt go towards CA.
Discussion: Surgery is indicated in simple goitre if:
* There is clinical or radiological evidence of compression of surrounding structures, especially the trachea.
* There are substernal goitres, which are best removed surgically, as biopsy is difficult and clinical observation
without frequent CT or MRI scans is impossible.
* The goitre continues to grow.
* There are cosmetic reasons - for example, large or unsightly.
Types of thyroid operations:
* Thyroid lobectomy to remove a nodule (solitary hot or cold nodules) and goitres that occur in one lobe.
* Partial thyroid lobectomy to remove a solitary nodule in one specific part of the thyroid.
* Thyroid lobectomy with isthmectomy for benign Hrthle cell tumours and for non-aggressive thyroid
cancers.
* Subtotal thyroidectomy (leaving enough of the gland to produce some hormones) is now little used and has
been replaced by total thyroidectomy or thyroid lobectomy alone.
* Total thyroidectomy for thyroid cancers, Hrthle cell tumours and also increasingly for multinodular goitres
and patients with Graves' disease.
* Robotic surgery: advantages include three-dimensional imaging and tremor elimination. Robotic thyroid
surgeries include thyroid lobectomy, total thyroidectomy, central compartment neck dissection, and radical
neck dissection for benign and malignant thyroid diseases.
Thyroid surgery is safe in the elderly, assuming careful preoperative evaluation and risk stratification.
Complications:
Possible complications following thyroid surgery include:
* Minor complications such as collections of serous fluid (they resolve spontaneously if small and
asymptomatic but may require single or repeated aspiration if large) and poor scar formation.
* Bleeding, which may cause tracheal compression.
* Recurrent laryngeal nerve injury:
o Innervates all of the intrinsic muscles of the larynx, except the cricothyroid muscle.
o Patients with unilateral vocal fold paralysis present with postoperative hoarseness.
o Presentation is often subacute and voice changes may not present for days or weeks.
o Unilateral paralysis may resolve spontaneously.
o Bilateral vocal fold paralysis may occur following a total thyroidectomy and usually presents immediately
after extubation.
o Both vocal folds remain in the paramedian position, causing partial airway obstruction. *
Hypoparathyroidism: the resulting hypocalcaemia may be permanent but is usually transient. The cause of
transient hypocalcaemia postoperatively is not clearly understood. * Thyrotoxic storm: is an unusual
complication of surgery but is potentially lethal.
* Superior laryngeal nerve injury:
o The external branch provides motor function to the cricothyroid muscle.
o Trauma to the nerve results in an inability to lengthen a vocal fold and thus to create a higher-pitched
sound.
o The external branch is probably the most commonly injured nerve in thyroid surgery.
o Most patients do not notice any change but the problem may be career-ending for a professional singer.
* Infection: occurs in 1-2% of all cases. Peri-operative antibiotics are not recommended for thyroid surgery. *
Hypothyroidism.
* Damage to the sympathetic trunk may occur but is rare.
1582. A 46yo labourer reports swelling in the right groin. The non-painful swelling is observable in both the
erect and the recumbent positions. Exam: non-tender irreducible 4 cm mass in the right groin below and on
the medial side of the inguinal ligament. Which is the most likely dx in this pt.?
a. Indirect inguinal hernia
b. Femoral hernia
c. Saphenous Vein Varicocele
d. Hydrocele
Key: Femoral Hernia
Reason: The swelling is below and medial to the inguinal ligament. This rules out an inguinal hernia and a
hydrocele since they are present in the scrotum, away from the region described in the scenario. The non-
painful swelling doesnt disappear on lying down and is irreducible which rules out a saphenous vein
varicocele. Femoral hernia is the answer and in this case, is an irreducible hernia. This hernia can also get
incarcerated, which means it may or may not have tenderness. An incarcerated hernia can be strangulated
when it develops systemic symptoms and severe pain and tenderness.

1583. A camel rider sustained a kick to the lateral side of his right leg just below the knee caused by the camel
stick. The site is slightly bruised and tender to touch. During physical
examination, he is unable to either dorsiflex or evert the foot. There is loss of sensation over the front and
outer half of the leg and dorsum of the foot. If these observations are the result of damage to a nerve bundle,
which is the most likely nerve affected?
a. Lateral popliteal
b. Peroneal
c. Tibia
d. Sural
Key: Peroneal (B)
Reason: The fibular nerve or the peroneal nerve supplies the muscles and skin of the area stated. Only
possible answer.

1584. A 46yo woman presents with sudden episode of abdominal pain which started about 2h ago. The pain is
located in the epigastrium and radiates to her back. She has vomited twice since the onset of attack. The pain
is made worse by lying flat on her back and she is more comfortable sitting up and bending forwards. She was
informed of the presence of gallstones in her gall bladder four weeks earlier when she reported pain in the
right hypochondrium. The oral
temp=39C, BP=120/80mmHg and the radial pulse=118/min. There is no jaundice but there is
marked tenderness in the epigastrium both on deep and superficial palpation. Which is the
most appropriate inv for the cause of the patients pain?
a. Plain abdominal X-ray
b. Serum Amylase
c. Serum bilirubin
d. Barium Swallow
Key: Serum Amylase (B)
Reason: Sudden onset abdominal pain, radiating to back, accompanied by vomiting, made worse by lying flat,
gallstones +ve, epigastric tenderness, tachycardia, fever all point to Acute Pancreatitis. Serum Amylase is the
most appropriate test, Lipase is more sensitive and specific but takes time to rise. Serum bilirubin will not help
us in any way, neither will a needless Barium Swallow. An XRAY will waste useful time which can be used to
the patients benefit.
Discussion:
This is a case of acute pancreatitis, usually managed in the surgical ward. This is self-perpetuating pancreatic
inflammation by enzyme mediated autodigestion. Edema and fluid shifts causing hypovolemia as extracellular
fluid is trapped in the gut, peritoneum and retroperitoneum.
Causes: (GET SMASHED)
* Gallstones
* Ethanol
* Trauma
* Steroids
* Mumps
* Autoimmune (PAN)
* Scorpion venom
* Hypercalcemia, Hypothermia, Hyperlipidemia
* ERCP and emboli
* Drugs
* Pregnancy and Neoplasia also possible causes.
Symptoms:
* Gradual or sudden epigastric or central abdominal pain (severe, radiates to back, sitting forward may relieve)
* Vomiting
Signs:
* May be mild in serious disease - tachycardia, fever, jaundice, shock, ileus, rigid abdomen +/- local/general
tenderness.
* Periumbilical bruising (Cullens sign), Flank bruising (Grey Turners sign) from blood vessel autodigestion
and retroperitoneal haemorrhage.
Tests:
* Raised serum amylase >1000 units/ml. Starts to fall after 24-48 hours.
* Serum lipase raised more sensitive and specific for pancreatitis.
* ABGs to monitor acid base status and oxygenation.
* Abdominal X-ray no psoas shadow, retroperitoneal fluid increased.
* Erect chest X-ray helps exclude perforation (gas under diaphragm).
* CT is a standard choice of imaging, MRI may be even better. USG if gallstones + raised AST.
* ERCP if LFTs worsen. CRP >150 mg/L at 36 hours after admission is a predictor of severe pancreatitis.
Management:
* Severity assessment is essential, modified Glasgow criteria pg. 639.
* NPO, pass NG tube decreased pancreatic stimulation.
* IV Line, lots of 0.9% saline to counter third place sequestration until vitals are stable and urine flow is
>30mL/hour.
* Catheterize with Foleys.
* Analgesia with Pethidine 75-100mg/4 hours IM OR Morphine (may cause Oddi sphincter to contract more
but it is a better analgesic and not contraindicated.
* Vital monitoring.
* Daily CBC, urea & electrolytes, calcium, glucose, amylase, ABGs.
* If worsening, ITU. Give O2 if decreased O2-sats.
* In suspected abcess formation or pancreatic necrosis on CT, consider TPN +/- laparotomy and
debridement/necrosectomy.
* Antibiotics may help in severe disease Imipenem in more than 30% necrosis. There is no consensus on
prophylactic use if necrosis is present.
* ERCP + gallstone removal may be needed if there is progressive jaundice. Repeat imaging, CT is performed in
order to monitor progress.
Complications:
EARLY
* Shock
* ARDS
* Renal failure
* DIC
* Sepsis
* Hypocalcemia
* Hyperglycemia (transient 5% may need insulin)
LATE
* Pancreatic necrosis and pseudocyst
* Abcesses
* Bleeding from elastase eroding a major vessel like Splenic artery
* Thrombosis leading to bowel necrosis
* Fistula
* Recurrent edematous pancreatitis

1585. A 75yo Japanese woman reports repeated episodes of vomiting of undigested food mixed with blood.
She has lost 5 kgs in weight over the last one month. Clinical exam: shows a frail woman with mild conjunctival
pallor. Exam: non-tender slightly mobile mass in the epigastric region. Which is the most likely dx?
a. Colon cancer
b. Gastric cancer
c. Gall bladder cancer
d. Oesophageal cancer
Key: Gastric CA (B)
Reason: Undigested food vomiting with blood rules out Colon CA, Gallbladder CA. Oesophageal CA would have
dysphagia as a symptom. Gastric CA is the most likely diagnosis.
Discussion: Associated with pernicious anaemia, blood group A, H. pylori, atrophic gastritis, adenomatous
polyps, smoking, nitrosamine exposure, diet (high nitrates, high salt, pickling, low Vit. C).
Borrmanns classification:
i. Polypoid.
ii. Excavating.
iii. Ulcerating and raised.
iv. Diffusely infiltrative.
Presents with non specific symptoms like dyspepsia (>1 month and age>50 warrants investigation), weight
loss, vomiting, dysphagia, anaemia.
Signs suggesting incurable disease = epigastric mass, hepatomegaly, jaundice, ascites, Virchows
node/Troisiers sign, Acanthosis nigricans.
Spread is local, lymphatic, blood-borne, transcoelomic eg. Krukenberg tumour.
Tests: Gastroscopy + multiple ulcer edge biopsies. Endoscopic ultrasound to evaluate depth of invasion.
CT/MRI to stage. Staging laparoscopy for locally advanced tumours. Cytology of peritoneal washing can
identify peritoneal mets.
Treat by Surgery Bilroth I & II, Roux-en-Y loop gastrectomy + combination chemotherapy.

1586. A 45yo man, known to be chronically addicted to alcohol, presents in the ED and reports two episodes
of vomiting fresh bright red blood in the previous 6h. He estimated the volume blood vomited at each bout to
be more than 500mls. Clinical exam: the radial pulse=120/min,
BP=90/60mmHg. There is no mass or tenderness in the epigastrium. The liver is palpable for 3
cm below the costal margin and not tender. The patient is not jaundiced. The physician
resuscitates the patient with oxygen by face mask, rapid infusion of intravenous normal saline
while he requests for haemoglobin level and whole blood for transfusion. Which is next
appropriate step in management?
a. Barium Swallow
b. Exploratory laparotomy
c. CT scan of the abdomen
d. Upper gastrointestinal endoscopy
Key: Upper GI Endoscopy (D)
Reason: The patient presented with emergency Upper GI Bleed, he is an alcoholic who vomited fresh red
blood with a palpable liver. Probable variceal bleed secondary to CLD brought on by Chronic Alcoholism.
Endoscopy should be performed on an urgent basis after resuscitation. Should be performed within 4 hours of
resus or when bleeding is ongoing within 24h according to OHCM pg. 254.

1587. A 42yo woman reports to the surgeon that she is worried about a lump that she feels the right breast.
The surgeon observes a 2 cm by 3 cm mass in the right lower quadrant of the breast. There are no associated
skin changes and the mass has limited mobility. There is no discharge from the nipple. There is no axillary
lymph node enlargement. Examination of the left breast and axilla was completely normal. A mammogram
report suggests the presence of
microcalcifications. Which is the most appropriate next step in the management of this pt?
a. Observation for one year and repeat the mammography
b. A needle-guided biopsy of the breast
c. Excision biopsy of the breast
d. Partial mastectomy
Key: Needle Guided Biopsy (B)
Reason: Biopsy all breast lumps!!! Looks harmless, doesnt have any danger signs in the history or
examination. Observation is advisable but only with a biopsy, not without it. Excision and mastectomy are
radical choices for an undiagnosed lump.
1588. A 45yo man presents with a mass on the right side of the face. The mass was first observed three
months ago but has recently become visibly larger. He feels pain over the mass and is unable to blow a
whistle. Clinical examination shows that the mass is likely to be the parotid gland. An oral examination shows
a foul smelling discharge from the duct of the gland and gentle probing shows that it is stenosed at the
meatus. Which of the following features suggests that the mass might be malignant?
a. Presence of pain
b. Recent enlargement
c. Facial nerve palsy
d. Stenosed duct meatus
Key: Facial nerve palsy (C)
Reason: Out of the options given, only facial nerve palsy is a sign of malignant invasion showing invasion. The
rest can be explained by benign ailments.
1589. A 6yo boy presents with jaundice following treatment with sulfathiazole. Investigations
suggest that the jaundice is due to haemolysis caused by G6DP deficiency. Which is true
regarding etiology of G6DP deficiency?
a. Inherited as autosomal dominant condition
b. Inherited as sex-linked dominant condition
c. Inherited as sex-linked recessive condition
d. Results from auto-antibodies to red cell antigens
Key: X-Linked recessive (C)
Reason: G6PD is inherited as an X-Linked recessive condition, which means it can occur only in homozygotes.
Usually happens in males.
Discussion: The enzyme glucose-6-phosphate dehydrogenase (G6PD) is one of the enzymes of the pentose
phosphate pathway. This pathway is involved in keeping an adequate amount of the coenzyme nicotinamide
adenine dinucleotide phosphate (NADPH) in cells.
Classes of G6PD deficiency enzyme variants:
* Severe (I) - chronic non-spherocytic haemolytic anaemia.
* Severe (II) - less than 10% of normal enzyme activity.
* Moderate (III) - 10-60% of normal enzyme activity.
* Mild to none (IV) - 60-150% of normal enzyme activity.
* None (V) - greater than 150% of normal enzyme activity.
History:
* Depends upon the severity of the enzyme deficiency.
* Most are asymptomatic.
* May be a history of neonatal jaundice, severe enough to require exchange transfusion.
* History of drug-induced haemolysis.
* Gallstones are common.
Examination:
* Most often, examination is unremarkable.
* Pallor of anaemia.
* During a crisis jaundice occurs.
* Back or abdominal pain (usually occurs when >50% haemolysis occurs).
* Splenomegaly may occur.
Investigations:
* FBC - anaemia.
* Macrocytosis - due to reduced folic acid which is required for erythropoiesis.
* Reticulocyte count - raised; gives indication of the bone marrow activity (bone marrow sampling thus not
needed).
* Blood film - acute haemolysis from G6PD deficiency can produce Heinz bodies, which are denatured
haemoglobin and bite cells (cells with Heinz bodies that pass through the spleen have part of the membrane
removed).
* Haemolysis - reduced levels of haptoglobin and elevated levels of bilirubin; haemoglobinuria.
* Direct antiglobulin test - to look for other causes of haemolysis; should be negative in G6PD deficiency.
* Renal function - to ensure no renal failure as a precipitant.
* LFTs - to exclude other causes of raised bilirubin.
* G6PD enzyme activity - is the definitive test (as opposed to the amount of G6PD protein).
* Performing assays for G6PD during haemolysis and reticulocytosis may affect levels and not reflect baseline
values.
* Ultrasound examination of the abdomen may reveal splenomegaly and gallstones.
Management:
* Avoidance of substances that may precipitate haemolysis is essential. Usually no further management is
required, although if haemolysis is marked there may be benefit from folate supplementation.
MANAGEMENT OF ACUTE HAEMOLYSIS
* Seek specialised advice.
* Blood transfusions may be needed.
* Dialysis may be required in acute kidney injury.
* Infants - more susceptible to neonatal jaundice, especially if premature, and exchange transfusion may be
required.
MANAGEMENT OF CHRONIC HAEMOLYSIS OR STABLE DISEASE
* Splenectomy may help.
* Supplementation with folic acid.
* Avoidance of precipitating drugs, and broad beans (usually favism occurs in the Mediterranean variety of the
disease).
* Avoid naphthalene - found in mothballs.

1590. A 5yo previously healthy child has a 1-day history of severe pain in the throat, breathing
difficulties and fever. On examination you find an anxious, septic-looking child with drooling of
saliva and stridor. Which is the most appropriate initial management?
a. Intubation under general anaesthesia
b. Insertion of nasogastric tube
c. Fluid resuscitation and antibiotics IV
d. Anteroposterior & lateral neck x-ray
Key: Intubation under GA (A)
Reason: This is Acute Epiglottitis, a respiratory emergency caused by H. influenza causing severe edema of the
throat and a huge inflamed cherry red epiglottis. Throat exam is severely contraindicated in this case and the
intervention of choice is intubation with an ETT under GA. NG tube makes no sense, fluid resuscitation and
antibiotics have a definite role along with
radiographs, but all those things come after the initial management. Antibiotic management is Penicillin and
Ceftriaxone 2g IV x 12 hourly.
Discussion:
Presentation:
Many of the signs and symptoms associated with acute epiglottitis are common, and may occur in many less
serious disorders. Patients with epiglottitis may present with any of the following, and the symptoms may
evolve very quickly over a period of a few hours.
The most common symptoms:
* Sore throat.
* Odynophagia (painful swallowing).
* Drooling (inability to swallow secretions).
* Fever.
* Anterior neck tenderness over the hyoid bone.
Other features:
* High temperature.
* Tachycardia.
* Ear pain.
* Cervical lymphadenopathy.
* The 'tripod sign' - the patient leans forward on outstretched arms to move inflamed structures forward,
thereby easing the upper airway obstruction.
With more severe epiglottitis:
* Dyspnoea.
* Dysphagia.
* Dysphonia.
* Stridor (late finding - indicates airway obstruction).
* Respiratory distress.
Differential diagnosis:
The differential diagnosis will depend on the presenting symptoms and age of the patient; however, generally
included are: * Pharyngitis * Laryngitis * Inhaled foreign body * Croup * Retropharyngeal abscess
Investigations:
Patients who are suspected of having acute epiglottitis should not have their throat examined with the aid of a
tongue depressor, due to the risk of laryngeal obstruction; rather, they should be urgently referred for
laryngoscopy.
* Fibre-optic laryngoscopy remains the 'gold standard' for diagnosing epiglottitis, as the epiglottis can be
seen directly. Laryngoscopy in these patients should only be performed in areas such as operating theatres
which are prepared for intubation or tracheostomy in the event of upper airway obstruction.
* Lateral neck X-ray may be useful if laryngoscopy is not possible. Soft-tissue radiograph of the neck may show
the 'thumbprint sign'.
* Throat swabs may be taken when the airway is secure, or when intubation/tracheostomy facilities are at
hand. Streptococci are becoming the major pathogens in acute epiglottitis now.
* Blood cultures may be taken if the patient is systemically unwell.
* CT or MRI scans may be performed if abscess formation is suspected.
Management:
* The incidence of acute epiglottitis is falling in children but rising in adults. Initial presentation may resemble
a viral sore throat, so a high index of suspicion is needed. Emergency referral is required if signs of airway
obstruction are present (stridor). Deterioration in symptoms may be rapid. Management is usually
conservative but intubation is occasionally needed (if >50% airway obstruction occurs).
* Surgical tracheostomy may be required in patients with severe airway obstruction in whom intubation has
not been possible.
* Abscess formation is being increasingly seen as epiglottitis cases in adults increase. Drainage may be
required in some patients.
Complications:
* Epiglottitis, if not adequately treated, may occasionally result in: * Abscess formation * Meningitis *
Septicaemia * Pneumothorax * Pneumo-mediastinitis (very rare)
1591. A 6yo boy has been noticed to have problems with co-ordinating his voluntary movements over the last
two years. He has a waddling gait and needs to support himself on his hands when rising from the floor. He
has larger calves than other boys but he runs more slowly. Which is the most likely dx?
a. Myotonia
b. Myasthenia gravis
c. Duchenne muscular dystrophy
d. Muscular atrophy
Key: Duchenne Muscular Dystrophy (C)
Reason: Presentation by 6 years, difficulty co-ordinating movements, waddling gait, Positive Gowers sign,
pseudohypertrophy of calf muscles all point to Duchenne Muscular Dystrophy as the cause in this boy.
Diagnose by muscle biopsy and DNA testing. Myasthenia Gravis would have descending paralysis, Myotonia
(Myo from Greek; muscle, and Tonus from Latin; tension) is a symptom of a small handful of certain
neuromuscular disorders characterized by delayed relaxation (prolonged contraction) of the skeletal muscles
after voluntary contraction or electrical stimulation and Muscular atrophy can never have hypertrophied
calves.

1592. A previously healthy, 10 month female child presents to your clinic with a 1-day history of high fever,
runny nose and conjunctivitis. The child looks unwell and is irritable. Exam: child's
oropharynx shows that it is inflamed and there are small white spots on the oral mucosa. Which is the most
likely dx?
a. Kawasaki disease
b. Parvovirus infection
c. Herpes zoster
d. Measles
Key: Measles (D)
Reason: These symptoms along with the pathognomonic Koplik spots on the oral mucosa point only to
measles. It cannot be herpes zoster because there isnt a lot of pain and dermatomal distribution, parvovirus
would cause red cell crisis or aplastic crisis and would not present this way. Kawasaki disease would have rash,
a strawberry tongue and limb involvement like arthralgias and swelling of hands or feet. Also predisposes to
coronary arteritis, check for aneurysms by ECHO!
Discussion:
MEASLES:
Rash for at least three days.
Fever for at least one day and at least one of the following:Cough
Coryza
Conjunctivitis
Prodrome: This lasts 2-4 days with fever, cough, runny nose, mild conjunctivitis and diarrhoea.
Koplik's spots are pathognomonic and appear on the buccal mucosa - opposite the second molar teeth - as
small, red spots, each with a bluish-white speck (sometimes compared with a grain of rice) in the centre. They
occur in 60-70% of patients during the prodrome and for up to 2-3 days after the rash disappears.
Rash (morbilliform = measles-like):This is first seen on the forehead and neck and spreads, involving the
trunk and finally the limbs, over 3-4 days. It may become confluent in some areas.
The rash then fades after 3-4 days in the order of its appearance.
It leaves behind a brownish discoloration, sometimes accompanied by fine desquamation.
Often, there is high fever (may be >40C) and a non-productive cough, with the patient being clearly ill.
Also, swelling around the eyes and photophobia may be present.
Diagnosed clinically!!
Management:
Uncomplicated measles is usually self-limiting and treatment is mainly symptomatic, with paracetamol or
ibuprofen and with plenty of fluids. Patients should remain at home to limit disease spread.
Monitor patients carefully for signs of complications and consider hospitalisation if these appear.

1593. A 3d term, breast-fed infant is brought by the mother who reports that the child has not been active and
not feeding well. She also notices jaundice, which was not present at birth and is increasing. Exam: the
temp=35.4C, and the liver is palpable 2 cm below the costal margin.
Which is the most likely dx?
a. Rhesus isoimmunisation
b. Inadequate breast milk
c. Congenital biliary tract obstruction.
d. Sepsis
Key: Sepsis (D)
Reason: Biliary atresia would have conjugated bilirubin and pale stools clinically, breastfeeding jaundice, which
manifests in the first 3 days of life is caused by insufficient production or intake of breast milk. Rhesus disease
doesnt make sense, and the hypothermia would point towards Sepsis as being the likely cause of jaundice and
other symptoms. Liver is also normally palpable in children of this age so take that symptom with a pinch of
salt.

1594. A 65yo woman with DM, HTN and normal kidney function underwent a total right hip
replacement. She had massive haemorrhage during the operation and was given 8 units of
packed RBC. The blood pressure dropped to 60/40 mmHg for about two hours before it was
corrected with blood transfusion. Two days after the surgery the serum creatinine level rose to
4.2 mg/dl (normal <1.5 mg/dl), BUN was 50 mg/dl (normal 10-20 mg/dl) and potassium 5.0
mmol/L (normal 3.5-5.0 mmol/l). There were brown granular casts in the urine sediment. Which
is the most likely cause of this complication?
a. Diabetic nephropathy
b. Malignant hypertension
c. Acute tubular necrosis
d. Interstitial nephritis
Key: Acute Tubular Necrosis (C)
Reason: This woman recently underwent major surgery and even though the blood loss was corrected with
blood transfusion of 8 units of packed cells, the blood pressure dropped to 60/40 mmHg before being
corrected which could possibly lead to Acute Kidney Injury or ATN in this case. The rising Creatinine, Potassium
and BUN all point to the kidney being unable to undertake its normal functions. Diabetic Nephropathy would
have a setting other than post-op patients, malignant hypertension doesnt make sense because there is no
HTN in this case, interstitial nephritis would mention history of certain drugs that cause this.

1595. A 78yo pt is diagnosed with metastatic lung cancer; there is no cure for his condition. His son tells the
physician that in the case of a diagnosis of cancer, the physician must not tell his father. He wishes that his
father does not suffer any psychological distress caused by the knowledge of a terminal diagnosis. Which one
of the following ethical principles supports the sons request?
a. Patient autonomy
b. Beneficence
c. Justice
d. Non-maleficence
Key: Non-Maleficence (D)
Reason: Justice and Autonomy would involve the patient knowing about the diagnosis so they are
automatically ruled out, Beneficence entails a paternalistic approach in which the doctor does what is best for
the patient, again regarding him as aware of the diagnosis. Non-maleficence is the appropriate choice
considering it basically means to do no harm on the doctor's part, by listening to the wishes of the
attendant.

1596. A 23yo single male was brought to Emergency exhausted and frightened. His father tells you that his
son, who was previously healthy, had, for no apparent reason, a sudden attack of fear, dizziness, sweating,
palpitations and the feeling that his heart is going to stop beating. The
symptoms started to decrease gradually after about 10 minutes. Which is the most likely dx?
a. Panic attack
b. Delirious state
c. Alcohol withdrawal phenomena
d. Social phobia
Key: Panic attack (A)
Reason: The short term nature of the symptoms point to panic attack. We have no alcohol history to regard
withdrawal, there is no public setting to regard social phobia and a delirious state would present with
symptoms of mania and would almost never need intervention, wouldnt resolve on its own.

1597. A 30yo woman, G2P1, at 37 weeks gestation mentions that her 3-year-old son has just
developed chickenpox. She is not certain whether she has had the disease herself. Which is the
next step in management?
a. Administration of varicella-zoster immune globulin IM
b. Measurement of varicella IgM level
c. Acyclovir tablets orally
d. Measurement of varicella IgG level
Key: Measurement of Varicella IgG level (D)
Reason: If you don't know the immunity status you have to measure varicella IgG level. If +ve nothing to do
but reassure the patient. If negative VZIG to be given.

1598. A 24yo primigravida presents to the ED with a history of 8-week amenorrhoea followed by
heavy vaginal bleeding and severe, crampy abdominal pain. Exam: HR=110/min and
BP=120/80mmHg. The uterus is bulky. The cervix is dilated and there is active bleeding from the cervical os,
but no tissue has been expelled. Which of the following is the most likely dx?
a. Inevitable abortion
b. Threatened Abortion
c. Incomplete abortion
d. Missed Abortion
Key: Inevitable abortion (A)
Reason: The cervix is dilated which rules out missed abortion completely. Threatened abortion would only
have bleeding. Incomplete abortion will have some expulsion of tissue, this is clearly a case of inevitable
abortion.
Discussion:
Classification of miscarriage is as follows:
Threatened miscarriage: mild symptoms of bleeding. Usually little or no pain. The cervical os is closed.
Inevitable miscarriage: usually presents with heavy bleeding with clots and pain. The cervical os is open. The
pregnancy will not continue and will proceed to incomplete or complete miscarriage.
Incomplete miscarriage: this occurs when the products of conception are partially expelled. Many incomplete
miscarriages can be unrecognised missed miscarriages.
Missed miscarriage: the fetus is dead but retained. The uterus is small for dates. A pregnancy test can remain
positive for several days. It presents with a history of threatened miscarriage and persistent, dirty brown
discharge. Early pregnancy symptoms may have decreased or gone.
Habitual or recurrent miscarriage: three or more consecutive miscarriages.
Risk factors:
Age: it is more frequent in women aged >30 years and even more common in those aged >35 years (due to
an increased risk of random chromosomal abnormalities).
Incidence increases with the number of births: 6% in the first and second pregnancies and 16% in further
pregnancies.
Cigarette smoking of >14 per day doubles the risk over non-smokers.
Excess alcohol. Even low amounts - four units a week of alcohol consumption during early pregnancy - have
been shown to increase the risk of spontaneous abortion substantially.[2]
Illicit drug use.
Uterine surgery or abnormalities - eg, incompetent cervix.
Connective tissue disorders (systemic lupus erythematosus, antiphospholipid antibodies - lupus
anticoagulant/anticardiolipin antibody).
Uncontrolled diabetes mellitus
Management:
Anti-D rhesus prophylaxis (250 IU) should be offered to all rhesus-negative women who have a surgical
procedure to manage a miscarriage.
However, anti-D rhesus prophylaxis does not have to be given to those women who:
* Receive solely medical management for an ectopic pregnancy or miscarriage.
* Have a threatened miscarriage.
* Have a complete miscarriage.
* Have a pregnancy of unknown location
Conservative management:
If a scan at the EPAU confirms a first-trimester miscarriage, expectant management (waiting to see if the
miscarriage will resolve naturally without intervention) for 7-14 days can be offered as the initial management
strategy.
If bleeding and pain have not started or bleeding and pain are persisting and/or increasing then these women
should have a repeat ultrasound examination performed. Alternative management may be offered to those
whose miscarriage is incomplete or has not started.
Those women who have resolution of bleeding and pain should perform a pregnancy test after three weeks.
If this is still positive, they need to be reviewed and considered for either medical or surgical management.
Women should be counselled so they are fully aware of what to expect. In most cases, resorption of fetal
tissue occurs without much bleeding. However, loss of fetal tissue vaginally can be associated with heavy
bleeding and pain and the patient may prefer to opt for medical or surgical management rather than cope
with this.
Medical management:
Women may opt for medical management at the initial stage or following expectant treatment.
Medical management can cause more pain and bleeding than surgical management but patients who opt for
this approach cite 'being in control' and avoiding general anaesthesia as the main reasons for their choice.
Vaginal misoprostol should be offered for the medical treatment of missed or incomplete miscarriage.
Oral misoprostol is an acceptable alternative if this is the woman's preference.
Mifepristone should no longer be given as a treatment for missed or incomplete miscarriage.
Women should be advised that bleeding can continue for up to three weeks.
Women should perform a pregnancy test three weeks after receiving medical management, unless they have
worsening symptoms. If these occur, they should be reviewed to ensure there is no molar or ectopic
pregnancy.
Surgical management
Clinical indications for offering surgical evacuation include persistent excessive bleeding, haemodynamic
instability, evidence of infected retained tissue and suspected gestational trophoblastic disease.
Where clinically appropriate, women should be offered a choice of:Manual vacuum aspiration under local
anaesthetic in an outpatient or clinic setting.
Surgical management in a theatre, under general anaesthetic.
Vacuum aspiration is safe, quick to perform and less painful than sharp curettage
Serious complications of surgery include perforation, cervical tears, intra-abdominal trauma, intrauterine
adhesions and haemorrhage.
Tissue obtained at the time of miscarriage should be examined histologically to confirm pregnancy and to
exclude ectopic pregnancy or gestational trophoblastic disease.
Complications
Expectant management has been shown to lead to a higher risk of incomplete miscarriage, need for
unplanned (or additional) surgical emptying of the uterus, bleeding and need for transfusion
However, risk of infection and psychological outcomes are similar for expectant and surgical management.
After complete miscarriage, bleeding normally ceases within 10 days. If part of the placenta remains,
bleeding may continue with cramps. If this occurs then a repeat ultrasound should be undertaken and surgery
is often required.

1599. A 46yo woman comes for a routine gynaecological visit. On pelvic examination, a 1-cm red, granular
lesion is noted on the posterior cervical lip, which is firm and bleeds on contact. Which is the next best step for
establishing a dx?
a. Cervical cytological smear
b. Punch biopsy
c. Transvaginal ultrasound
d. Colposcopy
Key: Punch biopsy (B)
Reason: The most appropriate or next best step for establishing a diagnosis would only be punch biopsy in this
case. Smear is irrelevant because the lesion has already been observed, most likely a polyp. TVU also not likely
because it wouldnt help at all. Colposcopy is unneeded because a biopsy can easily be obtained with a Cuscos
speculum.
Discussion:
Polyps are the most common benign neoplasms of the cervix (found in 4% of the gynaecological population).
They may be endocervical or cervical:
Endocervical polyps are most usually found in the fourth to sixth decade of life. They are cherry red lesions
which may be single or multiple and may appear as a pedunculated lesion on a stalk of varying length.
Cervical polyps are equally benign and tend to occur as single, smooth grey-white lesions that bleed easily if
touched.
Polyps tend to be asymptomatic but may also present with abnormal bleeding (such as during intercourse,
heavy menstrual periods, between menstrual periods or after the menopause). Occasionally, they may grow
big enough to obstruct the external os and so cause infertility. Malignancy is rare.
Polyps may be removed but should all be sent to histology. If the woman is asymptomatic, this can simply be
done by twisting them off. Haemorrhage may also occur and require cautery for haemostasis. Surgical
dilatation and curettage, electrosurgical excision or hysteroscopic polypectomy may be performed for more
persistent lesions. Occasionally, the appearance of the healed cervix following a cone biopsy can mimic a large
polyp.
All patients with suspicious lesions or who are symptomatic should be referred to a specialist clinic and the
lesions sent to histology.

1600. A 31yo woman, G5P4, who has amenorrhoea for 12 weeks and a positive pregnancy test
presents to the ED with vaginal bleeding. Symphyseal-fundal height measurement corresponds
to 22 weeks gestation. Ultrasound examination reveals bilateral cystic masses. No fetal parts are seen during
the examination. The cervix is closed. Which is the most likely dx?
a. Tubal pregnancy
b. Endometriosis
c. Hydatidiform mole
d. Threatened abortion
Key: Hydatidiform Mole (C)
Reason: The inconcordant amenorrhea and SFH point towards point towards a molar pregnancy as the likely
cause with the bilateral cystic masses. No fetal parts are seen which rule out threatened abortion.
Endometriosis would not present with amenorrhea and a tubal pregnancy wouldnt have the SFH seen in this
patient.
Discussion:
Complete molar pregnancies, all the genetic material comes from the father. An empty oocyte lacking
maternal genes is fertilised. Most commonly (75-80%) this arises from a single sperm duplicating within an
empty ovum. Less often an empty ovum is fertilised by two sperm. There is no fetal tissue.
Partial molar pregnancies, the trophoblast cells have three sets of chromosomes (triploid). Two sperm are
believed to fertilise the ovum at the same time, leading to one set of maternal and two sets of paternal
chromosomes. Around 10% of partial moles are tetraploid or mosaic in nature. There is usually evidence of
fetal tissue or fetal blood cells in a partial molar pregnancy. An embryo may be present at the start.
Tests:
Urine and blood levels of hCG. A urine pregnancy test should be performed in all cases of persistent or
irregular vaginal bleeding after a pregnancy event. Levels of hCG may be of value in diagnosing molar
pregnancies but are far more important in disease follow-up.
Histology. Definitive diagnosis is made by histological examination of the products of conception.
Ultrasound: in the first trimester may not be reliable. The typical 'snowstorm' appearance occurs mainly in
the second trimester, showing a heterogeneous mass with no fetal development, and theca-lutein ovarian
cysts.
Because of the lack of diagnostic reliability of ultrasound, products of conception from all non-viable
pregnancies should undergo histological examination in order not to miss the diagnosis, and the chance of
monitoring to prevent complications.
Management of hydatidiform moles:
Suction curettage is the method of choice of evacuation for complete molar pregnancies.
Suction curettage is the method of choice of evacuation for partial molar pregnancies except when the size
of the fetal parts deters the use of suction curettage and then medical evacuation can be used.
A urinary pregnancy test should be performed three weeks after medical management of failed pregnancy if
products of conception are not sent for histological examination.
Anti-D prophylaxis is required following evacuation of a PHM.
Follow up:
Two-weekly serum and urine samples until hCG concentrations are normal.
After hCG levels return to normal, monthly urine hCG testing. This continues for six months from evacuation
if levels have normalised within eight weeks; if not, monitoring continues for six months from when levels
became normal
Future pregnancy:
Women being monitored after molar pregnancy should be advised not to conceive until their hCG levels have
been normal for six months.
Women with GTD should be advised to use barrier methods of contraception until hCG levels revert to
normal. Once hCG levels have normalised, the combined oral contraceptive pill (COCP) may be used.

1601. A married 25yo woman presents with 6h hx of abdominal pain located in the LIF. The pain is persistent,
of increasing intensity and not radiating first experienced while she was lying down. She feels giddy when she
tries to stand erect. The last menstrual period was 6 weeks ago. The radial pulse=130/min and BP=80/40
mmHg. Pelvic US shows free intra-peritoneal fluid. What is the most appropriate next step in management?
a. Immediate laparoscopy.
b. Immediate laparotomy.
c. Pregnancy test (urine or serum).
d. Observation for 24 hours in the ICU
Key: Pregnancy test (urine or serum) (C)
Reason: This is a classic case of ruptured Ectopic pregnancy and the best step would pregnancy test to
establish the diagnosis. The immediate next step if the patient is hemodynamically unstable would be an
immediate laparoscopy. Observation wouldnt help anyone and a laparotomy would be considered in third-
world countries.
Discussion:
The majority of ectopic pregnancies occur in the ampullary or isthmic portions of the Fallopian tubes.
Risk factors:
Fertility treatments and intrauterine contraceptive devices (IUCDs) are the most important associated risk
factors.
Pelvic inflammatory disease may cause complete tubal occlusion or delay the transport of the embryo so that
implantation occurs in the tube. Adhesions from infection and inflammation from endometriosis may play a
part.
Ectopic pregnancy has been reported in tubes that have been divided in a sterilisation operation and where
they have been reconstructed to reverse one.
Ectopic pregnancy can occur in the treatment of infertility.
Right-sided tubal pregnancy is more common than left-sided. This is thought to be from spread of infection
from appendicitis.
Presentation:
Symptoms and signs of ectopic pregnancy can resemble those of other more common conditions, including
urinary tract infections and gastrointestinal conditions.
The most common symptoms are:
* Abdominal pain.
* Pelvic pain.
* Amenorrhoea or missed period.
* Vaginal bleeding (with or without clots).
* Other symptoms may include:
* Dizziness, fainting or syncope.
* Breast tenderness.
* Shoulder tip pain.
* Urinary symptoms.
* Passage of tissue.
* Rectal pain or pressure on defecation.
There may be a history of a previous ectopic pregnancy. After one ectopic pregnancy the chance of another
in the other tube is much increased.
If the ectopic pregnancy has ruptured, bleeding is profuse and there may be features of hypovolaemic shock,
including feeling dizzy on standing. Most bleeding will be into the pelvis and so vaginal bleeding may be
minimal and misleading.
Diarrhoea and vomiting are possible, atypical clinical features of ectopic pregnancy.
Tests:
The most accurate method to detect a tubal pregnancy is transvaginal ultrasound.
* This can identify the location of the pregnancy and also whether there is a fetal pole and heartbeat.
* Human chorionic gonadotrophin (hCG) levels are performed in women with pregnancy of unknown location
who are clinically stable.
* hCG levels are taken 48 hours apart. If there is a change in concentration between 50% decline and 63% rise
inclusive over 48 hours then the woman should be referred for clinical review in an early pregnancy
assessment service within 24 hours
Management:
* Admit as an emergency if the diagnosis of ectopic pregnancy is considered a possibility. A bedside pregnancy
test should be performed on all women of childbearing age presenting with lower abdominal pain where
pregnancy is even the remotest possibility.
* Anti-D rhesus prophylaxis should be given (at a dose of 250 IU) to all rhesus negative women who have a
surgical procedure to manage an ectopic pregnancy.
* Conservative management may be appropriate if the levels of hCG are falling and the patient is clinically
well. Repeat hCG levels are performed in these cases.

1602. A 40yo man has fallen off a roof. He is shocked and has chest pain. There is a delay between the radial
and femoral pulse. His CXR=widening of the mediastinum. What is the single most likely dx?
a. Cardiac tamponade
b. Diaphragmatic rupture
c. Fx ribs
d. Tension pneumothorax
e. Traumatic rupture of aorta
Key: Traumatic rupture of the aorta (E)
Reason: The wide mediastinum and the radio-femoral delay make Traumatic rupture of the Arch of the Aorta
the most likely diagnosis in this case. Tension Pneumothorax will not present this way and a rib fracture would
be less severe than the patient is currently. Diaphragmatic rupture would have bowel sounds in the chest and
cardiac tamponade would have Becks triad Muffled heart sounds, distended neck veins and decreased
blood pressure.

1603. A 36yo woman presents with swelling in the groin. Exam: swelling is diffuse and soft and lies below the
inguinal ligament. It empties with minimal pressure and refills with release. There is a cough impulse and it
disappears on lying down. On the calf of the same leg there are varicosities on the medial aspect. What is the
most likely dx?
a. Varicose vein
b. Varicocele
c. Saphena varix
d. Femoral hernia
e. Inguinal hernia
Key: Saphena Varix (C)
Reason: The emptying on lying down and the coexistence of varicose veins make a saphena varix the most
likely answer to this question. It is a dilatation of the saphenofemoral junction due to incompetent valves
when the saphenous vein drains into the femoral vein. Femoral and Inguinal hernia not possible because the
swelling is diffuse and empties with minimal pressure. Varicose veins are present but they are not the choice
because the saphena varix is immediately below the inguinal ligament.

1604. A man presents with a swelling above the groin crease in the abdomen. He has not had any med
problems of note. What is the most probable dx?
a. Inguinal hernia
b. Spigelian hernia
c. Testicular tumor
d. Epidydimal cyst
e. Irreducible hernia
Key: Spigelian Hernia (B) Reason: A Spigelian hernia (or lateral ventral hernia) is a hernia through the spigelian
fascia, which is the aponeurotic layer between the rectus abdominis muscle medially, and the semilunar line
laterally. There is a common misconception that they protrude below the arcuate line owing to deficiency of
the posterior rectus sheath at that level, but in fact the defect is almost always above the arcuate line. These
are generally interparietal hernias, meaning that they do not lie below the subcutaneous fat but penetrate
between the muscles of the abdominal wall; therefore, there is often no notable swelling. All the other
options lie below the groin crease i.e. the inguinal ligament.
Discussion:
Spigelian: this is a hernia through the linea semilunaris muscle. Initially this causes localised pain exacerbated
by straining and coughing, but the pain may become less localised and more an ache with time. Bulge can
often be seen in the lower abdomen with the patient erect and straining. This can be reduced by pressure with
a 'gurgling' noise and then the hernia orifice can often be felt. However, the defect may not be palpable or a
bulge may be found distant from the site. This needs prompt repair.
1605. A 70yo man presents with acutely painful, pale paralysed and pulseless left leg. He is noted to have a-
fib. What is the most probable dx?
a. Intermittent claudication
b. Cardiovascular syphilis
c. Buergers disease
d. Chronic limb ischemia
e. Acute limb ischemia
Key: Acute limb ischemia (E)
Reason: This is acute presentation of limb ischemia because the atrial fibrillation has thrown a clot which has
blocked arteries in the leg leading to this acute picture. Chronic ischemia would present with amputation or
gangrene plus lipodermatosclerosis of limbs, intermittent claudication would present with pain on walking and
no pain on rest, Buergers disease would need a smoking history and syphilis of the CVS would have
aneurysms of the large arteries rather than small vessel involvement leading to limb symptoms.
Acute limb ischaemia is most often due to either acute thrombotic occlusion of a previously partially occluded,
thrombosed arterial segment, or to embolus from a distant site. Without surgical revascularisation, complete
acute ischaemia leads to extensive tissue necrosis within six hours.
Presentation: The affected part becomes pale, pulseless, painful, paralysed, paraesthetic and 'perishing with
cold' ('the 6 Ps')
Investigations:
Hand-held Doppler ultrasound scan may help demonstrate any residual arterial flow.
Blood tests
FBC (ischaemia is aggravated by anaemia).
ESR (inflammatory disease - eg, giant cell arteritis, other connective tissue disorders).
Glucose (diabetes).
Lipids.
Thrombophilia screen.
If diagnosis is in doubt, perform urgent arteriography.
Investigations to identify the source of embolus:
ECG.
Echocardiogram.
Aortic ultrasound.
Popliteal and femoral artery ultrasound.
Management
Urgent admission - this is an emergency and often requires urgent open surgery or angioplasty. Objective
sensory loss requires urgent treatment. Heparinization is needed immediately (this may double the limb
salvage rate), and provide analgesia.
The limb must be checked for evidence of compartment syndrome and, if necessary, a fasciotomy should be
performed.
If the occlusion is embolic, the options are surgical embolectomy (Fogarty balloon embolectomy catheter) or
local intra-arterial thrombolysis:

If embolectomy with a Fogarty catheter fails, an on-table angiogram is performed and bypass graft or
intraoperative thrombolysis considered. Routine intraoperative angiography for arterial thromboembolectomy
has been shown to be beneficial.
After successful embolectomy, anticoagulation with heparin is needed to prevent recurrence. Many surgeons
postpone heparin for six hours after surgery to reduce the risk of a haematoma.
If the occlusion is due to thrombotic disease the options are intra-arterial thrombolysis, angioplasty or bypass
surgery. If due to thrombosis of an arterial graft, then thrombolysis is the first step.
For patients with acute arterial emboli or thrombosis, treatment with immediate systemic anticoagulation
with unfractionated heparin has been recommended. This should be followed by long-term warfarin in
patients with embolism.
An arteriogram is performed and the catheter advanced into the thrombus. Streptokinase, urokinase or
tissue plasminogen activator (tPA) should be combined with heparin and thrombolysis continued for 48 hours
or until clot lysis.
Fibrinolysis usually takes between 6 and 72 hours to achieve clot lysis and so patients with limb-threatening
ischaemia are not candidates for local fibrinolysis, and require emergent embolectomy. Local thrombolytic
therapy is therefore reserved for patients with non-life-threatening limb ischaemia.
If a limb is irreversibly ischaemic, amputation will be required.

1606. A 50yo woman complains of several months hx of weakness and difficulty climbing stairs. Exam:
fissuring of the skin of her hands. CXR: pulmonary fibrosis. What is the single most likely positive antibody?
a. Anti Jo1
b. Anti Scl 70
c. Anti Ro
d. Anti dsDNA
e. Anti centromere
Key: Anti Jo1 (A)
Reason: Anticentromere would be present in limited scleroderma and Anti-Scl 70 would be present in diffuse
scleroderma. Anti-DSDNA would be positive in SLE, along with Anti-Ro. Anti-Ro is also positive in Sjogrens
syndrome and scleroderma. This is a case of Polymyositis because none of the others would have the muscular
weakness that is present in this case. The antibody of choice would be Anti Jo1 antibody.

1607. A 65yo woman complaining of symptoms suggestive of Raynauds phenomenon and difficulty in
swallowing. Exam: painful lesions on her finger tips and facial telangiectasis. What is the single most likely
positive antibody?
a. Anti Jo1
b. Anti Scl 70
c. Anti Ro
d. Anti ds DNA
e. Anti centromere
Key: Anti-Centromere (E)
Reason: Anti-centromere antibody would be present in CREST syndrome which is also called Limited
scleroderma. All the others are already accounted for and would be present in other connective tissue
diseases.

1608. A 6yo boy presented about 4h ago with acute severe pain on the testis with the left half slightly higher
than the right. Pain was not relieved by any strong analgesic. What is the initial
management?
a. Give strong analgesic
b. IV NS and monitor vital signs
c. Reassure
d. Immediate surgical referral
e. Cover with antibiotics
Key: Immediate surgical referral (D)
Reason: This is a case of possibly testicular torsion, if in any doubt, refer immediately to surgery/urology.
Testicular torsion presents with inflammation of one testes which is tender, hot and swollen. It may lie high
and/or transversely. Analgesics, IV fluids and antibiotics may have a role but none of those things would be
initial management. Immediate consent is sought for orchidectomy but first it is tried to unwind the testicle to
avoid surgery. Reassurance is an asinine choice.

1609. A 60yo man is brought to the ED in an agitated state. He is lashing out violently. Which drug in low
dosage due to its relative lack of autonomic side effects is a drug of choice in the tx of agitation in this pt?
a. Haloperidol
b. Diazepam
c. Fluoxetine
d. Clozapine
e. Chlorpromazine
Key: Haloperidol (A)
Reason: Acute psychosiss drug of choice is always Haloperidol. Diazepam would sedate the patient but would
not take care of his psychotic symptoms, fluoxetine is an SSRI and would take 2 weeks to act, clozapine and
chlorpromazine are not drugs of choice in psychotic episodes due to their adverse effects.
1610. A 32yo woman of 40wks gestation attends the antenatal day unit with sudden onset epigastric pain with
nausea and vomiting. She is clinically jaundiced. Her biochemistry results show a raised bilirubin, abnormal
liver enzymes, high uric acid and hypoglycemia. Whats the most likely dx?
a. Acute fatty liver of pregnancy
b. Obstetric cholestasis
c. Cholecystitis
d. HELLP syndrome
e. Acute hepatitis
Key: Acute Fatty Liver of Pregnancy (AFLP) (A)
Reason: Pain, nausea, vomiting, jaundice, fever with elevated liver enzymes and bilirubin is clinically indicative
of AFLP. Also can have elevated INR, TLC and hypoglycaemia. It isnt cholestasis because there is no pruritis,
cholecystitis isnt the answer because there is no history of evidence of gallstones, HELLP isnt the answer
because there isnt any hemolysis or thrombocytopenia and acute hepatitis would present subclinically or with
very less symptoms like diarrhoea and vomiting alongwith clinical history of food poisoning.
Discussion:
Epidemiology:
* It is a rare condition with an incidence of 5 in 100,000 pregnancies.
* Acute fatty liver of pregnancy (AFLP) tends to occur in late pregnancy.
* Risk factors include first pregnancies, pre-eclampsia, twin pregnancies and male fetuses.
* It may be associated with a mutant gene producing a defect in mitochondrial fatty acid oxidation and infants
born to mothers with AFLP should be screened for defects in this system.
Presentation:
* This usually presents acutely with nausea, vomiting and abdominal pain, fevers, headache and pruritus,
beginning typically at about 35 weeks of gestation but can occur much earlier. It may also appear immediately
after delivery.
* Jaundice appears soon after onset of symptoms and can become intense in a large proportion of patients.
Fulminant liver failure may follow.
Investigations:
* The white cell count is often elevated. There may also be neutrophilia and thrombocytopenia.
* Liver transaminases are moderately high.
* Raised serum bilirubin.
* Abnormal clotting with coagulopathy (prolongation of prothrombin and partial thromboplastin times with
depression of fibrinogen levels).
* Biopsy would be diagnostic but coagulation problems often preclude it. CT/MRI scanning may show reduced
attenuation in the liver.
Management:
Consider early delivery, as the condition usually resolves afterwards with complete recovery. Supportive ITU
care is frequently required.
Complications:
AFLP is a life-threatening condition with a reported 1.8% maternal and 23% fetal mortality rate. Serious
complications include: * Disseminated intravascular coagulation (DIC) and gastrointestinal bleeding.
* Hepatic coma. * Acute kidney injury. * Pancreatitis. * Hypoglycaemia.

1611. A 24yo man believes his bowels are blocked and his life is in ruin. What kind of delusion is he suffering
from?
a. Persecutory
b. Factitious
c. Guilt
d. Nihilistic
e. Hypochondriacal
Key: Nihilistic (D)
Reason: The man believes his bowels are blocked and his life is ruined. This is a presentation of nihilistic
delusions whereupon the individual believes his life or his organs are useless and leading to his death,
destruction or ruin. Persecutory delusions involve the belief that people are out to get him or her and
factitious delusions are when the person himself is lying, guilt is an emotion and requires a history of inciting
factor while hypochondriacal delusions are delusions that are disease oriented and targeted to receiving
medicines or care from a doctor.
1612. A 75yo man with declining vision, cornea and pupils are normal, fundus shows obscured margins. What
is the single most likely dx?
a. Macular degeneration
b. HTN retinopathy
c. MS
d. DM background
e. Proliferative DM retinopathy
Key: Macular degeneration (A)
Reason: The age of the patient, normal opthalmological examination and obscured margins of the fundus all
point towards age related macular degeneration. HTN would have other findings on the ocular exam, MS
presents with optic neuritis and RAPD +ve with red colour blindness developing, DM and proliferative DM
would present with other fundus findings according to degree and stage of diabetic retinopathy.

1613. A man under psychiatric tx develops GI distress and tremors. Which drug is most likely to cause these
symptoms?
a. Lithium
b. Diazepam
c. Citalopram
d. Clozapine
e. Imipramine
Key: Lithium (A)
Reason: Lithium causes these symptoms along with Diabetes insipidus.

1614. A 24yo man presents with painless hematuria. No other complaint and no abnormality is found on
physical exam. What is the most appropriate initial inv which is helpful to get a dx?
a. Coag screening
b. MSU
c. Cystoscopy
d. MRI spine
e. Abdominal US
Key: Abdominal USG (E)
Reason: Painless hematuria in a young male without any other findings on history or examination is likely
Polycystic Kidney Disease which frequently presents with painless gross hematuria. Coagulation screening in a
24y old man is only helpful with a history of blood dyscrasias, mid stream urine will not help us in establishing
a diagnosis in this case, Cystoscopy is used to visualize the urinary tract, better to perform in older age group
to rule out CA and MRI spine will not help us at all.
1615. A 29yo woman presents to her GP with troublesome heavy periods. The med tx that she has tried have
made little difference. She is known to have large uterine intramural fibroids. You
confirm that she is currently trying for more children. Select the most appropriate management
for menorrhagia in this pt?
a. Danazol
b. Endometrial ablation
c. Hysterectomy
d. Hysteroscopic resection of fibroids
e. Myomectomy
Key: Myomectomy (E)
Reason: She is currently trying for more children, hysterectomy will not be useful in this case obviously. OHCS
pg. 276 states that chance of subsequent pregnancies is better after myomectomy and it is the best treatment
in this case. Endometrial ablation will not affect the fibroids and danazol causes a post-menopausal state
which would not help her in conceiving.

1616. A 30yo schizophrenic female attacks her mother believing that aliens have replaced her with an exact
double. What condition is she suffering from?
a. Capgras syndrome
b. Ganser syndrome
c. Todd syndrome
d. Fregoli syndrome
e. Cotard syndrome
Key: Capgras Syndrome (A)
Reason: Capgras syndrome is an irrational belief that a familiar person or place has been replaced by a
duplicate.
Ganser syndrome is a fictitious disorder in which a patient deliberately acts as if he has a physical or mental
illness when he doesnt have it.
Todd syndrome/Alice In Wonderland syndrome/Lilliputian syndrome is a disorienting neurological condition
affecting human perception of size, shape and time.
Fregoli syndrome is a delusion of doubles, a delusional belief that different ppl are infact a single person in
disguise or change appearance.
Cotards syndrome/Nihilistic delusions is walking corpse syndrome, the person think they are dead or that
one of their organs has stopped functioning.

1617. A 38yo man has just returned from a holiday where he went swimming everyday. For the last few days
he has had irritation in both ears. Now his right ear is hot, red, swollen and acutely
painful. What is the single most likely dx?
a. Foreign body
b. Impacted earwax
c. OE
d. OM
e. Perforation of eardrum
Key: Otitis Externa (C)
Reason: The swimming history, irritation in both ears and ear being hot, red, swollen and painful indicates
inflammation of the external acoustic meatus called Otitis Externa. It isnt otitis media because of the lack of
Tympanic membrane signs, perforated eardrum would present with just pain and deafness, impacted earwax
would also present with pain and conductive deafness. Foreign body would have history of something being
used near or inside the ear and would be seen on examination of the ear canal.

1618. A healthy 2yo boy is brought to the ED having cut his hand playing in the garden. He has a 2cm clean
laceration. He has not received any routine immunizations as his parents are concerned about possible side
effects. There are no contraindications to immunizations. What is the single most appropriate follow up inv?
a. Courses of DPT vaccine
b. Courses of DT
c. Single inj of DPT vaccine
d. Single inj of DT
e. Single inj of tetanus Ig
Key: Courses of DPT Vaccine (A)
Reason: The child is unimmunized and has no contraindication to vaccination. Keeping in mind his age, single
injections would be useless and would not help the patient. Courses of DPT vaccine would be the best choice
in this case.

1619. A 6wk child has hx of frequent vomiting which became worse during the last weeks. He has no fever,
recently he has passed stool only once every 2-3d. What inv will you do to confirm the dx?
a. Abdominal US
b. Barium meal
c. Erect XR abdomen
d. Feed test
e. Reassure
Key: Abdominal USG (A)
Reason: Barium studies in this case are always the wrong answer, Abdominal USG is going to give you a
definitive diagnosis as soon as is realistically possible. The age, symptoms point towards pyloric stenosis as the
likely cause, X-ray Abdomen will not show pyloric stenosis at all, Feed test is a useless answer and reassuring
the patients parents will not work because this is something that needs intervention, possibly surgically by
Ramstedts pyloromyotomy.

1620. A 30yo woman had an IUCD inserted 8-9m ago. Now on routine follow up the thread is missing. Uterine
US showed no IUCD in the uterus. What is the best management?
a. Laparoscopy
b. Pelvic CT
c. Laparotomy
d. Pelvic XR
Key: Pelvic XR (D)
Reason: Thread is missing and it isnt seen in the uterus, Xray is the logical choice. Laparotomy and
laparoscopy are not needed unless it perforates an organ which is highly unlikely. USG is the first choice which
has been performed, Xray after that to check location and then advice surgical retrieval if needed.

1621. A pt comes with weight loss and sleep disturbance has mild depression. He has a hx of MI. What is the
single most appropriate tx?
a. Diazepam
b. ECT
c. Imipramine
d. Lithium
e. Antipsychotics
Key: Diazepam
Reason: Diazepam is the mildest option available here, ECT is very unnecessary, Imipramine and other TCAs
are severely contraindicated with a history of MI, Lithium causes MI as an adverse effect and antipsychotics
are used for psychosis.

1622. A pt. comes back from India and presents with night sweats and lymphadenopathy. XR:
Cavitation. What investigation should be done next?
a. CT scan
b. AFB stain
c. Blood culture
d. Bronchoscopy
Key: AFB Stain (B)
Reason: The symptoms and arrival from an endemic area for Pulmonary TB suggests the best course of action
would be to go for AFB staining via ZN stain. CT Scan would not help in initial diagnosis, blood culture isnt first
line for TB, sputum culture clearly is and bronchoscopy is unnecessarily invasive.

1623. A 45yo woman has been extensively investigation for a lump she believes to be cancer. She doesnt
think doctors take her seriously and demands another referral. What term best describes her condition?
a. Munchausen syndrome
b. Munchausens by proxy
c. Hypochondriasis
d. Malingering
e. Phobia
Key: Hypochondriasis (C)
Reason: This patient, with her history of extensive investigations or her fat folder syndrome warrant nothing
other than the label of being a Hypochondriac who wants treatment for imaginary illnesses she has even after
being counselled about the severity of her illness.
Muchausens syndrome describes a patient who lies vividly, is addicted to institutions and goes from hospital
to hospital feigning illnesses hoping for laparotomy or mastectomies.
Munchausens by proxy defines injury to a dependent person by carer to gain medical attention. Malingering
is the creation of a fictitious illness without even the lump which is present in this case. Phobia is a fear of
something irrational.

1624. A 15yo man presents with bitemporal hemianopia and spade-like hands. What is the definite test to
confirm the dx?
a. Early morning growth hormone
b. Insulin tolerance test
c. OGTT with growth hormone measurements
d. Random insulin-like growth factor (IGF-1)
e. Short ACTH test
Key: OGTT with growth hormone measurements (C)
Reason: The bitemporal hemianopia and spade-like hands point towards this being acromegaly. The best initial
test is insulin like growth factors but the definitive test that confirms the diagnosis is OGTT with serial growth
hormone measurements. Early morning growth hormone would be raised anyway, it is released in a pulsatile
manner, Short ACTH is used for Cushings disease and Insulin tolerance test is not used for growth hormone or
acromegaly assessment.
Discussion:
How is acromegaly diagnosed?
* A blood test can measure the level of growth hormone. However, a single test is not reliable. This is because
the levels of growth hormone in the body fluctuate a lot throughout the day in all people. * The diagnosis of
acromegaly is made by a glucose tolerance test. In this test you drink a sugar drink containing 75 grams of
glucose. You then have a series of blood tests over two hours. The glucose should lower the blood level of
growth hormone. However, if you have acromegaly, the growth hormone level remains high.
* A blood test to measure the level of IGF-1 (see above) may be measured if acromegaly is suspected. This
may also be used as an index of disease activity to assess how well treatment is working. * A magnetic
resonance imaging (MRI) scan can show the size of any tumour.
* Eye and visual tests can assess if the tumour is pressing on the optic nerve.
* If you are confirmed as having acromegaly, other tests will be needed to see if the tumour is causing a lack
or excess of other hormones made by the pituitary. * Other tests may include chest X-ray, electrocardiogram
(ECG) and X-rays of some of your joints.
What are the treatments for acromegaly?
The aim of treatment is to reduce the level of growth hormone in the blood to normal, and to reduce the size
of an enlarged tumour. Many of the symptoms and features of acromegaly will reverse or improve with
successful treatment (apart from any fixed extra bone growth that had occurred).
Surgical treatment
The most common treatment is to remove the adenoma by surgery. This is done using very fine instruments.
There are two different ways to operate on the pituitary gland:
o The first method is known as endonasal trans-sphenoidal surgery. This involves the surgeon reaching your
pituitary gland through a small cut (incision) in wall of one of your nostrils. In the other way, the surgeon
approaches the pituitary gland through a small incision behind your upper lip, just above your front teeth. The
instruments are passed through the base of your skull - the sphenoid bone. The aim is to remove the
adenoma, but to leave the rest of the pituitary gland intact.
o The operation is successful, with no further treatment needed, in around 9 out of 10 cases with smaller
tumours. The operation is less successful in those with larger tumours. However, sometimes it is not possible
to remove all the cells of the tumour. If not all is removed and your growth hormone level remains high
following surgery, other treatments listed below are likely to work.
o Your surgeon will advise on the possible complications which can sometimes occur. For example, sometimes
the operation may damage some other parts of the pituitary gland. This may cause a reduced production of
some other hormones. If this occurs, you will need to take replacement hormone therapy.
Medication
Medication can be used if surgery is not possible, or not wanted. It is also used whilst waiting for surgery or
radiotherapy. It can also be used in cases where surgery fails to remove the tumour totally and the level of
growth hormone remains high.
* Somatostatin analogues (octreotide and lanreotide) reduce the level of growth hormone to normal in over
half of cases, and reduce the size of the tumour in about 8 in 10 cases. However, these medicines need to be
given as an injection. They work in a similar way to somatostatin (described above) which is a hormone that
prevents growth hormone from being released from pituitary cells. These medicines used to be injected
several times a day. However, longer-acting preparations are now available as monthly or fortnightly
injections. Side-effects are not common with these medicines. Some people develop tummy (abdominal) pains
and diarrhoea, but these usually wear off with time. Gallstones can also occur but rarely cause problems. *
Dopamine agonists (such as cabergoline, bromocriptine and quinagolide) can be taken as tablets. They work
by preventing the release of growth hormone from tumour cells. However, they only work well in about 1 in 5
cases. Side-effects such as feeling sick and dizzy are also quite common.
* Pegvisomant (Somavert) is taken as a daily injection. However, unlike the other medicines listed above, it
does not act directly at the pituitary. Pegvisomant works by blocking the action of growth hormone on your
body's cells. Therefore, although many of the symptoms of growth hormone excess will be eased, it does not
reduce the size of the tumour and headaches are not eased.
* Radiotherapy * Radiotherapy is an option to reduce the size of the tumour and hence reduce the production
of growth hormone. Radiotherapy focuses high-intensity radiation at your pituitary tumour to destroy the
abnormal cells. It may be used if you are not able to have surgery, or if surgery was only partially successful.
However, it can take months or years after the radiotherapy is given for the level of growth hormone to
reduce to normal. You can take medication whilst waiting for the effects of radiotherapy to work.
A possible side-effect of pituitary radiotherapy is damage to other normal pituitary cells. This can cause a
reduced level of some other hormones. However, if this occurs you can take replacement hormone therapy.
Acromegaly and bowel cancer screening
As mentioned earlier, people with acromegaly have an increased chance of developing bowel (colonic) polyps
and bowel cancer. Therefore, if you are diagnosed with acromegaly and are aged 40 or more you will normally
be offered a routine colonoscopy every 3-5 years. A colonoscopy is a test where an operator (a doctor or
nurse) looks into your large bowel (colon) with a flexible telescope. It can diagnose bowel problems such as
polyps and bowel cancer. The aim is to detect those people who develop cancer as early as possible (before
symptoms develop) when the chance of a complete cure is high.However, you should always tell you doctor if
you develop any new symptoms from your bowel such as persistent diarrhoea, passing mucus, passing blood,
or tummy (abdominal) pain.

1625. A 22yo man has had an acute, painful, red right eye with blurring of vision for one day. He had a similar
episode 1y ago and has had episodic back pain and stiffness relieved by exercise and diclofenac for four years.
What is the SINGLE most likely cause of his red eye?
a. Chorioretinitis
b. Conjunctivitis
c. Episcleritis
d. Iritis
e. Keratitis
Ans)D
Key)The symptoms described are characteristic of ankylosing spondylitis.(lower back pain and stiffness which
gets better after moving around and taking NSAIDS) the extra articular manifestations of AS are uveitis.
Symptoms:
Anterior uveitis:eye pain,photophobia,redness,visual loss,pupil shape changes,symptoms develop in few
hours or days,maybe single eye is affected
Intermediate uveitis:painless blurred vision,FLOATERS,both eyes normally affected
Posterior uveitis:painless blurred vision,severe vision loss,FLOATERS and scotomas, take longer to develop.
the stem of the question talks about the symptoms of anterior uvitis, hence the answer is iritis. (chorioretinitis
is excluded as it comes under the symptoms of posterior uvitis)
in episcleritis, the sclera may look blue and visual acquity is NORMAL. conjunctivitis will present with similar
symptoms as well but there will be a discharge and the question will talk about eyes sticking together.Keratitis
will be identified with a white patch on the cornea.
Treatment of uvietis
1. Steroid eye drops
2. cycloplegic eye drops to relieve pain
3. dark sun glasses
4. painkillers

Reason: These symptoms point towards Ankylosing Spondylitis with the episodic back pain that is relieved by
exercise and NSAIDs for four years. The presentation is that of Uveitis which is associated with autoimmune
diseases, specifically Iritis with acutely painful red eye. Chorioretinitis would present in the same way, but 1/3
or AS patients have Iritis. Conjunctivitis and Episcleritis would not have blurring of vision and Keratitis wouldnt
be associated with systemic symptoms.

1626. A 40yo divorced man with bipolar affective disorder attends hospital following an OD of 30 TCA tablets.
His new partner has left him and he has stopped taking his medicine and begun drinking heavily. He appears
depressed, feels hopeless and is ambivalent about being alive. He is now fit for discharge from the medical
ward and acknowledges the benefits of previous tx. What is the SINGLE most appropriate next management?
a. Admission to the psychiatry ward
b. Arrange psychiatric outpatient follow-up
c. Discharge to the care of the general practitioner
d. Referral to local alcohol treatment team
e. Referral to clinical psychologist
Ans)A
Key) the trigger (partner leaving) precipitated an acute attack of maniac depression in the patient. there is no
other option BUT for him to be admitted in a psychiatric unit with his actions being constantly observed.the
stem of the question says he stopped taking his medication. abrupt cessation of Lithium precipitates mania in
50% of the patients as well and people with mania arent likely to adhere to their medications in the first place
either.Discharging the patient with an SSRI/antidepressant isnt a very good move since SSRIs take at least 2-4
weeks for their effect to build up fully. therefore, proving that the best possible answer to the question is still
A.
Reason: The patient is high risk on the suicide risk assessment scale and doesnt warrant out-patient follow up
or discharge to be seen by a GP. He should be admitted and treated in the Psychiatry ward as needed. Referral
to the local alcohol treatment team or clinical psychologist is never the right answer.

1627. A healthy baby boy is born at term to a woman who was unwell with confirmed acute hep B
during pregnancy. The mother is very concerned that she may have infected the baby with hep
B. What SINGLE preventative intervention should be given to the baby?
a. Full course of hepatitis B vaccine
b. Hepatitis B immunoglobulin alone
c. Hepatitis B vaccine and hepatitis B immunoglobulin
d. Hepatitis B vaccine as single dose
e. None until hepatitis B status confirmed
Ans)C
Key)Babies born to mothers infected with hepatitis B have a high risk of acquiring infection, which can be
prevented by vaccination at birth.All babies with seropositive mothers should have the full primary course of
hepatitis B immunisation and most should also have HBIG within 24 hours of birth.

Reason: If the mother has active confirmed acute Hep B, give Immunoglobulins and vaccinate the baby at
birth. Also perform serology of the baby at 12-15 months (1 year). If Hep B Antibodies +ve and HbSAg -ve, do
nothing. Full course of the vaccine is unnecessary because the aforementioned course will provide sufficient
coverage, Ig alone and Vaccine alone will not be enough for the baby. Full course with immunoglobulin will be
given at BIRTH, 1 MONTH, 2
MONTHS. (E) isnt a good option because the baby needs immediate coverage in case the infection has been
transmitted at delivery.

1628. A previously well 15yo girl had an acute onset of fever, sweating, bruising and petechiae. A
blood count showed: Hgb=63g/L, WBC=1.1mg/L, Neutrophils=0.1, plt=14. No abnormal white
cells were seen on the blood film. She was transfused and given IV antibiotics and her condition
improved. 3wks later her blood count has returned to a similar picture. What is the SINGLE most
likely underlying dx?
a. ALL
b. AML
c. Aplastic anemia
d. CML
e. Pernicious anemia

Key: Aplastic Anaemia (C)


Reason: The age of the patient and pancytopenic picture give us a clinical diagnosis of Aplastic anemia. Normal
WBC morphology rules out ALL, AML and pernicious anaemia while the age rules out CML as a diagnosis.

1629. An 83yo woman admitted with a chest infection becomes confused with impaired attention and poor
concentration. She is restless and frightened. She is verbally abusive and has perceptual
abnormalities. There is no significant prv psychiatric hx. What is the SINGLE most likely dx?
a. Delirium
b. Drug induced psychosis
c. Lewy body dementia
d. Multi-infarct dementia
e. Psychotic depression

Key: Delirium (A)


Reason: Drug induced psychosis would require a drug history, dementia and psychotic depression would have
a significant previous psychiatric history. Delirium or Acute Confusional States happen in the elderly in
response to stressors like acute infections and this is most likely brought on by the chest infection that has
developed.

1630. A town has a population of 500,000. In a five year period there are 1250 cases of bladder cancer
diagnosed at the only hospital. During the same period the occupational health department
diagnosed a further 500 cases. What is the annual incidence per million of bladder cancer in this
population?
a. 2100
b. 1750
c. 1400
d. 700
e. 350

Key: 700 (D)


Reason: There are 1750 cases a 5 year period in this scenario. The number of cases in 1 year in the 0.5 million
people in this town are 350 cases/0.5 million people. To calculate the incidence per million of bladder cancer,
we would simply double the number to get an estimate which would give us 350 x 2 = 700 cases/million.
1631. A 28yo woman who has had a prv pulmonary embolism in pregnancy wishes to discuss
contraception. She has menorrhagia but is otherwise well. What is the SINGLE most suitable
contraceptive method for this patient?
a. COCP
b. Copper IUCD
c. Levonorgestrel intra-uterine system
d. Progestogen implant
e. POP

Key: Levonorgestrel Intra-Uterine System (C)


Reason: The woman has a history of thromboembolic disease, which essentially rules out COCPs. POPs,
Copper IUCD (Copper T) and Progestogen implants would not help the menorrhagia. LNG-IUS (Mirena) is the
hormone releasing device that is most suitable in this patient and would be the contraceptive of choice.

1632. An 8yo girl has had left earache for 2d. The earache subsided about 2h ago with the onset of a purulent
discharge which relieved the pain. Her temperature is 39.2C. What is the SINGLE most appropriate antibiotic?
a. Amoxicillin
b. Ciprofloxacin
c. Clindamycin
d. Erythromycin
e. Flucloxacillin

Key: Amoxicillin (A)


Reason: This is the picture of Acute Otitis Media which has led to tympanic membrane perforation. PO
Amoxicillin for 7 days is the treatment of choice with appropriate analgesics. Amoxicillin +/- Clavulanate is the
first line drug because it fits the organisms responsible which are Strep pneumonia, Moraxella catarrhalis and
H. influenza.

1633. A 38yo man has disturbing thoughts about his house being infected by germs. He is anxious
about safety and checks the locks of his doors repeatedly before going to bed. For the last 8wks
he has been washing his hands every time he touches the lock, 20-30 times a day. What is the
SINGLE most appropriate management?
a. Antidepressant
b. Antipsychotic
c. Anxiolytic
d. CBT
e. Psychodynamic psychotherapy

Key: CBT (D)


Reason: This scenario describes a case of OCD for which the best management is CBT followed by SSRIs or
TCAs. The first line treatment is always CBT, not pharmacological therapy. Psychotherapy is indicated in
depression, psychosomatic disorders, dissociative or conversion disorders, personality disorders, relationship
problems or grief.
1634. A 65yo man had closure of colostomy performed 5d ago. He is not systemically unwell. There is
a tender, localised fluctuant swelling 4 cm in diameter in the wound. What is the SINGLE most
appropriate management?
a. Abdominal support
b. Antibiotics
c. Laparotomy and re-suture wound
d. Local exploration of wound
e. Observation

Key: Local exploration of wound (D)


Reason: The patient is not systemically unwell, he has had colostomy performed 5 days back. The swelling is
tender, localised and fluctuant. Local exploration with I&D if needed is the single most appropriate
management option. Abdominal support is unnecessary, so are antibiotics if the patient is systemically well.
Laparotomy is indicated if there is a bleed or some surgically correctable issue going on, observation is not the
right answer here because intervention is warranted.

1635. A 32yo woman has had a febrile illness and swelling of the small joints of her hands, feet, wrists and
knees for two days. She has a maculopapular rash and a few palpable, small cervical lymph nodes. She was
previously well. There is no history of relevant travel outside the UK. She has two young children. What is the
SINGLE most likely dx?
a. Psoriasis
b. Reactive arthritis
c. Rheumatoid arthritis
d. Sarcoidosis
e. SLE
key)B
Ans)
Reactive arthritis is caused when a joint reacts to an infection elsewhere in the body. The infection which
triggers reactive arthritis is not actually in the joint, but is usually in the gut (gastroentritis) or urethra
(uretheritis).it can also occur after viral infections causing sore throat skin rash etc.

Fever is not usually a feature of the other diseases mentioned here.

1636. A 16yo girl has had an enlarging mass in the right side of her neck for the last 6wks. She has had
no other symptoms. She has a 2 x 2 cm enlarged LN in the anterior triangle of the neck with
several smaller associated LN palpable. Oropharyngeal examination shows tonsillar membranes.
What is the SINGLE most likely dx?
a. Infectious mononucleosis
b. Leukaemia
c. Lymphoma
d. Sarcoidosis
e. Tuberculosis
key)A
Ans)infectious mononucleosis is seen primarily in young teenagers and college going students (16 year old
girl). the rest of the options are excluded because there is no mention of weight loss(all other 4 options), chest
involvement (sarcoidosis,TB), skin involvement (leukemia=bruises,sarcoidosis=skin rash), night sweats
(TB,lymphoma)
symptoms of infectious mononucleosis:
Sore throat; tonsillar enlargement is common, classically exudative and may be massive
Fine macular non-pruritic rash, which rapidly disappears.
Lymphadenopathy, especially neck glands.
Arthralgias and myalgias low grade fever
Later signs include:
o Mild hepatomegaly and splenomegaly
o Jaundice

investigation:detection of heterophile antibodies by the monospot test or the paul bunnell test

EBV is also associated with:

Burkitt's lymphoma.
B-cell lymphomas in patients with immunosuppression.
Undifferentiated carcinomas - eg, cancer of the nasopharynx and cancer of the salivary glands.

treatment is supportive.

1635. A 32yo woman has had a febrile illness and swelling of the small joints of her hands, feet, wrists
and knees for two days. She has a maculopapular rash and a few palpable, small cervical lymph
Nodes. She was previously well. There is no history of relevant travel outside the UK. She has
two young children. What is the SINGLE most likely dx?
a. Psoriasis
b. Reactive arthritis
c. Rheumatoid arthritis
d. Sarcoidosis
e. SLE
Key is B: Reactive Arthritis
Patient is having reactive arthritis because of any underlying infection, most commonly urethral
infection.
Reactive arthritis is caused when a joint reacts to an infection elsewhere in the body. The infection
which triggers reactive arthritis is not actually in the joint, but is usually in the gut or urethra.
This is because they are most at risk of urethral infection from sexually transmitted diseases.
However, it can occur at any age and in anyone.
Treatment:
Antibiotics for infection
NSAIDs for joint pain
1636. A 16yo girl has had an enlarging mass in the right side of her neck for the last 6wks. She has had
no other symptoms. She has a 2 x 2 cm enlarged LN in the anterior triangle of the neck with
several smaller associated LN palpable. Oropharyngeal examination shows tonsillar membranes.
What is the SINGLE most likely dx?
a. Infectious mononucleosis
b. Leukaemia
c. Lymphoma
d. Sarcoidosis
e. Tuberculosis
Key is A: infectious Mononucleosis
Infectious mononucleosis (IM) is usually a self-limiting infection, most often caused by Epstein-Barr
virus (EBV)
The incubation period is between 1 and 2 months
Clinical infection is most common in populations with many young adults
Presentation:
Low-grade fever, fatigue and prolonged malaise.
Sore throat; tonsillar enlargement is common
Fine macular non-pruritic rash, which rapidly disappears.
Transient bilateral upper lid oedema.
Lymphadenopathy, especially neck glands.
1637. A 60yo man has had increasing pain in both buttocks, thighs and calves on walking for three
months. He has also recently developed impotence. Femoral and distal pulses are absent in both
limbs. What is the SINGLE most likely site of arterial obstruction?
a. Aorto iliac
b. External iliac
c. Femoropopliteal
d. Internal iliac
e. Tibial
Key is A: Aorto iliac
Page 718 OHCM 8th edition.
Patient is suufering from leriches syndrome.
Classic triad of:
Pain and claudication of buttock and thighs
Erectile dysfunction from aorto iliac occlusive disease
Absent femoral and distal pulse.
1638. A 78yo man has collapsed. He has had a severe headache for 12 hours and had an URTI 3d ago.
He has a temp=39.2C, pulse=122bpm, BP=84/60mmHg and RR=34bpm but his chest is clear. He
has a GCS=10 and some neck stiffness. He has been started on high-flow oxygen. What is the
SINGLE most appropriate immediate management?
a. IV antibiotic; CT brain scan
b. IV antibiotic; LP
c. IV fluids; CT brain scan
d. IV fluids; IV antibiotic
e. IV fluids; LP
Key is D: IV Fluids; IV antibiotics
As his BP is low and has presented with nech stiffness so, IV fluids and V antibiotics should be given, after
giving him oxygen.
1639. A 16yo boy was brought to hospital in a comatose state having taken methadone belonging to
his sister. He was given naloxone and rapidly became alert. Some hours later, he gradually
becomes semi-conscious again.What is the SINGLE most likely reason for this patient becoming
semi-conscious again in hospital?
a. Methadone hepatotoxicity has caused acute liver failure
b. Methadone is eliminated from the body more slowly than naloxone
c. Naloxone is a partial agonist at the central nervous system opioid receptor
d. The pt has misused another substance that has caused an intracranial bleed
e. The pt has misused another substance that is absorbed more slowly than methadone
Key is B: Methadone is eliminated from the body more slowly than naloxone
All other options are not favoring the scenario
1640. A 27yo woman who takes the COCP has had painless vaginal spotting and discharge for 3 days.
Her last menstrual period, which lasted four days, finished 10 days ago. Her last cervical smear
two years ago was normal. Abdominal and vaginal examinations are normal apart from a mild
ectropion with contact bleeding. What is the SINGLE most appropriate initial inv?
a. Cervical smear
b. Colposcopy
c. Endocervical swab
d. Endometrial biopsy
e. Pelvic US
Key is C: Endocervical swab
As her cervical smear and examination of abdomen and vagina are normal, next would be to exclude a STD
for which Endocervical swab is taken.
Chlamydia is an sexually transmitted infection caused by a germ (bacterium) calledChlamydia
trachomatis
About 1 in 20 sexually active women in the UK are infected with chlamydia. It is most common in
women aged under 25. (About 1 in 12 women aged 20 are infected with chlamydia.)
Presentation:
Vaginal discharge. This is due to the neck of the womb (cervix) becoming inflamed.
Pain or burning when you pass urine.
Vaginal bleeding or spotting between periods. In particular, bleeding after you have sex.
Pain or discomfort in the lower tummy (abdomen) area (the pelvic area)
Diagnosis
Chlamydia can be confirmed by a swab taken from the neck of the womb (cervix) in womenhave
sex
Treatment
A short course of an antibiotic medicine usually clears chlamydial infection
Note: antibiotics can interfere with the combined oral contraceptive pill (COCP). If you take the
COCP you should use alternative methods of contraception until seven days after finishing a course
of antibiotics
1641. A 72yo man being investigated for anaemia is booked for a colonoscopy in 24 hours. What is the
SINGLE most appropriate management the night before the procedure?
a. Bisacodyl tablets
b. Glycerine suppository
c. Lactulose syrup
d. Magnesium citrate (orally)
e. Senna tablets
Key is D: Magnesium Citrate Orally
Colonoscopy is a test to assess your colon (large intestine).
What preparation do I need to do?
The colon needs to be empty so that the operator can obtain a clear view. You will be instructed on
how to take a special diet for a few days before the test. You will also be given some laxatives to
take.(magnesium citrate is the least harmful)
You will need somebody to accompany you home, as you will be drowsy with the sedative
1642. A 19yo woman has had progressive bilateral iliac fossa pain and dyspareunia for 3days. She has
an offensive vaginal discharge and feels unwell and feverish. Her temp=39C. An initial
antimicrobial regimen is commenced. What SINGLE set of organisms are the most appropriate
for the antimicrobial regimen to cover?
a. Neisseria gonorrhoeae and Candida albicans
b. Neisseria gonorrhoeae and Candida albicans and Gardnerellavaginalia
c. Neisseria gonorrhoeae and Chlamydia trachomatis
d. Neisseria gonorrhoeae and Chlamydia trachomatis and Candida albicans
e. Neisseria gonorrhoeae and Chlamydia trachomatis and Gardnerellavaginalis
Key is C: Neisseria gonorrhoeae and Chlamydia trachomatis
Most common antibiotic given for STDs is azithromycin and it covers gram negative bacteria which
are neisseria and chlamydia.
Candida albicans is a fungal infection
Gardnerella is a gram variable bacteria
1643. A 48yo man with renal cancer had radiotherapy for metastatic spinal cord compression at the
11th thoracic vertebra 4wks ago. He has retained sensation but is unable to stand. He has pain
in a band around his lower trunk controlled by regular oral morphine. He is distressed by
increasingly frequent episodes of painful muscle spasms in his right leg. What is the SINGLE most
appropriate management of his symptoms?
a. Amitriptyline
b. Baclofen
c. Fentanyl patch
d. Gabapentin
e. Increase morphine dose
Key is B: Baclofen
1644. A 4yo girl has had a temp=38.5C for 2days and has not wanted to eat her food. Yesterday she
developed a sore throat and small, painful ulcers inside her mouth. Today she has small blisters
on the palms of her hands and soles of her feet which are painful but not itchy. What is the
SINGLE most likely underlying cause?
a. Coxsackie virus
b. Herpes simplex virus
c. Staphylococcus aureus
d. Streptococcus pneumonia
e. Varicella zoster virus
Key is A: coxsakie virus
Patient is suffering from HFMD (Hand,Foot and Mouth Disease)
HFMD is due to an infection that usually causes a typical illness, including a typical rash. It is most
commonly caused by the Coxsackie A16 virus
HFMD most commonly affects children under 10 years of age
This might include a high temperature (fever). After this, a sore throat commonly occurs, quickly
followed by small spots that develop inside the mouth. These soon progress into small mouth ulcers
In many cases, spots also develop on the skin. This is typically a day or so after the mouth ulcers
develop. The spots are small lumps that are a few millimetres in diameter and usually appear on the
hands and feet, they are not usually itchy but sometimes they can be a little bit sore.
Treatment:
There is no treatment that will take away the virus
1645. A 32yo woman has had 3 episodes of slurred speech and 2 episodes of transient weakness of
both legs in the past 5yrs. Each episode has resolved in 3m. What is the SINGLE most likely dx?
a. Meningioma
b. Migraine
c. Multiple sclerosis
d. Stroke
e. Transient ischaemic attack
Key is C: Multiple sclerosis
Points in favor: symptoms have a relapsing-remitting form
MS is thought to be an autoimmune disease
About 1 in 600 people in the UK develop MS. It can affect anyone at any age, although it is rare in
young children. It most commonly first develops around the age of 30. MS is the most common
disabling illness of young adults in the UK. It is twice as common in women as in men.
symptoms include:
Visual problems
The first symptom of MS for around one in four people with MS is a disturbance of vision. Inflammation
(swelling) of the optic nerve can occur. This is called optic neuritis. This can cause pain behind your eye and
also some loss of your vision. This usually only affects one eye.
Muscle spasms and spasticity
Tremors or spasms of some of your muscles may occur. This is usually due to damage to the nerves that
supply these muscles. Some muscles may shorten (contract) tightly and can then become stiff and harder to
use. This is called spasticity.
Pain
There are two main types of pain that may occur in people with MS:
Neuropathic pain
Musculoskeletal pain
Fatigue
Extreme tiredness (fatigue) is one of the most common symptoms of MS.
Other symptoms which may occur include:
Numbness or tingling in parts of the skin. This is the most common symptom of a first relapse.
Weakness or paralysis of some muscles. Mobility may be affected.
Problems with passing urine.
Inability to have an erection in men.
Difficulty with speaking.
Investigation and Treatment:
A firm diagnosis of MS is often not made until two or more relapses have occurred
A magnetic resonance imaging (MRI) scan of the brain is a useful test. This type of scan can detect
small areas of inflammation and scarring in the brain which occur in MS.
At present, although there is no cure for MS.
1646. An 8yo girl is complying with her asthma treatment of low-dose inhaled corticosteroid
prophylaxis and short-acting bronchodilators as required. Her inhaler technique is good. She
now has a frequent night cough and mild exercise-induced wheeze. What would be the SINGLE
most appropriate change in her treatment?
a. Add leukotriene antagonist
b. Add oral theophylline
c. Add regular long-acting bronchodilator
d. Increase dose of inhaled corticosteroid
e. Short course of oral corticosteroid
Key is D: Increase dose of inhaled corticosteroid
A common treatment plan for a typical person with moderate asthma is:
A preventer inhaler (usually a steroid inhaler), taken each morning and at bedtime. This usually
prevents symptoms throughout the day and night.
A reliever inhaler (short acting bronchodilators) may be needed now and then if breakthrough
symptoms occur. For example, if symptoms flare up when you have a cough or cold.
If exercise or sport causes symptoms then a dose of a reliever inhaler just before the exercise
usually prevents symptoms.
The dose of the preventer inhaler may need to be increased for a while if you have a cough or cold,
or during the hay fever season
1647. A 38yo man with longstanding alcohol dependence has vertigo and a tremor every morning.
What is the SINGLE most likely dx?
a. Anxiety
b. Benign positional vertigo
c. Cerebellar degeneration
d. Optic neuritis
e. Temporal lobe epilepsy
Key is C: Cerebellar degeneration
Cerebellar degeneration is a process in which neurons in the cerebellum - the area of the brain that
controls coordination and balance - deteriorate and die
Associated diseases:
ischemic or hemorrhagic stroke, when there is lack of blood flow or oxygen to the cerebellum
cerebellar cortical atrophy, multisystem atrophy, and olivopontocerebellar degeneration,
progressive degenerative disorders in which cerebellar degeneration is a key feature
Friedreichs ataxia, and other spinocerebellar ataxias, which are caused by inherited genetic
mutations that result in ongoing loss of neurons in the cerebellum, brain stem, and spinal cord
transmissible spongiform encephalopathies (such as Creutzfeldt-Jakob disease) in which abnormal
proteins cause inflammation in the brain, including the cerebellum
multiple sclerosis, in which damage to the insulating membrane (myelin) that wraps around and
protects nerve cells can involve the cerebellum
Other diseases that can cause cerebellar degeneration include:
chronic alcohol abuse that leads to temporary or permanent cerebellar damage
1648. An 84yo woman with Alzheimer's dementia has recently become incontinent and more
confused than usual. What is the SINGLE most likely dx?
a. Detrusor overactivity
b. Neuropathic bladder
c. Nocturnal enuresis
d. UTI
e. Uterine prolapse
Key is D: UTI
1649. A 4yo boy complains of pain around his right eye. He is unwell, febrile and also suffers from pain
on the right side of his face. What is the most probable dx?
a. Allergic reaction
b. Furuncle
c. Folliculitis
d. Foreign body
e. Periorbital cellulitis
Key is E: Periorbital cellulitis (also called preseptal cellulitis)
preseptal cellulitis occurs at younger ages (80% of patients are under 10 years of age and most are
younger than 5 with a mean age of 21 months)
Preseptal cellulitis:
Acute onset of swelling, redness, warmth and tenderness of the eyelid.
Fever, malaise, irritability in children.
Ptosis
Diagnosis: is usually made based on the clinical findings
Management:
Most children are initially admitted to hospital (even for preseptal cellulitis) unless there is good
reason not to. This may be just for 24 hours. Children should be considered to have orbital cellulitis
until proven otherwise (ie repeated examinations normal, good response to antibiotics in first 24
hours and normal CT scan).
Oral co-amoxiclav may be used for both adults and children as long as there is no allergy to
penicillin. Clinical improvement should occur over 24-48 hours.
1650. A pt presents with irregularly irregular pulse of 162bpm. What drug is most useful initially?
a. Amiodarone
b. Digoxin
c. Bisoprolol
d. Warfarin
e. Heparin
Key is C: Bisoprolol
Bisoprolol is Beta blocker used as antihypertensive and to control the heart rate aswell.
Bisoprolol slows down the activity of your heart by stopping messages sent by some nerves to your heart. It
does this by blocking tiny areas (called beta-adrenergic receptors) where the messages are received by your
heart. As a result, your heart beats more slowly and with less force.
1651. A 59yo man has shown a change in his mood and personality over a 9m period. He has
subsequently developed difficulty with memory and conc, and then progressive fidgety
movements of his limbs and facial musculature. By the time of medical assessment he has frank
choreiform movements and a mini-mental state exam of 21/30. Other exam is normal. He was
adopted and therefore no information on his famhx is available. He has 3 adult children (27, 30,
33) of whom the 2 youngest are asymptomatic. However, the oldest son has recently been inv
by the neurology dept for slightly erratic behavior and fidgety restless movements of both legs.
Based on the likely clinical dx, which one of the following genetic patterns is most likely?
a. AD inheritance with anticipation
b. AD with variable penetrance
c. AR
d. X-linked
e. Mitochondrial disorder
Key is A: AD inheritance with anticipation
Patient is suffering from Huntingtons disease and that is autosomal dominant with anticipation
which means a genetic disorder is passed on to the next generation, the symptoms of the genetic
disorder become apparent at an earlier age with each generation.
Huntingtons Disease:
It is an inherited (genetic) condition that affects the brain and nervous system. It can interfere with
movements of your body, can affect your reasoning, awareness, thinking and judgement (cognition)
and can lead to a change in your behavior
This faulty gene is carried on chromosome 4.
HD affects between 5-10 people per 100,000 in the UK.
Presentation
The symptoms of HD can be grouped into three main areas:
Problems with movement
Problems with cognition
Mood and behavioural problems
Treatment:
At present there is no cure for HD. Also, there is no treatment that has been found to delay the
onset of symptoms or to delay the progression of symptoms
1652. A 35yo pt has been dx with schizophrenia. He mimics the doctors and attendants doing the
same physical actions as them. What symptom does this pt have?
a. Echopraxia
b. Echolalia
c. Perseveration
d. Apraxia
e. Anosognosia
Key:A
Ans)Echopraxia is the involuntary repetition or imitation of another person's actions. Similar to echolalia,
which is the involuntary repetition of sounds and language. Echopraxia has long been recognized as a core
feature of Tourette syndrome, and is considered a complex tic, but it also occurs in autism spectrum disorders,
schizophrenia and catatonia.Apraxia is a motor disorder caused by damage to the brain (specifically the
posterior parietal cortex), in which someone has difficulty with the motor planning to perform tasks or
movements when asked, provided that the request or command is understood and he/she is willing to
perform the task. Apraxia is an acquired disorder of motor planning, but is not caused by incoordination,
sensory loss, or failure to comprehend simple commands (which can be tested by asking the person to
recognize the correct movement from a series)Anosognosia is a deficit of self-awareness, a condition in which
a person who suffers a certain disability seems unaware of the existence of his or her disability.

Key is A: Echopraxia
Echopraxia is involuntary imitation of the movements and is a feature for the diagnosis of schizophrenia.
Schizophrenia:
Schizophrenia is a serious mental health condition that causes disordered ideas, beliefs and
experiences. In a sense, people with schizophrenia lose touch with reality and do not know which
thoughts and experiences are true and real and which are not.
Schizophrenia develops in about 1 in 100 people. It can occur in men and women. The most
common ages for it first to develop are 15-25 in men and 25-35 in women.
Presentation:
Delusions
Hallucinations.
Disordered thoughts.

1653. A pt has loss of sensation on the tip of her tongue and the inner aspect of the lip. Which nerve is
most likely to be involved?
a. Vagus nerve
b. Glossopharyngeal nerve
c. Lingual nerve
d. Buccal nerve
e. Facial nerve
Key is C: Lingual Nerve
a. Vagus nerve: It leaves the skull through the jugular foramen, passes within the carotid sheath in the neck,
through the thorax supplying the lungs, and continues on via the oesophageal opening to supply the
abdominal organs.
b. Glossopharyngeal nerve: Passes across the posterior fossa, through the jugular foramen and into the
neck, supplying tonsil, palate and posterior third of tongue.

c. Lingual nerve: lingual nerve is a branch of the mandibular division of the trigeminal nerve (CN V3), which
supplies sensory innervation to thetongue. It also carries fibers from the facial nerve, which return taste
information from the anterior two thirds of the tongue, via thechorda tympani

d. Buccal nerve:brach of mandibular division of trigeminal nerve, supplies the skin and mucous membrane
of the cheek

e. Facial nerve: Mainly motor (some sensory fibres from external acoustic meatus, fibres controlling
salivation and taste fibres from the anterior tongue).
1654. A 51yo woman complains of difficulty swallowing and also reddish dots on her skin. A pic of her
hand is seen. What is the most appropriate term for the condition you would expect to see?
a. Sclerodactyly
b. RA
c. Swan neck deformity
d. Polydactyly
e. Ulnar deformity
Key is A: sclerodactyly
Patient is suffering from crest syndrome, sclerodactyly its one of the features.
(page 554 OHCM)
Limited systemic sclerosis(CREST syndrome is a part of which) Calcinosis(sub cu tissue), Raynauds
phenomenon, esophageal and gut dysmotility, Sclerodactyly and telengectesia. Skin involvement is limited
to the face, hand and feet. It is associated with anti-centromere antibodies in 70-80%. Pulmonary
hypertension is often present subclinically and become rapid life threatening.
1655. A 37yo female working as a healthcare assistant in a nursing home comes to the ED with
complaints of severe itching all over her body. On asking she replies that she had applied cream
on the body of a resident in the nursing home who had similar itches. What is the mechanism of
itching?
a. Allergic reaction
b. Inflammation of keratinocytes
c. Allergic reaction developed due to use of topical steroid creams
d. Subcutaneous bleeding
e. None
Key is A: allergic reaction
A severe allergic reaction (anaphylaxis) will affect the whole body; in susceptible individuals it may
develop within seconds or minutes of contact with the trigger factor and is potentially fatal.
Possible triggers can include skin or airborne contact with particular materials, the injection of a
specific drug, the sting of a certain insect or the ingestion of a food such as peanuts
1656. A 65yo pt who had MI 1yr ago now comes to the ED complaining that his neighbor is conspiring
against him. When his son is asked, he denies it and also narrates that sometimes his father says
that everybody in his office is always talking about him, which is not the case. What is the most
appropriate med?
a. TCA
b. Clozapine
c. Olanzapine
d. Lorazepam
Key is C: Olanzapine
TCA is not given at this age
Clozapine needs strict monitoring and can only be given under the supervision of a psychiatrist
Olanzapaine is antipsychotic which is most suitable in this case
1657. You suspect Cushing's disease in a 50yo woman who has attended clinic with glycosuria, HTN
and a suggestive body habitus. Initial inv point you towards a dx of Cushing's disease. Which of
the following findings would be against this dx?
a. A normal 8am cortisol
b. Failure to suppress morning cortisol with dexamethasone
c. HTN requiring >2 antihypertensive agents
d. Impaired growth hormone response to glucose loading
e. Unilateral adrenal enlargement
Key is E: Unilateral adrenal enlargement
Cushings disease
Bilateral adrenal hyperplasia from an ACTH secreting pituitary adenoma.
Peak age 30-50 years, male female ratio 1:1
A low dose dexamethasone test leads to no change in plasma cortisol but 8 mg may be enough
1658. Which finding, on clinical examination of the pulse, suggests a diagnosis of hypertrophic
obstructive cardiomyopathy (HOCM)?
a. Irregularly irregular pulse suggesting A-fib
b. Pulsusalternans
c. Pulsusbigeminus
d. Pulsusbisferiens
e. Pulsusparadoxus
Key is C Pulsusbisferiens
pulsusbisferiens, is a sign where, onpalpation of the pulse, a double peak per cardiac cycle can be
appreciated. Bisferious means striking twice. Classically, it is detected when aortic insufficiency exists in
association with aortic stenosis,[1] but may also be found hypertrophic obstructive cardiomyopathy.
Hypertrophic Cardiomyopathy:
Hypertrophic cardiomyopathy (HCM) is an autosomal dominant genetic disorder characterised by
left ventricular hypertrophy (LVH), impaired diastolic filling, and abnormalities of the mitral valve.
These features can cause dynamic obstruction of the left ventricular outflow tract, diastolic
dysfunction, myocardial ischaemia, and an increased risk of supraventricular and ventricular
tachyarrhythmias.
HCM is the most common genetic cardiovascular disease.
Epidemiology:The prevalence of HCM is about one in 500 and it tends to affect men and black
people more often. The obstructive form is seen in 25% of cases.
Hypertrophy can occur in any part of the left ventricle, although it is most common in the anterior
ventricular septum.
The presentation: is variable and includes dyspnoea (the most common presenting symptom),
chest pain, palpitations and syncope.
Examination: Classic examination findings are a forceful apex beat, with double
impulse(pulsusbisferiens) if the left ventricular outflow tract is obstructed and a late ejection systolic
murmur, which can be augmented by standing or Valsalvamanoeuvre and diminished by squatting. [2]
Investigation: Electrocardiogram (ECG): most patients have an abnormal ECG, although
electrocardiographic features are nonspecific and include LVH, ST segment changes and T-wave
inversion.
1659. A 60yo male is admitted with a 2d hx of lower abdominal pain and marked vomiting. On
examination he has abdominal swelling, guarding and numerous audible bowel sounds. What is
the likely dx?
a. Gallstone ileus
b. Ischemic colitis
c. Large bowel obstruction
d. Sigmoid volvulus
e. Small bowel obstruction
Key is D: sigmoid volvulus
History is not suggestive of ischemic colitis.
There are no bowel sounds heard in gallstone ileus, large bowel obstruction and small bowel obstruction,
hence it is sigmoid volvulus.
Sigmoid Volvulus:
In sigmoid volvulus, a large sigmoid loop full of faeces and distended with gas twists on its mesenteric
pedicle to create a closed-loop obstruction.
Risk factors:
The elderly.
Chronic constipation.
Megacolon, large redundant sigmoid colon and excessively mobile colon. [2]
It is more common in men.
Presentation:
lower abdominal pain associated with gross abdominal distension and a failure to pass either
flatus or stool.
Abdominal examination reveals a tympanitic, distended (but usually non-tender) abdomen and a
palpable mass may be present.
Investigation and Management:
Characterstic Abdominal X-Ray with an inverted U loop of bowel that looks a bit like a coffee bean.
Often managed by sigmoidoscopy and insertion of a flatus tube. Sigmoid colectomy is sometimes
required
1660. A 17-year-old boy is diagnosed with scabies. Which of the following statements regarding
scabies is correct?
a. Is best treated by salicylate emulsion
b. It can be spread by a droplet infection
c. It causes itchiness in the skin even where there is no obvious lesion to be seen
d. It is caused by Staphylococcus aureus
e. Typically affects the face
Key is C: It causes itchiness in the skin even where there is no obvious lesion to be seen
Excluded Points:
a. The usual scabies treatment is with permethrin cream. Permethrin is an insecticide that kills the mites
b. You need close skin-to-skin contact with an infected person to catch scabies. This is because the scabies
mite cannot jump or fly
d. Scabies is caused by a mite (like a tiny insect) called Sarcoptesscabiei. The mite is a parasite, meaning it
lives off the host (a human) with no benefit to the host
e. Itching is often severe and tends to be in one place at first (often the hands)
Scabies:
Scabies is caused by a mite (like a tiny insect) called Sarcoptesscabiei. The mite is a parasite,
meaning it lives off the host (a human) with no benefit to the host.
Scabies is common. In the UK, about 1 in 1,000 people develop scabies each month. Scabies is
more common in town (urban) areas, in women and children, in the winter, and in the North of the
country.
The skin-to-skin contact needs to be for a reasonable time to catch the mite. You usually need to
be in skin contact for 15-20 minutes to catch scabies.
Scabies symptoms usually take 2-6 weeks to occur after you are first
Treatment:
Scabies can stay in your skin for ever if not treated. Treatment is needed for:
Anybody who has scabies; AND
All household members, close contacts, and sleeping/sexual partners of the affected person - even
if they have no symptoms. This is because it can take up to six weeks to develop symptoms after you
become infected. Close contacts may be infected, but have no symptoms, and may pass on the mite.
Apply 5% permethrin over whole body including scalp, face (avoid eyes), neck and ears. Do not
forget the soles; wash off after 8-12 h, repeat after 7 days.

1661. An anemic young man is found to have a macrocytosis of 90%. The most likely cause is?
a. Zieves syndrome
b. Thalassemia minor
c. Chronic renal disease
d. IDA
e. Folate def
f. Chronic liver disease
g. HUS
h. Cytotoxic chemotherapy
i. Phenytoin

Ans: Floate Def

Zieve's syndrome is an acute metabolic condition that can occur during withdrawal from prolonged alcohol
abuse. It is defined byhemolytic anemia (with spur cells and acanthocytes), hyperlipoproteinaemia (excessive
blood lipoprotein), jaundice, and abdominal pain.[1] The underlying cause is liver delipidization

1662. An association with HPV is a most characteristic feature of?


a. Torus
b. Exotosis
c. Pleomorphic adenoma
d. Verruca vulgaris
e. Fibroma
f. Epulis fissuratum
g. Mucocele
h. Pyogenic granuloma
i. Parulis
j. Ranula
Ans.D, it is most commonly associated with warts or verruca vulgaris.

Exotosis:benign outgrowth cartilaginous tissue


Plemorphic Adenoma: A Locally invasive benign tumor
Fibroma: benign tumors composed of fibrous tissue
Epulis Fissuratum:benign hyperplasia of fibrous connective tissue which develops as a reactive lesion to
chronic mechanical irritation produced by the flange of a poorly fitting denture
Mucocele: a benign swelling containing mucin.
pyogenic granuloma: Pyogenic granulomata are common benign vascular lesions of the skin and mucosa.a
reactive inflammatory mass of blood vessels and a few fibroblasts within the dermis of the skin.
Parulis: an elevated nodule at the site of a fistula draining a chronic periapical ABSCESS
Ranula: A ranula is a type of mucocele found on the floor of the mouth

1663. For the following type of surgery what is the most likely agent that may cause post-operative infection --
aorto-iliofemoral reconstruction with a Dacron vascular prosthesis?
a. Proteus
b. E.coli
c. Bacteroides fragilis
d. Staphylococcus aureus
e. Staphylococcus epidermis
f. C.perfringens
g. Pseudomonas aeruginosa
h. Streptococcus fecalis
i. Streptococcus pneumonia
j. Brucella melitensis

Ans: Staph epidermidis is most common cause of infections in prosthesis.

1664. A primigravida in the 17th week of her symptomless gestation is found, on US, to have evidence
of placental tissue covering the cervical os. By the end of her pregnancy she is likely to develop?
a. Placental migration
b. Uterine myoma
c. Uterine rupture
d. Choriocarcinoma
e. Chorangioma
f. Vasa previa
g. Subplacental abruption
placenta
h. Subchorionic abruption
placenta
i. Placenta accrete
j. Placenta previa

Ans: A
In 90% of pregnancies, an initial low lying placenta will be pulled upwards by the growing uterus and assume a
normal position in the upper segment. This phenomenon is referred to as Migration

1665. An elderly lady with COPD has chronic SOB. She is listed for cataract extraction. What is the
anaesthetic of choice?
a. Facial nerve block
b. Bupivacaine infiltration of the
peri-orbital skin
c. IV midazolam
d. Peribulbar acupuncture
e. Peribulbar lignocaine
infiltration
f. Topical xylocaine
g. IV alfentanil
h. Epidural anesthesia
i. General anesthesia
j. Retrobulbar xylocaine Inj

Ans:E
The most used mode of anaesthesia in ophthalmic feild is peribulbar using lingocaine

1666. A 55yo chronic alcoholic with known hepatic cirrhosis has been on a heavy bout of alcohol the
night before and was brought home by friends after falling several times in the pub. While being
taken up the stairs to his bedroom he falls down the flight of 5 steps but sustains no obvious
injuiry. His wife calls the ED the next day because she could not rouse him in the morning. He is
brought in in a comatose state and both pupils appear dilated. Skull vault XR appears normal.
a. Hepatic encephalopathy
b. Intracerebral hematoma
c. Brain stem injury
d. Extradural hematoma
e. Chronic subdural hemorrhage
f. Despressed skull fx
g. Vertibrobasilar ischemia
h. Acute subdural hematoma
i. SAH
j. Severe migraine attack

ANS: H
Acute subdural hemotoma, typical history of alcholics, falls and usually debilliated or elderly, hepatic cirrhosis
increases coagulopathy and chances for bleed.

1667. A 58yo man complains of nose disfigurement. He has a hx of facial erythema particularly of the
cheeks and nose. Papules and pustules have been erupting at intervals over the last 10yrs. He
admits to a moderate regular consumption of alcohol. Exam: noted to have rhinophyma. The
most likely dx is?
a. Eczema
b. Herpes simplex
c. Epidermolysis bullosa
d. Dermatomyositis
e. Tinea versicolor
f. Pemphigus vulgaris
g. Acne rosacea
h. Malignant melanoma
i. Psoriasis
j. Atopic dermatitis
Ans: Acne Rosacea

Reason: Acne Rosacea is characterised by recurrent episodes of facial flushing with persistent erythema,
telangiectasia, papules and pustules.It is a chronic acneform disorder of the facial pilosebaceous glands with
an increased reactivity of capillaries to heat, causing flushing and eventually telangiectasia.Rhinophyma is an
enlarged nose associated with rosacea which occurs almost exclusively in men.

Management: Reassurance, benign disease, sunscreens, facial massage reduces oedema.avoid topical
steroids, avoid astringents.
Topical .75% MTZ firstline.
Azaleic Acid 15% gel.
Oral oxytetracyline, doxycyline or erythromycin.
1% Ivermection cream (better than MTZ)
Topical Brimonidine for flushing

1668. A 60yo man who presented with metastatic adenocarcinoma of unknown source. He developed
rapidly progressive weakness of his arms and was found to have a deposit of tumour in his
cervical spine. This was emergently treated with radiation. He developed considerable nausea
and vomiting during his therapy and at the end of the course began to have bloody vomiting.
Following rescusitation with 6 units of blood, what is the next test of choice?
a. Apt test
b. Neck, chest, abdominal XR
c. 24h esophageal pH probe test
d. CT abdomen
e. US abdomen
f. MRI abdomen
g. Barium swallow
h. Angiography
i. Nuclear scan
j. Endoscopy

Ans: Endoscopy
Reason: whenever there hemetemesis endoscopy should be carried out immediately if the patients condition
allows or it should be delayed till resucitation ,Underlying cause for hemetemesis needs to be sorted out.(
High dose radiation is a cause Ulceration and any active bleeders must be treated)

1669. A pt has fine nail pitting, small yellow-brown areas of discoloration in the nailbed involving the nails on
both hands. These findings a
re commonly associated with?
a. Yellow nail syndrome
b. Leukonychia
c. Onychomycosis
d. Lichen planus
e. Pellagra
f. Thallium toxicity
g. Contact dermatitis
h. Zinc deficiency
i. Hypoalbuminemia
j. Psoriasis

Ans: Psoriasis
Characteristic nail changes include pitting, discolouration,subungual hyperkeratosis, crumbling of the nail
plate, and onycholysis.
Oil drop or salmon patch: translucent yellow-red discoloration in the nail bed.
Leukonychia: areas of white nail plate due to foci of parakeratosis within the body of the nail plate.

1670. A young man develops nonfluent, effortful speech with dysarthria. He is able to undertsand
speech. He fails to repeat the sentence. What would you do next?
a. XR skull
b. Non-contrast CT brain
c. Contrast CT brain
d. Contrast MRI optic nerves
e. 4-vessel cerebral angiogram
f. Single vessel cerebral
angiogram
g. Cerebral angiography
h. MRI frontal lobe
i. MRI pituitary gland
j. MRI temporal lobe

Ans: MRI Frontal lobe (Brocas area)

Production (Broca's) dysphasia/aphasia - lesions are located in the left pre-central areas. This is a non-fluent
or expressive aphasia since there are deficits in speech production, prosody and syntactic comprehension.
Patients will typically exhibit slow and halting speech but with good semantic content. Comprehension is
usually good. Unlike Wernicke's aphasia, Broca's patients are aware of their language difficulties. Prosody is
the study of the meter of verse. Here it means the rhythm of speech

Sensory (Wernicke's) dysphasia/aphasia - lesions are located in the left posterior perisylvian region and
primary symptoms are general comprehension deficits, word retrieval deficits and semantic paraphasias.
Lesions in this area damage the semantic content of language while leaving the language production function
intact. The consequence is a fluent or receptive aphasia in which speech is fluent but lacking in content.
Patients lack awareness of their speech difficulties. Semantics is the meaning of words. Semantic paraphrasia
is the substitution of a semantically related but incorrect word.

1671. A pt being sedated with fentanyl develops severe respiratory depression. This is best reversed
using?
a. Ethanol
b. Naloxone
c. Phyostigmine
d. Atropine
e. Methylene blue
f. Diphenhydramine
g. Calcium disodium ethylene
diamine tetra-acetic acid
h. Deferoxamine mesylate
i. Flumazenil
j. Folic acid

Ans: Naloxone
Opioid Antagonist, reverses the effects of fentanyl, though it has to be administeres for a longer period of time
due long half life of fentanyl.

1672. A pt presented with the following blood work, MCV: Decreased Serum ferritin: Decreased Total
iron binding capacity: Increased Serum iron: Decreased Marrow iron: Absent. What is your dx?
a. Thalassemia trait
b. Hypoparathyroidism
c. Hereditary sideroblastic anemia
d. Protein energy malnutrition
e. Chronic renal failure
f. Anemia of chronic disease
g. Acute blood loss
h. IDA
i. Oral contraceptives
j. Megaloblastic anemia

H: Iron deficiency Anemia


Reason: S/S pallor, koilonychia,angular cheilitis, atrophic glossitis, IN marked Anemia ( Cardiac
enlargemnet,Flow Murmurs,ankle oedema and heart failure)
Inv: FBC : shows microcytic hypochromic anemia
Serum ferrtitin Level reduced, normal 12-15 mcg/L, ( serum ferritin is falsely raised during infections)
Anisocytosis and poikilocytosis
Total iron binding capacity is increased

Treatment: Iron supplememtation with B12 and folic acid


1673. A 20yo prv healthy woman presents with general malaise, severe cough and breathlessness
which has not improved with a seven day course of amoxycillin. There is nothing significant to
find on examination. The x-ray shows patchy shadowing throughout the lung fields. The blood
film shows clumping of red cells with suggestion of cold agglutinins.
a. Mycobacterium avium complex
b. Coxiella burnetii
c. Escherichia coli (Gram -ve)
d. Haemophilus influenza
e. Legionella pneumophila
f. Strep pneumococcus
g. TB
h. Mycoplasma pneumonia
i. PCP
j. Staph aureus

Ans: Mycoplasma pneumonia


Reason : inablity to respond to a seven day course of amoxicillin suggests atypical pneumonia, patchy shadows
throughout lung fields and cold agglutination points towards mycoplasma.

M. pneumoniae:[9]
Vague and slow-onset history over a few days or weeks of constitutional upset, fever, headache, dry cough
with tracheitic pleuritic pain, myalgia, malaise and sore throat.

This is like many of the common viral illnesses but the persistence and progression of symptoms is what helps
to mark it out.

In otherwise healthy individuals, it usually resolves spontaneously over a few weeks.

The hacking, dry cough can be very persistent.

Extra-respiratory features include rashes such as erythema multiforme, erythema nodosum and urticaria;
neurological complications like Guillain-Barr syndrome, transverse myelitis, cerebellar ataxia and aseptic
meningitis; haematological complications such as cold agglutinin disease and haemolytic anaemia; joint
symptoms like arthralgia and arthritis; cardiac complications such as pericarditis and myocarditis; rarely, may
cause pancreatitis

TReatment: Macrolides, Fluroquinolones

1674. An 18yo male works in a company where lunches are often catered. One day, the water at the
company facility is not working, but they manage to have the lunch anyway. 2wks later, he
becomes sick. He develops anorexia, nausea, malaise and jaundice. During the course of the
next 4wks, 7 people who shared in the lunch become ill with similar symptoms. After a few wks,
each of the 7 people completely recovers and they replace their caterer. What is a likely dx?
a. Pancreatic ca b. Hemochromatosis
c. Laennecs cirrhosis
d. Hep A
e. HCC
f. Rotors syndrome
g. Primary biliary cirrhosis
h. Gilberts syndrome
i. Hep B
j. Hemolysis

Ans: Hepatitis A

Symptoms of Hepatitis A range from mild nauseas to liver failure (very rare).Spread is normally by the faecal-
oral route although there are occasional outbreaks through food sources.Hand washing and good hygiene
around food and drink prevent spread of infection.Increasing age is a direct determinant of disease severity.

S/S The incubation period is 2-6 weeks with a mean of 4 weeks.


There is a prodrome of mild flu-like symptoms (anorexia, nausea, fatigue, malaise and joint pain) preceding
the jaundice. Smokers often lose their taste for tobacco. Diarrhoea can occur, particularly in children.
This can progress to the icteric phase with:Dark urine (appears first).Pale stools (not always). Jaundice
occurring in 70-85% of adults with acute HAV infection. Abdominal pain occurring in 40% of patients. Itch or
pruritus. Arthralgias and skin rash.Tender hepatomegaly, splenomegaly, and lymphadenopathy

Inv:IgM antibody to HAV is positive with onset of symptoms (usually about 3 to 4 weeks after exposure but up
to 6 weeks). The test is sensitive and specific. It remains positive for between 3 and 6 months (up to 12
months). It remains positive in relapsing hepatitis.

IgG antibody to HAV appears soon after IgM and persists for many years. In the absence of IgM it indicates
past infection or vaccination rather than acute infection. IgG remains detectable for life

LFTs

Mangement: Supportive, avoid alcohol.

1675. A 35yo 1st time donor suddenly passes out as she is donating blood. Which of the following
steps would be least useful in managing this adverse event?
a. Ensure donor is adequately hydrated and has not skipped a meal
b. Elevating the donor's legs as this is usually due to a vasovagal syncope
c. Haemoglobin of the donor meets the minimum requirement for donation
d. The donation is usually continued along with simultaneous normal saline infusion
e. The donor should be encouraged to mobilise after they have recovered

Ans: C
Option C has no role after such an event has occured.

1676. An infant is being examined as part of a routine examination. The child can hold its head up and
lifts its chest off a table. He has a palmer and rooting reflex as well as a social smile. He is not
afraid of strangers. What is the most likely age of this child?
a. neonate
b. 2 months
c. 6 months
d. one year
e. one and a half years
f. two years
g. four years
h. seven years
i. ten years
j. fourteen year

Ans:6 months
Reason : holding head and raising chest upto 90 degrees from a surface while prone (4months)
social smile 6 weeks, becomes increasingly socially responsive.
palmer and rooting reflexes are primitive reflexes
1677. A mother is concerned because her 1m boy has a swelling in his scrotum. He was born
prematurely. On examination the swelling is seen to transilluminate. The likely cause is?
a. Lymphogranuloma Venereum
b. Testicular Torsion
c. Hydrocele
d. Epididymitis
e. Seminoma
f. Mature teratoma
g. Varicocele
h. Lymphoma
i. Orchitis
j. Spermatocele

Ans: Hydrocele
Reason: 1-2% neonates present with congenital hydrocele which disappears by 1-2 years.

Spermatoceles:Smooth, extratesticular, spherical cysts in the head of the epididymis are not uncommon in
adult men. They are benign and do not usually require treatment.Epididymal cysts usually develop in adults
around the age of 40. Epididymal cysts are rare in children and when they occur, usually present around
puberty.

1678. A 2m girl has an ante-natal diagnosis of right hydronephrosis. Postnatal serial US exams revealed
increasing dilatation of the right pelvicalyceal system. No reflux was demonstrated on a MUCG.
Appropriate management should include?
a. Surgical repair
b. Intermittent catheterization
c. Diuresis renography
d. Anticholinergic agents
e. Phenylpropanolamine
f. Gellhorn pessary
g. Biofeedback-assisted behavioral
treatment
h. Oral Estrogen therapy
i. Vaginal Estrogen therapy
j. Ring pessary

1679. Jean is a 72yo woman with recurrent bowel cancer following a hemi-colectomy 2y ago. She is
known to have both local recurrence and liver mets and her pain has been under control on MST
90mg bd. She has had quite severe pain in the RUQ for the past hour despite having taken her
normal dose of MST. You find that she has an enlarged liver which is hard and irregular. There is
marked localised tenderness over the right lobe of her liver. Her abdomen is otherwise soft and
non-tender and the bowel sounds are normal. She is apyrexial. The tx of choice would be?
a. Oral NSAIDs
b. TENS
c. radio therapy to the liver
d. IM diamorphine
e. Paracetamol
f. Prednisolone
g. Physiotherapy
h. epidural anaesthetic
i. Pitocin
j. Aspirin

Ans: I/M diamorphine . Pain ladder

1680. Titubation is a feature of disease involving the?


a. Cerebellum b. Basal ganglia
c. Corpus callosum
d. Pons
e. Temporal lobe
f. Occipital lobe
g. Optic chiasma
h. 3rd ventricle
i. Hypothalamus
j. Pituitary gland

Ans: Cerebellum
titubations mostly occur due to cerebellar lesions.

1681. A 50yo farmer complains of pain in his left arm. Exam: he appears to have a neuropathy
affecting isolated nerves in multiple, random areas of his left arm. He also has a palpable
purpura and tender nodules on both of his upper and lower limbs. A likely diagnosis is?
a. Carpal tunnel syndrome
b. Polyarteritis nodosa
c. Angina Pectoris
d. Gout
e. Cellulitis
f. Rheumatoid arthritis
g. Erysipelas
h. Fascitis
i. Reiter's Syndrome
j. Polymyalgia Rheumatica

Ans: PAN

PAN is necrotising arteritis of medium or small arteries without glomerulonephritis or vasculitis in arterioles,
capillaries, or venules, and not associated with antineutrophil cytoplasmic antibodies (ANCAs).It can affect any
organ but, for unknown reasons, it spares the pulmonary and glomerular arteries.

Presentaion:Peripheral nerves and skin are the most frequently affected tissues.
PURPURA,LIVEDOID,SUBCUTANEOUS NODULES and NECROTIC ULCERS. Neurologically, MONONEURITIS
MULTIPLEX>...involvemnet of CNS,Git,kidneys and heart means higher mortality.RENAL
INVOLVEMENT:hypertension,AKI, GIT:necrosis,perforation.Myalgia
Investigations: Hepatitis B surface antigen is positive in 30%.
The p-ANCA test is usually negative in PAN.
There is a prominent acute phase response but this is nonspecific.
FBC shows leukocytosis with raised neutrophils.
Hypergammaglobulinemia occurs in 30%.
Biopsy

Arteriography shows aneurysms

TREATMENT: Corticosteroids..RELAPSE add Cyclophosphamide.=>


Azathioprine useful in maintenance therapy.
(IV-Ig) and aspirin are effective in childhood PAN.

1682. A patient with chronic neutropenia develops a chronic cough. A CXR reveals a cavitating
intrapulmonary lesion containing a movable rounded ball lesion. A likely dx is?
a. Tuberculosis
b. Bronchiectasis
c. Cystic fibrosis
d. Pulmonary hemosiderosis
e. Mitral stenosis
f. Aspergillosis
g. Wegeners granulomatosis
h. Goodpastures syndrome
i. Pulmonary embolism
j. Non-SCLC

Ans: Aspergillosis.
mostly affects people with reduced immunity, reduced neutrophil count is also predilection for aspergillosis.
five clinical types of Aspergillosis
APBA
Severe Asthma with fungal sensitization
Aspergilloma : (fungal ball in already caivitated space due to TB, Sarcoidosis)
Invasive aspergillosis
Chronic Necrotizing Pulmonary Aspergillosis.

Aspergilloma usually presents with massive hemoptysis, cough and fever are rare, Discovered occasionally in
asymptomatic patiets on xray showing a mass in upper lobe surrounded by air border. CT demonstrated fungal
structure more accurately.

Surgical removal
Long term itraconazole therapy
Instillation of amphotericin
hemoptysis needs to be treated with bronchial artery embolization.

1683. A mother brings her 1yo infant to her pediatrician. She describes that following a common cold
her child's voice has become hoarse and has developed a cough that sounds harsh and brassy
and was worse at night. Exam: the child was noted to have trouble drawing air into its lungs
between coughs and had trouble drawing air into its lungs. There was visible stridor on
inhalation. The cause is most likely to be?
a. EBV
b. Rhinovirus
c. Parainfluenza
d. Flavivirus
e. HIV
f. Rotavirus
g. CMV
h. Kemerovo
i. Creutzfeld-Jacob
j. Rubella

Ans: Parainfuenza
Parainfluenza virus causes several respiratory problms, laryngitis, bronchitis, pneumonia and bronchiolitis,
A rough barking cough with croup
hoarsness and wheezing
labored breathing
runny nose, fever, cough
decreased appetitie, diarrhea.

Investigation: Viral culture of secretions.


Treatment: Symptoatic supportive treatment
Antibiotics incase of secondary infection.

1684. INR:Normal, APTT:Elevated, Thrombin time:Elevated, Plt count:Normal, Bleeding time: Normal.
A likely aetiology is?
a. Waldenstrm's
macroglobulinaemia
b. Heparin
c. Szary cell leukaemia
d. Pelger-Het anomaly
e. von Willebrand's disease
f. Haemophilia
g. HIV infection
h. DIC
i. Acanthocytosis
j. Vit K deficiency

Ans: Heparin

PT-test for extrinsic system


INR- 0.9-1.2 (PT Control), Increased INR in warfarin, Vitamin K and liver disease
APTT- intrinsic system, increased PTT (Heparin,Haemophillia (Factor 8 affected))
THrombin Time- 10-15 seconds, increased in heparin, increasedd in DIC
Bledding Time (normal 7 min) - commonest ccause Von willlbrand disease
1685. An infant has diarrhea for 3d with weight loss from 10 kg to 9 kg. Exam: he is noted to have dry
mucous membranes, poor skin turgor, markedly decreased urine output, and tachycardia. His
BP=normal and compression-release of the nail beds shows satisfactory refilling. Appropriate
treatment would include?
a. Plasmapheresis and plasma
infusion
b. 0.5% Normal Saline
c. Lactated Ringer's injection
d. Packed cells
e. Whole blood
f. Platelets
g. FFP
h. double strength Normal Saline
i. 5% dextrose in 0.5N saline
solution
j. IV heparin

ANS: C

1-5% body weight loos Mild dehydration


skin turgor decreassed, mucous membrane dry, urine low, HR raissed, BP normal,perfusion normal,skin colour
pale,irritable
GIVE ORS

5-10% body weight loss Moderate Dehydration


skin turgor decreased,mucous menbrane very dry,urine oligouric,HR raised,BP normal,Perfusion prolonged
capilllary refill,skin colour grey, lethargic,
GIVE ORS 50-100 mL/kg body weight over 2-4 hours,again starting with 5 mL every 5 minutes. if ors fails then
give IV Bolus of 20 ml/kg Isotonic solution.

10-15% body weight loss Severe Dehydration


skin turgor poor,m ucous membrane parched,urine anuric,HR raised, Bp Decreased, perfusion prolonged ,
skin colour mottledd, Comatose.
intravenous isotonic fluid boluses (20-60 mL/kg) until perfusion improves.

EXAMPLES OF ISOTONIC SOLUTIONS


0.9% Saline (aka Normal Saline)
Ringers Solution
Lactated Ringers
5% Dextrose in 0.225% Saline
5% Dextrose in Water (technically, Isotonic, but physiology hypotonic

1686. A 4yo boy has the sudden onset of bone pain. He begins experiencing bleeding of his gums and
frequent bloody noses. His mother takes him to his pediatrician. Exam: he is pale and has
numerous petechiae over his body, with lymphadenopathy and hepatosplenomegaly. He has
WBC=100,000mm and numerous circulating blast cells. He is admitted to the hospital. A bone
marrow biopsy=35% blast cells. Which of the following is most likely?
a. Mantle cell lymphoma
b. Infectious lymphocytosis
c. Waldenstroms
macroglobulinemia
d. CML
e. CLL
f. Burkitt lymphoma
g. ALL
h. Mycosis fungoides
i. Hairy cell leukemia
j. AML
1687. A 63yo male has anal canal carcinoma with no evidence of spread to the pelvic wall, pelvic
muscles or lymph nodes. This is typically managed by?
a. Resection of the sigmoid colon
b. Right hemicolectomy
c. Left hemicolectomy
d. Transverse colectomy
e. Internal sphincterotomy
f. CT guided drainage
g. Diverticulectomy
h. Transverse colostomy
i. Chemotherapy and
radiatherapy
j. Abdominal perineal resection
1688. A 2m baby develops a life-threatening anemia. Blood tests show a normal serum iron, ferritin
and TIBC. Hemoglobin electrophoresis reveals a markedly decreased Hemoglobin A content and
an increased hemoglobin F content. This baby's anemia is likely to be secondary to?
a. Failure of alpha chain
production
b. Failure of beta chain production
c. Deficiency of B12
d. Lead poisoning
e. IDA
f. Presence of hemoglobin S
g. Presence of hemoglobin M
h. Deficiency of folate
i. Bone marrow failure
j. Inability to manufacture heme
1689. A 30yo caucasian man presented with a 2wk hx of gradually worsening vision in his left eye. The
patient had been seen once by a neurologist 2yrs prv for flashes. At that time a head CT was
normal. The patient was lost to follow up with the neurologist, but the flashes had continued for
the 2yr period. The patient did not experience visual changes with activity or movement. The
patient reported continued decreasing vision. Goldmann visual fields were done and showed a
central scotoma. A MRI was done at this time and showed inflammation of the left optic nerve.
A likely diagnosis is?
a. Pseudotumor
b. Orbital teratoma
c. Optic neuritis
d. Sarcoidosis
e. Optic glioma
f. Lymphangioma
g. Rhabdomyosarcoma
h. Retinal vascular shunts
i. Retinoblastoma
j. Mucormycosis
1690. A pregnant woman in an early stage of labour expresses the wish to have pain relief during
labour. The anesthetist describes that if the patient wishes he can use medication as a local
anesthetic to block the pain sensations of labour. Into which space should the local anaesthetic
be normally injected?
a. Anterior pararenal space
b. Aryepiglottic space
c. Vestibule space
d. Epidural space
e. Sub-arachnoid space
f. Space of Disse
g. Middle ear
h. Posterior pararenal space
i. Supraglottic space j. Lesser sac

1691. A 29yo Afro-Caribbean man presents with a non-productive cough mild aches in the ankles. The
symptoms have been present for 2m. His ESR is elevated. Ca: 2.69 mmol/l; PO43-: 1.20 mmol/l;
ALP: 80 iu/L. Serum 25(OH) D: 180 nmol/l.
Normal values for Calcium: 2.12-2.65mmol/l; Phosphate: 0.8-1.45mmol/l; ALP 30-300iu/L;
Serum 25(OH) D: 20-105nmol/l; Urea: 2.5-6.7mmol/l; Creatinine: 70-120mol/l
a. Osteoporosis
b. Thiazide diuretics
c. Skeletal metastases
d. Primary hyperparathyroidism
e. Hypoparathyroidism
f. Osteomalacia
g. Multiple myeloma
h. Paget's disease of bone
i. Sarcoidosis
j. Hyperthyroidism

1692. A 22yo has had recent chickenpox. He now presents with confusion. He is noted to have low
urine output and large petechiae all over his body. CXR: a large patch of consolidation is seen.
The management of choice should be :
a. Ventilatory support
b. Open surgical debridement
c. Resection of superficial
petechiae with wide margin
d. Booster vaccine
e. TENS
f. Lontophoresis
g. Nephrostomy
h. Oral Corticosteroids
i. Brivudin
j. IV acyclovir

1693. A young girl with a psychiatric hx on med tx is brought to the dermatologist by her mother
because of recurrent patchy hair loss. Exam: the hair shafts revealed twisting and fractures. This
suggests the following pathology:
a. Infection with Trichophyton tonsurans
b. Infection with Microsporum canis
c. Alopecia areata
d. Telogen Effluvium
e. Androgenetic Alopecia
f. Lichen planus
g. Traction Alopecia
h. Alopecia totalis
i. Trichorrhexis nodosa
j. Trichotillomania

1694. Syphilis typically causes


a. Lymphogranuloma Venereum
b. Testicular Torsion
c. Hydrocele
d. Epididymitis
e. Seminoma
f. Mature teratoma
g. Varicocele
h. Lymphoma
i. Orchitis
j. Spermatocele

1695. A middle aged woman has severe collapse of the right femoral head requiring replacement. The
removed femoral head is sent for pathology and is found to contain enlarged fat cells. The pathologist explains
that this is the likely cause of the patient's femoral head collapse. A likely aetiology is
a. Septic emboli
b. Impaired venous drainage
c. Hgb SS disease
d. Steroid use
e. Alcoholism
f. Gaucher's disease
g. missed fracture
h. Cushing's disease
i. Radiation
j. Vasculitis
Ans: Gauchers disease. Fat laden macrophages. Causes avascular necrosis of the bones.

1696. A 7yo boy with frequent episodic asthma is on tx with sodium cromoglycate. His physician wants to add
a non-steroid preventer. The mother of the boy, a teacher, has just read about a nonsteroidal medication
which acts on the mast cells, stopping them from releasing harmful
chemicals. Her physician agrees to add this medication to the boy's drug regimen. Which
medication is the physician most likely to add to the boy's treatment?
a. Inhaled short acting bronchodilator
b. SC adrenaline
c. Nedocromil Sodium
d. Inhaled long acting bronchodilator
e. Inhaled sodium cromoglycate
f. Inhaled steroids
g. Inhaled SABA
h. Oral steroids
i. Nebulised bronchodilators
j. Oral theophylline
key)C
Investigations for asthma
Spirometry:
FEV1:FVC ratio, A low value indicates that you have narrowed airways which are typical in asthma Therefore,
spirometry may be repeated after treatment. An improvement in the value after treatment with a
bronchodilator to open up the airways is typical of asthma.
PEFR:morning readings are usualy lower than evening readings in asthmatics

1697. A 3yo
boy is playing
with his
brother
when he
falls. He cries
immediately
and refuses
to walk. His
mother
carries him
to hospital.
He had a full
term NVD
with no
neonatal
complication
s. His
immunisatio
ns are up to
date. Exam:
looks well
and well-
nourished,
no
dysmorphic features. He
has slight swelling, warmth and discomfort on the lower 1/3 of the left tibia, and refuses to
weight bear. AP and lateral x rays of the tibia are normal. What is the most likely dx?
a. Ankle fx
b. Ankle sprain
c. Fibular fx
d. Knee dislocation
e. Tibial fx
Key)E
Ans)Toddler's fracture
Undisplaced spiral fractures of the tibial shaft in children under 7 years old often follow minimal
trauma and may not be visible on initial X-ray.[8]
Can be difficult to diagnose but should be suspected whenever a child presents with a limp or fails
to bear weight on the leg.
Treatment consists of immobilisation for a few weeks to protect the limb and to relieve pain.
Subperiosteal bone formation is usually apparent on X-rays by two weeks.

1698. Which one of the following electrocardiographic changes is found in hypercalcaemia?


a. Increased QRS interval
b. Prolonged Q-T interval
c. Short P-R interval
d. Short Q-T interval
key)D

1699. An elderly male pt with prior hx of hematemesis is having hx of long term use of aspirin and other drugs,
now presents with severe epigastric pain, dysphagia and vomiting. He was
connected to vital monitors which were not reassuring. What is the management?
a. Oral antacids
b. IV PPI
c. Oral PPI
d. Endoscopy
e. Analgesia
key)D
Ans) long term use of an nsaid predisposes to peptic ulcers. the symptoms described are of a possibly
perforated peptic ulcer or acute upper Gi bleed caused by the ulcer. The guidelines suggest that
Endoscopy is the primary diagnostic investigation in patients with acute UGIB

Endoscopy should be undertaken immediately after resuscitation for unstable patients with severe
acute UGIB.
Endoscopy should be undertaken within 24 hours of admission for all other patients with UGIB.

1700. A 68yo man presents with bruising and hx of falls. He is found to have a mask-like face, pill rolling
tremor and shuffling gait. EEG=normal. Which of the following conditions is he most likely being treated for?
a. HTN
b. DM
c. Psychosis
d. TIA
e. Complex partial seizure
key) C
Ans) features are classic for parkinsons disease. and people with PD usualy go on to develop parkinson related
dementia or psychosis.
Symptoms of Parkinsons:
Slowness of movement (bradykinesia). For example, it may become more of an effort to walk or
to get up out of a chair. This is a 'shuffling' walk with some difficulty in starting, stopping, and
turning easily.
Stiffness of muscles (rigidity), and muscles may feel more tense. Also, your arms do not tend to
swing as much when you walk.
Shaking (tremor) is common, but does not always occur. It typically affects the fingers, thumbs,
hands, and arms, but can affect other parts of the body. It is most noticeable when you are resting.
It may become worse when you are anxious or emotional. It tends to become less when you use
your hand to do something such as picking up an object.

it is diagnosed clinically and treatment includes

1. levodopa plus a dopa decarboxylase inhibitor


2. dopamine agonists
3. anticholinergics (tremor)
4. MAO inhibitors

You might also like